You are on page 1of 449

@

A
SH
W
A
TT
H
M
A
COMPLETELY
REVISED
EDITION

A
M
IN ALL COMPETITIVE

H
EXAMINATIONS

TT
A
W
SH
A

Contains More Than 2500 Objective Questions on


@

Verbal, Analytical &


Non-Verbal Reasoning

Jaikishan Premkishan

ARIHANT PUBLICATIONS (INDIA) LTD.


Arihant Publications (India) Ltd.
All Rights Reserved

A
M
H
TT
© PUBLISHERS
No part of this publication may be re-produced, stored in a retrieval system or distributed
A
in any form or by any means, electronic, mechanical, photocopying, recording, scanning,
W
web or otherwise without the written permission of the publisher. Arihant has obtained all
the information in this book from the sources believed to be reliable and true. However,
SH

Arihant or its editors or authors or illustrators don’t take any responsibility for the absolute
accuracy of any information published and the damages or loss suffered there upon.
A

All disputes subject to Meerut (UP) jurisdiction only.


@

ADMINISTRATIVE & PRODUCTION OFFICES


Regd. Office
‘Ramchhaya’ 4577/15, Agarwal Road, Darya Ganj, New Delhi -110002
Tele: 011- 47630600, 43518550; Fax: 011- 23280316

Head Office
Kalindi, TP Nagar, Meerut (UP) - 250002
Tele: 0121-2401479, 2512970, 4004199; Fax: 0121-2401648

SALES & SUPPORT OFFICES


Agra, Ahmedabad, Bengaluru, Bareilly, Chennai, Delhi, Guwahati,
Hyderabad, Jaipur, Jhansi, Kolkata, Lucknow, Meerut, Nagpur & Pune

ISBN : 978-93-13166-75-7

Published by Arihant Publications (India) Ltd.


For further information about the books published by Arihant
log on to www.arihantbooks.com or email to info@arihantbooks.com
PREFACE

Reasoning is the process of thinking in logical way in order to form the conclusion or judgement from
information. Reasoning basically tests candidates logical thinking and skill to draw inferences. Importance of
reasoning is increasingly moving centre stage in general competitive or entrance examinations. The questions that
are asked in different examinations are not easy to solve and one cannot solve these problems without having a

A
proper practice, once you know the basic concepts behind problems, you can solve them in least time.

M
H
TT
The book ‘How to Crack Test of Reasoning’ will certainly help the candidates to become
familiar with the concepts and approaches, which are used to solve the various reasoning
A
problems. The book covers all the three types of reasoning comprehensively i.e. Verbal,
W
Analytical and Non-Verbal Reasoning divided into chapters. Each chapter is further divided
into types of questions asked in competitive exams. Each section also contains previous year
SH

questions asked in different competitive exams (like UPSC Civil Services, State PSCs, Bank
(PO & Clerk), LIC (AAO & ADO), SSC (CGL, CPO, 10+2, MTS) Railway recruitment, etc.)
A

Latest trends and level of difficulty of questions being asked in various competitive exams
@

have been incorporated in this book. The questions and their accurate solutions will help the
candidates to practice more in minimum time.

Some prominent features of this book are


1. Each chapter in the book is incorporated with basic theory supported with the help of
example to make the chapter not only interesting, but comprehensible in a friendly
manner. This will certainly help the candidate to grasp the topic in an effective manner.
2. The exercises in each chapter, in the book is being divided into two parts i.e., ‘Base level’
and ‘Expert level’ depending upon the difficulty level of the questions.
3. Detailed explanation with answers have been provided at the end of each chapter to
help the candidate.
We are very thankful to Mohd. Shahid Saifi and Mr. Amogh Goel for their sincere efforts for
the compilation of this book.
We are sure that this book will add a new dimension to the preparation for ‘The Test of
Reasoning’ for various competitive examinations and prove a great help in cracking the
examination.

Jai Kishan, Prem Kishan


CONTENTS

VERBAL REASONING

A
1. Analogy 1-13

M
2. Classification 14-23

H
TT
3. Alphabet Test 24-34
4. Word Formation 35-42
A
W
5. Coding-Decoding 43-61
SH

6. Number Series 62-71


7. Letter Series 72-81
A
@

8. Logical Order of Words 82-86


9. Inserting the Missing Character 87-95
10. Number, Ranking and Time Sequence Test 96-107
11. Blood Relations 108-123
12. Mathematical Operations 124-135
13. Direction Sense Test 136-152
14. Clock and Calendar 153-161
15. Sitting Arrangement 162-182
16. Puzzles 183-199
17. Matrix Coding 200-205
18. Mathematical Reasoning 206-209
ANALYTICAL REASONING
19. Logical Venn Diagrams 210-224
20. Eligibility Test 225-233
21. Syllogism 234-253
22. Statement and Assumptions 254-262
23. Statement and Conclusions 263-274

A
M
24. Statement and Arguments 275-284

H
25. Statement and Course of Action 285-291

TT
26. Cause and Effect 292-299
27. Assertion and Reason A 300-304
W
28. Data Sufficiency 305-315
SH

29. Input-Output 316-334


A
@

NON-VERBAL REASONING
1. Analogy 335-344
2. Classification 345-352
3. Series 353-363
4. Mirror Image 364-371
5. Water Image 372-375
6. Paper Folding and Cutting 376-386
7. Grouping of Identical Figures 387-391
8. Formation of Figures 392-397
9. Counting of Figures 398-406
10. Embedded Figures 407-414
11. Figure Completion 415-425
12. Figure Matrix 426-430
13. Cube and Dice 431-442
@
A
SH
W
A
TT
H
M
A
Chapter 01 ANALOGY 1
VERBAL REASONING

01
Analogy
Analogy means similarity or correspondence, side (::) while another term is to be found from the
i.e. having similar features. given alternatives, having the same relation with
In questions based on analogy, a particular this term as the terms of the given pair bear.

A
relationship between words/letters/numbers is Choose the correct alternative.

M
given and another similar relationship has to Illustration 1. Newspaper : Press :: Cloth : ?
be identified from the alternatives provided. (a) Tailor (b) Fibre

H
(c) Mill (d) Textile

TT
Types of Analogy Solution (c) As, ‘Newspaper’ is prepared in ‘Press’, in
the same way, ‘Cloth’ is manufactured in the ‘Mill’.
Analogy may be broadly divided into the A
Here, some relationships are given, which are
W
following types useful for solving questions based on analogy
SH

(i) Word Analogy Country Capital Currency


(ii) Number Analogy
Bangladesh Dhaka Taka
A

(iii) Alphabet Analogy


Iraq Baghdad Dinar
@

(iv) Mixed Analogy


China Beijing Yuan
Word Analogy India New Delhi Rupee
This type of analogy covers every type of UK London Pound
relationship that one can think. There are Japan Tokyo Yen
many ways of establishing a relationship like USA Washington DC Dollar
quantity and unit, worker and tools, cause and
Russia Moscow Ruble
effect, word-synonym, word-antonym, country
and capital, state and capital, country and Greece Athens Euro
currency, animal and its young one (kid), male
Occupation Working Place Job Profile
and female, animal and its resting place, game
and place of playing, occupation, the working Lawyer Court Legal Practice
place and the work, study and topic etc. Servant House Service
In word analogy, the candidates are required Beautician Parlour Make up
to discover the relationship between question Mechanic Garage Repairing
pair and then to find the required pair of Waiter Restaurant Serving
words that is most similar to that relationship.
Farmer Field Farming
Direction (Illustration 1) In this question, there Chef Kitchen Cooking
is a certain relationship between two given terms
Sailor Ship Sailing
on one side of (::) and one term is given on another
2 How to Crack Test of Reasoning VERBAL

States of States of Individual Dwelling Individual Dwelling


Capital Capital
India India (Place) (Place)
Sikkim Gangtok Maharashtra Mumbai Lion Den Spider Web
West Madhya Horse Stable Bird Nest
Kolkata Bhopal
Bengal Pradesh King Palace Eskimo Igloo
Andhra
Goa Panaji Amaravati Instrument Measurement
Pradesh
Hygrometer Humidity
Tamil Nadu Chennai Haryana Chandigarh
Sphygmomanometer Blood pressure
Odisha Bhubaneshwar Telangana Hyderabad
Seismograph Earthquake
Bihar Patna Rajasthan Jaipur
Barometer Pressure
Male Female Male Female
Ammeter Current
Dog Bitch Horse Mare
Speedometer Speed
Bull Cow Cock Hen
Thermometer Temperature
Stag Doe Monk Nun
Balance Mass
Nephew Niece Drone Bee Anemometer Wind

A
Individual Class Individual Class Screw Gauge Thickness

M
Frog Amphibian Rat Rodent Taseometer Stress

H
Cattle Herd Snake Reptile Game Place of Playing

TT
Man Mammal Ostrich Bird Tennis Court
Whale Mammal Wrestling
A Arena
W
Animal Sound Animal Sound Boxing Ring
SH

Cock Crow Duck Quack Race Track


Cat Mew Frog Croak Badminton Court
A

Snake Hiss Owl Hoot Hockey Ground


@

Jackal Howl Donkey Bray Athletics Stadium


Exercise Gymnasium
Physical Physical
Unit Unit Cricket Pitch
Quantity Quantity
Power Watt Pressure Pascal Occupation Product Occupation Product
Current Ampere Area Hectare Farmer Crop Architect Designs
Time Second Mass Kilogram Producer Films Teacher Education
Work/Energy Joule Volume Litre Tailor Clothes Choreographer Dance
Force Newton Resistance Ohm Animal Young One Animal Young One
Word Synonym Word Synonym Dog Puppy Hen Chick

Assign Allot Substitute Replace Cat Kitten Cow Calf

Abduct Kidnap Haughty Proud Bear Cub Insect Larva


Dearth Scarcity Brim Edge Study Topic Study Topic
Word Antonym Word Antonym Human
Anthropology Botany Plants
beings
Robust Weak Deep Shallow
Classification
Chaos Peace Cruel Kind Taxonomy Ornithology Birds
of organisms
Gentle Harsh Notice Ignore Seismology Earthquake Herpetology Amphibians
Lethargy Alertness Mourn Rejoice Hematology Blood Physiology Body
Chapter 01 ANALOGY 3

Specialist Subject Specialist Subject Solution (b) Firstly, we will check the relation
between first pair i.e. 20 and 11
Cardiologist Heart Anthropologist Man 20
We can see, + 1 = 10 + 1 = 11
Orthopaedist Bone Pedologist Soil 2
Dermatologist Skin Mycologist Fungi Similarly, the number in the place of ? is obtained as
102
Ophthalmologist Eye Pathologist Diseases + 1 = 51 + 1 = 52
2
Nephrologist Kidney Phycologist Algae
Alphabet Analogy
Number Analogy In this type of analogy, there is a relationship
In this type of analogy, numbers are given in between the given alphabets or group of
a pair or group on one side. The candidate letters. The candidate has to remember the
has to find the relationship between them and position of letters in English alphabetical
then find the number or group of numbers order and find out the relationship between
which will replace the question mark in the the given letters or group of letters in the first
other pair on other side. pair and then find a set of letters to fit in place
The relationship in number analogy can be of of question mark in second pair. The
relationships between the letters can be of

A
following types
Even and odd numbers following types

M
l

Multiplication and division of numbers l


Vowels or consonants

H
l

l
Addition and subtraction of numbers l
Increasing/decreasing position of letters.

TT
l
Square and cube roots of numbers, etc. l
Reverse order of position of letters.
Skipped letter relation.
Directions (Illustrations 2-4) In the following
l

A
Here, following table is showing letters’
W
questions, select the related number pair from the
given alternatives. position in English alphabetical series.
SH

Forward Alphabet Backward Forward Alphabet Backward


Illustration 2. 23 : 13 : : 54 : ? BSSC (CGL) 2015
1 A 26 14 N 13
A

(a) 40 (b) 41 (c) 44 (d) 39


2 B 25 15 O 12
@

Solution (c) Firstly, we will check the relation


3 C 24 16 P 11
between first pair i.e. 23 and 13
4 D 23 17 Q 10
We can see, 23 − 10 = 13
5 E 22 18 R 9
Similarly, the number in the place of ? will be 6 F 21 19 S 8
54 − 10 = 44 7 G 20 20 T 7
8 H 19 21 U 6
Illustration 3. 14 : 225 : : 17 : ? 9 I 18 22 V 5
(a) 250 (b) 340 (c) 300 (d) 324 10 J 17 23 W 4
11 K 16 24 X 3
Solution (d) Firstly, we will check the relation
12 L 15 25 Y 2
between first pair i.e. 14 and 225.
13 M 14 26 Z 1
We can see,
(14 + 1) 2 = (15) 2 = 225 Directions (Illustrations 5-7) Choose the
Similarly, the number in the place of ? is obtained word/letter(s) which will replace the question
as mark.
(17 + 1) 2 = (18) 2 = 324
Illustration 5. AFKP : BGLQ : : CHMR : ?
BSSC (CGL) 2015
Illustration 4. 20 : 11 : : 102 : ?
(a) 49 (b) 52 (a) DINS (b) DNIS (c) DFKP (d) DJOT
(c) 65 (d) 85 Solution (a) Firstly, we will check the relation
between first pair i.e. AFKP and BGLQ
4 How to Crack Test of Reasoning VERBAL

We can see, A F K P Directions (Illustrations 8-9) Choose the


+1 +1 +1 +1 number which will replace the question mark.
B G L Q Illustration 8. NEWS : 14,5,23,19 : : PAPER : ?
Similarly, the term in the place of ? is obtained as (a) 16,5,16,1,18 (b) 18,5,16,1,16
C H M R (c) 16,1,16,5,18 (d) 32,2,32,10,36
+1 +1 +1 +1 Solution (c) Given, N E W S
D I N S

Illustration 6. SNOP : ONSP : : CLAY : ? 14 5 23 19


CISF (ASI) 2017 Each letter assigned its position from left in English
(a) ALCY (b) LCYA alphabet.
(c) LYCA (d) ACLY So, PAPER → 16,1,16,5,18
Solution (a) Firstly, we will check the relation
between first pair i.e. SNOP and ONSP Illustration 9. FILM : 10 : : HOTEL : ?
As, 1 2 3 4 3 2 1 4 (a) 12 (b) 15
S N O P → O N S P (c) 18 (d) 30

A
Similarly, the term in the place of ? is obtained as Solution (a) As, F I L M
↓ ↓ ↓ ↓

M
1 2 3 4 3 2 1 4
Position value 6 + 9 + 12 + 13 = 40 , 40 ÷ 4 = 10
C L A Y → A L C Y

H
[Q number of letters = 4]

TT
Illustration 7. ASTN : ZTSO : : MSUB : ? Similarly, H O T E L
(a) LTTC (b) LRRC ↓ ↓ ↓ ↓ ↓
Position value 8 + 15 + 20 + 5 + 12 = 60
(c) LTTA (d) LRRO A [Q number of letters = 5]
W
Solution (a) Firstly, we will check the relation
Hence, 60 ÷ 5 = 12
between first pair i.e. ASTN and ZTSO
SH

As, A S T N
Types of Questions
A

–1 +1 –1 +1
Different types of questions based on analogy
@

Z T S O that are asked in various competitive exams,


Similarly, the term in the place of ? is obtained as have been given below
M S U B
–1 +1 –1 +1
Type 1
L T T C
Direct/Simple Analogy
Mixed Analogy In this type of analogy, two elements are
In this type of analogy, there is a certain given which are related to each other in a
relationship between the given group of letters particular manner and other element is given
and numbers on one side. The candidates are followed by four alternatives. Firstly, the
asked to find out the correct alternative based candidate is required to identify the
on the given relationship. The relationship in relationship between the first two elements.
mixed analogy can be of following types Then, the candidate is required to pick that
l
The position of the alphabets or letters element from the alternatives which bear
exactly the same relationship to the third
l
Sum or product of the position of alphabets
element, as the first two bear.
or letters etc.
Chapter 01 ANALOGY 5

Illustration 10. ‘College’ is related to ‘Student’ in Similarly, in second pair,


the same way as ‘Hospital’ is related to Reverse
(a) Doctor (b) Nurse F R U I TS ST I U R F
(c) Medicine (d) Patient
Solution (d) In the ‘College’, education is given to
‘Students’, in the same way, treatment is given to the
‘Patient’ in ‘Hospital’. Directions (Illustrations 14-15) The
following questions, consist of two
Illustration 11. ‘Major’ is related to ‘Lieutenant’ in words/numbers having a certain relationship
the same way as ‘Squadron Leader’ is related to to each other, followed by same pair of
(a) Group Captain (b) Flying Attendent words/numbers select the related pair that
(c) Flying Officer (d) Pilot Officer has the same relationship as the original pair
Solution (c) ‘Major’ and ‘Squadron Leader’ are equivalent of words.
ranks in the Army and the Air Force, respectively. Same
Illustration 14. Pigeon : Peace : : ?
as ‘Lieutenant’ and ‘Flying Officer’ are equivalent ranks
(a) Crown : Head (b) White Flag :
in the Army and Air force, respectively.
Surrender
(c) Laurels : Victory (d) War : Liberty
Type 2 Solution (b) As, ‘Pigeon’ is a symbol of ‘Peace’.

A
Similarly, ‘White Flag’ is a symbol of ‘Surrender’.
Completing/Choosing the

M
Analogous Pair

H
Illustration 15. 9 : 50 : : ? SSC (CGL) 2015

TT
(a) 20 : 105 (b) 22 : 110
In this type of questions, two components are (c) 18 : 190 (d) 15 : 225
given, which are related to each other in a
particular way. The candidate is required to find A
Solution (a) In first pair, we have relation
between 9 and 50 as (9 × 5) + 5 = 45 + 5 = 50
W
out the relationship between the first two Similarly, (20 × 5) + 5 = 100 + 5 = 105
SH

components and choose the component/pair from


the given alternatives, which have the same Type 3
A

relationship as there in between the first two.


Choosing a Similar Word/Number
@

Directions (Illustrations 12-13) In each of the


following questions, select the related word/letters In this type of questions, a group of three
group from the given alternatives. words/numbers is given, followed by four
alternatives. The candidate is required to
Illustration 12. Boat : Oar : : Bicycle : ? choose the alternative, which is similar to
(a) Pedal (b) Seat
the given group of words/numbers.
(c) Road (d) Wheel
Solution (a) Second denotes that part of the first, on Directions (Illustrations 16-17) In each of
which the effort is applied to move it. the following questions, choose that
number/word which has the same
Illustration 13. FLOWER : REWOLF : : relationship among the given the three
FRUITS : ? BSSC (CGL) 2015 numbers/words.
(a) STUIRF (b) STUIFR
(c) STIURF (d) STRUIF Illustration 16. 144, 256, 324
Solution (c) In first pair, second term is the reverse order (a) 625 (b) 175
of first term. (c) 188 (d) 189
Reverse Solution (a) The given numbers are perfect
F L OW ER REWO L F squares.
As, 144 = (12) 2; 256 = (16) 2; 324 = (18) 2
Similarly, 625 = (25) 2
6 How to Crack Test of Reasoning VERBAL

Illustration 17. Bhilai, Durgapur, Bokaro Illustration 18. Pen : Pencil : Ink
(a) Baroda (b) Chennai (a) Orange : Banana : Juice
(c) Chandigarh (d) Rourkela (b) Table : Chair : Wood
Solution (d) All cities are famous for steel plants. (c) Cow : Milk : Curd
(d) Fish : Shark : Water
Solution (a) ‘Pen’, ‘Pencil’ belong to the same
Type 4 category i.e. stationary and pen contains ‘Ink’.
Similarly, ‘Orange’ and ‘Banana’ belong to the
Multiple Word/Number Analogy same category i.e. fruits and orange contains ‘Juice’.
In this analogy, a group of three inter-related
words/numbers is given. The candidate is Illustration 19. Given set (6, 10, 16)
(a) 9, 12, 18 (b) 5, 10, 15
required to trace out the relationship among
(c) 6, 11, 12 (d) 7, 11, 17
these words/numbers and choose another
group with similar relationship from the given Solution (d) As, 6 10 16
alternatives.
+4 +6
Directions (Illustrations 18-19) In each of the Similarly,
following questions, choose that set of

A
7 11 17
numbers/words from the four alternatives sets

M
that is similar to the given set. +4 +6

H
TT
Let us Practice A
W
SH

1. A ‘Cow’ is to a ‘Calf’ as ‘Goat’ is to 6. As ‘Bald’ is related to ‘Blond’, in the same


UPPSC (RO) 2014 way, as ‘Barren’ is related to SSC (FCI) 2012
(a) Vegetation (b) Farm
A

(a) Puppy (b) Kid (c) Lamb (d) Colt


(c) Fertile (d) Inhibited
@

2. ‘Pitch’ is related to ‘Cricket’, in the same way


as ‘Arena’ is related to 7. ‘Atom’ is related to ‘Molecule’, in the same
(a) Tennis (b) Gymnastic way as ‘Cell’ is related to RRB (ASM) 2006
(c) Badminton (d) Wrestling (a) Matter (b) Nucleus
(c) Organism (d) Battery
3. Choose the option that expresses the same
relationship as the following. 8. ‘Flower’ is related to ‘Petal’, in the same way
Tobacco : Cancer as ‘Book’ is related to LIC (AAO) 2005
(a) Milk : Food (b) Bud : Flower (a) Pages (b) Content
(c) Soil : Erosion (d) Mosquito : Malaria (c) Author (d) Library
(e) None of these
4. ‘Cyclone’ is related to ‘Anti-cyclone’, in the
same way as ‘Flood’ is related to 9. ‘Duma’ is related to ‘Russia’, in the same
(a) Devastation (b) Havoc way as ‘Shora’ is related to
(c) River (d) Drought (a) Malaysia (b) Afghanistan
(c) France (d) Germany
5. ‘Doctor’ is related to ‘Patient’, in the same
way as‘ Lawyer’ is related to 10. ‘Konkani’ is related to ‘Goa’, in the same
UP (B.Ed.) 2008 way as ‘Dogri’ is related to
(a) Customer (b) Criminal (a) Madhya Pradesh (b) Odisha
(c) Magistrate (d) Client (c) Jammu and Kashmir (d) Gujarat
Chapter 01 ANALOGY 7

Directions (Q. Nos. 11-43) In each of the 23. Hive : Bee :: Eyrie : ? SSC (CPO) 2015
following question, there is a certain relationship (a) Parrot (b) Sparrow
between two given terms on one side of (::) and one (c) Eagle (d) Pigeon
term is given on another side (::) while another
term is to be found from the given alternatives, 24. Nightingale : Warble :: Frog :? UPPSC (RO) 2014
having the same relation with this term as the (a) Yell (b) Cackle (c) Squeak (d) Croak
terms of the given pair bear. Choose the correct 25. Apes : Gibber :: Camels : ? SSC (CGL) 2016
alternative. (a) Grunt (b) Cheep (c) Bleat (d) Whine
11. Maharashtra : India :: Texas : ? SSC (10+2) 2014
(a) Canada (b) Mexico (c) Brazil (d) USA
26. Adversary : Enemy :: Adversity : ?
SSC (Steno) 2016
12. Bow : Arrow :: Pistol: ? (a) Friend (b) Dynamic
(a) Bullet (b) Gun (c) Shoot (d) Rifle (c) Love (d) Difficulty

13. Axe : Woodcutter :: Needle : ? 27. YTOJ : XSNI :: WRMH : ? SSC (CGL) 2014
UP Police (Constable) 2018 (a) VQLG (b) TOJE (c) RMHC (d) UPKF
(a) Butcher (b) Carpenter 28. TSR : FED :: WVU : ? SSC (CGL) 2016
(c) Tailor (d) Painter (a) CAB (b) MLK (c) PQS (d) GFH

A
14. King : Throne :: Rider : ? SSC (FCI) 2012
29. MAD : JXA :: RUN : ? SSC (Steno) 2016

M
(a) Chair (b) Horse (c) Seat (d) Saddle (a) OSQ (b) PRJ (c) UXQ (d) ORK

H
15. Coins : Mint :: Bricks : ? WBSC 2018
30. UHCDN : VIDEO :: OKZXDQ : ?

TT
(a) Foundry (b) Cemetery SSC (10+2) 2017
(c) Furnace (d) Kiln (a) REPLAY (b) REPOSE
16. River : Stream :: Ocean : ? SSC (CGL) 2017
(c) PLAYER A (d) OPPOSE
W
(a) Current (b) Pond 31. ECEIN : NIECE :: VANISH : ? SSC (10+2) 2017
SH

(c) Dam (d) Sea (a) HSINVA (b) SHINAV


(c) HSINAV (d) HSNIAV
17. Prediction : Future :: Regret :? SSC (Steno) 2016
A

(a) Past (b) Present (c) Sin (d) Time 32. AZBY : CXDW :: EVFU : ? SSC (Steno) 2016
@

(a) GHTS (b) TGSH (c) GTHS (d) GSTH


18. Hongkong : China :: Vatican : ?
SSC (Multitasking) 2013 33. AHOP : CKSU :: BJMR : ? SSC (CGL) 2013
(a) France (b) Mexico (a) EZUQ (b) DMQW (c) DQKM (d) CJWN
(c) Canada (d) Rome
34. DCBA : WXYZ :: IJKL : ? SSC (CPO) 2015
19. Moderate : Intensify :: Nominal : ? (a) SRQP (b) QPON (c) RQPO (d) PONM
(a) Memorial (b) Expensive
(c) Distance (d) Chacs 35. 63 : 21 :: 27 :?
(a) 6 (b) 9 (c) 1 (d) 3
20. Oxygen : Burn :: Carbon dioxide : ?
IBPS (Clerk) 2012
36. 85 : 42 : : 139 : ? SSC (CGL) 2016
(a) Isolate (b) Foam (a) 68 (b) 69 (c) 70 (d) 67
(c) Extinguishes (d) Explode
37. 9 : 24 :: ? : 6 SSC (CPO) 2015
(e) None of these
(a) 1 (b) 2 (c) 5 (d) 3
21. Sitar : Guitar :: Tanpura : ? CLAT 2015
38. 08 : 66 :: ? : 38 SSC (10+2) 2013
(a) Trumpet (b) Violin
(a) 2 (b) 6 (c) 12 (d) 19
(c) Harmonium (d) Mridanga

22. Money : Misappropriation :: Writing : ? 39. 64 : 8 :: 289 : ? SSC (10+2) 2017


(a) 17 (b) 27 (c) 26 (d) 19
SSC (CGL) 2014
(a) Deception (b) Mistake 40. 23 : 72 :: 38 : ? SSC (CPO) 2017
(c) Plagiarism (d) Theft (a) 110 (b) 117 (c) 123 (d) 112
8 How to Crack Test of Reasoning VERBAL

41. 13 : 156 :: 17 : ? SSC (10+2) 2017 54. Rabbit : Animal :: ? : ? SSC (10+2) 2018
(a) 135 (b) 270 (c) 272 (d) 130 (a) Sun : Moon (b) Pentagon : Figure
(c) Lion : Bird (d) Animal : Cow
42. 25 : 625 :: 35 : ? SSC (CGL) 2012
(a) 1575 (b) 1205 (c) 875 (d) 635 55. Horse : Hoof :: ? SSC (DEO & LDC) 2012
(a) Man : Foot (b) Dog : Black
43. 354 : 351 :: 478 : ? SSC (10+2) 2018 (c) Paise : Rupee (d) Pen : Pencil
(a) 481 (b) 447 (c) 475 (d) 477
56. Sailor : Compass :: ? PNB (Clerk) 2011
Directions (Q. Nos. 44-45) In the following (a) Student : Exam (b) Doctor : Stethoscope
questions, select the related number pair from the (c) Pen : Officer (d) Painter : Artist
given alternatives. (e) Bricks : Plumber
44. 63 : 36 :: ? : ? SSC (10+2) 2018
57. Create : Destroy :: ? SSC (CPO) 2017
(a) 94 : 49 (b) 35 : 54 (c) 47 : 72 (d) 73 : 39 (a) Thin : Small (b) Long : Short
(c) Fat : Huge (d) Long : Topmost
45. 62 : 145 :: ? : ? SSC (CPO) 2017
(a) 79 : 168 (b) 119 : 226 58. Bird : Worm :: ? SSC (Steno) 2016
(c) 167 : 291 (d) 34 : 122 (a) Horse : Stable (b) Trap : Cheese
(c) Lion : Cave (d) Cat : Mouse

A
Directions (Q. Nos. 46-53) Choose the
59. Cells : Cytology :: ?

M
word/letter(s) /number which will replace the RRB (ASM) 2009
question mark. (a) Worms : Ornithology (b) Insects : Entomology

H
(c) Diseases : Physiology (d) Tissues : Morphology
46. XMAE : 16 :: VTNG : ?

TT
(a) 21 (b) 17 (c) 35 (d) 18 60. Mendacity : Honesty :: ? MPPSC 2018

47. CEGI : RTVX :: IKMO : ? A


(a) Truth : Beauty
(b) Sportsmanship : Fortitude
W
(a) JKNP (b) MNQP (c) LNPR (d) DFHI (c) Courageous : Craven
SH

48. Palaeontology : Fossil : : Phrenology : ? (d) Turpitude : Depravity


SSC (CGL) 2015 61. Crumb : Bread :: ? CLAT 2017
A

(a) Pancreas (b) Thyroid (a) Water : Vessel (b) Powder : Face
@

(c) Lungs (d) Skull (c) Splinter : Wood (d) Inch : Unit
49. F : 216 :: L : ? SSC (CGL) 2013 62. Nuts : Bolts :: ? CLAT 2013
(a) 1728 (b) 1700 (c) 1600 (d) 1723 (a) Nitty : Gritty (b) Bare : Feet
(c) Naked : Clothes (d) Hard : Soft
50. PST : 01 :: NPR : ?
(a) 3 (b) 4 (c) 1 (d) 7 63. Book : Author :: ? CLAT 2013
(a) Rain : Flood
51. GREAT : 25 :: NUMBER : ? SSC (FCI) 2012 (b) Light : Switch
(a) 36 (b) 38 (c) 27 (d) 24 (c) Symphony : Composer
(d) Song : Music
52. AFI = M : ADD = L :: ABA = ?
SSC (CGL) 2015 64. Preamble : Constitution :: ? SSC (CGL) 2016
(a) I (b) K (c) N (d) O (a) Word : Dictionary (b) Contents : Magazine
(c) Explanation : Poetry (d) Preface : Book
53. 1, 2, 4, 7 : 3, 4, 6, 9 :: ? : 2, 3, 5, 8 SSC (10+2) 2012
(a) 0, 1, 3, 6 (b) 2, 4, 5, 8 Directions (Q. Nos. 65-67) In each of the
(c) 1, 3, 4, 7 (d) 3, 5, 6, 8 following questions, choose that set of numbers
from the four alternatives sets that is similar to
Directions (Q. Nos. 54-64) The following the given set.
questions, consist of two words having a certain
relationship to each other, followed by same pair 65. Given set : (6 : 12 : 18) SSC (10+2) 2013
of words. Select the related pair that has the same (a) (4 : 8: 14) (b) (12 : 24 : 36)
relationship as the original pair of words. (c) (6 : 20 : 26) (d) (30 : 36 : 45)
Chapter 01 ANALOGY 9

66. Given set : (7, 77, 140) 73. Consider the relationship given below.
(a) (8, 80, 166) (b) (9, 88, 180) Road : Bus : Driver
(c) (8, 88, 160) (d) (10, 110, 300)
Now, choose the group of words from the
67. Given set : (31, 19, 41) SSC (CPO) 2015 following which show the same
(a) (27, 29, 47) (b) (23, 33, 51) relationship as given below. CGPSC 2014
(c) (29, 17, 37) (d) (16, 27, 11) (a) Water : Ship : Driver
(b) Sky : Aeroplane : Pilot
Directions (Q. Nos. 68-71) In each of the
following questions, choose that word which has the (c) Paper : Letter : Reader
same relationship among the given three words. (d) Track : Train : Passenger
(e) None of the above
68. Yuan, Kyat, Lira
(a) Dirham (b) Turkey (c) Madrid (d) Mass 74. In this pyramid if 11 22 31 : 12 21 32 :: 9 12 21 : ?
1
69. Yellow, Blue, Red
(a) Black (b) Orange (c) White (d) Paint 2 3 4
9 8 7 6 5
70. 525, 813, 714
10 11 12 13 14 15 16
(a) 353 (b) 329 (c) 606 (d) 520
25 24 23 22 21 20 19 18 17

A
71. 8, 1331, 4913 26 27 28 29 30 31 32 33 34 35 36

M
(a) 121 (b) 1330 (c) 64 (d) 9
Now, choose the numbers which will

H
72. In given question, choose that word which has replace the question mark.

TT
the same relationship among the given three (a) 2, 7, 14 (b) 8, 13, 20
words Bhopal, Panaji, Gandhinagar (c) 6, 15, 18 (d) 10, 23, 30
(a) Amritsar (b) Allahabad (c) Patna (d) Baroda
A
W
SH

Answers
A

1. (b) 2. (d) 3. (d) 4. (d) 5. (d) 6. (c) 7. (c) 8. (a) 9. (b) 10. (c)
@

11. (d) 12. (a) 13. (c) 14. (d) 15. (d) 16. (a) 17. (a) 18. (d) 19. (b) 20. (c)
21. (b) 22. (c) 23. (c) 24. (d) 25. (a) 26. (d) 27. (a) 28. (b) 29. (d) 30. (c)
31. (c) 32. (c) 33. (b) 34. (c) 35. (b) 36. (b) 37. (d) 38. (b) 39. (a) 40. (b)
41. (c) 42. (d) 43. (c) 44. (a) 45. (b) 46. (d) 47. (c) 48. (d) 49. (a) 50. (a)
51. (a) 52. (b) 53. (a) 54. (b) 55. (a) 56. (b) 57. (b) 58. (d) 59. (b) 60. (c)
61. (c) 62. (c) 63. (c) 64. (d) 65. (b) 66. (c) 67. (c) 68. (a) 69. (b) 70. (c)
71. (c) 72. (c) 73. (b) 74. (b)

Answer with Explanations


1. (b) ‘Calf’ is the young one of ‘Cow’. Similarly, ‘Kid’ is 4. (d) As, ‘Cyclone’ is opposite of ‘Anti-cyclone’.
the young one of ‘Goat’. Similarly, Flood’ is opposite of ‘Drought’.
2. (d) ‘Cricket’ is played on ‘Pitch’. Similarly, ‘Wrestling’ 5. (d) ‘Doctor’ deals with ‘Patient’. Similarly,
is done in ‘Arena’. ‘Lawyer’ deals with ‘Client’.
3. (d) As, ‘Tobacco’ is a cause of ‘Cancer’. 6. (c) As, ‘Bald’ is related to ‘Blond’ in the same
Similarly, ‘Mosquito’ is the cause of ‘Malaria’. way, ‘Barren’ is related to ‘Fertile’.
10 How to Crack Test of Reasoning VERBAL

7. (c) First constitutes the second. As combining 27. (a) As, Similarly,
‘Atom’ we get ‘Molecule’ in the same way Y T O J W R M H
combining ‘Cell’ we get ‘Organism’.
–1 –1 –1 –1 –1 –1 –1 –1
8. (a) ‘Flower’ is made of ‘Petals’. Similarly, ‘Book’ is
made of ‘Pages’. X S N I V Q L G

9. (b) ‘Russian’ Parliament is called ‘Duma’. Similarly,


‘Afghanistan’s Parliament is called ‘Shora’. 28. (b) As, T S R F E D

10. (c) ‘Konkani’ is the language of ‘Goa’. Similarly, –1 –1 –1 –1


‘Dogri’ is the language of ‘Jammu and Kashmir’. Similarly,
11. (d) As, ‘Maharashtra’ is one of the states in ‘India’, in
W V U M L K
the same way, ‘Texas’ is one of the states in ‘USA’.
12. (a) As, ‘Arrow’ is released from ‘Bow’. Similarly, –1 –1 –1 –1
‘Bullet’ is released from ‘Pistol’.
29. (d) As, 13 1 4 10 24 1
13. (c) As, ‘Woodcutter’ cuts wood by ‘Axe’. Similarly, M A D J X A
‘Tailor’ stitches clothes by ‘Needle’.
–3
14. (d) As, ‘King’ sits on the ‘Throne’. in the same way –3
‘Rider’ sits on the ‘Saddle’.

A
–3
15. (d) ‘Coins’ are made in ‘Mint’, same as ‘Bricks’ are Similarly,

M
made in ‘Kiln’. 18 21 14 15 18 11

H
R U N O R K
16. (a) As, ‘Stream’ is produced by ‘River’, same as

TT
–3
‘Current’ is produced by ‘Ocean’.
–3
17. (a) ‘Predictions’ are made for ‘Future’, same as, –3
‘Regrets’ are made for ‘Past’. A
W
18. (d) As, ‘Hongkong’ is in ‘China’. Similarly, ‘Vatican’ 30. (c) As,
U H C D N
SH

is in ‘Rome’.
+1 +1 +1 +1 +1
19. (b) ‘Moderate’ and ‘Intensify’ are antonyms and
A

‘Nominal’ and ‘Expensive’ are antonyms. V I D E O


@

20. (c) ‘Oxygen’ enhances the fire and ‘Carbon Similarly,


dioxide’ extinguishes the fire. O K Z X D Q
21. (b) ‘Sitar’, ‘Guitar’, ‘Tanpura’ and ‘Violin’ are all +1 +1 +1 +1 +1 +1
string instruments.
P L A Y E R
22. (c) As, ‘Money’ can be ‘Misappropriation’, in the
same way ‘Writing’ can be ‘Plagiarism’. 31. (c) The alphabets of the words are written in
23. (c) As, ‘Bee’ residence is called ‘Hive’, in the same reverse order.
way ‘Eagle’ residence is called ‘Eyrie’. 32. (c) As,
24. (d) As, sound of ‘Nightingale’ is ‘Warble’. Similarly, 1 26 2 25 3 24 4 23
A Z B Y C X D W
the sound of ‘Frog’ is ‘Croak’.
+2
25. (a) ‘Gibber’ is the sound made by ‘Apes’ whereas
–2
‘Grunt’ is the sound made by ‘Camels’.
+2
26. (d) Synonym of ‘Adversary’ is ‘Enemy’. Similarly, –2
Synonym of ‘Adversity’ is ‘Difficulty’.
Chapter 01 ANALOGY 11

Similarly, 40. (b) As, 23 72


5 22 6 21 7 20 8 19
×3 +3
E V F U G T H S
Same as, 38 117
+2
–2 ×3 +3
+2 41. (c) As, 13 156
–2
(13)2 –13
∴ ? = GTHS
Similarly, 17 272
33. (b) As, A H O P
+2 +3 +4 +5 (17)2 –17

C K S U 42. (d) As, New Number


Similarly,
B J M R 25 : 625
+2 +3 +4 +5

D M Q W Similarly, New Number

A
M
34. (c) Here, each letter has its opposite letter. 35 : 635

H
As, D C B A

TT
43. (c) As, 354 – 3=351
W X Y Z Similarly,
A
478 – 3= 475
W
Similarly, I J K L
44. (a) As,
63 : 36
SH

R Q P O Similarly,
A

94 : 49
35. (b) As, 63 ÷ 3 = 21
@

Similarly, 27 ÷ 3 = 9 45. (b) As, 62 145

36. (b) As,


85 42 (8)2 –2 (12)2 +1

×½ – ½ +4
Similarly, Similarly, 119 226
139 69

×½ –½ (11)2 –2 (15)2 +1
37. (d) As, 9 × 3 − 3 = 27 − 3 = 24 +4
Similarly, ?× 3 − 3 = 6
46. (d) As, X M A E
⇒ ?× 3 = 6 + 3 = 9
9
⇒ ?= =3 ⇒?= 3
3 Positional 24 13 1 5
values
38. (b) As, (08)2 + 2 = 64 + 2 = 66 (2+4)(1+3)(0+1)(0+5)
Similarly, (06)2 + 2 = 36 + 2 = 38
39. (a) As, 64 ⇒ 64 = 8 6 + 4 + 1 + 5 =16
Same as, 289 ⇒ 289 = 17
12 How to Crack Test of Reasoning VERBAL

Similarly, 52. (b) As, AFI = 169


V T N G
= 13 = M

Positional 22 20 14 7 and ADD = 144


values = 12 = L
(2+2)(2+0)(1+4)(0+7) Similarly, ABA = 121
= 11 = K
4 + 2 + 5 + 7 = 18
53. (a) As,
47. (c) As, 1 2 4 7
C E G I R T V X
+2 +2 +2 +2
opposite
opposite
3 4 6 9
opposite
Similarly,
opposite
0 1 3 6
Similarly,
–2 –2 –2 –2
I K M O L N P R
2 3 5 8

A
opposite
54. (b) As, ‘Rabbit’ is a kind of ‘Animal’. Similarly,

M
opposite
opposite ‘Pentagon’ is a kind of ‘Figure’.

H
opposite 55. (a) The ‘Hoof’ is the hard lower part of ‘Horse’ feet.

TT
In the same way, ‘Foot’ is the lower part of ‘Man’.
48. (d) As, ‘Palaeontology’ is the study of ‘Fossils’ in the 56. (b) ‘Sailor’ judges the direction through ‘Compass’
same manner ‘Phrenology’ is the study of ‘Skull’. A
same as ‘Doctor’ judges the breathing and heart
W
49. (a) As, F → (6)3 = 216 beat through ‘Stethoscope’.
SH

[since, positional value of F is 6] 57. (b) As opposite word of ‘Create’ is ‘Destroy’, same
Similarly, L → (12 )3 = 1728 as opposite word of ‘Long’ is ‘Short’.
A

[since, positional value of L is 12] 58. (d) As ‘Bird’ eats ‘Worm’. Similarly, ‘Cat’ eats.
@

‘Mouse’.
50. (a) As, T
P S
Positional 59. (b) ‘Cytology’ is the science of study of ‘Cells’. In
value the same way ‘Entomology’ is the science of study
16 + 19 + 20 = 55 of ‘Insects’.
Digits’s sum = 5 + 5 = 10 60. (c) Both are antonyms of each other. ‘Mendacity’
Again, digits’s sum = 1 + 0 = 1 means untruthfulness, which is opposite of
‘Honesty’. In the same way, ‘Craven’ means coward
Similarly, which is antonym of ‘Courageous’.
N P R
Positional 61. (c) ‘Crumb’ is small piece of ‘Bread’ and ‘Splinter’ is
value small piece of ‘Wood’.
14 + 16 + 18 = 48
62. (c) As, ‘Nuts’ are covered with ‘Bolts’. Similarly,
Digit’s sum = 4 + 8 = 12 ‘Naked’ is covered with ‘Clothes’.
Again, digit’s sum = 1 + 2 = 3 63. (c) ‘Book’ is written by ‘Author’. Similarly,
‘Symphony’ is composed by ‘Composer’.
51. (a) In word GREAT, the number of alphabets = 5
64. (d) As ‘Constitution’ have ‘Preamble’, same as
∴ GREAT = (5)2 = 25 ‘Book’ have ‘Preface’.
In the same way, in word NUMBER, the number of 65. (b) As,
alphabets = 6
6 12 6 18
∴ NUMBER = (6)2 = 36 ;
6×2 6×3
Chapter 01 ANALOGY 13

Similarly, 71. (c) All the numbers are perfect cubes.


12 24 12 36 Let us see
12 × 2 12 × 3 As, 8 = 23
66. (c) As, 1331 = 113
7 77 7 140
; 4913 = 17 3
7 × 11 7 × 20
Similarly, 64 = 43
Similarly,
8 88 8 160 72. (c) All are the capitals of Indian states.
; 73. (b) As, bus runs on the road and for running it we
8 × 11 8 × 20
need driver. In the same way, aeroplane flies in the
sky and for flying it we need pilot.
67. (c) All the numbers in the given group are prime
numbers. Similarly, the numbers in option (c) are 74. (b) 1
prime numbers.
2 3 4
68. (a) All are the currencies of different countries.
9 8 7 6 5
69. (b) All are the colours of rainbow.

A
10 11 12 13 14 15 16
70. (c) 5 + 2 + 5 = 12

M
25 24 23 22 21 20 19 18 17
8 + 1 + 3 = 12

H
7 + 1 + 4 = 12 26 27 28 29 30 31 32 33 34 35 36

TT
Similarly, 6 + 0 + 6 = 12 In this pyramid, you can see the pattern.
So, the correct option is 8, 13, 20.
A
W
SH
A
@
14 How to Crack Test of Reasoning VERBAL

02
Classification
Classification means to group the given items Number Classification
on the basis of certain common quality that
they possess. In number classification, there is a similarity
among the given numbers or group of
In this chapter, the questions consist of numbers. The candidate has to identify the

A
four/five elements. All the elements except odd number or group of numbers which does

M
one, follow a certain pattern/rule or have some not belong to the group.

H
common quality and thus form a group. The
The number classification is generally based
candidate has to find the element that does

TT
on the following similarities:
not belong to the group. l
Square and square root of a number

Types of Classification
l
A
Cube and cube root of a number
W
l
Even and odd number
SH

Classification may be broadly divided into the l


Prime number
following types: l
Divisibility test of a number
A

(i) Word Classification l


Sum of digits of a number
@

(ii) Number Classification Illustration 3. Choose the odd one.


(iii) Letter Classification SSC (CGL) 2014
(a) 27 (b) 35 (c) 18 (d) 9
Word Classification Solution (b) Except 35, all others are multiples of 9,
but 35 is the multiple of 5 and 7.
In word classification, the candidate is given a
group of words in which all except one are Illustration 4. Choose the odd one.
related to one another in a particular manner (a) 16 — 28 (b) 32 — 84 (c) 12 — 52 (d) 49 — 77
thus form a group. The candidate has to Solution (c) 2 × 8 = 16
identify the odd word that does not belong to 8 × 4 = 32
the group.
7 × 7 = 49
Illustration 1. Choose the odd one. WBPSC 2018 But, 5 × 2 = 10
(a) Square (b) Circle Thus, 12— 52 is an odd pair.
(c) Parallelogram (d) Rectangle
Solution (b) Except circle, all other given shapes are Letter Classification
made up of straight lines.
In letter classification, there is a similarity or
Illustration 2. Choose the odd one. pattern among the letters in the group and the
(a) Eyes (b) Nose (c) Ears (d) Lungs candidate has to identify the pattern and
Solution (d) Except lungs, all others are external body select the one which does not follow the
parts, but lungs are internal parts of body. common pattern in the group.
Chapter 02 CLASSIFICATION 15

The relationship among the letters is generally


based on the following similarities: Types of Questions
l
Position of letters There are several types of questions that can
l
Vowel and consonant be asked from classification. Some of the types
l
Reverse order of letters are discussed below:
l
Addition or multiplication of positional value
Illustration 5. Choose the odd one. Type 1
(a) H (b) Q
(c) T (d) Z
Choosing the Odd Word/Letter
Solution (b) All other letters except Q, occupy the /Letters’ Group/Number
even-numbered positions in the English alphabets.
In this type of problems, four/five words/letters
As, H → 8 (even), Q → 17 (odd) /letters’ groups/numbers are given having
T → 20 (even), Z → 26 (even) some common features except the odd one.
You are required to find the ‘odd one out’.
Illustration 6. Choose the odd one.
(a) LNJ (b) RTP Illustration 8. Choose the odd one.
(c) NPK (d) FHD (a) Strong (b) Hesitant

A
Solution (c) +2 +2 +2 (c) Daring (d) Brave

M
Solution (b) Hesitant is different as all others are

H
L N J , R T P, N P K synonyms of each other.

TT
–2 –2 –3 Illustration 9. Choose the odd one.
+2
(a) Marigold A
(b) Tulip (c) Lotus
SSC (CPO) 2015
(d) Rose
W
and F H D
Solution (b) Except Tulip, all others are flowers
SH

whereas Tulip is a kind of plant.


–2
Clearly, letters group NPK follows different pattern Illustration 10. Choose the odd one.
A

from others.
BSSC (CGL) 2015
@

Hence, NPK does not belong to the group. (a) MTF (b) SLE
Note : Besides word, letter and number classification, there (c) RKD (d) UNG
also exists mixed classification in which combination of Solution (a) Here, the letters’ groups follow the below
letters and numbers are given. pattern
Illustration 7. Choose the odd one. 13 20 6 19 12 5
(a) OE3 (b) XD6 (c) JB5 (d) VK2 M T F S L E
(e) PH3 ,
+7 – 14 –7 –7
Solution (e) Here, putting the positional values of
letters according to English alphabetical order as 18 11 4 21 14 7
R K D U N G
A = 1, B = 2, C = 3, ... , Z = 26
and
and solving the alternatives, we have –7 –7 –7 –7
OE3 ⇒ OE = O ÷ E = 15 ÷ 5 = 3;
Except MTF, all others follow similar pattern. So,
XD6 ⇒ XD = X ÷ D = 24 ÷ 4 = 6; letters’ group MTF is different from others.
JB5 ⇒ JB = J ÷ B = 10 ÷ 2 = 5;
VK2 ⇒ VK = V ÷ K = 22 ÷ 11 = 2; Illustration 11. Choose the odd one. MPPSC 2014
(a) HGFED (b) PONML
PH3 ⇒ PH = P ÷ H = 16 ÷ 8 = 2 ≠ 3 (c) NLKJI (d) TSRQP
In PH3, the numerical value should be 2 instead of Solution (c) Except option (c), in all the other groups
3 because 16 ÷ 8 = 2. the consecutive letters are given in reverse order
Hence, PH3 does not belong to the group. But in option (c), letter M is missing.
16 How to Crack Test of Reasoning VERBAL

Illustration 12. Choose the odd one. Illustration 14. Choose the odd pair.
(a) 144 (b) 169 (c) 256 (d) 288 (a) Gold : Ornaments (b) Cloth : Garments
(e) 324 (c) Wood : Furniture (d) Leather : Footwear
Solution (d) Except 288, all other numbers are square (e) Earthen pots : Clay
of natural numbers. Solution (e) Except pair (e), in all other pairs, the first
As, 12 2 = 144, 13 2 = 169 is the raw material used to make the second.
16 2 = 256, 18 2 = 324 Illustration 15. Choose the odd pair.
Illustration 13. Choose the odd one. (a) Broad - Wide (b) Light - Heavy
(c) Tiny - Small (d) Big - Large
[SSC (10+2) 2014]
(a) 512 (b) 625 (c) 1296 (d) 2401 Solution (b) Light is an antonym of heavy. But in
Solution (a) Here, 512 is the cube of 8, whereas other pairs, words are synonyms of each other.
other numbers are square of 25, 36 and 49,
respectively. So, 512 is different from others.
Illustration 16. Choose the odd pair.
(a) (21 : 24) (b) (28 : 32)
(c) (14 : 16) (d) (70 : 80)
Type 2 (e) (54 : 62)
21 7 28 7
Choosing the Odd Pair of Solution (e) Here, = ; = ;

A
24 8 32 8
Words/Numbers

M
14 7 70 7
= ; =
In this type of questions, different pairs of 16 8 80 8

H
words/numbers having some common features/ 54 27
=

TT
and
properties like names, places, uses, situations, 62 31
origin, operations etc. are given, except one
pair. You have to identify the odd pair. A
Clearly, 54 : 62 is different from others.
W
SH

Let us Practice
A
@

Directions (Q. Nos. 1-30) In the following 6. (a) Shimla (b) Ooty
question, three/four out of the four/five (c) Darjeeling (d) Agra
alternatives are same in a certain way and so BSSC (CGL) 2015
form a group. Find the odd one that does not 7. (a) Bridge (b) Escalator
belong to the group. (c) Ladder (d) Staircase
SSC (10+2) 2017
1. (a) Sun (b) Mercury
(c) Mars (d) Venus 8. (a) Tired (b) Tardy
SSC(10+2) 2017 (c) Slow (d) Late
2. (a) Cloth (b) Plastic SSC (Steno) 2016
(c) Thread (d) Cotton OPSC 2018 9. (a) Islamabad (b) Kabul
3. (a) Barter (b) Purchase (c) Canberra (d) Sydney
(c) Sale (d) Borrow SSC (10+2) 2017
SSC (10+2) 2013 10. (a) Mother-in-law (b) Nephew
4. (a) Flower (b) Stem (c) Grandson (d) Great Grandfather
(c) Branch (d) Roots OPSC 2018 SSC (CGL) 2017
5. (a) Sky (b) Star 11. (a) Geometry (b) Trigonometry
(c) Planet (d) Comet (c) Algebra (d) Mathematics
(e) Moon
Chapter 02 CLASSIFICATION 17

12. (a) Sea (b) River Directions (Q. Nos. 31-49) In each of the
(c) Ocean (d) Swimming Pool following question, some groups of letters are
SSC (CPO) 2017 given, all of which, except one, share a common
13. (a) Gold (b) Silver (c) Bronze (d) Iron feature while one is different. Choose the odd one
out.
14. (a) Exact estimate (b) Only choice
(c) Clearly visible (d) Open secret 31. (a) MNW (b) OPY
CLAT 2017 (c) JKT (d) GHO OPSC 2018

15. (a) Rival (b) Opponent 32. (a) PLH (b) MHD (c) NJF (d) KGC
(c) Foe (d) Ally SSC (CGL) 2014 SSC (10 + 2) 2017

16. (a) Brass (b) Gun metal 33. (a) HKI (b) UXV (c) CFD (d) MQN
(c) Bronze (d) Germanium (e) GJH
SSC (Steno) 2013 34. (a) EDC (b) MLK (c) NPR (d) XWV
17. (a) Rain (b) Shower (c) Sleet (d) Raisin SSC (Steno) 2016

18. (a) Distinguish (b) Scatter 35. (a) HJN (b) JLP (c) PRU (d) QSW
(c) Differentiate (d) Classification (e) ACG PNB (Clerk) 2011
SSC (CGL) 2016 36. (a) TPLI (b) YUQM (c) RNJF (d) SOKJ
19. (a) Disperse (b) Congregate SSC (CGL) 2015

A
(c) Accumulate (d) Aggregate OPSC 2018
37.

M
(a) MORV (b) CEHL (c) CENT (d) JLOS
20. (a) Mis deed (b) Corruption SSC (CGL) 2013

H
(c) Failure (d) Offence 38. (a) NQTW (b) PSVZ (c) WZCF (d) BEHK

TT
(e) Villainy SSC (CPO) 2017

21. (a) Shorthand (b) Morse 39. (a) MOQS (b) ZADE
(c) Semaphore (d) Record MPPSC 2018 (c) CEGI A (d) SUWY
W
SSC (Steno) 2016
22. (a) Chameleon (b) Crocodile
SH

(c) Alligator (d) Locust 40. (a) DACB (b) EBCD


SSC (10+2) 2014 (c) SPQR (d) XUVW
23. 41.
A

(a) Cruise (b) Crusade (a) XTCG (b) NJMQ


(c) Campaign (d) Expedition CLAT 2017 (c) EAUZ (d) SOHL
@

24. (a) Blackmail (b) Smuggling 42. (a) BdEg (b) PrSu
(c) Snobbery (d) Forgery (c) KmNp (d) TwXz
(e) Sabotage (e) HjKm SSC (CPO) 2017

25. (a) Scurvy (b) Rickets 43. (a) FIJL (b) RUVX
(c) Night-blindness (d) Influenza (c) DGHJ (d) NPQS
SSC (CGL) 2013
44. (a) AEFJ (b) KOPT
26. (a) Herd (b) Flight (c) UYZD (d) EHIL SSC (CGL) 2016
(c) Hound (d) Swarm
SSC (Steno) 2013 45. (a) ABYZ (b) CDWX
(c) EFUV (d) GHTV
27. (a) Foal (b) Hen (c) Lamb (d) Leveret RRB (ASM) 2012
BSSC (CGL) 2015
46. (a) DFIMR (b) CEHLQ
28. (a) Agitation (b) Confusion (c) GILPU (d) HJMPT SNAP 2012
(c) Commotion (d) Annihilation
SSC (Steno) 2016 47. (a) B C D G (b) G I J L
(c) P R S U (d) U W X Z
29. (a) Poland (b) Korea
SSC (10+2) 2014
(c) Spain (d) Greece
SSC (10+2) 2014 48. (a) CDFE (b) JKLM
30. (a) Bang (b) Hiss (c) STVU (d) WXZY
(c) Whistle (d) Wink (e) HIKJ LIC (ADO) 2012
18 How to Crack Test of Reasoning VERBAL

49. (a) ZOIV (b) QIEM (c) HIUL (d) TEAP 66. (a) Pen, Stationery (b) Earth, Moon
SSC (CPO) 2015 (c) Sun, Star (d) Painter, Artist
SSC (Steno) 2016
50. Arrange these letters of each group to make
a meaningful word and then find the odd 67. (a) Circle : Arc (b) Line : Dot
one out. SSC (CGL) 2015 (c) Hexagon : Angle (d) Square : Line
SSC (10+2) 2013
(a) GHIET (b) VEENS (c) ORFU (d) VIDEID
Directions (Q. Nos. 51-61) In each of the
68. (a) Captain-Team
(b) Boss-Gang
following question, four/five numbers are given.
(c) Prime Minister-Cabinet
Out of these, three/four are alike in a certain way
(d) Artist-Troupe RRB (Group ‘D’ ) 2012
but one is different. Choose the one which is
different from the rest three/four. 69. (a) Apple : Jam (b) Lemon : Citrus
(c) Orange : Squash (d) Tomato : Purie
51. (a) 144 (b) 169 (c) 196 (d) 210
52. (a) 8110 (b) 9100 (c) 1189 (d) 1234
70. (a) Cow : Fodder (b) Crow : Carrion
SSC (CPO) 2015
(c) Poultry : Farm (d) Vulture : Prey

53. (a) 68 (b) 85 (c) 153 (d) 174 71. (a) Gold : Ornaments (b) Pitcher : Pottery
(c) Twigs : Nest (d) Wood : Furniture
SSC (CPO) 2015
72.

A
54. (a) 120 (b) 168 (c) 290 (d) 380 (a) Flag : Flagship (b) Court : Courtship
(c) War : Worship (d) Friend : Friendship

M
(e) 728
55. (a) 84 (b) 120 (c) 72 (d) 108 73. (a) Tree : Stem (b) Face : Eye

H
(e) 98 BOI (PO) 2008 (c) Chair : Sofa (d) Plant : Flower

TT
56. (a) 143 (b) 257 (c) 195 (d) 15 74. (a) Lion : Fox (b) Dog : Cat
(c) Crow : Cuckoo (d) Chief Minister : MLA
(e) 63 SBI (PO) 2008
A
(e) Husband : Wife
W
57. (a) 2378 (b) 7562 (c) 6662 (d) 1155
SSC (10+2) 2017 75. (a) Needle-Prick (b) Gun-Fire
SH

58. (a) 272 (b) 210 (c) 240 (d) 304 (c) Auger-Bore (d) Chisel-Carve
SSC (CGL) 2013 SSC (10+2) 2012
A

76. (a) Chandragupta : Mauryan


59. (a) 4867 (b) 5555 (c) 6243 (d) 6655 (b) Babar : Mughal
@

RRB (Group ‘D’) 2011


(c) Kanishka : Kushan
60. (a) 1256 (b) 5397 (c) 8765 (d) 9842 (d) Mahavira : Jainism SSC (10+2) 2012
SSC (10 + 2) 2017
77. (a) Principal : School (b) Soldier : Battalion
61. (a) 26 (b) 34 (c) 72 (d) 46 (c) Artist : Troupe (d) Singer : Chorus
(e) 38 Vijaya Bank (PO) 2008
78. (a) Stale : Fresh (b) Truth : Lie
Directions (Q. Nos. 62-63) Choose the odd term (c) Slow : Sluggish (d) Teach : Learn
from the given alternatives. (e) Kind : Cruel IBPS (PO) 2013
62. (a) KQ14 (b) AY13 (c) MR11 (d) GW15 79. (a) Cockroach : Antennae
MAT 2014 (b) Lizard : Flagella
63. (a) A8C (b) D22G (c) H42M (d) F34H (c) Hydra : Tentacles
Directions (Q. Nos. 64-80) In each of the (d) Plasmodium : Cilia
following question, four/five word pairs of words 80. (a) Social Science : Physics
are given. Out of these words, one pair does not (b) Mathematics : Algebra
bear the common relationship which others bear. (c) University : Harvard
You are required to find that odd pair. (d) Fruits : Banana UP Police (Constable) 2018
64. (a) Lion-Roar (b) Snake-Hiss Directions (Q.Nos. 81-99) Choose the odd numeral
(c) Bees-Hum (d) Frog -Bleat pair/group in each of the following question.
(e) Dog-Bark IBPS (Clerk) 2011
81. (a) 57 − 53 (b) 47 − 41
65. (a) Death : Disease (b) Milk : Butter (c) 67 − 63 (d) 71 − 67
(c) Grape : Wine (d) Water : Electricity SSC (10+2) 2017
Chapter 02 CLASSIFICATION 19

82. (a) (29, 31) (b) (47, 49) (c) (5, 7) (d) (11, 13) 93. (a) 9 − 72 (b) 8 − 56 (c) 11 − 115 (d) 10 − 90
SSC (CPO) 2015 SSC (CGL) 2014
83. (a) 8-15 (b) 25-36 (c) 49-64 (d) 81-100 94. (a) 1 : 2 (b) 3 : 28 (c) 4 : 65 (d) 2 : 7
UP (B.Ed.) 2011
95. (a) 22 : 42 (b) 4 : 6
84. (a) 46, 57 (b) 38, 49 (c) 41, 52 (d) 64, 73 (c) 11 : 20 (d) 5 : 14
85. (a) 12-24 (b) 14-28 (c) 23-46 (d) 36-70 96. (a) 6 : 23 (b) 3 : 11
SNAP 2011 (c) 1 : 3 (d) 5 : 18

86. (a) (42, 14) (b) (69, 23) (c) (108, 36) (d) (56,19) 97. (a) (13, 2028) (b) (11, 1210)
SSC (CGL) 2015 (c) (7, 336) (d) (9, 648)
SSC (CPO) 2017
87. (a) (81, 243) (b) (16, 64) (c) (64, 192) (d) (25, 75)
SSC (CGL) 2016
98. (a) 2,15,13 (b) 7,12,4
(c) 4,15,11 (d) 6,18,12
88. (a) 14, 17, 23 (b) 19, 22, 28
(c) 17, 20, 26 (d) 21, 23, 30 99. (a) 0-3-8 (b) 9-12-16
SSC (Steno) 2013 (c) 17-20-24 (d) 51-54-58
SSC (Steno) 2011
89. (a) 20, 16, 18 (b) 14, 11, 13
(c) 18, 14, 16 (d) 16, 12, 14 100. Which one set of the following sets
of numbers (3, 5, 19, 13), (11, 19, 2, 3), (3, 29, 2,

A
90. (a) 9, 49 (b) 13, 121 (c) 10, 61 (d) 7, 25
13), (2, 31, 11, 4) is different (if any) from the

M
SSC (CPO) 2013
other sets, in accordance with some rule?
91.

H
(a) 5-21 (b) 29-45 (c) 48-68 (d) 71-87
CGPSC (Pre) 2014
SSC (10+2) 2013

TT
(a) (3, 5, 19, 13) (b) (11, 19, 2, 3)
92. (a) 7 : 98 (b) 9 : 162 (c) 12 : 288 (d) 7 : 572 (c) (3, 29, 2, 13) (d) (2, 31, 11, 4)
SSC (CPO) 2014
(e) None of these
A
W
Answers
SH
A

1. (a) 2. (b) 3. (d) 4. (a) 5. (a) 6. (d) 7. (a) 8. (d) 9. (d) 10. (a)
@

11. (d) 12. (d) 13. (c) 14. (d) 15. (d) 16. (d) 17. (d) 18. (a) 19. (a) 20. (c)
21. (d) 22. (d) 23. (a) 24. (c) 25. (d) 26. (c) 27. (b) 28. (b) 29. (b) 30. (d)
31. (d) 32. (b) 33. (d) 34. (c) 35. (c) 36. (c) 37. (c) 38. (b) 39. (b) 40. (a)
41. (c) 42. (d) 43. (d) 44. (d) 45. (d) 46. (d) 47. (a) 48. (b) 49. (c) 50. (d)
51. (d) 52. (c) 53. (d) 54. (d) 55. (e) 56. (b) 57. (d) 58. (c) 59. (d) 60. (d)
61. (c) 62. (c) 63. (d) 64. (d) 65. (a) 66. (b) 67. (c) 68. (d) 69. (b) 70. (c)
71. (b) 72. (c) 73. (c) 74. (d) 75. (a) 76. (d) 77. (a) 78. (c) 79. (b) 80. (a)
81. (b) 82. (b) 83. (a) 84. (d) 85. (d) 86. (d) 87. (b) 88. (d) 89. (b) 90. (c)
91. (c) 92. (d) 93. (c) 94. (d) 95. (d) 96. (d) 97. (c) 98. (b) 99. (a) 100. (d)

Answer with Explanations


1. (a) Except Sun, all other are planets. 4. (a) Stem, branch and roots supply water to different
parts of the plant. So, flower is the odd one.
2. (b) Cotton is used to make thread and thread is
used to make cloth. So, plastic is the odd one. 5. (a) Except ‘Sky’, all others belong to the same
3. (d) Except ‘Borrow’, all the options are the terms of class i.e. celestial bodies.
business. 6. (d) Except Agra, all others are hill stations.
20 How to Crack Test of Reasoning VERBAL

7. (a) Escalater, ladder and staircase are used to 29. (b) Except Korea, all others are European countries
climb to next floor, whereas bridge is used to join while Korea is an Asian country.
two mountains or cliffs. 30. (d) Except wink, all others are different types of
8. (d) Late is the result of the other given options. A sound.
tired, tardy or slow attitude makes a work late. 31. (d) Here,
9. (d) Except Sydney, all others are the capitals of M N W O P Y
different countries while Sydney is one of the city in
Australia. +1 +9 +1 +9
10. (a) Except mother-in-law, all others are males J K T G H O
whereas mother-in-law is a female. Hence,
mother-in-law is different from others. +1 +9 +1 +7
11. (d) Except Mathematics, all others come under the So, it is clear from above that GHO is different from
field of Mathematics. others.
12. (d) Except Swimming pool, all others are natural 32. (b) Here,
resources of water.
P L H M H D
13. (c) Except Bronze, all others are pure metals
whereas bronze is an alloy. –4 –4 –5 –4

A
14. (d) Except open secret, all other given options
N J F K G C

M
represent certainity. Open secret is odd as it
alludes uncertainity.

H
–4 –4 –4 –4
15. (d) Except Ally, all others are harmful for us.

TT
So, it is clear from above that MHD is different
16. (d) Brass, bronze and gun metal are alloys while
from others.
Germanium is an element.
33. (d) H K A I U X V C F D
W
17. (d) Here, all options are related to rain or water
, ,
except Raisin. ‘Raisin’ is a partially dried grapes, +3 –2 +3 –2 +3 –2
SH

which is different from all others.


M Q N G J H
18. (a) Except distinguish, all others are related to and
A

separation. +4 –3 +3 –2
@

19. (a) Except Disperse, all others are synonyms. So, it is clear from above that MQN is different
from others.
20. (c) Except ‘Failure’, all others are illegal activities.
34. (c) E D C M L K
21. (d) All except Record are the brief notations in a ,
language, while Record is a detailed account.
–1 –1 –1 –1
22. (d) Except Locust, all others belong to reptile
category. N P R X W V
23. (a) Cruise is a journey by ship, while all others are ,
synonyms. +2 +2 –1 –1

24. (c) Except ‘Snobbery’, all other words are related to So, it is clear from above that NPR is different
crime. from others.
25. (d) Except ‘Influenza’, all other diseases are 35. (c) H J N J L P P R U Q S W
caused by the lack of vitamins. , , ,
+2 +4 +2 +4 +2 +3 +2 +4
26. (c) Except Hound, all others represent group of
insects or animals. While Hound is a special breed A C G
of dog. and
+2 +4
27. (b) Except Hen, all others are young ones of So, it is clear from above that PRU is different
different animals. from others.
28. (b) Except confusion, all others are done in group.
36. (c) Except RNJF, all others contain one vowel.
Confusion can be felt by an individual.
Chapter 02 CLASSIFICATION 21

37. (c) M O R V C E H L 43. (d) F I J L R U V X


, ,
+2 +3 +4 +2 +3 +4 +3 +1 +2 +3 +1 +2
C E N T J L O S D G H J N P Q S
and and
+2 +9 +6 +2 +3 +4 +3 +1 +2 +2 +1 +2
From above, it is clear that letters’ group CENT is So, it is clear from above that NPQS is
different from all others. different from others.
38. (b) N Q T W P S V Z 44. (d) A E F J
,
+4 +1 +4
+3 +3 +3 +3 +3 +4 K O P T
W Z C F B E H K
, +4 +1 +4
+3 +3 +3 +3 +3 +3 U Y Z D
So, it is clear from above that PSVZ is different +4 +1 +4
from others.
E H I L
39. (b) M O Q S Z A D E
, +3 +1 +3

A
+2 +2 +2 +1 +3 +1 So, it is clear from above that EHIL is different from

M
C E G I S U W Y others.
,

H
45. (d) Except option (d), all three are same because in
+2 +2 +2 +2 +2 +2
those three options first and fourth and second

TT
So, it is clear from above that ZADE is different and third are opposite letters.
from others,
40. (a) D   −3
→ A   +2
→ C  −1
→B
46. (d) D F I
A M R C
,
E H L Q
,
W
−3 +1 +1
E 
→ B  → C  → D +2 +3 +4 +5 +2 +3 +4 +5
−3 +1 +1
SH

S 
→ P   → Q  →R G I L P U H J M P T
−3 +1 +1
X  → U   → V  →W and
+3 +4
A

+2 +5 +2 +3 +3
So, it is clear from above that DACB is different +4
from others. So, it is clear from above that HJMPT is
@

41. (c) X different from others.


T C G N J M Q
47. (a) B C D G G I J L
–4 +4 –4 +4
, ,
E A U Z S O H L +1 +1 +3 +2 +1 +2

P R S U U W X Z
–4 +5 –4 +4 and
So, it is clear from above that EAUZ is different +2 +1 +2 +2 +1 +2
from others. So, it is clear from above that BCDG is different
42. (d) B d E g P r S u from others.
, , 48. (b) C D F E J K L M
+2 +1 +2 +2 +1 +2 ,
K m N p +1 +2 –1 +1 +1 +1
S T V U W X Z Y
+2 +1 +2
,
T w X z H j K m +1 +2 –1 +1 +2 –1
and H I K J
+3 +1 +2 +2 +1 +2
and
+1 +2 –1
So, it is clear from above that TwXz is different
from others. So, it is clear, from above that JKLM is different
from others.
22 How to Crack Test of Reasoning VERBAL

49. (c) Z O I V, Q I E M, H I U L and T E A P 60. (d) Here,


Except H I U L, all others have two consecutive 7 12 13 11
vowels. But in H I U L, one vowel is skipped
between I and U. i.e. O. 1 2 5 6 5 3 9 7 8 7 6 5 9 8 4 2
50. (d) As, GHIET → EIGHT
VEENS → SEVEN 7 12 13 12
ORFU → FOUR Sum of middle two digits and of extreme two digits
VIDEID → DIVIDE are same, but in 9842, it is different.
So, it is clear that DIVIDE does not belong to 61. (c) Except ‘72’ all others give a prime number,
group. when divided by 2.
51. (d) Except 210, all others are square of natural 62. (c) In all other groups except (c), number at the
numbers. end is half of the sum of positional value of first
52. (c) Except 1189, sum of digits of all other numbers and second letters in the alphabet.
is 10. 63. (d) In all other groups except (d), number between
53. (d) Except 174, all others are multiple of 17. first and second letters is twice the sum of their
positional values.
54. (d) Each of the numbers except 380, is either one
64. (d) Except frog-bleat, in all other pairs the sound of

A
less or one more than the square of a certain
number. the given animal is written. Frog-bleat is different

M
because frogs don’t bleat, they croak.
55. (e) Except ‘98’, all others are multiples of 6.

H
65. (a) In all other pairs, second is a product obtained
84 → 6×14, 120 → 6×20,

TT
from the first.
72 → 6×12, 108 → 6×18
66. (b) Except (b), in all others, first thing falls under
56. (b) Here, 143 → 12 2 − 1257 →162 + 1195 → 142 − 1,
, ,
A
the second. Earth is a planet not moon.
W
15→ 4 − 1, 63 → 8 − 1
2 2
67. (c) Circle is made from arcs, line is made from
dots and square is made from lines but hexagon is
SH

Hence, ‘257’ is the odd one.


not made from an angle.
57. (d) Here, 2378 ⇒ 2 + 3 + 7 + 8 = 20
68. (d) In all pairs except (d), first is the head of
A

7562 ⇒ 7 + 5 + 6 + 2 = 20
second.
@

6662 ⇒ 6 + 6 + 6 + 2 = 20
69. (b) In all pairs except (b), second is made from first.
1155 ⇒ 1 + 1 + 5 + 5 = 12
70. (c) In all pairs except (c), second is the food for first.
Clearly, 1155 is the odd one.
58. (c) Here, 272 = 2 + 7 + 2 = 11 (Prime) 71. (b) In all pairs except (b), first is the material used
to make the second.
210 = 2 + 1 + 0 = 3 (Prime)
240 = 2 + 4 + 0 = 6 (Non-prime)
72. (c) In all other pairs except (c), second is the
abstract noun of first.
304 = 3 + 0 + 4 = 7 (Prime)
Clearly, 240 is the odd one. 73. (c) In all other pairs except (c), second is a part of
the first.
59. (d) As, 4867 → 4 + 8 + 6 + 7 = 25,
which is divisible by 5. 74. (d) In all other pairs except (d), the first is
masculine and the other is feminine.
5555 → 5 + 5 + 5 + 5 = 20,
which is divisible by 5. 75. (a) Except Needle-Prick, in all other options the
6243 → 6 + 2 + 4 + 3 = 15, work of the given instrument is written on the right.
which is divisible by 5. 76. (d) In all pairs except (d), first is the founder of
But, 6655 → 6 + 6 + 5 + 5 = 22, second.
which is not divisible by 5. 77. (a) In all other pairs, except (a) second is a
So, 6655 is different. collective group of the first.
Chapter 02 CLASSIFICATION 23

78. (c) In all other pairs except (c), both the words are 90. (c) Here, 9 − 2 = 7 → 7 2 = 49
antonyms of each other. 13 − 2 = 11 → 112 = 121
79. (b) In all other pairs except (b), second is the organ 10 − 2 = 8 → 82 = 64 ≠ 61
used by first for its movement.
But, when bacteria senses either presence of 7−2 = 5 → 52 = 25
nutrients or harmful conditions, the flagella play its So, (10, 61) is different from others.
role. If bacteria requires nutrients, flagella starts the
+16 +16
movement towards it. 91. (c) Here, 5 → 21, 29 → 45
80. (a) In all pairs except (a), second completely belongs +16 +20
71 → 87 and 48 → 68
to first.
81. (b) Except 47 − 41, all other number pairs have a So, ‘48-68’ is different from other three.
difference of 4. 92. (d) Except (d), in all other options, second
82. (b) Except (47, 49), all other options have prime number is the multiple of the first number.
numbers. 93. (c) As, 9 × (9 − 1) ⇒ 9 × 8 = 72
83. (a) Except 8-15, in all other options, both 8 × (8 − 1) ⇒ 8 × 7 = 56
the numbers are the square of natural numbers. 10 × (10 − 1) ⇒ 10 × 9 = 90
84. (d) Except 64-73, all others are having a difference of 11. But, 11 × (11 − 1) ⇒ 11 × 10 = 110 ≠ 115

A
85. (d) Except in option (d), the second number is twice So, 11-115 is different from others.

M
of first number in all other options.
94. (d) In all other pairs except (d), second number

H
86. (d) The pattern is as follows is one more than the cube of the first number.

TT
(42, 14) → 14 × 3 = 42 95. (d) In all other pairs except (d), first number is
(69, 23) → 23 × 3 = 69 one more than the half of the second number.
(108, 36) → 36 × 3 = 108 A
96. (d) In all other pairs except (d), second number
W
But (56, 19) → 19 × 3 = 57 ≠ 56 is one less than four times the first number.
97. (c) Here, 13 − 2028 → (13)3 − (13)2 = 2028
SH

So, option (d) is the odd one.


×3
87. (b) Here, 81  → 243 ; ×3
16  → 48 ≠ 64 11 − 1210 → (11)3 − (11)2 = 1210
A

×3
64  → 192 ; ×3
25  → 75 7 − 336 → (7 )3 − (7 )2 = 294
@

9 − 648 → (9)3 − (9)2 = 648


Hence, (16, 64) is different from all others.
88. (d) Here, 14 + 3 = 17 → 17 + 6 = 23 So, it is clear from above that 7 − 336 is different
19 + 3 = 22 → 22 + 6 = 28 from others.
17 + 3 = 20 → 20 + 6 = 26 98. (b) Except (b) in all others, middle number is the
sum of other two numbers.
21 + 2 = 23 → 23 + 7 = 30
99. (a) Here,
So, (21, 23, 30) is different.
0 3 8 9 12 16 17 20 24 51 54 58
89. (b) The pattern is as follows and
, ,
18 14 11 13 +3 +5 +3 +4 +3 +4 +3 +4
20 16

+2 +2
Hence, 0-3-8 is different from other three.
+4 +3
18 14 16 16 12 14 100. (d) Except set in option (d), all other sets
contain only prime numbers. But set (d)
+4 +2 +4 +2 contains one composite number, i.e. 4.
Hence, (14, 11, 13) is the odd one.
How to Crack Test of Reasoning VERBAL

03
Alphabet Test
‘Alphabet Test’ has questions based on letters 5. Opposite letter of a letter can be obtained by
of English alphabet. These questions require subtracting the position of letter from 27. The
l
the understanding of the position of letters in answer obtained is the position of the
English alphabet. opposite letter. You can also remember the
opposite letters of English alphabet by
l
detection of the rule in which letters are

A
following the below trick:
arranged.

M
l
finding a letter or number following a Opposite Opposite
Trick Trick
Letters Letters

H
particular rule in an alpha-numeric sequence
etc. A–Z A to Z or Aaza B–Y By

TT
Some important points related to Alphabet test C–X CT Scan-X-RAY D–W Dew
are given below: E–V A Evening F–U First U (You)
W
1. Positional values of letters in English G–T GT Road H–S High School
alphabet.
SH

I–R Indian-Railway J–Q Jack-Queen


Forward Alphabet Backward Forward Alphabet Backward
K–P Kevin Peterson L–O Light ON
A

1 A 26 14 N 13
2 B 25 15 O 12 M–N Man
@

3 C 24 16 P 11
4 D 23 17 Q 10
5
6
E
F
22
21
18
19
R
S
9
8
Types of Questions
7 G 20 20 T 7 There are four types of questions based on the
8 H 19 21 U 6 alphabet test which are generally asked in
9 I 18 22 V 5 various competitive examinations.
10 J 17 23 W 4
11 K 16 24 X 3
12 L 15 25 Y 2 Type 1
13 M 14 26 Z 1
Finding a Particular Letter in
2. A, E, I, O and U are vowels of English
alphabet and remaining letters are English Alphabet
consonants of English alphabet. In this type, the questions asked are based on
3. A to M letters are called the first half of finding out the letter at a specific position
English alphabet.
with respect to another letter in the English
4. N to Z letters are called the second half of
alphabetical order.
English alphabet.
Chapter 3 ALPHABET TEST 25

Some important formulae to find the position of Illustration 3. How many letters are there
a letter in English Alphabet are given below: between 8th letter from left and 7th letter from
1. nth letter to the left of mth letter from right right in the English alphabet?
end = (m + n)th letter from the right end. (a) 7 (b) 11 (c) 8 (d) 9
2. nth letter to the right of mth letter from left Solution (b) Let us see
end = (m+ n)th letter from the left end. 1 2 3 4 5 6 7 8 9 10 11 1213 14 1516 17 18 19 20 21 2223 24 25 26
3. nth letter to the left of mth letter from left end A BCD E FGH I J K L M N O P Q R S T U V W X Y Z
= (m − n)th letter from the left end. 11 letters
From left From right
4. nth letter to the right of mth letter from right 8th 7th
end = (m − n)th letter from the right end.
So, there are 11 letters between 8th letter from left
5. Total number of letters between mth letter
from left and nth letter from right and 7th letter from right.
= 26 − (Position of mth letter + Position of nth Alternate Method (By Formula 5)
letter) Total number of letters in the English alphabet = 26
Number of letters in between
Illustration 1. Find the 11th letter to the left of
= Total number of letters in English alphabet
20th letter from left in the English alphabet.
– (letter from the left + letter from the right)
(a) D (b) J (c) K (d) I

A
∴ Required number of letters = 26 − (8 + 7) = 11
Solution (d) Let us see

M
Illustration 4. If 1st half of the English alphabet is

H
1 2 3 4 5 6 7 8 9 10 11 12 13 14 15 16 17 18 19 20 21 22 23 24 25 26
A B CD E F GH I J K L M NOPQR S T U VWX Y Z written in reverse order, then find the 15th

TT
To the left letter from right. Canara Bank (Clerk) 2008

11th (a) A (b) B


From left (c) C A (d) D
W
20th (e) E
Hence, 11th letter to the left of 20th letter from left
SH

Solution (b) According to the question,


is I.
15th 1st
Alternate Method (By Formula 3) From right
A

In English alphabet, 11th letter to the left of


@

20th letter from left = (20 − 11)th letter from left M C B A N O Z


= 9th letter from left = I

Illustration 2. If English alphabet is written in 13 letters of 1st 13 letters of 2nd half


backward order, then what will be the 13th half are reversed remain unchanged
letter to the left of the 3rd letter from right?
(a) P (b) N ∴ The required letter is B.
(c) R (d) Q
Solution (a) Backward order is written as
1 2 3 4 5 6 7 8 9 10 11 12 1314 15 16 1718 19 20 21 22 23 24 25 26
Type 2
Z Y XWV U T S R Q P O N M L K J I H G F E D C B A
Alpha-Numeric Sequence
13th To the left
From right
In this type of questions, a jumbled sequence of
3rd some letters, numbers and symbols are given.
Therefore, the 13th letter to the left of the 3rd letter Sometimes only English alphabets from A to Z
from right is P. are given.
Alternate Method (By Formula 1) The candidate is required to find the number
In backward order of alphabet, 13th letter to the left of times a particular letter/number/symbol
of 3rd letter from right = (3 + 13) th letter from right occurs in the sequence following a certain
= 16th from right = P condition.
How to Crack Test of Reasoning VERBAL
26

Illustration 5. In the following letter series, how


many times do PQR occur in such a way that Q Type 3
is in the middle of P and R?
Letter-Word Problems
QMPNPQRROPQNOPPQRPMQROPQRPPRR
PQRP In this type of questions, a word is given and
(a) 5 (b) 6 (c) 4 (d) 3 candidates are required to answer the questions
asked on the basis of given word e.g. how many
Solution (c) Here, Q M P N P Q R R O P Q N O letter(s) in the word is as far from the beginning
PPQRPMQROPQRPPRR PQRP of the word as it is from the beginning of the
So, in the above arrangement, PQR occur 4 times English alphabet, how many letters remain
in such a way that Q is in the middle of P and R. same in its position, if they are arranged in
alphabetical order etc.
Directions (Illustrations 6-8) Study the
following arrangement carefully and answer the Illustration 9. How many such pairs of letters are
questions given below. there in the word ‘CHANNEL’, which have as
F@39 HADI%4E $MK2URP5Wδ many letters between them in the word as in
81TJV7 the English alphabet?
(a) None (b) One
Illustration 6. How many consonants are there in

A
(c) Two (d) Three
the given arrangement, which are preceded by (e) More than three

M
a number and not followed by a letter? Solution (c) According to the question,

H
(a) None (b) One (c) Two (d) Three
C H A N N E L

TT
(e) More than three
Solution (c) Here, F @ 3 9 H  A D I % 4 E $M
K2URP5Wδ 81TJV7
A
So, there are two such pairs i.e. AC and LN.
W
In the given arrangement, 2 such consonants are
Illustration 10. How many such letters are there
there viz. 9 H  and 5 W δ which are preceded by
SH

in the word ‘PMCLNOB’ which remain same in


a number and not followed by a letter.
its position, if they are arranged in an
Illustration 7. Four of the following five options alphabetical order?
A

are alike in a certain way, based on their (a) None (b) One
@

positions in the above arrangement and so form (c) Two (d) Three
a group. Which is the one that does not belong (e) More than three
to that group? SSC (CPO) 2015 Solution (c) According to the question,
(a) D %  (b) 5 δ R (c) T V 8 (d) E % $
Given word P M C L N O B
Solution (d) Here, F @ 3 9 H  A D I % 4 E $ Alphabetical order B C L M N O P
MK2URP5Wδ 81TJV7
So, such type of letters are N and O.
In all the groups except ‘E % $’, second element is
two positions ahead of first element and the third Illustration 11. If each vowel in the word
element is two positions behind to the first element. ‘GLADIOLUS’ is substituted with the next
letter of the English alphabetical series and
Illustration 8. If all the symbols from the above each consonant is substituted with the letter
arrangement are dropped, then which of the preceding it. How many vowels are present in
following will be the twelfth from the left end? the new arrangement?
(a) 4 (b) K (c) 2 (d) U (a) None (b) One (c) Two (d) Three
(e) None of these (e) None of these
Solution (c) After dropping all the symbols, the Solution (a) According to the question,
resultant arrangement will be Given word G L A D I O L U S
F39HADI4EMK 2 URP5W81TJV7 ↓ ↓ ↓ ↓ ↓ ↓ ↓ ↓ ↓
New word F K B C J P K V R
12th from left
So, no vowel is present in the new arrangement.
So, twelfth element from the left is 2.
Chapter 3 ALPHABET TEST 27

Type 4 Solution (b) According to the question,


G J M P S
Rule Detection HI KL NO QR
In this type of questions, the alternatives are Hence, GJMPS follows the pattern.
given as the group of letters and out of these
groups, candidates are asked to choose the Illustration 13. Number of letters skipped in
correct alternative which follows the rule given between adjacent letters in the series decreases
in the question. by one. Which of the following series is
observing the rule?
Illustration 12. Find out the correct alternative in
(a) OXMYA (b) AZXUA (c) QUXZA (d) AEIKL
which number of letters skipped in between
adjacent letters in the series, is two. Solution (c) According to the question,
(a) LORTW Q U X Z A
(b) GJMPS
+4 +3 +2 +1
(c) KNPSV
(d) EHKLO Hence, QUXZA follows the pattern.

A
Let us Practice

M
H
TT
Base Level Exercise
1. Which of the following letters is 14th to the 7. If the 1st half of the English alphabet is
right of 6th letter from the left in the A
reversed and so is the 2nd half, then which
W
English alphabet? letter is 7th to the right of the 12th letter from
SH

(a) R (b) P (c) W (d) T the left side?


(a) S (b) U (c) R (d) T
2. Which letter is between midway of 8th letter
A

from left and 7th letter from right in the 8. All the English alphabets are arranged
@

English alphabet? alphabetically in the reverse order. The


(a) N (b) M (c) P (d) O difference of the positions of two vowels
3. If the English alphabet is written in which appear in the beginning and in the
backward order, then which letter will be 5th end, is CGPSC 2014
(a) 6 (b) 12
to the left of letter M? MAT 2011
(c)16 (d) 18
(a) G (b) H (c) S (d) R
(e) None of these
4. If English alphabet is written in the
backward order, then which letter is 7th to 9. In the following series, how many KGN
the right of K? UP (B.Ed.) 2011 occur in such a way that ‘G’ is in the middle
(a) A (b) B (c) C (d) D and ‘K’ and ‘N’ are adjacent to it on both
sides? SSC (CGL) 2014
5. If the English alphabet is written in reverse AKGLMNDQKGCSNGKTGKGNDZPUXGKE
order, then which letter is 7th to the left of
(a) 3 (b) 5
11th letter from right? SSC (10+2) 2008
(c) 1 (d) 2
(a) W (b) H (c) R (d) D
6. If the 2nd half of the English alphabet is 10. In the following letter series, how many M’s
written in the reverse order, then find the are followed by N, but not preceded by N?
15th letter from right. LIC (ADO) 2009 SSC (CPO) 2014
(a) M (b) L (c) K (d) J NMWVMNMVWNMNMMNWVMN
(e) O (a) 1 (b) 2 (c) 3 (d) 4
How to Crack Test of Reasoning VERBAL
28

11. If the following series is written in reverse 18. How many such pairs of letters are there in
order and then all the vowels are deleted, the word ‘PREAMBLE’, each of which has as
which will be the 8th letter from the right in many letters between them as in the English
the new series? alphabet?
ABCDEFGHIJKLMNOPQRSTU (a) None (b) One (c) Two (d) Three
VWXYZ OPSC 2018 19. How many such pairs of letters are there in
(a) L (b) H the word ‘STREAMING’ each of which has
(c) K (d) None of these as many letters between them in the word as
12. In the following letter series, how many BCN in the English alphabet (in both forward and
occur in such a way that C is in the middle backward directions)? IBPS (Clerk) 2011
and B and N are on any one side? (a) None (b) One (c) Two (d) Three
(e) More than three
[SSC (10+2) 2013]
BCMXNCXNBXNCBNCBYBCX 20. How many such pairs of letters are there in
NBCNABONMZCB the word ‘SUBSTANCE’, each of which has
(a) 4 (b) 2 (c) 5 (d) 3 as many letters between them in the word
(in both forward and backward directions) as
13. How many T’s are both immediately in the English alphabet? IBPS (PO) 2012
preceded and followed by ‘E’ in the

A
(a) None (b) One (c) Two (d) Three
following series? UP Police (Constable) 2018 (e) More than three

M
ET ETTMEEETETETETTEETTT
21. How many such pairs of letters are there in

H
EEETETETETTEETE
the word ‘STONED’, each of which has as

TT
(a) 7 (b) 6
(c) 5 (d) 8
many letters between them in the word
(in both forward and backward directions) as
Directions (Q. Nos. 14-15) Study the following A
they have between them in the English
W
alphabetical sequence and answer the questions alphabetical series?
based on it. IBPS Clerk (Pre) 2017 Punjab Grameen Bank (Clerk) 2011
SH

ABBCDEFEIBCAFECBBACAOBNUVW (a) None (b) Three (c) Two (d) One


(e) More than three
14. If all the vowels are dropped from the series,
A

then which alphabet will be eighth from the 22. If the first and fifth letters of the word
@

left end? ‘BILINGUAL’ were interchanged, also the


(a) C (b) B (c) N (d) F second and sixth letters, and so on, which of
(e) None of these the following would be 7th letter from your
right? OPSC 2018
15. How many vowels are there in the (a) A (b) I (c) G (d) U
alphabetical series which are immediately
preceded by a consonant? 23. How many such pairs of letters are there in
(a) One (b) Two the word REFRESHING, each of which has
(c) Four (d) More than five as many letters between them in the word as
(e) None of these they have in the English alphabet?
16. If the letters of the word ‘DOLPHIN’ are RBI Astt. (Mains) 2017, 2016
arranged as they appear in the English (a) None (b) One (c) Two (d) Three
alphabetical order, which of the following (e) More than three
letter will be the 5th from left? UP (B.Ed.) 2011 24. How many such pairs of letters are there in
(a) O (b) D the word ‘TEACH’ each of which has as
(c) I (d) None of these many letters between them in the word
17. Which letter in the word ‘WASHINGTON’ is (in both forward and backward directions) as
at the same position in the word (counting they have between them in the English
alphabetical series? SBI (Clerk) 2016
from the beginning) as it is in the English
(a) More than three (b) None
alphabet? UPPSC (RO) 2014
(c) Two (d) Three
(a) N (b) T (c) O (d) G
(e) One
Chapter 3 ALPHABET TEST 29

25. If in the word EQUALITY, the positions of 30. Each consonant in the word ‘TIRADES’ is
first and the fifth letters are interchanged, replaced by the previous letter and each vowel
similarly the positions of the second and the is replaced by the next letter in the English
sixth letters are interchanged and so on, alphabet and the new letters are rearranged
which letter will be third from the right end? alphabetically. Which of the following will be
RBI Astt. (Mains) 2017, 2016 the fourth from the right end?
(a) Q (b) U (c) I (d) T (a) F (b) J (c) Q (d) C
(e) None of these (e) None of these
26. The positions of how many alphabets will Directions (Q. Nos. 31-33) In each of the
remain unchanged if each of the alphabets in following questions, find out which of the
word ‘FORGET’ is arranged in alphabetical letter-series follows the given rule.
order from left to right? UBI (PO) 2011 31. Number of letters skipped in reverse order
(a) None (b) One (c) Two (d) Three in between adjacent letters in the series is
(e) More than three constant.
27. If the letters of the word ‘VERTICAL’ are (a) SQOLJ (b) SPNLJ (c) SPMJG (d) WUTRQ
arranged alphabetically, how many letters 32. Number of letters skipped in between the
will remain at the same position? adjacent letters in the series is equal

A
Syndicate Bank (Clerk) 2009 (a) SUXADF (b) RVZDHL

M
(a) Four (b) Three (c) Two (d) One (c) HKNGSW (d) RVZDFG

H
(e) None of these
33. Number of letters skipped in between the

TT
28. If the first and second letters in the word adjacent letters in the series is one.
‘COMMUNICATIONS’ were interchanged, SSC Matric Level 2011
also the third and the fourth letters, the fifth
A
(a) KMPQR (b) HJLMO (c) PRSUW (d) EGIKM
W
and sixth letters, and so on.
34. In a series of letters, find the group in which
Which letter would be the tenth letter
SH

the number of letters skipped in between the


counting from your right? SSC (CGL) 2012
adjacent letters in the series is constant.
(a) U (b) A (c) T (d) N
(a) ZXTN (b) EZUP (c) PRTX (d) RSAB
A

29. In the word ‘MATHEMATICS’, if M is


@

35. In the series given below, find the group in


replaced by P, A by D, T by W and so on, which two letters are skipped in between
then in the new arrangement of the letters, adjacent letters.
the letter of the 9th place from left is (a) BEHKLM (b) LQRUXY
UPPSC (RO/ARO) 2014 (c) EHKNQT (d) FHKOTZ
(a) K (b) L (c) F (d) W

Expert Level Exercise


1. How many such pairs of letters are there in (e) More than three
the word ‘JOURNEY’, each of which has as 3. The position of the first and the fifth letters
many letters between them in the word (in of the word HERITAGE are interchanged,
both forward and backward directions) as similarly, the position of second and sixth
they have between them in the English letters and third and seventh letters and
alphabetical order? UCO Bank (Clerk) 2011 fourth and eighth letters are interchanged. In
(a) None (b) One (c) Two (d) Three the new arrangement thus formed, how
(e) More than three many letters are there between the letters
2. How many such pairs of letters are there in which is fourth from the left and first from
the word ‘CONFUSED’ each of which has as the right in the English alphabetical series?
many letters between them in the word as in IBPS Clerk (Pre) 2016
the English alphabet? LIC (ADO) 2011 (a) Two (b) None
(a) Nil (b) One (c) Two (d) Three (c) More than three (d) Three
How to Crack Test of Reasoning VERBAL
30

4. Which single letter is exactly between the two 13. Number of letters skipped in between the
letters–one is 22nd from left side and the other adjacent letters in the series are 3 and 4
is 19th from right side in the following series? respectively. SSC Matric Level 2011
CGPSC 2014 (a) SWBFKPT (b) NRWBFKO
ZYXWVUTSRQPONMLKJIHGFEDCBA (c) HLPUYDI (d) EINRWAF
(a) M (b) N (c) L (d) K 14. Number of letters skipped in between
(e) None of these adjacent letters decrease in order.
5. If the last four letters of the word (a) SYDHK (b) HNSWA (c) NSXCH (d) AGMRV
‘TABULATION’ are written in reverse order 15. In the following question, number of letters
followed by next two in reverse order and skipped in between adjacent letters in the
next three in the reverse order, counting series increases by one. Which of the
from the end, which letter would be eighth following series observe the rule?
in the new arrangement?
(a) KORYBGJ (b) LMEYIPK
(a) N (b) T
(c) KMPTYEL (d) KPTYELM
(c) E (d) None of these
(e) RVZDFG
6. If the 2nd half of the letters of the word
‘TRANSPORTATIONAL’ are reversed and Directions (Q. Nos. 16-20) Study the following
arrangement carefully and answer the questions

A
placed before 1st half of the letters, which
letter will be the third to the right of the 13th IBPS (Clerk) 2013

M
letter from the right? CEBACDBCDACEBEDCABADAC

H
(a) R (b) N (c) L (d) A EDUBAUBDBU

TT
Directions (Q. Nos. 7-10) In the following word 16. How many such pairs of alphabets are there
‘ELECTROCARDIOGRAPH’, the first half of the in the series of alphabets given in BOLD
letters are reversed, but one letter ‘P’, then prefixed A
(D to C) in the given arrangement each of
W
and finally letter ‘S’ is suffixed. which has as many letters between them (in
both forward and backward directions) as
SH

7. Which letters will be exactly in the middle?


(a) L and E (b) R and D (c) D and I (d) E and R they have between them in the English
alphabetical series?
A

8. How many vowels will be to the left of the (a) Three (b) One
@

middle letter? (c) Two (d) None


(a) 2 (b) 1 (c) 4 (d) 3
(e) More than three
9. Which vowel will have a consonant to the 17. Which of the following is the ninth to the
left but a vowel to the right of it?
right of the twenty-second from the right
(a) I (b) O (c) A (d) E
end of the given arrangement?
10. Name the letter sandwiched between two (a) D (b) E (c) B (d) C
vowels? (e) U
(a) R and T (b) C and L
(c) R and L (d) None of these
18. If all the alphabets of the series are written in
reverse order, then which of the following
11. In a series of letters, find the group in which will be 8th to the right of the 7th from the
the number of letters skipped in between left end of the series?
adjacent letters are increasing as they move (a) A (b) B (c) C (d) D
forward. (e) None of these
(a) A C F J L Q (b) B D G K P V
(c) C E H L Q V (d) A I L M U X 19. How many such A’s are there in the given
arrangement each of which is immediately
12. In a series of letters, find the group in which preceded by a ‘B’ and also immediately
the number of letters skipped in between followed by a consonant?
adjacent letters decrease as they move
(a) One (b) None
forward?
(c) More than three (d) Two
(a) BGKNPR (b) ACEGI
(e) Three
(c) EJNQST (d) LWUXAP
Chapter 3 ALPHABET TEST 31

20. How many such D’s are there in the given 22. How many letters are there between the
arrangement each of which is immediately eighth element from the right and the
preceded by a consonant and also seventh letter from the left end of the given
immediately followed by a vowel? arrangement?
(a) More than four (b) Four (a) Nine (b) Six
(c) Two (d) Three (c) Eight (d) Five (e) Ten
(e) One 23. Which of the following will be ninth to the
Directions (Q. Nos. 21-24) Study the following right of the sixteenth element from the right
arrangement of letters, numbers and symbols end of the given arrangement?
carefully to answer the given questions. (a) # (b) 3
RBI Astt. (Pre) 2016
(c) Z (d) 6 (e) N

9Ω1&LYEKSR8%WH7$5UG4#62NA3@ZÚD 24. If all the letters from the given arrangement


are deleted, then which of the following will
21. As per the given arrangement, four of the
following five are alike in a certain way and represent the fifth element to the right of ‘4’
hence form a group, which one of the and the fourth element to the left of ‘%’
following does not belong to the group? respectively?
(a) W$H (b) %H8 (c) U#G (d) 3Ú@ (e) 1Y& (a) 3, & (b) Ú, @ (c) @,  (d) 3, 1 (e) @, 1

A
M
Answers

H
TT
Base Level Exercise
1. (d) 2. (a) 3. (d) 4. (d) 5. (c) 6. (b) 7. (b)
A
8. (e) 9. (d) 10. (c)
W
11. (c) 12. (d) 13. (d) 14. (d) 15. (d) 16. (d) 17. (d) 18. (c) 19. (d) 20. (d)
SH

21. (b) 22. (d) 23. (d) 24. (c) 25. (a) 26. (b) 27. (e) 28. (d) 29. (b) 30. (b)
31. (c) 32. (b) 33. (d) 34. (b) 35. (c)
A

Expert Level Exercise


@

1. (d) 2. (e) 3. (d) 4. (c) 5. (d) 6. (d) 7. (d) 8. (d) 9. (a) 10. (b)
11. (b) 12. (c) 13. (d) 14. (a) 15. (c) 16. (e) 17. (a) 18. (b) 19. (d) 20. (e)
21. (b) 22. (c) 23. (e) 24. (e)
How to Crack Test of Reasoning VERBAL

Answer with Explanations


Base Level Exercise
1. (d) 14th to the right of 6th letter from the left 8. (e) Reverse order of an English alphabet is as
= (6 + 14)th letter from left follows
= 20 th letter from left = T (by formula 2) ZYXWVUTSRQPONMLKJIHGFEDCBA
2. (a) 7th letter from right = (27 − 7 )th ↓ ↓ ↓ ↓
6 12 18 22 26
= 20 th letter from left
8 + 20 Vowel that appear in beginning in new arrangement
∴Required middle letter =
2 =U
= 14th letter from left = N U’s position in above arrangement = 6
3. (d) 5th letter to the left of M in the backward order and vowel that appear in end in new arrangement
=A
= 5th letter to the right of M in the forward order
A’s position in the above arrangement = 26
= (13 + 5)th letter in the forward order = 18 th letter
∴ Required difference = 26 − 6 = 20
Q Position of M in forward order = 13
∴18th letter in the forward order = R 9. (d) Given that,

A
AKGLMNDQKGCSNGKTGKGNDZP

M
4. (d) 7th letter to the right of K in backward order
UXGKE
= 7th letter to the left of K in forward order

H
Hence, there are two KGN in the above series, in
= (11 − 7 )th letter in the forward order

TT
which G is in the middle and ‘K’ and ‘N’ are
= 4 th letter in the forward order = D adjacent to it.
5. (c) 11th letter from right in reverse order
= 11th letter from left in forward order = K
10. (c) Given series
A
W
NMWVMNMVWNMNMMNWVMN
Now, 7th letter to the left of K in reverse order Hence, there are three M’s, which are followed by
SH

= 7th letter to the right of K in forward order N but not preceded by N.


= Position of K in forward order + 7
11. (c) Reverse Order: I Series : Z Y X W V U T S R Q
= (11 + 7 )th letter in forward order
A

PONMLKJIHGFEDCBA
= 18th letter in forward order = R
@

II Series: Z Y X W V T S R Q P
6. (b) 15th letter from the right falls in the 1st half
which remains unchanged. Hence, 15th letter has N M L K J H G F D C B
no relation with the change of 2nd half, i.e. reverse
order of the last 13 letters of English alphabet. 8th letter from right
∴Required 15th letter from right = (27 − 15)th
= 12th letter from left = L 12. (d) B C M X N C X N B X N C B N C B Y B C
XN BCNABO NMZCB
Alternate Method
Let us see So, 3 ‘BCN’ occur in such a way, that C is in the
15th 1st
From right middle and B and N are on any one side.

A B K L M Z Y P O N 13. (d) E T E T T M E E E T E T E T E T T E E T

TTEEETETETETTEETE
13 letters of 1st 13 letters of 2nd
half remain unchanged half get reversed There are ‘8’ T’s here which are immediately
7. (b) According to the question, preceded and followed by ‘E’
13 12 1110 9 8 7 6 5 4 3 2 1 26 25 24 23 22 21 20 19 18 17 16 15 14 14. (d) B B C D F B C F C B B C B N V W
M L K J I HG F E DC B A Z Y XWV U T S RQPON
7th to right Hence, ‘F’ is eighth from the left end.
12th from left
∴ Required letter = U
Chapter 3 ALPHABET TEST 33

15. (d) A B B C D E F E I B C A F E C B 26. (b) Given word F O R G E T


B A C A O B N U V W Alphabetical order E F G O R T

Hence, more than five vowels are there which are So, after arranging the letters alphabetically ‘T’ is
immediately preceded by a consonant. the only letter whose position remain unchanged.

16. (d) According to the question, 27. (e) Given word V E R T I C A L


Original word D O L P H I N Alphabetical Order A C E I L R T V
New word D H I L N O P There is no letter which will remain at the same
position.
5th from left
28. (d) According to the question,
17. (d) In given word, letter G is at 7th place and it
Given word C O M M U N I C A T I O N S
comes at 7th place in English alphabet too.
18. (c) According to the question, New word O C M M N U C I T A O I S N
P R E A M B L E 10th from right
Hence, required letter is N.
So, there are two such pairs i.e. AE and BE. 29. (b) According to the question,
19. (d) According to the question, Original Word – M A T H E M A T I C S

A
+3 +3 +3 +3 +3 +3 +3 +3 +3 +3 +3
After replacement – P D W K H P D W L F V

M
S T R E A M I N G
9th from left

H
So, there are three such pairs i.e. ST, GI and NT. ∴ Letter at 9th place from left = L

TT
20. (d) According to the question, 30. (b) According to the question,
S U B S T A N C E
i. Change, A
Original word, T I R A D E S
S J Q B C F R
W
ii. Change, B C F J QR S
SH

So, there are three such pairs i.e. ST, AC and SU.
So, J will be the fourth from the right end.
21. (b) According to the question, 31. (c) S P M J G
A

S T O N E D
Q, R N, O K, L H, I
@

So, there are three such pairs i.e. ST, NO and DE. 32. (b) R V Z D H L
22. (d) After interchanging the letters of the word,
S, T, U W, X, Y A, B, C E, F, G I, J, K
N G U A B I L I L
33. (d) 5 7 9 11 13
7th from right E G I K M
23. (d) According to the question, F H J L

R E F R E S H I N G
34. (b) E Z U P
DCBA YXWV TSRQ
Hence, there are three such pairs i.e. EF, HI and GI.
35. (c) E H K N Q T
24. (c) According to the question,
T E A C H FG IJ LM OP RS

Expert Level Exercise


Hence, there are two such pairs i.e. EH and CE.
1. (d) According to the question,
25. (a) Given word = EQUALITY
After interchanging the letters, new word is formed, J O U R N E Y
LITYEQUA
So, Q is the third from the right. So, there are three such pairs i.e. JN, UY and EJ.
How to Crack Test of Reasoning VERBAL
34

2. (e) According to the question, 14. (a) S Y D H K


C O N F U S E D
T, U, V, W, X Z, A, B, C E, F, G I, J
15. (c) K M P T Y E L
Hence, there are four such pairs i.e. CF, NO, OS
and DE. L NO QRS UVW ZABC FGHI
X D JK
3. (d) According to the question,
Original word H E R I T A G E 16. (e) Series of alphabets given in BOLD
New word T A G E H E R I
1st from right
D C A B A D A C
4th from left

Required number of letters = 3


i.e. F, G and H. 17. (a) Ninth to the right of the twenty second from the
right end = (22 − 9) i.e., 13th from the right end = D
4. (c) In the given series, 22nd letter from left = E (By formula 4)
and 19th letter from right = S
Now, total number of letters between 18. (b) The given series of alphabets in reverse order
E and S =13 UBDBUABUDECADABACDEBECAD
13 + 1 14 CBDCABEC
∴ Letter exactly between E and S = = =7

A
2 2 Now, 8th to the right of the 7th from the left end

M
Hence, 7th letter = L = (8 + 7 ) th i.e., 15th from the left end = B

H
5. (d) The new word formed will be NOITALUBA’, then ↓ ↓
19. (d) C E B A C D B C D A C E B E D C A B A D A C

TT
eighth letter from the end would be O.
EDUBAUBDBU
6. (d) 1 2 3 4 5 6 7 8 9 10 11 12 13 14 15 16
L A NO I T A T T R A N S P O R A
Thus, there are two such A’s.
W
13th letter from right = (16 +1 − 13) = 4 th letter 20. (e) C E B A C D B C D A C E B E D C A B A D A C
from left = O EDUBAUBDBU
SH

∴ Required letter = 3 rd to the right of O Thus, there is only one such D.


= 4 + 3 = 7 th from left = A 21. (b)
A
@

7. (d) Following the instructions as given in the W $ H % H 8 U # G


question, the new arrangement of letters would be
PACORTCEL ER DIOGRAPHS and therefore letters +3 –2 +2 –3 +3 –2
E and R exists exactly in the middle. 3 @ 1 Y &
8. (d) There are three vowels A, O, E to the left of +3 –2 +3 –2
middle letter E.
Hence, it is clear from above that %H8 is different
9. (a) Vowel I has a consonant D to the left but a from others.
vowel to the right of it. P A C O R T C E L E R D I O
22. (c) Eighth element from the right = 2
GRAPHS
Seventh element from the left = 
10. (b) Letters C and L are sandwiched between two
vowels A (C) O and E (L) E. ∴ Number of letters between  and 2 = Eight.

11. (b) B D G K P V 23. (e) Sixteenth element from the right end = 7
Now, ninth element of the right of 7= N .
C EF HIJ LMNO QRSTU
12. (c) 24. (e) After deleting all the letters from the given
E J N Q S T
arrangement, new arrangement is as follows.
FGHI KLM OP R 9 Ω1&8%7$54#623@Ú
13. (d) E I N R W A F Now, fifth element to the right of 4=@
and fourth element to the left of % =1
FGH JKLM OPQ STUV XYZ BCDE ∴ Required elements = (@, 1)
Chapter 04 WORD FORMATION 35

04

Word Formation
In word formation, a word is given in question Illustration 1. CHARACTER
and we have to choose that word from the (a) TRACER (b) CHARTER
given options, which can or cannot be formed (c) HEARTY (d) CRATE
from the letters of the given word.

A
Solution (c) By using the letters of the given word,
Sometimes, a set of English letters is given in HEARTY can’t be formed because in the given word,

M
a jumbled order and the candidates are letter Y is absent.

H
asked to arrange them in a meaningful order.

TT
In some situations, we are asked to choose Illustration 2. ADMINISTRATION
particular letters from a word and arrange (a) RATION (b) MINISTER
them to form a meaningful word. (c) MIND
A (d) STATION
W
Solution (b) By using the letters of the given word,
MINISTER can’t be formed because in the given
Types of Questions
SH

word letter E is absent.


Directions (Illustrations 3-4) In each of the
A

There are three types of questions based on


following question, a word has been given, followed
@

the word formation which are generally asked


in various competitive examinations. by four other words, one of which can be formed by
using the letters of the given word. Find that word.

Type 1 Illustration 3. DEMOCRACY


(a) SECRECY (b) MICRO
SSC (CGL) 2014

Word Formation Using Letters (c) MARCY (d) DEMON


Solution (c) MARCY—All the letters of this word are
from a Given Word present in the main word.
In this type of questions, a word has been Hence, ‘MARCY’ can be formed from the letters of
given, followed by four other words. The the given word ‘DEMOCRACY’.
candidate has to identify the word which can
or cannot be formed by using the letters of Illustration 4. COMPENSATION
main word. (a) TINY (b) COPY
(c) MENTION (d) MOTIVE
Directions (Illustrations 1-2) In the following Solution (c) MENTION—All the letters of this word are
questions, select the word from the given present in the main word. Hence, ‘MENTION’ can
alternatives which cannot be formed using the be formed from the letters of the given word
letters of the given word. SSC (CPO) 2015
‘COMPENSATION’.
36 How to Crack Test of Reasoning VERBAL

Illustration 7 How many meaningful English


Type 2 words can be formed by using letters of the
Forming Word with Selected word ‘ALEP’? SC (10+2) 2012
(a) One (b) Two
Letters of Given Word (c) Three (d) More than three
In this type of questions, a word has been given and Solution (c) Such meaningful words are ‘PEAL’,
the candidate is required to make new meaningful ‘LEAP’ and ‘PALE’.
words using letters at different positions like 1st,
2nd, 5th, 8th etc of the given word. Illustration 8. How many meaningful English
words can be made with the letters IFEL,
Illustration 5. If it is possible to make only one using each letter only once in each word?
meaningful word from the 3rd, the 6th, the 9th (a) None (b) One (c) Two (d) Three
and the 10th letters of the word ‘PARENTHESIS’,
Solution (c) Such meaningful words are ‘FILE’ and
using each letter only once, last letter of the word
‘LIFE’.
is your answer. If no such word can be formed,
your answer is ‘X’ and if more than one such
word can be formed, your answer is Y. Type 4
(a) R (b) T (c) X (d) Y
Word Formation by

A
Solution (a) P A R E N T H E S I S
Unscrambling Letters

M
3rd 6th 9th 10th

H
In this type of questions, a set of English
Since, 3rd, 6th, 9th and 10th letters are R, T, S and I

TT
letters is given in a jumbled order. The
respectively. Hence, only one meaningful word STIR
candidate is required to arrange these
can be formed. Last letter of this word is R, so correct
option is (a). A
letters to form a meaningful word.
W
Note Always try to place the letters according to the
Illustration 6. If it is possible to make a meaningful numbers provided in options rather than doing it on
SH

word with the 1st, 4th, 7th and 11th letters from the the basis of your vocabulary knowledge.
word ‘INTERPRETATION’, then which of the
following will be the third letter of that word? If Illustration 9. Select the combination of
A

more than one such word can be made, then give numbers that forms a meaningful word.
@

‘M’ as the answer and if no such word can be P N O A C L M I


formed, then give ‘X’ as the answer. 1 2 3 4 5 6 7 8
(a) T (b) R (c) M (d) X (a) 2, 7, 8, 6, 4, 3, 1, 5 (b) 4, 7, 5, 2, 6, 8, 1, 3
Solution (c) I N T E R P R E T A T I O N (c) 7, 1, 8, 5, 6, 2, 4, 3 (d) 5, 3, 7, 1, 6, 4, 8, 2
Solution (d) C O M P L A I N
1st 4th 7th 11th
5 3 7 1 6 4 8 2
Since, 1st, 4th, 7th and 11th letters are I, E, R and T,
respectively. Hence, three meaningful words RITE, TIRE Clearly, the given letters, when arranged in the
and TIER can be formed. Since, more than one words are order of ‘5, 3, 7, 1, 6, 4, 8, 2,’ form the word
formed from the given letters, so answer is M. ‘COMPLAIN’.

Illustration 10. Rearrange the letters A, R, T,


Type 3 Y and D to form a meaningful word and
Forming Meaningful Words select from the given alternatives, the word
which is almost opposite in meaning to the
Using Given Letters word so formed.
(a) Dirty (b) Quiet (c) Quick (d) Queek
In this type of questions, a word has been given
and the candidates are required to form as many Solution (c) The word that can be formed by
meaningful English words as possible from the rearranging the given letters is ‘Tardy’ which
means ‘Sluggish’ and opposite of this word is
given word, using each letter only once in each
‘Quick’.
word.
Chapter 04 WORD FORMATION 37

Solution (b) Letter D is suffixed to the given words


Type 5 LEN + D = LEND
SAN + D = SAND
Formation of Meaningful WOR + D = WORD
word by adding letters/words SEE + D = SEED
in the given word(s) Illustration 12. Which single letter can be
prefixed to the following words to form
In this type of questions, some words are
entirely new words? SSC (CGL) 2015
given and the candidate has to either suffix or
prefix same letter in each of the given words TILL, TABLE, PILE, TAB, PRING
(a) H (b) S (c) B (d) C
to make new meaningful words.
Solution (b) Letter S is prefixed to the given words
It also includes questions based on forming S + TILL = STILL
two meaningful words by inserting a word in S + TABLE = STABLE
between the given word.
S + PILE = SPILE
Illustration 11. Given below are some group of S + TAB = STAB
letters. Which of the options can be joined at S + PRING = SPRING
the end of these letters to make them

A
Illustration 13. Choose the middle words.
meaningful words? SSC (CGL) 2015

M
MIRR ( ) ANGE
LEN, SAN, WOR, SEE

H
(a) A (b) D (a) ORED (b) AGE (c) ICK (d) OR

TT
(c) B (d) K Solution (d) MIRR (OR) ANGE
The two words formed are MIRROR and ORANGE.

A
W
Let us Practice
SH
A

Base Level Exercise


@

Directions (Q. Nos. 1-13) In each of the 5. MERCHANDISE BSSC (CGL) 2015
following question, a word has been given, (a) MESH (b) DICE
followed by four other words, one of which cannot (c) CHARM (d) CHANCE
be formed by using the letters from the given
word. Find that word. 6. BENEVOLENT SSC (10+2) 2018
(a) BEEN (b) NEVIN
1. LEGALIZATION SSC (CPO) 2014 (c) LENT (d) BEN
(a) ALERT (b) ALEGATION
(c) GALLANT (d) NATAL 7. SEGREGATION SSC (CGL) 2016
(a) EAGER (b) SEA (c) GATE (d) NATION
2. COMMUNICATION SSC (Multitasking) 2014
(a) ACTION (b) UNION 8. PORTFOLIO SSC (Multitasking) 2014
(c) NATION (d) UNISON (a) RIFT (b) ROOF (c) FORT (d) PORTICO

3. CONTEMPTUOUS SSC (10+2) 2018 9. AUTOBIOGRAPHY Delhi Police (SI) 2016


(a) CON (b) TOM (a) TROOP (b) BRIGHT
(c) PRETTY (d) POST (c) GRAPHIC (d) TROPHY

4. INCARCERATION SSC (CGL) 2014 10. GEMDISTIONARY Delhi Police (ASI) 2016
(a) RELATION (b) TERRAIN (a) MEGASITY (b) DISTART
(c) INACTION (d) CREATION (c) STAR (d) GAME
38 How to Crack Test of Reasoning VERBAL

11. RELINQUISHED SSC (10+2) 2017 22. If it is possible to make a meaningful word
(a) LINE (b) QUITE (c) RELIED (d) RUN with 1st, 5th, 6th and 11th letters of the word
‘COURAGEOUSLY’, which of the following
12. NUMISMATIC SSC (10+2) 2017
(a) MUST (b) SINCE (c) MATIC (d) MAT
will be third letter of that word? If no such
word can be made, give ‘X’ as your answer
13. ADMINISTRATION DSSSB (PRT) 2017 and if more than one such word can be
(a) SITUATION (b) STRAIN made, give ‘M’ as the answer.
(c) TRADITION (d) RATION (a) C (b) A (c) G (d) X
(e) M
Directions (Q. Nos. 14-18) In each of the
following question, a word has been given, 23. How many meaningful English words can be
followed by four other words. One of which can be made with ‘EPRY’ using each letter only
formed by using the letters from the given word. once in each word? UPPSC (RO) 2014
Find that word. (a) Two (b) One
(c) Three (d) More than four
14. RECOMMENDATION SSC (FCI) 2012
(a) COMMUNICATE (b) REMINDER 24. How many meaningful English words can be
(c) MEDICO (d) MEDIATES made with the letters ‘ERDU’ using each
letter only once in each word?
15. PREPARATION SSC (10+2) 2013 (a) None (b) One (c) Two (d) Three

A
(a) PAMPER (b) REPEAT (e) More than three

M
(c) PARTITION (d) PARROT
25. How many meaningful English words can be

H
16. ULTRANATIONALISM SSC (CPO) 2014 made with the letters ‘NWTI’ using each

TT
(a) ULTRAMONTANE (b) ULTRAMODERN letter only once in each word?
(c) ULTRAIST (d) ULULATE Allahabad Bank (Clerk) 2012

17. PREMONITION SSC (CGL) 2017


(a) One
A
(b) Two (c) None (d) Three
W
(e) More than three
(a) ACTION (b) NATION
26. How many meaningful English words can be
SH

(c) MONITOR (d) REMOVE


made with the letters ‘LAME’ using each
18. COMPANIONSHIP SSC (10+2) 2013
letter only once in each word? SSC (10+2) 2012
A

(a) OPEN (b) OPIUM


(a) None (b) One (c) Two (d) Three
(c) OPINION (d) NATION
@

(e) More than three


19. How many meaningful English words can be Directions (Q. Nos. 27-30) Letters of the words
formed, starting with S, with the second, the given below have been jumbled up and you are
fourth, the fifth and the eighth letters of the required to construct the words. Each letter has
word PERISHED, using each letter only once been numbered and each word is followed by four
in each word? (to be counted from left) options. Choose the option which gives the correct
IBPS (Clerk) 2011
order of the letters as indicated by the numbers to
(a) None (b) One (c) Two (d) Three
form words. SSC (CGL) 2014
(e) More than three
27. 1 2 3 4 5 6
20. If by arranging the letters of the word G I C O D N
‘NABMODINT’, the name of a game is (a) 2, 1, 4, 3, 6, 5 (b) 4, 3, 2, 6, 5, 1
formed. What are the first and the last letters (c) 6, 5, 2, 3, 1, 4 (d) 3, 4, 5, 2, 6, 1
of the word so formed? SSC (CGL) 2016
(a) B, T (b) B, N (c) N, D (d) A, T 28. V A R S T E
1 2 3 4 5 6
21. How many meaningful English words can be (a) 2, 3, 1, 6, 4, 5 (b) 3, 2, 4, 5, 6, 1
formed with the third, the fifth, the seventh (c) 4, 5, 2, 3, 1, 6 (d) 6, 3, 4, 5, 2, 1
and the ninth letters of the word
‘DOWNGRADED’ once in each word?
29. E M I H T R
IBPS (Clerk) 2012
1 2 3 4 5 6
(a) None (b) One (c) Two (d) Three (a) 1, 2, 3, 4, 5, 6 (b) 4, 1, 6, 2, 3, 5
(e) More than three (c) 5, 1, 6, 4, 3, 2 (d) 6, 1, 2, 3, 5, 4
Chapter 04 WORD FORMATION 39

30. R T A N U E 32. Which single letter can be prefixed to the


1 2 3 4 5 6 following words in order to obtain entirely
(a) 1, 3, 2, 6, 4, 5 (b) 3, 2, 4, 6, 1, 5 new words?
(c) 4, 3, 2, 5, 1, 6 (d) 4, 6, 5, 2, 3, 1 (Same letter has to be prefixed in all five
31. Name a single letter that can be suffixed to words) SSC (CPO) 2015
the following words to form new words. EAT OUR IS AS AT
SSC (CPO) 2015 (a) S (b) H
HAT BAR BAT PIN BATH (c) C (d) B
(a) A (b) E (c) B (d) D

Expert Level Exercise


1. When we arrange the letters of the word such word can be formed, give your answer
‘VRAKIE’ in correct order, we get the name as ‘Y’. IBPS (Clerk) 2012
of a river. Fifth letter from left in that name, (a) C (b) T (c) N (d) X (e) Y
is CGPSC 2014 6. What is the last letter of the word which is
(a) R (b) E (c) I (d) K formed by rearranging the following letters?
(e) None of these

A
C P A O E C K (a bird) EPFO 2011

M
2. If a meaningful word can be formed by (a) E (b) K (c) P (d) C
rearranging the letters ‘USCALA’. The first (e) None of these

H
letter of the word so formed is, the answer. If 7. The letters of the word ‘NUMKIPP’ are

TT
no such word can be formed the answer is X. disorder. If they are arranged in proper
SSC (CGL) 2016 order, the name of a vegetable is formed.
(a) C (b) S (c) A (d) L A
What is the last letter of the word so formed?
W
3. If it is possible to make a meaningful word SSC (CGL) 2016
(a) K (b) M (c) N (d) P
SH

with the 2nd, the 6th, the 9th and the 12th
letters of the word ‘CONTRIBUTION’, which 8. If it is possible to make only one meaningful
of the following will be the last letter of that
A

word with the first, the third, the fourth and


word? If more than one such word can be the sixth letters of the word ‘LEARNING’,
@

formed, give ‘M’ as the answer and if no using each letter only once, which of the
such word can be formed, give ‘X’ as the following will be the third letter of that
answer. SSC (FCI) 2012 word? If more than one such word can be
(a) T (b) O (c) M (d) X formed, give ‘Y’ as the answer and if no such
word can be formed, give Z as the answer.
4. If it is possible to make a meaningful word (a) I (b) R (c) A (d) Y (e) Z
with the second, the fifth, the tenth
and the twelfth letters of the word 9. If it is possible to make only one meaningful
‘METROPOLITAN’, which of the following word with the first, the second, the fifth and
will be the third letter of that word? If no the ninth letters of the word MEDITATION,
such word can be made, give X as the which of the following will be the third letter
answer and if more than one such word can of that word? If no such word can be made,
be made, give M as the answer. give ‘X’ as the answer and if more than one
UPSC (CSAT) 2012 such word can be made, give Z as the answer.
(a) N (b) Q (c) M (d) X (a) A (b) N (c) M (d) X (e) Z
5. If it is possible to make only one meaningful Directions (Q. Nos. 10-12) In each of the
word with the first, second, third and fifth following questions, there are a set of words.
letters of the word ‘TECHNOLOGY’, which Choose the option which can be put before the set
of the following would be the third letter of of words to make them meaningful words.
that word? If no such word can be made,
give ‘X’ as your answer and if more than one 10. CLOSE, MISS, ABLE, COUNT
(a) EN (b) DIS (c) FORE (d) RE
40 How to Crack Test of Reasoning VERBAL

11. SULE, TAIN, ABLE, E 19. A L I R E M C


(a) SAG (b) MOV (c) MAN (d) CAP 1 2 3 4 5 6 7
(a) 6, 3, 4, 1, 7, 2, 5 (b) 6, 5, 1, 4, 7, 3, 2
12. MOVE, MARK, QUEST (c) 7, 4, 5, 1, 6, 3, 2 (d) 7, 5, 4, 1, 6, 2, 3
(a) RE (b) MIS (c) PRE (d) LESS
20. C N A S P H I
Directions (Q. Nos. 13-14) In each of the 1 2 3 4 5 6 7
question given below, there are set of words. (a) 5, 7, 4, 2, 6, 1, 3 (b) 2, 3, 7, 5, 4, 1, 6
Choose the option which can be put after the set (c) 4, 5, 7, 2, 3, 1, 6 (d) 7, 2, 3, 4, 5, 1, 6
of words to make them meaningful words. 21. G E D E M E N L C K N O W A T
13. ST, ENG, DAM, SEW 1 2 3 4 5 6 7 8 9 10 11 12 13 14 15
(a) EIL (b) ARM (c) AGE (d) RAP (a) 14, 9, 10, 11, 12, 13, 8, 4, 3, 1, 2, 5, 6, 7, 15
(b) 2, 4, 6, 8, 10, 12, 14, 1, 3, 5, 7, 9, 11, 13, 15
14. H, CH, F, SW (c) 8, 7, 5, 4, 3, 2, 1, 6, 9, 10, 11, 13, 12, 14, 15
(a) REE (b) ARM (c) EAP (d) EART (d) 6, 7, 8, 9, 10, 1, 2, 3, 4, 5, 15, 14, 13, 12, 11

Directions (Q. Nos. 15-18) In each of the 22. E C O T I H T Y H O N P A


following question two groups of letters are given 1 2 3 4 5 6 7 8 9 10 11 12 13
on both sides of a bracket. Each question has (a) 13, 7, 12, 6, 11, 5, 10, 4, 9, 3, 8, 2, 1
(b) 13, 12, 11, 10, 1, 2, 3, 4, 5, 6, 7, 8, 9
four options. Which of the option has correct set

A
(c) 10, 9, 8, 7, 6, 5, 4, 3, 2, 11, 12, 13, 1
of letters that will be brought inside the bracket

M
(d) 9, 8, 12, 3, 7, 6, 1, 2, 13, 4, 5, 10, 11
so that two meaningful words are formed. The

H
first word will be formed by including the Directions (Q. Nos. 23-26) In each of these

TT
bracket and the set of letters before the bracket. question, jumbled letters of a meaningful word are
Whereas the second word will be formed by given. You are required to rearrange these letters and
including the bracket and the letters after it.
A
select from the given alternatives, the word which is
almost similar in meaning to the rearranged word.
W
15. PLAT (?) ATION SSC (Steno) 2013
(a) TENT (b) FORM (c) TERR (d) EAU LIC (ADO) 2011
SH

23. DAXEPN
16. REA ( ) URCH (a) INCREASE (b) REDUCE
A

(a) CT (b) DER (c) SON (d) CH (c) STILL (d) DECREASE
(e) None of these
@

17. PROFESS ( ) PHAN


(a) ION (b) OR (c) ED (d) TION 24. HRADTE
(a) DECREASE (b) LOSS
18. PRACT ( ) BERG (c) REDUCTION (d) SCARCITY
(a) ICE (b) ICAL (c) DNA (d) ORN (e) None of these

Directions (Q. Nos. 19-22) Letters of the 25. DNHBEI


words given here have been jumbled up and you (a) FRONT (b) SIDE (c) BACK
are required to construct the words. Each letter (d) LAST (e) None of these
has been numbered and each word is followed by 26. HNAGSRI
four options. Choose the option which gives the (a) DECORATE (b) COMPOSE
correct order of the letters as indicated by the (c) IMPRESS (d) IMPOSE
numbers to form words. (e) None of these

Answers
Base Level Exercise
1. (a) 2. (d) 3. (c) 4. (a) 5. (d) 6. (b) 7. (d) 8. (d) 9. (c) 10. (b)
11. (b) 12. (b) 13. (a) 14. (c) 15. (d) 16. (c) 17. (c) 18. (c) 19. (b) 20. (b)
21. (b) 22. (b) 23. (b) 24. (b) 25. (a) 26. (d) 27. (d) 28. (c) 29. (b) 30. (c)
31. (b) 32. (b)
Chapter 04 WORD FORMATION 41

Expert Level Exercise


1. (a) 2. (a) 3. (b) 4. (c) 5. (c) 6. (b) 7. (c) 8. (d) 9. (e) 10. (b)
11. (d) 12. (a) 13. (c) 14. (b) 15. (b) 16. (d) 17. (b) 18. (a) 19. (a) 20. (c)
21. (a) 22. (d) 23. (a) 24. (d) 25. (c) 26. (a)

Answer with Explanations


Base Level Exercise
1. (a) By using the letters of given word, ‘ALERT’ 13. (a) By using the letters of given word, ‘SITUATION’
cannot be formed from ‘LEGALIZATION’ due to cannot be formed from ‘ADMINISTRATION’ due to
absence of letter ‘R’. absence of letter ‘U’.
2. (d) By using the letters of given word, ‘UNISON’ 14. (c) ‘MEDICO’ can be formed from
cannot be formed because letter ‘S’ is not present RECOMMENDATION.
in the given word. 15. (d) ‘PARROT’ can be formed from ‘PREPARATION’.

A
3. (c) By using the letters of given word, ‘PRETTY’ 16. (c) ‘ULTRAIST’ can be formed from

M
cannot be formed because letters ‘R’ and ‘Y’ are ‘ULTRANATIONALISM’.
not present in the given word.

H
17. (c) ‘MONITOR’ can be formed from
4. (a) By using the letters of given word, ‘RELATION’

TT
‘PREMONITION’.
cannot be formed from ‘INCARCERATION’ due to
18. (c) ‘OPINION’ can be formed from ‘COMPANIONSHIP’.
absence of letter ‘L’.
5. (d) By using the letters of given word ‘CHANCE’ A
19. (b) Second, fourth, fifth and eighth letters of the
W
word PERISHED → E, I, S, D
cannot be formed from ‘MERCHANDISE’ because
∴ Meaningful English word → SIDE
SH

an extra ‘C’ is not present in ‘MERCHANDISE’.


6. (b) By using the letters of given word, ‘NEVIN’ 20. (b) The name of a game is ‘BADMINTON’. The first
and last letters are ‘B’ and ‘N’ respectively.
A

cannot be formed because letter ‘I’ is not present


21. (b) Given, D O W N G R A D E D
@

in the given word.


7. (d) By using the letters of the given word, ‘NATION’ Only one meaningful English word ‘WAGE’ can be
cannot be formed because letter N’ is used only formed by using the letters W, G, A and E.
once. 22. (b) From letters C, A G and L, one meaningful word
8. (d) By using the letters of given word, ‘PORTICO’ can be formed, i.e. ‘CLAG’
cannot be formed from ‘PORTFOLIO’ due to the 23. (b) Only one meaningful word ‘PREY’ can be
absence of letter ‘C’. formed by using the letters of ‘EPRY’.
9. (c) By using the letters of given word, ‘GRAPHIC’ 24. (b) Only one meaningful word ‘RUDE’ can be
cannot be formed because letter ‘C’ is not present formed by using the letters of ‘ERDU’.
in the given word.
25. (a) Only one meaningful word ‘TWIN’ can be
10. (b) By using the letters of given word, ‘DISTART’ formed by using the letters of ‘NWTI’.
cannot be formed because letter ‘T’ is used only
26. (d) Three meaningful words ‘LAME’, ‘MALE’ and
once.
‘MEAL’ can be formed by using the letters of
11. (b) By using the letters of given word, ‘QUITE’ ‘LAME’.
cannot be formed from ‘RELINQUISHED’ due to
27. (d) CODING → (3, 4, 5, 2, 6, 1)
absence of letter ‘T’.
28. (c) STARVE → (4, 5, 2, 3, 1, 6)
12. (b) By using the letters of given word, ‘SINCE’
cannot be formed from ‘NUMISMATIC’ due to 29. (b) HERMIT → (4, 1, 6, 2, 3, 5)
absence of letter ‘E’. 30. (c) NATURE → (4, 3, 2, 5, 1, 6)
42 How to Crack Test of Reasoning VERBAL

31. (b) Letter E is suffixed to the following words (iii) NAME (iv) AMEN
HAT + E → HATE So, number of words formed by the given letters = 4.
BAR + E → BARE Since, the number of meaningful words is more
BAT + E → BATE than one, so our answer is ‘Z’.
PIN + E → PINE 10. (b)The meaningful words are
BATH + E → BATHE ‘DISCLOSE’, ‘DISMISS’, ‘DISABLE’ and
32. (b) Letter H is prefixed to the following words ‘DISCOUNT’.
H + EAT → HEAT 11. (d) The meaningful words are
H + OUR → HOUR ‘CAPSULE’, ‘CAPTAIN’, ‘CAPABLE’ and ‘CAPE’.
H + IS → HIS 12. (a) The meaningful words are
H + AS → HAS ‘REMOVE’, ‘REMARK’ and ‘REQUEST’.
H + AT → HAT
13. (c) The meaningful words are
‘STAGE’, ‘ENGAGE’, ‘DAMAGE’ and ‘SEWAGE’
Expert Level Exercise 14. (b) The meaningful words are
1. (a) After arranging the given letters in proper ‘HARM’, ‘CHARM’, ‘FARM’ and ‘SWARM’.
sequence, we get the river name KAVERI and its

A
15. (b) ‘FORM’ completes the first word as PLATFORM
fifth letter from left is ‘R’.

M
and begin the second as ‘FORMATION’.
2. (a) The correct word is CASUAL and first letter is ‘C’.
16. (d) REA (CH) URCH

H
3. (b) From letters O, I, T, N, only one meaningful
The meaningful words are ‘REACH’ and

TT
word ‘INTO’ can be formed and last letter of this
‘CHURCH’.
word is ‘O’.
4. (c) The second, fifth, tenth and twelfth letters of the A
17. (b) PROFESS (OR) PHAN
W
word METROPOLITAN are E, O, T and N, The meaningful words are ‘PROFESSOR’ and
respectively. The words formed are ‘NOTE’ and ‘ORPHAN’.
SH

‘TONE’. 18. (a) PRACT (ICE) BERG


5. (c) From letters T, E, C, N, only one meaningful The meaningful words are ‘PRACTICE’ and
A

word ‘CENT’ can be formed and third letter of this ‘ICEBERG’.


@

word is ‘N’.
19. (a) MIRACLE → (6, 3, 4, 1, 7, 2, 5)
6. (b) The name of the bird is ‘PEACOCK’ and the 20. (c) SPINACH → (4, 5, 7, 2, 3, 1, 6)
last letter is ‘K’.
21. (a) According to the question,
7. (c) The name of that vegetable would be 14 9 10 11 12 13 8 4 3 1 2 5 6 7 15
‘PUMPKIN’ and last letter of the word is ‘N’. ↓ ↓ ↓ ↓ ↓ ↓ ↓ ↓ ↓ ↓ ↓ ↓ ↓ ↓ ↓
A C K N O W L E DG E ME N T
8. (d) The first, third, fourth and sixth letters of the
word are L, A, R and I respectively. 22. (d) According to the question,
L E A R N I N G 9 8 12 3 7 6 1 2 13 4 5 10 11
1 2 3 4 5 6 7 8 ↓ ↓ ↓ ↓ ↓ ↓ ↓ ↓ ↓ ↓ ↓ ↓ ↓
Words formed using these letters are as follows. H Y P O T H E C A T I O N
(i) LAIR (ii) LIAR (iii) RAIL (iv) LIRA 23. (a) The word is ‘EXPAND’ and the meaning is
‘INCREASE’.
9. (e) The first, second, fifth and ninth letters of the
words 24. (d) The word is ‘DEARTH’ and the meaning is
‘SCARCITY’.
M E D I A T I O N are M, E, A and N respectively
1 2 3 45 6 7 8 9 25. (c) The word is ‘BEHIND’ and the meaning is
‘BACK’.
Words formed using these letters are as follows.
26. (a) The word is ‘GARNISH’ and the meaning is
(i) MANE (ii) MEAN ‘DECORATE’.
Chapter 05 CODING-DECODING 43

05
Coding-Decoding
Coding means to hide the meaning of any Methods to Remember the
message and decoding means to understand the
actual meaning of that message.
Position of Letters in English
In questions based on coding-decoding, a word or Alphabet

A
sentence is coded in a particular way and the Some methods/techniques are given below
candidates are asked to code other word or to remember the positions of English

M
sentence in the same way or to decode the word alphabets in forward or backward order.

H
or sentence. 1. By using EJOTY and CFILORUX

TT
To solve these questions, the candidates are formulae, we can easily remember the
required to understand the rule that has been position of letters of English alphabet
followed to code a particular word/message. A
in forward order.
W
C F I L O R U X
Position of Letters in English Alphabet
SH

Sometimes a word or a sentence is coded by 3 6 9 12 15 18 21 24


changing the position of English alphabet letters
A

according to a definite pattern, so it is necessary (sounds like a medicine name)


@

to remember the positions of all the letters in E J O T Y


English alphabetical order, both in forward and
backward order. 5 10 15 20 25
Order of the English Alphabet (sounds like a girl’s name Joti)
Forward Letters Backward Forward Letters Backward Note Backward order position of a letter
order order order order = 27 − Forward order position of letter
position position position position
e.g. Backward order position of B
1 A 26 14 N 13
= 27 − Forward order position of B
2 B 25 15 O 12
= 27 − 2 = 25
3 C 24 16 P 11
4 D 23 17 Q 10 2. By using V Q L G B and X U R O L I
5 E 22 18 R 9 F C, we can easily remember the
6 F 21 19 S 8 position of letters of English alphabet
7 G 20 20 T 7 in reverse order.
8 H 19 21 U 6 X U R O L I F C
9 I 18 22 V 5
10 J 17 23 W 4 3 6 9 12 15 18 21 24
11 K 16 24 X 3 and
12 L 15 25 Y 2 V Q L G B
13 M 14 26 Z 1
5 10 15 20 25
44 How to Crack Test of Reasoning VERBAL

Similarly,
Types of Coding-Decoding M A T H E M A T I C A L
Following are the categories/types of
coding-decoding, which are generally asked in
various competitive examinations.
H T A M T A M E L A C I
Type 1
Illustration 3. If GOODNESS is coded as
Letter/Number Coding HNPCODTR, then how GREATNESS can be
written in that code? SSC (CPO) 2015
In this type of coding, we deal with questions,
(a) HQZSMFRT
in which the letters of a word are replaced by
(b) HQFZUFRTM
certain other letters or numbers according to a
(c) HQFZUODTR
specific pattern/rule to form a code. You are (d) HQFZUMFRT
required to detect the coding pattern/rule and
Solution (d) Clearly, the letters in the word
answer the question(s) that are asked, based
GOODNESS are moved one step forward and one
on that coding pattern/rule.
step backward, alternately to obtain the letters of

A
Illustration 1. In a certain code language, ‘GIVE’ the code.

M
is written as ‘VIEG’ and ‘OVER’ is written as As, G O O D N E S S
‘EVRO’. How will ‘DISK’ be written in that

H
+1 –1 +1 –1 +1 –1 +1 –1
same code?

TT
H N P C O D T R
(a) SIDK (b) KISD
(c) KDSI (d) SIKD Similarly, G S
(e) None of these A
R E A T N
+1
E
–1
S
+1
W
+1 –1 +1 –1 +1 –1
Solution (d) Here,
SH

H Q F Z U M F R T
G V and O E
I I V V Illustration 4. If GRASP is coded as TIZHK, what
A

V E E R will be coded as OVTZXB? MPPSC 2018


@

E G R O
(a) LEGATE (b) LEAGUE
Similarly, (c) LEGACY (d) LEDGER
D S Solution (c) Here, each letter is coded by its opposite
I I letter.
S K
As,
K D
G R A S P T I Z H K
Illustration 2. If in a certain code 7+20=27
‘INTELLIGENCE’ is written as 18+9=27
‘ETNIGILLECNE’, then how can 1+26=27
‘MATHEMATICAL’ be written in the same 19+8=27
code? SSC (CGL) 2014 16+11=27

(a) AMHTMETACILA (b) TAMMEHITALAC Similarly,


(c) HTAMTAMELACI (d) LACITAMEHTAM L E G A C Y O V T Z X B
12+15=27
Solution (c) As, 5+22=27
I N T E L L I G E N C E 7+20=27
1+26=27
3+24=27
25+2=27
E T N I G I L L E C N E
Chapter 05 CODING-DECODING 45

Illustration 5. If ‘WORK’ is coded as ‘4-12-9-16’, Illustration 8. If DEVICE is coded as BAREXA


then how will you code ‘WOMAN’? and ORNATE is coded as ZVMESA, what
(a) 4-12-14-26-13 (b) 4-26-14-13-12 VIDEO be coded as? WBPSC 2018
(c) 23-12-26-14-13 (d) 23-15-13-1-14 (a) RIBAZ (b) REBAZ
(e) None of these (c) RBEAZ (d) ZABER

Solution (a) We have, Then, Solution (b) Using direct letter coding method,
W 4 W 4 As, D→B and O→Z Similarly, V → R
O 12 O 12
E→A R→V I→ E
R 9 M 14
K 16 A 26 V→R N→M D→ B
N 13 I→E A→E E→ A
Here, each letter is coded by the numerical value C→X T→S O→ Z
obtained by subtracting its ‘position value’ in
E→A E→A
English alphabetical order, from 27, e.g. letters W,
O, M, A, N are at 23rd, 15th, 13th, 1st and 14th
Illustration 9. In a certain code language, FILE is
position in alphabetical order. So, their codes are
written as 7465 and IDEAL is written as 43586.
(27–23), (27–15), (27–13), (27–1), (27–14), i.e. 4,
How will DEAF be written in that code

A
12, 14, 26, 13, respectively i.e. their backward
language? UP Police (Constable) 2018

M
order position.
(a) 3478 (b) 3588 (c) 3587 (d) 4578

H
Illustration 6. If CUP = 40, then KITE = ?
Solution (c) Using direct number coding method,

TT
(a) 10 (b) 20
As, F I L E and I D E A L
(c) 30 (d) 45
Solution (d) Using forward letter positions A
W
3 21 16 7 4 6 5 4 3 5 8 6
As, C U P ⇒ 3 + 21 + 16 = 40
SH

Similarly, D E A F
11 9 20 5
Similarly, K I T E ⇒ 11 + 9 + 20 + 5 = 45
A

3 5 8 7
Illustration 7. If BAG = 71 , then VICE = ?
@

(a) 69 (b) 70 Illustration 10. In a certain code ‘GONE’ is


(c) 75 (d) 90 written as 5 % 2 # and ‘MEDAL’ is written as
Solution (a) Using backward letter positions 4 # 3 $ @, then how will ‘GOLD’ be written in
25 26 20 that code?
As, B A G ⇒ 25 + 26 + 20 = 71 (a) 5@%3
(b) 5% @ 3
Similarly, V I C E
(c) 5#@3
(d) 5%#3
5 18 24 22
Solution (b) Using direct coding method,
= 5 + 18 + 24 + 22 = 69
As, G O N E and M E D A L

Type 2 5 % 2 # 4 # 3 $ @
Direct Coding
Similarly, G O L D
In direct coding, the code letters/numbers/
symbols occur in the same sequence as the
5 %@ 3
corresponding letters occur in the words. This
is basically a direct substitution method. ∴ GOLD ⇒ 5 % @ 3
46 How to Crack Test of Reasoning VERBAL

Type 3 12. (b) R B U K A E

Conditional Coding 6 7 0 8 9 2
In this type, letters/numbers are given along This question does not follow any condition.
with their codes. Candidates are required to
find the codes for a particular letter 13. (b) From condition (i),
group/word/number according to certain E B N A P I
conditions given in the questions.
$ 7 3 9 5 $
Directions (Illustrations 11-13) Study the
following letters and their corresponding digits
codes followed by certain conditions of coding and
then answer the questions given below them by
Type 4
finding out which of the digits combinations given Fictitious Language Coding
in (a), (b), (c) and (d) is the coded form of the
letters-groups given in each question and mark
Here, coded form of two or more sentences is
your answer accordingly. given and you are required to find the code of
a particular word or message.

A
Letters P N A J I R E B U K
To analyse such codes, any two messages
5 3 9 1 4 6 2 7 0 8

M
Digits/Codes
bearing a common word/number are picked
up. The common code will represent that

H
Conditions
particular word/number.

TT
(i) If both the first and the last letters in the
group are vowels, both should be coded Proceeding similarly by picking up all possible
as $. A
combinations of two, the entire message can
be decoded and the codes for every individual
W
(ii) If both the first and the last letters in the
group are consonants, both should be word/number can be found.
SH

coded as #. In some cases, there is no common


word/number in the message given. In those
Illustration 11. KUNAJB
A

cases, the words/numbers are coded


@

(a) 803917 (b) $0391$ separately. You have to identify the correct
(c) #0391# (d) #0391$ pattern of coding and find the code for the
(e) None of these word or message given in the question.
Illustration 12. RBUKAE Illustration 14. In a certain code language, ‘it pit
(a) #70892 (b) 670892 sit’ means ‘I am boy’, ‘it nit sit’ means ‘I am
(c) 670982 (d) 607892 girl’, which of the following means ‘girl’?
(e) None of these
(a) it (b) pit (c) sit (d) nit
Illustration 13. EBNAPI (e) None of these
(a) 273954 (b) $7395$ Solution (d) According to the question, we have
(c) #7395# (d) $7395# it pit sit I am boy
(e) None of these
it nit sit I am girl
Solutions (Illustrations 11-13) We know that, in
English alphabets A, E, I, O, U letters are vowels Here, ‘it’ and ‘sit’ are common in both the codes
and remaining letters are consonants. and ‘I’ and ‘am’ are common in both messages.
Hence, code for girl will be ‘nit’.
11. (c) From condition (ii),
Illustration 15. In a certain code language, 481
K U N A J B
means ‘sky is blue’, 246 means ‘sea is deep’
and 698 means ‘sea looks blue.’ What number
# 0 3 9 1 # is the code for ‘blue’? SSC (CGL) 2015
(a) 1 (b) 6 (c) 8 (d) 9
Chapter 05 CODING-DECODING 47

Solution (c) According to the question, we have 16. (b) Code for translate →@E9
17. (d) %→ g
8 1 sky is blue
4 ...(i) So, from the options %D4 →Good
2 4 6 sea is deep ...(ii)
...(iii)
Type 5
6 9 8 sea looks blue
Substitution Coding
From Eqs. (i) and (iii), we see that the code for
blue is ‘8’.
In this type of coding, some particular words
are coded with certain substituted word and on
Directions (Illustrations 16-17) Study the the basis of substituted word, the code of given
following information carefully and answer the word is derived.
questions given below.
Illustration 18. If ‘white’ is called ‘blue’, ‘blue’ is
In a certain code language called ‘red’, ‘red’ is called ‘yellow’, ‘yellow’ is
‘given time simple plan’ is written as called ‘green’, ‘green’ is called ‘black’, ‘black’ is
‘@E4 & N4 % N5 # E6’ called ‘violet’ and ‘violet’ is called ‘orange’, then
‘tired solution plant great’ is written as what would be the colour of human blood?
‘#N8 @ D5 % T5 & T5’ (a) Red (b) Green

A
‘sick point good turn’ is written as (c) Yellow (d) Violet
‘#K4 % D4 @ N4 & T5’ (e) Orange

M
‘garden sister phone team’ is written as Solution (c) We know that, the colour of human blood

H
‘&E5 # R6 % N6 @ M4’ is ‘red’ but here ‘red’ is called ‘yellow’. So, the

TT
Illustration 16. Which of the following is the colour of human blood will be ‘yellow’.
code for ‘translate’? Illustration 19. If ‘parrot’ is known as ‘peacock’,
(a) @E8 (b) @E9 A
‘peacock’ is known as ‘swallow’, ‘swallow’ is
W
(c) #E8 (d) #T8 known as ‘pigeon’ and ‘pigeon’ is known as
SH

(e) #T9 ‘sparrow’, then what would be the name of


Illustration 17. In the given code language, Indian National Bird? SSC (10+2) 2016
A

what does the code ‘%D4’ represent? (a) Parrot (b) Swallow
@

(a) Point (b) Turn (c) Peacock (d) Pigeon


(c) Sick (d) Good Solution (b) We know that, peacock is the Indian
(e) None of these National Bird but here peacock is known as swallow.
Solution (Illustration 16-17) Here, symbols are So, the answer is swallow.
coded as per the first letter of the word. Illustration 20. On another planet the local
terminology for ‘earth’, water,’ ‘light’ air and sky
As for ‘g’ symbol is %.
are sky, light, ‘air’, water and ‘earth’ respectively. If
Similarly, ‘p’→ &, ‘t’→ @ and ‘s’→ # someone is thirsty there what would he drink?
The number in the code represents the number of (a) Light (b) Air
letters in the word and the letter in the code (c) Sky (d) Water
represents the last letter of the word. Solution (a) One drinks water when he is thirsty.
e.g. In the word ‘‘given’’ g = %, last letter = N and Since, ‘water’ drink ‘light when he is thirsty there. So,
number of letters = 5 the answer is ‘light’.
∴Given = % N5
48 How to Crack Test of Reasoning VERBAL

Let us Practice
Base Level Exercise
1. In a certain code, ‘SOBER’ is written as 8. If ‘MUSICAL’ is written as ‘KWQKACJ’, then
‘RNADQ’. How ‘LOTUS’ can be written in how can ‘SPRINKLE’ be written?
SSC (CGL) 2013
that code? SSC (Multitasking) 2013
(a) QRBKCNJG (b) QNPGLIJC
(a) KNSTR (b) MPUWT (c) QRPKLMJG (d) URTKPMNG
(c) KMSTR (d) LMRST
9. In a code language, if ‘BUTTER’ is coded as
2. If ‘WATER’ is written as ‘YCVGT’, then what is ‘EXWWHU’ and ‘MILK’ is coded as ‘PLON’,
written as ‘HKTG’? SSC (CGL) 2013 then in the same code language, how will
(a) IRFE (b) FIRE ‘EARTH’ be spelled? UP Police (Constable) 2018
(c) REFI (d) ERIF (a) HDWUK (b) HDUWK
3. In a certain code, ‘TERMINAL’ is written as (c) KHDUW (d) KHUWD
‘NSFUMBOJ’ and ‘TOWERS’ is written as 10. If ‘DEAR’ is coded as ‘FGCT’, then how will

A
‘XPUTSF’. How is ‘MATE’ written in the ‘READ’ be coded as? OPSC 2018

M
same code? IBPS (Clerk) 2012 (a) TGCF (b) FGCF (c) TSFC (d) TCGF
(a) FUBN (b) UFNB

H
(c) BNFU (d) BNDS 11. If ‘BARLEY’ is written as ‘CBTMGZ’

TT
(e) None of these SOUND will be written as DSSSB 2017
(a) URWOF (b) TPWOF
4. In a certain code, ‘CERTAIN’ is coded as
‘XVIGZRM’ ‘SEQUENCE’ is coded as
(c) UPWPF
A (d) None of these
W
‘HVJFVMXV’. How would ‘REQUIRED’ be 12. If ‘PORTER’ is written as "QNSSFQ’, then
"BRIGHT’ would be coded as WBPSC 2018
SH

coded? SSC (CGL) 2012


(a) FJIVWVIR (b) VJIFWTRV (a) CNJHIU (b) CQJFGS (c) CNJHIS (d) CQJFIS
(c) WVJRIFVI (d) IVJFRIVW 13. In a certain code language ‘MOCKS’ is
A

5. In a certain code, ‘BUILDER’ is written as written as ‘NNDJT’. How is ‘FLAME’ written


@

‘JVCKSFE’. How is ‘SEALING’ written in the in that code language? SSC (10+2) 2017
same code? RRB (TC/CC) 2009 (a) KFBMA (b) GKBLF
(a) BFTKHOJ (b) JOHKBFT (c) LBHGK (d) LBGHG
(c) TFBKHOJ (d) BFTKJOH
14. If ‘THEN’ is written as ‘RLBS’, then how
6. In a certain code language, ‘CURATIVE’ is may ‘CASE’ be written in that code?
written as ‘BSVDDUHS’. How ‘STEAMING’ SSC (CGL) 2016
(a) AEPJ (b) APEP (c) EPAP (d) PAEJ
is to be written in the same code language?
(a) BFUTFMHL (b) TUFBFMHL 15. If ‘ CARING‘ is coded as ‘EDVGKC‘ , and
(c) BFUTLHMF (d) BFUTHOJN ‘SHARES’ is coded as ‘UKEPBO’, then how
(e) None of these will ‘CASKET’ be coded as in the same code?
SSC (CPO) 2013
7. In a certain code, ‘MOUSE’ is written as (a) EDXIBP (b) EDWIAP
‘PRUQC’. How is ‘SHIFT’ written in the (c) EDWPAI (d) EDWIBP
same code? Vijaya Bank (Clerk) 2010
(a) VKIRD 16. In a certain code language, ‘RULE’ is coded
(b) VKIDR as ‘GNWT’ and ‘BAND’ is coded as ‘FPCD’.
In the same code language, ‘SOFA’ will be
(c) VJIDR
coded as SBI (Clerk) 2016
(d) VIKRD (a) BIPT (b) CHQU (c) BILV (d) DHMV
(e) None of the above (e) CHDU
Chapter 05 CODING-DECODING 49

17. In a certain code language ‘ROUTE’ is 27. If each of the letters in the English alphabet
written as ‘URTWH’ and ‘LOWER’ is written is assigned an even numerical value by
as ‘ORVHU’. How will ‘PRINT’ be written in giving A = 2, B = 4 and so on, then what
that code language? would be the total value of the letters for the
Cooperative Bank (Clerk) 2016 word ‘LADY’ when similarly coded?
(a) TVMRX (b) SVGPV SSC (CGL) 2013
(c) TUJRX (d) STIQV (a) 74 (b) 72 (c) 84 (d) 82
(e) SUHQW
28. In a certain code language, ‘FRONT’ is
18. In a certain code language, ‘JUGS’ is coded written as ‘618151420’ and ‘BORNE’ is
as ‘KTFT’ and ‘WHEN’ is coded as ‘XGDO’. written as ‘21518145’. How is ‘MORNS’
In the same code language, ‘ONCE’ will be written in that code language? SSC (10+2) 2017
coded as SBI Clerk (Pre) 2016 (a) 1315181419 (b) 1314161718
(a) PMDF (b) PMBD (c) 81291114 (d) 141154321
(c) NODD (d) PMBF
(e) NODF 29. If ‘REASON’ is coded as 5 and ‘BELIEVED’
as 7, then what is the code number for
19. In a certain code, DURABLE is written as ‘GOVERNMENT’? SSC (Multitasking) 2012
QTCBDKA. How is COUNTRY written in (a) 10 (b) 6 (c) 9 (d) 8
that code? NIACL (AO Mains) 2016

A
(a) VPDOZSU (b) TNBOXQS 30. If ‘LAMP’ is coded as 30-52-28-22, then ‘TOY’

M
(c) VPDMZSU (d) TNBOZSU will be coded as MPPSC 2018
(a) 14-24-4 (b) 20-15-25

H
(e) None of these
(c) 14-4-24 (d) 20-25-15

TT
20. In a code language, ’FRIEND‘ is coded as
‘HUMJTK’. In the same language, ’BHOPAL‘ 31. If ‘MACHINE’ is coded as 19-7-9- 14-15-20-11,
then how will you code ‘DANGER’ in the
will be coded as
(a) DJQRCN
MPPSC 2018
(b) CJRTEQ same code? A
W
(c) DKSUGS (d) DLSVHT (a) 11-7-20-16-11-24 (b) 13-7-20-9-11-25
SH

(c) 10-7-20-13-11-24 (d) 13-7-10-11-25


21. In a code language, ‘TINY’ is coded as
‘RGLW’. In the same language, ‘ZEBRA’ will 32. If ‘JUICE’ is coded as ‘19-41-17-5-9’, then
A

be coded as MPPSC 2018 ‘TOY’ will be coded as MPPSC 2014


@

(a) XCZYP (b) XCZPY (c) XZCPY (d) XZCYP (a) 39-29-49 (b) 41-31-51 (c) 13-23-3 (d) 15-25-5
22. If DEMOCRATIC is written as 33. If in a code language, ‘PARENT’ is written as
EDMORCATCI, then how CONTINUOUS ‘BDFGJK’ and ‘CHILDREN’ is written as
will be written in the same code? CGPSC 2013 ‘MOXQUFGJ’, then how is ‘REPRINT’
(a) OCTNNIOUSU (b) OTCNINUOUS written in the same code?
(c) OCNTNIUOSU (d) OTNCINUOSU (a) FGBFXJK (b) FGBUXJK
(e) CONNITUOSU (c) FGBFXGD (d) BGFXJK
23. 165135 is to ‘PEACE as 1215225 is to 34. In a language, FIFTY is written as CACTY,
IB (ACIO) 2013 CAR as POL, TAR as TOL. How can TARIFF
(a) LEAD (b) LOVE (c) LOOP (d) AURA
be written in that language? SSC (CPO) 2015
24. If A = 1 , ACE = 9, then ART = ? SSC (10+2) 2013 (a) TOEFDD (b) TOEFEL
(a) 10 (b) 39 (c) 29 (d) 38 (c) TOLADD (d) TOLACC
25. If CAT = 12, then MAN =? 35. In a code language, ‘ORGANISATION’ is
(a) 14 (b) 24 written as ‘CBDWLQJWYQCL’ and
(c) 16 (d) 15 ‘OPERATION’ is written as ‘CXFBWYQCL’.
(e) None of these How would ‘SEPARATION’ be coded?
26. If ASHA equals 79, then VINAY (a) EJXEBYQCL
BHUSHAN = ? BSSC (CGL) 2015 (b) JFQYWBCXQL
(a) 211 (b) 200 (c) JFXWBWYQCL
(c) 144 (d) 180 (d) QCLYWBFXJE
50 How to Crack Test of Reasoning VERBAL

Directions (Q. Nos. 36-39) In a defence 46. In a certain code, ‘BASKET’ is written as
message, GETAWAY, FIRE, BACKWARDS, ‘5$3%#1’ and ‘TRIED’ is written as
MOVE, SLOW is coded as BENCDCI, QHOE, ‘14★#2’. How is ‘SKIRT’ written in that
PCTLDCOXU, ZMWE, UFMD. Based on this code? IBPS (PO) 2011
coding scheme, spot the codes of the following (a) 3%★41 (b) 3★%41 (c) 3%#41 (d) 3#4%1
words. (e) None of these
36. OVER 47. In a certain code language, ‘SAFER’ is
(a) MWED (b) MWEO written as ‘5@3#2’ and ‘RIDE’ is written
(c) MWOE (d) MWZO as ‘2 %#’, then how would ‘FEDS’ be
37. DEADLY written in that code? RBI (Grade B) 2009
(a) 3#5 (b) 3@%5 (c) 3#%5 (d) 3#%2
(a) XECXEI (b) XEEXC
(e) None of these
(c) XECXFI (d) XENXFI
48. In a certain code, ‘LONG’ is written as
38. REWARD ‘51 #4’ and ‘GEAR’ is written as ‘4 % $ 9’.
(a) OEDCOU (b) OEDCOX
How is ‘ROLE’ written in that code?
(c) OEDNXE (d) OTDCOX NIACL (AO Mains) 2016
39. GREAT (a) 915% (b) 951% (c) 915# (d) 415%

A
(a) BOECN (b) BOENC (e) None of these

M
(c) BOEHC (d) BOEQN 49. In a certain code language, ‘AUSTRALIA’ is

H
40. If PALE is coded as 2134, EARTH is coded as written as @$!#^@★?@.

TT
41590, then how is PEARL coded as? How is ‘STRAIT’ written in that code
SSC (10 + 2) 2014 language? SSC (10+2) 2017
(a) 29530 (b) 24153
(c) 25413 (d) 25430
(a) !#^@?#
(c) !#^?@# A (b) !#@^?#
(d) !#^?@#?
W
41. If the code for STABLE is 123456 and code
Directions (Q. Nos. 50-53) Study the following
SH

for LABOUR is 534789, then what will be the


information carefully and answer the questions
code for BOTTLE? UPPSC (RO) 2014
given below it. IBPS (Clerk) 2012
A

(a) 472256 (b) 472556


Digits in the numbers are to be coded as follows
@

(c) 472265 (d) 475526


Digits 9 2 1 7 5 3 6 4 8
42. If 73429186 denotes PURCHASE and 54064
denotes ORDER, then what will 7452688 Codes B V M L D P A F R
denote? WBPSC 2018
(a) PROGRES (b) PROCEED Conditions
(c) PROGRAM (d) PROCESS (i) If the first as well as the last digits are
43. If ‘LINGER’ is ‘123456’ and ‘FORCE’ is even, both are to the coded by the code for
the first digit.
‘56789’, then ‘FIERCE’ will be
(a) 345667 (b) 456678 (ii) If the first as well as the last digits are
(c) 345677 (d) 556789
odd, both are to be coded by the code for
the last digit.
(e) Cannot be determined
50. 562183
44. If in a certain code, the word MILITARY is (a) PAVMRP (b) DAVMRD
written as 12324567, then in the same code, (c) PAVMRD (d) DAVMRP
the word TAIL will be written as CLAT 2017 (e) None of these
(a) 3254 (b) 4523 (c) 2345 (d) 5432
51. 627851
45. In a certain code language, ‘DANGER’ is (a) PULRDM
written as ‘145237’ and ‘RANCOR’ is written (b) AVLDRM
as ‘745967’. How is ‘RAGE’ written in that (c) AVLFDM
code language? SSC (10+2) 2017 (d) AVLRDM
(a) 7231 (b) 7234 (c) 7423 (d) 7441 (e) None of the above
Chapter 05 CODING-DECODING 51

52. 812354 59. In a certain code ‘ter ner ger’ denotes ‘you
(a) RLVPDF (b) FMVPDF can determine’, ‘fer ler ter kar’ denotes ‘can
(c) RMVPDR (d) MFPVFD she help me’ and ‘mer der ger’ denotes ‘how
(e) None of these are you’. Which word denotes ‘determine’ in
53. 397416 that code? UPPSC (RO) 2014
(a) ger (b) ter (c) fer (d) ner
(a) PBLFMP (b) ABLFMA
60. If ‘Lily’ is called ‘Lotus’, ‘Lotus’ is called
(c) PVLFMA (d) PBLFMA
‘Rose’, ‘Rose’ is called ‘Sunflower’ and
(e) None of these
‘Sunflower’ is called ‘Marigold’, then which
54. In a certain code, ‘975’ means ‘throw away will be the national flower of India?
garbage’, ‘528’ means ‘give away smoking’ (a) Lily (b) Lotus
and ‘213’ means ‘smoking is harmful’. Which (c) Rose (d) Marigold
digit in that code means ‘smoking’? (e) Sunflower
(a) 5 (b) 8 61. If ‘ROAD’ is called ‘CAR’, ‘CAR’ is called
(c) 2 (d) 3 ‘TRAIN’, TRAIN’ is called ‘SCHOOL’,
55. In a certain code language, ‘po ki top ma’ SCHOOL’ is called ‘HOUSE’, ‘HOUSE’ is
means ‘Usha is playing cards’, ‘kop ja ki ma’ called ‘OFFICE’, then where do children go
means ‘Asha is playing tennis’, ‘ki top sop to study? [UCO Bank (Clerk) 2011]

A
ho’ means ‘they are playing football’ and ‘po (a) HOUSE (b) TRAIN (c) SCHOOL

M
sur kop’ means’ cards and tennis. Which (d) OFFICE (e) None of these

H
word in that language means ‘Asha’? 62. If the animals which can walk are called

TT
UPPSC 2013
(a) ja (b) ma ‘swimmers’, animals who crawl are called
(c) kop (d) top ‘flying’, those living in water are called
A
‘snakes’ and those which fly in the sky are
W
56. In a certain code if ‘on the day’ is coded as called ‘hunters’, then what will a lizard be
‘la si ko’ and ‘the setting day’ is coded as ‘si called?
SH

mu la’, what is the code for ‘setting’ in the (a) Swimmers (b) Snakes
given code language? (All the codes are two (c) Flying (d) Hunters
A

letter codes only) SBI Clerk (Pre) 2013 (e) None of these
@

(a) mu (b) ko
(c) si (d) Either ‘la’ or ‘ko’ 63. If ‘bucket’ is known as ‘tub’, ‘tub’ is known
(e) la as ‘glass’, ‘glass’ is known as ‘saucer’,
‘saucer’ is known as ‘spoon’, then which
57. In a certain code language, ‘job requires utensil will be used for drinking water?
expertise’ is written as ‘la nu si’. ‘expertise in (a) Tub (b) Saucer (c) Glass (d) Spoon
area’ is written as ‘li bo la’ and ‘requires area (e) None of these
inspection’ is written as ‘si dm bo’. How is
‘inspection’ written in that code language? 64. If ‘orange’ is called ‘butter’, ‘butter’ is called
(All the given codes are two letter codes only) ‘soap’, ‘soap’ is called ‘ink’, ‘ink’ is called
SBI (PO Pre) 2017 ‘honey’ and ‘honey’ is called ‘orange’, then
(a) si (b) either ‘nu’ or ‘si’ which of the following will be used for
(c) Either ‘bo’ or ‘si’ (d) dm washing clothes?
(e) bo (a) Honey (b) Butter (c) Orange (d) Soap
(e) Ink
58. In a certain code language, ‘give your book’
is coded as ‘sk dg ap’ and ‘book my tickets’ 65. If ‘BEAR’ is coded as ‘FISH’, ‘FISH’ as
is coded as ‘dg le nb’. What is the code for ‘CROW’, ‘CROW’ as ‘DOG’, ‘DOG’ as
‘give’ in the given code language? ‘ELEPHANT’ and ‘ELEPHANT’ as ‘ASS’,
IPPB Assist. Manager (Pre) 2016 then who can not remain alive in other place
(a) dg (b) nb than water? OPSC 2018
(c) Either ‘ap’ or ‘sk’ (d) Either ‘dg’ or ‘nb’ (a) FISH (b) ELEPHANT
(e) le (c) DOG (d) CROW
52 How to Crack Test of Reasoning VERBAL

Expert Level Exercise


Directions (Q. Nos. 1-5) In each question Directions (Q. Nos. 6-9) In each of the
below, a group of digits/symbols is given, followed following question, a group of numbers/ symbols
by four combinations of letters numbered (a), (b), followed by five combinations of letter codes is
(c) and (d). You have to find out which of the given. You have to find out which of the
combinations (a), (b), (c) and (d) correctly combinations correctly represents the group of
represents the group of digits/symbols based on numbers/symbols based on the given coding
the following coding system and the conditions system and the conditions and mark that
those follow and mark the number of that combination as your answer.
combination as your answer. If none of the four IBPS Bank Clerk (Pre) 2015
combinations correctly represents the group of Number or 5 $ % 9 4 # 8 ★ 2 @ & 3 + 7 6
digits/symbols, mark (e) i. e. ‘None of these’ as the Symbol
answer. Vijaya Bank (Clerk) 2012 Letter P E H U W A C X B G K Y V Z J
Code
Digits/ 5 9 @  3 8 1 $ % 4 2 6 ★ 7 δ #
Symbols Conditions
Letters/ B E P A K D F H Q I R J U M V T (i) If the first and the last elements are odd

A
Codes
numbers then their codes are to be

M
Conditions interchanged.

H
(i) If the first unit in the group is an even (ii) If an even number is immediately
followed as well as immediately preceded

TT
digit and the last unit is a symbol, both
these are to be coded as the code for the by a symbol then that even number is to
be coded as ‘O’.
symbol.
(ii) If the first unit in the group is an odd digit
A
(iii) If the third element is symbol then the
W
and the last unit is an even digit their first element is to be coded as the code of
SH

codes are to be interchanged. that symbol.


(iii) If both the first and the last units in the (iv) If the second last element is an odd
A

group are symbols, both these are to be number then the code of that odd number
coded as ‘X’. is to be interchanged with the code of last
@

1. @91$26 element.
(a) JEFHRP (b) PEFHRP (Please Note) : All the elements have to be counted from left
(c) XEFHRX (d) PEFHRJ to right to fulfil the conditions.
(e) None of these
6. 9 7 @ ★ + 4
2. 387#9 (a) WZGXVU (b) UZGXDW
(a) KMDTAE (b) KDMATE (c) UZGXVW (d) GZGXVW
(c) EDMTAK (d) KDMTAE (e) GZGVXW
(e) None of these 7. 7 # 6 8 & 3
3. 4@312δ (a) YADCKZ (b) YAJCKZ
(a) VPKFRV (b) VPKFRI (c) XPKFRX (d) IPKFRV (c) ZAJCKY (d) KAJCKY
(e) None of these (e) ZAJCYK

4. %4187★ 8. 4 # 5 % 9 2
(a) PAWHUB (b) WAOHBU
(a) QIFDMU (b) UNIFDMQ
(c) BAPHUW (d) BAPWUH
(c) XIFDMX (d) UIFDMU
(e) WAPHBU
(e) None of these
9. 7 5 4 % 2 $
5. 9124δ 6 (a) ZPWHBE (b) HPWHBE
(a) EFRIVJ (b) JFRIVE (c) EFRIVE (d) XFRIVX (c) ZPWHOE (d) HPWBOE
(e) None of these (e) EPWHBZ
Chapter 05 CODING-DECODING 53

Directions (Q. Nos. 10-13) Study the given 15. What is the code for ‘supply’ in the given
information carefully to answer the given code language?
questions. SBI (PO) 2015 (a) Only ta (b) Only mo
Codes (c) Either pa or mo (d) Only pa

‘cinderella shouted for rescue’ is written as ‘pr (e) Either mo or ta
co ly bu’.
16. What may be the possible code for ‘demand

‘rescue all the bugs’ is written as ‘ke mt co rx’. only more’ in the given code language?

‘bugs ate all carrots’ is written as ‘vg rx ke sh’. (a) xi ne mo (b) mo zi ne

‘carrots for pretty cinderella’ is written as ‘ly (c) ki ne mo (d) mo zi ki
pr vg as’. (All codes are two-letter codes only). (e) xi ka ta
10. In the given code language, what does the 17. What may be the possible code for ‘work
code ‘pr’ stand for? and money’ in the given code language?
(a) Either ‘bugs’ or ‘shouted’ (a) pa ga la (b) pa la lu
(b) Rescue (c) mo la pa (d) tu la ga
(c) Either ‘cinderella’ or ‘for’ (e) pa la ne
(d) For
(e) Pretty Directions (Q. Nos. 18-21) Study the following

A
information carefully to answer the given
11. What is the code for ‘bugs’ in the given code questions. Indian Bank PO (Pre) 2016

M
language? In a certain code language

H
(a) Other than those given as options ‘plot for all persons’ is written as ‘fn bo dl sw’

TT
(b) co
‘find the hidden plot’ is written as ‘dl et ga nu’
(c) sh
‘try and find out’ is written as ‘ga yc mp zh’
(d) Either ‘co’ or ‘vg’
(e) Either ‘ke’ or ‘rx’ A
‘for the lock out’ is written as ‘nu mp fn rv’
W
(All codes are two letter codes only)
12. What may be the possible code for ‘pretty’ in
SH

the given code language? 18. If ‘try the key’ is coded as ‘nu ka yc’ in the
(a) ly (b) pr (c) vg (d) as given code language, then how will ‘key and
A

(e) vg or as lock’ be coded as?


@

(a) ka bo zh (b) zh ga ka
13. What may be the possible code for ‘shouted (c) ka zh rv (d) bo rv ga
and ate’ in the given code language? (e) Other than those given as options
(a) bu sh mt (b) rx co gy
(c) ly rx vg (d) gy sh as 19. In the given code language, what does the
(e) sh gy bu code ‘sw’ stand for?
(a) either ‘for’ or ‘find’ (b) either ‘persons’ or ‘all’
Directions (Q. Nos. 14-17) Study the following
(c) hidden (d) for
information carefully and answer the following
(e) plot
questions. SBI (PO) 2013
Codes 20. Which of the following may represent the

‘economics is not money’ is written as ‘ka la ho code for ‘find friends’ in the given code
ga’. language?

‘demand and supply economics’ is written as (a) bo ga (b) ga cl
‘mo ta pa ka’. (c) fn ga (d) et bo

‘money makes only part’ is written as ‘zi la ne ki’. (e) cl et

‘demand makes supply economics’ is written as
‘zi mo ka ta’.
21. What is the code for ‘for hidden plot’ in the
given code language?
14. What is the code for ‘money’ in the given (a) fn et dl (b) fn dl bo
code language? (c) ga bo fn (d) mp rv et
(a) ga (b) mo (c) pa (d) ta (e) dl et ga
(e) la
54 How to Crack Test of Reasoning VERBAL

Directions (Q.Nos 22-24) Study the Directions (Q. Nos. 25-27) Study the following
information below and answer the following information carefully to answer the given
questions. questions. IBPS Clerk (Mains) 2017
In a certain code language, In a certain code language ‘work for earning
‘Thin paper neatly folded’ is written as @D6, %R5, money’ is coded as ‘Go3 None5 Xor 4 Farnin7’
!N4, ?Y6 ‘like six years passed’ is coded as ‘Ti3 Qasses6
‘Four people from USA’ is written as @M4, %E6, Zear5 Mik4’ ‘hence good amount received’ is coded
#A3, @R4 as ‘Seceive8 lenc5 Hoo4 Bmoun6’
‘Urban development programme launched’ is 25. What is the code for ‘last earning was
written as %E9, *T11, #N5, &D8 money’?
‘Dhaya likes forties hero’ is written as @S7, &S5, (a) Xa3 Mas4 None5 Farnin7
*A5, $O4 (b) None5 Xa3 Mas4 Darnin7
(c) Mone5 Mas4 Farnin7 Ya3
22. The code for the word ‘people’ is (d) Mas4 one5 Farnin7 Xa3
(a) @M4 (b) %E6 (e) None of the above
(c) #A3 (d) @R4
(e) None of these 26. What is the code for ‘hence always wrong
hance’?
23. The code ‘*A5’ denotes which of the

A
(a) Blway6 lance5 Xron5 Lenc5
following word?

M
(b) Lanc5 Xron5 Blway5 Lenc5
(a) Likes (b) Hero (c) Lanc5 Xron5 Blway5 Lenc5

H
(c) Forties (d) Dhaya (d) Blway6 Ienc5 Ianc5 Xron5

TT
(e) None of these (e) Lanc6 Xron6 Blway5 Lenc5
24. The code word of ‘paper’ is
(a) @R4 (b) %E6 (c) @M4 (d) #A3 A
27. ‘Farming’ is coded as
W
(a) Garmin7 (b) Gramin8 (c) Garing8 (d) Earnin7
(e) None of these (e) Earnin8
SH

Answers
A
@

Base Level
1 (a) 2 (b) 3 (c) 4 (d) 5 (a) 6 (a) 7 (b) 8 (c) 9 (b) 10 (a)
11 (b) 12 (d) 13 (b) 14 (a) 15 (d) 16 (b) 17 (e ) 18 (d) 19 (b) 20 (c)
21 (b) 22 (c) 23 (b) 24 (b) 25 (a) 26 (d) 27 (c) 28 (a) 29 (c) 30 (a)
31 (c) 32 (a) 33 (a) 34 (d) 35 (c) 36 (b) 37 (c) 38 (b) 39 (a) 40 (b)
41 (a) 42 (d) 43 (e) 44 (b) 45 (c) 46 (a) 47 (c) 48 (a) 49 (a) 50 (a)
51 (d) 52 (c) 53 (d) 54 (c) 55 (a) 56 (a) 57 (d) 58 (c) 59 (d) 60 (c)
61 (a) 62 (c) 63 (b) 64 (e) 65 (d)

Expert Level
1 (d) 2 (d) 3 (a) 4 (c) 5 (b) 6 (d) 7 (b) 8 (e) 9 (c) 10 (c)
11 (e) 12 (d) 13 (e) 14 (e) 15 (e) 16 (a) 17 (b ) 18 (c) 19 (b) 20 (b)
21 (a) 22 (b) 23 (d) 24 (e) 25 (a) 26 (d) 27 (a)
Chapter 05 CODING-DECODING

Answer with Explanations


Base Level Exercise
1. (a) As, S O B E R 6. (a) As, Similarly,
–1 –1 –1 –1 –1 C +1 B S +1 B
U S T F
R N A D Q R V E U
A D A T
Similarly, T D M F
L O T U S
I U I M
–1 –1 –1 –1 –1 V H N H
–1 –1
E S G L
K N S T R
7. (b) As,
2. (b) As, W A T E R +3 Similarly, +3
M P S V
+2 +2 +2 +2 +2 +3 +3
O R H K
Y C V G T

A
U U I I
−2 −2

M
Similarly, H K T G S Q F D
F I R E
−2 −2

H
E C T R
+2 +2 +2 +2

TT
8. (c) As, M U S I C A L
H K T G
–2 +2 –2 +2 –2 +2 –2
3. (c) As, T E R M I N A L A
W
K W Q K A C J
SH

Similarly,
N S F U M B O J S P R I N K L E
+1 +1 +1 +1 +1 +1 +1 +1
A

and –2 +2 –2 +2 –2 +2 –2 +2
T O W E R S Similarly, M A T E
@

Q R P K L M J G
X P U T S F B N F U
+1 +1 +1 +1 +1 +1 +1 +1 +1 +1 9. (b) As, B U T T E R E X W W H U
4. (d) The letters given here are opposite letters to each
other. +3
As, C E R T A I N and S E Q U E N C E +3
+3
+3
+3
X V I G Z R M H V J F V M X V +3
Similarly, R E Q U I R E D and
M I L K P L O N

I V J F R I V W +3
+3
5. (a) As, Similarly, +3
+3
B +1 J S +1 B
Similarly, E A R T H H D U W K
U V E F
I C A T
L –1 K L –1 K +3
+3
D S I H +3
E F N O +3
R +1 E G +1 J +3
56 How to Crack Test of Reasoning VERBAL

10. (a) As, Similarly,


C A S K E T
D E A R Similarly, R E A D +2 +3 +4 –2 –3 –4
+2 +2 +2 +2 +2 +2 +2 +2
E D W I B P
F G C T T G C F 16. (b) As,
11. (b) As, B A R L E Y R U L E and B A N D

+1 +1 +2 +1 +2 +1 +2 +2
+2 +2 +2 +2 +2 +2
C B T M G Z G N W T F P C D
Similarly, S O F A
Similarly,
+2
S O U N D
+2 +2 +2
+1 +1 +2 +1 +2
C H Q U

T P W O F 17. (e) As,


12. (d) As, P O R T E R R O U T E and L O W E R
+3 +3 –1 +3 +3 +3 +3 –1 +3 +3
+1 –1 +1 –1 +1 –1

A
U R T W H O R V H U

M
Q N S S F Q Similarly,
Similarly,

H
P R I N T
+3 +3 +3 +3

TT
B R I G H T –1

+1 –1 +1 –1 +1 –1 S U H Q W

C Q J F I S 18. (d) As, A


W
J U G S and W H E N
13. (b) As, M O C K S
SH

+1 –1 –1 +1 +1 –1 –1 +1
+1 –1 +1 –1 +1
K T F T X G D O
A

N N D J T Similarly,
O N C E
@

Similarly, +1 –1 –1 +1
F L A M E
P M B F
+1 –1 +1 –1 +1
19. (b) As, D U R A B L E
G K B L F –1 +1 –1

14. (a) As, T H E N Q T C B D K A


–2 +4 –3 +5 Similarly,
R L B S C O U N T R Y
Similarly, –1 +1 –1
C A S E
–2 +4 –3 +5
T N B O X Q S

A E P J 20. (c) As, F R I E N D


C A R I N G +2 +3 +4 +5 +6 +7
15. (d) As,
+2 +3 +4 –2 –3 –4 H U M J T K
E D V G K C Similarly, B H O P A L
and +2 +3 +4 +5 +6 +7
S H A R E S
+2 +3 +4 –2 –3 –4 D K S U G S
U K E P B O
Chapter 05 CODING-DECODING 57

21. (b) As, T I N Y 28. (a) Here, each letter is coded by its position in
–2 –2 –2 –2 English alphabetical order.
∴ MORNS → 1315181419
R G L W
29. (c) Given, REASON = 5
Similarly, Z E B R A
BELIEVED = 7
–2 –2 –2 –2 –2
Here, pattern is (number of letters –1).
X C Z P Y Now, number of letters in GOVERNMENT =10
∴Required code number = Number of letters −1
22. (c) As,
= 10 − 1 = 9
30. (a) Here, each letter is coded as twice its position in
reverse English alphabetical order as,
Similarly, 15 26 14 11
L A M P
C O N T I N U O U S

30 52 28 22
O C N T N I U O S U Similarly, 7 12 2

A
T O Y
23. (b) As, P E A C E

M
14 24 4

H
TT
16 5 1 3 5 31. (c) As, Similarly,
(positional value in alphabetical order) +6 +6
13 M 19 4 D 10
Similarly, L O V E +6 A +6
W
1 A 7 1 A 7
+6 +6
3 C 9 14 N 20
SH

+6 +6
12 15 22 5 8 H 14 7 G 13
+6 +6
A

9 I 15 5 E 11
24. (b) As, A = 1 (positional value)
@

+6 +6
14 N 20 18 R 24
and ACE = 1 + 3 + 5 (positional values of A,C,E) +6
=9 5 E 11
Similarly, ART = 1 + 18 + 20
32. (a) As,
= 39 (positional values of A,R,T)
3 1 20
10 21 9 3 5
25. (a) As, C A T = (3 + 1 + 20) ÷ 2 = 24 ÷ 2 = 12 J U I C E
13 1 14
Similarly, M A N 10×2 21×2 9×2 3×2 5×2
= (13 + 1 + 14) ÷ 2 = 28 ÷ 2 = 14 –1 –1 –1 –1 –1

26. (d) Here, the positional values of letters in backward 19 41 17 5 9


order are added. Similarly,
ASHA = 26 + 8 + 19 + 26 = 79
20 15 25
Similarly, V I N A Y B H U S H A N T O Y
= 5 + 18 + 13 + 26 + 2 + 25 + 19 + 6 + 8
+19 + 26 + 13 = 180
20×2 15×2 25×2
27. (c) Given, A = 2, B =4, C = 6, D = 8, …, Z = 52 –1 –1 –1
LADY = 2(12 + 1 + 4 + 25)= 2 × 42 = 84 39 29 49
58 How to Crack Test of Reasoning VERBAL

33. (a) As, P B and C M Solutions (Q. Nos. 36-39) According to the given
A D H O information,
R F I X G B F Q
E G L Q E E I H
D U T N R O
N J
R F A C E E
T K E G W D
N J A C
Similarly, Y I
R F
E G B P M Z
P B A C O M
R F
I X C T V W
N J K L E E
T K
W D and S U
C A R and T A R A C L F
34. (d) As, F I F T Y
R O O M
D X W D
C A C T Y, P O L T O L S U
,

A
Similarly, T A R I F F

M
36. (b) O M
V W

H
E E

TT
T O L A C C R O
35. (c) As, O C and O C 37. (c) D X
R B P X E E A
W
A C
G D E F
D X
SH

A W R B L F
N L A W Y I
A

I Q T Y
38. (b) R O
@

S J I Q E E
A W O C W D
A C
T Y N L
R O
I Q
D X
O C
N L 39. (a) G B
R O
Similarly, S J E E
E F A C
T N
P X
A W 40. (b) As, P A L E E A R T H
R B
and
A W 2 1 3 4 4 1 5 9 0
T Y
I Q Similarly, P E A R L
O C
N L 2 4 1 5 3
Chapter 05 CODING-DECODING 59

41. (a) As, and R A N C O R


S T A B L E L A B O U R
and 7 4 5 9 6 7

1 2 3 4 5 6 5 3 4 7 8 9 Similarly, R A G E

Similarly, B O T T L E
7 4 2 3

46. (a) As, B A S K E T


4 7 2 2 5 6

42. (d) As, 7 3 4 2 9 1 8 6 5 $ 3 % # 1


and T R I E D

P U R C H A S E
1 4 * # 2
Similarly,
Similarly,
S K I R T
5 4 0 6 4 7 4 5 2 6 8 8

A
M
3 % * 4 1
O R D E R P R O C E S S

H
47. (c) As, S 5 and R 2

TT
43. (e) As, L 1 and F 5
A @ I c
I 2 O 6
D %
N 3 R 7
F
A 3
E #
W
G 4 C 8 E #
R 2
E 5 E 9
SH

R 6 Similarly, F 3
Similarly, E #
A

F 5
D %
@

I 2 S 5
E 9/5
R 6/7 48. (a) As,
C 8 L O N G and G E A R
E 9/5
Hence, cannot be determined. 5 1 # 4 4 % $ 9

44. (b) As, Similarly, R O L E


M I L I T A R Y

1 2 3 2 4 5 6 7 9 1 5 %

49. (a) As, A U S T R A L I A


Similarly, T A I L

4 5 2 3 @ $ ! # ^ @ « ? @

45. (c) As, Similarly, S T R A I T


D A N G E R

1 4 5 2 3 7 ! # ^ @ ? #
60 How to Crack Test of Reasoning VERBAL

50. (a) 5 6 2 1 8 3 58. (c) According to the question,


give your book sk dg ap
P A V M R P book my tickets dg le nb
[condition (ii) follows] Hence, the code for ‘give’ is either ‘sk’ or ‘ap’.
51. (d) 6 2 7 8 5 1 59. (d) According to the given information,
ter ner ger you can determine
A V L R D M
fer ler ter kar can she help me
[no condition follows]
mer der ger how are you
52. (c) 8 1 2 3 5 4
It is clear from the above that the code for
determine is ner.
R M V P D R 60. (c) We know that, national flower of India is Lotus
[condition (i) follows] and here, Lotus is called Rose.
53. (d) 3 9 7 4 1 6 61. (a) Children study in ‘school’ but here ‘school’ is
called ‘house’. Hence, in this case ‘house’ is the
place where children go to study.
P B L F M A

A
62. (c) Lizard crawls and here animals who crawl are
[no condition follows] called flying.

M
54. (c) According to the given information, 63. (b) Glass is used for drinking water and here glass

H
975 throw away garbage …(i) is called saucer.

TT
528 give away smoking …(ii) 64. (e) Soap is used for washing clothes and here soap
is called ink.
213 smoking is harmful
From Eqs. (ii) and (iii), smoking → 2
…(iii)
A
65. (d) We know that, fish cannot live outside water and
W
here, fish is coded as crow. Hence, crow cannot
55. (a) According to the given information,
SH

live in place other than water.


po ki top ma Usha is playing cards
Expert Level Exercise
A

kop ja ki ma Asha is playing tennis


@

1. (d) @ 9 1 $ 2 6
ki top sop ho they are playing football

po sur kop P E F H R J
cards and tennis
[no condition follows]
So, Asha ja
2. (d) 3 8 7 # C 9
56. (a) As, on the day la si ko
K D M T A E
the setting day si mu la [no condition follows]
∴setting = mu 3. (a) 4 @ 3 1 2 δ
57. (d) According to the question,
V P K F R V
job requires expertise la nu si [condition (i) follows]
expertise in area li bo la 4. (c) % 4 1 8 7 «

requires area inspection si dm bo


X I F D M X
∴inspection → dm [condition (iii) follows]
Chapter 05 CODING-DECODING 61

5. (b) 9 1 2 4 δ 6 14. (e) money → la


15. (e) supply → either mo or ta
J F R I V E
16. (a) demand only more → xi ne mo
[condition (ii) follows]
17. (b) work and money → pa la lu
6. (d) 9 7 @ + 4
Solutions (Q. Nos. 18-21) The given information can
be represented as
G Z G X V W
[condition (iii) follows] plot for all persons fn bo dl sw

7. (b) 7 # 6 8 & 3 find the hidden plot dl et ga nu

try and find out ga yc mp zh


Y A J C K Z
[condition (i) follows] for the lock out nu mp fn rv

8. (e) 4 # 5 % 9 2 persons = bo/sw, try = yc/zh and = yc/zh


18. (c) ‘try the key’ → ‘nu ka yc’
W A P H B U
∴ try = yc and key = ka
[condition (iv) follows]
Hence, key and lock → ka zh rv

A
M
9. (c) 7 5 4 % 2 $ 19. (b) ‘sw’ stands for either ‘persons’ or ‘all’.

H
20. (b) find → ga
Z P W H O E

TT
∴find friends → ga cl
[condition (ii) follows]
21. (a) ‘for hidden plot’ → fn et dl
Solutions (Q. Nos. 10-13) According to the given
information, A
Solutions (Q. Nos. 22-24) First code-Symbol is as per
W
cinderella shouted for rescue pr co ly bu ...(i) the first letter of the word.
SH

T P N F U D L H
rescue all the bugs ke mt co rx ...(ii)
! % ? @ # * & $
A

vg rx ke sh ...(iii) Second code - last letter of the word.


@

bugs ate all carrots


...(iv) Third code - Number of letters in the word
carrots for pretty cinderella ly pr vg as
22. (b) Code of ‘People’ is %E6.
10. (c) ‘pr’ either cinderella or for
23. (d) *A5 is the code for ‘Dhaya’.
11. (e) bugs either ke or rx
24. (e) Code for ‘paper’ is %R5.
12. (d) pretty as Solutions (Q. Nos. 25-27) In the code, the number
13. (e) shouted and ate bu gy sh represent the number of letters in the word.
Solutions (Q. Nos. 14-17) On the basis of given The first letter in the code represents the next letter
information, of the first letter of the word.
The letters in the middle of the code are the letters
economics is not money ka la ho ga ...(i) after removing the first and last letter.

demand and supply economics mo ta pa ka ...(ii) 25. (a) Last earning was money → Mas4 Farnin7 Xa3
None5 ⇒ or Xa3 Mas4 None5 Farmin7
zi la ne ki ...(iii)
money makes only part 26. (d) Hence always wrong hence → Ienc5 Blway6
Xron5 Ianc5 or Blway6 Ienc5, Ianc5Xron5
demand makes supply economics
27. (a) Farming → Garmin7
zi mo ka ta ...(iv)
62 How to Crack Test of Reasoning VERBAL

06
Number Series
Number series is a sequential arrangement of Solution (b) Pattern of the series is as shown below
numbers following a certain defined pattern. 40 30 22 16 12
In this chapter, we deal with questions in
which a series of numbers, (which are –10 –8 –6 –4

A
generally called the terms of the series) is This series consists of a pattern of subtraction of
consecutive even numbers.

M
given. These numbers/terms follow a certain
2. Addition/subtraction of prime

H
pattern throughout the series. Candidates are
asked either to find a missing term or to find numbers These series follow the pattern

TT
the wrong term of the series. of addition/subtraction of prime numbers.

Important Patterns of the A


Illustration 3. 1, 3, 6, 11, 18, ?
W
(a) 25 (b) 29 (c) 32 (d) 35
Solution (b) Pattern of the series is as shown below
Series
SH

1 3 6 11 18 29
Different series based on some particular
A

patterns are discussed below +2 +3 +5 +7 +11


@

This series consists of a pattern of addition of prime


I. Based on Addition/Subtraction numbers.
of Numbers Illustration 4. 32, 19, 8, ?
1. Addition/subtraction of consecutive (a) 12 (b) 3 (c) 1 (d) 5
odd /even numbers These series follow Solution (c) Pattern of the series is as shown below
the pattern of addition or subtraction of
32 19 8 1
even or odd numbers.
Illustration 1. 5, 6, 9, 14, 21, ? –13 –11 –7
(a) 26 (b) 28 (c) 30 (d) 34 This series consists of a pattern of subtraction of
prime numbers.
Solution (c) Pattern of the series is as shown below
5 6 9 14 21 30 II. Based on Multiplication or
+1 +3 +5 +7 +9
Division of Numbers
This series consists of a pattern of addition of These series follow the pattern based on
consecutive odd numbers. multiplication and division of numbers.
Illustration 2. 40, 30, 22, 16, ? Illustration 5. 1, 3, 9, 27, 81, ?
(a) 10 (b) 12 (c) 15 (d) 17 (a) 216 (b) 220 (c) 243 (d) 250
Chapter 06 NUMBER SERIES 63

Solution (c) Pattern of the series is as shown below Illustration 9. 1, 2, 10, 37, 101, ?
1 3 9 27 81 243 (a) 402 (b) 206
(c) 226 (d) 320
×3 ×3 ×3 ×3 ×3 Solution (c) Pattern of the series is as shown below
Here, next number of this series is multiplication of
3 with previous one. 1 2 10 37 101 226

Illustration 6. 1296, 648, 216, ?, 36, 18, 6, 3


+ 13 + 23 + 33 + 43 + 53
(a) 112 (b) 108
This series is based on addition of cubes of
(c) 106 (d) 110
consecutive natural numbers.
Solution (b) Pattern of the series is as shown below
1296 648 216 108 36 18 6 3
Illustration 10. 225, 100, 36, 9, 1, ?
(a) 7 (b) 6
÷2 ÷3 ÷2 ÷3 ÷2 ÷3 ÷2
(c) 0 (d) 1
Solution (c) Pattern of the series is as shown below
Here, the terms are divided by 2 and 3 alternately.
216 225 100 36 9 1 0
So, missing term is obtained as = 108
2
– 53 – 43 – 33 – 23 – 13
III. Based on Addition/Subtraction of

A
This series is based on subtraction of cubes of

M
Squares/ Cubes of Natural Numbers consecutive natural numbers.

H
1. Addition/subtraction of squares of
IV. Based on Multiple Operation

TT
natural numbers These series follow the
pattern based on addition/subtraction of These series follow the pattern based on more
squares of natural numbers. A
than one mathematical operations. The
W
multiple operations can be multiplication and
Illustration 7. 1, 2, 6, 15, 31, ?
addition, multiplication and subtraction,
SH

(a) 56 (b) 64 division and subtraction etc.


(c) 60 (d) 58
Illustration 11. 13, 25, 51, 101, 203, ? CLAT 2015
A

Solution (a) Pattern of the series is as shown below


(a) 405 (b) 406
@

1 2 6 15 31 56
(c) 407 (d) 411
+1 2
+2 2
+3 2
+4 2
+5
2 Solution (a) Pattern of the series is as shown below
This series is based on addition of squares of 13 25 51 101 203 405
consecutive natural numbers.
×2–1 ×2+1 ×2–1 ×2+1 ×2–1
Illustration 8. 71, 55, 46, 42, ?
(a) 40 (b) 41 (c) 39 (d) 38 This sequence is based on multiplication of 2 and
then subtraction and addition of 1 alternately.
Solution (b) Pattern of the series is as shown below
71 55 46 42 41
Illustration 12. 5, 11, 23, 47, 95, ?
(a) 198 (b) 194
2 2 2 2 (c) 191 (d) 185
–4 –3 –2 –1
This series is based on subtraction of squares of Solution (c) Pattern of the series is as shown below
consecutive natural numbers. 5 11 23 47 95 191
2. Addition/subtraction of cubes of natural
×2+1 ×2+1 ×2+1 ×2+1 ×2+1
numbers These series questions, follow
the pattern based on addition/ subtraction This series is based on the pattern,
of cubes of natural numbers. next number = ( Previous number × 2 + 1)
64 How to Crack Test of Reasoning VERBAL

V. Based on Combination of Types of Questions


Two or More Series
These series contains the combination of two or There are mainly two types of questions which
more series. are asked in various competitive exams.
Illustration 13. 1, 2, 2, 4, 3, 8, 7, 10, ?
SSC (CGL) 2015 Type 1
(a) 9 (b) 8 (c) 11 (d) 13
Solution (c) Pattern of the series is as shown below To Find a Missing Term
+2 +4 +2
Here, we are given a series of numbers
1 2 2 4 3 8 7 10 11 including one or more missing numbers. This
series follows a pattern, keeping this pattern
+1 +1 +4 +4
in mind, we have to find the missing terms.
This series contains two separate series. In first
series, 1 is added in first two steps and 4 is added in Directions (Illustrations 15-17) In the
third and fourth steps. In second series, 2 and 4 is following questions, a series is given with one
added alternately. term missing. Choose the correct alternative from
the given ones that will complete the series.
Illustration 14. 3, 8, 6, 14, ?, 20

A
WBPSC 2018
Illustration 15. 831, 842, 853, 864, 875, ?
(a) 12 (b) 20 (c) 42 (d) 9

M
SSC (10+2) 2017
Solution (a) The pattern of the series is as shown

H
(a) 896
below 3 8 6 14 12 20 (b) 876

TT
(c) 886
×2 ×2
(d) 880
+6
This sequence contains two separate series. In first
+6
A
Solution (c) Pattern of the series is as shown below
W
series, each term is multiplied by 2. 831 842 853 864 875 886
SH

In the second series, 6 is added to the previous term


+ 11 + 11 + 11 + 11 + 11
to obtain the next term.
Hence, the missing term is 886.
A

Important Points Illustration 16. 8, 16, ?, 64, 128 SSC (10+2) 2017
@

(a) 30 (b) 32
l
If the change is slow or gradual, then it is a difference
(c) 28 (d) 48
series.
e.g. 20 19 17 14 10 Solution (b) Pattern of the series is as shown below
8 16 32 64 128
–1 –2 –3 –4
l
If the change is equally sharp, then it is a ratio series. ×2 ×2 ×2 ×2
e.g. 4 8 16 32 64 Hence, the missing term is 32.

×2 ×2 ×2 ×2 Illustration 17. 7, 8, 18, 57, ?, 1165


l
If the rise is sharp initially, but slows down later, then it SSC (CGL) 2014
(a) 174
is formed by adding squared, or cubed numbers.
(b) 232
e.g. 4 40 65 81 90 94 (c) 224
(d) 228
+ 62 + 52 + 42 + 32 + 22
Solution (b) Pattern of the series is as shown below
l
If the series is alternating, then it may be either a mixed
series or two different operations going on alternately. 7 8 18 57 232 1165
e.g. +5 +5
×1+1 ×2+2 ×3+3 ×4+4 ×5+5
2 5 4 10 8 15 16 Hence, the missing term is 232.
×2 ×2 ×2
Chapter 06 NUMBER SERIES 65

Solution (d) Pattern of the series is as shown below


Type 2 136
10 14 28 32 64 68 132
To Find the Wrong term
+4 ×2 +4 ×2 +4 ×2
Here, we are given a series of numbers.
Whole series except one number follow a So, 132 is wrong term, it must be replaced by
certain rule. We have to find the number/term 68 × 2 =136
which does not follow the rule. Illustration 19. 2, 3, 12, 37, 86, 166, 288
Directions (Illustrations 18-19) In each of the (a) 2 (b) 3 (c) 166 (d) 86
following question, one number is wrong in the Solution (c) Pattern of the series is as shown below
series. Find out the wrong number. 167
2 3 12 37 86 166 288
Illustration 18. 10, 14, 28, 32, 64, 68, 132
(a) 28 (b) 32 + 12 + 32 + 52 + 72 + 92 + 112
(c) 64 (d) 132
So, 166 is wrong term, it must be replaced is 167.

A
Let us Practice

M
H
TT
Directions (Q. Nos. 1-35) In each of the 1 2 5 2 2
10. 7 , 8 , 9 , 12 , 16 ,? SSC (CPO) 2015
following questions, a number series is given with 7 6 5 4 3
one of the terms missing. Choose the correct (a) 15
2 A
(b) 16
4
(c) 35 (d)
50
W
alternative that will continue the same pattern 4 4 2
and replace the question mark (?) in the given 11. 24, 6, 18, 9, 36, 9, 24, ?
SH

series. United Bank (Clerk) 2011


(a) 24 (b) 12 (c) 8 (d) 6
1. 23, 28, 34, 41, 49, ?
A

SSC (10+2) 2018 (e) 26


(a) 57 (b) 59 (c) 56 (d) 58
@

12. 24, 35, 20, 31, 16, 27,? SSC (CPO) 2015
2. 5, 11, 17, 25, 33, 43, ? SSC (CGL) 2016 (a) 5 (b) 12
(a) 49 (b) 51 (c) 52 (d) 53 (c) 8 (d) 9
3. 11, 13, 17, 19, 23, 25, ? SSC (10+2) 2018 13. 1, 4, 27, 256, ?
(a) 29 (b) 27 (c) 31 (d) 37
(a) 625 (b) 3125 (c) 3025 (d) 1225
4. 2, 5, 10, 17, 26, 37, ? UP Police (Constable) 2018
14. 8, 3, 11, 14, 25, ?
(a) 40 (b) 42 (c) 49 (d) 50
(a) 50 (b) 39 (c) 29 (d) 11
5. 4, 7, 11, 18, 29, 47, ? , 123, 199 WBPSC 2018
(a) 76 (b) 70 (c) 84 (d) 102
15. 1, 6, 21, 66, ? SSC (10+2) 2017
(a) 250 (b) 201 (c) 310 (d) 308
6. 3, 9, 6, 36, 30, ? SSC (10+2) 2013
16. 2, 10, 30, 68, ? UP Police (Constable) 2018
(a) 270 (b) 250 (c) 200 (d) 300 (a) 101 (b) 102 (c) 130 (d) 104
7. 97, 90, 76, 55, ? 17. 4, ?, 144, 400, 900, 1764 SSC (Steno) 2012
(a) 28 (b) 27 (c) 26 (d) 25 (a) 25 (b) 36 (c) 49 (d) 100
8. 0.5, 1.5, 5, 18, 76, ? SSC (CPO) 2017 18. 3, 6, 9, 15, 24, 39, 63, ? SSC (CPO) 2014
(a) 380 (b) 385 (c) 390 (d) 375 (a) 100 (b) 87 (c) 102 (d) 99

9. 198, 202, 211, 227,? BSSC (CGL) 2015 19. 30, 68, 130, 222, ?, 520, 738 SSC (FCI) 2012
(a) 236 (b) 252 (a) 420 (b) 350
(c) 275 (d) 245 (c) 250 (d) 280
66 How to Crack Test of Reasoning VERBAL

20. 29, 38, 47, ? SSC (CGL) 2014 Directions (Q. Nos. 37-39) What will come in
(a) 59 (b) 56 (c) 52 (d) 58 place of the question marks in the following
number series? IBPS (PO/MT) (Pre) 2016
21. 3.5, 7, 10.5, 14, ? SSC (Multitasking) 2013
(a) 15.5 (b) 16.5 (c) 18.5 (d) 17.5 37. 11, 20, 38, 74, ?
(a) 146 (b) 154 (c) 128 (d) 132
22. 3, 128, 6, 64, 9, ?, 12, 16, 15, 8 IB (ACIO) 2012
(e) 136
(a) 32 (b) 12 (c) 108 (d) 72
38. 24, 28, 19, 35, 10, ?
23. 2, 6, 4, 9, 8, 13, 16, 18, 32, ? SSC (CPO) 2017
(a) 26 (b) 36 (c) 16 (d) 46
(a) 24 (b) 26 (c) 22 (d) 28
(e) 15
24. 0, 7, 26, 63,? SSC (CGL) 2015
39. 12, 19, 35, 59, 90, ?
(a) 187 (b) 96 (c) 123 (d) 124
(a) 134 (b) 127 (c) 132 (d) 98
25. 14, 14, 21, 42, 105, ? SSC (CGL) 2015 (e) 114
(a) 285 (b) 315 (c) 305 (d) 215
Directions (Q. Nos. 40-59) In each of the
26. 0, 3, 8, 15, 24, ?, 48 SSC (Multitasking) 2014 following question, one of the terms in the number
(a) 41 (b) 29 (c) 37 (d) 35 series is wrong. Find out the wrong term.
27. 7, 25, 61, 121, ? 40. 146, 74, 40, 23, 19.5, 18.75

A
SSC (10+2) 2013
(a) 210 (b) 211 (c) 212 (d) 209

M
(a) 74 (b) 23 (c) 19.5 (d) 18.75
(e) 40
28. 5, 16, 51, 158, ? SSC (CGL) 2013

H
(a) 1452 (b) 483 (c) 481 (d) 1454 41. 2, 4, 16, 256, 65535

TT
29. 534, 543, 559, 584, 620, ? SSC (CPO) 2017 (a) 65535 (b) 256 (c) 4 (d) 16
(a) 648 (b) 676 (c) 669 (d) 671
A
42. 888, 454, 237, 128.5, 76.25, 47.125
W
30. 198, 194, 185, 169, ? SSC (CPO) 2013 (a) 76.25 (b) 454 (c) 128.5 (d) 237
(a) 92 (b) 136 (c) 144 (d) 112
SH

43. 1788, 892, 444, 220, 112, 52, 24


31. 0, 1, 1, 2, 3, 5, 8, 13, 21, ? CLAT 2013 RBI (Grade B) 2011
A

(a) 34 (b) 35 (c) 33 (d) 36 (a) 52 (b) 112 (c) 220


(d) 444 (e) None of these
@

32. 17, 36, 74, 150, ?, 606 CLAT 2013


(a) 250 (b) 303 (c) 300 (d) 302 44. 9, 16, 25, 36, 49, 61 SSC (Steno) 2012
(a) 16 (b) 9 (c) 49 (d) 61
33. 1, 0, 3, 2, 5, 6, ?, 12, 9, 20 SSC (CGL) 2014
(a) 9 (b) 10 45. 2, 8, 3, 27, 4, 64, 5, 225 IB (ACIO) 2011
(c) 7 (d) 8 (a) 27 (b) 8 (c) 225 (d) 64

34. 7, 4, 5, 9, ? 52.5, 160.5 46. 20, 40, 200, 400, 2000, 4000, 8000
Andhra Bank (Clerk) 2011
(a) 32 (b) 16
(a) 200 (b) 2000 (c) 8000 (d) 4000
(c) 14 (d) 20
(e) None of these
35. 4, 18, ?, 100, 180, 294. IB (ACIO) 2013
47. 9, 14, 19, 25, 32, 40 EPFO 2012
(a) 32 (b) 36
(a) 14 (b) 25 (c) 32 (d) 9
(c) 48 (d) 40 (e) 19
36. What should come next in the following 48. 11, 5, 20, 12, 40, 26, 74, 54
series? (a) 5 (b) 20 (c) 40 (d) 26
98765432198765432987654398
49. 2, 10, 50, 500, 1000
7654987?
(a) 2 (b) 10 (c) 50 (d) 1000
(a) 5 (b) 4
(c) 6 (d) 1 50. 1, 5, 5, 9, 7, 11, 11, 15, 12, 17
(e) None of these (a) 11 (b) 12 (c) 17 (d) 15
Chapter 06 NUMBER SERIES 67

51. 8, 13, 21, 32, 47, 63, 83 Directions (Q. Nos. 60-64) Find the one that
(a) 13 (b) 32 (c) 47 (d) 63 does not belong to the group. SSC (FCI) 2012

52. 4, 31, 130, 349, 728, 1339 60. 75, 79, 72, 80, 69, 83, 66
(a) 31 (b) 130 (c) 349 (d) 728 (a) 79 (b) 83 (c) 69 (d) 72

53. 1, 4, 8, 16, 31, 64, 127, 256 UP (B.Ed.) 2011 61. 1, 8, 27, 64, 127, 216
(a) 31 (b) 16 (a) 216 (b) 64 (c) 27 (d)127
(c) 8 (d) None of these
62. 6, 12, 21, 32, 45, 60
54. 2, 3, 6, 12, 24, 58 (a) 6 (b) 12 (c) 21 (d) 32
(a) 2 (b) 3 (c) 12 (d) 58
63. 8, 27, 125, 343, 1331
55. 33792, 8448, 2148, 528, 132, 33, 8.25 (a) 8 (b) 343
(a) 2148 (b) 528 (c) 8448 (d) 132 (c) 1331 (d) None of these

56. 2, 7, 24, 77, 236, 723, 2180 64. 2, 3, 9, 27, 112, 565, 3396
(a) 7 (b) 77 (c) 24 (d) 236 (a) 565 (b) 9
(c) 112 (d) 27
57. 160, 118, 83, 65, 34, 20

A
(a) 83 (b) 118 (c) 34 (d) 65 65. Which one of the following numbers 587, 875,

M
695, 767, 867 is different from the other
58. 10, 11, 15, 24, 40, 67 IBPS (RRB Scale-I) 2016 numbers, in accordance with some rule?

H
(a) 87 (b) 114 (c) 95 (d) 67 CGPSC 2014

TT
(e) None of these (a) 695 (b) 767
(c) 867 (d) 875
59. 10, 14, 28, 32, 64, 68, 132
(a) 28 (b) 32 (c) 64 (d) 132
CDS 2011
(e) None of these
A
W
SH

Answers
A

1. (d) 2. (d) 3. (a) 4. (d) 5. (a) 6. (a) 7. (b) 8. (b) 9. (b) 10. (d)
@

11. (b) 12. (b) 13. (b) 14. (b) 15. (b) 16. (c) 17. (b) 18. (c) 19. (b) 20. (b)
21. (d) 22. (a) 23. (a) 24. (d) 25. (b) 26. (d) 27. (b) 28. (c) 29. (c) 30. (c)
31. (a) 32. (d) 33. (c) 34. (d) 35. (c) 36. (c) 37. (a) 38. (d) 39. (b) 40. (b)
41. (a) 42. (a) 43. (b) 44. (d) 45. (c) 46. (c) 47. (d) 48. (c) 49. (d) 50. (b)
51. (c) 52. (d) 53. (c) 54. (d) 55. (a) 56. (d) 57. (d) 58. (d) 59. (d) 60. (a)
61. (d) 62. (a) 63. (d) 64. (b) 65. (c)

Answer with Explanations


1. (d) Pattern of the series is as shown below 3. (a) Pattern of the series is as shown below
+6 +6 +6
23 28 34 41 49 58
11 13 17 19 23 25 29
+5 +6 +7 +8 +9
+6 +6
Hence, the question mark will be replaced by 58. Hence, the question mark will be replaced by 29.
2. (d) Pattern of the series is as shown below 4. (d) Pattern of the series is as shown below
5 11 17 25 33 43 53 2 5 10 17 26 37 50
+6 +6 +8 +8 +10 +10
+3 +5 +7 +9 +11 +13
Hence, the question mark will be replaced by 53. Hence, the question mark will be replaced by 50.
68 How to Crack Test of Reasoning VERBAL

5. (a) Pattern of the series is as shown below 14. (b) Pattern of the series is as shown below
4 7 11 18 29 47 76 123 199 8 + 3 = 11, 3 + 11 = 14, 11 + 14 = 25
14 + 25 = 39
+3 +4 +7 +11 +18 +29 +47 +76
Hence, the question mark will be replaced by 39.
15. (b) Pattern of the series is as shown below
3+4 4+7 7+11 11+18 18+29 29+47
1 6 21 66 201
Hence, the question mark will be replaced by 76.
+5 +15 +45 +135
6. (a) Pattern of the series is as shown below
3 9 6 36 30 270 ×3 ×3 ×3
Hence, the question mark will be replaced by 201.
×3 –3 ×6 –6 ×9 16. (c) Pattern of the series is as shown below
Hence, the question mark will be replaced by 270. 2 10 30 68 130
7. (b) Pattern of the series is as shown below
97 90 76 55 27
(1)3 +1 (2)3 +2 (3)3 +3 (4)3 +4 (5)3 +5
–7 – 14 – 21 – 28 Hence, the question mark will be replaced by 130.
Hence, the question mark will be replaced by 27. 17. (b) Pattern of the series is as shown below

A
8. (b) Pattern of the series is as shown below 900 1764
4 36 144 400

M
0.5 1.5 5 18 76 385

H
×1+1 ×2+2 ×3+3 ×4+4 ×5+5 (2)2 (6)2 (12)2 (20)2 (30)2 ( 42) 2

TT
Hence, the question mark will be replaced by 385.
+4 +6 +8 +10 +12
9. (b) Pattern of the series is as shown below
A
Hence, the question mark will be replaced by 36.
W
198 202 211 227 252 18. (c) Pattern of the series is as shown below
SH

+(2)2 +(3)2 +(4)2 +(5)2 3 6 9 15 24 39 63 102


Hence, the question mark will be replaced by 252.
A

+3 3+6 6+9 9+15 15+24 24+39 39+63


10. (d) Pattern of the series is as shown below
@

Hence, the question mark will be replaced by 102.


1 2 5 2 2
7 , 8 , 9 , 12 , 16
7 6 5 4 3 19. (b) Pattern of the series is as shown below
50 50 50 50 50 50
→ → → → → 30 68 130 222 350 520 738
7 6 5 4 3 2
50
Hence, the question mark will be replaced by . +38 +62 +92 +128 +170 +218
2
11. (b) Pattern of the series is as shown below
24 6 18 9 36 9 24 12 +24 +30 +36 +42 +48

÷4 ÷2 ÷4 ÷2 +6 +6 +6 +6
Hence, the question mark will be replaced by 12. Hence, the question mark will be replaced by 350.
12. (b) Pattern of the series is as shown below
–4 –4 20. (b) Pattern of the series is as shown below
29 38 47 56
24 35 20 31 16 27 12
+9 +9 +9
–4 –4 –4
Hence, the question mark will be replaced by 56.
Hence, the question mark will be replaced by 12.
21. (d) Pattern of the series is as shown below
13. (b) Pattern of the series is as shown below
(1) = 1, (2 ) = 4, (3) = 27, (4) = 256,
1 2 3 4 3.5 7 10.5 14 17.5

(5)5 = 3125 +3.5 +3.5 +3.5 +3.5


Hence, the question mark will be replaced by 3125. Hence, the question mark will be replaced by 17.5.
Chapter 06 NUMBER SERIES 69

22. (a) Pattern of the series is as shown below 30. (c) Pattern of the series is as shown below
+3 +3 +3 +3
198 194 185 169 144
3 128 6 64 9 32 12 16 15 8
2 2 2 2
–2 –3 –4 –5
÷2 ÷2 ÷2 ÷2
Hence, the question mark will be replaced by 32. Hence, the question mark will be replaced by 144.
23. (a) Pattern of the series is as shown below 31. (a) Pattern of the series is as shown below
×2 ×2 ×2 ×2 The number is the addition of previous two
numbers
2 6 4 9 8 13 16 18 32 24
So, 0 + 1 = 1 ; 1+ 1 = 2
+3 +4 +5 +6 1+ 2 = 3 ; 2+3=5
Hence, the question mark will be replaced by 24. 3+ 5 = 8; 5 + 8 = 13
24. (d) Pattern of the series is as shown below 8 + 13 = 21 ; 13 + 21 = 34
0 7 26 63 124 Hence, the question mark will be replaced by 34.
32. (d) Pattern of the series is as shown below
(13 –1) (23 –1) (33 –1) (43 –1) (53 –1) 17 36 74 150 302 606
Hence, the question mark will be replaced by 124.
(× 2 + 2 ) (× 2+2)(× 2+2) (× 2+2) (× 2+2)

A
25. (b) Pattern of the series is as shown below

M
14 14 21 42 105 315 Hence, the question mark will be replaced by 302.

H
33. (c) Pattern of the series is as shown below
×1 × 1.5 ×2 × 2.5 ×3

TT
+2 +4 +6 +8
Hence, the question mark will be replaced by 315. 1 0 3 2 5 6 7 12 9 20
26. (d) Pattern of the series is as shown below A
W
+2 +2 +2 +2
12 − 1 = 0 ; 22 − 1 = 3
Hence, the question mark will be replaced by 7.
SH

32 − 1 = 8 ; 42 − 1 = 15
34. (d) Pattern of the series is as shown below
52 − 1 = 24 ; 62 − 1 = 35
7 × 0.5 + 0.5 = 4
A

7 2 − 1 = 48 4 × 1+ 1= 5
@

Hence, the question mark will be replaced by 35. 5 × 1.5 + 1.5 = 9


27. (b) Pattern of the series is as shown below 9 × 2 + 2 = 20
7 25 61 121 211 20 × 2.5 + 2.5 = 52.5
52.5 × 3 + 3 = 160.5
+18 +36 +60 +90 Hence, the question mark will be replaced by 20.
+18 +24 +30 35. (c) Pattern of the series is as shown below
4 18 48
+6 +6 ↓ ↓ ↓
Hence, the question mark will be replaced by 211 2 2 × 1 32 × 2 42 × 3
28. (c) Pattern of the series is as shown below 100 180 294
5 × 3 + 1 = 16 ↓ ↓ ↓
52 × 4 62 × 5 7 2 × 6
16 × 3 + 3 = 51
51 × 3 + 5 = 158 Hence, the question mark will be replaced by 48.
158 × 3 + 7 = 481 36. (c) Pattern of the series is as shown below
Hence, the question mark will be replaced by 481. 9 8 7 6 5 4 3 2 1;
29. (c) Pattern of the series is as shown below 9 8 7 6 5 4 3 2;
534 543 559 584 620 669 9 8 7 6 5 4 3;
9 8 7 6 5 4;
2 2 2 2 2
+(3) +(4) +(5) +(6) +(7) 987 6
Hence, the question mark will be replaced by 669. Hence, the question mark will be replaced by 6.
70 How to Crack Test of Reasoning VERBAL

37. (a) Pattern of the series is as shown below 45. (c) Pattern of the series is as shown below
11 20 38 74 146 2 3 = 8, 33 = 27, 43 = 64, 53 = 125 ≠ 225
Hence, 225 is wrong term, it should be replaced by
×2–2 ×2–2 ×2–2 ×2–2
125.
Hence, the question mark will be replaced by 146.
46. (c) Pattern of the series is as shown below
38. (d) Pattern of the series is as shown below 20000
24 28 19 35 10 46 20 40 200 400 2000 4000 8000
+22 –32 +42 –52 +62
× 10 × 10 × 10
Hence, the question mark will be replaced by 46. × 10 × 10
39. (b) Pattern of the series is as shown below Hence, 8000 is wrong term, it should be replaced
12 19 35 59 90 127 by 20000.
47. (d) Pattern of the series is as shown below
+7 +16 +24 +31 +37
10
+9 +8 +7 +6 9 14 19 25 32 40
–1 –1 –1
+4 +5 +6 +7 +8
Hence, the question mark will be replaced by 127. Hence, 9 is wrong term, it should be replaced by 10.

A
40. (b) Pattern of the series is as shown below 48. (c) Pattern of the series is as shown below

M
25
74 40 23 19.5 18.75 ×2+2 ×2+2 ×2+2
146

H
38

TT
(÷2)+1 (÷2)+3 (÷2)+5 (÷2)+7 (÷2)+9 11 5 20 12 40 26 74 54
Hence, 23 is wrong term, it should be replaced by 25.
41. (a) Pattern of the series is as shown below +9
A + 18 + 36
W
65536 Hence, 40 is wrong term, it should be replaced by
2 4 16 256 65535 38.
SH

49. (d) Pattern of the series is as shown below


×2 ×4 × 16 × 256
5000
A

Hence, 65535 is wrong term, it should be replaced 2 10 50 500 1000


@

by 65536
42. (a) Pattern of the series is as shown below
×5 ×5 × 10 × 10
74.25
888 454 237 128.5 76.25 47.125 Hence, 1000 is wrong term, it should be replaced
by 5000.
(÷2)+10 (÷2)+10 (÷2)+10 (÷2)+10 (÷2)+10 50. (b) Pattern of the series is as shown below
Hence, 76.25 is wrong term, it should be replaced +4 +2 +4 +2
by 74.25
43. (b) Pattern of the series is as shown below 13
1 5 5 9 7 11 11 15 12 17
108
1788 892 444 220 112 52 24
+4 +2 +4 +2
÷2– 2 ÷2–2 ÷2– 2 ÷2 – 2 ÷2– 2 ÷2 – 2
Hence, 12 is wrong term, it should be replaced by 13.
Hence, 112 is wrong term, it should be replaced by 51. (c) Pattern of the series is as shown below
46
108. 8 13 21 32 47 63 83
44. (d) Pattern of the series is as shown below
64 +5 +8 + 11 +14 + 17 + 20
9 16 25 36 49 61
+3 +3 +3 +3 +3
(3)2 (4)2 (5)2 ( 6) 2 (7)2 (8)2
Hence, 61 is wrong term, it should be replaced by Hence, 47 is wrong term, it should be replaced by
64. 46.
Chapter 06 NUMBER SERIES 71

52. (d)Pattern of the series is as shown below 59. (d) Pattern of the series is as shown below
736 136
4 31 130 349 728 1339 10 14 28 32 64 68 132

+4 ×2 +4 ×2 +4 ×2
(13 + 3) (33 + 4) (53 + 5) ( 7 3 + 6) (93 + 7) (113 + 8)
Hence, 132 is wrong term, it should be replaced by
Hence, 728 is wrong term, it should be replaced by 136.
736. 60. (a) Pattern of the series is as shown below
53. (c) Pattern of the series is as shown below +3 +3
7 77
75 79 72 80 69 83 66
1 4 8 16 31 64 127 256
–3 –3 –3
×2 + 2 ×2 – 1 ×2 + 2 ×2 – 1 ×2 + 2 ×2 – 1 ×2 + 2
Hence, 79 is wrong term, it should be replaced by
Hence, 8 is wrong term, it should be replaced by 7.
77.
54. (d) Pattern of the series is as shown below 61. (d) Pattern of the series is as shown below
48
2 3 6 12 24 58 125
1 8 27 64 127 216

A
2 +1 2 +3 +1 2 +3 +6 +1 2 +3 +6 +12 +1 2 +3 +6 +12 +24 +1

M
Hence, 58 is wrong term, it should be replaced by 13 23 33 43 53 63

H
48.
Hence, 127 is wrong term, it should be replaced by

TT
55. (a) Pattern of the series is as shown below
2112
125.
33792 8448 2148 528 132 33 8.25
A
62. (a) Pattern of the series is as shown below
W
÷4 ÷4 ÷4 ÷4 ÷4 ÷4 5
6
SH

12 21 32 45 60
Hence, 2148 is wrong term, it should be replaced
by 2112.
+7 + 9 + 11 + 13 + 15
A

56. (d) Pattern of the series is as shown below


Hence, 6 is wrong term, it should be replaced by 5.
@

238
2 7 24 77 236 2180 63. (d) The numbers are cubes of prime numbers, i.e.
723
2 3 , 33 , 53 , 7 3 , 113 . Clearly, none is wrong.
(×3)+1 (×3)+3 (×3)+5 (×3)+7 (×3)+9 (×3)+11 64. (b) Pattern of the series is as shown below
Hence, 236 is wrong term, it should be replaced by 8
238. 2 3 9 27 112 565 3396
57. (d) Pattern of the series is as shown below
55 ×1+1 ×2+2 ×3+3 ×4+4 ×5+5 ×6+6
160 118 83 65 34 20
Hence, 9 is wrong term, it should be replaced by 8.
–(7 × 6) –(7 × 5) –(7 × 4) –(7 × 3) –(7 × 2)
65. (c) Given numbers are 587, 875, 695, 767 and 867.
Hence, 65 is wrong term, it should be replaced by 587 ⇒ 5 + 8 + 7 = 20
55. 875 ⇒ 8 + 7 + 5 = 20
58. (d) Pattern of the series is as shown below 695 ⇒ 6 + 9 + 5 = 20
65 767 ⇒ 7 + 6 + 7 = 20
10 11 15 24 40 67
867 ⇒ 8 + 6 + 7 = 21
2 2 2 2 2 So, sum of digits of all numbers is 20 whereas sum
+(1) +(2) +(3) +(4) +(5)
of digits of number 867 is 21.
Hence, 67 is wrong term, it should be replaced by Hence, number 867 is different from others.
65.
How to Crack Test of Reasoning VERBAL

07
Letter Series
Letter series is a logical arrangement of letters Illustration 2. Find the next two terms of the
of English alphabet arranged in a specified given series.
pattern. Z, S, W, O, T, K, Q, G, ?, ?
In this chapter, a series of letters, groups of (a) N, C (b) N, D

A
letters or combinations of letters and numbers (c) O, C (d) O, D

M
is given. Each group or single element is Solution (a) The given series is a combination of two
called term. The terms of the series form a

H
series.
certain pattern. Candidates are required to

TT
I. Z, W, T, Q, ? and II. S, O, K, G, ?
identify this pattern and find the missing
term(s) in the given series which will satisfy Now,
the pattern. Z
–3
W A
–3
T
–3
Q
–3
N
W
–4 –4 –4 –4
S O K G C
SH

Types of Letter Series Hence, the missing letters are N and C.


A

There are mainly three types of letter series. Illustration 3. Find the missing term in the given
@

They are as follows series.


ABP, CDQ, EFR, ? CLAT 2015
Type 1 (a) GHS (b) GHT
(c) HGS (d) GHR
Alphabet Series Solution (a) The given series is a combination of
In this type, letters of English alphabet are three series.
arranged in a particular pattern like reverse I. A, C, E, ? II. B, D, F, ?
order of letters, skipping of letters, position of III. P, Q, R, ?
letters in alphabetical order etc. Now,
+2 +2 +2
Illustration 1. Find the next term in the series. A C E G
+2 +2 +2
A, D, G, J, ? B D F H
+1 +1 +1
(a) K (b) L (c) M (d) N P Q R S
Solution (c) Pattern of the series is as shown below
Hence, the missing term is GHS.
A D G J M
Illustration 4. Find the missing term in the given
+3 +3 +3 +3
series.
Every next term of the series is next third letter of AB, DEF, HIJK, ?, STUVWX
the alphabets (according to position). (a) LMNO (b) LMNOP
Hence, the missing term is M. (c) MNOPQ (d) QRSTU
Chapter 07 LETTER SERIES 73

Solution (c) Pattern of the series is as shown below Type 3


AB DEF HIJK MNOPQ STUVWX
Mixed Series
+2 +2 +2 +2
[Alpha-numeric Series]
The number of letters in the terms of the given In this type, the series is based on the
series increases by one at each step. The first letter combination of both the letters and numbers.
of each term is two steps ahead of the last letter of
Each term in the series follows a certain
the preceding term. However, each term consists of
pattern based on either the alphabetical
consecutive letters in order.
position of the letters or the numbers in
Hence, the missing term is MNOPQ. different correlation.
Illustration 7. Replace the question mark (?) in
Type 2 the following series with suitable option.
Continuous Pattern Series C-3, E-5, G-7, I-9, ?, ?
SSC (CGL) 2013, BSSC (CGL) 2015
In this type, a series of small/capital letters are
(a) M-18, K-14
given which follow a particular pattern.
(b) K-11, M-13

A
However, some letters are missing from the
(c) X-24, M-21

M
series. The series follows a specific pattern
(d) O-15, X-24
and candidates are required to find the letters

H
Solution (b) Pattern of the series is as shown below
which should come in place of the blank

TT
spaces or question marks. C-3 E-5 G-7 I-9 K - 11 M - 13

Directions (Illustrations 5-6) These questions +2 A


+2 +2 +2 +2
W
are based on the letter series. In each of these
series, some of the letters are missing. Select the In this series, every letter is given its alphabetical
SH

correct alternatives. position and one letter is skipped in between each


term.
Illustration 5. abca_bcaab_ca_bbc_a
A

Hence, the missing terms are K-11 and M-13.


(a) ccaa (b) bbaa
@

(c) abac (d) abba Illustration 8. What comes in place of question


Solution (c) Pattern of the series is as shown below mark (?) in the series given below?
P3C, R5F, T8I, V12L, ? CLAT 2015
abc / aabc / aabbc / aabbcc / a ⇒ abac
(a) Y17O
Hence, the missing letters are abac. (b) X16M
Illustration 6. _cb_ca_bacb_ca_bac_d (c) X17O
SSC (10 + 2) 2014
(d) X16O
(a) badddb (b) bbbddd (c) addddb (d) addbbb Solution (c) Pattern of the series is as shown below
Solution (c) Pattern of the series is as shown below +2 +2 +2 +2
P R T V X
acbd / cadb / acbd / cadb / acbd ⇒ addddb +2 +3 +4 +5
17
3 5 8 12
The sequence is made of repetition of two blocks +3 +3 +3 +3
‘acbd’ and ‘cadb’ alternatively. C F I L O
Hence, the missing letters are addddb. Hence, the missing term is X17O.
How to Crack Test of Reasoning VERBAL

Let us Practice
Directions (Q. Nos. 1-30) In each of the following question, various terms of an alphabet series are
given with one or more terms missing as shown by (?). Choose the missing terms out of the given
alternatives.
1. T, Q, N, K, ? SSC (10+2) 2017 17. DKY, FJW, HIU, JHS, ? SSC (10+2) 2013
(a) H (b) I (c) G (d) F (a) LFQ (b) LGQ (c) KGR (d) KFR

2. A, D, H, M, ?, ? 18. BMK, DLM, FKO, HJQ, ? DSSSB 2017


(a) P, T (b) R, X (c) S, Z (d) T, Y (a) JHS (b) JIS (c) JGK (d) JIR

3. H, V, G, T, F, R, E, P, ?, ? IB ACIO (Pre) 2017 19. RTM, QRJ, PPG, OND, ? SSC (10+2) 2017
(a) K, L (b) D, N (c) C, D (d) L, K (a) NLZ (b) MMZ
(c) MKB (d) NLA
4. X, Q, K, F, ?
20. AZY, EXW, IVU, ?

A
(a) E (b) B (c) C (d) D SSC (10+2) 2011
(a) MTS (b) MQR

M
5. A, D, E, H, I, L, ?, ? (c) NRQ (d) LST

H
(a) M, P (b) M, N (c) M, O (d) M, Q
21. DKM, FJP, HIS, JHV, ? SSC (CPO) 2013

TT
6. W, T, P, M, I, F, B, ?, ? [SSC (CPO) 2012]
(a) LGY (b) HGY (c) IGZ (d) IGY
(a) Z, V (b) X, U (c) Y, U (d) Y, V
A
22. LMN, NPP, PSR, ? SSC (10+2) (Tier-I) 2017
W
7. Z, A, U, F, P, ? UPPSC (RO) 2014 (a) QRS (b) RQT
(a) K (b) M (c) N (d) O (c) PQR (d) RVT
SH

8. CE, GI, KM, OQ, ? SSC (10 + 2) 2014 23. XBF UDK RFO OHR ? IBPS RRB (PO) Pre 2016
(a) TW (b) TV (c) SU (d) RT
A

(a) LKU (b) MKS


(c) LJT (d) MJS
@

9. TU, ?, NO, XY SSC (10+2) (Tier-I) 2017


(e) LJV
(a) IJ (b) FG
(c) DF (d) DE 24. bc, cde, de, efg, fg, ? SSC (CGL) 2013
(a) ghi (b) fgh (c) hij (d) ijk
10. LT, MS, NR, OQ, ? SSC (10 + 2) 2017
(a) LP (b) QP 25. BEH, DGJ, (?), EJO, GLQ, INS
(c) PR (d) PP IB ACIO (Pre) 2017
(a) FLR (b) FIS (c) FKO (d) FIL
11. FD, HF, KH, OJ, ? SSC (10 + 2) (Tier) 2017
(a) GH (b) EF (c) RS (d) TL 26. ?, LMN, RST, WXY SSC (10+2) (Tier-I) 2017
(a) EGF (b) DFE
12. AC, FH, KM, PR, ? SSC (Multitasking) 2014
(c) DEF (d) EFG
(a) UX (b) TV (c) UW (d) VW
27. AFI, JOR, MRU, ? SSC (CGL) 2013
13. DF, GJ, KM, NQ, RT, ? SSC (CGL) 2012 (a) GJN (b) HMP
(a) UX (b) UW (c) YZ (d) XZ
(c) PMO (d) RJL
14. BEG, DGI, FIK, HKM, ? SSC (10+2) 2018
28. ABCD, BCDA, CDAB, ?, ABCD SSC (FCI) 2011
(a) JNP (b) NMO (c) JMO (d) KLO
(a) DADC (b) DABC
15. CPK, FSN, IVQ, LYT, ? SSC (10+2) 2017 (c) DBAC (d) DACB
(a) OBY (b) PCX (c) PCY (d) OBW
29. BEAG, DGCI, FIEK, ? SSC (CGL) 2013
16. AEN, MQZ, CGP, ? SSC CGL (Tier-I) 2016 (a) HMIE (b) HKGM
(a) OSB (b) PUE (c) MPX (d) OTC (c) HGKJ (d) HKLJ
Chapter 07 LETTER SERIES 75

30. ELFA, GLHA, ILJA, ?, MLNA 37. BA_BA_BAC_ACB_CBAC IB ACIO (Pre) 2017
SSC CGL (Tier-I) 2016 (a) AACB (b) BBCA (c) CCBA (d) CBAC
(a) OLPA (b) KLMA
38. P_R_S_QRR_ SSC (10+2) (Tier-I) 2016
(c) LLMA (d) KLLA
(a) QPRS (b) QRPS (c) QRPP (d) QPSR
31. What comes in place of question mark in th e
following series? SBI (PO) 2010
39. mc_m_a_ca_ca_c_mc SSC (10+2) 2013
(a) acmmma (b) camcam
N A T U R A L, N A T U R A, (c) aaacmm (d) acmmca
A T U R A,?
(a) ATUR (b) TURA 40. LU_TUPLUBTU_LUBT_P_UBTUP
(c) RUTA (d) ARUT SSC (CPO) 2015
(e) None of these (a) BPUL (b) LBPU (c) PBUL (d) BUPL
32. Letters are arranged in a particular order 41. _ bb_c_bg_b_g SSC (10+2) 2011
with some criterion. Find out the right one to (a) cbgbc (b) cgbcb (c) cgbcc (d) gbcbb
fill in the blank.
A, CD, GHI, __ , UVWXY
42. _b_baaabb_a_ _bb_ a_ RRB (ASM) 2012
(a) abbaaba (b) ababbaa
(a) LMNO (b) NOPQ (c) KLMN (d) MNOP (c) babaaba (d) baabaab

A
33. Which of the following terms follows the 43. _ml_nl_mlm_l_mlmn_

M
SSC (CSI & ASI) 2017
trend of the given list? SSC (10+2) 2017 (a) nnmmll (b) nmlnml

H
YXXXXXX, YYXXXXX, YYYXXXX, (c) mnlnlm (d) nmnnnl

TT
YYYYXXX, YYYYYXX,__________ 44. rtx _ sx _ z _ txy _ _ yz SSC (CPO) 2014
(a) XXXXXXX (b) YXXXXXX (a) yyrxs
(c) yyrsx A (b) yysxr
(d) yyxrs
W
(c) YYYYYYX (d) YYXXXXX
45. aa_aabb_b_aa_aabb_bb
SH

34. What should come next in the following UP (B.Ed.) 2011


letter series? (a) bbbaa (b) bbbba (c) aabbb (d) babba
A

ABCDPQRSABCDEPQRSTABC 46. ZYX_W_YZZ_XWWXY_


DEFPQRST
@

(a) WXYZ (b) WYXZ (c) WXZY (d) XYZW


(a) A (b) V (c) U (d) W
(e) None of these 47. adb_ac_da_cddcb_dbc_cbda SSC (CGL) 2015
(a) bccba (b) cbbaa
35. What should come next in the following (c) ccbba (d) bbcad
letter series?
ABCDEFZYXWVUABCDEZYX
48. B_CCABB_CABBC_AB_CCA SSC (CPO) 2015
(a) BBBC (b) BBCC
WVUABCDZYXWV
(c) BCCB (d) BCBC
(a) U (b) A (c) B (d) Z
(e) None of these 49. bc_bca_ cab_ ab_a_ca SSC (10+2) 2013
(a) abcab (b) cabac (c) abccb (d) cabca
Directions (Q. Nos. 36-61) In each of the
following question, a letter series is given. Some of 50. CDEF_EDC_DE_FED_ SSC (10+2) (Tier-I) 2017
the letters of the series are missing. This missing (a) FCFC (b) FCCD (c) DCCC (d) DEFC
letters are given in same order as one of the
four/five alternatives below the series. Find out 51. ab_d_ _ b_dm_ _x_ _
the correct alternative. (a) xmaxabda (b) xmaxabdm
(c) mxaxabdm (d) mxaxbadm
36. s_nx_mnx_mn_sm_x
Delhi Police (SI and ASI) 2016 52. a_b_c a_b_c_a_c c SSC (CGL) 2015
(a) mssxn (b) smnxx (a) ababac (b) ababca
(c) nmxxn (d) nxyms (c) acacab (d) acbcab
76 How to Crack Test of Reasoning VERBAL

53. abc_ya bc_xw abc dev __ Directions (Q. Nos 66-80) What should come in
(a) zdu (b) dxu (c) cdu (d) zyw the place of question mark (?) in the following
(e) None of these alpha numeric series?
54. c_bba_cab_ac_ab_ac 66. A1, C3, F6, J10, O15, ?
(a) bcacb (b) babcc (c) abcbc (d) acbcb (a) U21 (b) V21 (c) T20 (d) U20

55. _zy_zxy_yxzx_zyx_xy SSC (Steno) 2011 67. G13T, I11R, L9O, ?


(a) yxzyz (b) zxyzy (c) yzxyx (d) xyzzy (a) P0K (b) P5L (c) P5J (d) P7K

56. a_ba_c_aad_aa_ea SSC (CGL) 2012


68. A2I, E4J, I8K, O16L, ? MPPSC 2018
(a) U32M (b) U16N (c) U18N (d) U32L
(a) babbb (b) babbd (c) babbc (d) bacde
69. XW2, TS6, PO10, LK14, ? IBPS Clerk (Pre) 2016
57. ZX_TR_NLJ_FD_ (a) HG18 (b) JI18 (c) HG16 (d) IH18
(a) VPHB (b) VRHB (c) UPJB (d) VPJB
(e) JG12
58. _ _r_ttp_ _s__tp_ _ _s_ _ _ 70. AZ-21, CY-18, FX-15, JW-12, ? IBPS (RRBs) 2016
(a) pqrssqtpqrtp (b) pqstrsqptrst
(a) PU-7 (b) OV-9 (c) PU-8 (d) NV-9
(c) pqsqrsqrrtpq (d) None of these
(e) OV-7

A
59. pqr_ _rs _ rs _ _s_q _ SSC (FCI) 2012
71. AY-12, CW-15, EU-18, GS-21,? IBPS (Clerk) 2017

M
(a) spqpprr (b) pqrrppq (a) IL-24 (b) IQ-24 (c) JQ-27 (d) JR-21

H
(c) sqppqpr (d) sqprrqr (e) JQ-24

TT
60. _A′B_B′ AA′_ BB′_A SSC (FCI) 2012 72. 1CV, 5FU, 9IT, ?, 17OR SBI PO 2011
(a) A′BAB′ (b) A′ B′AB (c) ABAB (d) AB′A′B (a) 11LS (b) 14JS (c) 15JS (d) 13LS
61. a_bc_a_bcda_ccd_bcd_ A
(e) None of these
W
(a) abddbd (b) acbdbb (c) adbbad (d) adbcad 73. 2B, 4C, 8E, 14H, ? SBI PO 2011
SH

62. In the following question find out the group (a) 22L (b) 16K (c) 20L (d) 22I
of letters in the place of question mark in the (e) None of these
A

letter series. OPSC 2018 74. C4X, F9U, I16R, ? IBPS (PO) 2012
@

H3M, I5O, L9S, Q17A, ? (a) L25P (b) K25P (c) L27P (d) L25O
(a) X33Q (b) Z33P (c) X33P (d) W33R (e) None of these
63. In the following question find out the group 75. ZW3, UR6, PM10, KH15, ? LIC (Assistant) 2013
of letters in the place of question in the letter (a) FC21 (b) FB21 (c) FC22 (d) FC20
series. OPSC 2018 (e) None of these
1/R, 3/O, 5/K, 9/F, 13/Z, ?
(a) 19/S (b) 20/T (c) 19/T (d) 21/R
76. Z9A, X7D, ?, T3J, RIM SBI (Clerk) 2013
(a) W6F (b) S3H (c) G9V (d) V5G
64. Which of the following terms follows the (e) None of these
trend of the given list? SSC (10+2) 2011
77. Q1F, S2E, U6D, W24C, ?
PRRQPQPQ, PQRRPQPQ, PQPRRQPQ,
(a) Y120B (b) Y44B (c) Z88B (d) Y66B
PQPQRRPQ, PQPQPRRQ,____________
(e) None of these
(a) RRPQPQPQ (b) PRRQPQPQ
(c) PQPQPQRR (d) PQRRPQPQ 78. 9Z-20, 11A-18, 13Y-16, 15B-14? IBPS (RRBs) 2016
(a) 14D-10 (b) 16B-12 (c) 17X-12 (d) 16X-10
65. What should come next in the following (e) 17D-10
letter series?
79. 8 H 24 X 7 G 15 O 17 ? UP Police (Constable) 2018
PQPQRPQRSPQRSTPQRSTUP
(a) T (b) R (c) Q (d) P
QRSTUVPQRST
(a) S (b) U (c) Q (d) R 80. B 2 I 9 R 18 T 20 H ? UP Police (Constable) 2018
(e) None of these (a) 6 (b) 7 (c) 8 (d) 9
Chapter 07 LETTER SERIES

Answers
1. (a) 2. (c) 3. (b) 4. (b) 5. (a) 6. (c) 7. (a) 8. (c) 9. (d) 10. (d)
11. (d) 12. (c) 13. (a) 14. (c) 15. (d) 16. (a) 17. (b) 18. (b) 19. (d) 20. (a)
21. (a) 22. (d) 23. (c) 24. (a) 25. (d) 26. (d) 27. (b) 28. (b) 29. (b) 30. (d)
31. (a) 32. (d) 33. (c) 34. (c) 35. (a) 36. (a) 37. (c) 38. (b) 39. (a) 40. (a)
41. (b) 42. (a) 43. (d) 44. (c) 45. (d) 46. (a) 47. (b) 48. (c) 49. (c) 50. (a)
51. (b) 52. (c) 53. (a) 54. (d) 55. (a) 56. (d) 57. (a) 58. (c) 59. (c) 60. (c)
61. (c) 62. (a) 63. (a) 64. (c) 65. (b) 66. (a) 67. (d) 68. (a) 69. (a) 70. (b)
71. (b) 72. (d) 73. (a) 74. (d) 75. (a) 76. (d) 77. (a) 78. (c) 79. (c) 80. (c)

Answer with Explanations

A
M
1. (a) Pattern of the series as shown below 6. (c) Pattern of the series as shown below

H
T Q N K H W T P M I F B Y U

TT
–3 –4 –3 –4 –3 –4 –3 –4
–3
Hence, the missing term is H.
–3 –3 –3
A
Hence, the missing terms are Y and U.
W
2. (c) Pattern of the series as shown below 7. (a) Pattern of the series as shown below
SH

+5 +5
A D H M S Z
Z A U F P K
A

+3 +4 +5 +6 +7
@

–5 –5
Hence, the missing terms are S and Z.
Hence, the missing term is K.
3. (b) Pattern of the series as shown below
8. (c) Pattern of the series as shown below
–1 –1 –1 –1
+4 +4 +4 +4

H V G T F R E P D N
CE GI KM OQ SU
–2 –2 –2 –2
+4 +4 +4 +4
Hence, the missing terms are D and N. Hence, the missing term is SU.
4. (b) Pattern of the series as shown below 9. (d) Pattern of the series as shown below
X Q K F B +10 +10 +10

–7 –6 –5 –4
TU DE NO XY
Hence, the missing term is B. +10 +10 +10
5. (a) Pattern of the series as shown below Hence, the missing term is DE.
A D E H I L M P
10. (d) Pattern of the series as shown below
+1 +1 +1 +1
L M N O P
–1 –1 –1 –1
+3 +1 +3 +1 +3 +1 +3 T S R Q P

Hence, the missing terms are M and P. Hence, the missing term is PP.
78 How to Crack Test of Reasoning VERBAL

11. (d) Pattern of the series as shown below 19. (d) Pattern of the series as shown below
+2 +3 +4 +5 –1 –1 –1 –1
F H K O T R Q P O N
+2 +2 +2 +2 –2 –2 –2 –2
D F H J L T R P N L
Hence, the missing term is TL. M
–3
J
–3
G
–3
D
–3
A
12. (c) Pattern of the series as shown below
A C F H K M P R U W Hence, the missing term is NLA.
20. (a) Pattern of the series as shown below
+2 +3 +2 +3 +2 +3 +2 +3 +2 +4 +4 +4
A E I M
Hence, the missing term is UW. –2 –2 –2
Z X V T
13. (a) Pattern of the series as shown below
–2 –2 –2
+3 +4 +3 +4 +3 Y W U S

DF GJ KM NQ RT UX Hence, the missing term is MTS.


21. (a) Pattern of the series as shown below
+4 +3 +4 +3 +4
+2 +2 +2 +2
Hence, the missing term is UX. D F H J L
–1 –1 –1 –1
14. (c) Pattern of the series is as shown below. K J I H G
+3 +3

A
+2 +2 +2 +2 +3 +3
B D F H J M P S V Y

M
+2 +2 +2 +2
E G I K M Hence, the missing term is LGY.

H
+2 +2 +2 +2
G I K M O 22. (d) Pattern of the series as shown below

TT
Hence, the missing term is JMO. +2 +2 +2
L N P R
15. (d) Pattern of the series as shown below +3 +3 +3

C
+3
F
+3
I
+3
L
+3
O
M
+2
P
A +2
S
+2
V
W
N P R T
+3 +3 +3 +3
P S V Y B
SH

+3 +3 +3 +3
Hence, the missing term is RVT.
K N Q T W
23. (c) Pattern of the series as shown below
A

Hence, the missing term is OBW. X B F U DK R F O OH R L J T


@

16. (a) Pattern of the series as shown below –3 –3 –3 –3


+2 +2 +2 +2 +2
+2 +5 +4 +3 +2
+2
Hence, the missing term is LJT.
A E N M Q Z C G P O S B 24. (a) Pattern of the series as shown below
+2
+2
b c, cd e, d e, e f g, f g, ghi
+2

Hence, the missing term is OSB.


Hence, the missing term is ghi.
17. (b) Pattern of the series as shown below
+2 +2 +2 +2 25. (d) There are two series in the question,
D F H J L
+2 +2 +2 +2
–1 –1 –1 –1
B D F E G I
K J I H G +2 +2 +2 +2
E G I and J L N
–2 –2 –2 –2 +2 +2 +2 +2
Y W U S Q H J L O Q S
Hence, the missing term is LGQ. Hence, the missing term is FIL.
18. (b) Pattern of the series as shown below 26. (d) Pattern of the series as shown below
+2 +2 +2 +2 +5 +4 +3
B D F H J
–1 –1 –1 –1
M L K J I E F G L M N R S T W X Y
+2 +2 +2 +2
K M O Q S
+1 +1 +1 +1 +1 +1 +1 +1

Hence, the missing term is JIS. Hence, the missing term is EFG.
Chapter 07 LETTER SERIES 79

27. (b) Pattern of the series as shown below 34. (c) Pattern of the series as shown below
A F I J O R ABCD/PQRS/ABCDE/PQRST/ABCDEF/ PQRST
U
+5 +3 +5 +3
Hence, the missing term is U.
M R U H M P
35. (a) Pattern of the series as shown below
+5 +3 +5 +3 ABCDEF/ZYXWVU/ABCDE/ZYXWVU/ABCD/
ZYXWV U
Hence, the missing term is HMP.
Hence, the missing term is U.
28. (b) Pattern of the series as shown below
36. (a) Pattern of the series as shown below
iftin iftin iftin iftin
Sh g Sh g Sh g Sh g smnx/smnx/smnx/smnx ⇒ mssxn
ABCD BCDA CDAB DABC Hence, the missing letters are mssxn.
ABCD 37. (c) Pattern of the series as shown below
Hence, the missing term is DABC. BAC/BAC/BAC/BAC/BAC/BAC ⇒ CCBA
29. (b) Pattern of the series as shown below Hence, the missing letters are CCBA.
B
+2
D
+2
F
+2
H 38. (b) Pattern of the series as shown below
+2 +2 +2 PQRRS/PQRRS ⇒ QRPS
E G I K

A
Hence, the missing letters are QRPS.
+2 +2 +2
A C E G 39. (a) Pattern of the series as shown below

M
+2 +2 +2 m c a/ m c a/ m c a/ m c a/m c a/ mc
G I K M

H
⇒ acmmma

TT
Hence, the missing term is HKGM. Hence, the missing letters are acmmma.
30. (d) Pattern of the series as shown below 40. (a) Pattern of the series as shown below
E
+2
G
+2
I
+2
K
+2
M A
LUB/TUP/LUB/TUP/LUB/TUP/LUB/TUP
W
⇒ BPUL
L L L L L
SH

The series is made of repetition of patterns


+2 +2 +2 +2 ‘LUB’ and ‘TUP’ alternatively.
F H J L N
Hence, the missing letters are BPUL.
A

A A A A A
41. (b) Pattern of the series as shown below
@

Hence, the missing term is KLLA. cbbg/cbbg/cbbg ⇒ cgbcb


31. (a) Last and first letters get disappear alternately. Hence, the missing letters are cgbcb.
Hence, the missing term is ATUR. 42. (a) Pattern of the series as shown below
32. (d) Pattern of the series as shown below abb/baa/abb/baa/abb/baa ⇒ abbaaba
A, CD, GHI, MNOP UVWXY The series is repetition of alternative two blocks
‘abb’ and ‘baa’.
+2 +3 +4 +5 Hence, the missing letters are abbaaba.
The number of letters in the terms of the series 43. (d) Pattern of the series as shown below
increases by one at each step. nml/mnl/nml/mnl/nml/mnl ⇒ nmnnnl.
Each term consists of consecutive letters in Hence, the missing letters are nmnnnl.
order.
44. (c) Pattern of the series as shown below
Hence, the missing term is MNOP.
rtxy/sxyz/rtxy/s xyz ⇒ yyrsx
33. (c) Pattern of the series as shown below
Hence, the missing letters are yyrsx.
Y XXXXXX YY XXXXX YYY XXXX YYYY XXX
45. (d) Pattern of the series as shown below
1 6 2 5 3 4 4 3 aabaa/bbabb/aabaa/bbabb ⇒ babba
YYYYY XX YYYYYY X The sequence is made of two patterns ‘aabaa’
5 2 6 1 and ‘bbabb’.
Hence, the missing term is YYYYYYX. Hence, the missing letters are babba.
80 How to Crack Test of Reasoning VERBAL

46. (a) Pattern of the series as shown below 58. (c) Pattern of the series as shown below
ZYXW/WXYZ/ZYXW/WXYZ ⇒ WXYZ The last letter of each previous block is moved
Hence, the missing letters are WXYZ. to the beginning of the successive block.
Now, pqrst/tpqrs/stpqr/rstpq ⇒ pqsqrsqrrtpq
47. (b) Pattern of the series as shown below
Hence, the missing letters are pqsqrsqrrtpq.
adbc/acbd/abcd/dcba/dbca/cbda ⇒ cbbaa 59. (c) Pattern of the series as shown below
Hence, the missing letters are cbbaa. pqrs/qrsp/rspq/spq r ⇒ sqppqpr
48. (c) Pattern of the series as shown below Hence, the missing letters are sqppqpr.
BBCCA/BBCCA/BBCCA/BBCCA ⇒ BCCB 60. (c) Pattern of the series as shown below
Hence, the missing letters are BCCB AA′B/ BB′A/A′AB/ B′BA ⇒ ABAB.
49. (c) Pattern of the series as shown below Hence, the missing letters are ABAB.
bca/bca/bca/bca/bca/bca ⇒ abccb 61. (c) Pattern of the series as shown below
bc/abc/abc/abc/abc/abc/a ⇒ abccb aabcd/abbcd/abccd/abcdd ⇒ adbbad
Hence, the missing letters are abccb. Hence, the missing letters are adbbad.
62. (a) Pattern of the series as shown below
50. (a) Pattern of the series as shown below
+1 +3 +5 +7
CDEF/FEDC/CDEF/FEDC ⇒ FCFC H I L Q X

A
+2 +4 +8 +16
Hence, the missing letters are FCFC 3 5 9 17 33

M
51. (b) Pattern of the series as shown below +2 +4 +8 +16
M O S A Q

H
Here, repeated pattern is ‘abxdm’.

TT
Now, abxdm /abxdm/abxdm ⇒ xmaxabdm Hence, the missing term is X33Q.
63. (a) Pattern of the series as shown below
Hence, the missing letters are xmaxabdm.
1
+2
3
+2 A 5
+4
9
+4
13
+6
19
W
52. (c) Pattern of the series as shown below –3 –4 –5 –6 –7
R O K F Z S
aabcc/aabcc/aabcc ⇒ acacab
SH

Hence, the missing letters are acacab. Hence, the missing term is 19S.
53. (a) abczy/abcdxw/abcdevu 64. (c) Pattern of the series as shown below
A

Required answer = zdu PRRQPQPQ


@

54. (d) Pattern of the series as shown below PQ R R P Q P Q


cab/bac/cab/bac/cab/bac ⇒ acbcb
The sequence is made of repetition of two PQPRRQPQ
patterns cab and bac alternatively
PQPQRRPQ
Hence, the missing letters are acbcb.
55. (a) Pattern of the series as shown below PQPQPRRQ
y zy/xzx/yzy/xzx/yzy/xzx/y ⇒ yxzyz.
The sequence is made of repetition of patterns PQPQPQRR
‘yzy’ and ‘xzx’ alternately. Hence, the missing term is PQPQPQRR.
Hence, the missing letters are yxzyz. 65. (b) Pattern of the series as shown below
56. (d) Pattern of the series as shown below PQ/PQR/PQRS/PQRST/PQRSTU/PQRSTUV/PQ
abba/acca/adda/aeea ⇒ bacde RST U
Hence, the missing letters are bacde. Hence, the missing term is U.
57. (a) Pattern of the series as shown below 66. (a) Pattern of the series as shown below
ZXVTRPNLJHFDB ⇒ VPHB +2 +3 +4 +5 +6
A1 C3 F6 J 10 O15 U 21
The sequence is made of (–2) letters,
respectively. +2 +3 +4 +5 +6
Hence, the missing letters are VPHB. Hence, the missing term is U21.
Chapter 07 LETTER SERIES 81

67. (d) Pattern of the series as shown below 74. (d) Pattern of the series as shown below
+2 +3 +4 +3 +3 +3
G I L P C F I L
–2 –2 –2 +5 +7 +9
13 11 9 7 4 9 16 25
–2 –3 –4 K –3 –3 –3
T R O X U R O

Hence, the missing term is P7K. Hence, the missing term is L25O.
68. (a) Pattern of the series as shown below 75. (a) Pattern of the series as shown below
First letter of each term is a series of consecutive Z
–5
U
–5
P
–5
K
–5
F
vowels, numbers are moved as 2,2 2 , 2 3 , 2 4 , 2 5 –5 –5 –5 –5
W R M H C
and third letter of each term is increasing
consecutively. +3 +4 +5 +6 21
3 6 10 15
Hence, the missing term is U32M.
Hence, the missing term is FC21.
69. (a) Pattern of the series as shown below
–4 –4 –4 –4
76. (d) Pattern of the series as shown below
X T P L H –2 –2 –2 –2
–4 –4 –4 –4 Z X V T R
W S O K G –2 –2 –2 –2
9 7 5 3 1

A
+4 +4 +4 +4 18
2 6 10 14 +3 +3 +3 +3 M
A D G J

M
Hence, the missing term is HG18

H
70. (b) Pattern of the series as shown below Hence, the missing term is V5G.

TT
+2 +3 +4 +5 77. (a) Pattern of the series as shown below
A C F J O
–1 –1 –1 –1 +2 +2 +2 +2
Z Y X W V Q S U W Y
–3 –3 –3 –3 1
×2
A
2
×3
6
×4
24
×5
120
W
21 18 15 12 9
–1 –1 –1 –1 B
Hence, the missing term is OV9. F E D C
SH

71. (b) Pattern of the series as shown below Hence, the missing term is Y120B.
A

+2 +2
E
+2 +2 78. (c) Pattern of the series as shown below.
A C G I
@

–2 –2 –2 –2 +2 +2 +2 +2
Y W U S Q 9 11 13 15 17
+1 –2 +3 –4
+3 +3 +3 +3 24 Z A Y B X
12 15 18 21
–2 –2 –2 –2 12
20 18 16 14
Hence, the missing term is IQ24.
72. (d) Pattern of the series as shown below Hence, the missing term is 17X12.
+4 +4 +4 +4 79. (c) Here, the numbers are followed by letters at that
1 5 9 13 17
particular place in English alphabet.
+3 +3 +3 +3
C F I L O X 7 G 15 O 17 Q
8 H 24
–1 –1 –1 –1 R
V U T S
Hence, the missing term is Q.
Hence, the missing term is 13LS.
80. (c) Here, the letters are followed by numbers, which
73. (a) Pattern of the series as shown below denotes its positional value in English alphabet.
+2 +4 +6 +8
2 4 8 14 22 B 2 I 9 R 18 T 20 H 8
+1 +2 +3 +4
B C E H L

Hence, the missing term is 22L. Hence, the missing term is 8.


How to Crack Test of Reasoning VERBAL
82

08
Logical Order
of Words

A
M
‘Logical order of words’ is basically the Illustration 1.

H
arrangement of words according to a certain 1. Frog 2. Eagle

TT
order which can be their size, occurrence, 3. Grasshopper 4. Snake
dictionary order etc. 5. Grass SSC (CGL) 2015
In these types of questions, four/five/six words (a) 3, 4, 2, 5, 1 A (b) 1, 3, 5, 2, 4
W
are given and the candidate is required to (c) 5, 3, 1, 4, 2 (d) 5, 3, 4, 2, 1
Solution (c) From the given words, it is deduced that
SH

arrange the given words either in a logical


sequence or in an order according to grass is eaten by grasshopper, grasshopper is eaten
dictionary. by frog, frog is eaten by snake and finally, eagle
A

eats snake. So, the correct logical arrangement of


@

words is 5, 3, 1, 4, 2.
Types of Questions
Illustration 2.
In this chapter, there are two types of 1. Country 2. Furniture
questions, generally asked in various 3. Forest 4. Wood
competitive exams. 5. Trees
(a) 1, 3, 5, 4, 2 (b) 1, 4, 3, 2, 5
Type 1 (c) 2, 4, 3, 1, 5 (d) 5, 2, 3, 1, 4
Solution (a) From the given words, it is deduced that
Arrangement According to Logic a country contains forests, a forest has trees, trees
give wood that is used to make furniture.
As the name implies, in this type of questions, So, the correct logical arrangement of words is
a sequence is formed with a certain number of 1, 3, 5, 4, 2.
words given in such a way that it gives a
logical step-by-step completion of the process Illustration 3.
or the activity described. 1. Pupa 2. Larva
3. Moth 4. Eggs SSC (CGL) 2013
Directions (Illustrations 1-4) In each of the (a) 4, 2, 1, 3 (b) 4, 1, 2, 3 (c) 4, 3, 2, 1 (d) 4, 3, 1, 2
following question, arrange the given words in a Solution (a) The given words represent the life cycle
logical and meaningful order. of a moth.
Chapter 08 LOGICAL ORDER OF WORDS
83

Stage 1. The Egg – Embryonic stage For this arrangement, first we shall take the
Stage 2. Larva – larval stage first letter of each word and then arrange the
Stage 3. Pupa – Pupal stage words in the order in which they appear in the
English alphabet, then take the second letter
Stage 4. Adult moth – Imaginal stage
and so on.
∴The correct sequence is eggs → Larva → Pupa →
Moth ⇒ 4, 2, 1, 3 Illustration 5. Arrange the following words
according to dictionary.
Illustration 4. 1. Fenestration 2. Feather
1. Travel 2. Destination 3. Feed head 4. Feature
3. Payment 4. Berth/Seat number 5. Feminine SSC (CPO) 2015
(a) 4, 2, 3, 5, 1
5. Reservation (b) 2, 4, 1, 5, 3
6. Availability of Berth/Seat for reservation (c) 2, 4, 3, 5, 1
(a) 6, 2, 5, 4, 3, 1 (b) 5, 3, 4, 6, 1, 2 (d) 4, 2, 3, 1, 5
(c) 2, 6, 3, 5, 4, 1 (d) 1, 2, 5, 4, 3, 6
Solution (c) Sequence of given words as per
Solution (c) From the given words, it is clear that in dictionary is as follows
order to perform a journey, first, a destination is to
Feather → Feature → Feed head → Feminine
be identified, secondly availability of berth/seat is
→ Fenestration, i.e. 2, 4, 3, 5, 1

A
ascertained, followed by payment for reservation.

M
As a result, berth is allotted and traveling is
completed. Illustration 6. If the following five words are

H
arranged in alphabetical order, then which

TT
word will come in the middle?
Type 2 (a) Electric (b) Elector

Arrangement According
(c) Electrode
A (d) Elect
W
(e) Electron
to Dictionary Solution (a) Arranging the words in alphabetical
SH

order we have
Arranging words in alphabetical order implies
‘to arrange them in the order as they appear Elect, Elector, Electric, Electrode, Electron.
A

in a dictionary’. Hence, Electric will come in the middle.


@

Let us Practice
Directions (Q. Nos.1-21) In each of the following question, arrange the words in a meaningful, logical
order and then select the appropriate sequence from the alternatives given below for each of the groups of
words.
1. 1. Adulthood 2. Infancy 3. 1. District 2. Village
3. Childhood 4. Adolescence 3. State 4. Block
SSC (10+2) 2014 (a) 2, 1, 4, 3 (b) 2, 3, 4, 1
(a) 1, 3, 4, 2 (b) 2, 3, 4, 1 (c) 2, 4, 1, 3 (d) 3, 2, 1, 4
(c) 2, 4, 3, 1 (d) 1, 2, 3, 4
4. 1. Farmer 2. Seed
2. 1. Weaving 2. Cotton 3. Food 4. Cultivation
3. Cloth 4. Thread SSC (10+2) 2013
(a) 2, 4, 1, 3 (b) 2, 4, 3, 1 (a) 1, 2, 4, 3 (b) 2, 1, 3, 4
(c) 4, 2, 1, 3 (d) 3, 1, 4, 2 (c) 4, 2, 3, 1 (d) 3, 1, 4, 2
How to Crack Test of Reasoning VERBAL
84

5. 1. Lung 2. Nostrils 14. 1. Vegetable 2. Market


3. Windpipe 4. Blood 3. Cutting 4. Cooking
SSC (Steno) 2011 5. Food SSC (10+2) 2011
(a) 1, 2, 3, 4 (b) 2, 3, 1, 4 (a) 1, 2, 3, 4, 5 (b) 2, 1, 3, 4, 5
(c) 1, 3, 4, 2 (d) 4, 3, 2, 1 (c) 3, 1, 2, 5, 4 (d) 5, 2, 1, 3, 4

6. 1. Sending 2. Encoding 15. 1. Site 2. Plan


3. Receiving 4. Decoding 3. Rent 4. Money
SSC (10+2) 2013 5. Building
(a) 2, 4, 3,1 (b) 4, 2, 1, 3 (a) 4, 1, 2, 5, 3 (b) 3, 4, 2, 5, 1
(c) 1, 2, 3, 4 (d) 2, 1, 3, 4 (c) 2, 3, 5, 1, 4 (d) 1, 2, 3, 5, 4
7. 1. Twilight 2. Dawn 16. 1. Compose 2. Appreciation
3. Noon 4. Night 3. Money 4. Think
SSC (10+2) 2013 5. Sing
(a) 2, 1, 3, 4 (b) 2, 3, 1, 4
(a) 1, 4, 2, 5, 3 (b) 4, 1, 5, 2, 3
(c) 1, 2, 3, 4 (d) 1, 3, 2, 4
(c) 2, 4, 3, 5, 1 (d) 5, 4, 2, 1, 3
8. 1. Mother 2. Child (e) 3, 2, 4, 1, 5
3. Milk 4. Cry

A
17. 1. Collector 2. Governor
5. Smile SSC (Multitasking) 2014

M
3. Chief Secretary 4. President
(a) 1, 5, 2, 4, 3 (b) 2, 4, 1, 3, 5
5. Clerk

H
(c) 2, 4, 3, 1, 5 (d) 3, 2, 1, 5, 4
(a) 1, 2, 3, 4, 5 (b) 5, 1, 3, 2, 4

TT
9. 1. Destination 2. Booking (c) 5, 1, 3, 4, 2 (d) 5, 1, 4, 3, 2
3. Boarding 4. Travel
5. Planning
18. 1. South America
A 2. Africa
W
3. Europe 4. Australia
(a) 1, 2, 3, 4, 5 (b) 3, 4, 5, 1, 2
5. North America
SH

(c) 4, 3, 1, 2, 5 (d) 5, 2, 3, 4, 1
(a) 2, 1, 5, 3, 4 (b) 2, 5, 1, 3, 4
10. 1. Weekly 2. Daily (c) 2, 5, 1, 4, 3 (d) 2, 1, 5, 4, 3
A

3. Monthly 4. Fortnightly
19 1. Curd 2. Milk
@

5. Bimonthly
3. Butter milk 4. Cow
(a) 1, 4, 3, 2, 5 (b) 2, 1, 4, 3, 5
(c) 4, 1, 2, 3, 5 (d) 5, 1, 2, 3, 4
5. Ghee 6. Butter
SSC (10+2) 2014
11. 1. Shoulder 2. Wrist (a) 2, 5, 6, 4, 1, 3 (b) 4, 6, 2, 1, 3, 5
3. Elbow 4. Palm (c) 4, 2, 1, 3, 6, 5 (d) 2, 6, 4, 5, 3, 1
5. Finger 20. 1. Seed 2. Plant
(a) 5, 4, 2, 3, 1 (b) 3, 4, 5, 2, 1
3. Germination 4. Embryo
(c) 3, 1, 4, 2, 5 (d) 2, 4, 5, 3, 1
5. Flower 6. Fruit SSC (CGL) 2014
12. 1. House 2. Street (a) 1, 5, 3, 4, 2, 6 (b) 1, 4, 3, 2, 5, 6
3. Room 4. Town (c) 6, 5, 4, 3, 1, 2 (d) 1, 5, 6, 4, 2, 3
5. District SSC (Steno) 2011
(a) 3, 2, 1, 4, 5 (b) 3, 1, 4, 2, 5 21. 1. Orange 2. Indigo
(c) 3, 1, 2, 4, 5 (d) 3, 1, 2, 5, 4 3. Red 4. Blue
5. Green 6. Yellow
13. 1. Consultation 2. Illness 7. Violet SSC (CPO) 2014
3. Doctor 4. Treatment (a) 7, 2, 4, 5, 6, 1, 3
5. Recovery (b) 7, 2, 4, 6, 5, 1, 3
(a) 2, 3, 1, 4, 5 (b) 2, 3, 4, 1, 5 (c) 7, 2, 6, 4, 5, 1, 3
(c) 4, 3, 1, 2, 5 (d) 5, 1, 4, 3, 2 (d) 7, 2, 6, 4, 1, 5, 3
Chapter 08 LOGICAL ORDER OF WORDS
85

Directions (Q. Nos. 22-31) In the following 28. 1. Drink 2. Drinking


questions, arrange the given words as per order in 3. Drive 4. Dictionary
the dictionary. 5. Dracula SSC (CPO) 2017
22. 1. Regulate 2. Remedy (a) 4, 5, 1, 2, 3 (b) 4, 1, 5, 3, 2
3. Reassemble 4. Research (c) 4, 2, 5, 1, 3 (d) 4, 5, 2, 1, 3
SSC (10+2) 2017 29. 1. Mobile 2. Mandate
(a) 3, 1, 2, 4 (b) 1, 2, 3, 4
3. Mandarin 4. Monkey
(c) 4, 2, 1, 3 (d) 1, 3, 4, 2
5. Master SSC (CPO) 2017
23. 1. Joke 2. Jockey
(a) 3, 2, 5, 1, 4 (b) 3, 1, 2, 5, 4
3. Jocular 4. Jocund (c) 3, 5, 2, 4, 1 (d) 3, 5, 2, 1, 4
SSC (10+2) 2017
(a) 4, 2, 1, 3 (b) 2, 3, 4, 1 30. 1. Consume 2. Consciousness
(c) 2, 3, 1, 4 (d) 4, 2, 3, 1 3. Conscience 4. Conservation
5. Consequence SSC (CGL) 2015
24. 1. Aqueous 2. Aquarium (a) 3, 2, 5, 4, 1 (b) 3, 1, 2, 5, 4
3. Aquiline 4. Aquatic (c) 3, 5, 2, 4, 1 (d) 3, 2, 1, 5, 4
(a) 4, 3, 2, 1 (b) 1, 2, 3, 4
31. 1. Convince 2. Converge

A
(c) 2, 4, 1, 3 (d) 3, 1, 4, 2
3. Convenience 4. Convalesce

M
25. 1. Directional 2. Directory 5. Converse SSC (CGL) 2015

H
3. Directive 4. Direction (a) 1, 4, 3, 2, 5 (b) 4, 3, 2, 1, 5

TT
Delhi Police (ASI) 2017 (c) 4, 3, 2, 5, 1 (d) 4, 2, 5, 3, 1
(a) 1, 3, 4, 2 (b) 3, 4, 2, 1
32. If the given words are arranged according to
(c) 3, 4, 1, 2 (d) 4, 1, 3, 2
A
English dictionary, which word will be in
W
26. 1. Infect 2. Inferior third place? SSC (CGL) 2016
3. Indulge 4. Induce
SH

(a) KNOW (b) KNACK


5. Indirect SSC (10+2) 2018 (c) KNIT (d) KNOB
(a) 5, 2, 1, 4, 3 (b) 1, 2, 4, 5, 3
A

(c) 5, 4, 3, 1, 2 (d) 1, 4, 5, 3, 2
33. The following words SHRISHRI, SRISRI,
@

SHRISRI, SRISHRI, SHREESHREE are


27. 1. Exception 2. Exceptional arranged according to dictionary rule. Which
3. Exchanging 4. Exchanged of them will be in the end? CGPSC (Pre) 2014
(a) SRISHRI (b) SRISRI
5. Excess SSC (10+2) 2018
(c) SHRISHRI (d) SHRISHRI
(a) 1, 2, 5, 4, 3 (b) 2, 1, 4, 5, 3
(e) None of these
(c) 4, 2, 5, 3, 1 (d) 3, 2, 1, 5, 4

Answers
1. (b) 2. (a) 3. (c) 4. (a) 5. (b) 6. (d) 7. (b) 8. (b) 9. (d) 10. (b)
11. (a) 12. (c) 13. (a) 14. (b) 15. (a) 16. (b) 17. (b) 18. (b) 19. (c) 20. (b)
21. (a) 22. (a) 23. (b) 24. (c) 25. (d) 26. (c) 27. (a) 28. (a) 29. (a) 30. (a)
31. (c) 32. (d) 33. (b)
How to Crack Test of Reasoning VERBAL
86

Answer with Explanations


1. (b) The correct logical order is as given below 21. (a) The correct logical order is as given below.
Infancy → Childhood → Adolescence → Adulthood Here, the colours of rainbow are
2. (a) The correct logical order is as given below Violet → Indigo → Blue → Green → Yellow →
Cotton → Thread → Weaving → Cloth Orange → Red
3. (c) The correct logical order is as given below 22. (a) According to English dictionary, the correct
Village → Block → District → State sequential order is as given below
4. (a) The correct logical order is as given below Reassemble → Regulate → Remedy → Research
Farmer → Seed → Cultivation → Food 23. (b) According to English dictionary, the correct
sequential order is as given below
5. (b) The correct logical order is as given below
Nostrils → Windpipe → Lung → Blood Jockey → Jocular → Jocund → Joke
24. (c) According to English dictionary, the correct
6. (d) The correct logical order is as given below
sequential order is as given below
Encoding → Sending → Receiving → Decoding
Aquarium → Aquatic → Aqueous → Aquiline
7. (b) The correct logical order is as given below
Dawn → Noon → Twilight → Night 25. (d) According to English dictionary, the correct
sequential order is as given below
8. (b) The correct logical order is as given below

A
Direction → Directional → Directive → Directory
Child → Cry → Mother → Milk → Smile

M
26. (c) According to English dictionary, the correct
9. (d) The correct logical order is as given below sequential order is as given below Indirect → Induce

H
Planning → Booking → Boarding → Travel → Indulge → Infect → Inferior
→ Destination

TT
27. (a) According to English dictionary, the correct
10. (b) The correct logical order is as given below sequential order is as given below
Daily → Weekly → Fortnightly → Monthly
→ Bimonthly A
Exception → Exceptional → Excess → Exchanged
W
→ Exchanging
11. (a) The correct logical order is as given below 28. (a) According to English dictionary, the correct
SH

Finger → Palm → Wrist → Elbow → Shoulder sequential order is as given below


12. (c) The correct logical order is as given below Dictionary → Dracula → Drink → Drinking → Drive
A

Room → House → Street → Town → District 29. (a) According to English dictionary, the correct
@

13. (a) The correct logical order is as given below sequential order is as given below
Illness → Doctor → Consultation → Treatment Mandarin → Mandate → Master → Mobile →
→ Recovery Monkey
14. (b) The correct logical order is as given below 30. (a) According to English dictionary, the correct
Market → Vegetable → Cutting → Cooking → Food sequential order is as given below
15. (a) The correct logical order is as given below Conscience → Consciousness → Consequence →
Money → Site → Plan → Building → Rent Conservation → Consume
16. (b) The correct logical order is as given below 31. (c) According to English dictionary, the correct
Think → Compose → Sing → Appreciation sequential order is as given below
→ Money Convalesce → Convenience → Converge →
17. (b) The correct logical order is as given below Converse → Convince
Clerk → Collector → Chief Secretary → Governor 32. (d) According to English dictionary, the correct
→ President sequential order is as given below
18. (b) The continents arranged in descending order of KNACK → KNIT → KNOB → KNOW
area: Africa > North America > South America
It is clear from above that KNOB is on the third
> Europe > Australia
place.
19. (c) The correct logical order is as given below
33. (b) According to English dictionary, the correct
Cow → Milk → Curd → Butter Milk → Butter →
sequential order is as given below
Ghee
SHREESHREE → SHRISHRI → SHRISRI →
20. (b) The correct logical order is as given below SRISHRI → SRISRI
Seed → Embryo → Germination → Plant → Flower
It is clear that SRISRI will come in the end.
→ Fruit
Chapter 09 INSERTING THE MISSING CHARACTER 87

09

Inserting the
Missing Character

A
M
In this chapter, we deal with questions which Illustration 2. ? 13 49

H
have a set of numbers or letters or both
following a definite pattern represented 9 17 69

TT
pictorially. These figures could be any 13 11 59
geometrical shapes like circle, triangle,
matrices, etc. The pattern can be based on (a) 9 A
(b) 5 (c) 10 (d) 21
W
certain Mathematical operations, positional Solution (b) Here, the pattern is as follows
SH

values of the letters in English alphabetical Number in third column


order etc. The candidate is required to find = 2 × Number in first column
+ 3 × Number in second column
A

this pattern and accordingly find the missing


character in the figure. So, (9 × 2) + (17 × 3) = 69
@

Examples given below, will give you a better and (13 × 2) + (11 × 3) = 59
idea about the types of questions asked in the Hence, (? × 2) + (13 × 3) = 49
various competitive exams. ⇒ ? × 2 = 49 − 39 = 10
10
Directions (Illustrations 1-7) Find the missing ∴ ?= = 5
character in the following figures. 2
Illustration 3. 4 9 9 ?
Illustration 1. 6 8 ? 123 232
1 4
(a) 2 (b) 6 (c) 4 (d) 16
25 17 38 18 89 16 Solution (c) The numbers along the sides of triangles
SSC (CGL) 2016; SSC (CPO) 2017 are the squares of digits of numbers at the centre.
(a) 13 (b) 15 As, 12 = 1
(c) 17 (d) 19 123 22 = 4
42
Solution (b) In figure I, 25 + 17 = 42 ⇒ =6 32 = 9
7 Similarly, 22 = 4
56
In figure II, 38 + 18 = 56 ⇒ =8 232 32 = 9
7
22 = 4
105
Similarly, in figure III, 89 + 16 = 105 ⇒ = 15 Hence, ?= 4
7
88 How to Crack Test of Reasoning VERBAL

Illustration 4. Illustration 6.
260 132
16 3 20 5 18 5
? 68
SSC (CPO) 2014 1 8 ?
20 36 8 4 4 3 6 4

(a) 12 (b) 10 SSC (CPO) 2014


(c) 9 (d) 8 (a) 3 (b) 10 (c) 15 (d) 60
Solution (a) Starting from question mark (?) moving in Solution (a) In figure I,
anti-clockwise direction
16 − (3 + 4 + 8)
? 20 36 68 132 260 = 16 − 15 = 1
In figure II, 20 − (5 + 3 + 4)
16 32 64 128
= 20 − 12 = 8
Similarly, in figure III,
×2 ×2 ×2
18 − (5 + 4 + 6)
So, the difference between first and second number = 18 − 15 = 3
16
=8

A
must be Illustration 7.
2 C 18 B 21

M
i.e. ? + 8 = 20 D15 A 24

H
⇒ ? = 20 − 8 ? H3

TT
⇒ ? = 12 F9 G6
Hence, the missing number is 12.
(a) E31 A
(b) E12 (c) F12 (d) H 8
W
Illustration 5. Solution (b) Alphabet appears in anti-clockwise
P M Q
SH

direction and the numerals are the multiple of


F C F 3 appear in clockwise direction.
SSC (10+2) 2017
Hence, the missing character is E12.
A

J J ?
@

(a) R
Illustration 8. Which one of the given options fits
(b) K correctly in the blank space so that the pattern
(c) L is maintained?
(d) O 3 4 5
Solution (b) P (16) M (13) Q (17) 25 2 49 3 ? 3
F (6) C (3) F (6) (a) 49 (b) 64 (c) 81 (d) 100
J (10) J (10) ? Solution (b) The pattern is as follows
As, 16 − 6 = 10 and 13 − 3 = 10 a ⇒ ( a + c) 2 = b
(columnwise) b c
Similarly, 17 − 6 = 11 As, (3 + 2) 2 = 25 and ( 4 + 3) 2 = 49
And, 11 is the positional value of K. So, missing number = (5 + 3) 2 = 8 2 = 64
Chapter 09 INSERTING THE MISSING CHARACTER

Let us Practice
Directions (Q. Nos. 1-17) Find the missing 6. A
character in each of the following question. I 4 7 E

1. 5 1 64 3
2 4 3 9 1 7 ? I
11 ? 216
8 IGNOU 2012
8 2
A O
20 90 ? U
WBPSC 2018 (a) U and 28 (b) I and 26
(a) 160 (b) 100 (c) 50 (d) 75 (c) E and 27 (d) A and 25

2. 7. 4 9 19

A
5 2 1 7 3 2 13 ? 5
64 36 81

M
121
21 ? 6 6 11

H
49 100 MPPSC 2018

TT
SSC (CGL) 2015 (a) 4 (b) 6 (c) 8 (d) 10
(a) 40 (b) 30 (c) 20 (d) 10

3.
8. 4 2 A 9 4 6 20
W
41 ? 72 720 ?
SH

21 16 3 3 10 2 1 6
SSC (CPO) 2011
A

35 40 (a) 72 (b) 720


@

(c) 7200 (d) 38


14 42
9.
MPPSC 2018 D M V
(a) 12 (b) 13 (c) 14 (d) 15 F P Z

4. 16 25 9 H S ?
CGPSC 2013
36 64 81
10 13 ? (a) D (b) C (c) I (d) G
SSC (CGL) 2013
(e) K
(a) 14 (b) 11 (c) 12 (d) 13 10. 27 9 35 7 36 4

5. 86 480 38 2 3 4
72 510 21
36 ? 12 1 2 ?
SSC (CGL) 2016
SSC (Multitasking) 2013
(a) 120 (b) 360 (a) 3 (b) 4
(c) 100 (d) 240 (c) 5 (d) 6
90 How to Crack Test of Reasoning VERBAL

11. 6 5 3 10 18. Consider the following matrix.


2 8 ? 4 3 370 7
2 224 6
4 6 3 8
1 730 X
5 9 15 3
Delhi Police (SI & ASI) 2016
What is the number at ‘X’ in the above matrix?
(a) 5 (b) 4 (c) 6 (d) 7 UP (Lower Subordinate) 2016
(a) 5 (b) 8
12. (c) 9 (d) 11
5
222 12 19. Examine the following three figures in which
the numbers follow a specific pattern.
110 26
84 81 88
?
SSC (Multitasking) 2013
14 12 18 9 ? 11
(a) 54 (b) 51 (c) 48 (d) 44

13. 24 20 37 The missing number (?) in the third figure

A
above is UPSC (CSAT) 2013

M
31 25 ? (a) 7 (b) 16
(c) 21 (d) 28

H
26 36 19 MPPSC 2018

TT
Directions (Q. Nos. 20-28) Numbers/Letters in
(a) 23 (b) 26 (c) 30 (d) 25
following questions have been arranged according
14. 11 12 13 A
to an identical pattern. Find out the missing
W
14 15 16 numbers.
2 3 4 20.
SH

39 57 ? 92 70 48
SSC (Steno) 2013
64 53 42
A

(a) 87 (b) 75 (c) 85 (d) 77


SSC (Multitasking) 2014
@

52 45 ?
15. 5 ?
(a) 36 (b) 40
8 39
(c) 38 (d) 42
13 22
SSC (CPO) 2013
21. 2 9 11 7
(a) 66 (b) 72 (c) 61 (d) 78
8 5 13 –3
16. 4 3 2
7 ? 10 –4
6 4 10 ?
6 9 10 SSC (CGL) 2013
SSC (CGL) 2013 (a) 3 and ( −2 ) (b) ( −3 ) and ( −2 )
9 27 ?
(c) 3 and 2 (d) ( −3 ) and 2
(a) 30 (b) 20 (c) 50 (d) 54

17. 22. 17 8 5 5
96 100 132
13 7 5 4
6 4 6 6 12 6 3
5 7 3 10 6 4 ?
IB (ACIO) 2013
21 32 ? SSC (10+2) 2013
(a) 4 (b) 5
(a) 25 (b) 30 (c) 32 (d) 20 (c) 6 (d) 7
Chapter 09 INSERTING THE MISSING CHARACTER 91

23. 3 5 8 7 Directions (Q. Nos. 30-37) Find out the


missing numbers and letters.
4 6 4 6
5 2 2 3 30. ? O 25
SSC (CGL) 2013 ? J
58 58 62 ?
(a) 126 (b) 122 (c) 128 (d) 124 49 16
A F
24. 24 25 50 4 C 9 Hotel Management 2011

24 20 10 (a) Y and 40 (b) U and 36


(c) W and 64 (d) X and 81
4 9 3
31. 7 A 5 8 I 4 5 Q 1
12 5 ?
SSC (10+2) 2013
G 49 C O 40 K W ? ?
(a) 10 (b) 15 (c) 20 (d) 5
4 E 3 6 M 2 4 U 2
25. 9 7 4
MAT 2012
5 4 3 (a) V and 9 (b) T and 3

A
(c) S and 6 (d) Y and 5
6 8 ?

M
32. C E
49 52 25

H
SSC (CGL) 2014 A 64 47 G

TT
(a) 6 (b) 7 (c) 4 (d) 5 ? 34
8 23
26. AZ BY CX ?
M
11 16
A K
I MAT 2012
W
DW EV FU
(a) J and 80 (b) O and 83
SH

GT ? IR
SSC (CPO) 2011
(c) N and 74 (d) P and 85
(a) HR (b) HS (c) HV (d) HU
A

33.
8 12 14
@

27. 18 21 24 6 54 7 8 51 4 9 ? 5
3 9 3 4 7 9
6 4 8
21 26 ? SSC (CPO) 2015
SSC (CGL) 2015
(a) 53 (b) 76 (c) 71 (d) 68
(a) 22 (b) 24 (c) 27 (d) 29
34. 6 7 10 7
28. 18 11 6 12 ?
9 38 6 19 32 9 26 44 3 9 ? 20 5 4
17 11 15 8 8 6
SSC (10+2) 2014
Delhi Police (SI & ASI) 2016
(a) 9 (b) 40 (c) 7 (d) 36
(a) 7 (b) 8 (c) 6 (d) 5
29. In the following question, select the number
which can be placed at the sign of question 35. 9 8
mark from the given alternatives. 3 Q 7
E
2 I 6
4 5 6 125
5 4
8 9 7 513
11 ? 6 536 UP (Lower Subordinate) 2016
SSC (CPO) 2017 (a) R (b) N
(a) 7 (b) 10 (c) 9 (d) 8 (c) M (d) L
92 How to Crack Test of Reasoning VERBAL

36. 39. Find out the missing term.


3 9 2 8 4 7
5 45 8 22
7 4 ? 5 ?
81 5 64 6 49 5 40 25 16 48
(a) 5 (b) 6 (c) 12 (d) 9
SSC (CPO) 2014
(a) 1 (b) 8 (c) 6 (d) 16 40. Find out the missing term.
37. A M Y
DL 10 14 FR
7 9 17 8 10 21
RX 23 18 SM
J 35 D V 53 P H ? B
KM ? ? PV MAT 2011 4 5 6 3 3 6
G S ?
(a) 18, 34 (b) 14, 21
MAT 2012
(c) 56, 84 (d) 12, 18 (a) F and 75 (b) E and 65
(c) D and 60 (d) C and 58
38. Which of the given options fits correctly in the
blank space to exhibit the similar pattern of 41. Find out the missing term.
the three sets below? 8 14 5
7 9 11 3 15 S 6 18 D ? 8 K

A
9 3 6
9 4 4 7 ? 7

M
(a) 12 (b) 16 (c) 18 (d) 25 (a) 9 (b) 3 (c) 6 (d) 2

H
TT
Answers W
A
1. (c) 2. (b) 3. (d) 4. (c) 5. (d) 6. (c) 7. (b) 8. (b) 9. (a) 10. (c)
SH

11. (b) 12. (a) 13. (d) 14. (d) 15. (b) 16. (c) 17. (a) 18. (c) 19. (b) 20. (c)
21. (a) 22. (a) 23. (d) 24. (c) 25. (b) 26. (b) 27. (d) 28. (a) 29. (d) 30. (b)
A

31. (c) 32. (b) 33. (d) 34. (a) 35. (c) 36. (c) 37. (b) 38. (b) 39. (b) 40. (b)
@

41. (d)

Answer with Explanations


1. (c) In figure I, 2 2 + 42 = 4 + 16 = 20 4. (c) As, in column I, 16 + 36 = 4 + 6 = 10
In figure II, 3 + 9 = 9 + 81 = 90
2 2
Also, in column II, 25 + 64 = 5 + 8 = 13
Similarly in figure III, 12 + 7 2 = 1 + 49 = 50 Similarly, in column III, 9 + 81 =?
⇒ 3+ 9=?
2. (b) In figure I, ⇒ ? = 12
64 + 36 + 49 = 8 + 6 + 7 = 21
5. (d) As, (86 − 38) × 10 = 480
Similarly, in figure II,
and (72 − 21) × 10 = 510
121 + 81 + 100 = 11 + 9 + 10 = 30
Same as, (36 − 12 ) × 10 = 240
3. (d) As, 21 + 35 = 56
14 + 42 = 56 6. (c) Starting from letter A in vowel sequence in
16 + 40 = 56 clockwise direction
Similarly, A, E, I, O, U, A, E (outer side)
41 + ? = 56
In inner side, (right half part)
⇒ ? = 56 − 41
(7 − 3)3 = 64 and (8 − 2 )3 = 216
⇒ ? = 15
Chapter 09 INSERTING THE MISSING CHARACTER 93

In inner side, (left half part) Similarly,


(5 − 4)3 = 1 (2 × 8) ÷ ? = 4
(11 − 8)3 = 27 16
and ⇒ ?= =4
Hence, ? = E and 27 4
(4 + 6) − (5 + 1) 10 − 6 Hence, ?= 4
7. (b) In figure I, = =2
2 2 12. (a) Pattern given in figure is as follows
(9 + 6) − (7 + 2 ) 15 − 9 5 × 2 + 2 = 12
In figure II, = =3
2 2 12 × 2 + 2 = 26
Similarly, in figure III, 26 × 2 + 2 = 54
(19 + 11) − (13 + 5) 30 − 18
= =6 54 × 2 + 2 = 110
2 2
110 × 2 + 2 = 222
8. (b) In figure I, 4 × 2 × 3 × 3 = 72 So, the missing number is 54.
In figure II, 9 × 4 × 10 × 2 = 720 13. (d) As, 24 + 20 + 37 = 81
Similarly, in figure III, 6 × 20 × 1 × 6 = 720 and 26 + 36 + 19 = 81
9. (a) Consider columnwise, Similarly,
+2 +2 31 + 25 + ? = 81

A
As, D  → F  → H
⇒ 56 + ? = 81

M
+3 +3
M  → P  → S ⇒ ? = 81 − 56

H
+4 +4 ⇒ ? = 25
Similarly, V  → Z  → D

TT
10. (c) As, 14. (d) As, 11 + 14 × 2 = 11 + 28 = 39
27 9 12 + 15 × 3 = 12 + 45 = 57
and
A
Similarly, 13 + 16 × 4 = 13 + 64 = 77
W
2
15. (b) The pattern of the series is as follows
SH

1
5 8 13 22 39 72
Figure is solved as,
A

27 ÷ 9 − 2 = 1 ⇒ 3 − 2 = 1 ⇒ 1 = 1 +3 +5 +9 +17 +33
@

Similarly, second figure +2 +4 +8 +16


35 7
×2 ×2 ×2
3 3 3
16. (c) Consider column wise 4 × = 6, 6 × =9
2 2 2
and 3 × 3 = 9, 9 × 3 = 27
⇒ 35 ÷ 7 − 3 = 2
⇒ 5 − 3 = 2 ⇒2 = 2 Similarly, 2 × 5 = 10, 10 × 5 = 50
Then, Hence, 50 will come in place of question mark.
36 4
17. (a) As, 96 ÷ 6 = 16 and 16 + 5 = 21
4
and 100 ÷ 4 = 25
? and 25 + 7 = 32
Similarly, 132 ÷ 6 = 22 and 22 + 3 = 25
⇒ 36 ÷ 4 − 4 = ?
⇒ 9− 4=? ⇒ ? = 5 18. (c) As, 33 + 7 3 = 27 + 343 = 370
and 2 3 + 63 = 8 + 216 = 224
11. (b) As, (6 × 5) ÷ 3 = 10
Similarly, 13 + X 3 = 730
(4 × 6) ÷ 3 = 8
(5 × 9) ÷ 15 = 3 ⇒ X 3 = 729
⇒ X = 3 729 = 9
94 How to Crack Test of Reasoning VERBAL

19. (b) In the first two figures, the number at the top is 27. (d) As, 6 − 3 = 3 + 18 = 21
divided by the number at the right and is then
84 and 9 − 4 = 5 + 21 = 26
doubled. As in figure I, = 7 and 2 × 7 = 14. Similarly, 8 − 3 = 5 + 24 = 29
12
In figure II,
81
= 9 and 2 × 9 = 18. 28. (a) As, (18 + 9 + 17 ) − 6 = 44 − 6 = 38
9 and (11 + 19 + 11) − 9 = 41 − 9 = 32
88
In figure III, = 8. and (6 + 26 + 15) − 3 = 47 − 3 = 44
11
Similarly, (12 + 9 + 8) − 20 = 29 − 20 = 9
And the missing number will be 2 × 8 = 16
29. (d) In first row, {4 × 6 + 1} × 5 = 125
20. (c) As, 92 + 64 + 52 = 208 –40 ⇒ 25 × 5 = 125
and 70 + 53 + 45 = 168
–40 In second row, {8 × 7 + 1} × 9 = 513
Similarly, 48 + 42 + 38 = 128
⇒ 57 × 9 = 513
21. (a) As, 2 + 9 = 11
In third row, {11× 6 + 1} × ? = 536
9 − 2= 7
⇒ (66 + 1)? = 536
Also, 8 + 5 = 13, 5 − 8 = − 3
Now, 7 + ? = 10 ⇒ 67 × ? = 536
⇒ ? = 10 − 7 = 3 536
⇒ ?= =8

A
Also, 4− 6=? 67

M
⇒ ? = −2 ⇒ ?= 8

H
17 + 8 30. (b) Starting from letter A in anti-clockwise direction
22. (a) Here, in row 1, =5

TT
5 +2 +3 +4 +5
13 + 7 A 
→ C  
→ F  
→ J  
→ O
In row 2, =4
5 A +6

→ U
W
6 + 12
In row 3, =3 Starting from number 4 in anti-clockwise
6
SH

direction
10 + 6 2 2 = 4, 32 = 9, 42 = 16, 52 = 25,
In row 4, =?
4 62 = 36 and 7 2 = 49
A

⇒ ?= 4 ? = U and 36
@

Hence,
23. (d) As, (3 × 4 × 5) − 2 = 60 − 2 = 58 31. (c) In first figure, starting from letter A in clockwise
and (5 × 6 × 2 ) − 2 = 60 − 2 = 58 direction
+2 +2 +2
and (8 × 4 × 2 ) − 2 = 64 − 2 = 62 A 
→ C  
→ E  
→ G
Similarly, (7 × 6 × 3) − 2 = 126 − 2 = 124 and (7 2 + 52 ) − (42 + 32 )
Hence, 124 will come in place of question mark.
= (49 + 25) − (16 + 9) = 49 (middle number)
24. (c) As, (24 + 24) ÷ 4 = 12
In second figure, starting from letter I in
and (25 + 20) ÷ 9 = 5 clockwise direction
(50 + 10) ÷ 3 = 20 +2 +2 +2
Similarly, I 
→ K  
→ M  
→ O
25. (b) As, 9 × 6 − 5 = 49 and (82 + 42 ) − (62 + 2 2 )
and 7 × 8 − 4 = 52 = (64 + 16) − (36 + 4) = 40 (middle number)
Similarly, 4 × ? − 3 = 25 Similarly, in third figure, starting from letter Q in
⇒ 4 × ? = 25 + 3 = 28 clockwise direction
+2 +2 +2
⇒ ?=
28
= 7 ⇒? = 7 Q 
→ S  
→ U  
→ W
4 and (52 + 12 ) − (42 + 2 2 )
26. (b) In each cell, pair of opposite letters is given.
= (25 + 1) − (16 + 4) = 6
+1
∴ G —→ H and opposite letter of H is S. Hence, ? = S and 6
So, HS will come in place of question mark.
Chapter 09 INSERTING THE MISSING CHARACTER 95

32. (b) Starting from letter A in clockwise direction P V


+2 +2 +2 +2 ↓ ↓
A 
→ C  
→ E  
→ G  
→ I 16 + 22 =
38
= 19 + 2 = 21
+2 +2 +2 2
→ K   → M   → O
Starting from number 8 in anti-clockwise direction 38. (b) In figure I, (7 − 4)2 = 9
add (prime number sequence). In figure II, (9 − 7 )2 = 4
+3 +5 +7 +11
8 → 11  → 16  → 23  
→ 34 Similarly, in figure III, (11 − 7 )2 = 42 = 16
+19
+13
 +17
→ 47  → 64  → 83 So, missing number = 16
Hence, ? = O and 83 39. (b) In figure I,
25
33. (d) In figure I, 6 × 7 + (8 + 4) = 42 + 12 = 54 = 45 − 40 = 5 ⇒ 5 = 5 = 5
5
In figure II, 8 × 4 + (12 + 7 ) = 32 + 19 = 51 Similarly, in figure II,
Similarly, in figure III, 48
= 22 − 16 = ? ⇒ 6 = 6 = ?
9 × 5 + (14 + 9) = 45 + 23 = 68 8
34. (a) 6 7 ? 10 7 So, missing number = 6
I II
40. (b) In first figure, starting from letter A in clockwise

A
III
5 4 direction
8 6 +3 +3 +3

M
A  → D   → G   → J
In triangle I, 5 + 7 = 12 ÷2 = 6

H
and (7 + 9) × 2 + 4 + 5 = 35 (middle number)
In triangle II, 10 + 4 = 14 ÷2 = 7

TT
In second figure, starting from letter M in
Similarly, in triangle III, 8 + 6 = 14 ÷2 = 7 clockwise direction
So, question mark is replaced by 7. +3 +3 +3
A
M  → P   → S   → V
W
35. (c) As, 5 + 4 = 9 ⇒I, 3 + 2 = 5 ⇒E
and (17 + 8) × 2 + 6 + 3 = 53 (middle number)
9 + 8 = 17 ⇒Q
SH

Similarly, in third figure, starting from letter Y in


Similarly, 7 + 6 = 13 ⇒ M clockwise direction
+3 +3 +3
Y  → B   → E   → H
A

81
36. (c) In figure I, − (5 − 3)
9
@

and (10 + 21) × 2 + 3 + 6


= 9−2 = 7 (middle number)
= 65 (middle number)
64
In figure II, − (6 − 2 ) Hence, ? = E and 65
8
= 8− 4 = 4 (middle number) 41. (d) Position of S in English alphabetical order = 19
49
Similarly, in figure III, − (5 − 4) Position of D in English alphabetical order = 4
7
= 7 −1= 6 (middle number) Position of K in English alphabetical order = 11

37. (b) As, D L Now, in figure I, (8 × 9) − 15 = 3 × 19 (S)


↓ ↓ ⇒ 72 − 15 = 57 ⇒ 57 = 57
16 Again, in figure II,
4 + 12 = = 8 + 2 = 10
2 (14 × 3) − 18 = 6 × 4(D)
It means, positional values of letters will be added ⇒ 42 − 18 = 24
and divided by 2 and after division, 2 will be ⇒ 24 = 24
added.
Similarly, K M Similarly, in figure III, (5 × 6) − 8 = ? × 11 ( K )
↓ ↓ ⇒ 30 − 8 = ? × 11 ⇒ 22 = ? × 11
24
11 + 13 = = 12 + 2 = 14 ⇒ ?=
22
= 2
2 11
96 How to Crack Test of Reasoning VERBAL

10
Number, Ranking and
Time Sequence Test
Number, ranking and time sequence test is Illustration 2. How many even numbers are

A
based on arrangement of characters/ persons/ there in the following series of numbers, each

M
objects in a particular order based on some of which is preceded by an odd number, but

H
specific parameters. not followed by an even number?
5348971653298735

TT
In this chapter, we deal with questions which
have either a sequence of numbers, position of (a) 0
a person/object in a row/queue or time (b)
(c)
1
2 A
W
sequence test accompanied by some conditions.
(d) 3
SH

Types of Questions Solution (d) 5 3 4 8 9 7 1 6 5 3 2 9 8 7 3 5


There are three such even numbers 6, 2 and 8
A

Generally three types of questions based on each of which is preceded by an odd number and
@

number, ranking and time sequence test are not followed by an even number.
asked in various competitive exams.
Directions (Illustrations 3-4) Following
questions are based on the five three digit
Type 1 numbers given below.
519 364 287 158 835
Number Test
Illustration 3. If the positions of the first and the
In this type of questions, a number/a set of third digits within each number are
numbers/a series of digits is given and the interchanged, which of the following will be
candidates are asked to find out digits the third digit of the second lowest number?
following certain given conditions.
(a) 9
Illustration 1. How many 5’s are there in the (b) 5
following sequence which are preceded by 3 (c) 7
but not followed by 8? SSC (CGL) 2015 (d) 8
45832735178935831352 Solution (d) According to the question, after
(a) 4 (b) 3 (c) 2 (d) 1 interchanging the position of the first and third
Solution (c) 4 5 8 3 2 7 3 5 1 7 8 9 3 5 8 3 1 3 5 2 digits, new numbers are 519 ⇒ 915, 364 ⇒ 463,
So, it is clear from above that there are two 5’s in 287 ⇒ 782, 158 ⇒ 851, 835 ⇒ 538, so second
the given sequence which are preceded by 3 but lowest number = 538.
not followed by 8. Hence, third digit of the second lowest number is 8.
Chapter 10 NUMBER, RANKING AND TIME SEQUENCE TEST 97

Illustration 4. Which of the following is the Important Formulae Related to


difference between the second digit of the
lowest and the highest of these numbers? Ranking Test
(a) 3 (b) 1 The position or rank can be calculated with the
(c) 2 (d) 0 help of following formuale
Solution (c) According to the question, 1. Total number of persons in a row = (Rank of a
person from upper or left end) + (Rank of that
Highest number = 835
person from lower or right end) −1
Lowest number = 158 [because that particular place counts two times]
∴Difference between the second digit of the lowest 2. Rank of a person from upper or left end = (Total
and the highest number = 5 − 3 = 2 number of persons in row) − (Rank of that person
Illustration 5. The positions of how many digits from lower or right end) + 1
in the number 351462987 will remain 3. Rank of a person from lower or right end
unchanged after the digits are rearranged in = (Total number of persons in row)
ascending order within the number? − (Rank of that person from upper or left end) + 1
(a) None 4. In case of interchanging of positions
(b) One (a) Total number of persons =
(c) Two

A
[Initial position of 1st person + Interchanged
(d) Three
position of 2nd person] −1

M
Solution (c) Given number 3 5 1 4 6 2 9 8 7
(b) New position of 2nd person =

H
After rearrangement 1 2 3 4 5 6 7 8 9
[Differente in the two positions of Ist person] +

TT
Hence, there are two numbers, 4 and 8 which
[Initial position of 2nd person]
remain unchanged after the rearrangement.
Illustration 6. The positions of the first and the A
Illustration 7. In a class, boys stand in a single
W
row. One of the boy is nineteenth in order
sixth digit in the number 5109238674 are
from both the ends. How many boys are there
SH

interchanged. Similarly, the positions of the


in the row?
second and the seventh digit are interchanged
(a) 27 (b) 37 (c) 38 (d) 39
A

and so on. Which of the following will be the


third digit from the right end after the Solution (b) According to the question,
@

rearrangement?
(a) 9 (b) 0
19th from left 19th from right
(c) 6 (d) 3
Solution (b) Given number = 5109238674 Clearly, number of boys in the row
= (18 + 18 + 1) = 37
After interchanging the digits,
Alternate Method (Using Formula 1)
new number = 3867451092
Total number of boys in the row = 19 + 19 − 1 = 37
Hence, third digit from the right end = 0
Illustration 8. In a class of 42 students, Mahesh’s
rank is 16th from the bottom. What is his rank
Type 2 from the top?
Ranking Test (a) 25th (b) 26th (c) 24th (d) 27th
Solution (d) Number of students ahead of Mahesh
In this type of questions, generally, the rank = 42 − 16 = 26
of a person either from the top/bottom or from Now, Mahesh’s rank from top = 26 + 1 = 27
right/left or the total number of persons is to
So, Mahesh’s rank is 27th from the top.
be find out based on conditions given in
question. Alternate Method (Using Formula 2)
∴ Mahesh’s rank from the top = 42 − 16 + 1 = 27
98 How to Crack Test of Reasoning VERBAL

Illustration 9. Anu and Vinay are ranked seventh Combining Eqs. (i), (ii) and (iii), we get,
and eleventh, respectively from the top in a Gautam > Ramesh > Mohan > Shyam > Rajat
class of 31 students. What will be their Hence, Rajat is the shortest among all.
respective ranks from the bottom in the class?
(a) 20th and 24th (b) 24th and 20th
(c) 25th and 21st (d) None of these
Type 3
Solution (c) Number of students after Anu in the class Time Sequence Test
= (31 − 7) = 24
Now, Anu’s rank from bottom = 24 + 1 = 25 In this type of questions, candidates are
So, Anu is 25th from the bottom. required to find out a particular day or time
Number of students after Vinay in the class based on some given conditions.
= (31 − 11) = 20 To solve such type of problems, the candidate
Now, Vinay’s rank from bottom = 20 + 1 = 21 must have a basic knowledge regarding
So, Vinay is 21st from the bottom. calender and clock.
Alternate Method (Using Formula 3) Illustration 12. Sunita leaves her house at 20 min
∴Anu’s rank from the bottom = 31 − 7 + 1 = 25 to seven in the morning, reaches Vineeta’s
Vinay’s rank from the bottom = 31 − 11 + 1 = 21 house in 25 min, they finish their breakfast in

A
Illustration 10. In a row of students, Anil is 7th another 15 min and leave for their office

M
from left, while Sunil is 18th form right. Both which takes another 35 min. At what time, did
they leave Vineeta’s house to reach their

H
of them interchanged their positions such that
Anil becomes 21st from left. What will be the office? KVS (PRT) 2015

TT
total number of students in the class? (a) 7 : 40 am (b) 7 : 20 am
(c) 7 : 45 am (d) 8 : 15 am
(a) 30 (b) 33 (c) 31 (d) 30
A
Solution (b) Sunita leaves her house at 6 : 40 am.
W
Solution (a)
She reaches Vineeta’s house in 25 min, i.e.
6 : 40 + 0 : 25 at 7 : 05 am.
SH

7th from left 18th from left

Anil Sunil Both leave for office, 15 min after 7 : 05 am i.e.


7 : 05 + 0 : 15 at 7 : 20 am.
A
@

Illustration 13. Kamala would like to complete


all her work before 10 : 00 pm in order to
watch an important TV programme. She has
Sunil Anil 40 min assignment in each of her five
prepared subjects. What is the least time at
21Ist from left which she can start and still complete her
home work in the time for the programme?
Total number of students = [Initial position of Sunil MPPSC 2018
+ Interchanged position of Anil] −1 (a) 6 : 40 pm (b) 6 : 30 pm
= [18 + 21] − 1 = 38 (c) 7 : 10 pm (d) 7 : 20 pm
Solution (a) Time required to complete the
Illustration 11. Mohan is taller than Shyam but homework
shorter than Ramesh. Ramesh is taller than = 5 × 40 = 200 min = 3 h 20min
Rajat but shorter than Gautam. If Shyam is Now, the required time = 10 : 00 − 03 : 20
taller than Rajat, then who is the shortest = 06 : 40 pm
among all? MPPSC 2018
(a) Gautam (b) Rajat (c) Shyam (d) Ramesh Illustration 14. If every second Saturday and all
Solution (b) From given information, Sundays are holidays in a 30 days month
Ramesh > Mohan > Shyam …(i) beginning on Saturday, then how many
Gautam > Ramesh > Rajat …(ii) working days are there in that month?
Shyam > Rajat …(iii) (a) 20 (b) 21 (c) 22 (d) 23
Chapter 10 NUMBER, RANKING AND TIME SEQUENCE TEST 99

Solution (d) As, the month begins with Saturday. (a) 16th (b) 18th
So, 2nd, 9th, 16th, 23rd, 30th are Sundays, while (c) 19th (d) 17th
8th and 22nd are second Saturday. Thus, there are Solution (d) According to Samant, 16, 17
7 holidays in all.
∴ Number of working days = 30 − 7 = 23 According to Possible dates
Samant 16, 17
Illustration 15. Samant remembers that his
His sister 17, 18
brother’s birthday is after 15th but before 18th
of February, while, his sister remembers that Here, 17th is common.
her brothers’ birthday is after 16th but before
Hence, brother’s birthday is on 17th.
19th of February. On which date, of February,
is Samant’s brother’s birthday?

Let us Practice
1. How many ‘8’ are followed by even number 5. How many such even digits are there in the

A
and preceded by an odd number? above arrangement each of which is

M
SSC (Multitasking) 2013 immediately preceded and also immediately

H
184381483287848568784186 followed by odd numbers?

TT
(a) 3 (b) 4 (c) 9 (d) 5 (a) Three (b) Four
(c) Five (d) Six
2. How many 9’s are followed by and preceded
(e) None of these
by numbers divisible by 2? EPFO 2011
A
W
89653596834965269737294137941734 6. If two is subtracted from each odd digit and
98453976153195742968532957489451 three is added to each even digit in the
SH

(a) 12 (b) 6 (c) 8 (d) 10 number 3675249. How many digits will
(e) 4 appear twice in the new number thus
A

formed? SSC (Steno) 2012


3. How many such 5’s are there in the (a) 1 (b) 2
@

following number sequence, each of which (c) 3 (d) 4


is preceded by 3 or 4 but not followed by
8 or 9? 7. The positions of how many digits in the
35954553584567357554523510 number 837912 will remain unchanged after
(a) Six (b) Three the digits within the number are rearranged
(c) Four (d) Five in descending order (from left to right)?
(e) None of these IBPS (Clerk) 2012
(a) None (b) One (c) Two (d) Three
Directions (Q. Nos. 4-5) Study the following (e) More than three
arrangement of numbers carefully and answer 8. How many such pairs of digits are there in
the questions that follow. PNB (Clerk) 2012 the number 75938462, each of which has as
5 7 9 4 8 2 4 2 5 7 8 4 2 many digits between them in the number as
9 2 4 8 7 5 2 1 2 5 4 9 7 they have between them when arranged in
4. How many such digits are there in the above ascending order? Bank of Baroda (PO) 2000
(a) None (b) One (c) Two (d) Three
arrangement each of which is followed by a (e) More than three
digit which has a greater value than the first
digit? 9. If the following numbers are written in
(a) Eleven ascending order then what will be the
(b) Seven middle digit of the middle term?
(c) Eight 815, 686, 795, 835, 765, 822, 719 OPSC 2018
(d) Nine (a) 8 (b) 1 (c) 3 (d) 9
(e) None of these
100 How to Crack Test of Reasoning VERBAL

10. If all the digits of the number ‘6379452’, are 18. In a row of girls, Veena is 12th from the start
arranged in ascending order from left to and 19th from the end. In another row of
right, what will be the sum of the fourth girls, Sunita is 14th from the start and 20th
digit from the left and second digit from the from the end. How many girls are there in
right after rearrangement? IBPS (Clerk) 2016 both the rows together?
(a) 10 (b) 12 (c) 15 (d) 13 (a) 72 (b) 65 (c) 63 (d) 61
(e) 11
19. In a line of boys Aman is 12th from top and
11. In the number 76534218, each digit is Baman is 18th from bottom. If there are 6
replaced by the next digit i.e., ‘1’ is replaced boys between Aman and Baman, then how
by ‘2’, ‘2’ is replaced by ‘3’ and so on and many maximum boys are there in the row?
then the digits are arranged in ascending SSC (SI and Asst.) 2017
order from left to right, which digit will be (a) 34 (b) 36
fifth from the left end? SBI (PO) 2017 (c) 35 (d) 37
(a) None (b) 7 (c) 6 (d) 5
(e) 4 20. Consider five people A, B, C, D and E, each
having different age. A is younger than only
12. In a given number 7834329513, we B. C is older than D. D is not the youngest.
interchange the first and the second digits, Who amongst the following are older
the third and the fourth digit and so on.

A
than C? CDS 2012
Then which digit will be sixth from the (a) A and B (b) E, B and A

M
right? DSSSB 2018 (c) A and E (d) E and B

H
(a) 3 (b) 2 (c) 9 (d) 4
21. Five birds Crow, Pigeon, Little pigeon, Big

TT
13. Sohan ranks seventh from the top and crow and Eagle fly one after other from a
twenty-sixth from the bottom in a class. How tree branch. Big crow flew after Crow but is
many students are there in the class?
SSC (10+2) 2013 A
ahead of Eagle. Pigeon is between Crow and
W
(a) 31 (b) 32 Big crow. Little pigeon is before Crow.
Which bird is in the last? SSC (Constable) 2012
SH

(c) 33 (d) 34
(a) Pigeon (b) Big crow
(c) Eagle (d) None of these
14. Bharati is 8 ranks ahead of Divya, who ranks
A

22. In a group of five districts, Akbarpur is


26th in a class of 42. What is Bharati's rank
@

smaller than Fatehpur, Dhanbad is bigger


from the last? WBPSC 2018
than Palamu and Bara Banki is bigger than
(a) 9th (b) 24th (c) 25th (d) 34th Fatehpur but not as big as Palamu. Which
15. In a row at a bus stop, A is 7th from the left district is the biggest? SSC (10+2) 2011
and B is 9th from the right. They both (a) Akbarpur (b) Fatehpur
interchange their positions. Now, A becomes (c) Dhanbad (d) Palamu
11th from the left. How many people are 23. Keshav is taller than Vijay but shorter than
there in the row? MPPSC 2018 Nitin, Nitin is taller than Kishan but shorter
(a) 10 (b) 20 (c) 19 (d) 18 than Amar. If Vijay is taller than Kishan,
16. In a class of 45 students, a boy is ranked 20th. then who is the shortest among them?
When two boys joined, his rank was dropped UPPSC (RO) 2014
(a) Keshav (b) Vijay
by one. What is his new rank from the end?
(c) Nitin (d) Kishan
UPSC (CSAT) 2013
(a) 25th (b) 26th 24. Among five people J, K, L, M and N (each
(c) 27th (d) 28th running at a different speed), who ran at the
17. There are 25 boys in a horizontal row. Rahul third fastest speed?
was shifted by three places towards his right M run faster than L, but slower than K. J ran
side and he occupies the middle position in slower than only one person. L did not run
the row. What was his original positon from at the slowest speed. NICL (Asst.) 2015
(a) Cannot be determined
the left end of the row ? CGPSC 2013
(b) M (c) N
(a) 15th (b) 16th (c) 12th (d) 10th
(d) L (e) K
(e) None of these
Chapter 10 NUMBER, RANKING AND TIME SEQUENCE TEST 101

25. 37 girls are standing in a row facing the (a) 10th


school building. Ayesha is fifteenth from the (b) 11th
left end. If she is shifted six places to the (c) 12th
right, what is her position from the right (d) Cannot be determined
end? SBI (Clerk) 2016 32. Sneha correctly remembers that her father’s
(a) 16th (b) 21st birthday is before 16th June but after 11th
(c) 20th (d) 18th June whereas, her younger brother correctly
(e) None of these remembers that their father’s birthday is
26. In a class of 50 students, Raghu's rank is after 13th June but before 18th June and her
twice that of Paul. There are 10 students who elder brother correctly remembers that their
have ranks worse than that of Raghu. Paul's father’s birthday is on an even date. On what
rank in the class is MP PSC 2018 date, in June, is definitely their father’s
(a) 5th (b) 10th (c) 15th (d) 20th birthday? Vijaya Bank (Clerk) 2010
(a) Sixteenth (b) Twelfth
27. Umar reached the place of meeting
(c) Fourteenth (d) Data inadequate
30 minutes before time. Arvind reached
(e) None of these
45 minutes late for the meeting. The meeting
finished at 11 : 00 am, 30 minutes after 33. Reaching the place of meeting on Tuesday
Arvind reached the meeting. At what time 15 min before 8:30 h, Akshat found himself

A
did Umar arrive for the meeting? MPPSC 2018 half an hour earlier than the man who was

M
(a) 9 : 00 am (b) 9 : 15 am 40 min late. What was the scheduled time of
(c) 9 : 30 am (d) 9 : 45 am the meeting? SSC (10+2) 2010

H
28. Praveen correctly remembers that his father’s (a) 8 : 00 h (b) 8 : 05 h (c) 8 : 15 h (d) 8 : 45 h

TT
birthday is after 21st May but before 27th 34. A bus for Delhi leaves every 30 min from a
May whereas his sister correctly remembers bus stand. An enquiry clerk told a passenger
that their father’s birthday is after 24th May A
that the bus had already left 10 min ago and
W
but before 30th May. On which day in May next but will leave at 9:35 am. At what time
was definitely their father’s birthday?
SH

did the enquiry clerk give this information to


MAT 2012
the passenger?
(a) 25th (b) 24th
(a) 9 : 10 am (b) 8 : 55 am (c) 9 : 08 am (d) 9 : 15 am
A

(c) 26th (d) 25th or 26th


35. If it is possible to make one such number
@

29. The priest told the devotee, ‘‘The temple bell with the first, the fourth and the sixth digits
is rung at regular intervals of 50 minutes.
of the number 531697, which is the perfect
The last bell was rung ten minutes ago. The
square of a two-digit even number, which of
next bell is due to be rung at 7 : 50 am’’ At
the following will be the second digit of the
what time did the priest give this
two digit even number. If no such number
information to the devotee? RRB 2002
(a) 7 : 20 am (b) 7 : 30 am can be made, give ‘@’ as the answer and if
(c) 7 : 10 am (d) 7 : 00 am more than one such numbers can be made,
give ‘’ as the answer.
30. As I reached the station, I came to know that (a) 4 (b) 2 (c) 6 (d) @
I reached 30 minutes earlier. And the train
(e) 
was 1 hr and 30 minutes late, so it arrived at
6 : 20. When did I reach the station? 36. There are twenty people working in an
RRB Gorakhpur 2014 office. The first group of five workers
(a) 4 : 40 O’ clock (b) 5 : 10 O’ clock between 8 : 00 am to 2 : 00 pm. The second
(c) 5 : 20 O’ clock (d) 4 : 20 O’ clock group of ten workers between 10 : 00 am to 4
: 00 pm and the third group of five workers
31. Sangeeta remembers that her father’s between 12 noon to 6 pm. There are three
birthday was certainly after 8th but before computers in the office which all the
13th of December. Her sister Natasha employees frequently use. During which of
remembers that their father’s birthday was the following hours the computers are likely
definitely after 9th but before 14th of to be used most? RRB (ASM) 2012
December. On which date of December was (a) 10 : 00 am-12 : 00 noon (b) 12 noon-2 : 00 pm
their father’s birthday? (c) 1 : 00 pm-3 : 00 pm (d) 2 : 00 pm-4 : 00 pm
102 How to Crack Test of Reasoning VERBAL

37. A person comes to meet his friend in the 43. If ‘2’ is added to the second digit of all even
afternoon at 12 : 30, he again comes at 1 : numbers and ‘3’ is subtracted from the first
20 pm then he comes third time at 2 : 30 pm digit of all odd numbers, in how many numbers
and the fourth time he comes at 4 : 00 pm thus formed will a digit appear twice?
When will he come again? MPPSC 2017 (a) None (b) Three
(a) 5 : 00 pm (b) 5 : 20 pm (c) More than three (d) Two
(c) 5 : 50 pm (d) 6 : 10 pm (e) One
Directions (Q. Nos. 38-40) Study the sets of 44. Reaching the place of meeting 20 min before
numbers given below and answer the questions. 8 : 50 h, Satish found himself 30 min earlier
972 682 189 298 751 LIC (ADO) 2012 than the man who came 40 min late. What
38. If one is added to the lowest number and was the scheduled time of the metting?
SSC (10+2) 2002
two is added to the highest number, then (a) 08 : 20 (b) 08 : 10 (c) 08 : 05 (d) 08 : 00
what will be the difference between the
second digit of the smallest number and 45. In a row of boys, Rajan is 10th from the right
third digit of the highest number? and Suraj is 10th from the left. When Rajan
(a) 5 (b) 7 and Suraj interchange their positions, Suraj
(c) 9 (d) 8 will be 27th from the left. Which of the
(e) None of these following will be Rajan’s position from the

A
right? IBPS (Clerk) 2012
39. If in each number, second and third digit are

M
(a) 10th (b) 26th (c) 29th (d) 25th
interchanged, then what will be the sum of (e) None of these

H
first digit of the smallest number and last
46. In a queue, Mr. X is fourteenth from the

TT
digit of highest number?
(a) 7 (b) 6 front and Mr. Y is seventeenth from the end,
while Mr. Z is exactly in between Mr. X and
(c) 9
(e) None of these
(d) 8
A
Mr. Y. If Mr. X is ahead of Mr. Y and there
W
are 48 persons in the queue, how many
40. If one is added to the smallest odd number persons are there between Mr. X and Mr. Z?
SH

and one is subtracted from the highest odd UPSC (CSAT) 2013
number, then which of the following will be (a) 6 (b) 7 (c) 8 (d) 9
A

obtained, if the second digit of that odd


47. In a vertical queue of 13 people, all facing
@

highest number is subtracted from the


North, K stands exactly at the centre of the
second digit of that odd lowest number thus
queue. No one stands between K and W.
formed?
(a) 6 (b) 5 (c) 4 (d) 3
Only five people stand between W and P. L
(e) 2 stands at one of the positions before P, but
not at the beginning of the queue. How
Directions (Q.Nos 41-43) The following many people stood after W? SBI (PO) 2016
questions are based on the given 3-digit (a) Three (b) None (c) Five (d) Seven
numbers. (e) Nine
821 547 452 935 368 SBI (Clerk) 2016 48. Anil reached a place on Friday. He came to
41. The product of the second and the third digit know that he was three days earlier than the
in which of the following numbers is the scheduled day. If he had reached there on
second highest? the following Sunday, how many days
(a) 547 (b) 935 (c) 368 (d) 452
late/early he would have been?
SSC CGL 2000
(e) 821 (a) One day earlier (b) One day late
42. The difference of the first and the second (c) Two days late (d) Two days earlier
digits in how many of the given numbers is 49. In a class, Anil ranks 7th from the top, Rohit
greater than the third digit of the same is 7 ranks ahead of Sumit and 3 ranks behind
number? Anil, Vijay, who is 4th from the bottom is 32
(a) Two (b) Three ranks behind Sumit. How many students are
(c) More than three (d) One there in the class? MPPSC 2014
(e) None (a) 39 (b) 49 (c) 52 (d) 62
Chapter 10 NUMBER, RANKING AND TIME SEQUENCE TEST 103

50. Aman is 16th from the left end in a row of 53. If D is 5 cm less than F, what would be the
boys and Vivek is 18th from the right end. length of D?
Gagan is 11th from Aman towards the right (a) 7 cm
and 3rd from Vivek towards the right end. (b) 8 cm
How many boys are there in the row? (c) 9 cm
SBI (PO) 2007
(a) 40 (b) 42 (c) 48 (d) 41 (d) Cannot be determined
(e) None of these (e) None of the above

51. In a queue of fifteen people facing North, 54. Which wire has least length?
Rahul’s position is ninth from the end of the (a) B (b) A (c) C (d) E
queue. Only four people are standing (e) None of these
between Rahul and Tom, Karan is standing 55. If A carry 10 cm length and B carry 5 cm
immediately after Tom. Bharti is standing length, then what would be the length of C?
exactly between Karan and Rahul. Sonali is (a) 6 cm (b) 2 cm (c) 7 cm (d) 9 cm
standing before Bharti, but after Tom. What (e) None of these
is position of Sonali from the beginning of
the queue? (Note All people are standing Directions (Q. Nos. 56-58) Study the given
one behind the other) IBPS (Officer) 2016 information carefully to answer the given
questions.

A
(a) Cannot be determined (b) Seventh
(c) Eleventh (d) Fourth

M
Six people J, K, L, M, N and O are holding a
(e) Sixth
different number of flowers, but not necessarily

H
52. The Chairman of the Selection committee in the same order. J holds less than L but more

TT
arrived at the interview-room for conducting than N. M holds less flowers than N but more
an interview at 10 min to 12 : 30 h. He was than O. L does not hold the most number of
earlier by 20 min than the other members of A
flowers. M holds 16 flowers. NABARD (AM) 2016
W
the board, who arrived late by 30 min. At
what time were the interview scheduled? 56. Who amongst the following holds the third
SH

SSC Investigator 2005 highest number of flowers?


(a) 12 : 10 (b) 12 : 20 (c) 12 : 30 (d) 12 : 40 (a) N (b) O (c) L (d) J
A

(e) Other than those given as options


Directions (Q. Nos. 53-55) Study the
@

57. How many people hold more flowers than


following information and answer the given L?
question. IBPS (PO) 2016 (a) Three (b) Two (c) None (d) Four
There are six wires in an extension box A, B, C, (e) One
D, E and F. They have different length, but not 58. If the difference between the number of
necessarily in the same order. E is greater than flowers hold by M and J is 12, then how
C, but less than D and B. A is greater than D many flowers does N possibly hold?
and B. A is not longest wire. F is 13 cm long (a) 9 (b) 4 (c) 34 (d) 22
and E is 4 cm long. (e) 43

Answers
1. (b) 2. (b) 3. (d) 4. (a) 5. (a) 6. (b) 7. (b) 8. (e) 9. (d) 10. (b)
11. (c) 12. (b) 13. (b) 14. (c) 15. (c) 16. (c) 17. (d) 18. (c) 19. (b) 20. (a)
21. (c) 22. (c) 23. (d) 24. (b) 25. (e) 26. (d) 27. (b) 28. (d) 29. (c) 30 (d)
31. (d) 32. (c) 33. (b) 34. (d) 35. (a) 36. (b) 37. (c) 38. (a) 39. (d) 40. (c)
41. (a) 42. (a) 43. (e) 44. (a) 45. (e) 46. (c) 47. (d) 48. (a) 49. (c) 50. (d)
51. (d) 52. (a) 53. (b) 54. (c) 55. (b) 56. (d) 57. (e) 58. (d)
How to Crack Test of Reasoning VERBAL

Answer with Explanations


1. (b) 1 8 4381483287 8 485687 8 41 8 6 Now, fourth digit from the left = 5
There are four such 8’s which are followed Second digit from the right = 7
by even number and preceded by an odd Sum of fourth and second digit = 5 + 7 = 12
number. 11. (c) Original number - 7 6 5 3 4 2 1 8
2. (b) 8 9 653596834 9 652697372 9 4137941734 9 8 Step I Arrangement - 8 7 6 4 5 3 2 9
453976153195742 9 6853295748 9 451 Step II Arrangement - 2 3 4 5 6 7 8 9
In the given series, there are six 9’s followed by
and preceded by number divisible by 2.
5th from Left
3. (d) There are five such 5’s as shown below Hence, required digit = 6
35954 5 53584 5 673 5 75 54 5 12. (b) Given number : 7 8 3 4 3 2 9 5 1 3
23 5 10 After interchanging the digits,
4. (a) 579, 425, 257, 578, 429, 248, 487, 125, 254, 549 New number : 8 7 4 3 2 3 5 9 3 1
and 497, i.e. Eleven 6th digit from right end

A
5. (a) 5 7 9 4 8 2 4 2 5 7 8 4 2 9 2 4 8 7 5 2 1 2 5 13. (b) Total students in the class

M
4 9 7 i.e. Three = 7 + 26 − 1 = 33 − 1 = 32 (Using formula 1)
14. (c) Divya's rank from the last = 42 − 26 + 1 = 17

H
6. (b) Subtract 2 from odd digits in given number and

TT
add 3 in even digits ∴Bharati's rank from the last = 17 + 8 = 25
3 6 7 52 4 9 15. (c)
↓ ↓ ↓ ↓ 7th from left 9 from left
−2 −2 −2 −2 A
W
A B
16 5 3 2 4 7
↓ ↓ ↓ A
SH

+3 +3 +3
1 9 5 3 5 7 7
A
@

So, in new number there are two digits of five


and two digits of seven. B A

7. (b) Given number — 83 7 912 11th from left


Descending order — 9 8 7 3 2 1 ∴Total number of persons in row = (11 + 9) − 1
Hence, the position of only one digit, i.e. 7, = 19
remains unchanged.
16. (c) Total number of boys after 2 new boys joined
8. (e) Given number 7 5 9 3 8 4 6 2 = 47. Since, the rank of the boy dropped by one
Arrange the digits in ascending order, it becomes 21th from 20th and it means that one
boy who joined was in front of him and one
2 3 4 5 6 7 8 9 behind him, so his rank is 21th among 47
students.
Therefore, such pair = six Then his rank from end = 47− 21 + 1 = 27th
i.e. (7,9), (5,8), (3,6), (8,6), (5,3) and (4,2). 17. (d) When Rahul was shifted by three places towards
his right side, then he occupies the middle
9. (d) Ascending order:
position, i.e. 13th position.
686, 719, 765, 795 , 815, 822, 835
Middle number 10th 13th
Thus, his original position from the left end of the
∴Middle digit of 795 is 9.
row = 13 − 3 = 10th
10. (b) When the digits of the number ‘6379452’ are 18. (c) In first row, Veena is 12th from starting and 19th
arranged in ascending order, then from end.
rearrangement will be 2 3 4 5 6 7 9
Chapter 10 NUMBER, RANKING AND TIME SEQUENCE TEST 105

Total number of girls = (12 + 19 −1) = 30 27. (b) Q Meeting finished at 11 : 00 am.
(Using formula 1) ∴ Arvind reached the meeting at
In second row, Sunita is 14th from starting and 11 : 00 am – 30 min = 10 : 30 am
20th from the end.
Meeting actual time = 10 : 30 am − 45 min
Then, total number of girls = (14 + 20 − 1) = 33
∴Total number of girls in both rows = 9 : 45 am
= 30 + 33 = 63 Umar arrived for the meeting
19. (b) Number of boys between Aman and Baman = 9 : 45 am – 30 min = 9 : 15 am
= Total number of boys in row − (Aman’s rank 28. (d) According to Praveen — 22, 23, 24, 25 , 26
from top + Baman’s rank from bottom)
⇒ 6 = Total number of boys in row − (12 + 18) According to his sister — 25 , 26 , 27, 28, 29
∴Total number of boys in row = 6 + 30 = 36 Here, common dates are 25 and 26.
20. (a) Arrangement of the people according to their Hence, their father’s birthday will be either on
age, 25th or 26th.
B>A>C>D>E 29. (c) The last bell rang 50 min before 7 : 50 am, i.e. at
Clearly, A and B are older than C. 7 : 00 am but it happened 10 min before the
21. (c) Arrangement of birds in the order of their flight, priest gave the information to the devotee. So,
the information was given at 7 : 10 am
Eagle → Big crow → Pigeon → Crow →

A
Little pigeon 30. (d) The right time of arrival of train

M
Clearly, Eagle is in the last. = 6 : 20 − 1 hr 30 min = 4 : 50

H
22. (c) Here the descending order of five districts is as ∴ The time when I reached the station

TT
follows = 4 : 50 − 0 : 30 = 4 : 20 O’ clock
Dhanbad > Palamu > Bara Banki > Fatehpur 31. (d) According to Sangeeta — 9, 10 , 11 , 12
> Akbarpur. Hence, Dhanbad district is the A
According to Natasha — 10 , 11 , 12 , 13
W
biggest amongst them.
Here, 10, 11 and 12, three dates are common.
23. (d) Let us deduce the order of height as given in the
SH

So, date of her father’s birthday cannot be


question
determined.
Nitin > Keshav > Vijay …(i)
A

Amar > Nitin > Kishan …(ii) 32. (c) According to Sneha, 12, 13, 14 , 15
@

and Vijay > Kishan …(iii) According to younger brother, 14 , 15, 16, 17
Now, combining above equations, we have According to elder brother …, 14 , 16, …
Amar > Nitin > Keshav> Vijay > Kishan Hence, confirm date is 14th.
Hence, Kishan is the shortest.
33. (b) Akshat reached the place of meeting at 8 : 15 h.
24. (b) According to the information, the arrangement is He reached 30 min earlier than the man who
as follows was 40 min late, so he reached 10 min late.
K>J>M>L>N Hence, the scheduled time of the meeting was
8:05 h.
Third from the left
34. (d) Next but will leave at 9 : 35 am.
So, ‘M’ ran at the third fastest speed.
So, last but left at (9 : 35 − 0 : 30) = 9 : 05 am
25. (e) Present position of Ayesha from left So, enquiry clerk gave the information to the
= 15 + 6 = 21st passenger = (9: 05 + 0:10) = 9:15 am
∴Position of Ayesha from right
35. (a) 5 3 1 6 9 7
= 37 − 21 + 1 = 17th (Using formula 3)   
26. (d) Raghu's rank in the class = 50 − 10 = 40 5 6 7
40 Perfect square = 576,
Hence, Paul's rank in the class = = 20
2 Square root = 576 = 24
Hence, second digit = 4
106 How to Crack Test of Reasoning VERBAL

36. (b) Shifts


Hence, there are two numbers in which
Twenty people
difference of first two digits is greater than the
First group of 5 8 am-2 pm third digit.
Second group of 10 10 am-4 pm 43. (e) 821 547 452 935 368
Third group of 5 12 noon-6 pm –3 –3 +2 –3 +2

521 247 472 635 388


Here, timing 12 noon-2 pm is commonly exist in
all the shifts, so the computers are likely to be Hence, there is only one number 388 that has a
used most during this time. digit appear twice.
37. (c) The person comes to meet his friend 44. (a) Satish reached at 08 : 50 − 00 : 20 = 8 : 30
1st time at = 12 : 30 pm He was 30 min earlier than the man who came
2nd time at = 1 : 20 pm 40 min late. It implies that Satish was 10 min
(12 : 30 + 50 min) late.
3rd time at = 2 : 30 pm (1 : 20 + 70 min) ∴ Scheduled time of the meeting
4th time at = 4 pm ( 2 : 30 + 90 min) = 8 : 30 − 0 : 10
5th time at = 4 : 00 pm + 110 min = 8 : 20
= 4 : 00 + 1 : 50 = 5 : 50 pm 45. (e) 10th from left 10th from right

A
38. (a) Given, 972, 682, 189, 298, 751 Rajan

M
Suraj
Make Changes +2 +1

H
Changed Series 974, 682, 190, 298, 751

TT
∴Required Difference = 9 − 4 = 5
39. (d) After interchange 927, 628, 198, 289, 715 A
W
Rajan Suraj
∴Required Sum = 1 + 7 = 8
SH

27th from left


972, 682, 189, 298 , 751
40. (c) Series +1 −1 New position of Rajan from right = (27 − 10) + 10
A

972, 682, 190, 298, 750 [using formula 4(b)]


@

∴Required Difference = 9 − 5 = 4 = 27
41. (a) The product of second and third digit of each 46. (c) The arrangement can be drawn as
number is as follows.
13 8 8 16
821 ⇒ 2 × 1 = 2
547 ⇒ 4 × 7 = 28
Front End
452 ⇒ 5 × 2 = 10 X Z Y
14 17
935 ⇒ 3 × 5 = 15
368 ⇒ 6 × 8 = 48 So, position of Z will be after (14+8)
∴ Second highest product is 28 and number is = 22nd place.
547. Thus, Z is at 23rd position
42. (a) The difference of first and second digit of each ∴ There are 8 persons sitting between Mr. X and
number is as follows Mr. Z.
821 ⇒ 8 − 2 = 6 > 1 47. (d) According to the question,
547 ⇒ 5 − 4 = 1 < 7 WK P
452 ⇒ 5 − 4 = 1 < 2 North
facing
935 ⇒ 9 − 3 = 6 > 5
368 ⇒ 6 − 3 = 3 < 8 Clearly, 7 people stood after W.
Chapter 10 NUMBER, RANKING AND TIME SEQUENCE TEST 107

48. (a) Anil reached the place on Friday and he was Q The others members were late by 30 min
three days earlier than the scheduled day. ∴Scheduled time of interview
Therefore, the scheduled day = (12 : 40) − 30 min
= Friday + 3 days = Monday = 12 : 10 h
If he had reached on Sunday, then he would Solution (Q. Nos. 53-55) By given information,
have been earlier by one day.
D/B > E > C …(i)
49. (c) 7 10 17 31 49 and A > D/B …(ii)
Anil Rohit Sumit Vijay
Now, from Eqs. (i) and (ii), we have
A > D/B > E > C
∴Total number of students in the class It is given that A is not the longest wire.
= 49 + 3 = 52 Hence, F would be the longest wire.
50. (d) Vivek is 18th from the right end and Gagan is ∴ F > A > D/B > E > C …(iii)
3rd to the right of Vivek. So, Gagan is 15th right F =13cm
end. There are 15 boys to the left of Aman,
E = 4 cm
10 boys between Aman and Gagan and 14 boys
to the right of Gagan 53. (b) Given, F = 13 cm
10 ∴Length of D = F − 5

A
15 2 14 = 13 − 5 = 8 cm

M
A V G
15th 54. (c) It is clear from Eq. (iii), wire C has the least

H
16th 18th length.

TT
∴Number of boys in the row 55. (b) It is clear from Eq. (iii), wire C has the least
= 15 + 1 + 10 + 1 + 14 = 41 length, so length of wire C would be 2 cm.
51. (d) (14) Tom A
Solution (Q. Nos. 56-58) From given information,
W
(13) Karan L > J> N …(i)
N> M> O
SH

(12) Sonali and …(ii)


(11) Bharti On combining Eqs. (i) and (ii), we have
(9) Rahul L > J> N > M> O
A

…(iii)
@

As per the question,


L does not hold the most number of flowers.
∴ K > L > J> N > M> O …(iv)
56. (d) It is clear from Eq. (iv) that J has the third highest
number of flowers.
Total people = M + N − 1 57. (e) One person holds more flower than L.
15 = 12 + N − 1 58. (d) Given, M hold 16 flowers.
∴ N=4 and difference between the flowers of M and
Hence, Sonali is fourth from beginning of the J = 12
queue. ∴Number of flowers J holds = 16 + 12 = 28
52. (a) The chairman of the selection committee arrived Hence, N has the number of flowers between 16
at (12 : 30) − 10 min = 12 : 20 h and 28.
Since, he was earlier than other members, so So, as per option (d), number between 16 and
other members arrived at 12 : 20 + 20 min 28 is 22. Hence, N holds 22 flowers.
= 12 : 40 h
How to Crack Test of Reasoning VERBAL

11
Blood Relations
Blood Relations means any relation Relations from maternal side
between two or more persons which is Relation Relation name
acquired by them by the virtue of their
Mother’s father Maternal grandfather
birth.

A
Mother’s mother Maternal grandmother
Questions Based on ‘Blood Relations’ are

M
related to our day-to-day life. In the Mother’s brother Uncle

H
questions examiner defines the simple Mother’s sister Aunt
relationship by using complicated set of

TT
Children of maternal uncle or Aunt Cousin
definitions and candidates have to
Wife of maternal uncle Aunt
comprehend these definitions quickly.
Husband of maternal aunt A Uncle
W
In order to solve these questions one
should have an adequate knowledge of
SH

Other Relations
‘blood relations’. These relations may be
divided into two main categories. Relation Relation name
A

(i) Maternal Relations Relations on Grandfather’s or Grandmother’s son Father or Uncle


@

the mother’s side are called Grandfather’s or Grandmother’s only son Father
maternal relations.
Grandfather’s or Grandmother’s daughter Aunt
(ii) Paternal Relations Relations on
Mother’s or father’s son Brother
the father’s side are called paternal
relations. Mother’s or father’s daughter Sister
Son’s wife Daughter-in-law
Types of Relations
Husband’s or wife’s sister Sister-in-law
Relations from paternal side
Relation Relation name
Relation Relation name
Daughter’s husband Son-in-law
Father’s father Grandfather
Husband’s or wife’s brother Brother-in-law
Father’s mother Grandmother
Brother’s son Nephew
Father’s brother Uncle
Brother’s daughter Niece
Father’s sister Aunt
Sister’s husband Brother-in-law
Children of uncle or aunt Cousin
Brother’s wife Sister-in-law
Wife of uncle Aunt
Grandson’s or Granddaughter’s Great granddaughter or son
Husband of aunt Uncle daughter or son
Chapter 11 BLOOD RELATIONS 109

Relations from One Generation to Next Generation


Generation I Generation II Generation III

Grandfather, grandmother, Mother, father, mother-in-law, Self, sister, sister-in-law,


maternal grandfather, father-in-law, uncle, aunt, brother, brother-in-law
maternal grandmother maternal uncle, maternal aunt cousin, husband, wife

Generation VI Generation V Generation IV

Great Grandson, Grandson and granddaughter Son, daughter, nephew, niece,


Great Granddaughter daughter-in-law, son-in-law

Types of Questions
There are three types of questions based on blood relations which are generally asked
in various competitive examinations.

A
M
Type 1 Approach to Draw a

H
Relation Diagram
Deciphering Jumbled-Up

TT
To solve this type of questions the approach
Descriptions you need to follow is ‘‘Last to First’’.
In this type of questions, a person gives A
It means begining with last information and
W
information to the another person either. In a proceed backward.
SH

photograph or infront of him. Using this e.g. Pointing to a man, Rahul said ‘‘The boy playing
information a relation is established to identify cricket is the son of the my father's wife. ‘‘How that
the person that has been described. man related to Rahul?
A

Now, let us draw the relation diagram.


@

(i) Rahul's father's wife means Rahul's Mother.


Important Notations/Symbols to Solve r Husband-wife Mothers
Blood Relation Questions Father
Given below are some standard symbols and
notations, they will help you to solve the questions
by expressing the given information in the form of a Rahul
relation tree. (ii) Son of Rahul’s Mother is his brother
+ Father-Daughter r Husband-wife
+ Male Father s
Mother
– Female
– Son
+ – Husband-Wife
+ + Brother-Brother – Mother-Son Brother
Rahul r
– – Sister-Sister
– Brother-Sister + Illustration 1. Pointing to a man, a lady said ‘‘His
+
+ Father-Son – Mother-Daughter mother is the only daughter of my mother’’.
How is the lady related to the man?
+ – SSC (CPO) 2014
(a) Mother (b) Daughter (c) Sister (d) Aunt
110 How to Crack Test of Reasoning VERBAL

Solution (a) The only daughter of Mother (–)


lady’s mother is the lady herself.
Only
Type 2
So, she is the mother of the man. daughter
By relation tree Relation Puzzles
Lady (–)
It is clear from the given diagram In this type of questions, more than one
that the lady is the mother of the Mother information in the form of puzzle is given and
man. based on the information, it is required to
Man (+) deduce the relation between the two
Illustration 2. Q’s mother is the individuals which is asked by drawing a
sister of P and daughter of M. family chart.
S is the daughter of P and sister of T. How is M
related to T? Approach to Draw a Relation
(a) Grandmother (b) Father Diagram
(c) Grandfather To solve this type of questions, divide the
(d) Grandfather or Grandmother given information in smaller parts and then
Solution (d) We can summarise the information in a draw the relation diagram.
family tree e.g. In a family, there are six members- A, B, C, D, E

A
Either and F. A and B is a married couple, A being a male
M gran

M
dfa member. D is the only son of C, who is the brother
th er
Daughter of A. E is the sister of D. B is the daughter-in-law of

H
or
Sister g F, whose husband has died. How is F related to A?

TT
s P
ra
nd

Now, let us draw the family diagram,


mo

Mother Daughter (i) ‘A and B’ is a married couple, A being a male


A
the

Sister member. So, B will be the female member


r

W
Q Ss T
r s
A B
SH

Hence, M is either the grandfather or the Married Couple


grandmother of T.
(ii) D is the only son of C, who is the brother of A.
A

Illustration 3. Introducing a man to her r Brother r Married Couple s


@

husband, a woman said, ‘‘His brother’s A B


C
father is the only son of my grandfather’’. Only son
How is the woman related to the man”? r
SBI (PO) 2014
D
(a) Mother-in-law (b) Sister (iii) E is the sister of D.
(c) Daughter (d) Sister-in-law r Brother r Married Couple s
(e) None of these A B
C
Solution (b) According to the question, Only son
+ r Sister
Grandfather D E
s
Only son (iv) B is the daughter-in-law of F, whose husband has
+ died. It means F is a female member.
Father
Da Fs
ug hte
+ + r s Daughter-in-law
Man Brother Woman r Brother r Married Couple s
Brother A B
Sister C
Only son
The only son of woman’s grandfather will be r Sister
D s
woman’s father, who is the father of man’s brother E
and therefore of man also. Thus, the woman is the From the above diagram it is clear that ‘F’ is the
sister of that man. mother of A.
Chapter 11 BLOOD RELATIONS 111

Directions (Illustrations 4-6) Read the following Illustration 9. Which of these is a pair of
information and answer the questions given below. brothers?
CLAT 2015
(a) P and X (b) P and Z (c) Q and X (d) R and Y
Anita is the niece of Prateek’s mother. Anita’s
mother is Prateek’s aunt. Rohan is Anita’s mother’s Solutions (Illustrations 7-9) According to the given
brother. Rohan’s mother is Anita’s grandmother. information, we can draw a relation diagram is as
shown below
Illustration 4. Rohan’s mother is ...... of Anita’s married
mother. Y+ R+ couple P– Z+

(a) Aunt (b) Mother (c) No relation (d) Sister


Q+ X–
Illustration 5. Prateek’s and Anita’s mother are
(b) Z is brother-in-law of R
(a) Cousin sister (b) Sister-in-law
(b) X and P are females.
(c) Friends (d) Sisters
(d) R and Y is the pair of brothers.
Illustration 6. Rohan is Prateek’s
(a) Brother (b) Brother-in-law Type 3
(c) Uncle (d) Cousin brothers
Coded Relations

A
Solutions (Illustrations 4-6) According to the given

M
information, we can draw a relation diagram is as In this type of questions, certain symbols and

H
shown below codes are used to represent the information
s
Mother in coded form. The candidate is required to

TT
Sisters decode the information with the help of
Mothers Brother
s
Mother Rohanr
Aunt A
symbols to be used for making family
diagram and then find out the relationship
W
c le
Un er between the persons asked in the question.
Niece oth
dm
SH

Prateek Anita G ran


Cousin Approach to Draw a Relation
A

(b) It is clear from the diagram that Rohan’s Diagram


@

mother is mother of Anita’s mother.


To solve this type of questions firstly we have
(d) Prateek’s and Anita’s mother are sisters. to decode the given information and then
(c) Rohan is Prateek’s uncle. apply them in the question asked.
Directions (Illustrations 7-9) Read the following e.g. P + Q means P is the daughter of Q. P − Q means
information carefully to answer the questions that P is the wife of Q; P × Q means P is the sister of Q
follow. ; P ÷ Q means P is the father of Q. How ‘M’ is
related to ‘Q’ in M – N ÷ Q?
A family consists of six members P, Q, R, X, Y and
Z. Q is the son of R but R is not the mother of Q. P Here, (i) P + Q ⇒ Q
and R are married couple. Y is the brother of R. X Daughter
is the daughter of P and Z is the brother of P. s
P
Illustration 7. Who is the brother-in-law of R? (ii) P − Q ⇒ s Wife r
P Q
(a) P (b) Z
(iii) P × Q ⇒ s Sister
(c) Y (d) X P Q
(iv) P ÷ Q ⇒ r
Illustration 8. How many female members are P
there in the family? Father
(a) One (b) Two
(c) Three (d) Four Q
112 How to Crack Test of Reasoning VERBAL

Now, applying the above information in the question Illustration 11. Which of the following represents
asked. Given equation M − N ÷ Q
‘J is son of F’?
(v) Now, M − N ⇒ M Wife Nr
s
(a) J ÷ R − T × F (b) J + R − T × F
and N ÷ Q ⇒ N
r (c) J ÷ M − N × F (d) Cannot be determined
(e) None of these
Father
Solution (e) From option (a), J ÷ R − T × F
Q
r s r
(vi) Now, combining the above diagrams, we have J R T
s Wife r Brother Sister
M N Father

Father F
Mother
Q From option (b), J + R – T × F
It is clear from the above diagram that M is the J
s R
mother of Q. Mother
Directions (Illustrations 10-12) Following So, initially it is proved that J is female, since J is
questions are based on the information given the mother of R. So, J cannot be a son of F.

A
below. From option (c), J ÷ M − N × F

M
(i) ‘P × Q’ means ‘P is father of Q’. r
J
s
M
r N
‘P − Q’ means ‘P is sister of Q’.

H
(ii) Brother Sister
Father
(iii) ‘P + Q’ means ‘P is mother of Q’.

TT
(iv) ‘P ÷ Q’ means ‘P is brother of Q’. F
Illustration 10. In the expression B + D × M ÷ N, A
None of the options shows that J is son of F.
W
how is M related to B? Illustration 12. Which of the following represents
SH

(a) Granddaughter ‘R is niece of M’?


(b) Son
(a) M ÷K × T − R
(c) Grandson
A

(b) M−J+R−N
(d) Granddaughter or Grandson
@

(e) None of the above (c) R − M × T ÷W


(d) Cannot be determined
Solution (c) B + D × M ÷ N (e) None of these
s
B Solution (b) Option (a) does not clarify the gender of R.
Hence, it is not possible.
Mother
Now, option (b) M − J + R − N
r s s
Grandson D M J
Sister
Father Mother
r
M N Niece s
Brother R N
Sister
So, M is grandson of B.
So, R is niece of M.
Chapter 11 BLOOD RELATIONS 113

Let us Practice
Base Level Exercise
Looking at a woman sitting next to him, Pointing to a photograph a man said ‘I have
Amit said, “She is the sister of the husband no brother or sister but that man’s father is
of my wife”. How is the woman related to my father’s son’. Whose photograph was it?
Amit? MPPSC 2018 OPSC 2018
(a) Daughter (b) Sister (c) Wife (d) Niece (a) His son’s (b) His own
A woman introduces a man as the son of (c) His father’s (d) His nephew’s
the brother of her mother. How is the man A man said to a woman, ‘‘The only son of
related to the woman? MAT 2013 your brother, is the brother of my wife.’’ How
(a) Nephew (b) Son is that woman related to the wife of that man?
(c) Cousin (d) Uncle SSC (10+2) 2013
(a) Aunt (b) Sister

A
Introducing a boy, a girl says, He is the son (c) Mother (d) Grandmother

M
of the only sister of my mother's brother'.
Looking at the photo of a man, Sunil said,

H
How is the boy related to that girl?
SSC (10+2) 2017 ‘‘His mother is the wife of my father’s son.

TT
(a) Father-in-law (b) Brother Brothers and sisters, I have none.’’ How the
(c) Cousin (d) Niece man in the photo is related to Sunil?
A and B is a married couple. C and D are A CGPSC 2014
W
(a) Sunil’s son
brothers. C is the brother of A. How is D (b) Sunil’s uncle
SH

related to B? SSC (Steno) 2016 (c) Sunil’s nephew


(a) Brother-in-law (b) Brother (d) Sunil’s cousin
A

(c) Son-in-law (d) Cousin (e) None of the above


@

Pointing to a gentleman, Deepak said, “His In a family of five persons, Dinesh is Jairam’s
only brother is the father of my daughter’s son and Gopal’s brother while Meeta is
father.” How is the gentleman related to Gopal’s mother and Jayanti’s daughter. If
Deepak? there are no step brothers or half brothers in
(a) Grandfather (b) Father the family, which of the following statements
(c) Brother-in-law (d) Uncle is true? IB (ACIO) 2012
(e) None of these (a) Jayanti is Dinesh’s mother
If Neena says, “Anita’s father Raman is the (b) Meeta is Dinesh’s mother
only son of my father-in-law, Mahipal”, (c) Jayanti is Jairam’s grandmother
then how is Bindu, who is the sister of (d) All of the above
Anita, related to Mahipal? MAT 2013
Deepak said to Nitin, “That boy playing with
(a) Niece (b) Daughter
the football is the younger of the two brothers
(c) Wife (d) Granddaughter
of the daughter of my father’s wife”. How is
Pointing towards a photo, Rakesh said, the boy playing football related to Deepak?
“She is the daughter of the only son of my CLAT 2013
grandfather.“ How is the girl related to (a) Son
Rakesh? SSC (CGL) 2016 (b) Brother
(a) Sister (b) Daughter (c) Cousin
(c) Grand Daughter (d) Cousin (d) Brother-in-law
114 How to Crack Test of Reasoning VERBAL

Ravi was showing a photograph to his A family consists of a man, his wife, his three
friend, Gopi. Pointing at a boy in the sons, their wives and three children in each
photograph, Ravi said ‘The boy sitting at son’s family. How many members are there in
the left is the son of the wife of the only son the family? SSC (10+2) 2014
of the grandmother of my younger brother‘. (a) 12 (b) 13 (c) 15 (d) 17
What is the relation between the boy in the
If A is the son of Q, Q and Y are sisters, Z is
photograph and Ravi? CLAT 2017
the mother of Y, P is the son of Z, then which
(a) Nephew and uncle
of the following statements is correct?
(b) Ravi's brother-in-law IB (ACIO) 2017
(c) First Cousins (a) P is maternal uncle of A(b) P and Y are sisters.
(d) Brothers (c) A and P are cousins. (d) None of the above
Raghu and Babu are twins. Babu’s sister is Given that,
Reema. Reema’s husband is Rajan. Raghu’s I. A is the brother of B
mother is Lakshmi. Lakshmi’s husband is II. C is the father of A.
Rajesh. How is Rajesh related to Rajan?
SSC (10+2) 2012 III. D is the brother of E.
(a) Uncle (b) Son-in-law IV. E is the daughter of B.
(c) Father-in-law (d) Cousin Then, the uncle of D is UPSC (CSAT) 2013

A
A is the father of B, C is the brother of A, F (a) A (b) B (c) C (d) E

M
is the sister of B. If M is the father of A, then If the uncle of the father of Rani is the

H
establish relationship between F and C. grandson of the father of Anup and Anup is

TT
WBPSC 2018
the only son of his father, then how Anup is
(a) Daughter and Father (b) Husband and Wife
related to Rani? BSSC (CGL) 2015
(c) Brother and Sister (d) Niece and Uncle
(a) Grandfather A (b) Uncle
W
S is the only son of V.V is married to R. M is (c) Maternal Uncle (d) Great grandfather
the daughter of R. R is the grandmother of
SH

A, Q, Y, Z are different persons. Z is the


A. How is S definitely related to A?
SBI (PO) 2017 father of Q. A is the daughter of Y and Y is
A

(a) Uncle the son of Z. If P is the son of Y and B is the


@

(b) Cannot be determined brother of P, then CGPSC 2012


(c) Father (a) B and Y are brothers (b) A is the sister of B
(d) Brother (c) Z is the uncle of B (d) Q and Y are brothers
(e) Sister (e) None of the above
Annu is daughter of my mother’s brother Q’s mother is sister of P and daughter of M.S
Abahi. Pari is granddaughter of my mother. is the daughter of P and Sister of T. How is M
Pari should call Annu as CLAT 2013 related to T? OPSC 2018
(a) Maternal Aunt (b) Sister (a) Father (b) Grandfather
(c) Cousin (d) Niece (c) Grandmother
(d) Maternal grandfather or grandmother
A is wife of B. D is brother of A. P and Q
are the children of E, who is wife of D. How Directions (Q. Nos. 26 and 27) Study the
is B related to Q? NlCL (AO) 2017 following information carefully and answer the
(a) Father (b) Grandmother given questions. LIC (AAO) 2016
(c) Uncle (d) Father-in-law A is the mother of both U and T. T is the sister of
(e) Cousin J. J is the son of M. P is married to U. U is the
daughter-in-law of K.
Javed’s brother’s sister’s father’s brother is
Karim and Karim’s daughter is Dolly. How How is P related to M?
is Javed related to Dolly? (a) Son-in-law (b) Nephew
UP PSC (RO) 2014 (c) Son (d) Cannot be determined
(a) Brother (b) Cousin (c) Sister (d) Uncle (e) Cousin
Chapter 11 BLOOD RELATIONS 115

If R is the husband of K, then how is K If ‘A $ B’ means that ‘A is the father of B’;


related to P? ‘A★ B’ means that ‘A is the mother of B’;
(a) Cannot be determined (b) Mother-in-law ‘A @ B’ means that ‘A is the wife of B’, then
(c) Grandmother (d) Sister-in-law which of the following means that ‘M is the
(e) Mother grandmother of N’? CLAT 2015

If ‘P + Q’ means ‘P is the husband of Q’, (a) M ★ R $ T @ N (b) M ★ R @ T @ N


‘P/Q’ means ‘P is the sister of Q’ and ‘P × Q’ (c) M ★ T $ N @ R (d) M $ T $ N @ R
means ‘P is son of Q’, which of the following Directions (Q. Nos. 32 and 33) Read the
shows ‘A is the daughter of B’? following information carefully and answer the
OPSC 2018
questions which follow.
(a) A/D × B (b) D × B + C/A
(c) B + C × A (d) C × B/A (i) ‘P × Q’ means ‘P is brother of Q’.
(ii) ‘P − Q’ means ‘P is mother of Q’.
A3P means A is mother of P. A4P means A is
(iii) ‘P + Q’ means ‘P is father of Q’.
brother of P. A9P means A is husband of P.
A5P means A is daughter of P. Which of the (iv) ‘P Q’ means ‘P is sister of Q’.
following means that K is the mother-in-law Which of the following means ‘M is niece of
of M and J is the brother-in-law of M? N’?

A
MPPSC 2018 (a) M × R − N (b) N ÷ J + M ÷ D
(a) M9N3K4J (b) M9N5K3J (c) N ÷ J + M (d) N × J − M ÷ D

M
(c) K5J9M3N (d) None of these (e) None of these

H
If A × B means A is the brother of B and Which of the following means B is the

TT
A + B means that A is the father of B then grandfather of F?
which one means that 'M is the nephew of (a) B + J − F (b) B − J + F
N’? DSSSB 2017 (c) B × T − FA (d) B ÷ T +
W
(a) N + M × K (b) N × K + M F
(c) M × K + N
SH

(d) None of these (e) None of these


A

Expert Level Exercise


@

A woman going with a boy is asked by T, S and R are three brothers. T’s son Q is
another woman about the relationship married to K and they have one child Rahul
between them. The woman replied, “My blessed to them. M, the son of S is married to H
maternal uncle and the uncle of his maternal and this couple is blessed with a daughter
uncle is the same.” How is the lady related Madhvi. R has a daughter N who is married to
to that boy? P. This couple has one daughter Karuna born to
(a) Grandmother and Grandson them. How is Madhvi related to S? MAT 2013
(b) Mother and son (a) Daughter (b) Niece
(c) Aunt and Nephew (c) Granddaughter (d) None of these
(d) Cannot be determined Directions (Q. Nos. 4-6) Study the following
(e) None of the above information and answer the given questions.
There are six members in a family. A is the SBI (PO) Pre 2015
father of D, E is the grandfather of D. B is the T is the sister of D. D is married to P. P is the son
daughter-in-law of C. F is the uncle of D. of M. T is the mother of J. Y is the father of U. Y
What is the relationship of C with F? has only one son and only one daughter. U is the
(a) Sister MAT 2013 daughter of T. Q is the son of D.
(b) Mother-in-Law How is P related to T?
(c) Nephew (a) Brother (b) Uncle
(d) Data inadequate (c) Brother-in-law (d) Cousin brother
(e) Cannot be determined
116 How to Crack Test of Reasoning VERBAL

How is J related to D? Directions (Q. Nos. 13-14) Study the following


(a) Son (b) Niece information and answer the given questions.
(c) Son-in-law (d) Nephew SBI (Clerk) 2016
(e) Daughter S is the father of R. R is the only son of U. U is the
daughter of J. J has only two children. K is the son
How is Q related to M? of J.
(a) Son-in-law (b) Grandson
(c) Nephew (d) Son If X is the grand-daughter of J, then how is U
(e) Cannot be determined related to X?
(a) Sister (b) Mother-in-law
Directions (Q. Nos. 7-9) Study the following (c) Grandfather (d) Sister-in-law
information and answer the given questions. (e) Cannot be determined
IBPS PO (Pre) 2016
D is daughter of N, E is wife of N. G is sister of How is S related to K?
D. C is married to G. N has no son. K is mother (a) Brother (b) Brother-in-law
of E. Q is only daughter of C. (c) Cousin (d) Father
(e) Uncle
How Q is related to D?
(a) Daughter (b) Cousin Read the following information carefully to
(c) Niece (d) Sister-in-law choose best option for the question CLAT 2013

A
(e) Cannnot be determined ‘L % M’ means that ‘M is brother of L’.

M
How N is related to K? ‘L × M’ means that ‘L is the mother of M’.

H
(a) Brother-in-law (b) Cousin ‘L ÷ M’ means that ‘L is the sister of M’.

TT
(c) Son-in-law (d) Sister ‘L = M’ means that ‘M is father of L’.
(e) Brother Which of the following means ‘I is the
How many daughters N have? nephew of Q’? A
W
(a) One I. Q % J = I II. Q ÷ M × B % I
III. C ÷ I = B % Q
SH

(b) Three
(c) Two (a) Only III (b) Only I
(d) Cannot be determined (c) Only II (d) None of these
A

(e) None of the above


@

Directions (Q. Nos. 16-19) Study the following


Directions (Q. Nos. 10-12) Study the given information carefully to answer these questions.
information carefully to answer the given Delhi Police (Constable) 2009
questions. IBPS (PO) 2016
(i) ‘P H Q’ means ‘P is father of Q’.
D is the son of P. P is the daughter of M. S is
(ii) ‘P # Q’ means ‘P is sister of Q’.
married to M. R and H are children of S. W is the
husband of H. K is the son of W. (iii) ‘P + Q’ means ‘P is brother of Q’.
(iv) ‘P − Q’ means ‘P is mother of Q’.
If M does not have any son, then how is R
related to D? (v) ‘P/Q’ means ‘P is son of Q’.
(a) Uncle (b) Son-in-law (vi) ‘P = Q’ means ‘P is daughter of Q’.
(c) Sister (d) Brother If Z H X − Y is given, then which of the
(e) Aunt following is true?
How is D related to K? (a) Z is maternal uncle of Y
(a) Father-in-law (b) Cousin (b) Z is maternal grandfather of Y
(c) Grandfather (d) Uncle (c) X is nephew of Z
(e) Brother-in-law (d) Z is grandmother of Y
How is W related to P? Which of the following means ‘Z has two
(a) Brother-in-law (b) Uncle children’?
(c) Father (d) Father-in-law (a) Z + Y + X (b) Z H Y # X
(e) Grandfather (c) Z # Y − X (d) Z + X/Y
Chapter 11 BLOOD RELATIONS 117

Which of the following means ‘X is the What should come in place of the question
grandfather of Y’? mark, to establish that T is the sister-in-law
(a) X H Z H Y (b) Y H Z/X of Q in the expression?
(c) Z # X = Y (d) X + Y/Z R%T×P?Q+ V
Which of the following means ‘Y is the (a) ÷ (b) %
mother of X and Z’? (c) × (d) $
(a) X + Y − Z (b) Y − X/Z (e) Either $ or ×
(c) Y − X + Z (d) Y + X H Z Directions (Q. Nos. 23-24) Read the following
Directions (Q. Nos. 20-22) Read the following information carefully and answer the questions
information carefully and answer the following which follow. RBI Astt. (Mains) 2017
questions. SBI (PO) 2013 If ‘A × B’ means ‘A is the son of B’.
(i) If ‘A + B’ means ‘A is the father of B’. If ‘A + B’ means ‘A is the father of B’.
(ii) If ‘A × B’ means ‘A is the sister of B’. If ‘A > B’ means ‘A is the daughter of B’.
If ‘A < B’ means ‘A is the wife of B’.
(iii) If ‘A $ B’ means ‘A is the wife of B’.
(iv) If ‘A % B’ means ‘A is the mother of B’. Which of the following pairs of people
represent first cousins with regard to the
(v) If ‘A ÷ B’ means ‘A is the son of B’.
relations given in the expressions, if it is

A
Which among the following options is true, provided that A is the sister of J:‘L>V<J+P’

M
if the expression ‘I + T % J × L ÷ K’ is and ‘S × A<D+F<E+K’.

H
definitely true? (a) LP (b) SP
(a) L is the daughter of T

TT
(c) SK (d) SF
(b) K is the son-in-law of I (e) Can not be determined
(c) I is the grandmother of L
(d) T is the father of J A
What will come in the place of the question
W
(e) J is the brother of L
mark, if it is provided that M is the
grandmother of F in the experssion
SH

Which among the following expressions is ‘F × R < S? M’.


true, if Y is the son of X is definitely false? (a) > (b) <
A

(a) W % L × T × Y ÷ X (b) W + L × T × Y÷ X (c) + (d) ×


(c) X + L × T × Y ÷ W (d) W $ X + L + Y + T
@

(e) Can not be determined


(e) W % X + T × Y ÷ L

Answers
Base Level Exercise
1. (b) 2. (c) 3. (b) 4. (a) 5. (d) 6. (d) 7. (a) 8. (a) 9. (a) 10. (a)
11. (b) 12. (b) 13. (d) 14. (c) 15. (d) 16. (b) 17. (a) 18. (c) 19. (b) 20. (d)
21. (a) 22. (a) 23. (d) 24. (b) 25. (d) 26. (a) 27. (e) 28. (a) 29. (d) 30. (d)
31. (c) 32. (b) 33. (a)

Expert Level Exercise


1. (c) 2. (d) 3. (c) 4. (c) 5. (d) 6. (b) 7. (c) 8. (c) 9. (c) 10. (e)
11. (b) 12. (a) 13. (e) 14. (b) 15. (c) 16. (b) 17. (b) 18. (a) 19. (c) 20. (b)
21. (d) 22. (d) 23. (b) 24. (d)
118 How to Crack Test of Reasoning VERBAL

Answer with Explanations


Base Level Exercise
(b) “Husband of my wife” means Amit himself. So, the (a) That man’s father’s son means that man himself
woman sitting next to Amit is the sister of Amit. or we can say, that this man is the father of the
person in the photograph. Hence, the man is
(c) According to the question,
pointing towards his son’s photograph.
Brother Woman’s mother
X (a) According to the question,
Son Brother r
Woman Womans
Man Only
Aunt
So, from above, the man is the cousin of woman. Wife son
Manr
s Brother r
(b) Girl’s mother’s brother’s only sister is her mother
and the son of her mother is her brother. Brother-in-law

A
(a) From the given information, So, woman is aunt to the wife of that man.

M
Married (a) According to the question,
+ Brother + Brother Couple

H
D C A B Fatherr

TT
Brother-in-law Wife
Mothers Sunil r
∴D is brother-in-law of B. A Son
W
(d) According to the question, Man in photographr
SH

Gentlemanr Xr Since, Sunil has no brother and sister. So, Sunil’s


Brother father’s son is Sunil himself. Thus, the mother of
Father
A

the man in photograph is the wife of Sunil.


@

Uncle Deepakr Hence, the man in the photograph is Sunil’s


son.
Daughter Father
(b) For making the relation tree
Y Jayantis
Coup Daughter
Hence, gentleman is Deepak’s uncle. Jairamr le
Meetas
(d) According to the question, Son r
the
Mo Mother
Mahipalr Fa Gopalr
th er- Dineshr
Brother
Granddaughter

- la in
Father w
r Couple From above relation tree and given information,
Raman Neenas
we can say that Meeta is Dinesh’s mother.
Father
(b) According to the question,
Sister
Anita Bindus s Wife r
X Deepak’s father
Daughter
So, Bindu is granddaughter of Mahipal. r Brother s
(Boy) Z Y Deepakr
(a) It is clear from given information that only son of
Rakesh's grandfather is Rakesh's father and his Brother
daughter is the sister of Rakesh. Hence, girl in
So, the boy is brother of Deepak.
photograph is the sister of Rakesh.
Chapter 11 BLOOD RELATIONS 119

(d) The blood relation is as follows (c) The relation diagram is shown as follows
s s Couple r Brother s Couple r
E D A B
Mother

Grandmother
s Wife r
Children
Son P Q Uncle
Father
r r So, B is uncle of Q.
Ravi Boy
Brother (b) According to the question,
Hence, boy and Ravi are brothers. r
g hte Karimr
Brother
Z
r
(c) According to the question, Dau
s Husband
Lakshmi Rajeshr Father
Fa
r

th
the

er-
r
Dolly s Javed
r
X Y
Mo

Sister
in

Father Brother
-la
w

Sister Couple Cousin


Raghur Babur Reemas Rajanr
Brother Husband
Hence, Javed is the cousin of Dolly.

A
(d) According to the question,

M
From relation tree, we can say that Rajesh is the
Father Mother 2

H
father-in-law of Rajan.

TT
(d) We can summarise the information in a family tree
+
M
Father wife A son son wife son wife 6
W
+Brother
A C+
SH

son son son son son son son son son 9


Father Niece
Total members in the family = 2 + 6 + 9 = 17
A

Sister
B F–
(a) From the given information,
@

From the above diagram it is clear that, F is the


daughter of A and C is the brother of A. Hence, C s
Z
is the uncle of F while F is the niece of C. Son
Mother
(b) From given information, relation diagram is as below
s Sister s r
Q Y P
r Married Couple s Brother
V R
Son
Son Daughter
r
A
r s
S M Q P is the brother of Q.
A Grandmother ∴P is the maternal uncle of A.
S might be the Father or Uncle of A. (a) According to the statements,
So, relation of S with A is cannot be determined. ⊕
C
(a) According to the question, Father
Brother ⊕ Brother B
Abahir My Mothers A
Granddaughter

Daughter Daughter
Brother ⊕
Myself E D
Annus
Daughter Uncle
Pari s
From above, A is the Uncle of D.
So, Pari should call Annu her maternal aunt.
120 How to Crack Test of Reasoning VERBAL

(d) According to the question, (a) P is son-in-law of M.


Fatherr Father (e) If R is husband of K.

Grandson
Grandson + Husband K–
R
Anupr Anup
Uncler
Son Son Son Mother
Uncle
r
he

Fatherr +
at

P
df
an
gr

Father So, K is the mother of P.


at

Ranis
re
G

(a) From option (a) A/D × B


Rani
B
Since, Anup is the only son of his father and ter
uncle of Rani’s father is grand son of Anup’s u gh
Da Son
father. So, he is the son of Anup. Therefore,
Anup is Rani’s father’s uncle’s father. Hence, he Sister ⊕
A D
is the great-grandfather of Rani.
(b) According to the question, So, A is the daughter of B.

A
(d) From option (a), M9N3K4J
Zr

M
Married
So Couple
n M⊕

H
Father N

TT
Q r
Y Son Son Mother
Daughter
J A K⊕
W
s Brother r Brother
A Pr B
Sister Here, K is the son of M and J is either the son or
SH

daughter of M.
From above diagram, it is clear that A is sister of B.
From option (b), M9N5K3J
A

(d) According to the question,


@

w K
-la
M Grandfather r-in
or Grandmother the daughter
Daughter Mo
Married Mother
Mother Couple
M N J
of Q Sister P
Daughter
s Sister
S T Brother-in-law or Sister-in-law

From options, option (d) is correct. Here, J is the brother-in-law or sister-in-law of M.


From option (c), K5J9M3N
Solutions (Q. Nos. 26 and 27) On the basis of given
information, the blood relation diagram is as follows Married
r Couple s
J M
Couple
M+ A– w K Daughter Mother
-la
M
ot r-in
hte
s
h g K N
Son Mother er u Son
Da
Here, J is the husband of M.
Sister – Sister – Couple +
J+ T U P Hence, option (d) is correct.
Chapter 11 BLOOD RELATIONS 121

(d) From option (a) N + M × K (a) B is grandfather of F. So, B should be male. Hence,
⊕ options (b) and (d) are redundant. Now, by taking
N option (a).
Son (a) B + J – F r
Father B
Father
⊕ Brother
M K s
Grandfather J
From option (b), N × K + M
⊕ Brother ⊕
Mother
N K
F
Hence, option (a) is true.
Nephew
or Niece
M Expert Level Exercise
N is nephew or niece of M. From option (c), (c) According to the question,
M×K+N Brother
⊕ Brother K P Z
M
Mother Uncle

A
Father Brother
Woman NeYp X

M
Uncle
N hew
Mother

H
Thus, none of the above is correct option. Aunt

TT
Boy
(c) From option (c), M H T $ N @ R
M ★ T → M is the mother of T Hence, their relationship is Aunt and Nephew.
T $ N → T is the father of N A
W
(d) Cannot be determine, since the relation of B and C is
N @ R → N is the wife of R
not given with other family members. Hence, data is
SH

From the given information, we can make a inadequate.


relation diagram
(c) According to the question,
A

s
M
Brother Brother
@
Grandmother

Mother T ( +) S ( +) R ( +)
r
T Son Son Daughter
(–) ( +) ( +) (–) (–)
Father K Q M H N P ( +)
s Wife r Son Daughter Daughter
N R
So, M is the grandmother of N. Rahul Madhvi(–) Karuna (–)
(b) M is niece of N. So, M is female. Here,
option (a) is redundant. Also, gender of M From the above generation tree, we see that Madhvi
cannot be determined using options (c). is the granddaughter of S.
Now, option (b), i.e. N ÷ J + M ÷ D
Solutions (Q. Nos. 4-6) According to the information given in
s r
N J the question, the relation chart is given below
Sister
Father M
Niece s (+)
M D (–) Sister (–) (+)
Sister Y T D P

So, option (b) represents that M is the niece (–) (+) (+)
of N. U J Q
122 How to Crack Test of Reasoning VERBAL

(c) P is the husband of T’s sister, i.e. brother-in-law (e) We cannot determined the answer as X might be
of T. the daughter or niece of U.
(d) J is the son of D’s sister, therefore J is the (b) It is clear from the diagram that S is the
nephew of D. brother-in-law of K.
(b) Q is the son of M’s son and thus Q is the
(c) I. Q % J = I +
grandson of M. I
Solutions (Q. Nos. 7-9) Making relation diagram as per Father
the given information, + Brother
J Q
s
K Here, I is the father of Q.
II. Q ÷ M × B % I
Mother
– Sister –
s Q M
E Nr Mother
Ne
Father ph Brother
ew B
D
s Gs Cr
Sister
So, I is the nephew of Q.

A
Daughter
III. C ÷ I = B % Q

M
Niece Qs + Brother +

H
B Q
(c) Q is niece of D.

TT
(c) N is son-in-law of K. Father
– Sister
(c) N have two daughters. C
A I
W
Solutions (Q. Nos. 10-12) The relationship diagram is Here, the gender of I is not defined.
as follows Hence, only statement II means ‘I is the nephew
SH

M S Da of Q’.
u
Daughter

gh
te (b) According to the question,
A

r
Z ★X − Y
@

P– R– H– W+ Z
r
Sister Sister
Son Son Father

D+ K+ Grandfather
X
s

(e) If M does not have any son then R is a female. Mother


So, R is the sister of D’s mother.
Hence, R is the aunt of D. Y
Hence, option (b) is true.
(b) K is D’s aunt's son. Hence, K is the cousin of D.
(b) According to the question,
(a) W is P’s sister’s husband. Hence, W is the
(a) Z + Y + X
brother-in-law of P.
r r
Z Y X
Solutions (Q. Nos. 13-14) From given information Brother Brother
relation diagram is as follows. (b) Z ★ Y # X
Grand daughter
J r
Z
Daughter Father
Son
Sr Us s
Y X
Son Kr Sister

Rr X
s Clearly, Z has two children X and Y.
Chapter 11 BLOOD RELATIONS 123

(c) Z # Y − X (d) From option (d),


s s W$X+L+Y+T
Z Y
Sister
Mother
s Wife
W Xr
X
Father
(d) Z + X/Y Y r
L
Son

Z
r
X
r Father
Brother r
Y
(a) According to the given information,
Father
From option (a), X H Z H Y T
r
X So, expression W $ X + L + Y + T is definitely
Father false.
Grandfather

r
Z (d) From option (d), R % T X P $ Q + V
Father s
R
Y

A
Hence, X is grandfather of Y. Mother

M
s s Wife r
T

H
(c) From option (a), X + Y – Z Sister P Q

TT
r s Father
X Y
Brother
Mother Sister-in-law V
A
W
Z So, T is the sister-in-law of Q in the given
expression.
SH

From option (b), Y–X/Z


(b)
s s r Sister s r
Y Z V J A D
A

Mother Daughter Son Daughter


@

X
r Son Ls P S
r
F
s
E
r

From option (c), Y–X + Z


K
s
Y Hence, S and P are first cousins.
Mother (d) F × R < S ? M
X
r Z Here,
Brother
s
Hence, Y is mother of X and Z. M

(b) From option (b), Son


I + T % J X L ÷K
er

s r
R S
oth

Son-in-law
Ir
dm
an

Father
Gr

s Couple r F
r
T K
Mother Son Q S is the son of M.
Sister
J
s
Lr ∴ S × M is correct.
So, K is son-in-law of I. Hence, ‘?’= ‘×’.
124 How to Crack Test of Reasoning VERBAL

12
Mathematical
Operations

A
Mathematical operations are basically the

M
simplification of an expression containing Types of Questions

H
numbers and different Mathematical
There are four types of questions based on

TT
operations.
mathematical operations which are usually
In this chapter, we deal with questions asked in various competitive examinations.
having four fundamental Mathematical A
W
operations– addition (+), subtraction (–),
multiplication (×) and division (÷) and also Type 1
SH

statements such as ‘less than (<)’, ‘greater Problem Solving by Substitution


than (>)’, ‘equal to (=)’, ‘not equal to (≠)’, etc.
A

All these operations are represented by In this type of questions, you have substitutes
@

symbols, different from usual ones. The for various mathematical symbols or numerals
candidate is required to substitute the real followed by a question involving calculation of
signs in place of artificial symbols to solve the an expression or choosing the correct/incorrect
questions. equation.
We need to follow the VBODMAS rule for Illustration 1. If + means ÷,– means ×, × means
simplification of Mathematical operations. +, ÷ means –, then give the value for
This rule gives us the correct order, in which 45 + 9 – 3 × 15 ÷ 2 BSSC (CGL) 2015
various operations regarding simplification are (a) 40 (b) 36 (c) 56 (d) 28
to be performed. Order of various operations is Solution (d) Given, 45 + 9 − 3 × 15 ÷ 2
as same as the order of letters in the
Replacing the proper signs in the given expression,
‘VBODMAS’.
we have 45 ÷ 9 × 3 + 15 – 2
Letter Rule VBODMAS Using VBODMAS rule,
Do First V Vinculum — or bar 5 × 3 + 15 − 2 = 15 + 15 − 2 = 30 − 2 = 28
B Brackets ( ){}[]
Illustration 2. If ‘K’ means ‘minus’, ‘L’ means
O Of ×
‘divided by’, ‘M’ means ‘plus’ and ‘D’ means
D Division ÷
‘multiplied by’, then
M Multiplication × 117 L 3 K 5 M 12 D 8 = ? SSC (10+2) 2016
A Addition + (a) 150 (b) 125
Do Last S Subtraction – (c) 130 (d) 145
Chapter 12 MATHEMATICAL OPERATIONS 125

Solution (c) Given, 117 L 3 K 5 M 12 D 8 = ? Solution (a) Given, 4 − 8 + 12 = 0


Now, change the alphabet into the signs, On interchanging signs ‘+’ and ‘−’ and numbers 4
⇒ ? = 117 ÷ 3 − 5 + 12 × 8 and 8 in option (a), we get
117 8 + 4 − 12 = 0 ⇒ 12 − 12 = 0 ⇒ 0 = 0
⇒ ?= − 5 + 12 × 8 ⇒ ? = 39 − 5 + 96
3 Illustration 5. Which one of the four
⇒ ? = 135 − 5 ⇒ ? =130 interchanges in signs and numbers would
make the given equation correct?
Illustration 3. If ‘–’ stands for ‘+’, ‘+’ stands for ‘ 6 × 4 + 2 = 16
×’, ‘×’ stands for ‘–’, then which one of the (a) + and × , 2 and 4 (b) + and × , 4 and 6
following is not correct? SSC (CPO) 2014 (c) + and × , 2 and 6 (d) None of these
(a) 22 + 7 – 3 × 9 = 148 (b) 33 × 5 − 10 + 20 = 228
Solution (b) Given, 6 × 4 + 2 = 16
(c) 7 + 28 − 3 × 52 = 127 (d) 44 − 9 + 6 × 11 = 87
On interchanging signs ‘+’ and ‘×’ and number
Solution (c) From option (a), 4 and 6,
LHS = 22 + 7 − 3 × 9 = 22 × 7 + 3 − 9 4 + 6 × 2 = 4 + 12 = 16
= 154 + 3 – 9 = 157 – 9 = 148 = RHS
From option (b), Type 3
LHS = 33 × 5 − 10 + 20 = 33 − 5 + 10 × 20

A
= 33 − 5 + 200 = 233 − 5 = 228 = RHS
Miscellaneous Patterns

M
From option (c),
In this type, questions are based on balancing

H
the equations by replacing the actual
LHS = 7 + 28 − 3 × 52 = 7 × 28 + 3 − 52

TT
mathematical signs with some other signs and
= 196 + 3 − 52 = 199 − 52 tricks.
= 147 ≠ 127 (RHS)
From option (d), A
Illustration 6. If the following equation has to
W
be balance, then the signs of which of the
LHS = 44 − 9 + 6 × 11 = 44 + 9 × 6 − 11 following options will be used?
SH

= 44 + 54 − 11 = 98 − 11 = 87 = RHS 24 6 12 16 = 0
Hence, equation in option (c) is not correct. (a) ÷, + and – (b) − , ÷ and +
A

(c) − ,− and − (d) ÷, + and ÷


@

Type 2 Solution (a) From option (a), 24 ÷ 6 + 12 − 16 = 0


24
Interchanging the Signs LHS = + 12 − 16 = 4 + 12 − 16 = 16 − 16 = 0
6
and Numbers ∴ LHS = RHS
In this type of questions, certain signs or Illustration 7. If 5 × 4 = 15 , 7 × 8 = 49 and
numbers interchange among each other. The 6 × 5 = 24, then 8 × 4 is equal to
candidate is required to change the given (a) 24 (b) 26 (c) 28 (d) 30
signs or numbers with each other and select Solution (a) As, 5 × 4 = 5 × ( 4 − 1) = 5 × 3 = 15
the equation which is correct from the given
7 × 8 = 7 × (8 − 1) = 7 × 7 = 49
alternatives.
and 6 × 5 = 6 × (5 − 1) = 6 × 4 = 24
It can also be asked to find the correct pair of
signs and numbers to be interchanged from Similarly, 8 × 4 = 8 × ( 4 − 1) = 8 × 3 = 24
the alternatives to make the given equation Illustration 8. If 64 × 52 = 17, 48 × 56 = 23 and
correct.
74 × 35 = 19, then 84 × 37 = ?
Illustration 4. If signs ‘+’ and ‘−’ and numbers (a) 32 (b) 28 (c) 22 (d) 20
4 and 8 interchange between each other, then Solution (c) As, 64 × 52 ⇒ (6 + 4) + (5 + 2) = 17
which one of the following four equations 48 × 56 ⇒ ( 4 + 8) + (5 + 6) = 23
would be correct? and 74 × 35 ⇒ (7 + 4) + (3 + 5) = 19
(a) 4 − 8 + 12 = 0 (b) 8 − 4 ÷ 12 = 8
(c) 4 ÷ 8 − 12 = 16 (d) 8 ÷ 4 − 12 = 24 Similarly, 84 × 37 ⇒ (8 + 4) + (3 + 7) = 22
126 How to Crack Test of Reasoning VERBAL

Type 4 Directions (Illustrations 12-14) In the


following questions, the symbols δ, @, ©, % and
Simple and Coded Inequalities are used with the meaning as indicated below.
In this type of questions, certain relationship n
‘P © Q’ means ‘P is not smaller than Q’.
between different sets of elements is given n
‘P % Q’ means ‘P is neither smaller than
(in terms of ‘less than’, ‘greater than’, or nor equal to Q’.
‘equal to’), using either the real symbols or n
‘P H Q’ means ‘P is neither greater than
substituted symbols. The candidate is required
nor equal to Q’.
to read the given statements and then choose
which of the conclusions is/are definitely true.
n
‘P δ Q’ means ‘P is not greater than Q’.
‘P @ Q’ means ‘P is neither greater than
n

Directions (Illustrations 9-11) In each of the


following questions, relationship between different nor smaller than Q’.
element is shown in the statements. These Now, in each of the following questions
statements are followed by two conclusions. assuming the given statements to be true, find
SBI (PO) 2014 which of the four conclusions I, II, III and IV
Give Answer given below them is/are definitely true, give
(a) If only conclusion I follows your answer accordingly.

A
(b) If only conclusion II follows
Illustration 12.

M
(c) If either conclusion I or II follows
Statements D δ T, T @ R, R © M, M % K

H
(d) If neither conclusion I nor II follows
Conclusions I. R @ D

TT
(e) If both conclusions I and II follow II. R % D
III. K H T
Illustration 9. Statements P ≥ Q = R, Q > S > T
Conclusions I. P ≥ T II. T < Q
A
IV. M δ T
W
(a) Only either I or II is true
Solution (b) On combining the two statements, we get (b) III and IV are true
SH

P≥ Q= R > S> T (c) Either I or II and III are true


Conclusions I. P ≥ T (false) (d) Either I or II and IV are true
A

(e) Either I or II and III and IV are true


II. T < Q (true)
@

So, only conclusion II follows. Illustration 13.


Statements J @ F, F δ N, N % H, H  G
Illustration 10. Statements A ≤ B = C, Conclusions I. G ★ N II. N  J
D> C= E
III. F ★ J IV. J δ G
Conclusions I. E ≥ A II. A < D
(a) I and II are true
Solution (e) On combining the two statements, we get (b) I, II and III are true
A≤ B= C= E< D (c) I, III and IV are true
Conclusions I. E ≥ A (true) (d) All I, II, III and IV are true
II. D > A (true) (e) None of the above
So, both conclusions I and II follow. Illustration 14.
Statements R H K, K % D, D @ V, V δ M
Illustration 11. Statements F ≥ G = H, G > J ≥ K
Conclusions I. R H D II. V H R
Conclusions I. F ≥ K II. K < H
III. D @ M IV. M % D
Solution (b) On combining the two statements, we get (a) None is true
F≥G =H>J ≥K (b) Only III is true
Conclusions I. F ≥ K (false) (c) Only IV is true
II. K < H (true) (d) Either III or IV is true
So, only conclusion II follows. (e) Either III or IV and II are true
Chapter 12 MATHEMATICAL OPERATIONS 127

Solutions (Illustrations 12-14) H  G ⇒ H≥ G


⇒≥ So, J = F≤ N> H≥ G
% ⇒> Now, conclusions
H⇒< I. G H N ⇒ G < N (true)
δ ⇒≤ II. N  J ⇒ N ≥ J (true)
@⇒= III. F H J ⇒ F < J (false)
IV. J δ G ⇒ J ≤ G (false)
12. (e) Here, D δ T ⇒ D ≤ T
Hence, I and II are true.
T@R⇒T=R
R  M ⇒ R≥ M 14. (d) Here, R H K ⇒ R < K
M % K ⇒ M> K K%D⇒K>D
So, D≤ T = R≥ M> K D@V⇒D=V
Now, conclusions VδM ⇒ V≤ M
I. R @ D ⇒ R = D (may be) So, R < K > D = V ≤ M
II. R % D ⇒ R > D (may be)
Now, conclusions
III. K H T ⇒ K < T (true)
I. R H D ⇒ R < D (false)
IV. M δ T ⇒ M ≤ T (true)
II. V H R ⇒ V < R (false)

A
From first two conclusions only one can be true.
Hence, either I or II and III and IV are true. III. D @ M ⇒ D = M (may be)

M
13. (a) Here, J @ F ⇒ J = F IV. M % D ⇒ M > D (may be)

H
FδN ⇒ F≤ N Hence, either III or IV is true.

TT
N % H ⇒ N> H

A
Let us Practice
W
SH

Base Level Exercise


A
@

1. If ‘+’ means ‘÷’, ‘×’ means ‘–’, ‘÷’ means ‘+’ 5. If the mathematical signs interchange from
and ‘–’ means ‘×’, then 16 ÷ 8 × 6 − 2 + 12 = ? – to +, + to ÷, × to − and ÷ to ×, then find out
DSSSB 2017 the correct answer of the given equation
(a) 23 (b) 24 (c) 17 (d) 22 6÷ 8 + 2 × 5 − 8 = ? SSC (CPO) 2013
(a) 27 (b) 18 (c) 32 (d) 28
2. If + means ‘subtraction’, × means ‘division’,
‘÷’ means ‘addition’ and ‘−’ means 6. If P denotes ‘÷’, Q denotes ‘×’, R denotes ‘+’
‘multiplication’, then and S denotes ‘−’, then
15 × 5 + 78 − 5 ÷ 4 = ? SSC (10+2) 2017 18 Q 12 P 4 R 5 S 6 = ? SSC (CGL) 2012
(a) − 383 (b) − 460 (a) 95 (b) 53 (c) 51 (d) 57
(c) − 475 (d) −743
7. In a certain code language, ‘÷’ represents ‘+’,
3. If + means ‘×’, − means ‘÷’, ‘×’ means ‘−’ and ‘–’ represents ‘×’ ‘+’ represents ‘÷’ and
÷ means ‘+’, then find the value of the given ‘×’ represents ‘–’. Find out the answer to the
equation 6 + 64 − 8 ÷ 45 × 8 = ? SSC (CGL) 2016 given question 8 − 14 + 7 × 10 ÷ 9 = ?
(a) 85 (b) 76 (c) 87 (d) 75 SSC (10 + 2) 2017
(a) 15 (b) 28 (c) 20 (d) 13
4. If ‘×’ means subtraction; ‘−’ means division,
‘o’ means addition and ‘%’ means 8. In a certain code language, ‘×’ represents ‘+’,
multiplication, then find the value of ‘÷’ represents ‘×’, ‘–’ represents ‘÷’ and ‘+’
13 o 3 × 6 % 8 − 4 o 14=? SSC (CGL) 2016 represents ‘–’. Find out the answer to the
(a) 18 (b) 14 given question 25 ÷ 2 − 10 + 10 × 6 = ?
SSC (10 + 2) 2017
(c) 12 (d) 8 (a) 1 (b) 21 (c) 19 (d) 20
128 How to Crack Test of Reasoning VERBAL

9. If ‘a’ represents ÷, ‘b’ represents +, ‘c’ Directions (Q. Nos. 17-19) Correct the following
represents −, and ‘d’ represents ×, then equations by interchanging the two signs.
24a 6d 4b 9c 8 = ? SSC (CPO) 2015 17. 51 ÷ 3 × 12 − 6 + 3 = 201
(a) 20 (b) 19 (a) + and ÷ (b) − and + (c) × and ÷ (d) − and ×
(c) 6 (d) 17 (e) None of these
10. If ‘>’ stands for division’, ‘∨’ stands for 18. 16 + 4 ÷ 2 − 21 × 7 = 21
‘multiplication’, ‘–’ stands for ‘greater than’,
(a) + and − (b) + and × (c) − and ÷ (d) × and ÷
‘×’ stands for ‘less than’, ‘<’ stands for
(e) None of these
‘addition’, ‘∧’ stands for ‘subtraction’, ‘+’
stands for ‘equal to ’, then which of the 19. 3 2 × 22 ÷ ( 62 − 9) + 12 = 33
following equations is correct? SSC (CGL) 2014 (a) − and ÷ (b) + and ×
(a) 10 ∧ 3 < 5 < 4 + 16 (b) 4 ∧ 6 < 4 < 4 + 16 (c) × and ÷ (d) + and –
(c) 12 ∧ 3 < 5 < 4 + 12 (d) 11 ∧ 12 < 5 < 4 + 12 (e) None of these
11. If ‘+’ stands for division; × stands for 20. The following equation is incorrect. Which
addition, ‘−’ stands for multiplication, ‘÷’ two signs should be interchanged to correct
stands for subtraction, which of the the equation?
following equation is correct? WBSC 2018 16 × 8 − 4 ÷ 9 + 14 = 37

A
(a) 36 + 6 − 3 × 5 ÷ 3 = 24
(a) + and × (b) + and × (c) ÷ and – (d) – and +

M
(b) 36 × 6 + 7 ÷ 2 − 6 = 20
(c) 36 ÷ 6 + 3 × 5 − 3 = 40 21. The following equation is incorrect. Which

H
two signs should be interchanged to correct
(d) 36 − 6 + 3 × 5 ÷ 3 = 74

TT
the equation?
12. If + , −, ×, ÷, =, > and < are represented as 8 + 2 − 40 ÷ 5 × 16 = 44
δ , •, γ , η , ω , β and α respectively, then which
A
(a) − and + (b) ÷ and × (c) × and − (d) + and ÷
W
of the following is correct? SSC (CPO) 2012
(a) 3 γ 6 η 2 δ 8 • 4 ω 5 (b) 3 η 6 γ 2 δ 8 • 4 β 5 Directions (Q. Nos. 22-23) In the following
SH

questions, which one of the four interchanges in


(c) 3 γ 6 • 2 δ 8 η 4 α 5 (d) 3 δ 6 • 2 γ 8 η 4 ω 5
signs and numbers would make the given
13. Select the correct combination of equation correct?
A

mathematical signs to replace ‘★’ signs and 22. 4 × 6 − 2 = 14


@

to balance the following equation.


(a) × to ÷, 2 and 4 (b) − to +, 2 and 6
6 ★ 4 ★ 12 ★ 12 SSC (Multitasking) 2014
(c) × to +, 4 and 8 (d) − to ÷, 2 and 6
(a) ÷ , − , = (b) + , − , ÷ (c) = , − , ÷ (d) × , − , =
23. (6 ÷ 2) × 3 = 0
14. Which of the following sets of operations (a) × to −, 2 and 6 (b) ÷ and × , 2 and 3
with the usual notations replacing the stars
(c) × to −, 2 and 3 (d) ÷ and × , 2 and 6
in the order given makes the statements
valid? SSC (Steno) 2013 Directions (Q. Nos. 24-27) If the given
100★ 16★ 225★ 1 interchanges are made in signs and numbers,
(a) ×, =, + (b) +, =, − (c) +, =, × (d) −, ×, = which one of the four equations would be correct?
15. Select the correct combination of 24. Given interchanges : signs ‘+’ and ‘−’;
mathematical signs to replace ‘’’ signs and numbers ‘5’ and ‘8’
to balance the given equation. (a) 82 − 35 + 55 = 2 (b) 82 − 35 + 55 = 102
(c) 85 − 38 + 85 = 132 (d) 52 − 35 + 55 = 72
24 ’ 34 ’ 2 ’ 5 ’ 12 SSC (10+2) 2013
(a) + ÷ × = (b) = ÷ + − (c) = ÷ − + (d) + ÷ = × 25. Given interchanges : signs ‘+’ and ‘×’;
16. Select the correct combination of numbers ‘3’ and ‘7’
mathematical signs to replace ‘’’ signs and (a) 23 + 17 × 73 = 1241
to balance the following equation. (b) 37 + 73 × 12 = 112
(8 ’ 7 ’ 6) ’ 5 ’ 10 SSC (CGL) 2014 (c) 23 × 17 + 37 = 428
(a) × − ÷ = (b) − × ÷ + (c) + − ÷ × (d) × + = ÷ (d) 23 + 17 × 73 = 388
Chapter 12 MATHEMATICAL OPERATIONS 129

26. Given interchanges : signs ‘×’ and ‘÷’; 38. If + , −,|, φ × and 1stand respectively for
numbers ‘4’ and ‘9’ greater than, not less than, less than, not
(a) 94 × 7 ÷ 47 = 324 (b) 94 × 7 ÷ 97 = 329 greater than, equal to and not equal to, then
(c) 49 × 7 ÷ 49 = 7 (d) 47 × 9 ÷ 94 = 18 x 1 y + z may give UPPSC (RO) 2014
(a) x + y 1 z (b) x × y| z
27. Given interchanges : signs ‘−’ and ‘÷’;
numbers ‘2’ and ‘6’ (c) x 1 y φ z (d) x − y φ z
(a) 32 − 12 ÷ 6 = 30 (b) 36 ÷ 12 – 2 = 1
(c) 32 − 16 ÷ 6 = 1 (d) 36 − 12 ÷ 2 = 30 Expert Level Exercise
28. If 17 × 64 = 4716, 28 × 34 = 4823, then Directions (Q. Nos 1-5) In these questions,
76 × 84 = ? DSSSB 2017 relationship between different elements is shown
(a) 6487 (b) 8764 (c) 4678 (d) 4768 in the statements. The statements are followed by
29. If 9 × 3 + 8 = 24, 10 × 2 + 7 = 35 and conclusions I and II. Study the conclusions based
on the given statements and select the appropriate
80 × 40 + 3 = 6, then find the value of
option. IBPS RRB (PO) 2016
12 × 4 + 3 = ? WBPCS 2018
(a) 7 (b) 9 (c) 12 (d) 16 Given Answer
(a) If only conclusion I is true
30. Some equations are solved on the basis of a

A
(b) If only conclusion II is true
certain system. On the same basis, find out (c) If both conclusions are true

M
the correct answer for the unsolved equation. (d) If none of the conclusions is true

H
4 − 5 − 1 = 514, 3 − 5 − 6 = 563, 0 − 6 − 8 = ? (e) If either conclusion I or II is true

TT
SSC (CGL) 2015 1. Statements P < E < T < R; T > K
(a) 860 (b) 680 (c) 806 (d) 068 Conclusions I. K > P II. R > K
31. If 4 H 2 @ 3 = 6, 18 H 6 @ 4 = 12, then what A
2. Statements X < W; A > C > H = W
W
will be the value of 24 H 3 @ 7 = ?
Conclusions I. C > X II. A > W
SH

(a) 21 (b) 27 (c) 72 (d) 56


3. Statements B < N < D; K = R < D < W
32. If 2 & 6 % 7 = 33, 4 & 6 % 8 = 54, then what Conclusions I. N = R II. B < W
A

will be the value of 6 & 8 % 9=?


@

(a) 95 (b) 77 (c) 99 (d) None 4. Statements J < L < B < S > Y < M
Conclusions I. J < M II. L ≥ Y
33. If 16 − 2 = 2, 9 − 3 = 0, 81 − 1 = 8, then what is
64 − 4 = ? BSSC (CGL) 2015
5. Statements B < N < D; K = R < D < W
(a) 4 (b) 2 (c) 6 (d) 8 Conclusions I. W > K II. N ≥ W
34. Change symbol and solve accordingly to find Directions (Q.Nos. 6-10) In the following
correct answer from the alternatives given questions, relationships between different
below 9 × 8 × 7 = 24, 4 × 7 × 3 = 14, 2 × 1 × 9 = ? elements is shown in the statements. The
SSC (CPO) 2015 statements are followed by conclusions. Study the
(a) 12 (b) 10 (c) 18 (d) 11 conclusions based on the given statements and
35. If 879 = 8, 625 = 1 and 586 = 9, then 785 = ? select the appropriate answer. IBPS PO (Pre) 2016

(a) 6 (b) 7 (c) 8 (d) 9 Give answer


(a) If only conclusion II is true
36. If given, 12 (169) 5, 4 (20) 2 and 1 (26) 5, then (b) If only conclusion I is true
7 (?) 3 (c) If both conclusions are true
(a) 78 (b) 68 (c) 58 (d) 36 (d) If either conclusion I or II are true
(e) If neither conclusion I nor II is true
37. Let J = 1, K = 2, L = 5, M = 7, N = 11, O = 13
6. Statements S < L < I= P > E > R; L > Q
and P = 17. Find the letter to be inserted in
Conclusions I. P > S II. I >R
the box in the relation given.
(N × 1+ M) ÷ K = 31 SSC (10+2) 2014 7. Statements G > R < E = A < T < S; D < A < J
(a) L (b) P (c) J (d) O Conclusions I. T > D II. R > S
130 How to Crack Test of Reasoning VERBAL

8. Statements A > B > C < D < E < F 17. Which of the following symbols should be
Conclusions I. A > E II. C < F placed in the blank spaces respectively (in
the same order from left to right) in order to
9. Statements G > R > E = A < T ≤ S; D ≤ A ≤ J complete the given expression in such a
Conclusions I. J > G II. J = G manner that makes the expressions ‘B > N’
10. Statements S < L < I = P > E > R; L > Q as well as ‘D ≤ L’ definitely true?
Conclusions I. L < R II. E > Q B __ L __ O __ N __ D
11. In which of the given expressions, does the (a) =, =, ≥, ≥ (b) >, ≥, =, >
expression ‘C < P’ definitely holds false? (c) >, <, = , ≤ (d) >, =, =, ≥
SBI (PO) Pre 2017 (e) >, =, ≥, >
(a) P > A > L < E; C> L > O > N
18. Which of the following should be placed in
(b) P < A < L > E; C > L < O < N the blank spaces respectively (in the same
(c) P = A > L = E; C = L > O < N order from left to right) in order to complete
(d) P > A > L > E; C < L < O < N the given expression in such a manner that
(e) P = A > L < E; C < L > O > N makes the expression ‘A < P’ definitely false?
__ ≤ __ < __ > __
12. Which of the following expression will be
(a) L, N, P,A (b) L, A, P, N
true if given expression K ≥ G > H ≤ F is

A
definitely true? SBI (PO) 2014 (c) A, L, P, N (d) N, A, P, L

M
(a) F ≥ K (b) H < K (c) F < G (d) K ≥ H (e) P, N, A, L

H
(e) None of these
19. Which of the following symbols should be

TT
13. Which of the following expression will be placed in the blank spaces respectively (in
false when given expression K < H ≤ J = D ≥ the same order from left to right) in order to
Q is definitely true? SBI (PO) 2014 A
complete the given expression in such a
W
(a) K < D (b) D ≤ H (c) J ≥ Q (d) J > K manner that makes the expression ‘F > N’
(e) None of these and ‘U > D’ definitely false?
SH

14. Which of the following expression will be F__ O __ U __ N ___ D


(a) <, <, >, = (b) <, =, =, >
A

true when the given expressions W > Z = L,


X ≥ F > W are definitely true? SBI (PO) 2014 (c) <, =, =, < (d) ≥, =, =, ≥
@

(a) L = F (b) Z ≥ F (c) F > L (d) Z > X (e) >, >, =, <
(e) L ≥ X
Directions (Q. Nos. 20-22) In the following
Directions (Q. Nos. 15-19) Read each statement questions, the symbols @, ©, $, % and H are used
carefully and answer the following questions. with the following meanings as illustrated below.
SBI (PO) 2013 RBI Assistant (Mains) 2017

15. Which of the following expressions will be ‘P © Q’ means ‘P is not greater than Q’.
true, if the expression R > O = A > S < T is ‘P % Q’ means ‘ P is not smaller than Q’.
definitely true? ‘P H Q’ means ‘P is neither smaller than nor
(a) O > T (b) S < R (c) T > A (d) S = O equal to Q’.
(e) T < R ‘P @ Q’ means ‘ P is neither greater than nor
16. Which of the following symbols should equal to Q’.
replace the question mark (?) in the given ‘P $ Q’ means ‘P is neither greater than nor
expression in order to make the expressions smaller than Q’.
‘P > A’ as well as ‘T < L’ definitely true? Now, in each of the following questions,
P>L?A≥N=T assuming the given statements to be true, find
(a) ≤ (b) > (c) < (d) ≥ which of the two conclusions I and II given
(e) Either ≤ or < below them is/are definitely true.
Chapter 12 MATHEMATICAL OPERATIONS 131

Give answer 23. Statements R # J, J $ D, D @ K, K % T


(a) If only conclusion I is true Conclusions I. T # D II. T @ D
(b) If only conclusion II is true III. R # K IV. J $ T
(c) If either conclusion I or II is true (a) Either I or II is true
(d) If neither conclusion I nor II is true (b) Only III is true
(e) If both conclusions I and II are true (c) III and IV are true
20. Statements K @ V, V © N, N % F (d) Either I or II and III are true
(e) None of the above
Conclusions I. F @ V II. K @ N
21. Statements H © W, W $ M, M @ B 24. Statements T % R, R $ M, M @ D, D © H
Conclusions I. D % R II. H # R
Conclusions I. B H H II. M % H
III. T © M IV. T % D
22. Statements D % B, B H T, T $ M (a) Only I is true (b) I and IV are true
Conclusions I. T © D II. M © D (c) II and IV are true (d) II and IV are true
(e) None of these
Directions (Q. Nos. 23-27) In the following
questions, the symbols @, $, #, © and % are used 25. Statements M @ B, B # N, N $ R, R © K
with the following meanings as illustrated below. Conclusions I. K # B II. R © B
IBPS (PO) 2012 III. M $ R IV. N © M

A
n
‘P $ Q’ means ‘P is not smaller than Q’. (a) I and III are true (b) I and II are true
(c) II and IV are true (d) II, III and IV are true

M
‘P © Q’ means ‘P is neither greater than nor
(e) None of these
equal to Q’.

H
26. Statements F # H, H @ M, M © E, E $ J
‘P # Q’ means ‘P is neither smaller than nor

TT
n

equal to Q’. Conclusions I. J © M II. E # H


III. M © F IV. F # E
n
‘P % Q’ means ‘P is not greater than Q’.
A
(a) I and II are true (b) II and III are true
W
n
‘P @ Q’ means ‘P is neither greater than nor (c) I, II and III are true (d) II, III and IV are true
smaller than Q’. (e) None of these
SH

Now, in each of the following questions 27. Statements D % A, A @ B, B0. © K, K % M


assuming the given statements to be true, find
A

Conclusions I. B $ D II. K #A
which of the four conclusions I, II, III and IV III. M # B IV. A © M
@

given below them is/are definitely true and (a) I, II and IV are true (b) I, II and III are true
give your answer accordingly. (c) II, III and IV are true (d) I, III and IV are true
(e) All are true

Answers
Base Level Exercise
1. (a) 2. (a) 3. (a) 4. (a) 5. (a) 6. (b) 7. (a) 8. (a) 9. (d) 10. (a)
11. (d) 12. (d) 13. (d) 14. (b) 15. (c) 16. (a) 17. (e) 18. (d) 19. (a) 20. (c)
21. (d) 22. (b) 23. (c) 24. (a) 25. (d) 26. (b) 27. (c) 28. (c) 29. (b) 30 (b)
31. (d) 32. (b) 33. (a) 34. (a) 35. (a) 36. (c) 37. (a) 38. (a)

Expert Level Exercise


1. (b) 2. (c) 3. (b) 4. (d) 5. (a) 6. (c) 7. (b) 8. (a) 9. (e) 10. (e)
11. (b) 12. (b) 13. (b) 14. (c) 15. (b) 16. (b) 17. (d) 18. (e) 19. (c) 20. (b)
21. (e) 22. (d) 23. (d) 24. (a) 25. (c) 26. (b) 27. (e)
How to Crack Test of Reasoning VERBAL

Answer with Explanations


Base Level Exercise
1. (a) 16 ÷ 8 × 6 − 2 + 12 9. (d) Given, ? = 24 a 6 d 4 b 9 c 8
Replacing the signs, 16 + 8 − 6 × 2 ÷ 12 ⇒ ? = 24 ÷ 6 × 4 + 9 − 8
6×2 ⇒ ?=
24
× 4+ 9−8
= 16 + 8 − = 16 + 8 − 1 = 24 − 1 = 23
12 6
2. (a) 15 × 5 + 78 − 5 ÷ 4 ⇒ ?=4×4+ 9−8
Now, changing the signs as per the questions, ⇒ ? = 16 + 9 − 8
15 ⇒ ? = 25 − 8 ⇒ ? = 17
− 78 × 5 + 4 = 3 − 390 + 4 = − 390 + 7 = − 383
5 10. (a) After changing the signs of equation given in
3. (a) Given equation, 6 + 64 − 8 ÷ 45 × 8 option (a) as per the question, we get
Now, changing the sign as per the question, 10 − 3 + 5 + 4 = 16
6 × 64 ⇒ 19 − 3 = 16 ⇒ 16 = 16
6 × 64 ÷ 8 + 45 − 8 = + 45 − 8
8 11. (d) On changing the signs, in option (d), we get

A
= 6 × 8 + 45 − 8 = 48 + 45 − 8 = 93 − 8 = 85 36 × 6 ÷ 3 + 5 − 3 = 74

M
4. (a) Given , ? =13 o 3 × 6% 8 − 4 o 14 ⇒ 36 × 2 + 5 − 3 = 74
⇒ 72 +5 − 3 = 74 ⇒ 77 − 3 = 74

H
Now, changing the symbols into the sign as per
question, ∴ 74 = 74

TT
? = 13 + 3 − 6 × 8 ÷ 4 + 14 12. (d) From option (d) 3δ 6• 2 γ 8 η 4ω 5
8
⇒ ? = 13 + 3 − 6 × + 14
4 A
3+ 6−2 × 8÷4= 5
W
⇒ ? = 13 + 3 − 12 + 14 ⇒ 9−2 ×2 = 5⇒ 5= 5
⇒ ? = 30 − 12
SH

13. (d) 6 H 4 H 12 H 12
⇒ ? =18 From option (d), 6 × 4 − 12 = 12 ⇒ 12 = 12
5. (a) 6 ÷ 8 + 2 × 5 − 8 = 6 × 8 ÷ 2 − 5 + 8
A

14. (b) From option (b), 100 + 16 = 225 − 1


=6×4−5+ 8
@

⇒ 10 + 4 = 15 − 1
= 24 − 5 + 8
⇒ 14 = 14
= 32 − 5 = 27
15. (c) From option (c), we have
6. (b) Given, 18 Q 12 P 4 R 5 S 6 24 = 34 ÷ 2 − 5 + 12
After changing the letters into signs as per given 24 = 17 − 5 + 12
in the question, we have
24 = 29 − 5
18 × 12 ÷ 4 + 5 − 6
12 24 = 24
= 18 × + 5 − 6 [by VBODMAS rule] 16. (a) Given equation is (8 H 7 H 6) H 5 H 10
4
= 18 × 3 + 5 − 6 = 54 + 5 − 6 Now, replacing H sign with the mathematical
= 59 − 6 = 53 signs in option (a),
7. (a) 8 − 14 + 7 × 10 ÷ 9 = 8 × 14 ÷ 7 − 10 + 9 (8 × 7 − 6) ÷ 5 = 10
= 8 × 2 − 10 + 9 = 16 − 10 + 9 ⇒ (56 − 6) ÷ 5 = 10 ⇒ 50 ÷ 5 = 10
= 25 − 10 = 15 ⇒ 10 = 10
8. (a) 25 ÷ 2 − 10 + 10 × 6 17. (e) None of the interchange is applicable in the
equation to produce number 201.
Replacing the signs,
18. (d) If we interchange the signs ‘×’ and ‘÷’, the
25 × 2 ÷ 10 − 10 + 6
25 equation becomes
= − 10 + 6 16 + 4 × 2 − 21 ÷ 7
5
= 5 − 10 + 6 = 11 − 10 = 1 = 16 + 8 − 3 = 21
Chapter 12 MATHEMATICAL OPERATIONS 133

19. (a) If we interchange symbols ‘−’ and ‘÷’, the 31. (d) Here, sign H implies division and sign @
equation becomes implies multiplication
36
9 × 4 − (36 ÷ 9) + 1 = 9 × 4 − +1 4 ÷ 2 × 3 = 6, 18 ÷ 6 × 4 = 12
9
= 36 − 4 + 1 = 33 Therefore, 24 ÷ 3 × 7 = 56
20. (c) From option (c), we have 32. (b) Here sign ‘&’ does not convey any meaning
16 × 8 ÷ 4 − 9 + 14 = 16 × 2 − 9 + 14 and sign ‘%’ signifies ‘+’
= 32 − 9 + 14 26 + 7 = 33, 46 + 8 = 54
= 46 − 9 = 37 Therefore, 68 + 9 = 77
21. (d) From option (d), we have 33. (a) As, 16 − 2 = 4 − 2 = 2,
8 ÷ 2 − 40 + 5 × 16 = 4 − 40 + 80 9−3=3−3=0
= 84 − 40 = 44 and 81 −1 = 9 − 1 = 8
22. (b) On interchanging ‘−’ to ‘+’ and 2 to 6, we get Similarly, 64 − 4 = 8 − 4 = 4
4 × 2 + 6 = 14 34. (a) As, 9 +8 + 7 = 24
⇒ 8 + 6 = 14 ⇒ 14 = 14 and 4 + 7 + 3 = 14
23. (c) On interchanging ‘×’ to ‘−’ and 2 to 3, we get Similarly, 2 + 1 + 9 = 12
(6 ÷ 3) − 2 = 0 ⇒ 2 − 2 = 0 ⇒ 0 = 0 Note Here, sign of ‘×’ is converted into ‘+’.

A
24. (a) (a) 52 + 38 − 88 = 2 (b) 52 + 38 − 88 ≠ 102 35. (a) As, 879 = (7 + 9) − 8 = 8

M
(c) 58 + 35 − 58 ≠ 132 (d) 82 + 38 − 88 ≠ 72 625 = (2 + 5) − 6 = 1

H
25. (d) (a) 27 × 13 + 37 ≠ 1241 (b) 73 × 37 + 12 ≠ 112 and 586 = (8 + 6) − 5 = 9

TT
Similarly, 785 = (8 + 5) − 7 = 6
(c) 27 + 13 × 73 ≠ 428 (d) 27 × 13 + 37 = 388
36. (c) As, 12 2 + 52 = 144 + 25 = 169
26. (b) (a) 49 ÷ 7 × 97 ≠ 324 (b) 49 ÷ 7 × 47 = 329
A
42 + 2 2 = 16 + 4 = 20
W
(c) 94 ÷ 7 × 94 ≠ 7 (d) 97 ÷ 4 × 49 ≠ 18
and 12 + 52 = 1 + 25 = 26
27. (c) (a) 36 ÷ 16 − 2 ≠ 30 (b) 32 − 16 ÷ 6 ≠ 1
SH

Similarly, 7 2 + 32 = 49 + 9 = 58
(c) 36 ÷ 12 − 2 = 1 (d) 32 ÷ 16 − 6 ≠ 30
37. (a) Given, (N × 1 + M) ÷ K = 31

A

28. (c) ↓
As, 17 × 64 = 4 7 1 6 (11 × 1 + 7 ) ÷ 2 = 31
@

11 × 1 + 7
⇒ = 31
2
28 × 34 = 4 8 2 3 ⇒ 11 × 1 + 7 = 62
and
⇒ 11 × 1 = 62 – 7
⇒ 11× 1 = 55
55
Similarly, 7 6 × 8 4 = 4 6 7 8 ⇒ 1= =5= L
11
38. (a) Symbol and given interpretation.
+ → greater than (>)
29. (b) As, (9 ÷ 3) × 8 = 24
− → not less than (≥)
(10 ÷ 2 ) × 7 = 35 | → less than (<)
and (80 ÷ 40) × 3 = 6
φ → not greater than (≤ )
Similarly, (12 ÷ 4) × 3 = 9
× → equal to (=) and
30. (b) As, 4 5 1 and 3 5 6
1 → not equal (≠ )
5 1 4 5 6 3 Hence, statement x 1 y + z
Similarly, 0 6 8 x ≠ y > z – (1)
Now, observing the options to check which of the
6 8 0 given may be valid
134 How to Crack Test of Reasoning VERBAL

Option (a) x > y ≠ z here x > y which is possible 10. (e) S ≤ L ≤ I = P ≥ E > R; and Q < L ≤ I = P ≥ E
because in the given statement (1) L>Q
x ≠ y, although x can also be less
than y and y ≠ z is also followed in
I. L < R (false)
y > z. II. E ≥ Q (false)
Option (b) x = y < z x = y is not possible because in Hence, neither conclusion I nor II is true.
statement 1 x ≠ y. 11. (b) P < A ≤ L ≥ E; C ≥ L ≤ O < N
Option (c) x ≠ y ≤ z x ≠ y is followed as in statement (1) Combine the above
but y ≤ z is not valid because y > z
is given in statement (1). P < A≤ L < C ⇒C > P
Option (d) x ≥ y ≤ z x ≥ y is not valid because statement (1) So, given expression in option (b) is definitely
say x ≠ y, y ≤ z cannot be true false.
because statement (1) say y > z. 12. (b) Here, we will check options one-by-one.
Hence, option (a) may be true and hence it is the (a) F ≥ K (it does not follow as no relation can be
answer. determined between F and K)
(b) H < K (it follows)
Expert Level Exercise (c) F < G (it does not follow as no relation can be
1. (b) P < E < T < R; P < E < T ≥ K found between F and G)
I. K > P (false)
(d) K ≥ H (it does not follow as K cannot be

A
II. R ≥ K (true) equal to H.)

M
2. (c) A > C ≥ H = W > X 13. (b) From the given statement K < H ≤ J = D ≥ Q

H
I. C > X (true)
Check each option one-by-one.

TT
II. A > W (true)
(a) K< D (true)
3. (b) K = R < D > N ≥ B and B ≤ N < D ≤ W
(b) D ≤ H (false)
I. N = R (false)
(c) J ≥ Q A (true)
W
II. B < W (true)
4. ( d) J ≤ L < B ≤ S > Y < M (d) J > K (true)
SH

I. J < M (false) So, only expression (b) is not true.


II. L ≥ Y (false) 14. (c) On combining the two expressions
A

5. (a) K = R < D > N ≥ B and B ≤ N < D ≤ W X≥ F> W> Z = L


@

I. W > K (true) Now, checking the expression one-by-one.


II. N ≥ W (false) (a) L = F (false)
6. (c) S ≤ L ≤ I = P ≥ E > R; L > Q (b) Z ≥ F (false)
I. P ≥ S (true) (c) F > L (true)
II. I > R (true) (d) Z > X (false)
Hence, both conlusions I and II are true. (e) L ≥ X (false)
7. (b) G > R ≤ E = A ≤ T ≤ S; and D ≤ A ≤ T So, only expression (c) is true.
I. T ≥ D (true) 15. (b) We will check each option one-by-one.
II. R > S (false) (a) O > T (false)
Hence, only conclusion I is true. (b) S < R (true)
8. ( a) A ≥ B > C ≤ D ≤ E < F (c) T > A (false)
I. A ≥ E (false) (d) S = O (false)
II. C < F (true)
(e) T < R (false)
Hence, only conclusion II is true.
16. (b) From option (b), P > L > A ≥ N = T
9. (e) G > R ≥ E = A ≤ T ≤ S; and G > R ≥ E = A ≤ J
I. J > G (false) Here, P > A and T < L both are definitely true.
II. J = G (false) 17. (d) From option (d), B > L = O = N ≥ D
Hence, neither conclusion I nor II is true. Here, B > N and D ≤ L both are definitely true.
Chapter 12 MATHEMATICAL OPERATIONS 135

18. (e) From option (e), P ≤ N < A > L 24. (a) Statements T % R ⇒ T < R
Here, A < P is definitely false. R $ M ⇒R> M
19. (c) From option (c), M @ D ⇒M = D
F<O=U=N<D D © H ⇒D < H
Here, F > N and U > D both are definitely false.
i.e. T<R>M =D < H
Solutions (Q. Nos. 20-22) According to the given
information, Conclusions I. D % R ⇒ D < R (true)
©→ < %→ > II. H # R ⇒ H > R (false)
H→ > @→ < III. T © M ⇒ T < M (false)
$→ =
IV. T % D ⇒ T < D (false)
20. (b) Statements K @ V→ K < V Hence, only conclusion I is definitely true.
V © N→ V≤N
25. (c) Statements M @ B ⇒ M = B,
N % F→ N≥F
B # N ⇒B > N
We have, K < V ≤ N ≥ F
N $ R ⇒ N > R,
Conclusions I. F @ V → F < V (false)
R © K ⇒ R<K
II. K @ N → K < N (true)
So, only Conclusion II is true. i.e. M = B > N> R < K
Conclusions I. K # B ⇒ K > B

A
21. (e) Statements H © W → H ≤ W (false)
II. R © B ⇒ R < B

M
W$M →W=M (true)
M @ B→ M < B III. M $ R ⇒ M ≥ R (false)

H
We have, H ≤ W = M < B IV. N © M ⇒ N < M (true)

TT
Conclusions I. B H H → B > H (true) Hence, II and IV are definitely true.
26. (b) Statements F # H ⇒ F > H
II. M % H → M > H (true)
A H @ M ⇒H = M
W
So, both Conclusions are true.
22. (d) Statements D % B → D ≥ B M © E ⇒M < E
SH

B H T→B > T E $ J ⇒E> J


T $ M→T = M i.e. F > H =M < E>J
A

We have, D ≥ B > T = M Conclusions I. J © M ⇒ J < M (false)


@

Conclusions I. T © D → T ≤ D (false) II. E # H ⇒ E > H (true)


II. M © D → M ≤ D (false)
III. M © F ⇒ M < F (true)
So, neither conclusion I nor II is true.
IV. F # E ⇒ F > E (false)
Solutions (Q. Nos. 23-26) $ ⇒ ≥,  ⇒ <, # → >, %
Hence, II and III are definitely true.
→ ≤, @ → =
27. (e) Statements D % A ⇒ D < A
23. (d) Statements R # J ⇒ R > J
A @ B ⇒A = B
J $ D ⇒J≥ D
B © K ⇒B < K
D @ K ⇒D = K
K % M ⇒K< M
K % T ⇒K≤ T
i.e. D<A =B < K<M
i.e. R > J≥ D = K≤ T
Conclusions I. T # D ⇒ T > D (may be) Conclusions I. B $ D ⇒ B > D (true)
II. T @ D ⇒ T = D (may be) II. K # A ⇒ K > A (true)
III. R # K ⇒ R > K (true) III. M # B ⇒ M > B (true)
IV. J $ T ⇒ J > T (false) IV. A © M ⇒ A < M (true)
Hence, either conclusion I or II and III are Hence, all the conclusions I, II, III and IV are
definitely true. definitely true.
136 How to Crack Test of Reasoning VERBAL

13
Direction
Sense Test
Direction is the measurement of position of
Cardinal Directions

A
one object with respect to another object and

M
displacement is the minimum A direction between two main directions is

H
distance between the starting point and final called cardinal direction or subdirection,

TT
point of an object. i.e. NE (North-East), NW N
(North-West), SE NW NE
In this chapter, we deal with the questions in
which a successive follow-up of directions is (South-East) and SW A 45°
W
formulated and the candidate is required to (South-West). W 90° E
ascertain the final direction or the distance Students are advised to use
SH

between two points. diagram as given here, for SW SE


Before moving ahead, let us familiarize with the purpose of sensing
A

S
some facts/concepts. directions.
@

Angle formed between two main directions


Main Directions (i.e. North-East; North-West; South-East or
South-West) is 90° and angle formed between
There are four main directions viz. East, West,
a cardinal direction and main direction is 45°.
North and South as shown in the figure given
below Angle of Movement
Left North Right For solving questions based on angle of
Right Left movement, it is necessary to know the
rotations which are given below
West East (i) Movement towards the right is called
Left Right
clockwise (CW) movement.
Sunset Sunrise
Right South Left 360°

315° 45°
Main directions
270° 90° Clockwise
Abbreviations for these directions are E (East),
W (West), N (North) and S (South).
225° 135°
180°
Chapter 13 DIRECTION SENSE TEST 137

(ii) Movement towards the left is called AB = Perpendicular


anti-clockwise (ACW) movement. BC = Base and
360° AC = Hypotenuse
0° Hence, the shortest distance between A and C
45° 315°
i.e. AC = AB2 + BC2
Anti-clockwise 90° 270°
Illustration 1. Ravi walks 8 km North-East and
135° 225° then 6 km South-East. Find the total distance as
180° well as shortest distance between starting and
end points.
Shadow at the Time of Sunrise (a) 14 km, 12 km (b) 14 km, 10 km
(c) 10 km, 8 km (d) 8 km, 6 km
In the morning, sunrises in the East, the shadow
of any person or object falls in the West. Solution (b) According to the question,
Sunrise B
N
N Person 8 km 90º 6 km NW NE
West Starting point End point W E
W E

A
East A C SW SE
Shadow

M
S
S
Here, total distance = AB + BC

H
Shadow at the Time of Sunset = 8 + 6 = 14 km

TT
In the evening, when the sunset in the West, the and shortest distance AC = AB2 + BC2
shadow of a person or an object falls in the East.
Person N A = 82 + 62
W
= 64 + 36
SH

W E
Sunset East = 100 = 10 km
A

West Shadow
S
Types of Questions
@

Shortest Distance There are four types of questions which are


If we draw a straight line from the initial generally asked in various competitive
point of the object to the final point, then examinations.
length of this line is called shortest distance.
e.g. An object starts from point A and reaches Type 1
to point C after going through point B.
Then, distance between initial and final
Based on Final Direction
position = AB + BC These questions are based on finding the final
But shortest distance between initial and direction of an object with respect to its starting
final position = AC point.
To find the shortest distance between two Illustration 2. Suman walks 15 km towards North.
points, it is necessary to know Pythagoras She turns right and walks another 15 km. She
theorem. turns right and walks another 15 km. In which
direction is she from the starting point?
Here, in ∆ ABC,
SSC (Steno) 2017
A Initial Position (a) North
(b) South
(c) East
B C Final Position (d) West
138 How to Crack Test of Reasoning VERBAL

Solution (c) According to the question, the direction Illustration 5. A clock is so placed that at
diagram of Suman is as given below. 12 noon its minute hand points towards
15 km B N North-East. In which direction does its hour
C
hand point at 1:30 pm? NIFT (UG) 2014
(a) North (b) South (c) East (d) West
15 km 15 km W E
Solution (c) In this question , the clock is placed, so
that at 12 noon its minute hand point towards
O A S North-East.
Starting 15 km Final point We know that, minute and hour hand point in the
point
same direction at 12 noon. Therefore, the clock
Let O be the starting point and A be the ending will look some what like this
point of Suman. It is clear from the diagram that, she N N
is in East direction from her starting point. NW NE NW NE
Illustration 3. After starting from a point, a man 9 12 9 12
walks 4 km towards West, then turning to his
W E W E
right he moves 4 km. After this, he again turns
right and moves 4 km. Which choice given 6 3 6 3

A
below indicates the correct direction in which he
SW SE SW SE

M
is from his starting point? SSC (CGL) 2014
S S
(a) North (b) East

H
12 : 00 Noon 01 : 30 PM

TT
(c) South (d) West
At 1 : 30 pm, the hour hand will point in the East
Solution (a) According to the question, the direction direction.
diagram of a man is as given below
A
Illustration 6. A girl is facing North. She turns
W
4 km N
C D Ending point
180º in the anti-clockwise direction and then
SH

225º in the clockwise direction. Which


4 km

W E
direction is she facing now? CG PSC 2013
A

(a) West (b) North-East


S (c) South-West (d) East
@

B A Starting point
4 km (e) North-West
Let A be the starting and D be the ending point of Solution (b) The girl turns 180º in anti-clockwise
man. It is clear from the diagram that, he is in North direction and then 225º in clockwise direction,
direction from his starting point. which means she finally turns 45º in clockwise
direction.
Illustration 4. If South-West becomes North, then N
what will North-East be? NW NE
(a) North (b) South-East (c) South (d) East 225°
Solution (c) The diagrammatic representation of W E
direction is as shown below
180°

N N
NW NE NE SW SE
S
135°
W E W E
Initially she was facing North. So, now she is
135° facing North-East direction.

SW SE SW Illustration 7. A person walks towards his


S S house at 8:00 am and observes his shadow to
his right. In which direction he is walking?
Clearly, directions are moving 135° clockwise. SSC (CPO) 2014
Hence, North-East will become South moving (a) North (b) South (c) East (d) West
135° clockwise.
Chapter 13 DIRECTION SENSE TEST 139

Solution (b) At 8:00 am the Sun is in the East Solution (b) According to the question, the
direction. movements of the person are as given below
N 15 km
Right D C N
Left O
Person W E
Person’s 9 km W E
Shadow Sun S
House A 3 km B S

Clearly, the person is walking towards the South According to the question,
direction. BC = (3 × 3) = 9 km
CD = (3 × 5) = 15 km
Type 2 ∴ OD = (15 − 3) = 12 km
Based on Displacement Now, required distance,

In this type of questions, we have to find out AD = OD 2 + OA 2


the final distance between the starting and the = 12 2 + 9 2 = 144 + 81
final points or distance between two = 225 = 15 km

A
points/persons/things.

M
Illustration 8. Vijay travelled 12 km Type 3

H
Southward, then turned right and travelled
Based on Direction and Displacement

TT
10 km, then turned right and travelled 12 km.
How far was Vijay from the starting point? In this type of questions, we have to
SSC (CGL) 2014
A
determine both the distance and direction.
W
(a) 22 km (b) 44 km (c) 12 km (d) 10 km
Illustration 10. Shyam goes 5 km in the North
Solution (d) According to the question, direction
SH

from his school. Now, turning to the left, he


diagram of Vijay is as given below.
goes to 10 km and again turns to left and goes
D A to 5 km. How far he is from his school and in
A

10 km N which direction? SSC (CGL) 2014


@

12 km 12 km (a) 10 km, South from school


W E (b) 10 km, North from school
(c) 10 km, West from school
C B
10 km S (d) 10 km, East from school
Let point A be the starting point and D be the final Solution (c) According to the question, the
point of travelling of Vijay. movements of Shyam are as given below
Now, Vijay’s distance from starting point 10 km
C B N
= AD = BC = 10 km
5 km 5 km W E
Illustration 9. A person starts from a point
A and travels 3 km Eastwards to B and then D
A S
turns left and travels thrice that distance to School
reach C. He again turns left and travels five Let point A be the starting point, i.e. school of
times the distance he covered between A and Shyam and point D be the final point.
B and reaches his destination D. The shortest From figure, AB = CD = 5 km
distance between the starting point and the and AD = BC = 10 km
destination is So, Shyam is 10 km far away from his school and
(a) 12 km (b) 15 km in West direction from school.
(c) 16 km (d) 18 km
140 How to Crack Test of Reasoning VERBAL

Illustration 11. X and Y start from the same Solution (d) According to the question, diagram is as
point. X walks 40 m North, then turns West given below.
and walks 80 m, then turns to his right and (Final X
(Final
walks 50 m. At the same time, Y walks 90 m Y position
position N
North. Where is Y now with respect to the
50 m 50 m
position of X ? SSC (CGL) 2017 90 m W E
(a) Y is 30 m to the East of X 80 m
(b) Y is 80 m to the West of X 40 m S
(c) Y is 30 m to the West of X X, Y Starting Point
(d) Y is 80 m to the East of X
It is clear from diagram that, Y is 80 m to the East
of X.

Let us Practice
Base Level Exercise

A
1. Ramesh goes towards North, turns right, 5. One morning at 7 O’clock, Naresh started

M
then turns right again and then goes to the walking with his back towards the Sun.

H
left. Which direction is Ramesh facing now? Then, he turned towards left, walked straight

TT
UP Police (Constable) 2018 and then turned towards right and walked
(a) East (b) West (c) South (d) North straight. Then, he again turned towards left.
2. Rakesh leaves from his house. He walks A
Now, in which direction is he facing?
W
10 m in North-West direction and then 20 m (a) North (b) East
in South-West direction. Next, he moves (c) West (d) South
SH

20 m in South-East direction. Finally, he


6. I am facing South. I turn right and walk
turns towards his house. In which direction
20 m. Then, I turn right again and walk
A

is Rakesh going? CGPSC 2014


10 m. Then, I turn left and walk 10 m and
@

(a) South-East (b) North-West then turn right walk 20 m.Then, I turn right
(c) South-West (d) North-East
again and walk 60 m. In which direction am
(e) None of these
I from the starting point? RRB (GG) 2012
3. From his house, Deepak went 25 km to (a) South-East (b) North-West
North. Then, he turned West and covered (c) South-West (d) North-East
15 km. Then, turned South and covered
10 km. Finally, turning to East, he covered 7. If you start running from a point towards
15 km. In which direction is he from his North and after covering 4 km you turn to
house? SSC (CPO) 2013
your left and run 5 km and then again turn
(a) North (b) South (c) West (d) East to your left and run 5 km and then turn to
left again and run another 6 km and before
4. A policeman left his police post and finishing you take another left turn and run
proceeded 4 km South on hearing a loud 1 km, then answer the following question
sound from point A. On reaching the place, based on this information. From the finishing
he heard another sound and proceeded 4 km point, if you have to reach the point from
to his left to the point B. From B, he where you started, then in which direction
proceeded left to reach another place C, 4 km will you have to run? SSC (CGL) 2015
away. In which direction, he has to go to (a) East (b) West
reach his police post? (c) North (d) South
(a) North (b) South (c) East (d) West
Chapter 13 DIRECTION SENSE TEST 141

8. Kajal left home for a hospital. She travelled 13. A watch reads 4 : 30. If the minute hand
5 km towards East, then turned towards points East, in which direction will the hour
South-East to travel another 10 km, then hand point?
again she turned towards North-East to (a) South-East (b) North-East
travel 10 km and finally, she reached the (c) North (d) North-West
hospital. In which direction is hospital with
respect to her house? SSC (10+2) (Pre) 2017
14. The time in a clock is quarter past twelve.
(a) South (b) North (c) West (d) East
If the hour hand points to the East, then
which is the direction opposite to the minute
9. Raju starts from a place P towards North and hand? UPPSC (Pre) 2013
reaches place Q. From there, he turns (a) South-West (b) South
towards North-West and reaches place (c) West (d) North
R. Then, he turns towards South-West and
walks to a place S. From there, he turns 15. A man is performing yoga with his head
towards North-West and finally reaches down and legs up. His face is towards the
place T. Which of the following figures West. In which direction, will his left hand
shows the movement of Raju? be? RRB (ASM) 2004
R (a) North (b) South
T T R

A
Q Q (c) East (d) West

M
(a) S (b) 16. One evening before sunset two friends Ajay
S

H
and Amal were talking to each other
face-to-face. If Amal’s shadow was exactly to

TT
P P
R
R his right side, which direction was Ajay
T facing? WBPSC 2018
Q S Q (a) North A (b) South
W
(c) S (d) (c) West (d) None of these
SH

P T P 17. Daily in the morning the shadow of Gol


Gumbaz falls on Bara Kaman and in the
10. Ram is facing North-West. He turns in
A

evening the shadow of Bara Kaman falls on


clockwise direction by 90º, then 180º in the
@

Gol Gumbaz exactly. So, in which direction


anti-clockwise direction and then another 90º is Gol Gumbaz to Bara Kaman? SSC (CPO) 2011
in the same direction. Which direction is he (a) Eastern side (b) Western side
facing now? SSC (10+2) 2002 (c) Northern side (d) Southern side
(a) South-West (b) West
(c) South (d) South-East
18. Ram goes 15 m North, then turns right and
walks 20 m, then again turns right and walks
11. A man is facing West. He turns 45° in the 10 m, then again turns right and walks 20 m.
clockwise direction and then another 180° in How far is he from his original position?
the same direction and then 270° in the SSC (10+2) 2013
anti-clockwise direction. Which direction is (a) 5 m (b) 10 m
he facing now? MAT 2013, RRB (TC/CC) 2005 (c) 15 m (d) 20 m
(a) South (b) North-West
(c) West (d) South-West 19. Sherley starting from a fixed point, goes 15 m
towards North and then after turning to his
12. A direction pole was situated on the road right, he goes 15 m. Then, he goes 10 m, 15 m
crossing. Due to an accident, the pole turned and 15 m after turning to his left each time.
in such a manner that the pointer which was How far is he from his starting point?
showing East, started showing South. Sita, a SSC (CGL) 2011
traveller went to the wrong direction thinking (a) 15 m
it to be West. In which direction actually she (b) 5 m
was travelling? SSC (CGL) 2013 (c) 10 m
(a) North (b) West (c) East (d) South (d) 20 m
142 How to Crack Test of Reasoning VERBAL

20. Rema walked 10 km South and turned right turns to North and walks 9 km. How far is
and walked 5 km. Then, again she turned he from his starting point? IBPS (Clerk) 2011
right and walked 10 km. Then, she turned (a) 3 km (b) 4 km
left and walked 10 km. How many km will (c) 5 km (d) 7 km
(e) None of these
Rema have to walk to reach the starting
point? SSC CGL (Pre) 2016 26. A boy leaves his home. He walks 45 m in
(a) 25 km (b) 20 km (c) 5 km (d) 15 km South-West direction and then 145 m in
North-East direction. Then, he moves 60 m
21. Lakshman went 15 km to the West from his
in South direction . How far is he now from
house, then turned left and walked 20 km.
his initial position? SSC (10+2) (Tier-1) 2017
Then, he turned towards East and walked 25
(a) 100 m (b) 80 m (c) 40 m (d) 60 m
km and finally turning left, he covered 20
km. How far is he now from his house? 27. A cyclist cycles 3.5 km West, then turns
(a) 15 km (b) 20 km (c) 25 km (d) 10 km North and cycles 2.5 km North, then turns to
his right and cycles 3.5 km, then turns North
22. A man travels 4 km due North, then travels and cycles 4.5 km. Where is he now with
6 km due East and further travels 4 km due respect to his starting position? SSC CGL 2017
North. How far he is from the starting point? (a) 2 km North (b) 7 km South
SSC (CGL) 2013

A
(c) 7 km North (d) 2 km South
(a) 6 km (b) 14 km (c) 8 km (d) 10 km

M
28. A man is facing South. He turns 135°
23. Anamika who is facing South walks 20 m, anti-clockwise and then 180° clockwise. Which

H
then she turns to her right and walks 15 m, direction he is facing now? WBPSC 2018

TT
then she turns to her right and walks 16 m. (a) North-East (b) West
Again, she turns to her right and walks 12 m.
(c) South-East (d) South-West
How far is she from her starting point? A
W
CGPSC 2013 29. Two airliners start from the same airport. A
(a) 4 m (b) 16 m (c) 12 m (d) 7 m flies 20 km South. B flies 35 km East, then
SH

(e) 5 m turns South and flies 45 km, then turns to its


24. Manoj starts from his house and moves towards right and flies 35 km. Where is B with
A

North. After walking a distance of 4 km, he respect to A now? SSC (10+2) 2017
@

turns left. After walking a particular distance, (a) 65 km South (b) 25 km North
he reaches Sunita’s house. Sunita’s house is at (c) 25 km South (d) 65 km North
distance of 5 km from Manoj’s house. Then, 30. Akash travelled 4 km straight towards
Manoj walked a total distance of EPFO 2011
South. He turned left and travelled 6 km
(a) 1 km (b) 5 km (c) 7 km (d) 3 km
straight, then turned right and travelled
(e) None of these
4 km straight. How far is he from the
25. A man walks 1 km towards East and then he starting point? MPPSC 2018
turns to South and walks 5 km. Again he (a) 18 km (b) 12 km
turns to East and walks 2 km, after this, he (c) 10 km (d) 8 km

Expert Level Exercise


1. A person starts walking from his home Turning 90° towards his right, he walks for
towards his friends place. He walks for 25 m 15 m to reach his friend’s place. How far and
towards West. He takes a 90° right turn and in which direction is the friend’s place from
walks for 20 m. He again takes a 90° right his home? SBI (PO) Pre 2015
turn and walks for 10 m. He then walks for (a) 30 m towards East (b) 30 m towards North
another 10 m after taking a 90° left turn. (c) 40 m towards South (d) 30 m towards South
(e) 40 m towards North
Chapter 13 DIRECTION SENSE TEST 143

2. Anoop starts walking towards South. After 7. The houses of A and B face each other on a
walking 15 m, he turns towards North. After road going North-South, A’s being on the
walking 20 m, he turns towards East and western side. A comes out of his house, turns
walks 10 m. He then turns towards South left, travels 5 km, turns right, travels 5 km to
and walks 5 m. How far is he from his the front of D’s house, B does exactly the
original position and in which direction? same and reaches the front of C's house. In
(a) 10 m, North (b) 10 m, South this context, which one of the following
(c) 10 m, West (d) 10 m, East statements is correct? UPSC (CSAT) 2013
(a) C and D live on the same street
3. A shopper in a mart loads his trolley and (b) C’s house faces South
walks 30 m through an alley which is going
(c) The houses of C and D are less than 20 km apart
South, then he turns to his left and walks
(d) None of the above
10 m, then he turns North and walks another
10 m, then he turns West and walks 45 m 8. Point A is 30 m to the South of point B. Point
and then he turns North and walks 20 m. C is 20 m to the East of point A. Point D is
Where is he now with reference to his 15 m to the South of point C. Point D is exactly
starting position? SSS (10+2) 2018 midway between points E and F in such a
(a) 35 m, West (b) 55 m, West manner that points E, D and F form a

A
(c) 35 m, East (d) 55 m, East horizontal straight line of 40 m. Point E is to

M
4. Starting from a point S, Mahesh walked 25 m West of point D. How far and in which direction
is point E from point B? SBI (PO) Pre 2015

H
towards South. He turned to his left and walked
50 m. Then, he again turned to his left and (a) 45 m towards South (b) 25 m towards South

TT
(c) 30 m towards West (d) 35 m towards North
walked 25 m. He again turned to his left and
(e) 45 m towards North
walked 60 m and reached a point W. How far
Mahesh is from point S and in which direction? A
9. Point D is 7 m to the West of Point R. Anand
W
(a) 10 m, West starts walking from point R toward East. He
walks for 3 m and takes a right turn. He
SH

(b) 25 m, North
(c) 10 m, East
further walks for 4 m takes another right
turn. He walks for 5 m and turns to his right.
A

(d) 25 m, West
He further walks for 4 m and stops at point
@

5. Rohit walked 25 m towards South. Then, he T. In which direction is point T with respect
turned to his left and walked 20 m. He then to point D? IBPS Clerk (Pre) 2017
turned to his left and walked 25 m. He again (a) South (b) West
turned to his right and walked 15 m. At (c) East (d) Cannot be determined
what distance is he from the starting point (e) North
and in which direction? UP (B.Ed.) 2012
10. One morning after sunrise Manajit and
(a) 35 m, East
Sangita were talking to each other face-to-
(b) 35 m, North
face at the park. If Manajit’s shadow was
(c) 40 m, East
exactly to the right of Sangita, which
(d) 60 m, East
direction Manajit was facing?
6. Rajnikanth left his home for office in car. He SSC (Steno) 2014
drove 15 km straight towards North and then (a) South (b) East (c) West (d) North
turned Eastwards and covered 8 km. Then, he 11. Raju wants to go to the planetarium. So, he
turned to left and covered 1 km. He again walked 1.5 km towards East from the place
turned left and drove for 20 km and reached and then turned to right and walked 2.5 km
office. How far and in which direction is his and then turned towards East walked 1 km
office from the home? IB (ACIO) 2013 and turned to South and walked 4 km and
(a) 21 km West reached the place by walking 2.5 km towards
(b) 15 km North-East West. What distance is he from the starting
(c) 20 km North-West point? SSC (10+2) 2014
(d) 26 km North-West (a) 6.5 km (b) 9.5 km (c) 10 km (d) 9 km
144 How to Crack Test of Reasoning VERBAL

12. A person starts from Point A, walks 30 m 16. If a person walks 4 m towards the South
towards South and reaches Point B. He then from point L, takes right turn and walks for
takes a right, walks 7 m, followed by a right another 3 m, then which of the following
turn and walks for 6 m. He then takes a right points would he reach?
turn and walks 7 m. He takes a final left (a) Q (b) G (c) K (d) H
turn, walks a certain distance and reaches (e) Cannot be determined
Point R. Point R is 17 m to the North of Point
17. If a person walks 8 m towards North from
B. What is the distance between Point A and
point S, then which of the following points
Point R? SBI (PO) Pre 2017
would he crossed and how far will he be
(a) 18 m (b) 23 m
from point R?
(c) 21 m (d) 27 m
(a) G, 4 m (b) H, 3 m (c) L, 6 m (d) L, 3 m
(e) 13 m (e) G, 8 m
13. A child was looking for his father. He went Directions (Q. Nos. 18-19) Read the given
90 m towards East before turning to his information carefully and answer the given
right. Then, he went 20 m before turning to questions. IBPS (PO) Pre 2014
his right again to look for his father at his
Point N is 8 m to the West of point O. Point P is
uncle's place 30 m from this point. His father
4 m to the South of point O. Point Q is 4 m to the

A
was not there. Then, he went 100 m to the
East of point P. Point R is 6 m to the North of

M
North before meeting his father in a street. point Q. Point S is 8 m to the West of point R.
How far from the starting point did the son

H
Point T is 2 m to the South of point S.
meet his father? MAT 2013

TT
(a) 80 m (b) 100 m 18. How far and in which direction is point T
from point N?
(c) 140 m (d) 200 m
A
(a) 4 m to the East (b) 8 m to the West
W
Directions (Q. Nos. 14 and 15) Study the given (c) 4 m to the West (d) 8 m to the East
information and answer the given questions. (e) 6 m to the South
SH

IBPS (Clerk) 2013


19. If point T is 4 m to the North of point E.
l
Point A is 11 m North of point B. Then, what is the distance between E and Q?
A

l
Point C is 11 m East of point B.
(a) 11m (b) 8 m (c) 15 m (d) 5 m
@

l
Point D is 6 m North of point C. (e) 9 m
l
Point E is 7 m West of point D.
l
Point F is 8 m North of point E. Directions (Q. Nos. 20-22) Read the given
l
Point G is 4 m West of point F. information carefully and answer the given
questions. IBPS (PO) Pre 2017
14. How far is point F from point A? Point B is 25 m South of point A. Point C is 10 m
(a) 43 m (b) 4 m (c) 3 m (d) 7 m East of point B. Point D is 30 m North of point C.
(e) 5 m Point E is 7 m East of point D. Point X is 18 m
15. How far and in which direction is point G South of point E. Point E is 7m East of point D.
from point A? Point X is 18 m South of point E. Point M is 12 m
(a) 3 m, North (b) 5 m, North South of point X. Point C is 7 m West of point M.
(c) 4 m, North (d) 4 m, South 20. B is in which direction from point D?
(e) 3 m, South
(a) South (b) South-West
Directions (Q. Nos. 16-17) Read the following (c) North-East (d) South-East
information carefully and answer the questions. (e) North

Point H is 6 m towards the East of point G. 21. If point W is 3 m to the North of point A,
Point R is 8 m North of point G. Point Q is exactly then what is the distance between point B
midway between point R and point G. Point K is and point W?
10 m to the South of point Q. Point L is 3m (a) 28 m (b) 15 m (c) 22 m (d) 24 m
towards the East of point Q. Point S is exactly (e) 17 m
midway between point G and point H.
Chapter 13 DIRECTION SENSE TEST 145

22. What is distance between points B and M? 23 What is the difference of the distance
(a) 17 m (b) 15 m between the points Z, J and points Y, F?
(c) 21 m (d) 19 m (a) 26 m (b) 22 m (c) 12 m (d) 8 m
(e) 13 m (e) 10 m

Directions (Q. Nos. 23-25) Study the given 24. If a person walks 20m towards North from
information carefully to answer the given point Z, takes a left turn and walks 8 m, how
questions. IBPS (PO) Pre 2017 far will be from point C?
Point E is 16 m to the South of point C. Point F is (a) 2 m (b) 6 m (c) 3 m (d) 4 m
4 m to the West of point E. Point H is 5 m to the (e) 5 m
South of point F. Point J is 12 m to the East of 25. In which direction is point Z with respect to
point H. Point Y is to the East of point F. A point H?
person walks 15m from point Y towards West, (a) North-West (b) North-East
reaches point Z, takes a left turn and reaches (c) South-West (d) East
point J. (e) South-East

Answers

A
M
Base Level Exercise

H
1. (a) 2. (d) 3. (a) 4. (d) 5. (d) 6. (d) 7. (b) 8. (d) 9. (a) 10. (d)

TT
11. (d) 12. (a) 13. (b) 14. (d) 15. (a) 16. (b) 17. (a) 18. (a) 19. (c) 20. (d)
21. (d) 22. (d) 23. (e) 24. (c) 25. (c) 26. (b) 27. (c) (c) (c) (c)
A 28. 29. 30.
W
Expert Level Exercise
SH

1. (b) 2. (d) 3. (a) 4. (a) 5. (a) 6. (c) 7. (c) 8. (a) 9. (c) 10. (d)
11. (a) 12. (e) 13. (b) 14. (e) 15. (a) 16. (b) 17. (d) 18. (a) 19. (b) 20. (b)
A

21. (a) 22. (a) 23. (b) 24. (d) 25. (b)
@

Answer with Explanations


Base Level Exercise
1. (a) According to the quasestion, the direction 2. (d) According to the question, the direction
diagram is as given below. diagram is as given below.
N B N
10 m NW NE
20 m A
W E W E
C
Starting SE
S SW
point 20 m S
End point D
It is clear from the diagram that, Rakesh is going in
Now, clearly Ramesh is facing towards East
North-East direction.
direction.
146 How to Crack Test of Reasoning VERBAL

3. (a) According to the question, the direction 7. (b) According to the question, the direction
diagram is as given below. diagram is as given below.
C 15 km B 5 km N
N 4 km
Finishing
10 km
point W E
D 15 km E 25 km W E 5 km
Starting point
S 1 km S
A 6 km
(House)
Clearly, you have to run in West direction to reach
Starting from the point A and passing through
the starting point.
the points B, C and D, Deepak reaches the
point E. Hence, he is in North direction from his 8. (d) According to the question, the direction
house. diagram is as given below.
N
NW NE
4. (d) According to the question, the direction 5 km Hospital
diagram is as given below. Home
10 km 10 km W E
Police post C N

A
SW SE

M
4 km

S
4 km W E It is clear from diagram that the hospital is in East

H
direction with respect to Kajal’s home.

TT
S 9. (a) According to the question, the direction
A 4 km B diagram is as given below.
To reach police post (starting point) from C, T A
R
W
N
policeman will have to move in the West direction. Q
SH

5. (d) According to the question, the direction S W E


diagram is as given below.
A

Naresh P S
@

This situation is traced by option (a).


Left Starting point
Right N 10. (d) Movement in clockwise direction = 90º
Movement in anti-clockwise direction
(Final Left W E = 180º + 90° = 270º
position or we can say he turns (270º − 90º) = 180º in
(South) S anti-clockwise direction.
Initially he was facing North-West direction. So,
So, Naresh’s facing towards South. now he is facing South-East direction.
6. (d) According to the question, the direction 11. (d) According to the question, the direction
diagram is as given below. diagram is as given below.
E 60 m F N Q
NW NE 270° N
G
20 m NW NE
10 m
C W E 45° O
D W E
10 m P

SE
18

B A SW SE
20 m S SW
S R S
Clearly, the final position is F which is to the
From figure, finally he is facing in the OS direction,
North-East of the starting point A.
which is South-West.
Chapter 13 DIRECTION SENSE TEST 147

12. (a) The pointer which was showing East started 17. (a) In the morning, an object casts shadow to the
showing South. West. In the evening, an object casts its shadow to
After accident, E→ S N the East. Therefore, Gol Gumbaz is to the Eastern
S→ W side of Bara Kaman.
W→ N W E 18. (a) According to the question, the direction
N→ E diagram is as given below.
∴ Pointer turned 90° clockwise.
S B 20 m C
So, Sita actually went in North
direction.
10 m
13. (b) Given information can be represented as
15 m E 20 m
W
NW N
SW
11 12 1 NW NE D
10 2 End point
S 9 3 N W E A
8 4 Starting point
7 6 5 NE
SE SE
SW Thus, Ram is at a distance of 15 − 10 = 5 m from
E S
Hence, when the minute hand points to the East, his starting point.

A
hour hand points to the North-East direction. 19. (c) According to the question, the direction
diagram is as given below.

M
14. (d) Given information can be represented as
E 15 m D

H
East
N

TT
11 12 1
10 15 m 10 m
Hour 2
North 9 hand 3 South
8
Minute
hand 4
W E
BA C
W
7
6 5 F 15 m
S 15 m
West
SH

12:15 pm A

The clock is showing 12:15 and according to the Here, A is the starting point and F is the end point
A

question, the direction of hour hand is in East. of Sherley.


@

So, the minute hand points towards South Q AE = AB + BE


direction. The direction opposite to South is North. = 15 + 10
Hence, the direction opposite to minute hand is = 25 m [Q BE = DC = 10]
North. Now, AF = AE − EF
15. (a) According to the question, the direction = 25 − 15
diagram is as given below. = 10 m
N Hence, Sherley is 10 m far from his starting point.
N
Left hand 20. (d) According to the question, the direction
Head E W
diagram is as given below.
E
W 10 km D 5 km A Starting
Right hand Final E point
S N position
S
Hence, it is clear from above figure that the left 10 km 10 km
hand points towards North. W E

16. (b) As we know that, sunsets in West direction. S C B


5 km
So, Ajay
N
Now, distance between starting point and final
(West)
W E position = ED + DA = 10 + 5
Amal Amal’s shadow = 15km
S Hence, Rema have to walk 15 km to reach the
Hence, Ajay was facing South. starting point.
148 How to Crack Test of Reasoning VERBAL

21. (d) According to the question, the direction 24. (c) It is clear from the figure that, it is a right angle
diagram is as given below. triangle.
N In ∆ABC by Pythagoras theorem,
B 15 km A E AB 2 + BC 2 = AC 2 Sunita’s A B
⇒ AB = AC − BC
2 2 2 house
20 km

20 km
W E
⇒ AB = AC 2 − BC 2 5 km 4 km
C 25 km D ⇒ AB = 25 − 16 C
S
⇒ AB = 9 Manoj’s
Points A and E show the starting and end positions house
⇒ AB = 3 km
respectively of Lakshman.
Thus, Manoj travelled a distance of 4 + 3 = 7 km
Q AE = CD − AB = 25 − 15 = 10 km
Hence, he is 10 km far away from his house. 25. (c) According to the question, the direction
diagram is as given below.
22. (d) According to the question, the direction Clearly, DF = BC = 5 km
diagram is as given below.
(End Point)
D End point E
4 km

A
4 km N
6 km NW NE 9 km

M
B C A F
1 km B

H
W E (Starting
4 km 4 km Point) 5 km 5 km

TT
SW SE
A E S
C 2 km D
Starting
point
6 km
A
W
EF = (DE − DF) = (9 − 5) = 4 km
Distance between the starting point and end point
BF = CD = 2 km
SH

is AD.
Now, AD2 = AE2 + DE2 AF = AB + BF = (1 + 2 ) = 3 km
∴ Man’s distance from starting point A,
A

[Pythagoras theorem in ∆AED]


@

AD2 = (6)2 + (8)2 AE = AF 2 + EF 2


∴ AD = 36 + 64 = 100 = 10 km = 32 + 42
23. (e) According to the question, the direction = 9 + 16 = 25 = 5 km
diagram is as given below.
26. (b) According to the question, the direction
A diagram is as given below.
N (N)
D 12 m E F NW NE C
m

W E
60 m
0

16 m 20 m
10

N
5

SW SE A NW NE
14

S (W) (E)
C B D
15 m W E
m

Here, point A and E are the starting and end


45

SW SE
positions of Anamika. S
B
In ∆ AEF, AE = (AF)2 + (EF)2 (S)
[Q AF = 20 − 16 = 4 m, EF = 15 − 12 = 3 m] Let starting point is A and end Point is D.
= (4) + (3) = 16 + 9 = 25
2 2 Required distance, AD = (100)2 − (60)2
AE = 5 m = 6400
So, she is 5 m away from her starting point. = 80 m
Chapter 13 DIRECTION SENSE TEST 149

27. (c) According to the question, the direction 29. (c) According to the question, the direction
diagram is as given below. diagram is as given below.
D (Starting
point) O 35 km
4.5 km N N
20 km
3.5 km W E A 45 km W E
B C
2.5 km 2.5 km S S
B 35 km
O
A
3.5 km Starting Point Let A and B start from a point O.
Then, AB = 45 − 20 = 25 km
Let O is the starting point, D is the final point.
∴ B is in 25 km South with respect to A.
∴ From the starting point to the end point,
30. (c) According to the question, the direction
Distance = OC + CD = 2.5 + 4.5 = 7 km
diagram is as given below.
And, direction → North
A Starting point N
∴Option (c) is the right answer.
28. (c) According to the question, the direction 4 km
6 km W E
diagram is as given below.

A
B 3 3 C
(N) M 4 km

M
6 km S
D
A

H
Let the person started from point A and ending

TT
O point is D.
(W) (E) There, BM = MC = 3 km
135°
180° A
Now, by Pythagoras theorem, in ∆ ABM,
W
AM2 = 32 + 42 = 9 + 16
⇒ AM = 25 = 5 km
SH

A′
(S) Again, using pythagoras theorem in ∆ DCM,
= MD2 = 32 + 42 = 9 + 16
A

OA′ is the final position at which the man is looking


finally. ⇒MD = 25 ⇒MD = 5 km
@

∴OA′ is South-East direction. ∴Total distance between starting point and ending
point is = AM + MD = 5 + 5 = 10 km

Expert Level Exercise


1. (b) According to the question, the direction 2. (d) According to the question, the direction
diagram is as given below. diagram is as given below.
15 m C 10 m
D
Friend’s
90° place 5m N
10 m A
E
10 m 90°
20 m

90° Starting point W E


20 m 15 m
90° S
25 m Home B B
where A → Starting point and E → End point
∴ The friend’s place is 20 m + 10 m = 30 m in the Clearly, from figure, point E is at a distance of 10 m
North direction from his home. from point A and in the East direction.
150 How to Crack Test of Reasoning VERBAL

3. (a) According to the question, the direction 7. (c) According to the question, the direction
diagram is as given below. diagram is as given below.
End Point Starting Point X 5 km D
B 35 m A
5 km Right
20 m N
30 m
45 m Left
W E A B 10 km
35 m 10 m Left
10 m
S
Right 5 km
10 m
So, it is clear from the figure that, he is 35 m C 5 km Y
towards West from his starting position in the end.
Now, AX = DB = BY
4. (a) According to the question, the direction = CA = 5 km
diagram is as given below. XD = CY = 5 km, XC = XA + AC
60 m
N =5+ 5
10 m S 50 m
W V = 10 km
W E
∴Distance between C and D,
25 m

A
25 m
50 m
CD = (XD)2 + (XC )2 [by Pythagoras theorem]

M
T U S
= 5 + 10
2 2
= 25 + 100

H
Mahesh is at a distance of 10 m and in West
= 125 = 5 5

TT
direction from his starting point S.
= 5 × 2.23 = 11.18 km
5. (a) According to the question, the direction So, it is clear from the figure, that the houses of C
diagram is as given below.
A
and D are less than 20 km apart.
W
A D
E 8. (a) According to the question, the direction
15 m N
SH

diagram is as given below.


25 m 25 m W E B
A

B C N
@

20 m S 30 m
∴ Rohit’s distance from starting point, A C W E
20 m
AE = AD + DE [Q DE = 15 m , BC = AD = 20 m] 15 m
= 20 + 15 = 35 m E D F
S
20 m 20 m
He is 35 m in East direction from the starting point.
Here, BE = AE + AB
6. (c) According to the question, the direction
Q AE = CD = 15 m
diagram is as given below.
and AB = 30 m
e)
f f ic 20 km ∴ BE = 15 + 30 = 45 m
(O E F D
N So, point E is 45 m away in the South direction
1 km NW NE
8 km
C
from B.
B
W E
9. (c) According to the question, the direction
15 km diagram is as given below.
SW SE
S
e) T 3m
A N
om D
(H R
7m
In ∆AFE, AE = AF 2 + EF 2 = (AB + BF )2 + (ED − FD)2 4m W E
4m
= (16) + (12 ) = 256 + 144 =
2 2
400 = 20 km
S
So, his office is 20 km in North-West direction from 5m
his home.
Point T is to the East of point D.
Chapter 13 DIRECTION SENSE TEST 151

10. (d) Here, Manajit Shadow is exactly to the right of ∴ AF = AB − FB = AB − DC [Q FB = DC]


Sangita. It means right hand of Sangita is in the = 90 − 30 = 60 m
West and her face is in the direction of South. EF = ED − FD
Manajit is opposite to Sangita that’s why, his face
is in the direction of North. = ED − BC [Q FD = BC]
= 100 − 20 = 80 m
11. (a) According to the question, the direction
diagram is as given below. Now, AE = (AF)2 + (EF)2
A 1.5 km B = (60)2 + (80)2
2.5 km 2.5 km N = 3600 + 6400
C 1 km
D = 10000 = 100 m
G
W E Solutions (Q. Nos. 14-15) According to the question,
4 km 4 km the direction diagram is as given below.
S 4m
G F
F 2.5 km E A N
8m

11 m
Let A be the starting point of Raju and F be the end 7m D W E
point. E

A
6m
Now, GF = ED = 4 km and AG = BC = 2.5 km S
B C

M
Now, distance of Raju from his starting point

H
= AG + GF = (2.5 + 4) km 14. (e) According to the question, the direction
= 6.5 km diagram is as given below.

TT
AG = BG − AB G
4m
12. (e) According to the question, the direction F
= (CD + EF ) − (AB )
diagram is as given below.
= (6 + 8) − 11 A 3m
W
A
= 3m
R
SH

and GF = 4 m A
N
30 m ∴ AF = AG 2 + GF 2
D 7m E
A

17 m W E = 9 + 16
@

6m = 5m
S
C B 15. (a) Point G is in the North direction from point A
7m
and AG = 3 m [from above solution]
Here, AB = 30m, BR = 17m
∴ AR = AB − BR = 30 − 17 = 13 m Solutions (Q. Nos. 16-17) According to the question,
Hence, distance between Point A and R is 13 m. the direction diagram is as given below.
R
13. (b) According to the question, the direction
diagram is as given below. 4m
8m 3m
Q L N
E
4m 3m 3m
S
100 m G H W E
6m
B N 6m S
A 90 m F 10 m
20 m W E
K
D C S 16. (b) He will reach at point G.
30 m
17. (d) He will cross point L and he will be 3 m far from
AE is the distance between the starting point and point R.
the end point.
152 How to Crack Test of Reasoning VERBAL

Solutions (Q. Nos. 18-19) 20. (b) It is clear from diagram that point B is in
South-West direction from point D.
S 8m R
21. (a) Distance between point B and Point W
2m N = AB + AW =25+ 3=28m
4m 4m
N O 22. (a) Distance between point B and point M
8m 6m W E
T = BC +CM
4m 4m
S =10+7 =17 m
E 4m P 4m Q Solution (Q. Nos. 23-25) According to the question,
the direction diagram is as given below.
18. (a) Clearly, point T is 4 m to the East of point N.
8m
19. (b) Distance between E and Q N
= PQ + PE [QPE = OT = 4 m ] NW NE
C
= 4m+ 4m = 8m 20m W E
Solutions (Q. Nos. 20-22) According to the question,
the direction diagram is as given below. 16 m SW SE
S
D 7m E F 4m E 8m Z 15 m
Y

A
W
5m 5m

M
3m 18 m N
A J

H
NW NE H 12 m

TT
30 m X W E 23. (b) Required distance
25 m = (15 + 12) − 5
12 m
SW
S
SE
A
= 27 − 5 = 22 m
W
24. (d) Required distance = (20 – 16) = 4m
SH

B 10 m C 7m M 25. (b) Point Z is to the North-East of point H.


A
@
Chapter 14 CLOCK AND CALENDAR 153

14
Clock and Calendar
Clock and calendar are the basic instruments Fast and Slow Clock
that are being used to measure time in If a watch or a clock indicates 6:15, when the
different units. correct time is 6, it is said to be 15 min fast.

A
On the other hand, if it indicates 5:45, when the
Clock correct time is 6, it is said to be 15 min slow.

M
H
A Clock is an instrument that indicates time. It
Clock Facts

TT
has three hands i.e. a short hand that indicates
hour (hour hand), a long hand that indicates
minutes (minute hand) and a thin long hand
l

hour hand. A
In 60 min, the minute hand gains 55 min space over
W
that indicates seconds (second hand).
l
In every hour, both the hands coincide once.
The face or dial of a clock is divided along the
SH

l
The hands are in the same straight line, when they
circumference into 12 equal parts called hour are coincident or opposite to each other.
spaces and 60 equal parts called minute
A

l
When the two hands are at right angles (90°), they
spaces. are 15 min space apart.
@

The basic structure of clock is as follows. l


When the hands are in opposite directions, they are
30 min space apart.
Minute hand l
Angle traced by hour hand in 12 h = 360°
12 360°
11 1 ∴ Angle traced by hour hand in 1 h = = 30°
10 2 12
30 1°
Second hand
9 3 ∴ Angle traced by hour hand in 1 min = =
Hour hand 60 2
8 4 l
Angle traced by minute hand in 60 min = 360°
7 6 5
360°
∴ Angle traced by minute hand in 1 min = = 6°
Clock 60
l
The angle between the minute hand and hour hand
Here, 60 minute spaces = 360° of a clock is,
11
∴ 1 minute space = 6° θ= m − 30h
2
Similarly,12 hour space = 360°
where, h = hours and m = minutes
∴ 1 hour space = 30° l
Both hands of a clock occurs at right angle twice in
60 min = 1 hour one hour, 22 times in 12 h and 44 times in 24 h.
30 min = Half an hour l
Both hands of a clock occurs in a straight line or are
opposite once in one hour, 11 times in 12 h and
15 min = Quarter of an hour 22 times in 24 h.
1 min = 60 s
154 How to Crack Test of Reasoning VERBAL

Illustration 1. When the minute hand covers a Illustration 4. A clock is set right at 8 am. The
distance of 1h 20 min, then what is the clock gains 10 min in 24 h. What will be the right
angular distance covered by it? time when the clock indicates 1 pm on the
(a) 310° (b) 192° (c) 480° (d) 420° following day?
Solution (c) Total minute spaces = 1 h + 20 min (a) 11 : 40 pm
= 60 min + 20 min = 80 min (b) 12 : 48 pm
(c) 12 pm
Q 1 min space = 6 °
(d) 10 pm
∴ 80 min spaces = 80 × 6 ° = 480 °
Note The relative speed of the minute hand with respect to
Solution (b) Time from 8 am of a particular day to 1 pm on
° the following day = 29 h
hour hand is 5 1 per min and at any time, they make
2 Now, the clock gains 10 min in 24 h, it means that 24 h 10
an angle between 0° and 180° with each other. min of this clock is equal to 24 h of the correct clock.
Whenever you get answer in decimals, then 10 145
remember that the base is 60. ∴ 24 + = h of this clock
60 6
Illustration 2. What will be the measurement = 24 h of the correct clock
of the angle made by the hands of a clock 24
when the time is 8 : 35? 29 h of this clock = × 6 × 29
145

A
(a) 32.4° (b) 37.5° (c) 45° (d) 47.5°
= 28 h, 48 min of correct clock

M
Solution (d) Angle traced by hour hand in 35 min 29 h of this clock = 28 h 48 min of the correct clock.

H
1
after 8 = 35 × = 17.5 ° It means that the clock is 12 min faster than the

TT
2
correct clock. Therefore, when clock indicates 1pm,
At 8 : 35, min hand is at 7, and angle between the correct time will be 48 min past 12.
8 and 7 = 30 °
∴ Required angle between two hand at 8 : 35
A
Illustration 5. At what time between 7 and
W
= 30 ° + 17.5 ° = 47.5 °
8 O’clock will the hands of a clock be in the
SH

straight line but not together?


Illustration 3. At what angle the hands of a (a) 5
5
min past 7
clock are inclined at 15 min past 5? 11
A

1° 1° 1° (b) 5 min past 7


(a) 72 (b) 67 (c) 58 (d) 64°
@

2 2 2 (c) 15 min past 7


Solution (b) At 15 min past 5, the minute hand is 3
(d) 7 min past 7
at 3 and hour hand slightly ahead of 5. 10
∴ Distance between 3 and 5 = 10 min space Solution (a) At 7 O’clock the hour hand is at 7 and the
Now, angle between 3 and 5 = 10 × 6 ° = 60 ° minute hand is at 12. Thus, they are 25 min spaces
[Q 1 min space = 6 °] apart.
Now, angle through which hour hand shifts To be in a straight line but not together, they have to
 1 ° 1° be 30 min spaces apart.
in 15 min = 15 ×  = 7
 2 2 So, the minute hand have to gain 5 min spaces.
 1 ° 1° Now, 55 min spaces are gained in 60 min.
∴ Angle at 15 min past 5 =  60 + 7  = 67
 2 2 ∴5 min spaces will be gained in
60
×5
55
Alternate method,
60
Here, m = 15 and h = 5 = min
11
11
∴ θ= (15) − 30 (5) = |82.5 − 150 | 5
2 = 5 min
11

= 67.5 ° = 67 ∴The hands will be in a straight line but not together
2 5
at 5 min past 7.
11
Chapter 14 CLOCK AND CALENDAR 155

Calendar l
The first day of century cannot be Wednesday,
Friday and Sunday.
A calendar is the record of all the days of the l
The last day of century cannot be Tuesday, Thursday
year.
and Saturday.
l
The smallest unit of a calendar is a day.
l
A week is group of 7 days. Illustration 6. If the day after tomorrow is a
l
A month is 12th part of a year. Sunday, what was it day before yesterday?
l
A year is the time taken by Earth to (a) Wednesday (b) Thursday
complete one revolution around the Sun. (c) Friday (d) Saturday
l
A period of 100 years is called a century. Solution (a) Today → Friday.
There are two types of years. So, the day before yesterday → Wednesday.
(a) Ordinary year : An ordinary year is a
year which has 365 days. Such years are How to Find the Day on a
not divisible by 4. e.g., 2001, 2002, 2003 Particular Date?
etc. Ordinary years in the form of century In order to find a particular day on the basis
are not exactly divisible by 400. e.g. 900, of given day and date the following steps
1800, 2100, etc. are to be taken.

A
(b) Leap year : A leap year is a year which Step I Find the total number of odd days

M
has 366 days. Such years are exactly between the reference date and the date for

H
divisible by 4. e.g. 2004, 2008, 2012 etc. which the day is to be determined (exclude

TT
Leap year in the form of a century are the reference day but count the given day
exactly divisible by 400. e.g. 2000, 2400, for counting the total number of odd days)
2800 etc. A
Step II (a) If the reference day was before
W
l
Odd days In a given period, the number of that particular date– The day of the week on
days more than the complete weeks are the particular date is equal to the total
SH

called odd days. Or, we can say, when total number of odd days ahead of the reference
number of days is divided by 7, then the day.
A

remainder is called odd days. Or


@

1 ordinary year = 365 days = (52 weeks+ 1day) (b) If the reference day was ahead of that
1 leap year = 366 days = (52 weeks + 2 days) particular date– The day of the week on the
particular date is equal to number of odd
days before the reference day.
Odd Days in Century Years
Note : In order to find a day on a particular date, when
l
100 yr = 76 ordinary years + 24 leap years reference day is not given, you have to find the number
= (76 × 1 + 24 × 2) odd days of odd days upto that date. Your required day will be
= 124 odd days = (17 weeks + 5 days) determined according to the given table.
= 5 odd days Day of the Week Related to Odd Days
∴Number of odd days in 100 yr = 5
Number of days Day
∴Number of odd days in 200 yr
= (5 × 2) = 10 = (7 + 3) 0 Sunday
= 3 odd days 1 Monday
l
Number of odd days in 300 yr 2 Tuesday
= 5 × 3 = 15 days = 2 weeks +1day
3 Wednesday
= 1odd day
4 Thursday
Number of odd days in 400 yr
5 Friday
= (5 × 4) + 1 (for leap year)
= 20 + 1 = 21days 6 Saturday
= 3 weeks = 0 odd day
156 How to Crack Test of Reasoning VERBAL

Illustration 7. If March 27, 2011 was Sunday, Illustration 9. What was the day on 26th Jan
what was the day on June 27, 2012? CLAT 2017 1950, when first Republic Day of India was
(a) Monday (b) Wednesday celebrated?
(c) Sunday (d) Saturday (a) Monday
Solution (b) Day on 27 March 2011 = Sunday (b) Tuesday
Number of odd days from 27 March 2011 to (c) Thursday
27 June 2011 = 4 + 30 + 31 + 27 = 92 (d) Friday
= 13 week + 1 odd day Solution (c) Number of odd days in 1600 yr = 0
Hence, day on 17 June 2011 = Sunday + 1 Number of odd days in 300 yr = 1
= Monday ∴Number of odd days in 1900 yr = 0 + 1 = 1
Now, as 2012 is a leap year Now, in 49 yr, there are 12 leap yr and 37 ordinary
∴Day on 27 June 2012 = Monday + 2= years.
Wednesday ∴Number of odd days in 49 yr = 2(12) + 37
Illustration 8. I have a calendar of year 1997. [Q a leap year has 2 odd days and
When can I use it again? an ordinary year has 1 odd day]
(a) 2001 (b) 2003 (c) 2005 (d) 2007 = 24 + 37 = 61

A
Solution (b) The year 1997 has 365 days, i.e. 1 odd = 8 weeks + 5 odd days.

M
day, year 1998 has 365 days, i.e. 1 odd day. Number of odd days in 1949 yr = 1 + 5 = 6
Likewise year 1999, 2001 and 2002 have 1 odd

H
Now, for 26 January
day each and year 2000 has 366 days, i.e. 2 odd

TT
days. ⇒ Number of odd days = 5
The sum of odd days, so calculated from years ∴Total number of odd days = 6 + 5 = 11
1997 − 2002 = 1 + 1 + 1 + 2 + 1 + 1 = 7 days A = 1 week + 4 odd days
W
= 0 odd days. Day related to 4 odd days = Thursday
SH

Hence, the year 2003 will have the same calendar Hence, the day on 26th January 1950 was
as that of the year 1997. Thursday.
A
@

Let us Practice
Base Level Exercise
1. How many times in a day are the hands of a 5. At half past 5 in the evening, the smaller
clock pointing opposite to each other? angle between the hour and minute hands of
UPPSC (RO) 2014 a clock is IB (ACIO) 2017
(a) 20 (b) 4 (c) 24 (d) 22 (a) 10° (b) 12° (c) 15° (d) 18°
2. How many times from 4 pm to 10 pm, the 6. The angle between the hour hand and the
hands of a clock are at right angle? CLAT 2017 minute hand of a clock when time is 3 : 25
(a) 11 (b) 6 (c) 90 (d) 10 will be WBPSC 2018
3. Find the angle traced by hour hand of a correct 1° 1°
(a) 47 (b) 42 (c) 60° (d) 45°
clock between 8 O’ clock and 2 O’ clock. 2 2
(a) 180° (b) 150° (c) 210° (d) 200° 7. A clock was set correct at 12 O'clock. It loses
4. What will be the angle between hour hand 10 minutes per hour. What will be the angle
and minute hand, if clock shows 8:30 pm? between the hour and minute hands of the
SSC (CPO) 2011
clock after one hour? CLAT 2017
(a) 90° (b) 85°
(a) 90° (b) 75° (c) 60° (d) 85°
(c) 75° (d) 105°
Chapter 14 CLOCK AND CALENDAR 157

8. At what time, (in minutes) between 12. Two clocks are set correctly at 9 am on
3 O’ clock and 4 O’clock, both the needles Monday. Both the clocks gain 3 min and
will coincide with each other? 5 min respectively in an hour. What time will
1 4 4 4 the second clock register, if the first clock
(a) 5 (b) 12 (c) 13 (d) 16
11 11 11 11 which gains 3 min in an hour shows the time
9. At what time between 5 and 6, are the hands as 27 min past 6 pm on the same day?
of a clock coincides? (a) 6 : 27 pm (b) 6 : 45 pm
(c) 6 : 25 pm (d) 6 : 50 pm
(a) 22 min past 5 (b) 30 min past 5
8 3 (e) None of these
(c) 22 min past 5 (d) 27 min past 5
11 11 13. A clock is set right at 5 am. The clock loss
10. At what time between 4 and 5 O’clock, will 16 min in 24 h. What will be the right time
the hands of a watch point in opposite when the clock indicates 10 pm on the 4th
directions? day?
(a) 11 : 15 pm (b) 11 : 00 pm
(a) 45 min past 4 (b) 40 min past 4
(c) 12 : 00 pm (d) 12 : 30 pm
4 6
(c) 50 min past 4 (d) 54 min past 4
11 11 14. The minute hand of a clock overtakes the
hour hand at intervals of 63 min of the
11. At what time between 5:30 and 6 O’clock,

A
correct time. How much does a clock gain or
will the hands of a clock be at right angle?

M
loose in a day?
3 7
(a) 43 min past 5 (b) 43 min past 5 8 8

H
11 11 (a) 54 min (gain) (b) 56 min (gain)
77 77

TT
(c) 40 min past 5 (d) 45 min past 5 8 8
(c) 53 min (gain) (d) 53 min (gain)
77 77
A
W
Expert Level Exercise
SH
A

15. The day before the day before yesterday is 19. If it is Saturday on January 1,2000, then
@

three days after Saturday. What day is it January 1, 2001 would have been CGPSC 2014
today? IB (ACIO) 2013, BSSC (CGL) 2015 (a) Monday (b) Tuesday
(a) Tuesday (b) Wednesday (c) Friday (d) Saturday
(c) Thursday (d) Friday (e) None of these

16. The last day of a century cannot be either 20. If January 1 is a Friday, then what is the first
day of the month of March in a leap year?
I. Tuesday II. Thursday
SSC (CGL) 2013
III. Saturday IV. Sunday MAT 2013 (a) Tuesday (b) Wednesday
(a) I and II (c) Thursday (d) Friday
(b) I and IV 21. In a month of 31 days, third Thursday falls
(c) I, II and III on 16th. What will be the last day of the
(d) I,III and IV month? SSC (CGL) 2013
17. If today is Thursday, then what will be the (a) 5th Friday
day on 363rd day? (b) 4th Saturday
(c) 5th Wednesday
(a) Sunday
(d) 5th Thursday
(b) Saturday
(c) Thursday 22. If Republic day was celebrated in 1996 on
(d) None of these Friday, on which day in 2000 independence
18. Today is Monday. After 61 days, it will be was celebrated? SSC (CGL) 2012
(a) Tuesday (b) Monday
(a) Wednesday (b) Saturday (c) Sunday (d) Saturday
(c) Tuesday (d) Thursday
158 How to Crack Test of Reasoning VERBAL

23. If 18th February, 1997 falls on Tuesday, then 30. Mrs. Susheela celebrated her wedding
what will be the day on 18th February, 1999? anniversary on Tuesday, 30th September,
(a) Monday (b) Tuesday 1997. When will she celebrate her next
(c) Thursday (d) Friday wedding anniversary on the same day?
24. If, in year 2000, birthday of Netaji be (a) 30th September, 2003
Wednesday, what was the Independence (b) 30th September, 2004
Day in that year? WBPSC 2018
(c) 30th September, 2002
(a) Wednesday (b) Thursday (d) 30th October, 2003
(c) Friday (d) Saturday 31. If the Republic Day of India in 1980 falls on
25. If 27 March, 1995 was a Monday, then what Saturday, X was born on March 3, 1980 and
day of the week was 1November, 1994? Y is 4 days older to X, then Y ’s birthday fell
MAT 2013 on IB (ACIO) 2013
(a) Sunday (b) Monday (a) Thursday
(c) Tuesday (d) Wednesday (b) Friday
26. What was the day on 2nd July, 1984? (c) Wednesday
(a) Wednesday (b) Tuesday (d) None of these
(c) Monday (d) Thursday 32. Nasir’s birthday is on Thursday May 18. On

A
27. It is observed that January 1, 2023 is a what day of the week will be Rehan's

M
Sunday. In which year again the January 1st Birthday in the same year if Rehan was born
on August 19? SSC (CGL) 2017

H
will fall on a Sunday? MPPSC 2018
(a) 2029 (b) 2030 (a) Wednesday

TT
(c) 2034 (d) 2037 (b) Saturday
(c) Friday
28. The year next to 1990 will have the same
calendar as that of the year 1990.
(d) Thursday
A
W
(a) 1995 (b) 1997 (c) 1996 (d) 1992 33. The birthday of Y was celebrated six days
SH

before X, who was born on October 4, 1999.


29. I was born on August 11. Mohan is older The independence day of that year fell on
than me by 11 days. This year’s Sunday. On which day did Y celebrate her
A

independence day falls on Monday. The day birthday, if it was not a leap year? CLAT 2017
@

on which Mohan’s birthday will fall this year (a) Wednesday


will be (b) Tuesday
(a) Monday (b) Tuesday (c) Monday
(c) Sunday (d) Thursday (d) Sunday

Answers
1 (d) 2 (a) 3 (a) 4 (b) 5 (c) 6 (a) 7 (b) 8 (d) 9 (d) 10 (d)
11 (b) 12 (b) 13 (b) 14 (b) 15 (d) 16 (c) 17 (d ) 18 (b) 19 (a) 20 (a)
21 (a) 22 (a) 23 (c) 24 (c) 25 (c) 26 (c) 27 (c) 28 (c) 29 (c) 30 (a)
31 (a) 32 (b) 33 (b)
Chapter 14 CLOCK AND CALENDAR

Answer with Explanations


15 min are gained in  × 15 min = 16 min
1. (d) The hands of the clock point in opposite 60 4
directions (in the same straight line) once, in every  55  11
hour but in between 5-7, they point opposite to 4
Hence, the hands will coincide at 16 min past 3.
each other only once at 6 O’ clock. Hence, there 11
are 22 instances, where they point opposite to 9. (d)
each other in 24 hours. 11 12 1
2. (a) Between 4 pm to 10 pm, the hands of a clock 10 2
are at right angle 11 times as between 8 and 9 3
10 O’clock there is a common position at 8 4
9 O’clock. 7 6 5
3. (a) Angle traced by hour hand in 12h = 360°
360° From the figure, we find that, minute hand is
∴ Angle traced by hour hand in 1 h = = 30°
12 25 min space apart from hour hand. In order to
coincide, it has to gain 25 min space. Now, 55 min
Time period between 8 O’clock and 2 O’ clock = 6 h
are gained by minute hand in 60 min. 25 min will
Hence, angle traced by hour hand in 6h
60 3
= 6 × 30° = 180° × 25 = 27

A
be gained in min
55 11

M
11 3
4. (b) We know that, θ = m − 30 h So, the hands will coincide at 27 min past 5.
2 11

H
Here, h = 8 and m = 30 10. (d)

TT
11 11 12 1 11 12 1
∴ θ= × 30 − 30 (8) 10 2 10 2
2
⇒ θ = 75°
9
A 3 9 3
W
8 4 8 4
5. (c) We know that, θ= 11 m − 30 h 7 6 5 7 6 5
SH

2
Fig. (i) Fig. (ii)
Here, m = 30 and h = 5
At 4 O’clock, the hands are 20 min spaces apart
A

11
∴ θ= × 30 − 30 × 5 from each other and to be in the opposite direction
2
@

they needs to be 30 min spaces apart.


⇒ θ = 15° From fig. (i) and fig. (ii), it is clear that minute hand
11 has to travel.
6. (a) We know that, θ = m − 30 h
2 (20 + 30) min space in order to be in opposite
Here, m = 25, and h = 3 direction of each other.
11 1°
∴θ = × 25 − 30 × 3 ⇒ θ = 47 Now, 55 min space is gained in 60 min.
2 2 Therefore, 50 min space will be gained in
7. (b) The clock loses 10 minutes per hour.  60 × 50 min or 54 6 min.
 
So, in one hour the clock will move 50 minutes  55  11
or, we can say that after 1 hour the clock will show Hence, the hands of the clock will be in opposite
time as 12:50.
6
direction at 54 min past 4.
11 11
Now, θ= m − 30h Here, m = 50 and h = 12
2 11. (b)
11 12 1 11 12 1 11 12 1
11
∴ θ= × 50 − 30 × 12 ⇒θ = 85° 10 2 10 2 10 2
2 9 3 9 3 9 3
8. (d) At 3 O’clock, the minute hand is 15 min away 8 4 8 4 8 4
7 6 5 7 6 5 7 6 5
from the hour hand. To coincide, it must gain 15 min.
As, 55 min are gained in 60 min. Fig. (i) Fig. (ii) Fig. (iii)
160 How to Crack Test of Reasoning VERBAL

27 60 × 24  4320
Gain in 24 h = 
8
At 5, both the hands are 25 min apart and to be at × = = 56 min
 11 63  77 77
right angle both the hands have to be 15 min apart
as shown in fig. (ii) and fig. (iii). Since, we have to As result is positive, therefore clock gains
8
take the position of clock between 5:30 and 56 min.
6 O’clock, therefore the positions of hands of clock 77
as above fig. (iii) is our answer. 15. (d) Three days after Saturday is Tuesday and
Now, it is clear from fig. (i) and fig. (iii) that minute Tuesday is a day before a day before yesterday.
hand needs to travel (25 + 15) = 40 min space in So, yesterday is Thursday.
order to form a right angle with the hour hand. Hence, today is Friday.
55 min space is gained in 60 min. Therefore, 16. (c) The last day of century cannot be either
Tuesday, Thursday or Saturday.
40 min spaces will be gained in  60 × 40 min or 363
 55  17. (d) Here, = 51 weeks + 6 odd days
7 7
43 min. Therefore, the hands are at right angle
11 ∴ Thursday + 6 days = Wednesday
7 18. (b) Each day of the week is repeated after 7 days.
at 43 min past 5. So, after 63 days, it will be Monday.
11
Therefore, after 61 days, it will be Saturday.
12. (b) According to the question,
19. (a)Q Saturday is on January 1, 2000.
First clock gains = 3 min/h

A
∴ Day on January 1,2001= Saturday + 2 =Monday
and second clock gains = 5 min/h

M
[Q 2000 is a leap year]
Total time from 9 am to 6 pm on Monday = 9 h
20. (a) Total number of days in a leap year from January

H
So, first clock gains in 9h = (9 × 3) min = 27 min
1 to March 1 = 30 + 29 + 1 = 60 days

TT
and shows the time as 6 : 27 pm. Also, the second
clock gains in 9 h = (9 × 5) min = 45 min and shows 60 ÷ 7 = 8 weeks and 4 odd days
the time as 6 : 45 pm. So, the fourth day from Friday = Tuesday
13. (b) Time from 5 am of a particular day to 10 pm A
21. (a) Friday will fall on 3,10, 17, 24, 31
W
on the 4th day is 89 h. Now, the clock loses So, it will be 5th Friday on 31st.
SH

16 min in 24 h or in other words, we can say that 22. (a) Day on 26th January 1996 = Friday
23 h 44 min of this clock is equal to 24 h of the Number of days from 26th January to 15th August
A

correct clock or  23 +
44  in a leap year
 = 5 + 29 + 31 + 30 + 31 + 30 + 31 + 15 = 202
 60 
@

= 28 weeks + 6 odd days


356
⇒ h of this clock = 24 h of the correct clock ∴ Day on 15th August 1996 = Thursday
15
Now, number of odd days from 15th August 1996
∴ 89 h of this clock to 15th August 2000 =1 + 1 + 1 + 2 = 5
24 × 15
=  × 89 h of correct clock ∴Day on 15th August 2000 = Thursday + 5
 356  = Tuesday
= 90 h of the correct clock 23. (c) 18th February, 1997 is Tuesday. So, 17th
Therefore, it is clear that in 90 h this clock loses 1 h February, 1998 will also be Tuesday.
and hence the correct time is 11 pm when this
Again 16th February, 1999 will also be Tuesday.
clock shows 10 pm.
Hence, 18th February, 1999 will be Thursday.
14. (b) As we know that, in a correct clock, the minute
hand gains 55 min spaces over the hour hand in 24. (c) We know that, the birthday of Netaji is
60 min. celebrated on 23rd Jan.
To be together again, the minute hand must gain Number of odd days between 23rd Jan and 15th
60 min over the hour hand. August 2000
= 8 + 29 + 31 + 30 + 31 + 30 + 31 + 15
60 min are gained in  × 60 min = 65 min
60 5
 55  11 = 205 ⇒ 29 weeks + 2 odd days
But they are together after 63 min ∴ The day on 15th August 2000
= Wednesday + 2 odd days
∴Gain in 63 min =  65 − 63 = 2 =
5 5 27
min
 11  11 11 = Friday
Chapter 14 CLOCK AND CALENDAR 161

25. (c) Here, 27th March 1995 was Monday. Now, for The sum of odd days, so calculated from
calculating total number of odd days. First, we years 1990-95 = (1 + 1 + 2 + 1 + 1 + 1)
calculate total number of days till 1 November 1994. = 7 days = 0 odd day
∴ Number of days in March 1995 = 27 Hence, the year 1996 will have the same calendar
as that of the year 1990.
Number of days in February 1995 = 28
Number of days in January 1995 = 31 29. (c) Mohan was born on 31st July, August 15 falls
on Monday. So, Monday falls on 1 August and
Number of days in December 1994 = 31
8 August. Now, 31st July will be Sunday.
Number of days in November 1994 = 29
30. (a) Total number of odd days
Total = 146 = 20 week + 6 odd days
30 September, 1997-98 = 1
Now, on November 1, 1994 = Monday – 6
30 September, 1998-99 = 1
= Tuesday
30 September, 1999-2000 = 2
26. (c) Number of odd days in 1600 yr = 0 30 September, 2000-01 = 1
Number of odd days in 300 yr = 1 30 September, 2001-02 = 1
∴ Number of odd days in 1900 yr = 1 30 September, 2002-03 = 1
Now, in 83 yr, there are 20 leap years and 63 =1+ 1+ 2 + 1+ 1+ 1 = 7
ordinary years.
∴Tuesday + 7 = Tuesday
∴ Number of odd days in 83 yr = 40 + 63 = 103

A
= 14 weeks + 5 odd days. So, the next Tuesday will come on the

M
∴ Number of odd days in 1983 yr = 5 + 1= 6 Mrs. Susheela’s wedding anniversary on 30th
September, 2003.

H
Number of odd days from 1st January 1984 to 2nd
July 1984 = 31 + 29 + 31 + 30 + 31 + 30 + 2 31. (a) Republic day in 1980 i.e. January 26, 1980

TT
= 184 = 26 weeks + 2 odd days = Saturday
∴ Total number of odd days = 6 + 2 = 8 Number of odd days till March 3, 1980
= 1week + 1 odd day A
5 + 29 + 3 = 37 = 5 weeks + 2 odd days
W
Day related to 1 odd day = Monday ∴March 3 is Monday and Y is 4 days older to X.
SH

Hence, the day on 2nd July 1984 was Monday. So, his birthday must be four days ahead of X, i.e.
on Thursday.
27. (c) The year 2023 will have 365 days i.e., 1 odd day.
32. (b) Day on May 18 = Thursday
A

The year 2024 will have 366 days i.e. 2 odd days.
Number of odd days from May 18 to August 19
@

Likewise year, 2025, 2026, 2027, 2029, 2030, 2031


= 13 + 30 + 31 + 19 = 93
and 2033 will have 1 odd day and year, 2028 and
2032 will have 2 odd days. = 13 weeks + 2 odd days
Hence, day on August 19 = Thursday + 2
The sum of odd days, so calculated from years
= Saturday
2023 – 2033 =
1+ 2 + 1+ 1+ 1+ 2 + 1+ 1+ 1+ 2 + 1 33. (b) The birthday of Y was celebrated six days
before October 4, 1999, i.e. September 28.
= 14 days = 0 odd days
Now, day on 15th August 1999 = Sunday
Hence, the year 2034 will have the same calendar
as that of the year 2023. Number of odd days from 15th August 1999 to
28th September 1999 = 16 + 28 = 44
28. (c) The year 1990 has 365 days, i.e. 1 odd day,
= 6 weeks + 2 odd days
year 1991 has 365 days, i.e. 1 odd day, year 1992
∴ Day on 28th September 1999 = Sunday + 2
has 366 days i.e. 2 odd days. Likewise year 1993,
= Tuesday
1994, 1995 have 1 odd day each.
162 How to Crack Test of Reasoning VERBAL

15
Sitting
Arrangement

A
M
Sitting arrangement is a sequential Before solving this type of questions, it is necessary

H
arrangement of objects/persons on the to know the given facts

TT
basis of predefined conditions. l
When it is not given in the question that in which
In this chapter, we deal with the direction the person(s) is facing, then we assume
questions based on sitting arrangement of A
it facing the North direction.
W
persons or objects. In this type of l
When A, B, C and D are facing in South direction
SH

questions, some conditions are given, on and P, Q, R and S are facing in North direction in
the basis of which candidates are required a line, then positions to their right and left will be
A

to arrange objects/person, either in a row


or in a circular order or in any other Southward facing
@

geometrical shape. While making Right A B C D Left North


arrangements, it should be noted that all
the conditions given are complied with. West East

Left P Q R S Right
South
Northward facing
Types of Questions l
When A, B are in one line and P, Q in A B
There are three types of questions, which other, then diagonally opposite
are generally asked in various competitive directions will be P Q
examinations.
A is diagonally opposite to Q and B is
diagonally opposite to P.
Type 1
Illustration 1. P, Q, R, S and T are sitting in a line
Arrangement in Linear Pattern facing West. P and Q are sitting together. R is sitting
at South end and S is sitting at North end. T is
Here, we deal with questions, where a
neighbour of Q and R. Who is sitting in the middle?
proper arrangement is required to be done SSC (Steno) 2017
in a linear format along a straight line. (a) P (b) Q (c) R (d) S
Chapter 15 SITTING ARRANGEMENT 163

Solution. (b) According to the given information, the 2. (d) It is clear from the diagram that there are four
sitting arrangement is as given below. persons to the right of D—A, C, E and B.
S 3. (a) It is clear from the diagram that F is sitting to the
left of D.
P N
4. (a) It is clear from the diagram that A is at the
immediate left of C.
W E
Q 5. (b) It is clear from the diagram that B is at the right
end.
T S
Illustration 6. Six persons A, B, C, D, E and F sit
R in two rows of three persons each. If E is not at
any end of rows, D is second to the left of F, C is
Clearly, from the above diagram, Q is sitting in the the neighbour of E and is sitting diagonally
middle. opposite to D and B is the neighbour of F, then
who will sit opposite B? SSC (CGL) 2015
Directions (Illustrations 2-5) Study the
following information carefully and answer the (a) A (b) E (c) C (d) D
questions given below. Solution (b) According to the question, the

A
arrangements are as follows
Six friends A, B, C, D, E and F are sitting in a

M
A E C F B D
row facing towards North. C is sitting between A Row 1

H
and E. D is not at the extreme end. B is sitting at Row 2
immediate right of E. F is not at the right end but D B F C E A

TT
D is sitting at 3rd left of E. Hence, E is sitting opposite to B.
Illustration 2. How many persons are there to the A
Note Linear arrangement also includes floor
W
right of D? arrangement in which people are arranged as
(a) One (b) Two (c) Three (d) Four per their floor numbers from top to bottom. The
SH

(e) None of these lowest floor is usually numbered 1.

Illustration 3. Which of the following is sitting to Directions (Illustration 7-9) Study the
A

the left of D? following information carefully and answer the


@

(a) F (b) C (c) E (d) A


questions given below.
(e) None of these There are eight persons namely S, T, U, V, W, X,
Y and Z lives on eight different floors from one to
Illustration 4. Who is at the immediate left of C? eight. Ground floor is number one and one above
(a) A (b) E (c) Either E or A that is number two and so on till the topmost floor
(d) Cannot be determined is number eight. X lives on odd number floor but
(e) None of these does not live on 3rd floor. Z lives immediate below
Illustration 5. Who is at the right end? X. More than two person lives between Z and Y.
(a) A (b) B (c) E There are six person lives between S and Y. V
(d) Cannot be determined lives immediate above W, but live below T. U does
(e) None of these not live above X. W does not live immediate
Solutions (Illustrations 2-5) According to the given above Y. IBPS (Clerk) Pre 2017
information, the sitting arrangement of six friends is Illustration 7. Who lives on floor number five?
as given below
(a) U (b) S (c) Z (d) T
N (e) None of these
F D A C E B Illustration 8. How many persons live between
W E
W and X?
Facing North S (a) One (b) Three (c) Five (d) Two
(e) None of these
164 How to Crack Test of Reasoning VERBAL

Illustration 9. Who lives immediately above V? Left or P


anti-clockwise Right or
(a) Z (b) T (c) Y (d) W direction clockwise
(e) None of these direction
Solutions (Illustration 7-9) According to the given S Q
information, arrangement is as follows
Floor Person
R
8 S
Note When it is not given in the question that in which
7 X
direction the persons are facing, then we
6 Z assume it facing the centre.
5 T
4 V Illustration 10. Five students are standing in a
3 W
circle. Abhinav is between Alok and Ankur.
2 U
Apurva is on the left of Abhishek. Alok is on
the left of Apurva. Who is sitting next to
1 Y
Abhinav on his right?
7. (d) T lives on floor number five. (a) Apurva (b) Ankur
8. (b) Three persons live between W and X. (c) Abhishek (d) Alok

A
9. (b) T lives immediately above V. Solution (d) We arrange the following students in a

M
circle based on the information given in the

H
question.
Type 2 Abhishek Apurva

TT
Circular Arrangement
In circular arrangement, persons/objects are A
W
arranged around a circle facing towards or
SH

outside the centre on the basis of information Ankur Alok


provided. Abhinav
A

Facing Towards the Centre Clearly, Alok is sitting next to Abhinav on his right.
@

Suppose A, B, C and D are four persons facing Directions (Illustration 11-13) Study the
towards the centre, then their right and left following information carefully and answer the
sides are shown by the arrows in the given given questions. NABARD (PO) 2009
diagram. B is on the right side of A and D is Eight persons I, J, K, L, M, N, O and P are sitting
on the left side of A. around a circular table but not necessarily in the
same order. Three of them are facing outwards
C
while five are facing towards the centre.
M is third to the right of K. N is sitting third to
Centre the left of M. Three persons are sitting between N
D of B
and J. O is sitting third to the right of N, who is
Left or circle not facing the centre. L is sitting third to the
Right or
clockwise anti-clockwise right of I, who is not facing the centre.
direction A direction
Illustration 11. Who sits between N and I?
(a) J (b) K (c) M (d) O
Facing Outside the Centre (e) None of these
Suppose there are four persons P, Q, R and S.
All the persons are facing outside the centre, Illustration 12. Who among the following is
then their left and right sides are shown in the second to the right of J?
diagram by arrows. Q is on the right of P and (a) L (b) K (c) M (d) N
S is on the left of P. (e) None of these
Chapter 15 SITTING ARRANGEMENT 165

Illustration 13. Which of the following Left Right


statements is/are true with respect to P? A
Right Left
(a) P is opposite of O
(b) P is fourth to the right of O Left D B Right
(c) P is fourth to the left of O
(d) All are true Left C Right
(e) None of the above Right Left
Solutions. (Illustration 11-13) According to the given Facing the direction opposite to centre
information the final arrangement of eight persons
around a circular table is shown below. 2. Pentagonal Arrangement
O In this arrangement, persons/objects are
K J placed around a pentagonal plane facing
either at the centre or the direction opposite to
L M the centre. The left and right of each person in
both the cases can be understood with the
help of the following diagrams
N I
P Right Left

A
11. (e) P is sitting between N and I. A

M
12. (b) K is sitting second to right of J. Left Right

H
13. (d) All statements are true with respect to P.

TT
Right E B Left
Right Left
Type 3 Left A
W
D C Right
Arrangement in Other Geometrical
SH

Shapes Right Left


Facing the centre
A

In this type, questions are asked to arrange


the persons in square, rectangle, pentagon,
@

Left Right
hexagon or any other geometrical shape.
A
1. Square/Rectangular Right Left
Arrangement Left E B Right
In this type of arrangement, objects/persons
Left Right
are placed around a rectangular or a square
shaped table facing either at the centre or the Right D C Left
direction opposite to centre. The left and right
of each person in both the cases can be Left
Right
understood with the help of following Facing the direction opposite to centre
diagrams
Left
Right
A 3. Hexagonal Arrangement
Left Right
In this arrangement, objects/persons are
Right D B Left placed around a hexagonal plane facing either
at the centre or the direction opposite to
Right Left
C centre. The left and right of each person/
Left Right object in both the cases can be understood
Facing the centre with the help of following diagrams
166 How to Crack Test of Reasoning VERBAL

Right Left neighbour of V. Only one person sits between V


Right and W. S sits second of right of Q. Q faces the
Left A centre. R is not an immediate neighbour of P.
F B Illustration 15. Who sits second to the left of Q?
Right Left
Right Left (a) V (b) P (c) T (d) Y
Left Right
(e) Cannot be determined
E C
Illustration 16. What is the position of T with
Right D Left
respect of V?
Left Right (a) Fourth to the left (b) Second to the left
(c) Third to the left (d) Third to the right
Facing the centre
(e) Second to the right
Left Right Illustration 17. Four of the following five are
Right A Left alike in a certain way and so form a group.
Which is the one that does not belong to that
F B group?
Left Right
Left Right (a) R (b) W (c) V (d) S
Right Left

A
C (e) Y
E

M
Solutions. (Illustration 15-17) On the basis of given
D Right

H
Left information, the seating arrangement is as given
below

TT
Right Left R
T S
Facing the direction opposite to centre
A Y V
W
Illustration 14. A, B, C and D are playing cards
P Q
on a square table.
SH

W
A and C and B and D are partners. D is to the 15. (b) Q faces centre and P is second to the left of Q.
right of C. The face of C is towards West. Find
A

the direction that D is facing. 16. (c) T is third to the left of V.


@

(a) West (b) East (c) South (d) North 17. (d) R, W, V and Y are sitting in the middle of the
sides of the table. S is sitting at the corner.
Solution (c) Arrangement according to the question is
as follows Illustration 18. A, B, C, D, E, F, G and H are eight
D N girls who are sitting around a octagonal
enclosure facing the centre. B is sitting between
Right A C Left W E
G and D, H is third to the left of B and second to
B S
the right of A, C is sitting between A and G, B
Clearly, D is facing South. and E are not sitting opposite to each other.
Who is sitting third to the left of D?
Directions (Illustration 15-17) Study the (a) H (b) G (c) F (d) E
following information carefully and answer the
Solution (c) Following sitting arrangement can be
questions given below. IBPS (PO) 2011
drawn from the given instructions as given below
Eight friends P, Q, R, S, T, V, W and Y are A C
sitting around a square table in such a way
that four of them sit at four corners of the
F G
square while four sit in the middle of each of
the four sides. The ones who sit at the four Right Left
corners face the centre while those who sit in H B
the middle of the sides face outside.
E D
P who faces the centre sits third to the right of
V.T who faces the centre, is not an immediate Clearly, F is sitting third to the left of D.
Chapter 15 SITTING ARRANGEMENT

Let us Practice
Base Level Exercise
1. There are five different houses, A to E, in a R doesn’t have S to his right. Who is
row. A is to the right of B and E is to the left standing on the West end? SSC (CPO) 2017
of C and right of A, B is to the right of D. (a) S (b) S or U
Which of the house is in the middle? (c) T (d) T or U
IB (ACIO) 2013
(a) A (b) B (c) C (d) D
Directions (Q. Nos. 8-9) Read the following
passage below and answer the given questions.
2. A, B, C, D, E and F are sitting in a row. ‘E’ Five friends are standing in a row facing South.
and ‘F’ are in the centre and ‘A’ and ‘B’ are Jayesh is to the immediate right of Ashok. Pravir
at the ends. ‘C’ is sitting on the left of ‘A’. is between Bablu and Suresh. Suresh is between
Then, who is sitting on the right of ‘B’? Jayesh and Pravir. DSSSB 2017

A
SSC (Multitasking) 2014
8. Who is at the extreme left end?

M
(a) A (b) D (c) E (d) F
(a) Ashok (b) Pravir
3. P, Q, R, S and T are sitting together. T is at

H
(c) Bablu (d) Suresh
one extreme end. P is the neighbour of T and

TT
9. Who is in the middle?
is third to the left of Q. Who is fourth to the (a) Bablu (b) Suresh (c) Pravir (d) Jayesh
right of T? SSC (10+2) 2017
(a) P (b) T (c) Q (d) S A
Directions (Q. Nos. 10-11) Read the following
W
information and answer the given questions.
4. In a gathering seven members are sitting in a SNAP 2014
SH

row. ‘C’ is sitting left to ‘B’ but on the right There are 3 females A, B and E and 4 males C, D,
to ‘D’. ‘A’ is sitting right to ‘B’. ‘F’ is sitting F and G standing in a straight line. No two
A

right to ‘E’ but left to ‘D’. ‘H’ is sitting left to


females are together. B is to right of C, F and D
‘E’. Find the person sitting in the middle.
@

SSC (10+2) 2013


are not together as A is placed between them. G is
(a) C (b) D (c) E (d) F not near B or E but E and F are together. D is not
to the right of B. E is not sitting at the extreme
5. Who among P, Q, R, S, T is in the exactly end.
middle while standing in a line? CGPSC 2018
10 Who is in the extreme end?
I. Q is to the immediate right of R.
(a) G and B (b) C and F
II. T is exactly between P and R.
(c) B and D (d) None of these
III. Q is exactly between R and S.
(a) P (b) Q (c) R (d) S 11. Who is exactly in the middle?
(e) None of these (a) A (b) F
6. Six persons, P, A, V, K, J and C are sitting in (c) E (d) None of these
a row facing North. C is to the immediate Directions (Q. Nos. 12-14) Study the following
left of V who is third to the right of P. J is to information to answer the given questions.
the immediate left of K and is at one of the SBI (PO) Pre 2015
end. Who is to the immediate left of A? Eight friends G, H, I, J, N, O, P and Q are seated
SSC (10+2) 2017
in a straight line, but not necessarily in the same
(a) C (b) P (c) K (d) J
order. H sits second to right of O. O sits at one of
7. 6 boys P, Q, R, S, T and U are standing in a the extreme ends of the line. Only three people sit
row facing North. P and Q cannot be either between H and N. I sits third to the left of J. Only
at 1st or 2nd place. R and S will always be two people sit between J and G. P is not an
together and R must be at one of the ends. immediate neighbour of N.
168 How to Crack Test of Reasoning VERBAL

12. Who amongst the following represents the 18. If we start counting from the left, on which
person seated at the extreme right end of number is C?
the line? (a) 1st (b) 2nd
(a) P (b) Q (c) O (d) I (c) 3rd (d) 5th
(e) G (e) None of these
13. Who amongst the following sit exactly 19. Eight persons A, B, C, D, E, F, G and H are
between J and G? sitting around a round table each facing the
(a) O, Q (b) H, O (c) O, P (d) N, Q centre. D is second to the left of F and third to
(e) H, N the right of H. A is second to the right of F
14. What is the position of P with respect to N? and an immediate neighbour of H. C is second
(a) Third to the left (b) Second to the left to the right of B and F is third to the right of
(c) Fourth to the right (d) Third to the right B. G is not an immediate neighbour of F. In
(e) Second to the left the above information who is to the
immediate left of A? WBPSC 2018
Directions (Q. Nos. 15-16) Read the following (a) H (b) E
information carefully and answer the questions (c) G (d) B
that follow.
20. 5 girls A, R, D, S and V are sitting around a
(i) Six flats on a floor in two rows facing pentagonal table. A is between R and D. S is

A
North and South are allotted to P, Q, R, to the left of V. R is to the left of S. Who is to

M
S, T and U. the right of A?

H
(ii) Q gets a North facing flat and is not next (a) S (b) D

TT
to S. (c) R (d) V
(iii) S and U get diagonally opposite flats. 21. Six persons are seated around a hexagonal
(iv) R next to U, gets a South facing flat and A
table. Arup is seated opposite Belal, who is
W
T gets a North facing flat. between Chirag and Derek. Arup is between
15. Whose flat is between Q and S? Ela and Farook. Ela is to the left of Derek.
SH

(a) T (b) U (c) R (d) P Which of the following pairs is facing each
(e) Data inadequate other? WBPSC 2018
A

(a) Ela and Farook


16. The flats of which of the other pair than SU,
@

(b) Derek and Ela


is diagonally opposite to each other? (c) Chirag and Ela
(a) PT (b) QP (c) QR (d) TS (d) Chirag and Derek
(e) None of these
Directions (Q. Nos. 22-23) Read the following
Directions (Q. Nos. 17 and 18) These information carefully to answer the questions that
questions are based on the information given follow. MPPSC 2018
below. Six girls are sitting in a circle. Sonia is sitting
A group of seven singers, facing the audience, opposite to Radhika. Poonam is sitting right of
are standing in a line on the stage as follows Radhika but left of Deepti. Monika is sitting left of
(i) D is to the right of C. Radhika. Kamini is sitting right of Sonia and left
(ii) F is standing beside G. to Monika. Now, Deepti and Kamini, Monika and
(iii) B is to the left of F. Radhika mutually exchange their positions.
(iv) E is to the left of A. 22. Who will be opposite to Sonia?
(v) C and B have one person between them. (a) Radhika (b) Monika
(c) Kamini (d) Sonia
(vi) A and D have one person between them.
17. Who is on the extreme right? 23. Who will be sitting left of Kamini?
(a) Poonam (b) Deepti
(a) D (b) F (c) G (d) E
(c) Radhika (d) Sonia
(e) None of these
Chapter 15 SITTING ARRANGEMENT 169

Directions (Q. Nos. 24 and 25) Read the Directions (Q. Nos. 31-35) Study the following
following information carefully and answer the information carefully and answer the questions
questions given below it. SSC (Constable) 2012 given below.
A, B, C, D, E, F and G are playing cards sitting in A, B, C, D, E, F, G, H and J are sitting around a
a circle. circle facing at the centre. C is third to the left of
(i) F is 2nd to the right of G. A. E is fourth to the right of A. D is fourth to the
(ii) B is neighbour of F but not of E. left of J, who is second to the right of A. F is third
(iii) E, the neighbour of C, is 4th to the right to the right of B. G is not an immediate neighbour
of G. of A. NIACL (AO) 2014

(iv) D is between E and A. 31. What is H’s position with respect to E?


(a) Third to the left (b) Fourth to the left
24. Who is fourth to the left of G? (c) Fifth to the right (d) Fifth to the left
(a) D (b) E (c) C (e) Sixth to the left
(d) Cannot be determined
32. Who is third to the right of G?
25. Who is to the immediate left of G? (a) B (b) D
(a) A (b) C (c) A (d) Data inadequate
(c) B (d) None of these (e) None of these

A
Directions (Q. Nos. 26-30) Study the following 33. Who is fifth to the right of E?

M
information carefully and answer the questions (a) F (b) C (c) H (d) A
given below. (e) None of these

H
P, Q, R, S, T, V, W and Z are sitting around a

TT
34. Who is second to the left of H?
circle facing at the centre. R is fourth to the left of (a) A (b) F
P, who is second to the right of S. V is fourth to
the right of S. Q is fourth to the left of W, who is
(c) D
A
(e) None of these
(d) Data inadequate
W
not an immediate neighbour of P or S. Z is not an
35. Who is to the immediate right of D?
SH

immediate neighbour of R. NIACL (AO) Mains 2016


(a) F (b) C
26. Who is to the immediate right of V? (c) A (d) Data indequate
A

(a) R (b) W (e) None of these


(c) Z (d) Data inadequate
@

(e) None of these Directions (Q. Nos. 36-40) Study the following
information carefully and answer the given
27. Who is to the immediate right of R? questions.
(a) T (b) S (c) W
(d) Data inadequate (e) None of these Eight people J, K, L, M, N, O, P and Q are sitting
around a circular table with equal distance
28. In which of the following pairs is the first between each other, but not necessarily in the
person sitting to the immediate right of the same order. Q sits to the immediate left of N.
second person? Only four persons sit between Q and L. P sits
(a) VW (b) RT second to the right of M. M is not an immediate
(c) WR (d) QP neighbour of L. Only three people sit between O
(e) ZP and K. K is neither an immediate neighbour of M
29. Who is third to the right of R? nor Q. IBPS (Clerk) 2016
(a) P (b) S 36. Which of the following statements is not true
(c) Q (d) Data inadequate as per the given arrangement?
(e) None of these
(a) P sits third to the left of K
30. Who is second to the left of Z? (b) All the given statements are true
(a) Q (b) V (c) Both M and L are immediate neighbours of J
(c) S (d) W (d) Only three people sit between N and P
(e) None of these (e) K sits to the immediate left of L
170 How to Crack Test of Reasoning VERBAL

37. As per the given arrangement, J : L in the 39. Who sits exactly between the one who is to
same way as K : N. Following the same the immediate right of P and K when
pattern, Q : _ ? counted from the left of K?
(a) J (b) M (a) N (b) O (c) J (d) K
(c) K (d) P (e) Q
(e) O
40. In the given arrangement, if O interchanges
38. What is Q’s position with respect to J? its position with N and P interchanges its
(a) Second to the left position with K, then how many persons will
(b) Fourth to the left sit between P and N, when counted from the
(c) Second to the right right of P?
(d) Immediate right (a) Two (b) One (c) More than three
(e) Third to the right (d) Three (e) None of these

Expert Level Exercise


Directions (Q. Nos. 1-5) Study the following 4. Who among the following is to the
information to answer the given questions. immediate left of F?
SBI (Clerk) 2012

A
(a) G (b) J
(c) I (d) K

M
(i) Eight persons E, F, G, H, I, J, K and L are
(e) None of these
seated around a square table-two on each

H
side. 5. How many persons are seated between K

TT
(ii) There are three lady members and they and F?
are not seated next to each other. (a) One (b) Two

(iii) J is between L and F.


(c) Three A (d) Cannot be determined
W
(e) None of these
(iv) G is between I and F.
SH

(v) H, a lady member, is second to the left Directions (Q. Nos. 6-10) Read the information
given below to answer the questions.
of J.
A

A, B, C, D, E, F, G and H want to have a


(vi) F, a male member, is seated opposite to E,
dinner on a round table and they have worked out
@

a lady member. the following seating arrangements. CLAT 2013


(vii) There is a lady member between F and I.
(i) A will sit beside C
1. Who among the following are the three lady (ii) H will sit beside A
members? (iii) C will sit beside E
(a) E, H and J
(iv) F will sit beside H
(b) E, G and J
(c) G, H and J (v) E will sit beside G
(d) H, E and G (vi) D will sit beside F
(e) None of the above (vii) G will sit beside B
2. Which of the following is true about J? (viii) B will sit beside D
(a) J is a male member 6. Which of the following is wrong?
(b) J is a female member (a) A will be to the immediate right of C
(c) Sex of J cannot be determined (b) D will be to the immediate left of B
(d) Position of J cannot be determined (c) E will be to the immediate right of A
(e) All of the above (d) F will be to the immediate left of D
3. Who among the following is seated between 7. Which of the following is correct?
E and H? (a) B will be to the immediate left of D
(a) I (b) J (b) H will be to the immediate right of A
(c) F (d) K (c) C will be to the immediate right of F
(e) None of these (d) B will be to the immediate left of H
Chapter 15 SITTING ARRANGEMENT 171

8. A and F will become neighbours, if 15. Four of the following five are alike in a
(a) B agrees to change her sitting position certain way and thus from a group. Which is
(b) E agrees to change her sitting position the one that does not belong to that group?
(c) G agrees to change her sitting position (a) T-E (b) Q-C (c) S-B
(d) H agrees to change her sitting position (d) R-A (e) P-D
9. During sitting Directions (Q. Nos. 16-18) Study the information
(a) A will be directly facing C carefully and answer the questions given below.
(b) B will be directly facing C
(c) A will be directly facing B
(i) Eleven students A, B, C, D, E, F, G, H, I, J
(d) B will be directly facing D and K are sitting in a row of the class
facing the teacher.
10. H will be sitting between (ii) D, who is to the immediate left of F, is
(a) C and B (b) A and F (c) D and G (d) E and G
second to the right of C.
Directions (Q. Nos. 11-15) Study the following (iii) A is second to the right of E, who is at one
information to answer the given questions. of the ends.
IBPS (PO) 2012 (iv) J is the immediate neighbour of A and B
Ten people are sitting in two parallel rows and third to the left of G.
containing five people each, in such a way that (v) H is to the immediate left of D and third

A
there is an equal distance between adjacent to the right of I.

M
persons. In row 1, P, Q, R, S and T are seated and
all of them are facing South. In row 2, A, B, C, D
16. Who is sitting in the middle of the row?

H
and E are seated and all of them are facing North. (a) C (b) I (c) B

TT
(d) G (e) None of these
Therefore, in the given seating arrangement each
member seated in a row faces another member of 17. Which of the following groups of friends is
the other row. D sits third to the left of A. P faces A
sitting to the right of G?
W
immediate neighbour of D. R sits second to the (a) IBJA (b) ICHDF (c) CHDF (d) CHDE
SH

right of P. Only one person sits between Q and S, B (e) None of these
and E are immediate neighbours of each other. E
18. In the above sitting arrangement, which of
does not face P and Q.
A

the following statements is superfluous?


11. How many persons are seated between Q
@

(a) Only (i) (b) Only (ii)


and T? (c) Only (iii) (d) None is superfluous
(a) None (b) One (c) Two (d) Three (e) Cannot be determined
(e) Cannot be determined
Directions (Q. Nos. 19-21) Study the given
12. Four of the following five are alike in a information carefully to answer the given
certain way and thus form a group. Which is questions. RBI (Asst.) 2016
the one that does not belong to that group? Eight people A, B, C, D, E, F, G and H are sitting
(a) R (b) S (c) C (d) T
in a straight line facing North with equal
(e) A
distances between each other, but not necessarily
13. Who amongst the following represent the in the same order. G sits fourth to the left of D. B
people sitting exactly in the middle of the sits second to the right of D. Only two people sit
rows? between F and H. F sits at one of the positions to
(a) P, E (b) S, D (c) S, A (d) A, R the left of D. Neither F nor H sits at an extreme
(e) P, B end of the line. A sits at one of the positions to the
right of H. More than three people sit between A
14. Which of the following is true regarding B? and C.
(a) A and C are immediate neighbours of B
(b) B sits at one of the extreme ends of the line
19. As per the given arrangement, E : D in the
(c) Q faces B same way as B : A. Following the same
(d) T is an immediate neighbour of the pattern, H : ……… ?
person facing B (a) E (b) B (c) G (d) C
(e) D sits to the immediate left of B (e) F
172 How to Crack Test of Reasoning VERBAL

20. How many persons sit between C and G? 26. In which of the given pairs of people, is even
(a) Three (b) More than three number of people living between them?
(c) Two (d) None of these
(a) G, C (b) H, E
(e) One
(c) E, D (d) I, H
21. If all the people are made to sit in (e) B, I
alphabetical order from right to left, the
position’s of how many people will remain Directions (Q. Nos. 27-31) Study the following
unchanged? information to answer the given questions.
IBPS RRB (Pre) 2016
(a) One (b) Three
(c) Two (d) More than three Nine persons, E, F, G, H, K, L, M, N and O are
(e) None of these seated in a straight line facing North, with equal
distance between each other, but not necessarily
Directions (Q. Nos. 22-26) Study the following in the same order.
information and answer the questions. Only two people sit between E and the one sitting
IBPS RRB (Pre) 2016
at extreme ends of the line. K sits second to the
Nine friends A, B, C, D, E, F, G, H and I live on right of E. H sits fourth to the left of M. M does
nine different floors of a building, but not not sit at any of the extreme ends of the line. M is
necessarily in the same order. The lower most not an immediate neighbour of E. The number of
floor of the building is numbered one, the one

A
people sitting between H and K is double than
above that is numbered two and, so on till the that between M and O. More than two people sit

M
topmost floor is numbered nine. I lives on floor between G and E. G is not an immediate

H
numbered six. neighbour of M. F is an immediate neighbour of

TT
E lives on an odd numbered floor above I. Only L, but not H.
three people live between E and G. A lives on an 27. Four of the following five are alike in a
even numbered floor immediately below D, but not
on the floor numbered eight. Only one person lives A
certain way based on the given arrangement
W
and thus form a group. Which is the one that
between A and F. C lives on one of the floors below
does not belong to that group?
SH

F. The number of people living above C is equal to


(a) N, E (b) M, H (c) K, L (d) O, N
the number of people living below H.
(e) F, O
A

22. Four of the following five are alike in a


28. Which of the following is true with respect to
@

certain way as per the given arrangement


and thus form a group. Which of the N as per the given arrangement?
(a) N is an immediate neighbour of H
following does not belong to that group?
(b) N sits second to the left of M
(a) HB (b) FC (c) DE (d) EI
(c) N sits at one of the extreme ends of the line
(e) AG
(d) More than two people sit between N and G
23. How many persons live between A and the (e) None of the given options is true
person living on the floor numbered seven? 29. Who sits second to the left of H?
(a) More than three (b) Two (c) One (a) N (b) K (c) F (d) E
(d) None (e) Three (e) No one as H sits at one of the extreme ends of
24. H lives on which of the following floor the line
numbers? 30. Who amongst the following sit exactly
(a) Eight (b) One (c) Five (d) Nine between N and O?
(e) Other than those given as options (a) H, L (b) F, O
25. Which of the following is not true about B as (c) F, E (d) M, K
per the given arrangement? (e) K, O
(a) B lives on an even numbered floor 31. In which of the given pairs of people, is even
(b) All the given statements are true number of people sitting between them?
(c) Only one person lives between B and E (a) G, E (b) K, L
(d) H lives immediately above B (c) F, M (d) O, M
(e) Only one person lives above B (e) K, N
Chapter 15 SITTING ARRANGEMENT 173

Directions (Q. Nos. 32-35) Study the following Directions (Q. Nos. 36-40) Study the following
information to answer the given questions. information to answer the given questions.
IBPS (PO) 2014 IBPS (PO) Pre 2017
A, B, C, D, E, F, G and H are sitting in a straight Eight friends G, H, I, J, K, L, M and N are seated
line equidistant from each other (but not in a straight line with equal distance between
necessary in the same order). Some of them are each other, but not necessarily in the same order.
facing South while some are facing North. Some of them are facing North while some are
Note Facing the same direction means, if one is facing South.
facing North, then the other also faces North and K is an immediate neighbour of the person sitting
vice-versa. Facing the opposite direction means, if at an extreme end of the line. Only three people
one is facing North, then the other faces South sit between K and M. J sits second to the right of
and vice-versa. M. J does not sit at an extreme end of the line. N
H faces North. C sits at one of the extreme ends of sits to the immediate left of G. N is not an
the line. A sits third to the left of C. D is not an immediate neighbour of M. Immediate
immediate neighbour of C. C sits third to the neighbours of G face opposite directions (i.e. if one
right of A. B sits to the immediate right of G. B neighbour faces North, then the other faces South
does not sit at any of the extreme ends of the line. and vice-versa.)
Only one person sit between F and D. G sits Person sitting at the extreme ends face opposite

A
second to the left of F. E sits second to the right of directions (i.e. if one person faces North, then the

M
B. Both the immediate neighbour of G face the other faces south and vice-versa). H sits second to
same direction. Both the immediate neighbour of the left of L. L faces North. L is not an immediate

H
A face the opposite direction. E faces the direction neighbour of K. Immediate neighbours of L face

TT
as B. H and C faces the opposite direction. the same direction (i.e. if one neighbour faces
32. Which of the following sits second to the North, then the other also face North and
right of F? A
vice-versa). Both K and H face a direction
W
(a) H (b) D opposite to that of J (i.e. if J faces North then K
and H faces South and vice-versa.)
SH

(c) C (d) G
(e) A 36. As per the given arrangement, which of the
following statements is true with respect to I?
A

33. Which of the following is true with respect to


(a) I sits at an extreme end of the line
@

the given information?


(b) K is an immediate neighbour of I
(a) G sits at one of the extreme ends of the line
(b) B sits exactly between A and F (c) Only four person sit between I and M
(d) I faces a direction to that of J
(c) B sits third to the right of E
(e) All the given statements are true
(d) A faces North
(e) A sits second to the left of B 37. Which of the following pairs represents the
immediate neighbours of J?
34. How many persons in the given
(a) L, I (b) K, L (c) L, H (d) I, K
arrangement are facing North?
(e) H, K
(a) Four
(b) One 38. What is the position of K with respect to L?
(c) Three (a) Immediate right (b) Second to the right
(d) Two (c) Third to the left (d) Third to the right
(e) More than five (e) Immediate left
35. Four of the following five are alike in a 39. Four of the given five are alike in a certain
certain way and so form a group. Which of way based on the given arrangement and
the following does not belong to the group? hence form a group. Which of them does not
(a) B, A (b) D, C belong to that group?
(c) E, B (d) A, E (a) JM (b) HL (c) HI (d) KL
(e) G, F (e) MG
174 How to Crack Test of Reasoning VERBAL

40. How many persons sit to the left of G? 44. How many persons sit to the left of T?
(a) One (b) Two (c) More than three (a) Three (b) None
(d) None (e) Three (c) More than three (d) One
(e) Two
Directions (Q. Nos. 41-45) Study the following
information to answer the given question. 45. Which of the following represents the
IBPS (PO) Pre 2016 immediate neighbours of P?
Eight friends P, Q, R, S, T, U, V and W are seated (a) R, T (b) S, V
in a straight line with equal distance between (c) W, P (d) T, S
each other, but not necessarily in the same order. (e) V, R
Some of them are facing North while some are
facing South. V is an immediate neighbour of the
Directions (Q. Nos. 46-50) Study the following
information to answer the given questions.
person sitting at an extreme end of the line. R sits IBPS (PO) Pre 2016
second to the left of V. Only one person sits
Eight people S, T, U, V, W, X, Y and Z are sitting
between R and T. As many people sits to the right
in a straight line equidistant from each other (but
of T as to the left of P. V and P face the same
not necessarily in the same order). Some of them
direction (i.e. if V faces North, then P also faces
are facing South while some are facing North.
North and vice-versa.)
S faces North. Only two people sit to the right of
Immediate neighbours of P face opposite

A
S. T sits third to the left of S. Only one person sits
directions (i.e. if one neighbour faces North, then

M
between T and X. X sits to the immediate right of
the other faces South and vice-versa.) Q sits
W. Only one person sits between W and Z. Both

H
fourth to the left of S. Q is not an immediate
the immediate neighbours of T face the same

TT
neigbhour of V.
direction. U sits fourth to the left of X. T faces the
Persons sitting at the extreme ends face opposite opposite direction as S. Y does not sit at any of
directions (i.e. if one person faces North, then the
A
the extremes ends of the line. V faces the same
W
other person faces South and vice-versa.) W faces direction as W. Both Y and U face the opposite
South. W does not sit at an extreme end of the direction of Z. U faces North.
SH

line. U sits to the immediate right of W. U and Q


face the same direction (i.e. if U faces North, then
46. How many persons in the given
arrangement are facing North?
A

Q also faces North and vice-versa.)


(a) More than four (b) Four (c) One
@

41. As per the given arrangement, which of the (d) Three (e) Two
following statements is not true with respect
to U?
47. Four of the following five are alike in a
(a) U sits at an extreme end of the line
certain way, and so form a group. Which of
(b) Only three persons sit between U and R the following does not belong to the group?
(c) U sits second to the left of T (a) W, X (b) Z, Y (c) T, S (d) T, Y
(d) All the given statements are true (e) V, U
(e) U is an immediate neighbour of Q 48. What is the position of X with respect to Z?
42. What is the position of Q with respect to W? (a) Second to the left (b) Third to the right
(a) Immediate left (b) Second to the right (c) Third to the left (d) Fifth to the right
(c) Third to the left (d) Third to the right (e) Second to the right
(e) Second to the left 49. Who amongst the following sits exactly
between Z and W?
43. Four of the following five are alike in a
(a) T (b) Y (c) X (d) W
certain way based on the given arrangement
(e) U
and hence form a group. Which of them does
not belong to that group? 50. Who is sitting second to the right of T?
(a) WS (b) QT (c) WR (d) UP (a) Z (b) V (c) X (d) W
(e) RV (e) None of these
Chapter 15 SITTING ARRANGEMENT 175

Directions (Q. Nos. 51-55) Study the given Directions (Q. Nos. 56-60) Study the following
information carefully and answer the given information to answer the given questions.
questions. IBPS (Clerk) 2016 IPPB (AM) 2016
Eight people are sitting in two parallel rows, Eight people R, S, T, U, V, W, X and Y are sitting
containing four people in such a way that there is around a square table (but not necessarily in the
an equal distance between adjacent persons. In same order), in such a way that four of them sit at
row-1, M, N, O and P are seated and all of them four corners while four sit in the middle of each of
are facing North and in row-2 W, X, Y and Z are the four sides. The ones sitting in the middle of
seated and all are facing South (but not the sides are facing outside (i.e. opposite to the
necessarily in the same order.) centre) and the ones sitting in the corners are
Thus, in the given arrangement each person facing the centre.
seated in a row faces another person of the other T sits in middle of one of the sides. Only two
row. O sits second to the right of one who sits at persons sits between T and Y. V sits second to the
an extreme ends of the line. O faces an immediate left X. X faces same direction as Y (i.e. if Y faces
neighbour of W. X sits to the immediate right of the centre, then X also faces the centre and
W. The one who faces X sits at one of the vice-versa). Only three persons sit between X and
positions to the left of P. P does not face W. M sits U. R is an immediate neighbour of U, but not Y.
to the immediate left of the one who faces Z. Only one person sits between R and S.

A
51. Which of the following represents the 56. Four of the following five are alike in a

M
immediate neighbours of the one who faces certain way based on the given arrangement

H
Z? and hence form a group. Which one of the
following does not belong to that group?

TT
(a) Only O (b) N and M
(c) Only N (d) Only M (a) U (b) R (c) Y (d) V
(e) M and P (e) X
A
57. What is the position of S with respect to R?
W
52. Which of the following represents the people
sitting to the immediate right of O and (a) Second to the right (b) Fourth to the right
SH

immediate right of Z respectively? (c) Third to the left (d) Third to the right
(a) P, W (b) N, W (e) Second to the left
A

(c) N, Y (d) P, X 58. Who sits exactly between T and Y when


@

(e) M, W counted from the left of T?


53. Who sits to the immediate right of the one (a) R, S (b) V, W (c) W, X (d) R, V
who faces an immediate neighbour of W? (e) U, V
(a) Only P (b) P and N 59. Which of the following is true regarding W
(c) Only M (d) Only O as per the given arrangement?
(e) M and P (a) W sits at one of the corners of the table
54. Which of the following represents the correct (b) W sits second to the left of S
position of N? (c) W is an immediate neighbour of both V and T
(a) Immediate left of the one who faces Y (d) None of the given options is true
(b) Second to the right of O (e) Only three people sit between W and Y
(c) Immediate left of P 60. How many people sit between U and V
(d) Second to the left of the one who faces Z when counted from the left of V?
(e) None of the given options (a) One
55. Who amongst the following faces P? (b) Three
(a) Either Z or W (b) X (c) More than three
(c) Z (d) Y (d) None
(e) W (e) Two
176 How to Crack Test of Reasoning VERBAL

Directions (Q. Nos. 61-65) Study the given Directions (Q. Nos. 66-70) Study the
information carefully to answer the given questions. information carefully and answer the questions.
IBPS RRB (Pre) 2016 SBI (PO) 2015
Eight people A, B, C, D, M, N, O and P are sitting N, O, P, Q, R, S, T and U are sitting around a
around a square table (but not necessarily in the circular area at equal distances between each
same order) in such a way that four of them sit at other, but not necessarily in the same order.
four corners while four sit in the middle of each of Some of the people are facing the centre while
the four sides. The ones sitting in the middle of the some face outside (i.e. in a direction opposite to
sides are facing the centre and the ones sitting at the centre).
the corners are facing outside (i.e. opposite to the R sits second to the right of T. T face the
centre) centre. O sits third to the left of R. R and O face
A sits in the middle of one of the sides. Only one opposite directions. Immediate neighbours of O
person sits between A and M. A sits third to the faces the centre. P sits second to the right of O.
right of B. Only three people sit between B and N. C U sits to the immediate left of P. N sits second
sits second to the right of N. O and C face the same to the left of Q. Q faces the same direction as O.
direction. M is not an immediate neighbour of O. P Q is not an immediate neighbours of T.
sits second to the right of D. Immediate neighbours R face opposite
61. Who is to the immediate left of M? directions. (i.e. if one neighbour faces the

A
(a) N (b) B centre the other neighbour faces outside and

M
(c) D (d) P vice-versa.)
(e) C 66. Four of the following five are alike in a

H
62. What is the position of O with respect to P? certain way based on the given seating

TT
(a) Fourth to the left arrangement and so form a group. Which
is the one that does not belong to that
(b) Third to the left
(c) Third to the right group? A
W
(c) Immediate right (a) U (b) S (c) P (d) N
SH

(e) Immediate left (e) Q


63. Which of the following statements is true with 67. Who sits second to the left of U?
A

respect to the given arrangement? (a) T (b) O (c) S


(a) None of the given statements is true
@

(d) Other than those given as options


(b) Only three people sit between O and D (e) Q
(c) D sits third to the left of A
(d) M sits at one of the corners of the table 68. As per the given seating arrangement,
(e) M is an immediate neighbour of P which of the following is true regarding S?
(a) U is one of the immediate neighbours of S
64. As per the given arrangement, four of the (b) S sits third to the right of R
following five are alike in a certain way and (c) S faces the centre
hence form a group. Which of the following (d) Only two people sit between S and T
does not belong to the group? (e) Only one person sits between S and N
(a) D (b) O
(c) P (d) A
69. What is P’s position with respect to N?
(e) M (a) Second to the left (b) Second to the right
(c) Third to the right (d) Third to the left
65. Who amongst the following sit exactly between (e) Fourth to the left
B and the one who sits to the immediate right
of N, when counted from the left of B?
70. How many people in the given
arrangement face outside?
(a) O, D (b) M, P
(a) Three (b) Two (c) Four (d) Five
(c) C, P (d) M, C
(e) One
(e) A, C
Chapter 15 SITTING ARRANGEMENT

Answers
Base Level Exercise
1. (a) 2. (b) 3. (c) 4. (b) 5. (c) 6. (b) 7. (d) 8. (a) 9. (b) 10. (a)
11. (b) 12. (e) 13. (d) 14. (a) 15. (a) 16. (b) 17. (c) 18. (c) 19. (b) 20. (c)
21. (c) 22. (b) 23. (a) 24. (c) 25. (a) 26. (b) 27. (a) 28. (e) 29. (c) 30. (a)
31. (a) 32. (e) 33. (d) 34. (b) 35. (a) 36. (d) 37. (d) 38. (b) 39. (a) 40. (d)

Expert Level Exercise


1. (d) 2. (a) 3. (d) 4. (b) 5. (c) 6. (c) 7. (b) 8. (d) 9. (c) 10. (b)
11. (c) 12. (b) 13. (e) 14. (e) 15. (d) 16. (b) 17. (c) 18. (d) 19. (b) 20. (d)
21. (d) 22. (c) 23. (b) 24. (d) 25. (c) 26. (d) 27. (d) 28. (e) 29. (c) 30. (d)
31. (a) 32. (b) 33. (e) 34. (a) 35. (d) 36. (e) 37. (c) 38. (c) 39. (c) 40. (a)

A
M
41. (e) 42. (e) 43. (c) 44. (e) 45. (e) 46. (d) 47. (a) 48. (b) 49. (a) 50. (c)

H
51. (e) 52. (a) 53. (e) 54. (d) 55. (d) 56. (b) 57. (a) 58. (c) 59. (d) 60. (a)

TT
61. (e) 62. (b) 63. (a) 64. (b) 65. (c) 66. (d) 67. (c) 68. (c) 69. (b) 70. (b)

A
W
Answer with Explanations
SH

Base Level Exercise


A

1. (a) Data is arranged as shown below 5. (c) According to the given information arrangement
@

D B A E C is as follows

P T R Q S
So, the house in the middle is A.
Hence, it is clear from above that R is exactly in
2. (b) According to the question, sitting arrangement the middle while standing in a line.
is as follows
B D E/F F/E C A 6. (b) According to the given information,
arrangement is as follows
Clearly, D is sitting on the right of B.
North
3. (c) According to the given information, J K P A C V
arrangement is as follows
Hence, P is at the immediate left of A.
T P R/S S/R Q 7. (d) According to the given information, the standing
It is clear from the diagram that Q sits fourth to arrangement is as given below.
the right of T. (↑)
T/U U/T P/Q Q/P S R
4. (b) According to the given information,
arrangement of the seven members in a row is It is clear from above diagram that T or U
as follows standing on the West end.
H E F D C B A

Hence, D is sitting in the middle.


178 How to Crack Test of Reasoning VERBAL

Solutions (Q. Nos. 8-9) According to the given 20. (c) Arrangement according to the question is as
information, arrangement is as follows follows
A

Bablu Pravir Suresh Jayesh Ashok South


8. (a) Ashok is at the extreme left end. R D
9. (b) Suresh is in the middle. Right Left
Solutions (Q. Nos. 10-11) According to the given
S V
information, the arrangement is as follows
Clearly, R is to the right of A.
G D A F E C B 21. (c) According to the given information,
10. (a) G and B are in extreme ends. arrangement is as follows
11. (b) F is exactly in the middle. Farook Chirag
Solutions (Q. Nos. 12-14) According to the given
information, the arrangement is as follows. Arup Belal

O I H P J Q N G Ela Derek

A
12. (e) G is seated at the extreme right end of the line. Hence, Chirag and Ela are facing each other.
13. (d) N,Q sit exactly between J and G.

M
Solutions (Q. Nos. 22-23) According to the given
14. (a) P is third to the left of N. information, the arrangement is as follows

H
Solutions (Q. Nos. 15-16) According to the given Radhika Monika

TT
information, the arrangement is as follows
N
South facing Right Poonam A Kamini Left
W
U R P W E
North facing
SH

Q T S
S Deepti Sonia
15. (a) Clearly, T’s flat is between Q and S.
Original positions
A

16. (b) As given in statement (iii), S and U are


Monika Radhika
@

diagonally opposite. Similarly, from the figure we


can say, P and Q are also diagonally opposite.
Solutions (Q. Nos. 17 and 18) According to the given
Right Poonam Deepti Left
information, the arrangement of group of seven
singers, facing the audience is as shown below
Kamini Sonia
E A C D B F G
17. (c) It is clear from the diagram that, singer G is at New arrangement
the extreme right of the line. 22. (b) Monika is opposite to Sonia.
18. (c) It is clear from the diagram that, C is the third 23. (a) Poonam is sitting left of Kamini.
from the left.
19. (b) According to the given information, the Solutions (Q. Nos. 24-25) According to the given
arrangement is as follows information, the sitting arrangement is as shown
G below
B H D
A E
D A
G C
C E
F B F
Hence, it is clear from diagram that E is to the 24. (c) Person C is fourth to the left of G.
immediate left of A.
25. (a) Person A is to the immediate left of G.
Chapter 15 SITTING ARRANGEMENT 179

Solutions (Q. Nos. 26-30) According to the given Solutions (Q. Nos. 36-40) According to the given
information, the sitting arrangement as shown below information, the sitting arrangement is as shown
T below
Q
S R
N P

Q W K O

V L M
P
Z J
36. (d) This statement is not true that three people sit
26. (b) W sits to the immediate right of V.
between N and P.
27. (a) T is to the immediate right of R.
37. (d) Following the same pattern of J : L and K : N, i.e.
28. (e) In pair ZP, Z is sitting to the immediate right of P. Q:P
29. (c) Q is third to the right of R. 38. (b) Q’s position with respect to J is fourth to the left
30. (a) Q is sitting second to the left of Z. of J.

Solutions (Q. Nos. 31-35) According to the given 39. (a) N sits between Q and K when counted from left

A
information, the sitting arrangement is as shown of K.

M
below 40. (d) After interchanging the positions, arrangement is
G J

H
as follows
Q

TT
E H
O K
B A
P A N
W
C F
D
SH

31. (a) H’s position with respect to E is third to the left. L M


32. (e) C is third to the right of G. J
A

33. (d) A is fifth to the right of E.


@

Three persons sit between P and N, when


34. (b) F is second to the left of H. counted from the right of P.
35. (a) F is to the immediate right of D.

Expert Level Exercise


Solutions (Q. Nos. 1-5) According the given 1. (d) It is clear from the figure that, the three lady
information, the sitting arrangement as shown below members are H, E and G.
Note (+) = Male; (–) = Female
2. (a) It is clear that, J is a male member.
I+ G– 3. (d) K is sitting between E and H.

E–
4. (b) J is sitting to the immediate left of F.
F+
5. (c) There are three persons seated between K and F
K+ J+ on either side.

H– L+
180 How to Crack Test of Reasoning VERBAL

Solutions (Q. Nos. 6-10) According to the given Solutions. (Q. Nos. 19-21) According to the given
information, the sitting arrangement is as shown information, the sitting arrangement is as shown
below below
B
G D G C F E D H B A

E F 19. (b) Following the same pattern as E : D and B : A,


there is H : B.
C H
20. (d) Nobody is sitting between C and G.
A
6. (c) E is not to immediate right of A. 21. (d) Five people positions remain unchanged, if all
the people are made to sit in alphabetical order
7. (b) H will be on immediate right of A. from right to left.
8. (d) A and F will become neighbour, if H agrees to Previous
change her sitting position. G C F E D H B A order
9. (c) A will be directly facing B.
10. (b) H is sitting between A and F. Alphabetical
H G F E D C B A order

A
Solutions (Q. Nos. 11-15) According to the given
information, the sitting arrangement is as shown

M
Solutions (Q. Nos. 22-26) According to the given
below information, sequence of all people is as shown

H
below

TT
facing South
Floor 9 8 7 6 5 4 3 2 1
Row 1, R Q P S T
Row 2, C D B E A
People H
A
B E I D A G F C
W
facing North −1 −1 +2 −1 −1
22. (c) H → B, F → C, D → E, E → I, A → G
SH

floor floor floor floor floor


11. (c) There are two persons seated between Q and T. 23. (b) Two people are living between A and the person
A

12. (b) Except S, all others are sitting at the extreme living on the floor numbered 7.
@

ends. 24. (d) H lives on 9th floor.


13. (e) P, B represent the people sitting exactly in the 25. (c) Only one person lives between B and E is not
middle of the rows. true.
14. (e) D sits to the immediate left of B is true. 26. (d) Two people are living between I and H.
15. (d) Except R-A, all others are sitting diagonally Solutions (Q. Nos. 27-31) According to the given
opposite to each other. information, the arrangement is as shown below
Solutions (Q. Nos. 16-18) According to the given
information, the sitting arrangement is as follows F L H E O K M N G

27. (d) Except O and N, in all other pairs three people


E K A J B I G C H D F sit between them.
16. (b) It is clear from the figure that student I is sitting 28. (e) None of the given options is true.
in the middle of the row.
29. (c) F sits second to the left of H.
17. (c) CHDF are sitting to the right of G. 30. (d) M and K both sit exactly between N and O.
18. (d) All the information is required to know the 31. (a) Four people are sitting between G and E.
position of all students in the row.
Chapter 15 SITTING ARRANGEMENT 181

Solutions (Q. Nos. 32-35) According to the given Solutions (Q. Nos. 46-50) According to the given
information, the sitting arrangement is as shown information, arrangement is as shown below.
below
C H D A F B G E
X W T Z U S Y V

32. (b) D sits second to the right of F. 46. (d) Three people are facing North.

33. (e) A sits second to the left of B, is definitely true 47. (a) Except W and X, in all other options both
with respect to the given information. persons have face in opposite direction whereas
W and X have face in same direction.
34. (a) Four persons (H, D, B and E) are facing towards
North. 48. (b) Position of X with respect to Z is third to the right.

35. (d) Except AE, all others have one person between 49. (a) T sits exactly between Z and W.
them but in pair AE, there are three persons 50. (c) X is sitting second to the right of T.
(F, B and G) between them.
Solutions (Q. Nos. 51-55) According to the given
Solutions (Q. Nos. 36-40) According to the given information, the sitting arrangement as shown below
information, arrangement is as shown below X W Z Y
Row 2 S

A
I K H J L M G N

M
Row 1 N

H
36. (e) All the given statements are true. N M O P

TT
37. (c) L, H are the immediate neighbours of J. 51. (e) ‘O’ faces Z and immediate neighbours of ‘O’ are
M and P.
38. (c) K sits third to the left of L. A
52. (a) People sitting immediate right of ‘O’ is P and
W
39. (c) Except HI, in all other group every person have immediate right of Z is W.
SH

the same direction whereas H and I have the


53. (e) Immediate neighbours of W are X and Z and N
opposite direction.
and O faces X and Z.
A

40. (a) Only one person sit to the left of G. M sits at immediate right of N and P sits at
@

Solutions. (Q. Nos. 41-45) According to given immediate right of O.


information, arrangement is as shown below 54. (d) ‘O’ faces Z and second to the left of ‘O’ is ‘N’
55. (d) Y faces P.
Solutions (Q. Nos. 56-60) According to the given
U W T Q R P V S information, the sitting arrangement is as shown
below
41. (e) U is an immediate neighbour of Q is not true. S
U Y
42. (e) Q is second to the left of W.
43. (c) Except WR, all other pairs are facing opposite
directions. R W
44. (e) Two persons sit to the left of T.
45. (e) V, R are immediate neighbours of P. V X
T
182 How to Crack Test of Reasoning VERBAL

56. (b) Except R, all others are sitting at the corners of 63. (a) None of the given statements is true.
the table and facing the centre, but R is sitting in 64. (b) Except O, all other persons are sitting at the
the middle of the side and facing outside. middle of the sides of the table.
57. (a) S is sitting second to the right of R. 65. (c) C, P sit exactly between B and the one who sits
58. (c) W and X are sitting between T and Y when to the immediate right of N, i.e. M.
counted from the left of T. Solutions (Q. Nos. 66-70) According to the given
59. (d) None of the given option is true. information, the sitting arrangement is as shown
below
60. (a) Only one person, i.e. R is sitting between U and
S
V when counted from the left of V.
O Q
Solutions (Q. Nos. 61-65) According to the given
information, arrangement is as shown below
P U R
C B

P N
M D T
66. ( d) Except N, all others are facing the centre.
67. (c) S sits second to the left of U.

A
N O
A 68. (c) S faces the centre is true regarding S.

M
61. (e) C is to the immediate left of M. 69. (b) P is second to the right of N.

H
62. (b) O sits third to the left of P. 70. (b) Only two people in the given arrangement face

TT
outside.

A
W
SH
A
@
Chapter 16 PUZZLES

16
Puzzles
A puzzle is the raw information given in Illustration 1.
jumbled form in a particular order. The 1. Tom, Dick and Harry are intelligent.
information given need to be arranged 2. Tom, Brown and Jack are hard working.
systematically, so that all the information can

A
3. Brown, Harry and Jack are honest.
be depicted correctly. 4. Tom, Dick and Jack are ambitious.

M
WBPSC 2018
In this chapter, we deal with the questions put

H
Which of the following person is neither
in the form of puzzles involving certain

TT
hard working not ambitious?
number of items, (may be persons or any
(a) Tom (b) Dick (c) Harry (d) Jack
object). You are required to analyse the given
informations, condense it in a suitable form A
Solution (c) All the given information can be
W
summarised in the tabular form as given below.
and answer the questions asked.
SH

The questions on puzzle test can be based on Intelligent Hardworking Honest Ambitious
classification, placing arrangement, Tom ✔ ✔ ✗ ✔
A

comparison, sequential order, based on family Dick ✔ ✗ ✗ ✔


@

etc. Harry ✔ ✗ ✔ ✗
Brown ✗ ✔ ✔ ✗

Types of Puzzles Jack ✗ ✔ ✔ ✔

There are various categories of questions based Clearly, Harry is neither hard working nor
on puzzles which are as follows ambitious.

Type 1 Type 2
Based on Classification Based on Seating/Placing
Certain items belonging to different groups or Arrangement
possessing different qualities are given
In this type of puzzles, some clues regarding
alongwith some clues, with the help of which
seating/placing arrangement (linear or
you are required to group the items having circular) of some persons/objects along with
common properties and analyse the given additional information (like their profession,
items to answer the questions accordingly. colour of wearing items, etc) are given. The
It is advisable to solve classification based candidates are required to form the proper
puzzle by presenting the given information in sequence, arrange the persons using these
form of a table. clues and answer the questions accordingly.
184 How to Crack Test of Reasoning VERBAL

Directions (Illustrations 2-4) Read the X is from Chanakyapuri and is sitting third to the
following information carefully to answer the right of the person from Dwarka. The person from
given questions. Mehrauli is sitting second to the left of person from
Five friends A, B, C, D and E are sitting on a Chanakyapuri. The person from Rohini is sitting
bench and wearing different colours shirt, i.e. second to the left of W. A, who is from Lajpat
blue, red, yellow, white and green (not Nagar, is sitting exactly between X and Z. The
necessarily in this order). person from Saket is sitting second to the right of
I. A is sitting next to B and wearing red shirt. the person from Lajpat Nagar. C is sitting third to
the left of X. Y is not from Karol Bagh.
II. C is sitting next to D and not wearing IBPS (PO) Pre 2016
either blue or green shirt.
Illustration 5. Who amongst the following persons
III. D is not sitting with E and E is wearing
belongs to Okhla?
blue shirt.
(a) Y (b) D (c) C (d) B
IV. E is on the left end of the bench.
(e) Z
V. C is on the second position from the right. Illustration 6. What is A’s position with respect
VI. A is on the right of B and E. to B?
VII. B is wearing green shirt. (a) Third to the right (b) Second to the right

A
VIII. A and C sitting together. (c) Third to the left (d) Second to the left

M
IX. D is not wearing white shirt. (e) Fourth to the right

H
Illustration 7. How many people are sitting
Illustration 2. Where is A sitting?

TT
between Z and C when counted in an
(a) Between B and D (b) Between D and C anticlockwise direction from C?
(c) Between E and D (d) Between B and C (a) One
A
(b) Two (c) Three (d) Four
W
(e) None of these
Illustration 3. What is the colour of C’s shirt?
Solutions (Illustrations 5-7) According to the given
SH

(a) Blue (b) Red (c) White (d) Yellow


information, arrangement is as follows
Illustration 4. Who is sitting in the centre? Chanakyapuri
A

(a) A (b) B (c) C (d) D Lajpat Nagar Okhla


@

X
Solutions (Illustrations 2-4) According to the given A Y
information, arrangement is as follows Rohini Z B Mehrauli

E B A C D D C
Blue Green Red White Yellow Saket W Dwarka
2. (d) A is sitting between B and C. Karol Bagh
3. (c) The colour of C’s shirt is white. 5. (a) Y belongs to Okhla.
4. (a) A is sitting in the centre. 6. (a) Position of A with respect to B is third to the right.
7. (d) When counted in an anticlockwise direction from
Directions (Illustrations 5-7) Study the C, then four people are sitting between Z and C.
following information carefully to answer the
questions based on it.
A, B, C, D, W, X, Y and Z are sitting in a circle
Type 3
(But not necessary in the same order). Their Conditions, Grouping and
faces are in the centre. W is sitting third to the Team Formation
left of Y. The person, who is from Dwarka is to
the immediate right of W and W is not from In this type of puzzles, conditions regarding the
Okhla. B is sitting fourth to the right of Z. Z is selection or non-selection of persons or objects
not the neighbour of Y. Neither B nor Z is an with respect to one another are given. On the
immediate neighbour of W. basis of the given information, a team is to be
formed and questions are answered accordingly.
Chapter 16 PUZZLES 185

Illustration 8. Four political parties W, X, Y and Z Chartered Accountant (CA), Engineer, Teacher,
decided to set up a joint candidate for the Doctor, Architect and Pharmacist. No lady is
coming parliamentary elections. either Pharmacist or Chartered Accountant.
The formula agreed by them was the acceptance Each of them has a different monthly income.
of a candidate by most of the parties. Four The Chartered Accountant earns the most. S,
aspiring candidates A, B, C and D approached the Engineer earns less than V, the Doctor.
the parties for their tickets. R, the Teacher, earns more than P and less
than S. W’s wife earns the least. T is an
A was acceptable to W but not Z.
unmarried lady Lawyer and she earns less than
B was acceptable to Y but not X. P and more than only Q. The Pharmacist’s
C was acceptable to W and Y. income is not the lowest.
D was acceptable to W and X. Illustration 9. Who earns the least?
When candidate B was preferred by W and Z, (a) P (b) Q
candidate C was preferred by X and Z and (c) P or Q (d) R
candidate A was acceptable to X but not to Y,
who got the ticket? UPSC (CSAT) 2012 Illustration 10. What is P’s profession?
(a) A (b) B (a) Pharmacist (b) Lawyer

A
(c) C (d) D (c) Teacher (d) Cannot be determined

M
Solution (c) According to the given information, Illustration 11. How many members earn less

H
arrangement is as follows. than the Doctor?

TT
Four political parties (a) 2 (b) 3
Candidates
W X Y Z (c) 4 (d) 5
A 3 3 A
W
Illustration 12. Which of the following
B 3 3 3
represents the three female members of
SH

C 3 3 3 3 the family?
D 3 3
(a) PTQ (b) TRQ (c) VTQ
A

So, C is accepted by all the four parties. (d) Cannot be determined


@

Solutions (Illustrations 9-12) According to the given


Type 4 information, arrangement is as follows

Based on Comparison Persons Professions Sex


P Pharmacist M
This type of puzzles are related to comparison.
Q Architect F
You are required to analyse all the given
information, arrange the data in R Teacher M/F
ascending/descending sequence and then S Engineer M/F
answer the questions accordingly. T Lawyer F
It is advisable to use the notation such as V Doctor M/F
greater than (>), smaller than (<) and equal to
W Chartered Accountant M
(=) properly according to given condition/
information. Q ← W
Wife
Directions (Illustrations 9-12) Read the following In terms of income, Q < T < P < R < S < V < W
information carefully and answer the given 9. (b) Q earns the least. 10. (a) P is Pharmacist.
questions. 11. (d) V is Doctor and Q, T, P, R and S earn less
P, Q, R, S, T, V and W are the seven members of than V.
a family. There are three female members. Each 12. (d) The females are Q, T and one from R,
of them has a different profession—Lawyer, S and V.
186 How to Crack Test of Reasoning VERBAL

Type 5 13. (a) N has to attend a seminar on Sunday.


14. (c) Except option (c), in all other options, the given
Sequential Order to Things day is either the previous day or the next day of the
day on which the given person has to attend a
In this type of puzzles, some clues are given seminar.
regarding the order of occurrence of certain
events. The candidate is required to analyse the
given information, frame the right sequence Type 6
and then answer the questions accordingly.
Based on Family Problems
Directions (Illustrations 13-14) Study the In this type of puzzles, some clues are
following information carefully to answer the
provided regarding relationship amongst
given questions. IBPS (PO) Pre 2017
different members of a family, together with
Seven people J, K, L, M, N, O and P have to
their professions, qualities, colours, dresses
attend a seminar on seven different days of the
etc. You have to analyse the whole
same week starting from Monday and ending on
information and then answer the given
Sunday, but not necessarily in the same order.
questions accordingly.
K has to attend a seminar on Wednesday. Only
one person has to attend a seminar between K Illustration 15. P, Q, R, S, T and U are 6 members

A
and P. J attends a seminar immediately after P. of family in which there are two married

M
The number of people who have to attend a couples. T, a teacher, is married to a doctor

H
seminar before J is same as who have to attend a who is mother of R and U. Q, the lawyer, is

TT
seminar after L. Only one person has to attend a married to P. P has one son and one grandson.
seminar between L and M. O has to attend a Of the two married ladies one is a housewife.
seminar immediately after M. A
There is also one student and one male
W
Illustration 13. On which of the following days engineer in the family.
SH

does N have to attend a seminar? Which of the following is true about the
(a) Sunday (b) Thursday granddaughter of the family? [SSC (CGL) 2015]
A

(c) Saturday (d) Wednesday (a) She is a lawyer


(e) Tuesday
@

(b) She is an engineer


Illustration 14. As per the given arrangement, (c) She is a student
(d) She is a doctor
four of the following five are alike in a certain
way and so form a group. Which one of the Solution (c) According to the given information,
following does not belong to the group? arrangement is as follows
(–) Couple
(a) M-Friday (b) M-Wednesday (Lawyer) Q P(+)(Housewife)
(c) O-Sunday (d) O-Thursday
(e) J-Wednesday Son
Solutions (Illustrations 13-14) According to the given
information, arrangement is as follows
(Teacher) T(+) Couple (–)
(Doctor)
Person Day S
P Monday mother mother
J Tuesday
K Wednesday R
M Thursday (Student/Engineer) U
O Friday (Engineer/Student)
L Saturday So, the granddaughter of the family is a student
N Sunday and either R or U is a student.
Chapter 16 PUZZLES 187

Type 7 Illustration 16. What is the colour of Kapil’s


shirt?
Some Miscellaneous Puzzles (a) White (b) Green
In this type of puzzles, some mixed clues (c) Blue (d) Data inadequate
regarding three or more conditions of Illustration 17. Who likes reading?
persons/objects are given. It is required to (a) Rohan (b) Amar
analyse this mixed information with respect to (c) Kapil (d) Data inadequate
different conditions and classify the objects
accordingly to answer the questions asked. Solutions (Illustrations 16-17) According to the given
information, arrangement is as follows
Directions (Illustrations 16-17) Read the
Hobbies Colours of shirts
following information carefully and answer the

Reading
questions mentioned below.

Singing
Playing

Writing
Outing

Yellow

Green
White
Blue
Red
I. Five friends Amar, Kapil, Sarvesh, Rohan
and Nagesh put on five shirts of different
colours, i.e. Red, Yellow, Blue, White and Amar 7 7 7 7 3 7 3 7 7 7
Green, while they were going to attend a Kapil 7 7 7 3 7 7 7
party. These colours are not in order.

A
Sarvesh 7 7 3 7 7 3 7 7 7 7
II. They have different hobbies as reading,

M
playing, outing, singing and writing. Rohan 3 7 7 7 7 7 7 7

H
III. Kapil, who likes singing, does not wear Nagesh 7 3 7 7 7 7 7 7 l

TT
yellow shirt. Sarvesh wears red shirt and he 16. (d) The colour of Kapil’s shirt may be either blue or
does not like reading or writing. Nagesh white or green. Hence, data is inadequate to
likes playing and he does not wear blue or
yellow shirt. Amar likes writing and Rohan
answer the question. A
W
17. (a) Rohan likes reading.
does not wear yellow or green shirt.
SH
A

Let us Practice
@

Base Level Exercise


Directions (Q. Nos. 1-3) Read the following 2. Who is good in Physics, History and
information carefully to answer these questions. Dramatics?
MAT 2013
(a) Poonam (b) Shobha (c) Madhu (d) Anjali
Madhu and Shobha are good in Dramatics and
Computer Science. Anjali and Madhu are good in 3. Who is good in History, Physics, Computer
Computer Science and Physics. Anjali, Poonam Science and Mathematics?
and Nisha are good in Physics and History. Nisha (a) Poonam (b) Nisha (c) Anjali (d) Madhu
and Anjali are good in Physics and Mathematics. 4. Six students A, B, C, D, E and F are sitting
Poonam and Shobha are good in History and on the ground. A and B belong to Ruby
Dramatics. House, while the rest belong to Emerald
1. Who is good in Physics, History and House. D and F are tall, while others are
Mathematics but not in Computer Science? short. C and D are wearing glasses, while
(a) Madhu others are not wearing. Which girl of
(b) Poonam Emerald House is tall and wearing glasses?
(c) Nisha SSC (CGL) 2015
(d) Anjali (a) B (b) C (c) D (d) A
188 How to Crack Test of Reasoning VERBAL

5. There are six books A, B, C, D, E and F. B, C Directions (Q. Nos. 11-13) Read the following
and E have blue covers while the rest have passage and answer the three items that follow.
red covers. D and F are new books while UPSC (CSAT) 2013
the rest are old books. A, C and D are law A tennis coach is trying to put together a team of
reports while the rest are of physics. Which four players for the forthcoming tournament. For
is the red covered new law report book? this, 7 players are available: males A, B and C and
WBPSC 2018 females W,X, Y and Z. All players have equal
(a) A (b) B (c) C (d) D capability and atleast 2 males will be there in the
Directions (Q. Nos. 6-10) Study the given team. For a team of four, all players must be able to
information and answer the given questions. play with each other. But, B cannot play with W,C
IBPS (Clerk) 2013 cannnot play with Z and W cannot play
A, B, C, D, E, F and G are standing in a straight with Y.
line facing North with equal distances between
11. If Y is selected and B is rejected, then the
them, not necessarily in the same order.
team will consist of which one of the
Each one is pursuing a different profession— following groups?
actor, reporter, doctor, engineer, lawyer, teacher (a) A, C, W and Y (b) A, C, X and Y
and painter not necessarily in the same order. (c) A, C, Y and Z (d) A, W, Y and Z

A
G is fifth to the left of C. The reporter is third to
the right of G. F is fifth to the right of A. E is 12. If B is selected and Y is rejected, then the

M
second to the left of B. The engineer is second to team will consist of which one of the

H
the left of D. There are only three people following groups?

TT
(a) A, B, C and W (b) A, B, C and Y
between the engineer and the painter. The doctor
(c) A, B, C and X (d) A, W, Y and Z
is to the immediate left of the engineer. The
lawyer is to the immediate right of the teacher. A
13. If all the three males are selected, then how
W
6. What is A’s profession? many combination of four member teams are
possible?
SH

(a) Painter (b) Doctor (c) Teacher (d) Actor


(e) Engineer (a) 1 (b) 2 (c) 3 (d) 4
Directions (Q. Nos. 14-16) Answer the following
A

7. Which of the following statements is true


according to the given arrangement? questions based on the statements given below.
@

CLAT 2013
(a) F is the teacher
I. There are 3 poles on each side of the road.
(b) F is third to the left of E
II. These six poles are labelled A, B, C, D, E
(c) The painter is to the immediate left of B
and F.
(d) The lawyer is standing in the exact middle of
the arrangement III. The poles are of different colours namely
Golden, Silver, Metallic, Black, Bronze and
(e) None of the above
White.
8. Who among the following is an actor? IV. The poles are of different heights.
(a) E (b) F (c) C (d) B V. E, the tallest pole, is exactly opposite to the
(e) A Golden coloured pole.
9. What is D’s position with respect to the VI. The shortest pole is exactly opposite to the
painter? Metallic coloured pole.
(a) Third to the left (b) Second to the right VII. F, the Black coloured pole, is located between
(c) Fourth to the right (d) Third to the right A and D.
(e) Second to the left VIII. C, the Bronze coloured pole, is exactly
10. Four of the following five are alike in a opposite to A.
certain way based on the given standing IX. B, the Metallic coloured pole, is exactly
arrangement and so form a group. Which of opposite to F.
the following does not belong to the group? X. A, the White coloured pole, is taller than C
(a) AED (b) DFC (c) GDB (d) EBF but shorter than D and B.
(e) BFC
Chapter 16 PUZZLES 189

14. What is the colour of the pole diagonally Directions (Q. Nos. 21-23) Study the
opposite to the Bronze coloured pole? information below and answer the given
(a) White (b) Silver questions. MPPSC 2018
(c) Metallic (d) Golden Seven members, viz. A,B,C,D, E, F and G, are
family members. Three of them are female
15. Which is the second tallest pole?
members. There are two couples in the family.
(a) A (b) D
Each of them has a different profession, viz.
(c) B (d) Cannot be determined
Architect, Teacher, Lawyer, Doctor, Engineer,
16. Which is the colour of the tallest pole? Manager and Music Director, but not necessarily
(a) Golden (b) Silver in the same order. B is a Lawyer and his wife is
(c) Bronze (d) None of these F, who is a Manager. A is brother of G, who is an
Architect. C, who is a Doctor is a female and is
Directions (Q. Nos. 17-20) Study the following
unmarried. D is a Teacher and sister of G. E is
information to answer the given question.
NIACL (AO) 2014 not an Engineer and is father of C.
Six lecturers are scheduled in a week starting 21. Which of the following combinations
from Monday and ending on Sunday of the same according to the given information is true?
week. Computer Science is not on Tuesday or (a) C- Female-Manager (b) B- Female-Lawyer

A
Saturday. Psychology is immediately after (c) F-Male-Manager (d) G-Male-Architect

M
Organisational Behaviour. Statistics is not on
22. Which of the following is true about A?
Friday and there is one day gap between

H
(a) A is husband of C
Statistics and Research methods. One day prior to

TT
(b) A is a Music Director
the schedule of Economics there is no lecture (as
(c) A is brother of G and D
that day is the ‘off’ day and Monday is not the ‘off’
day) A
(d) A is sister of E
W
23. If B is father of A, then what is the relation
17. Which of the following is the last lecture
between F and G?
SH

scheduled?
(a) G is mother of F
(a) Statistic (b) Research methods
(b) F is mother of G
A

(c) Psychology (d) Cannot be determined


(c) F is sister of G
(e) None of these
@

(d) Cannot be determined


18. How many lectures were scheduled between Directions (Q. Nos. 24-26) Read the following
Economics and Psychology? information carefully to answer these questions.
(a) One (b) Two
(c) Three (d) Cannot be determined Ramesh, Kailash, Avinash and Jagan are good
(e) None of these friends studying in a school. Three of them stay
away from the school and one stays nearer to it.
19. On which day is Computer Science Two study in class IV, one in class V and one in
scheduled? class VI. They study Hindi, Maths, Social Science
(a) Monday (b) Wednesday and Science.
(c) Thursday (d) Cannot be determined One of these four friends is good in all the four
(e) None of these subjects while another is weak in all the four
20. If Wednesday is the ‘off’ day, the code would subjects. Ramesh stays away from the school and
be 2-4. If Thursday was the ‘off’ day, the is good in Maths only while Kailash is weak in
code would be 3-3. Taking account the ‘off’ Maths only and stays close to the school. Neither
day which of the following code is correct? of these two study in class V or VI. One who is
(a) 2-4 good in all the subjects is studying in class V.
(b) 3-3
Avinash is not studying in class VI.
(c) 4-2 24. Name the boy who is good in all the
(d) Cannot be determined subjects.
(e) None of the above (a) Kailash (b) Jagan (c) Avinash (d) Ramesh
190 How to Crack Test of Reasoning VERBAL

25. Name the boy who is weak in all the subjects. 31. Which is V’s favourite city?
(a) Ramesh (b) Kailash (c) Jagan (d) Avinash (a) Hyderabad (b) Pune
(c) Mumbai (d) Data inadequate
26. Which two boys are good in Mathematics? (e) None of these
(a) Jagan and Kailash
(b) Kailash and Avinash Directions (Q. Nos. 32-36) Study the given
(c) Ramesh and Avinash information carefully to answer the given
(d) Ramesh and Jagan questions. Indian Bank (PO) Pre 2016
Seven people P, Q, R, S, T, U and V live on seven
Directions (Q. Nos. 27-31) Study the following different floors of a building, but not necessarily
information carefully and answer the given in the same order. The lower most floor of the
questions. IBPS (Clerk) 2011 building is numbered one, the one above that is
P, Q, R, S, T, V and W are seven friends. Each of numbered two and so on till the top most floor is
them likes a particular fruit, viz. Apple, Banana, numbered seven. Each one of them likes a
Pear, Guava, Orange, Mango and Watermelon and different watch-Casio, Citizen, Fossil, Seiko,
each of them has a favourite city, viz. Mumbai, Tissot, Omega and Fastrack, but not necessarily
Pune, Delhi, Kolkata, Chennai, Hyderabad and in the same order.
Cochin. U lives on floor number 4. Only two people live

A
The choices of fruit and favourite city of the seven between U and the one who likes Citizen. Only

M
friends are not necessarily in the same order. Q three people live between the one who likes
likes Mango and his favourite city is Chennai. The Citizen and the one who likes Fastrack. S lives

H
one whose favourite city is Pune likes on one of the even numbered floors above the one

TT
Watermelon. T’s favourite city is Kolkata. who likes Fastrack. Only one person lives
R likes Guava and his favourite city is not between S and the one who likes Seiko . V lives
Mumbai. W’s favourite city is Cochin and he does A
one of the odd numbered floors below the one
W
not like either Banana or Pear. The favourite who likes Seiko. Only one person lives between V
SH

city of the one who likes Orange is Hyderabad. and the one who likes Fossil. P lives either
T does not like Pear. P’s favourite city is immediately above or immediately below the one
neither Pune nor Hyderabad. S does not like who likes Fossil. T lives immediately above P.
A

Watermelon. Only one person lives between Q and the one


@

who likes Casio. The one who liks Tissot lives on


27. Who likes Apple?
one of the floors above the one who likes Omega.
(a) W (b) T (c) V (d) P
(e) Data inadequate 32. Who lives immediately below S?
(a) T
28. Which fruit does P like? (b) Q
(a) Apple (b) Orange (c) Pear (d) Watermelon (c) U
(e) None of these (d) The one who likes Omega
29. Which is R’s favourite city? (e) The one who likes Tissot
(a) Mumbai (b) Pune 33. How many people live above the one who
(c) Hyderabad (d) Delhi likes Seiko?
(e) None of these (a) More than three (b) Two
30. Which of the following combinations of (c) None (d) Three
Person–Fruit-City is incorrect? (e) One
(a) R–Guava–Kolkata 34. Which of the following watches does T like?
(b) V–Watermelon-Hyderabad (a) Omega (b) Fastrack
(c) T–Banana–Cochin (c) Fossil (d) Tissot
(d) S–Guava–Delhi (c) Casio
(e) All are incorrect
Chapter 16 PUZZLES 191

35. On which of the following floor numbers does 36. How many people live between the one
Q live? who likes Fastrack and the one who likes
(a) 7 (b) 5 Casio?
(c) 3 (d) 2 (a) None (b) One (c) More than three
(e) 6 (d) Two (e) Three

Expert Level Exercise


1. Seven men A, B, C, D, E, F and G are standing (b) R-Foreign, S-Industry and Commerce,
in a queue in that order. Each one is wearing a P-Agriculture, Q-Rural Development,
cap of a different colour like violet, indigo, T-Human Resources
blue, green, yellow, orange and red. D is able (c) P-Foreign, Q-Industry and Commerce,
T-Agriculture, S-Rural Development,
to see in front of him green and blue but not
U-Human Resources
violet. E can see violet and yellow but not red.
(d) Q-Foreign, U-Industry and Commerce,
G can see caps of all colours other than
T-Agriculture, R-Rural Development,
orange. If E is wearing an indigo coloured P-Human Resources
cap, then the colour of the cap worn by F is
UPSC (CSAT) 2013 4. If P gets Foreign and R gets Human

A
(a) Blue (b) Violet (c) Red (d) Orange Resources, then which is not a valid

M
assignment of Agriculture and Rural
2. In five flats, one above the other, live five Development?

H
professionals. The professor has to go up to
(a) S-Agriculture, Q-Rural Development

TT
meet his IAS officer friend. The doctor is
(b) U-Agriculture, Q-Rural Development
equally friendly to all and has to go up as
(c) Q-Agriculture, T-Rural Development
frequently as go down. The engineer has to go
up to meet his MLA friend above whose flat A
(d) Q-Agriculture, S-Rural Development
W
lives the professor’s friend. From the ground Directions (Q. Nos. 5-7) Read the following
SH

floor to the top floor, in what order do the five passage and answer the given questions.
professionals live? UPSC (CSAT) 2012 CGPSC (Pre) 2012
A

(a) Engineer, professor, doctor, IAS officer, MLA Seven friends Anu, Beena, Kanika, Mohini,
@

(b) Professor, engineer, doctor, IAS officer, MLA Samina, Easther and Rani study different
(c) IAS officer, engineer, doctor, professor, MLA disciplines such as Arts, Commerce, Science,
(d) Professor, engineer, doctor, MLA, IAS officer Engineering, Architecture, Management and
Directions (Q. Nos. 3-4) Read the following Pharmacy, not necessarily in the same order.
passage and answer the given questions. Each of them belongs to a different state viz.
Andhra Pradesh, Uttar Pradesh, Maharashtra,
The head of a newly formed Government desires to
Karnataka, Kerala, Chhattisgarh and Punjab
appoint five of the six elected ministers P, Q, R, S,
but not necessarily in the same order.
T and U to portfolios of Foreign, Industry and
Commerce, Agriculture, Rural Development and Kanika studies Engineering and does not belong
Human Resources. U does not wants any portfolio to either Uttar Pradesh or Punjab. The one who
if S gets one of the five. R wants either Foreign or belongs to Chhattisgarh does not study
Human Resources or no portfolio. Q says that if S Architecture or Pharmacy. Anu belongs to
gets industry and Commerce or Rural Maharashtra. Samina belongs to Kerala and
Development, then she must get the other one. T studies Science. The one who belongs to Andhra
insists on a portfolio if P gets one. Pradesh studies Commerce. Mohini studies
Management and Rani studies Arts. Beena
3. Which of the following is a valid assignment? belongs to Karnataka and does not study
(a) P-Foreign, Q-Industry and Commerce, Architecture. The one who studies Arts does not
R-Agriculture, S-Rural Development, T-Human
belong to Punjab.
Resources
192 How to Crack Test of Reasoning VERBAL

5. Which of the following student and subject 9. Who amongst the following likes Kolkata?
combination is correct? (a) R (b) P (c) T (d) S
(a) Anu-Architecture (b) Easther- Pharmacy (e) Q
(c) Beena-Commerce (d) Mohini-Engineering
10. Which of the following statements is true as
(e) None of the above
per the given arrangement?
6. Which of the following state subject (a) None of the given statements is true
combination is/are correct? (b) R likes Agra city
I. Chhattisgarh-Engineering (c) U works in the same department in which S
II. Punjab-Pharmacy works
(d) V works with only the one who likes
III. Andhra Pradesh-Commerce
Chandigarh
IV. Andhra Pradesh-Architecture (e) Both Q and T work in production department
(a) I and II (b) II and III
(c) I and IV (d) I and III 11. Which of the following combinations
(e) None of these represent the combination of people working
in IT department?
7. Which of the following student state (a) S, P and the one who likes Kolkata
combination is/are correct? (b) P and V
I. Kanika-Chhattisgarh

A
(c) R, the one who likes Mumbai
II. Samina-Andhra Pradesh (d) The one who likes Agra and T

M
III. Easther-Karnataka (e) T, the one who likes Patna and Agra

H
IV. Mohini-Punjab 12. Which of the following combinations

TT
(a) I and II (b) I and III (c) I and IV (d) II and III represents the department in which V works
(e) None of these and the city he likes?
Directions (Q. Nos. 8-12) Study the following (a) IT-Delhi A (b) IT-Kolkata
W
information and answer the given questions. (c) Production-Kolkata (d) Production-Patna
SH

Indian Bank (PO) Pre 2016 (e) HR-Patna


Seven people, namely P, Q, R, S, T, U and V like Directions (Q. Nos. 13-17) Study the following
A

seven different cities namely, Agra, Mumbai, information carefully and answer the questions
@

Delhi, Jaipur, Patna, Chandigarh and Kolkata. given below. SBI (PO) 2013
Each of them works in either of the three Eight people—E, F, G, H, J, K, L and M are
departments of a company viz. Production, HR sitting around a circular table facing the centre.
and IT with atleast two of them in a department. Each of them is of a different profession —
R works in HR department with the one who likes Chartered Accountant, Columnist, Doctor,
Mumbai. S works with the one who likes Patna. S Engineer, Financial Analyst, Lawyer, Professor
does not work with R. P works with only the one and Scientist, but not necessarily in the same
who likes Delhi. P neither likes Patna nor works order. F is sitting second to the left of K.
in production department. The one who likes Agra The Scientist is an immediate neighbour of K.
works with the one who likes Jaipur. Neither R There are only three people between the Scientist
nor Q likes Jaipur. Q works with only T. U likes and E. Only one person sits between the
Chandigarh. U does not work in IT department. Engineer and E. The Columnist is to the
Note None of the information given is necessarily in the immediate right of the Engineer. M is second to
same order. the right of K. H is the Scientist.
8. Four of the following five are alike in a G and J are immediate neighbours of each other.
certain way as per the given arrangement Neither G nor J is an Engineer. The Financial
and hence form a group. Which of the Analyst is to the immediate left of F. The
following does not belong to the group? Lawyer is second to the right of the Columnist.
(a) V-Kolkata (b) P-Patna The Professor is an immediate neighbour of the
(c) Q-Jaipur (d) S-Chandigarh Engineer. G is second to the right of the
(e) T-Agra Chartered Accountant.
Chapter 16 PUZZLES 193

13. Who is sitting second to the right of E? Only one person lives between O and the one who
(a) The Lawyer (b) G runs for 2200 m. The one who runs for 4000 m
(c) The Engineer (d) F lives immediately above O. Only one person lives
(e) K between the one who runs for 4000 m and the one
who runs for 1300m. The number of people living
14. Who amongst the following is the Professor?
between O and the one who runs for 1300 m is
(a) F (b) L (c) M (d) K
(e) J
same as the between the one who runs for 4000 m
and R. N lives on an odd numbered floor. N ran
15. Four of the following five are alike in a for 2000 m more than the one who lives on floor
certain way based on the given arrangement number 4. Only two people live between Q and
and hence form a group. Which of the the one who runs for 3300 m.
following does not belong to that group? 18. Who amongst the following live (s) between
(a) Chartered Accountant-H
P and the one who runs for 1300 m?
(b) M-Doctor
(a) Both Q and R
(c) J-Engineer
(b) Only S
(d) Financial Analyst-L
(c) Both R and the one who runs for 850 m
(e) Lawyer-K
(d) Only the one who runs for 4000 m

A
16. What is the position of L with respect to the (e) Both R and the one who runs for 2200 m

M
Scientist? 19. How many people live between S and N?
(a) Third to the left (b) Second to the right

H
(a) Three (b) One
(c) Second to the left (d) Third to the right (c) Five (d) Four

TT
(e) Immediate right (e) Two
17. Which of the following statements is true
according to the given arrangement? A
20. As per the given arrangement, four of the
W
following five are alike in a certain way and
(a) The Lawyer is second to the left of the Doctor so form a group. Which one of the following
SH

(b) E is an immediate neighbour of the Financial does not belong to the group?
Analyst (a) Q-3300 m
A

(c) H sits exactly between F and the Financial (b) Q-2300 m


Analyst
@

(c) Floor number 4-S


(d) Only four people sit between the Columnist (d) Floor number 2-R
and F (e) Floor number 7-1300 m
(e) All of the given statements are true
21. Which of the following statements is true
Directions (Q. Nos. 18-23) Study the given with respect to the given arrangement?
information carefully and answer the given
(a) Only two people live between P and O
questions. IBPS (PO) Pre 2017
(b) Q runs for 4000 m
Seven athletes M, N, O, P, Q, R and S live on (c) N lives on floor number 7
seven different floors of a building but not (d) The one who runs for 850 m lives immediately
necessarily in the same order. The lowermost above P
floor of the building is numbered 1, the one above (e) None of the above
that is numbered 2 and so on till the topmost floor
22. If the total distance covered by B and M is
is numbered 7. Each one of them runs for a
5300 m, then how much did B run alone?
different distance in marathon-850 m, 1300m,
(a) 2000 m (b) 4000 m
2200 m, 2800 m, 3300 m, 4000 m, and 4700 m,
(c) 3100 m (d) 1300 m
but not necessarily in the same order. The one (e) 600 m
who runs for 2200 m lives on floor numbered 3.
The one who runs for 2800 m lives on one of the 23. Who amongst the following runs for 2200 m?
(a) P (b) N
floors below Q but not on the floor number 2, only
(c) Q (d) R
two people live between M and S. The one who
(e) S
runs for 850 m lives immediately below M.
194 How to Crack Test of Reasoning VERBAL

Directions (Q. Nos. 24-28) Study the given 27. Which of the following combinations will be
information carefully to answer the given definitely true as per the given arrangement?
questions. NIACL AO (Main) 2016 (a) C-Squash (b) Thursday-D
Seven people A, B, C, D, E, F and G were (c) Saturday-Cricket (d) Monday-Volleyball
appointed to a company on seven different days (e) F-Kho-kho
of the same week starting from Monday to
28. How many people were appointed before G?
Sunday (but not necessarily in the same order).
(a) Two (b) One (c) Three (d) Four
Each person also plays a different game (e) None, as G was appointed on Monday
namely-Cricket, Hockey, Football, Squash,
Volleyball, Tennis and Kho-kho, but not Directions (Q. Nos. 29-35) Study the following
necessarily in the same order information carefully and answer the given
Only two people were appointed after the one question. IBPS (PO) 2014
who plays Hockey. E was appointed on one of the A, B, C, D, E, F, G and H are sitting around a
days after the one who plays Hockey. Only three square table in such a way that four of them sit at
people were appointed between E and G. Only four corners of the square while four sit in the
one person was appointed between G and the middle of each of the four sides.
one who plays Volleyball. A was appointed The one who sit at the four corners face the centre

A
immediately after the one who plays Volleyball. of the table while those sit in the middle of the
More than three people were appointed after sides face outside. Each one of them likes a

M
the one who plays Kho-kho. C was appointed different subject viz. Mathematics, Hindi, English,

H
immediately after F, but not on Friday. Only two Biology, Chemistry, Physics, History and

TT
people were appointed between F and the one Geography. (None of the information given is
who plays Cricket. B was appointed immediately necessary in the same order). C sits third to left of
after the one who plays Cricket. More than three A
the one who likes Geography.
W
people were appointed between D and the one The one who likes Geography faces outside. Only
who plays Tennis. G does not play Football. F two people sit between C and H. The one who likes
SH

Plays Squash. Mathematics sits to immediate right of H. The one


24. Who amongst the following was appointed who likes Chemistry sits second to the right of G.
A

on Wednesday? G is neither an immediate neighbour of H nor C.


@

(a) The one who plays Kho-kho G does not like Geography. Only one person sit
(b) A between A and the one who likes Chemistry.
(c) B D sits to the immediate left of the one who likes
(d) The one who plays Cricket Physics. G does not likes Physics. E likes History. E
(e) F is not an immediate neighbour of A. The one who
likes Hindi is an immediate neighbour of E. The one
25. Which of the following statements is true as who likes Biology is an immediate neighbour of F.
per the given arrangement?
(a) Only one person was appointed between F 29. Who amongst the following sits diagonally
and the one who plays Squash opposite the one who likes Mathematics?
(b) Only three people were appointed before C (a) The one who likes Hindi
(c) A plays Tennis (b) D
(d) B was appointed on Saturday (c) A
(e) None of the above (d) The one who likes English
(e) The one who likes Biology
26. The person who plays Tennis was
appointed on which of the following days? 30. Who amongst the following represents the
(a) Thursday immediate neighbour of the one who likes
(b) Tuesday Chemistry?
(c) Sunday (a) B, F (b) C, E
(d) Monday (c) B, E (d) D, F
(e) Saturday (e) F, H
Chapter 16 PUZZLES 195

31. Who amongst the following sits exactly 35. Who amongst the following likes Geography?
between H and B? (a) B (b) F (c) H (d) A
(a) C (e) D
(b) The one who likes Hindi
(c) The one who likes English
36. Eight people A, B, C, D, E, F, G and H work in
three different companies X, Y and Z. Out of
(d) G
these, two are females who work in different
(e) A
companies and have different specialisations.
32. Which of the following is true regarding B? Two of them are specialists in Finance, two in
(a) B is one of the immediate neighbours of D HR, two in marketing and one each is an
(b) The one who likes Geography is an engineer and a computer specialist.
immediate neighbour of B
D, working in a company X has specialised in
(c) B sits second to left of H
(d) B likes History
HR and her friend G, a finance specialist is
(e) B is an immediate neighbour of the one who
working in company Z. H, an HR specialist, is
likes Mathematics working with B, a marketing specialist and
does not work in company Y. C is not a
33. What is the position of the one who likes computer specialist.
Physics with respect to G?

A
(a) Second to the left No two people with the same specialisation,
work together. F is a specialist in marketing

M
(b) Third to the right
(c) Fourth to the left working in company Y and his friend, A has

H
(d) Second to the right specialist in finance and is working in

TT
(e) Third to the left company X with only another specialist. Not
more than three of them work in any company.
34. Which of the following subjects does D
likes? A
No female is an engineer or computer
W
specialist. In which two companies do the HR
(a) Biology (b) Mathematics
specialist work?
SH

(c) Hindi (d) Chemistry


(a) X and Y (b) Y and Z
(e) English
(c) X and Z (d) Data inadequate
A
@

Answers
Base Level Exercise
1 (c) 2 (a) 3 (c) 4 (c) 5 (d) 6 (b) 7 (d) 8 (c) 9 (e) 10 (e)
11 (b) 12 (c) 13 (b) 14 (d) 15 (d) 16 (b) 17 (e ) 18 (c) 19 (c) 20 (e)
21 (d) 22 (c) 23 (b) 24 (c) 25 (c) 26 (c) 27 (a) 28 (c) 29 (d) 30 (e)
31 (b) 32 (b) 33 (d) 34 (c) 35 (b) 36 (b)

Expert Level Exercise


1 (c) 2 (d) 3 (b) 4 (a) 5 (a) 6 (d) 7 (c) 8 (b) 9 (d) 10 (c)
11 (d) 12 (d) 13 (b) 14 (d) 15 (c) 16 (b) 17 (a ) 18 (b) 19 (a) 20 (c)
21 (c) 22 (e) 23 (e) 24 (e) 25 (b) 26 (c) 27 (c) 28 (b) 29 (a) 30 (c)
31 (e) 32 (e) 33 (e) 34 (a) 35 (b) 36 (c)
How to Crack Test of Reasoning VERBAL

Answer with Explanations


Base Level Exercise
Solutions (Q. Nos. 1-3) According to the given Solutions (Q. Nos. 6-10) According to the given
information, arrangement is as follows information, standing arrangement and profession
of the persons are as follows
Computer Mathe-
People Dramatics Physics History
Science matics Facing North
Madhu 3 3 3
Shobha 3 3 3 A G E D B F C
Anjali 3 3 3 3 Doctor (Eng.) (Teacher) (Lawyer)(Reporter) (Painter) (Actor)
Poonam 3 3 3
Nisha 3 3 3 6. (b) A is a Doctor.
7. (d) The lawyer is standing in the exact middle of
1. (c) Nisha is good in Physics, History and
the arrangement.
Mathematics.
8. (c) C is an Actor.
2. (a) Poonam is good in Physics, History and

A
Dramatics. 9. (e) D is second to the left of painter.

M
3. (c) Anjali is good in given four subjects. 10. (e) BFC does not belong to the group.

H
4. (c) We have total six students sitting on the ground 11. (b) From the question, it is clear that B is rejected.

TT
: Ruby House → A, B It means from males, both A and C must be the
Emerald House → C, D, E, F part of team as it is given that atleast two males
Tall → D, F A
must be there in the team. Now, from the given
table, it is clear that, if C is selected Z must be
W
Short → A, B, C, E rejected. So, Z must not be the part of team. It is
Wearing glasses → C, D
SH

also given in question that Y is selected.


Not Wearing glasses → A, B, E, F. From the above table, it is clear that if Y is
A

We have to find out the girl of Emerald House who selected, then W must be rejected.
@

is tall and wearing glasses. So, we have to find So, from females, X and Y will be the part of the
out the common students in the given condition. team. As a result, the team will consist of A, C, X
Therefore, D is the only student who is wearing and Y.
glasses, tall and belongs to Emerald House.
12. (c) Here, it is given that B is selected. It means that
5. (d) According to the given information, arrangement W is rejected and Y is also rejected as given.
is as follows Now, there are two possibilities of formation of
Blue Red New Old Law Physics team, i.e. ABXZ and ABCX. But from the
cover cover Report options given, ABCX will be the required team.
A ✗ ✔ ✗ ✔ ✔ ✗
13. (b) If all three males are selected then three of the
B ✔ ✗ ✗ ✔ ✗ ✔ team members are A, B and C. Now, W cannot
C ✔ ✗ ✗ ✔ ✔ ✗ be a part of the team because she cannot play
D ✗ ✔ ✔ ✗ ✔ ✗ with B and Z cannot be selected because she
E ✔ ✗ ✗ ✔ ✗ ✔ cannot play with C. Hence, there can be only
F ✗ ✔ ✔ ✗ ✗ ✔ two combinations of four member teams, i.e.,
ABCX and ABCY.
Hence, D is the red covered new law report.
Chapter 16 PUZZLES 197

Solutions (Q. Nos. 14-16) According to the given 21. (d) G-Male-Architect is a true combination.
information, arrangement is as follows 22. (c) A is brother of G and D.
E Silver B Metallic C Bronze 23. (b) F is mother of G.
Solutions (Q. Nos. 24-26) According to the given
Tallest
information, arrangement is as follows
Shortest
Friends Stay Stay IV V VI Subjects
away near Hindi Maths Social Science
D Golden F Black A White
Science
Also, E > B, D > A > C > F Ramesh ✔ ✔ ✔
14. (d) The colour of the pole diagonally opposite to the Kailash ✔ ✔ ✔ ✔ ✔
Bronze coloured pole is Golden. Avinash ✔ ✔ ✔ ✔ ✔ ✔
15. (d) As, comparison between B and D is not given, Jagan ✔ ✔
so it cannot be determined which pole is second
tallest. 24. (c) From the above table, Avinash is good in all the
subjects.
16. (b) Silver coloured pole is the tallest pole. 25. (c) Jagan is weak in all the subjects.
Solutions (Q. Nos. 17-20) According to the given 26. (c) Ramesh and Avinash are good in Mathematics.

A
information, arrangement is as follows Solutions (Q. Nos. 27-31) According to the given

M
Days Subjects information, arrangement is as follows
Monday Organisational Behaviour

H
Persons Favourite Fruits Favourite cities
Tuesday Psychology

TT
P Pear Mumbai
Wednesday Statistics
Thursday Computer Science Q Mango Chennai
Friday Research Methods R A
Guava Delhi
W
Saturday Off day S Orange Hyderabad
SH

Sunday Economics T Banana Kolkata


V Watermelon Pune
17. (e) Last lecture scheduled is of Economics.
A

W Apple Cochin
18. (c) Three lecture are scheduled between
@

Economics and Psychology. 27. (a) W likes Apple.


19. (c) Computer Science is scheduled on Thursday. 28. (c) P likes Pear.
20. (e) If wednesday is the off day the code is 2-4 29. (d) R’s favourite city is Delhi.
2 → Number of lectures before off day.
30. (e) All combinations are incorrect.
4 → Number of lectures after off day. 31. (b) V’s favourite city is Pune.
If Thursday is the off day the code is 3-3
Solutions (Q. Nos 32-36) According to the given
3 → Number of lectures before off day
information, the arrangement is as follows
3 → Number of lectures after off day.
Following the same pattern, if Saturday is the off Floor Person Watch
day the code will be 5-1. 7 R Casio
Solutions (Q.Nos. 21-23) As per the information, the 6 S Tissot
arrangement is as follows. 5 Q Fastrack
Married 4 U Seiko
+ Couple – 3 T Fossil
B (Lawyer) F (Manager)
er
Moth Married
2 P Omega
Couple 1 V Citizen
+ Brother + Sister Music
A G D– E+ Director
Engineer Architect Teacher 32. (b) Q lives immediately below S.
Father 33. (d) Three people Q, S and R live above the one who
(Doctor) C
– likes Seiko.
198 How to Crack Test of Reasoning VERBAL

34. (c) T likes Fossil. Solutions (Q. Nos. 8-12) According to the given
35. (b) Q lives on floor number 5 information, arrangement is as follows
36. (b) Only one person-S, lives between the one who likes Person City Department
Fastrack and the one who likes Casio. P Mumbai HR
Q Agra IT
Expert Level Exercise R Delhi HR
1. (c) Arrangement in queue is as follows S Kolkata Production
T Jaipur IT
A → Yellow or Green or Blue
U Chandigarh Production
B → Yellow or Green or Blue
V Patna Production
C → Yellow or Green or Blue
D → Violet 8. (b) Except P-Patna, in all others, the department
E → Indigo of the given person and the department of
F → Red the person whose favourite city is given are
G → Orange same.
Hence, from the above arrangement, F worn red 9. (d) S likes Kolkata.
colour cap. 10. (c) U and S both work in production department
2. (d) The order is as follows is true

A
4th → IAS officer 11. (d) The one who likes Agra, i.e. Q and T work in

M
3rd → MLA IT department.
2nd → Doctor 12. (d) V works in production department and he

H
1st → Engineer/Professor likes Patna city.

TT
Ground → Professor/Engineer Solutions (Q. Nos. 13-17) According to the given
information, sitting arrangement is as follows
But from the given options, only option (d) follows,
i.e. from ground floor to top floor, order is professor, A
E (Chartered Accountant)
W
engineer, doctor, MLA, IAS officer. J (Lawyer) M (Columnist)
SH

3. (b) Here, option (b) is valid assignment.


G (Financial
L (Engineer)
4. (a) S is not insisted to get agriculture. Analyst)
A

Solutions (Q.Nos. 5-7) According to the given information, F (Doctor) K (Professor)


@

arrangement is following H (Scientist)


Disciplines States
13. (b) G is sitting second to the right of E.
Management

Chhattisgarh
Maharashtra
Architecture
Engineering
Commerce

Karnataka
Pharmacy
Science

Punjab
Kerala

14. (d) K is the professor.


Arts

UP
AP

Friends
15. (c) Except J-Engineer, in all other options, there
are three persons between first and second
person.
Anu 7 7 7 7 3 7 7 7 7 3 7 7 7 7
Beena 7 7 7 7 7 7 3 7 7 7 3 7 7 7 16. (b) The position of L with respect to the scientist
is second to the right.
Kanika 7 7 7 3 7 7 7 7 7 7 7 7 3 7
Mohini 7 7 7 7 7 3 7 7 7 7 7 7 7 3
17. (a) Only Statement (a) is true.
Samina 7 7 3 7 7 7 7 7 7 7 7 3 7 7
Solutions (Q. Nos. 18-23) According to the given
information, arrangement is as follows
Easther 7 3 7 7 7 7 7 3 7 7 7 7 7 7
Rani 3 7 7 7 7 7 7 7 3 7 7 7 7 7 Floor Number Person Distance
7 N 3300 m
6 M 4700 m
5. (a ) Anu-Architecture combination is correct.
5 R 850 m
6. (d) Chhattisgarh-Engineering and Andhra 4 Q 1300 m
Pradesh-Commerce are correct.
3 S 2200 m
7. (c) Kanika-Chhattisgarh and Mohini-Punjab are 2 P 4000 m
correct.
1 O 2800 m
Chapter 16 PUZZLES 199

18. (b) Only S lives between P and the one who Solutions (Q.Nos. 29-35) According to the given information,
runs for 1300 m. sitting arrangement is as follows
19. (a) Three people live between S and N (History)
20. (c) Except pair, floor number 4-S, all other E
(Chemistry) C G (Hindi)
pairs have three place difference
between the elements. (English) B F (Geography)
21. (c) N lives on floor number 7 is true.
(Mathematics) A
H D (Biology)
22. (e) Here, M covered 4700 m.
(Physics)
Given, distance covered by B and
M = 5300 m 29. (a) Person who likes Hindi sits diagonally opposite to the
∴ B ran = 5300 − 4700 = 600m person who likes Mathematics.
23. (e) S runs for 2200 m. 30. (c) B and E are the immediate neighbours of the person
Solutions (Q. Nos. 24-28) According to the given who likes Chemistry.
information, arrangement is as follows 31. (e) A sits exactly between H and B.
Day Person Game
32. (e) B is an immediate neighbours of the one who likes
Mathematics is definitely true.
Monday D Football
33. (e) Person who likes Physics is third to the left of G.

A
Tuesday G Kho-kho
34. (a) D likes Biology.

M
Wednesday F Squash
35. (b) F likes Geography.

H
Thursday C Volleyball
36. (c) The given information can be represented as in the

TT
Friday A Hockey table given below
Saturday E Cricket
Sunday B Tennis A
Persons Companies Gender Occupation
W
A X Male Finance
24. (e) F was appointed on Wednesday. B Z Male/Female Marketing
SH

25. (b) Only three people were appointed before C C Y Male Engineer
is a true statement.
A

D X Female HR
26. (c) The person who plays tennis was
@

E Y Male Computer
appointed on Sunday.
F Y Male Marketing
27. (c) Saturday-Cricket is a correct combination.
G Z Male/Female Finance
28. (b) Only one person was appointed before G.
H Z Male/Female HR

Hence, HR specialist works in companies X and Z.


200 How to Crack Test of Reasoning VERBAL

17
Matrix Coding
Matrix is a set of elements laid out in The columns and rows of matrix I are numbered
tabular form (in rows and columns). from 0 to 4 and that of Matrix II are numbered
Matrix coding is a method to represent the from 5 to 9. A letter from these matrices can be
letters of English alphabet by two digits. represented first by its row and next by its

A
One digit is represented by the column.

M
corresponding row and the other digit is e.g. ‘M’ can be represented by 01, (Row, Column )

H
represented by the corresponding column. 20, (Row, Column ) etc., in the same way ‘T’ can

TT
The questions based on matrix coding be represented by 57, 85 etc. Similarly, you have
consist of two matrices i.e. Matrix I and to identify the set for the word given in each
Matrix II. Matrix I has five columns and question. A SSC (CPO) 2015
W
five rows marked from 0 to 4. Five letters
appear in each column and row but Illustration 1. LANE
SH

different in order. Matrix II has five Matrix I Matrix II


columns and five rows marked from 5 to 9. 0 1 2 3 4
A

5 6 7 8 9
Five letters appear in different order in each 0 Z M S R C 5 X K T E S
@

column and row. 1 J L D B G 6 Q A U Y P


Candidates are required to find the 2 M B C M H 7 U V O W E
assigned number to the letters of given 3 R L N S I 8 T Y A E U
word from the given matrices and select the 4 B D M R J 9 X O E V A
group of numbers which accompanies the (a) 11, 66, 33, 96 (b) 11, 67, 32,97
letter in the right format. (c) 31, 66, 33, 97 (d) 31, 87, 32, 97
Let us discuss some examples to understand
matrix coding. Illustration 2. A I R S
Directions (Illustrations 1-2) Read the Matrix I Matrix II
following information carefully and answer the 0 1 2 3 4 5 6 7 8 9
questions that follow. 0 A E M N P 5 I L R S T
In the following questions, a word is 1 N P A E M 6 R S T I L
represented by only one set of numbers as 2 E M N P A 7 T I L R S
given in any one of the alternatives. The sets 3 P A E M N 8 L R S T I
of numbers given in the alternatives are 4 M N P A E 9 S T I L R
represented by two classes of alphabets as in (a) 43, 55, 86, 95 (b) 12, 76, 99, 78
two Matrices given below. (c) 00, 68, 78, 88 (d) 24, 69, 56, 78
Chapter 17 MATRIX CODING 201

Solutions (Illustrations1-2) Illustration 3. A word is represented by only one


1. (d) set of numbers as given in anyone of the
Matrix I Matrix II alternatives. The sets of numbers given in the
0 1 2 3 4 5 6 7 8 9 alternatives are represented by two classes of
0 5 E alphabets as in the two matrices given below.
1 L 6 A
2 The columns and rows of matrix I are
7 E
3 L N 8 A E numbered from 0 to 4 and that of matrix II
4 9 E A are numbered from 5 to 9. A letter from these
Here, taking the row and column number of each matrices can be represented first by its row
letter, we have and next by its column, e.g. ‘O’ can be
L = 11, 31 represented by 01, 33, etc and ‘Q’ can be
A = 66, 87, 99 represented by 55, 78 etc. Similarly, you have
N = 32 to identify the set for the word METAL.
E = 58, 79, 88, 97 SSC (Steno) 2016
Now, taking each option and comparing them Matrix I Matrix II
with above values, we get 0 1 2 3 4

A
5 6 7 8 9
Option L A N E 0 M O R A L 5 Q U I E T

M
1 O R A L M 6 U I E T Q
(a) 11 66 33 96

H
5 5
2 R A L M O 7 I E T Q U
(b) 11 67 32 97

TT
5 3 A L M O R 8 E T Q U I
(c) 31 66 33 5 97 4 L M O R A 9 T Q U I E
(d) 31 87 32 97
A
(a) 23, 67, 96, 40, 44 (b) 23, 76, 95, 40, 44
W
(c) 32, 76, 95, 44, 04 (d) 32, 76, 44, 95, 04
Hence, only option (d) contains all the correct
values. Solutions (c)
SH

2. (a) Matrix I Matrix II


Matrix I Matrix II
A

0 1 2 3 4 5 6 7 8 9
0 1 2 3 4 5 6 7 8 9 0 M A L 5 E T
@

0 A 5 I R S 1 A L M 6 E T
1 A 6 R S I 2 A L M 7 E T
2 A 7 I R S 3 A L M 8 E T
3 A 8 R S I 4 L M A 9 T E
4 A 9 S I R
Here, taking the row and column number of each
Here, taking the row and column number of each letter, we have
letter, we have M → 00, 14, 23, 32, 41
A = 00, 12, 24, 31, 43 E → 58, 67, 76, 85, 99
I = 55, 68, 76, 89, 97 T → 59, 68, 77, 86, 95
R = 57, 65, 78, 86, 99 A → 03, 12, 21, 30, 44
S = 58, 66, 79, 87, 95 L → 04, 13, 22, 31, 40
Now, taking each option and comparing them Now, taking each option and comparing them
with above values, we get with above values, we get
Option A I R S Option M E T A L
(a) 43 55 86 95 (a) 23 67 96 5 40 5 44 5
(b) 12 76 99 78 5 (b) 23 76 95 40 5 44 5
(c) 00 68 78 88 5 (c) 32 76 95 44 04
(d) 24 69 5 56 5 78 5 (d) 32 76 44 5 95 5 04
Hence, only option (a), contains all the correct Hence, only option (c) contains all the correct
values. values.
202 How to Crack Test of Reasoning VERBAL

Let us Practice
Directions (Q. Nos. 1-16) Read the following 4. CART SSC (Multitasking) 2014
information carefully and answer the questions
Matrix I Matrix II
that follow.
0 1 2 3 4 5 6 7 8 9
A word is represented by only one set of
numbers as given in any one of the 0 E A R W P 5 S B K T C
alternatives. The sets of numbers given in the 1 W P A E R 6 B C T K S
alternatives are represented by two classes of 2 A W P R E 7 T S C B K
alphabets as in two matrices given below. 3 P R E A W 8 K T S C B
The columns and rows of Matrix I are 4 R E W P A 9 C K B S T
numbered from 0 to 4 and that of Matrix II (a) 65, 33, 40, 86 (b) 66, 12, 40, 58
are numbered from 5 to 9. A letter from these (c) 88, 44, 31, 89 (d) 59, 20, 32, 89
matrices can be represented first by its row
and next by its column.
5. VOLT SSC (10 + 2) 2018
Matrix I Matrix II

A
Now, you have to identify the set for the word
given in each question. 0 1 2 3 4 5 6 7 8 9

M
0 A F B I M 5 V U X Q X
1. MARX SSC ( 10 + 2) 2017

H
1 F C J C B 6 T V V Q X
Matrix I Matrix II

TT
2 B G F F F 7 W Y T X W
0 1 2 3 4 5 6 7 8 9
3 C L E J B 8 Z S N Z T
0 F L F B L 5 X V N X Z
1 K G A B H 6 U U V Q U 4 K A M C A E 9 N O S N X
W
2 G C L F E 7 P V Z R V (a) 67, 96, 31, 65 (b) 20, 89, 14, 59
3 F M J K C 8 O N T V W
SH

(c) 43, 58, 31, 65 (d) 30, 57, 30, 68


4 D E A G L 9 Q X R N X
6. BALD SSC (Multitasking) 2013
(a) 34, 67, 40, 67 (b) 13, 57, 33, 68
A

(c) 31, 42, 97, 99 (d) 43, 95, 13, 66 Matrix I Matrix II
@

2. PEAK SSC (10 + 2) 2017


0 1 2 3 4 5 6 7 8 9
Matrix I Matrix II 0 B T D I F 5 A L R E K
1 I D B F T 6 K A L R E
0 1 2 3 4 5 6 7 8 9
0 I F E I H 5 X Z Q Q T 2 F B I T D 7 E K A L R
1 E M I J C 6 O P W Y W 3 T I F D B 8 R E K A L
2 D M D G G 7 U Q P U S 4 D F T B I 9 L R E K A
3 A I C E H 8 X S U P Q
4 I K E C I 9 V U X N T (a) 12, 99, 65, 24 (b) 21, 88, 95, 24
(c) 43, 55, 67, 04 (d) 34, 77, 76, 42
(a) 43, 66, 33, 55 (b) 34, 76, 11, 87
(c) 41, 79, 31, 95 (d) 77, 42, 30, 41 7. EASE SSC (CGL) 2017, 16

3. ROST BSSC (CGL) 2015, SSC (CGL) 2014 Matrix I Matrix II


Matrix I Matrix II 0 1 2 3 4 5 6 7 8 9
0 1 2 3 4 5 6 7 8 9 0 E S U O H 5 E K A N S
4 K L M N O 9 P Q R S T 1 S U H E O 6 K A S E N
3 L M K O N 8 T S Q P R 2 O H E S U 7 N S E K A
2 N O L M K 7 R T S Q P 3 U E O H S 8 A E N S K
1 M N O K L 6 S P T R Q 4 H O U S E 9 S N K A E
0 O K N L M 5 Q R P T S
(a) 56, 44, 67, 40 (b) 97, 21, 66, 29 (a) 55, 85, 44, 42 (b) 77, 85, 88, 44
(c) 75, 00, 10, 92 (d) 68, 33, 65, 58 (c) 77, 66, 31, 44 (d) 00, 98, 23, 98
Chapter 17 MATRIX CODING 203

8. GEAR SSC (10+2) 2017 12. CARS Delhi Police (SI and ASI) 2016
Matrix I Matrix II Matrix I Matrix II
0 1 2 3 4 5 6 7 8 9 0 1 2 3 4 5 6 7 8 9
0 A S E G A 5 E T R R F 0 E A R W P 5 S B K T C
1 Q D H L S 6 G R P L R 1 W P A E R 6 B C T K S
2 W N T K D 7 I O U R M 2 A W P R E 7 T S C B K
3 H D Y A F 8 D F R K Q 3 P R E A W 8 K T S C B
4 R G A N G 9 S R W D E 4 R E W P A 9 C K B S T

(a) 03, 99, 57, 77 (b) 44, 55, 42, 66 (a) 96, 00, 23, 99 (b) 95, 01, 13, 77
(c) 41, 00, 02, 78 (d) 65, 02, 00, 43 (c) 66, 20, 31, 88 (d) 77, 33, 40, 69
9. GOES SSC (10+2) 2018 13. SILK Delhi Police (SI and ASI) 2016
Matrix I Matrix II Matrix I Matrix II
0 1 2 3 4 5 6 7 8 9 0 1 2 3 4 5 6 7 8 9
0 A A J M G 5 W V P V P 0 M L F H B 5 I K S U N
1 C J H G J 6 R S P Z U 1 H B M L F 6 U N I K S

A
2 H B L M D 7 R Z Z O T 2 L F H B M 7 K S U N I

M
3 I J L I C 8 Q T N S P 3 B M L F H 8 N I K S U
4 E E D M L 9 Y Q R V X

H
4 F H B M L 9 S U N I K

TT
(a) 11, 75, 12, 77 (b) 32, 76, 44, 95 (a) 76, 67, 32, 68 (b) 76, 67, 32, 65
(c) 13, 78, 41, 66 (d) 33, 88, 21, 89 (c) 76, 67, 33, 68 (d) 76, 66, 33, 68
10. EAST SSC (FCI) 2012 14. HALT A SSC (CPO) 2017
W
Matrix I Matrix II Matrix I Matrix II
SH

0 1 2 3 4 5 6 7 8 9 0 1 2 3 4 5 6 7 8 9
0 E A R W P 5 S B K T C 0 T E R A H 5 L O P U E
A

1 W P A E R 6 B C T K S 1 E R H T A 6 O P E L U
@

2 A W P R E 7 T S C B K 2 A H E R T 7 U E L O P
3 P R E A W 8 K T S C B 3 R T A H E 8 P L U E O
4 R E W P A 9 C K B S T 4 H A T E R 9 E U O P L

(a) 00, 12, 76, 58 (b) 32, 34, 76, 68 (a) 40, 03, 76, 24 (b) 21, 41, 68, 13
(c) 41, 20, 77, 59 (d) 24, 02, 55, 76 (c) 12, 14, 69, 00 (d) 34, 41, 87, 31
11. DOOR SSC (10+2) 2011 15. FAITH SSC (10+2) 2014
Matrix I Matrix II Matrix I Matrix II
0 1 2 3 4 5 6 7 8 9 0 1 2 3 4 5 6 7 8 9
0 F O M S R 5 A T D I P 0 F A N O I 5 S E H B T
1 S R F O M 6 I P A T D 1 I 0 F A N 6 H S E T B
2 O M S R F 7 T D I P A 2 A N O I F 7 B T S E H
3 R F O M S 8 P A T D I 3 O F I N A 8 E H T B S
4 M S R F O 9 D I P A T 4 N I A F O 9 T S E H B

(a) 69, 44, 20, 43 (a) 24, 31, 10, 59, 57


(b) 76, 01, 44, 24 (b) 12, 20, 40, 68, 65
(c) 95, 20, 44, 12 (c) 31, 34, 23, 76, 79
(d) 57, 13, 32, 23 (d) 43, 42, 41, 78, 89
204 How to Crack Test of Reasoning VERBAL

16. ORGAN SSC (CGL) 2015 Matrix I Matrix II


Matrix I Matrix II
0 1 2 3 4 5 6 7 8 9
0 1 2 3 4 5 6 7 8 9 0 P W N I S 5 A E R O H
0 P A G R Z 5 E M L N O 1 I S P W N 6 O H A E R
1 G R Z P A 6 L E O M N 2 W N I S P 7 E R O H A
2 Z P A G R 7 O N E L M 3 S P W N I 8 H A E R O
3 A G R Z P 8 N O M E L 4 N I S P W 9 R O H A E
4 R Z P A G 9 M L N O E
(a) 75, 03, 11, 22, 76
17. HIPS
(a) 85, 41, 24, 11 (b) 66, 21, 24, 11
(b) 67, 22, 31, 58, 22
(c) 86, 40, 23, 14, 96 (c) 67, 41, 24, 42 (d) 78, 34, 23, 04
(d) 98, 03, 44, 22, 58 18. SORROW
(a) 23, 86, 69, 88, 65, 33
Directions (Q. Nos. 17-21) Read the following
(b) 23, 43, 14, 33, 65, 78
information carefully and answer the questions
(c) 11, 66, 69, 65, 59, 97
given below.
(d) 42, 65, 95, 88, 77, 44
A word is represented by only one set of
numbers as given in anyone of the alternatives. 19. WEAR

A
(a) 44, 68, 67, 87 (b) 44, 87, 98, 69
The set of numbers given in the alternatives

M
(c) 20, 86, 67, 87 (d) 32, 87, 78, 95
are represented by two classes of alphabets as

H
in the two matrices given below. 20. HORROR
(a) 78, 69, 77, 89, 76, 97

TT
The columns and rows of Matrix I are numbered
(b) 85, 65, 57, 58, 97, 95
from 0 to 4 and that of Matrix II from 5 to 9. A
(c) 66, 58, 77, 57, 69, 76
letter can be represented first by its row and A
(d) 78, 65, 88, 95, 96, 57
W
next by its column number, e.g. 'N' can be
represented by 02,21, etc. 'O' can be represented 21. PENS
SH

by 65,96 etc. Similarly, you have to identify the (a) 12, 67, 21, 30 (b) 43, 56, 13, 23
correct set for the word given in each question. (c) 43, 56, 21, 42 (d) 31, 57, 21, 42
A
@

Answers
1. (c) 2. (d) 3. (d) 4. (b) 5. (a) 6. (b) 7. (b) 8. (b) 9. (c) 10. (a)
11. (d) 12. (d) 13. (a) 14. (b) 15. (c) 16. (d) 17. (a) 18. (d) 19. (b) 20. (d)
21. (c)) 22. (a) 23. (b) 24. (c) 25. (d) 26. (b) 27. (d) 28. (a) 29. (a) 30. (a)
Chapter 17 MATRIX CODING 205

Answer with Explanations


1. (c) According to the matrices, 8. (b) According to the matrices, I - 04, 10, 23, 32, 41
M - 31 G - 03, 44, 65 T - 59, 68, 76, 87, 95
A - 12, 42 E - 02, 55, 99 H - 57, 65, 79, 86, 98
R - 78, 97 A - 00, 04, 33, 42
∴ FAITH ⇒ 31, 34, 23, 76, 79
X - 55, 58, 96, 99 R - 40, 57, 58, 66, 69, 78, 87, 96
∴ GEAR ⇒ 44, 55, 42, 66 16. (d) According to the matrices,
∴ MARX ⇒ 31, 42, 97, 99
O - 59, 67, 75, 86, 98
2. (d) According to the matrices, 9. (c) According to the matrices,
R - 03, 11, 24, 32, 40
P - 66, 77 G - 04, 13
G - 02, 10, 23, 31, 44
E - 02, 10 , 33, 42 O - 78,
A - 01, 14, 22, 30, 43
A - 30 E - 40, 41
N - 58, 69, 76, 85, 97
K - 41, S - 66, 88
∴ GOES ⇒ 13, 78, 41, 66 ∴ ORGAN
∴ PEAK ⇒ 77, 42, 30, 41
3. (d) According to the matrices, 10. (a) According to the matrices, ⇒ 98, 03, 44, 22, 58
R - 97, 89, 75, 68, 56 E - 00, 13 , 24, 32, 41 17. (a) According to the matrices,

A
O - 44, 33, 21, 12, 00 A - 01, 12, 20 , 33, 44 H - 59, 66, 78, 85, 97

M
S - 98, 86, 77, 65, 59 S - 55, 69, 76, 87, 98 I - 03, 10, 22, 34, 41
T - 99, 85, 76, 67, 58 P - 00, 12, 24, 31, 43

H
T - 58, 67, 75, 86, 99
∴ ROST ⇒ 68, 33, 65, 58 S - 04, 11, 23, 30, 42

TT
∴ EAST ⇒ 00, 12, 76, 58
4. (b) According to the matrices, ∴ HIPS ⇒ 85, 41, 24, 11
C - 59, 66, 77, 88, 95 11. (d) According to the matrices,
A - 01, 12 , 20, 33, 44 D - 57, 69, 76, 88, 95 A
18. (d) According to the matrices,
W
S - 04, 11, 23, 30, 42
R - 02, 14, 23, 31, 40 O - 01, 13, 20, 32, 44 O - 58, 65, 77, 89, 96
SH

T - 58, 67, 75, 86, 99 R - 04, 11, 23, 30, 42 R - 57, 69, 76, 88, 95
∴ CART ⇒ 66, 12, 40, 58 ∴ DOOR ⇒ 57, 13, 32, 23 W - 01, 13, 20, 32, 44
A

5. (a) According to the matrices, 12. (d) According to the matrices, ∴ SORROW
⇒ 42, 65, 95, 88, 77, 44
@

V - 55, 66, 67 C - 59, 66, 59, 77, 88, 95


O - 96 A - 01, 12, 20, 33, 44 19. (b) According to the matrices,
R - 02, 14, 23, 31, 40 W - 01, 13, 20, 32, 44
L - 31 E - 56, 68, 75, 87, 99
S - 55, 76, 87, 98, 69
T - 65, 77, 89 ∴ CARS ⇒ 77, 33, 40, 69 A - 55, 67, 79, 86, 98
∴ VOLT ⇒ 67, 96, 31, 65 13. (a) According to the matrices, R - 57, 69, 76, 88, 95
6. (b) According to the matrices, S - 57, 69, 76, 88, 95 ∴ WEAR ⇒ 44, 87, 98, 69
B - 00, 12, 21, 34, 43 I - 55, 67, 79, 86, 98 20. (d) According to the matrices,
A - 55, 66, 77, 88, 99 L - 01, 13, 20, 32, 44 H - 59, 66, 78, 85, 97
L - 56, 67, 78, 89, 95 K - 56, 68, 75, 87, 99 O - 58, 65, 77, 89, 96
D - 02, 11, 24, 33, 40 ∴ SILK ⇒ 76, 67, 32, 68 R - 57, 69, 76, 88, 95,
∴ BALD ⇒ 21, 88, 95, 24 14. (b) According to the matrices, ∴ HORROR
7. (b) According to the matrices, H - 04, 12, 21, 33, 40 ⇒ 78, 65, 88, 95, 96, 57
E - 00, 13, 22, 31, 44 A - 03, 14, 20, 32, 41
21. (c) According to the matrices,
55, 68, 77, 86, 99 L - 55, 68, 77, 86, 99
P - 00, 12, 24, 31, 43
A - 57, 66, 79, 85, 98 T - 00, 13, 24, 31, 42
E - 56, 68, 75, 87, 99
S - 01, 10, 23, 34, 43 ∴ HALT ⇒ 21, 41, 68, 13
15. (c) According to the matrices, N - 02, 14, 21, 33, 40
59, 67, 76, 88, 95
S - 04, 11, 23, 30, 42
∴ EASE ⇒ 77, 85, 88, 44 F - 00, 12, 24, 31, 43
A - 01, 13, 20, 34, 42 ∴ PENS ⇒ 43, 56, 21, 42
206 How to Crack Test of Reasoning VERBAL

18
Mathematical
Reasoning
Mathematical reasoning involves the basic 3 x2

A
⇒ = 10800
mathematical and arithmetic problems. 4

M
Mathematical reasoning test is designed to 10800 × 4
⇒ x2 = = 14400

H
test the ability of a candidate to solve the 3

TT
various Mathematical problems which are ∴ x = 14400 = 120
encountered in day to day life. To solve the
problems on Mathematical reasoning, a A
Illustration 3. The difference of the ages of Rohit
W
candidate should have a knowledge of and Axar is 12 yr. The ratio of their ages is 3 : 5.
concepts of arithmetic or basic Mathematics. The age of Axar is WBPSC 2018
SH

(a) 32 yr (b) 24 yr
Examples given below, will give you a better
(c) 28 yr (d) 30 yr
idea about the types of questions asked in
A

various examinations. Solution. (d) Let present ages of Rohit and Axar be 3x
@

and 5x, respectively.


Illustration 1. If cost of 15 eggs is ` 75, then find According to the question, 5 x − 3 x = 12
out the cost of 4 dozens eggs. ⇒ 2 x = 12
(a) ` 240 (b) ` 300 (c) ` 150 (d) ` 185 ⇒ x =6
Solution (a) Q Cost of 15 eggs = ` 75 ∴The age of Axar = 5 x = 5 × 6 = 30 yr.
75
∴Cost of 1 egg = ` =`5 Illustration 4. There are deer and peacocks in a
15
∴Cost of 4 dozens ( 4 × 12 = 48) eggs = 5 × 48 zoo. By counting heads they are 80. The number
= ` 240 of their legs is 200. How many peacocks are
there? SSC (CGL) 2015
Illustration 2. If a number is multiplied by (a) 60 (b) 50
three-fourth of itself, then the value thus (c) 20 (d) 30
obtained is 10800. What is that number?
Solution. (a) Assuming deer = d and peacock = p
(a) 210 (b) 180
×4
(c) 120 (d) 160 Head → d + p = 80  → 4d + 4p = 320 …(i)
Solution (c) Let the number be x. Legs → 4d + 2 p = 200 → 4d + 2 p = 200 …(ii)
According to the question, From subtracting Eq. (ii) from Eq. (i), we get
 3 120
x ×  x ×  = 10800 2 p = 120 ⇒ p = = 60
 4 2
Chapter 18 MATHEMATICAL REASONING 207

Illustration 5. In an examination, 78% of the total Illustration 6. At the end of a business conference
students who appeared were successful. If the all the ten people present, shake hands with each
total number of failures was 176 and 34% got other only once. How many handshakes were
first class, then how many students got first there altogether?
class? SSC (CGL) 2014 (a) 20 (b) 45
(a) 272 (b) 112 (c) 210 (d) 254 (c) 55 (d) 90
Solution (a) Let total students appeared in an exam = x n (n − 1)
22 Solution. (b) Total number of handshakes =
Then, 22% of x = 176 ⇒ × x = 176 2
100 where, n = number of people
176 × 100
⇒ x= = 8 × 100 ⇒ x = 800 Total number of handshakes
22 10 (10 − 1)
Now, number of students who got first class =
2
34
= 34% of 800 = × 800 = 34 × 8 = 272 10 × 9
100 = = 45
2

Let us Practice

A
M
H
1. David divides 78 by half and adds 11. What 6. How many odd numbered pages are there in

TT
number he gets at the end ? IB (ACIO) 2017 a book of 1089 pages?
(a) 50 (b) 44 1 2 (c) 167 (d) 83 1 2 (a) 542
A
(b) 545 (c) 544 (d) 546
W
2. The sum of all the 3-digit numbers which are 7. A shepherd had 17 sheeps. All but nine died.
formed by the digits 1, 2 and 3 without How many was he left with?
SH

repetition of digits, is CGPSC 2014 (a) 17 (b) 9 (c) Nil (d) 8


(a) 1233 (b) 1321
A

(c) 1323 (d) 1332


8. A person is given 1 rupee for shooting at the
target and if he misses, then he has to give
@

(e) None of these


1 rupee. If he gets 30 rupees for 100 chances
3. 12 yr old Sami is three times as old as his then how many chances did he miss?
brother Vinay. How old will Sami be when Chattisgarh (PCS) 2008
he is twice as old as Vinay? (a) 25 (b) 35
Delhi Police (SI and ASI) 2016
(c) 40 (d) 45
(a) 14 yr (b) 16 yr
(c) 20 yr (d) 18 yr 9. There were a total of 10 bicycles and
tricycles. If the total number of wheels was
4. A man climbing up a wall of 24 m high. He 24, how many tricycles were there?
climbs 16 m in a day but slipped back by 3 m MCA NIMCET 2008
40 cm in the evening. How far had the man (a) 2 (b) 6 (c) 18 (d) 4
reached on that day? SSC (CPO) 2015
(a) 11.4 m (b) 12.6 m 10. P is greater than Q by 60% and greater than
(c) 12 m 40 cm (d) 19 m 40 cm R by 30%. The ratio of Q and R is WBPSC 2018
(a) 1 : 2 (b) 2 : 1
5. Two horses A and B run at a speed of 3 : 2
(c) 13 : 16 (d) 16 : 13
ratio in the first lap; during the second lap
the ratio differs by 4 : 7; during the third lap 11. In a group of cows and hens, the number of
the ratio differs by 8:9. What is the difference legs are 14 more than twice the number of
in ratio of speed altogether between the two heads. Find the number of cows?
horses? SSC (CPO) 2015 (a) 5 (b) 7
(a) 4 (b) 1 (c) 3 (d) 2 (c) 10 (d) 12
208 How to Crack Test of Reasoning VERBAL

12. If it takes two workers, working separately 16. In a company, 60% workers are males. If the
but at the same speed, 2 h and 40 min to number of female workers in the company is
complete a particular task, about how long 800, what is the number of male workers in
will it take one worker, working at the same the company? CLAT-2017
speed, to complete the same task alone? (a) 1600 (b) 1400
CLAT 2015
(c) 1900 (d) 1200
(a) 1 h 20 min (b) 4 h 40 min
(c) 5 h (d) 5 h 20 min 17. The flowers kept in a basket doubles in every
one minute. If the basket gets completely
13. Sonu and his friend Rahul went for filled by flowers in 30 min, then in how
shopping. Sonu had ` 500 with him while 1
Rahul had ` 240. Sonu spent twice as much many minutes th of the basket was filled
4
as Rahul on shopping. Now, Sonu has three with flowers? SSC (CPO) 2006
times as much money as is left with Rahul. (a) 15 min (b) 28 min
How much money did Sonu spend? 15 45
SSC (Steno) 2016 (c) min (d) min
(a) ` 220 (b) ` 60 (c) ` 440 (d) ` 120 2 2

14. A man has ` 480 in the denominations of 18. Some birds are sitting on two branches A
and B. If one bird of the branch A fly away

A
one-rupee notes, five-rupee notes and
and sit on branch B, then the number of

M
ten-rupees notes. The number of notes of
each denomination is equal. What is the total birds on both the branches will become

H
number of notes that he has? equal. But if a bird from branch B flies and

TT
sits on the branch A then the number of
(a) 45 (b) 60 (c) 75 (d) 90
birds on branch A is double of branch B.
15. The ratio of present ages of Sumit and Amit
is 3 : 4. If the age of Sumit 20 yr hence will be A
What number of birds were sitting on
W
branch A at the beginning?
62 yr, then what is the present age of Amit? (a) 3 (b) 4
SH

SSC (CPO) 2017


(c) 5 (d) 7
(a) 56 yr (b) 64 yr (c) 60 yr (d) 52 yr
A
@

Answers
1. (c) 2. (d) 3. (b) 4. (b) 5. (c) 6. (b) 7. (b) 8. (b) 9. (d) 10. (c)
11. (b) 12. (d) 13. (c) 14. (d) 15. (a) 16. (d) 17. (b) 18. (d)

Answer with Explanations


1. (c) According to the question, Then, 12 + x = 2 (4 + x )
78 78 × 2 12 + x = 8 + 2 x
Number = + 11 = + 11 = 156 + 11 = 167
1 1 ⇒ 4=x
2 In 4 more years, Sami will be twice his brother’s
Hence, David gets 167 at the end. age. So, Sami’s age = 12 + 4 = 16 yr
2. (d) The 3-digit numbers which are formed by digits 4. (b) Distance covered by man = (16 − 3.4) m
1, 2 and 3 without repetition of digits are given
below = 12.6 m
312 + 321 + 123 + 231 + 132 + 213 = 1332 5. (c) Difference between ratio of speed of both
3. (b) If Sami is 12, one-third of his age is 4, which is horses = (2 + 7 + 9) − (3 + 4 + 8)
his brother’s age = 18 − 15 = 3
Chapter 18 MATHEMATICAL REASONING 209

6. (b) Required number of pages 13. (c) Let expenditure of Rahul = `x


Total number of pages + 1
= and expenditure of Sonu = ` 2x
2
1089 + 1 1090 Now, 500 − 2 x = 3(240 − x )
= = = 545 ⇒ 500 − 2 x = 720 − 3x
2 2
7. (b) According to the question, ‘All but nine died’. ⇒ x = ` 220
This statement means that ‘All except nine died’ ∴ Expenditure of Sonu = 2 x = `440
i.e. nine sheeps remained alive and others died. 14. (d) Let the number of notes of each denomination = x
So, shepherd was left with 9 sheeps.
Total amount = 480
8. (b) Assuming the number of chances missed by x + 5x + 10x = 480
the person be x. ⇒ 16x = 480
Number of chances to shoot the target = 100 − x ⇒ x = 30
So, required total number of notes = x + x + x
According to the questions, (100 − x ) − x = 30 [number of notes of each denomination is equal]
⇒ 100 − 2 x = 30 = 3x = 3 × 30 = 90
⇒ 2 x = 100 − 30 = 70
15. (a) Let present ages of Sumit and Amit be 3x and
70
x= = 35 4x, respectively.
2 According to the question,

A
9. (d) Let the required number of tricycles = x 3x + 20 = 62 ⇒ 3x = 62 − 20

M
Then, the number of bicycles = 10 − x ⇒ 3x = 42 ⇒ x = 14

H
According to the question, 3x + 2(10 − x) = 24 Present age of Amit = 4x = 4 × 14 = 56 yr.

TT
[Q Tricycles = 3 wheels; Bicycle = 2 wheels] 16. (d) Let the total number of workers = x
⇒ x + 20 = 24
⇒ x=4 A
Given, number of female workers = 800
W
Therefore, the required number of tricycles = x = 4 Then, according to the question,
60
of x + 800 = x
SH

10. (c) Given, P = 160% of Q = 130% of R 100


⇒ 160% of Q = 130% of R 60x
∴ x− = 800
A

160 130
⇒ Q× = R× ⇒ Q × 16 = R × 13 100
@

100 100 800 8000


Q 13 ⇒ x= = = 2000
⇒ = ⇒ Q : R = 13 : 16 0.4 4
R 16 60
Number of male workers = × 2000 =1200
11. (b) Let the number of hens be x and cows be y. 100
Number of legs in the group = 2 x + 4y 17. (b)Q 30min→ 1 [Full]
Number of heads in the group = x + y 1
∴ (30 − 1)min ⇒ 29min →
According to the question, 2
legs = 2 (heads) + 14 1
∴ (29 − 1)min ⇒ 28min →
2 x + 4y = 2(x + y ) + 14 4
⇒ 2 x + 4y = 2 x + 2 y + 14 Therefore, the expected time is 28 min.
⇒ 2 x − 2 x + 4y − 2 y = 14 18. (d) Assuming the number of birds on A = x and B = y
⇒ 2 y = 14 In Ist stage, x − 1 = y + 1 …(i)
⇒ y =7 (therefore number of cows =7) In IInd stage, x + 1 = 2(y − 1) …(ii)
From Eqs. (i) and (ii), we get
12. (d) Given that the two workers, working separately
(x + 1) = 2(x − 2 − 1) ⇒ x + 1 = 2 (x − 3)
but at the same speed takes 2 h and 40 min to
complete a particular task. Then, one worker, ⇒ x + 1= 2x − 6
working at the same speed, to complete the same ⇒ 2x − x = 7 ⇒ x = 7
task will take double time i.e. 5 h 20 min. Therefore, total number of birds on branch ‘A’ = 7.
210 How to Crack Test of Reasoning ANALYTICAL
ANALYTICAL REASONING

19
Logical
Venn Diagrams

A
M
Venn diagrams are used to visualise logical

H
relationship among given groups of elements. Illustration 1. Vehicle, Car, Bus

TT
In this chapter, we deal with the questions Solution Venn diagram would be as follows
which aim at analysing a candidate’s ability to Vehicle
relate a given groups of items and illustrate it A
W
diagrammatically. Different geometrical figures
Car Bus
SH

like circles, triangles, squares and rectangles,


etc. are used to represent these diagrams.
A

Types of Questions Bus and car are entirely different but both are vehicles.
@

Illustration 2. Hospital, Nurse, Patient


Based on the variety of questions that are
asked in various competitive exams, we have Solution Venn diagram would be as follows
classified logical venn diagrams into two types Hospital
as follows

Type 1 Nurse Patient

Identification of Venn Diagram


In this type of questions, we deal with different Nurse and patient are entirely different but both
types of cases which are as follows are present in hospital.

Case I If two separate groups of items are Case II When two groups of items have some
completely unrelated to each other but they are common relationship and both of them are
all completely included in the third group, then completely included in the third group, the
the relationships can be diagrammatically relationships are shown by two smaller
shown as in the given Illustration. intersecting circles in a third large circle.
Chapter 19 LOGICAL VENN DIAGRAMS 211

Illustration 3. Lawyers, Teachers, Educated Illustration 6. Females, Mothers, Doctors


Solution Venn diagram would be as follows Solution Venn diagram would be as follows
Some lawyers are teachers and some teachers are
lawyers but all lawyers and teachers are educated. Females

Educated Mothers Doctors

Lawyers
Teachers

All mothers are females but some females and


some mothers can be doctors. So, the circle
Case III If one item belongs completely to the representing doctors would intersect both of the
class of second while, third item is entirely two concentric circles.
different from the two, then they may be
represented by the given below diagram in the
Illustration 7. Doctor, Educated, Employed
SSC (Steno) 2014
illustration.
Solution Venn diagram would be as follows
Illustration 4. Engineers, Human Beings, Rats
Solution Venn diagram would be as follows Educated

A
Doctor Employed

M
Human Beings

H
Engineers Rats

TT
All the doctors are educated but some are
employed.
We know that, all engineers are human beings but A
W
rats are entirely different from both of these. Case VI If one item belongs to the class of
SH

second and the third item is partly related


Case IV If one group of items is partly included to the second, then their relationship is as
A

in the second group of items and the third shown in the following illustration.
group is completely unrelated to these two
@

Illustration 8. Males, Fathers, Children


groups, then their relationship is
diagrammatically shown as in the following Solution Venn diagram would be as follows
illustration.
Males
Illustration 5. Wire, Copper, Paper
Solution Venn diagram would be as follows Fathers Children

Wire Copper Paper

All fathers are males. This would be represented


Some wires are made of copper but paper is entirely by two concentric circles but some males are
different. children and children cannot be fathers.

Case V If one item belongs to the class of Case VII If the items evidently belong to
second and the third item is partly related to three different groups, then they are
these two, then their relationship is as shown in represented by three non-intersecting
the following illustration. circles.
212 How to Crack Test of Reasoning ANALYTICAL

Illustration 9. Doctors, Lawyers, Engineers. Illustration 12. Married, Women, Teachers


Solution Venn diagram would be as follows Solution Venn diagram would be as follows

Doctors Married Women


Engineers

Lawyers Teachers

Some women are teachers. Some women are


These three have no relationship between each married. Some teachers are married. Some married
other. women are teachers.
So, they are represented by three disjoint circles.

Case VIII If one item belongs to the class of


second and the second belongs to the class of
Type 2
third, then they are represented by three Analysis of Venn Diagram
concentric circles.
In this type of questions, generally a Venn

A
Illustration 10. Seconds, Minutes, Hours diagram comprising of different geometrical

M
Solution Venn diagram would be as follows figures is given. Each geometrical figure in
the diagram represents a certain class. The

H
candidate is required to study and analyse the

TT
Hours
figure carefully and then answer the given
questions based on it.
Seconds
A
Directions (Illustrations 13-14) Study the
W
Minutes
following diagram to answer these questions.
SH

Clearly, hours consists of minutes and minutes


A

Triangle 5 Circle
consists of seconds.
@

9 2 1
Case IX If two items are partly related to the 8 Rectangle
third and are themselves independent of each
other, then they are represented by three
intersecting circles in a line. Illustration 13. Find out the number, that lies
inside all the figures.
Illustration 11. Dogs, Pets, Cats
(a) 2 (b) 5 (c) 9
Solution Venn diagram would be as follows (d) No such number is there
Solution (a) Such number should belong to all three
Dogs Pets Cats
figures, i.e. circle, rectangle and triangle. There is only
one number, i.e. 2 which belongs to all three figures.
Illustration 14. What are the numbers that lie
Clearly, some dogs and some cats are pets. But all inside any two figures?
the pets are not dogs or cats. Also dogs and cats
are not related to each other. (a) 2, 1 (b) 5, 1
(c) 5, 9 (d) 9, 1
Case X If the three items are partly related to Solution (b) Such numbers are 5 and 1 because 5 lies
each other, then they are represented as inside the triangle and circle. And, 1 lies inside the
shown in the following illustration. circle and rectangle.
Chapter 19 LOGICAL VENN DIAGRAMS 213

Directions (Illustrations 15-16) Study the Illustration 16. Which part shows Males, who are
figure given below carefully and answer the neither Doctor nor Literate?
questions that follow. (a) A (b) B (c) C (d) D
Literate Solution (c) To depict the required part, the figure should
Doctor
show the region common to only rectangle, i.e. C. C
shows Males, who are neither Doctor nor Literate.
B
G
A Illustration 17. Which one of the areas marked
E D
I-VII represents the urban educated who are
F not hard working? SSC (CGL) 2015
C
Males
I
Urban
II
Illustration 15. Which part shows Literate Males, III IV Hard working
who are Doctors? VI V VII Educated
(a) G (b) E
(c) D (d) F
(a) II (b) I (c) IV (d) V

A
Solution (b) To depict the required part, the figure
should show the region common to all the three Solution (c) The area which represents the urban

M
diagrams, i.e. E. E shows literate Males, who are educated who are not hard working lies outside the

H
Doctors. square but common to triangle and circle, i.e. IV.

TT
Let us Practice A
W
SH

Base Level Exercise


A
@

Directions (Q. Nos. 1-14) Each of the question given below, contains three elements. These three
elements may or may not have some link. Each group of the elements may fit into one of the diagrams
(a), (b), (c), (d) and (e). You have to select the diagram, which expresses the correct relationship among
given elements.
5. Words with no vowel, words with one
vowel, words with two vowels
6. School, Classroom, Blackboard

(a) (b) (c) 7. Planets, Earth, Sun


8. Room, Wall, Window
9. Grains, Wheat, Maize
(d) (e) 10. Fruits, Plums, Tomato
1. Week, Day, Year 11. Living beings, Men, Rodents
2. Mammals, Cows, Crows 12. Iron, Lead, Nitrogen
3. House, Bedroom, Bathroom 13. Cabinet, Minister, Home Minister
4. Metal, Iron, Chlorine 14. Homosapiens, Woman, Mother
214 How to Crack Test of Reasoning ANALYTICAL

15. Which of the diagram represents the 22. Truck, Ship, Goods
statement-some musicians are teachers, some 23. In a village, there are landlords of which
teachers are players but no musician is a some are literate. Which of the following best
player? DSSSB 2017
expresses the relationship between them?
(a) (b) SSC (CGL) 2016

(a) (b)

(c) (d) (c) (d) None of these

24. Identify the diagram that best represents the


Directions (Q. Nos. 16-20) Each question relationship among the classes given below
given below, has three items having certain Soda water, Mineral water, Liquid
relationship among them. The same relationship SSC (CGL) 2015
is expressed by sets of circles, each circle
representing one item irrespective of its size.
Match the items with right set of circles.

A
(a) (b)

M
H
(a) (b) (c)

TT
(d)
(c) (d)
A
W
25. Identify the diagram that best represents the
(e) relationship among the given classes.
SH

16. Rivers, Canals, Perennial source of water Males, Doctors, Brothers SSC (10+2) 2017
17. Rings, Ornaments, Diamond rings
A

18. Women, Married persons, Wives who work


@

19. Computer skills, Graduates, Employed


20. Students, First divisioners, Third divisioners
(b)
(a) (c) (d)
Directions (Q. Nos. 21 and 22) In each of
these question, three words are related in some 26. Which of the following figures give the
way or other. The relationship among the words proper relation to Fruit, Red and Shirt?
in the question can be best represented by one of SSC (Steno) 2016
the five diagrams (a), (b), (c), (d) and (e) given
below. Choose that correct diagram.

(a) (b) (c) (d)


27. Identify the diagram that best represents the
(a) (b) (c) relationship among the given classes.
Tree, Branches, Root SSC (CPO) 2017

(d) (e)

21. Goat, Carnivores, Lion (a) (b) (c) (d)


Chapter 19 LOGICAL VENN DIAGRAMS 215

28. Which of the following figures represents the 33. Manager, Labour Union and Worker
relation between Currency, Rupee and SSC (10+2) 2014
Dollar? UCO Bank (Clerk) 2011

(a) (b) (c) (a) (b) (c) (d)

34. Food, Curd, Spoons SSC (CPO) 2014

(d) (e)

29. Which one of the following diagram best


depicts the relationship among Elephants, (a) (b) (c) (d)
Wolves and Animals? SSC (Steno) 2012
35. Profit, Dividend, Bonus BSSC (CGL) 2015

A
M
(a) (b) (c) (d)
(a) (b)

H
30. Which of the following represents Granite, (c) (d)

TT
Tree and Water. CGPSC 2013
36. Choose from the 4 diagrams given below,
the one that illustrates the relationship
(a) (b)
A
among Languages, Japanese, German.
W
SSC (CPO) 2016
SH

(c) (d)
A
@

(e) (a) (b) (c) (d)

37. Identify the diagram that best represents the


31. Identify the diagram that best represents the relationship among classes given below.
relationship among the given classes. Sportsmen, Cricketers, Batsmen
Society, Friends, Enemies SSC (10+2) 2017 SSC (Multitasking) 2014

(a) (b)

(a) (b) (c) (d)


(c) (d)
Directions (Q. Nos. 32-35) In the following
questions, which one of the following diagrams
represents the correct relationship among the 38. Which figure will best represent the
given elements. relationship amongst the three classes Doctor,
Teacher, Women? SSC (CPO) 2013; AFCAT 2018
32. Travelers, Train, Bus SSC (CGL) 2014
(a) (b)

(c) (d)
(a) (b) (c) (d)
216 How to Crack Test of Reasoning ANALYTICAL

39. Which figure best represents the relationship 44. Which of the following diagrams indicates
between Editor, Newspaper and Journalist? the best relation between Singers, Athletes
SSC (CPO) 2015 and Girls? AFCAT 2018

(a) (b)

(a) (b) (c) (d)

40. Which of the following figures best depicts (c) (d)


the relationship among criminals, thieves
and judges? IB (ACIO) 2017

45. Which of the following diagrams indicates


C T J
the best relation between Clothes, Flowers
and White? AFCAT 2018

(a) (b)
(a) (b) (c) (d)

A
M
41. Which diagram best describes the (c) (d)

H
relationship between Human beings,

TT
Daughters, Mothers? DSSSB 2017

(a) (b)
A
46. Given below are three figures which
represent graduates, post graduates, officers.
W
Which part represents all the officers who
SH

(d)
are graduates and post graduates?
(c)
SSC (Multitasking) 2013
A

Post graduates
Graduates
@

B D A
42. Which of the following diagrams indicate the
best relation among Sports, Football, Lizard?. G
E F
UP Police (Constable) 2018
C Officers
(a) (b)
(a) G (b) D (c) B (d) C
47. Point out the letter which represents Indians
and Historians, but not politicians on the
(c) (d)
basis of these three circles. WBPSC 2018
Indian
a
43. Which of the following diagrams indicate the
best relation among Pen, Pencil, Paper? b c
UP Police (Constable) 2018 d
Historian Politician
f g e

(a) (b)
(a) b
(b) f
(c) b and f
(c) (d) (d) b and g
Chapter 19 LOGICAL VENN DIAGRAMS 217

48. Study the following Venn diagram and find


the region representing persons who are F H
educated and employed but not confirmed. G E
SSC (CGL) 2016
D
Employed C
B B A
Educated A E F C
D (a) B, C, D (b) E (c) G, F (d) G, F, H, A
Confirmed in job
50. In the following figure given numbers
(a) A, C indicate Indian, leader and singer. Study the
(b) A, B, C figure carefully. Which region among them
(c) B, D denotes Indian leaders who are not singers?
(d) A, BSSC (CGL) 2015

49. In the given figure, square represents 1 2 6


lawyers, triangle represents cyclists, circle Indian Leader
3
represents men and rectangle represents 4 5
post-graduates. Which set of letters 7 Singers

A
represents men who are not cyclists?

M
(a) 2 (b) 3 (c) 4 (d) 5

H
TT
Expert Level Exercise
1. In the given figure, circle P represents hard A
2. In the following figure, rectangle represents
W
working people, circle Q represents Mechanics, circle represents Artists, triangle
SH

intelligent people, circle R represents truthful represents Gamers and square represents
people and circle S represents honest people. Golfers. Which set of letters represents
Which region represents the people who are Mechanics who are either Artists or Gamers?
A

intelligent, honest and truthful but not hard SSC (10+2) 2017
@

working? UPSC (CSAT) 2013


B

P Q C
10 11 1
F
9 7 2 D E
8 6 I G
H
4 5 3 A
R S

(a) 6 (a) DEIGH


(b) 7 (b) DEIG
(c) 8 (c) EGH
(d) 11 (d) IGH
218 How to Crack Test of Reasoning ANALYTICAL

3. In the following figure, rectangle represents 6. In the given diagram, circle represents
Paramedics, circle represents Equestrians, professionals, square represents dancers,
triangle represents Photographers and triangle represents musicians and rectangle
square represents Fathers. Which set of represents europeans. Different region in the
letters represents Paramedics who are diagram are numbered 1 to 11. Who among the
photographers? SSC (10+2) 2017 following is neither a dancer nor a musician but
is professional and not a European?
SSC (CGL) 2013
D
E 1
F
A B 2 3
11
4 5 10
G H
6 7 9
C 8

(a) F (b) E (a) 8 (b) 11


(c) 1 (d) 10
(c) G (d) H
4. In the following figure, the boys who are 7. In the given figure, 10% are students

A
and parents, another 10% are students,
cricketer and sober are indicated by which
teachers and parents, 15% are teachers

M
number? SSC (CGL) 2013
Cricketer and parents. 35% are students and teachers.

H
How many percentage are only teachers,

TT
10 Sober parents and students? SSC (CGL) 2015
7
6
9 A
Students 35% Teachers
W
2 Boys
4 10%
SH

Girls 1
8 10% 15%
(a) 6 (b) 5 (c) 4 (d) 2
A

Parents
5. In the following diagram, police officer
@

represents circle, corrupt represents triangle,


(a) 40, 65, 45
poet represents square and married
(b) 65, 40, 45
represents rectangle.
(c) 45, 40, 65
(d) 40, 45, 65
11
Directions (Q. Nos. 8-11) Read the following
information carefully and answer the questions
1 10 based on them. CGPSC 2018
5
2 6
3 4 10 7
12
12 13 8 6 13
9 8 7 4

9 11 14
5
The area representing unmarried
— Rectangle represents males
police officer who are not corrupt but are
poets is SSC (Multitasking) 2013 — Triangle represents educated.
(a) 8 (b) 9 — Circle represents urban
(c) 2 (d) 4 — Square represents civil servants
Chapter 19 LOGICAL VENN DIAGRAMS 219

8. How many among the following is an 13. Which number represents the uneducated
educated male who are not the urban women, who have government jobs as well
resident? as jobs in private sectors?
(a) 11 (b) 9 (c) 5 (a) 6 (b) 4 (c) 12
(d) 4 (e) None of these (d) 9 (e) None of these
9. How many among the following is a female, 14. Number 10 represents
urban resident and also a civil servant? (a) Educated women in private job
(a) 6 (b) 7 (c) 10 (b) Uneducated men in government job
(d) 13 (e) None of these (c) Educated men working in private sector
10. How many among the following is (d) Educated men having private as well as
uneducated and also an urban male? government job
(a) 12 (b) 11 (c) 3 (e) None of the above
(d) 2 (e) None of these
Directions (Q. Nos. 15-16) The following figure
11. How many among the following are represents a group of persons. The triangle
educated male who hails from urban area? represents educated persons, the rectangle
(a) 2 (b) 4 represents the administrative persons, the square
(c) 5 (d) 11 represents the businessmen and circle represents

A
(e) None of these income tax payers. CGPSC 2013

M
Directions (Q. Nos. 12-14) Read the following

H
information carefully and answer the questions

TT
based on them.
In the Venn diagram given below, the square
represents women, the triangle represents A
W
persons who are in government services, the
circle represents educated persons and the
SH

rectangle represents persons working in


private sectors. Each section of the diagram is 15. From the above figure, it can be concluded that
A

numbered. Your task is to study the diagram (a) All businessmen pay income tax.
@

(b) Some administrative persons are income tax


and answer the questions that follow.
payers.
Educated Women (c) All income tax payers are educated.
persons Persons in (d) Educated persons which are in administrative
1 government
2 4 service service do not pay income tax.
7 (e) All businessmen are educated.
13
3
16. According to the given figure, it can be
5 6 9
10 12 inferred that
8
(a) None of the income tax payer is educated
11 (b) All the administrative persons are income tax
Persons in private sector payers
12. Which number represents educated women, (c) None of the income tax payer is administrative
who are in government jobs only? person
(a) 2 (b) 3 (d) Some of the persons who are neither
(c) 6 (d) 4 businessman nor income tax payers are
(e) None of these educated person
(e) None of the above
220 How to Crack Test of Reasoning ANALYTICAL

Answers
Base Level Exercise
1. (a) 2. (e) 3. (b) 4. (e) 5. (d) 6. (a) 7. (e) 8. (b) 9. (b) 10. (b)
11. (b) 12. (d) 13. (a) 14. (a) 15. (d) 16. (e) 17. (c) 18. (b) 19. (d) 20. (e)
21. (d) 22. (a) 23. (c) 24. (d) 25. (a) 26. (d) 27. (b) 28. (b) 29. (a) 30. (c)
31. (b) 32. (a) 33. (a) 34. (d) 35. (d) 36. (a) 37. (c) 38. (b) 39. (d) 40. (b)
41. (d) 42. (a) 43. (a) 44. (c) 45. (b) 46. (a) 47. (a) 48. (c) 49. (a) 50. (a)

Expert Level Exercise


1. (a) 2. (a) 3. (d) 4. (d) 5. (a) 6. (d) 7. (a) 8. (a) 9. (c) 10. (a)
11. (b) 12. (b) 13. (d) 14. (d) 15. (b) 16. (d)

Answer with Explanations

A
M
Base Level Exercise

H
TT
1. (a) According to the question, 4. (e) According to the question,
Metal
Year
Week A Iron
W
Day Chlorine
SH

Iron is a metal but chlorine is a liquid non-metal,


Year comprises of weeks and week comprises so it is totally different.
A

of days.
@

5. (d) According to the question,


2. (e) According to the question,
Mammals
Cows Words with Words with
no vowels Words with one vowel
two vowels
Crows
All are different in nature.
Cows comes under the class of mammals but 6. (a) According to the question,
crows does not belong to the class of mammals.
School
3. (b) According to the question,
Classroom
House
Blackboard
Bedroom Bathroom

Bedroom and bathroom both are parts of a Blackboard is found in a classroom and a
house but they are entirely different. classroom is found in a school.
Chapter 19 LOGICAL VENN DIAGRAMS 221

7. (e) According to the question, 12. (d) According to the question,

Planets
Iron Lead Nitrogen
Earth

Sun
All are different entities.
13. (a) According to the question,
Earth is a planet, but Sun is a star.
8. (b) According to the question, Cabinet
Minister
Room Home Minister
Window

Home minister is a minister and minister


belongs to cabinet.
Wall 14. (a) According to the question,

A
Wall and window both are entirely different, but
both are parts of a room. Homosapiens

M
Woman
9. (b) According to the question,

H
Mother

TT
Grains

Maize A
A mother is a woman and a woman is a
W
homosapien.
SH

15. (d) According to the question,


Wheat
A

Maize and wheat both are different entities, but


both are grains.
@

10. (b) According to the question, Teachers

Fruits

Plums Tomato Musicians Players

Some musicians are teachers, some teachers


Plum is a fruit and tomato is a red berry, type are players, but no musician is a player.
fruit. Both are totally different friuts.
16. (e) According to the question,
11. (b) According to the question,
Perennial source
Living beings of water

Rodents Canals
Rivers

Men
Men and rodents are different entities, but both Rivers and canals are different, but both are
are living beings. perennial source of water.
222 How to Crack Test of Reasoning ANALYTICAL

17. (c) According to the question, 22. (a) According to the question,
Ornaments

Rings
Diamond rings Truck Goods Ship

Some goods are transported by ships and


some by truck.
All diamond rings are rings and all rings are
23. (c) According to the question,
ornaments.
18. (b) According to the question,
Woman Landlords Literate

Married persons Some landlords are literate.


24. (d) According to the question,
Liquids
Wives who work

A
Soda water
Note Wives means woman and married only.

M
Mineral water

H
19. (d) According to the question, Soda water and mineral water both are liquids.

TT
25. (a) According to the question,

Employed
A
W
Brothers Doctors
SH

Computer
skills Graduates Males
A

Some graduates are employed and some have All brothers are males. Some doctors are males
@

computer skills. And some employed persons and brothers.


are graduates and some have computer skills 26. (d) According to the question,
and, some graduates are employed and have
computer skills. Fruit Red Shirt
20. (e) According to the question,
Some fruits are red and also some shirts can be red.
Students 27. (b) According to the question,
Tree
First divisioners

Third divisioners Branches Root


Some students are first divisioners and some Both branch and root are the parts of a tree but
are third divisioners. both are not related to each other.
21. (d) According to the question, 28. (b) According to the question,
Carnivores
Dollar Currency

Lion Goat
Rupee

Dollar and rupee both are currencies of different


Lion is carnivores but goat is not. countries.
Chapter 19 LOGICAL VENN DIAGRAMS 223

29. (a) According to the question, 36. (a) According to the question,
Languages

Wolves Elephants Japanese


German
Animals
Wolves and elephants both are animals, but German and Japanese, both are different
both are of different category. languages.
30. (c) According to the question, 37. (c) According to the question,
Cricketers
Granite Tree
Batsmen
Sportsmen
Water

All the batsmen are cricketers and all the


All are different entities. cricketers are sportsmen.
31. (b) According to the question, 38. (b) According to the question,
Society

A
Enemies Friends Doctor Teacher

M
H
Friend and Enemy both are present in society. Women

TT
32. (a) According to the question,

A
Some women may be doctor and teacher both.
And some doctors may be women and teachers
W
Train Bus
both.
SH

39. (d) According to the question,


Travelers
Some travelers travel by bus and some by train.
A

Newspaper
33. (a) According to the question,
@

Editor Journalist
Manager Worker
Some editors may be journalist and vice-versa.
Labour union But newspaper is different.
Worker is a part of labour union. But manager is 40. (b) According to the question,
different.
34. (d) According to the question, T J
C
Curd Spoon All thieves are criminals but judges are different.
Food 41. (d) According to the question,
Curd is a type of food, but spoon is a cutlery.
35. (d) According to the question,
Mothers

Profit
Daughters
Dividend
Bonus Human beings

All mothers are daughters and all daughters are


Both dividend and bonus are the parts of profit. human beings.
224 How to Crack Test of Reasoning ANALYTICAL

42. (a) According to the question, 45. (b) According to the question,

Football Flowers White Clothes


Lizard
Sports .
Some flowers are white and some clothes are
Football is a kind of sport, but Lizard is different. white. But flowers and clothes are different.
43. (a) According to the question, 46. (a) G represents officers, who are graduates and
post graduates.
Pen Pencil Paper
47. (a) Letter ‘b’ represents Indians and Historians
Pen, Pencil and Paper all are different from one but not politicians.
another. 48. (c) In the diagram letters B and D represent the
44. (c) According to the question, region of the persons who are educated and
employed, but not confirmed.

Singers Athletes 49. (a) Set of letters B, C and D represents men who
are not cyclists.
Girls 50. (a) Indian leaders who are not singers is denoted

A
by the region 2.

M
Some Girls are singers and Athletes.

H
TT
Expert Level Exercise
1. (a) Region 6 represents the people who are intelligent,
A
10. (a) 12 persons are uneducated and also and
W
honest and truthful but not hard working. urban male.
2. (a) From the figure, it is clear that DEIGH are sets of 11. (b) 4 persons are educated male who hails from
SH

letters which represents Mechanics who are either urban area.


Artists or Games. 12. (b) That number should occupy the space
A

3. (d) That letter should occupy the space common to common to circle, square and triangle. 3 is
that number.
@

triangle and rectangle. ‘H’ is that letter.


13. (d) That number should occupy the space which
4. (d) Number 2 represents the boys who are cricketer
is common to square, triangle and rectangle.
and sober. 9 is that number.
5. (a) The area which represents the unmarried police 14. (d) Number 10 represents educated men having
officers who are not corrupt but poet is 8. private as well as government jobs.
6. (d) Region number 10 represents the person who is Solutions (Q. Nos. 15-16) According to the question,
neither a dancer nor a musician but is professional Educated Persons
and not European.
For teacher = 100 − (35 + 10 + 15)
Administrative

7. (a) 4 Businessmen
persons

= 100 − 60 = 40% 5
For parents = 100 − (10 + 10 + 15) 3
1
= (100 − 35) = 65% 2
For students = 100 − (35 + 10 + 10)
= 100 − 55 = 45% Income tax payers
The percentage for only teachers, parents and 15. (b) From the figure, it is clear that some
students is 40, 65 and 45, respectively. administrative persons are income tax payers
8. (a) There are 11 educated male who are not urban and part 1 shows it.
resident. 16. (d) Some of the persons who are neither
9. (c) 10 females are urban resident and also a civil businessman nor income tax payers are
educated person and parts 2, 3, 4 and 5
servant.
show it.
Chapter 20 ELIGIBILITY TEST
225

20
Eligibility Test
Eligibility test is a process of checking a In case of a candidate, who fulfils all other
candidate’s qualifications and necessary criteria except
conditions to fulfil the given criteria for (A) at (iv) but has secured more than 72%
job/promotion/admission in college etc. marks in preliminary selection

A
In these type of questions, a set of necessary examination, his/her case is to be

M
qualifications and conditions required to be referred to Deputy General Manager.

H
fulfilled by the candidate for certain vacancies in (B) at (ii) but has secured atleast 65% marks

TT
job, promotion, etc. along with the bio-data of in post-graduation, his/her case is to be
the candidates who have applied for the same referred to General Manager.
posts. You are required to match personal data of A
In each of the questions given below. You
W
a person with the eligibility requirements. have to study the information provided
Following examples will illustrate the type of with reference to the conditions given
SH

questions, we may come across. above and decide whether the candidate
is to be called for an interview or some
A

Directions (Illustrations 1-5) Study the following


other course of action is to be taken.
@

information carefully and answer the questions given


below. You are not required to assume anything
other than the information provided to
Following are some conditions for short-listing
you in each question. All these cases are
candidates for the interview for management
given to you as on 1st Nov, 2017.
trainees in an organisation.
Now, read the information provided in
The candidate must each question and decide the courses of
(i) not less than 21 yr and more than 28 yr as on action with regard to each candidate.
1st Nov, 2017.
Give Answer
(ii) have secured atleast 60% marks in graduation. (a) if the candidate is to be called for an interview
(iii) have secured atleast 65% marks in the (b) if the case is to be referred to General Manager
preliminary selection examinations. (c) if the candidate will not be called for an
(iv) have secured atleast 55% marks in the final interview
selection examinations. (d) if the data provided are not sufficient to take a
decision
(v) be ready to join work immediately after the
(e) if the case is to be referred to Deputy General
interview.
Manager
How to Crack Test of Reasoning ANALYTICAL
226

Illustration 1. Neelam Srivastava has secured ready to join immediately after the interview.
75% marks in the preliminary selection She stood 3rd in the final selection
examination. She was 22 yr old as on 5th Dec, examination. She was born on 2nd July, 1994.
2013. She has secured 65% and 60% marks in Solutions (Illustrations 1-5) All the informations can
the final selection examination and in be summarised as given in the table
graduation, respectively. She is ready to join
Name Conditions fulfilled
immediately after the interview.
(i) (ii) (B) (iii) (iv) (A) (v)
Illustration 2. John D’costa has secured 54% Neelam P P — P P — P
marks in the final selection examination. He
has secured 75% marks each in graduation John D’costa P P — P O P P
and preliminary examination. He is ready to Subhash O P — P P — P
join work immediately after the interview. He Rahul P P — P P — P
was 22 yr old as on 4th Aug, 2014. Sudha P P — P — P
Illustration 3. Subhash Malhotra was 26 yr old as 1. (a) Neelam Srivastava satisfies all the conditions.
on 9th Sep, 2015. He has secured 75% marks Therefore, she may be called for the interview.
each in graduation, preliminary and final 2. (e) John D’costa satisfies all the conditions except

A
selection examination. He is ready to join (iv) but he fulfils the sub-condition (A). So, his

M
work immediately after the interview. case is to be referred to the Deputy General
Manager.

H
Illustration 4. Rahul Biswas was born on 21st Jan,

TT
3. (c) Subhash Malhotra does not satisfy condition (i).
1990. He is ready to join work immediately Hence, he should not be called for the
after the interview. He has secured 70% marks interview.
in all the graduation, preliminary and final A
W
4. (a) Rahul Biswas satisfies all the conditions.
selection examination. Hence, he should be called for the interview.
SH

Illustration 5. Sudha Nagpal has secured 75% and 5. (d) Sudha Nagpal’s final selection marks are not
65% marks in the graduation and preliminary mentioned in the given question. So, data
A

selection examination respectively. She is provided is not sufficient to take a decision.


@

Let us Practice
Directions (Q. Nos. 1-5) Study the following (iv) have secured atleast 50% marks in the
information carefully and answer the questions selection process.
given below. IBPS (Clerk) 2011 In the case of a candidate who fulfils all the
Following are the conditions for selecting conditions except
Accounts Officer in an organisation. (A) at (i) above, but atleast 21 yr old and not more
The candidate must than 28 yr old and has work experience of 5 yr
(i) be atleast 21 yr and not more than 26 yr as Accounts Assistant in an organisation,
as on 01.11.2011. his/her case is to be referred to GM-Accounts.
(ii) be a Commerce graduate (B.Com.) with (B) at (ii) above, but has secured atleast 50%
atleast 55% aggregate marks. aggregate marks in graduation and has
secured atleast 55% marks in the selection
(iii) have work experience of atleast 2 yr in
process, his/her case is to the referred to
the Accounts department of an
VP-Accounts.
organisation.
Chapter 20 ELIGIBILITY TEST
227

In each question below, details of one Directions (Q. Nos. 6-10) Read the following
candidate are provided. You have to take information carefully and answer the questions
one of the following courses of actions based given below. IDBI (Office Executive) 2018
on the conditions given above and the A company decided to appoint Content Manager
information provided in each question and (CM) to give a new impetus to its business.
mark the number of that course of action as Following are the criteria laid down by the
your answer. You are not required to company.
assume anything other than the The candidate must have
information provided in each question. All
(i) a Graduate degree from a recognized
these cases are given to you as on university with at least 65% marks.
01.11.2011.
(ii) qualified in at least 5 written examinations
Give Answer of Bank PO.
(a) if the case is to be referred to GM-Accounts
(b) if the case is to be referred to VP-Accounts (iii) obtained at least 60% marks in the written
(c) if the candidate is to be selected test (total marks = 200) conducted by the
(d) if the candidate is not to be selected company.
(e) if the data provided are inadequate to take a (iv) obtained at least 40% marks out of 75 marks
decision in the interview for the above post conducted

A
Now, read the information provided in each by the company.

M
question and mark your answer accordingly. (v) a working knowledge of computers.

H
1. Umesh Choksi was born on 25th November, (vi) completed 30 yr of age as on 14th January,

TT
1989. He has secured 60% aggregate marks 2012.
in B. Com. and 65% marks in the selection However, in case of a candidate who fulfils all
process. He has been working in the
Accounts department of an organisation for A
these criteria except
W
the past 3 yr. (A) (i) above, but is a graduate, is to be referred
to the Director of the company.
SH

2. Pratibha Kale was born on 6th June, 1988. (B) (iii) above, is to be referred to the Assistant
She has secured 60% aggregate marks in Vice President of the company.
A

B.Com. and 49% marks in the selection


Based on the above criteria and the
@

process. She has been working in the


information given in each of the following
Accounts department of an organisation for
cases, you have to take decision. You are not
the past 3 yr.
required to assume anything. In case you
3. Arun Patil has secured 55% aggregate find that the given data is insufficient to
marks in graduation. He has been working make a decision, given ‘data inadequate’ as
for past 4 yr in the Accounts department of your answer. The cases are being given to
an organisation. He has secured 50% marks you as on 14.01.2012.
in the selection process. He was born on Give Answer
12th July, 1988. (a) if the candidate is to be selected as CM
(b) if the candidate is not to be selected as CM
4. Parbha Dixit was born on 18th April, 1985.
(c) if the case is to be referred to the Director of the
She has been working as Accounts Assistant company
in an organisation for the past 5 yr. She has (d) if the case is to be referred to the Assistant Vice
secured 60% aggregate marks in B.Com. and President of the company
55% marks in the selection process. (e) if the data are inadequate to take any decision
5. Amul Verma has secured 50% aggregate 6. Surbhi Gaurav, was born on 12th January,
marks in B.Com. and 60% marks in the 1981, has done her Graduation with 67%
selection process. He has been working in the marks. It was the fifth interview for her when
Accounts department of an organisation for she appeared before the interview board
past 4 yr. He was born on 2nd January, 1987. constituted for Union Bank of India. She has a
working knowledge of computers. She got 121
How to Crack Test of Reasoning ANALYTICAL
228

marks and 38 marks in the written However, if a candidate fulfils all the above
examination and interview respectively, mentioned criteria except
conducted by the company.
(A) at (i) but has appeared to the last semester
7. Vineet Pehal has a working knowledge of examination and has obtained an aggregate
computers. He got 125 marks and 32 marks of minimum 65% marks in first seven
in the written examination and interview semesters his/her case may be referred to
respectively, conducted by the company. VP of the company.
When he appeared before the interview (B) at (iv) but is willing to pay an amount
board for PO for Bank of India, it was his atleast ` 25000 and has obtained atleast
sixth interview for Bank PO. He performed a 70% marks in Engineering degree; his case
good dance on 10th January, 2012, on the eve
may be referred to the General Manager of
of the birthday of his sister, who is 8 yr
the company.
younger than him.
In each of the following question, details of a
8. Malik Aanand a talented working computer candidate are given as regards his/her
engineer, is 31 yr of age. He has already candidature. You have to read the
qualified six exams for Bank POs. He is a information provided and decide his/her
Graduate with 65% marks. He got 60% marks status based on the conditions given above
in the interview and 40% marks in the written

A
and the information provided. You are not
test conducted by the company for CM. required to assume anything other than the

M
9. Shobha Vijay has a working knowledge of information provided in each of the

H
computers. She is a Graduate with 65% following questions. All these cases are

TT
marks. She has qualified five written given to you as on 01.05.2017.
examinations of Bank PO. She has obtained Give Answer
40% marks and 60% marks in interview and (a) A
if the candidate is to be selected
W
written examinations respectively, (b) if the case is to be referred to General Manager
conducted by the company for CM. For the (c)if the case is to be refered to the VP
SH

last three years her age has been more (d) if the data provided are not adequate to take a
than 18. decision
A

10. Alankrita Mangal is Graduate with 65% (e) if the candidate is not to be selected
@

marks. She has obtained 30 marks in the 11. Sachin who has just completed 23 yr age
interview and 123 marks in the written passed out degree in Civil Engineering with
examination conducted by the company, for 70% marks. He has cleared the selection test
CM. She is 32 yr old and possesses working with 61% marks. He is willing to pay the
knowledge of computers. She has given amount of ` 25000 only will not be able to
more than five interview of Bank PO exam. pay ` 50000.
Directions (Q. Nos. 11-15) Study the following 12. Anjali is an IT Engineer passed out in 2010
information carefully to answer these questions. with 52% marks. After getting the
Following are the conditions stipulated by XYZ Engineering degree, she has done MBA with
Companies for recruitment of Trainees specialisation in finance. She has cleared the
Engineers. selection test with 60% marks. She can pay
the deposit of ` 50000. Her date of birth is
The candidate must 15th Sep, 1993.
(i) be an Engineering graduate with atleast
13. Mohan Rao is an Electronics Engineer and
60% marks.
passed out in 2015 with 66% marks. He has
(ii) be not less than 21 yr and not more than passed the selection test with 59% marks and
25 yr of age as on 01.05.2017. is willing to pay the required amount of
(iii) have passed the selection test with atleast deposit.
55% marks. 14. K. Shiv Kumar is a student of Mechanical
(iv) be willing to pay a deposit of ` 50000 to be Engineering and has appeared for the last
refunded on completion of training. semester examination. Results of the last
Chapter 20 ELIGIBILITY TEST
229

semester examinations are expected in June answer. You are not required to assume
2017. He is expecting to score 65% marks in anything other than the information
the last semester as his aggregate percentage provided in each question. All these cases
of the first seven semesters is 67%. He has are given to you as on 01.01.2010.
passed the selection test with 60% marks and
Give Answer
has no problem in paying the amount of (a) if the case is to be referred to Executive Director
` 50000 as deposit. He is 22 yr old at present.
(b) if the case is to be referred to GM-Accounts
15. Rajeev Andhare has appeared for the last (c) if the data provided are not adequate to take a
semester of Engineering degree examination decision
and is hoping to score atleast 70% marks. (d) if the candidate is to be selected
His aggregate score upto first seven (e) if the candidate is not to be selected
semesters is 72%. He has just completed
16. Prashant Mishra has secured 60% in B. Com.
21 yr of age. He has scored 63% marks in the
and 65% marks in M.Com. He has been
selection test and is ready to pay an amount
working in the Accounts department of an
of ` 50000 as deposit.
organisation for the past 7 yr after
Directions (Q. Nos. 16-25) Study the following completing his M.Com. He has secured 50%
information carefully and answer the questions marks in personal interview. His date of

A
given below. IDBI Bank (PO) 2014 birth is 15.09.1984.

M
Following are the conditions for selecting 17. Samir Malhotra was born on 25th July 1982. He
Manager Accounts in an organisation. has been working in the Accounts department

H
The candidate must of an organisation for the past 6 yr after

TT
obtaining his M.Com. degree with 58% marks.
(i) be atleast 25 yr and not more than 35 yr as He has secured 70% marks in B.Com. and 60%
on 01.01.2010.
A
marks in personal interview.
W
(ii) be a graduate in Commerce with atleast
55% marks. 18. Sudha Agarwal was born on 5th January,
SH

1978. She has been working in the Accounts


(iii) be a post graduate in Commerce with
department of an organisation for the past
atleast 60% marks.
7 yr after obtaining her MBA in Finance with
A

(iv) have post qualification work experience of 70% marks. She has secured 68% marks in
@

atleast 6 yr in the Accounts Department of B.Com. and 52% marks in personal interview.
an organisation.
(v) have secured atleast 45% marks in the 19. Arun Ramnathan has secured 62% marks in
personal interview. M.Com. and 58% marks in B.Com. He has
been working in an organisation for the past 6
In the case of a candidate who satisfies all yr after completing his M.Com. He has
the conditions except secured 46% marks in the personal interview.
(A) at (iii) above, but is an MBA-Finance with His date of birth is 20th May, 1981.
atleast 65% marks, the case is to be referred
20. Mohan Das was born on 8th February, 1980.
to GM-Accounts.
He has been working for the past
(B) at (iv) above, but is a CA/ICWA and has 2 yr in an organisation after completing his
work experience of atleast 1 yr in an CA. He has secured 60% marks in both
organisation, the case is to be referred to B.Com. and M.Com. He has secured 50%
Executive Director. marks in personal interview.
In each question below, details of one 21. Atul Ghosh has secured 65% marks in
candidate are provided. You have to take B.Com. and 65% marks in M.Com. He has
one of the following courses of actions based been working for the past 8 yr in the
on the information provided and the Accounts department of an organisation
conditions and subconditions and mark the after completing his M.Com. He was born on
number of that course of action as your 12th march, 1981.
How to Crack Test of Reasoning ANALYTICAL
230

22. Seema Jaiswal was born on 19th January, In the case of a candidate who satisfies all the
1978. She has secured 62% marks in both above conditions except
B.Com. and M.Com. She has been (A) at (ii) above, but has secured atleast 55%
working in the Accounts department of
marks in graduation in any discipline and
an organisation for the past 6 yr after
atleast 70% marks in post-graduate
completing her M.Com. She has secured
degree/diploma in Personnel
48% marks in personal interview.
Management/HR. The case
23. Navin Gosh has secured 68% marks in is to be referred to GM-HR.
B.Com. and 57% marks in M.Com. He has
(B) at (iv) above, but has post qualification work
been working in the Accounts department
of an organisation for the past 7 yr after experience of atleast 4 yr out of which atleast
completing his M.Com. He has secured 2 yr as Deputy Manager-HR, the case is to be
47% marks in the personal interview. referred to President-HR.
In each question given below, details of one
24. Kapil Sonawane was born on 4th
candidate are given. You have to take one of
November, 1976. He has been working for
the following courses of actions based on the
the past 1 yr in an organisation after
completing his ICWA. He has secured 65% information provided and the conditions and
subconditions given above and mark the

A
marks in both B.Com. and M.Com. He has
number of that course of action as your

M
secured 60% marks in personal interview.
answer.

H
25. Sonam Khanna was born on 28th
You are not required to assume anything other
December, 1979. She has secured 62%

TT
marks in M.Com. She has been working than the information provided in each
for the past 8 yr in the Accounts question. All these cases are given to you as on
department of an organisation after 01.03.2012. A
W
completing her MBA- Finance with 75% Give Answer
SH

marks. She has secured 54% marks in (a) if the candidate is not to be selected
B.Com. She has secured 60% marks in (b) if the data provided are not adequate to take a
personal interview. decision
A

(c) if the case is to be referred to President-HR


@

Directions (Q. Nos. 26-29) Study the (d) if the case is to be referred to GM-HR
following information carefully and answer the (e) if the candidate is to be selected
questions given below. IBPS (PO) 2012
26. Rita Bhatt was born on July 25, 1978. She has
Following are the conditions for selecting secured 62% marks in graduation and 65%
Manager-HR in an organisation. marks in post-graduate diploma in personnel
The candidate must Management. She has been working for the
past 6 yr in the personnel Department of an
(i) be atleast 30 yr and not more than 35 yr
organisation after completing her post
as on 01.03.2012.
graduation. She has secured 55% marks in the
(ii) have secured atleast 60% marks in selection process.
graduation in any discipline.
(iii) have secured atleast 65% marks in the 27. Ashok pradhan was born on August 8, 1980. He
post-graduate degree/diploma in has been working in the Personnel Department
personnel Management/HR. of an organisation for the past 4 yr after
completing his post-graduate degree in
(iv) have post qualification work experience
Personnel Management with 67% marks. Out of
of atleast 5 yr in the Personnel/HR
his entire experience, he has been working for
Department of an organisation.
the past 2 yr as Deputy Manager-HR. He has
(v) have secured atleast 50% marks in the secured 62% marks in graduation and 58%
selection process.
marks in the selection process.
Chapter 20 ELIGIBILITY TEST
231

28. Alok Verma was born on March 4, 1976. He has (ii) have secured atleast 40% marks in the selection
been working in the Personnel Department of interview.
an organisation for the past 6 yr after (iii) have post qualification work experience of
atleast five years in the marketing division of
completing his post-graduate diploma in
an organisation.
Personnel Management with 66% marks. He
(iv) have secured atleast 45% marks in the selection
has secured 57% marks in the selection process examination.
and 63% marks in graduation. (v) have a post Graduate degree/diploma in
29. Swapan Ghosh has been working in the Marketing-Managment with atleast 60%
Personnel Department of an organisation for the marks.
past 5 yr after completing his post graduate Study the following information carefully
degree in HR with 72% marks. and find which of the following condition
He has secured 56% marks in graduation. He show candidate is not selected?
was born on May 12, 1977. He has secured 58% (a) Candidate is daughter of a renowned freedom
marks in the selection process. fighter from another state SBI (PO) 2017
30. Study the following information carefully and (b) Candidate has a post Graduate degree in
answer the question given below. Following Finance with 60% marks
(c) Candidate has completed his graduation with
are the conditions for selecting Marketing
80% marks.
manager in an organisation.

A
(d) Candidate does not own a house in Noida

M
The candidate must (e) Candidate has secured 56% marks in cap
(i) be a Graduate in any discipline with atleast 55% Gemini’s interview

H
marks.

TT
Answers A
W
SH

1. (c) 2. (d) 3. (e) 4. (a) 5. (b) 6. (e) 7. (e) 8. (d) 9. (e) 10. (a)
11. (b) 12. (e) 13. (d) 14. (c) 15. (c) 16. (d) 17. (e) 18. (b) 19. (c) 20. (a)
A

21. (c) 22. (d) 23. (e) 24. (a) 25. (e) 26. (e) 27. (c) 28. (a) 29. (d) 30. (b)
@

Answer with Explanations


1. (c) Umesh Choksi 3. (e) Arun Patil
(i) Age (on 01.11.2011)-21 yr and 11 months. (i) Age (on 01.11.2011)-23 yr and 3 months
(ii) Marks in graduation (B.Com.)-60% (ii) Marks in graduation-55%
(iii) Experience in Account department-3 yr (iii) Experience in Account department-4 yr
(iv) Marks in selection process-65% (iv) Marks in selection process-50%
So, Umesh Choksi is to be selected. Here, nothing is given about the
specialisation in graduation. So, the data are
2. (d) Pratibha Kale inadequate to take the decision.
(i) Age (on 01.11.2011)-23 yr and 4 months
4. (a) Parbha Dixit
(ii) Marks in graduation (B.Com.)-60%
(i) Age (on 01.11.2011)-26 yr and 6 months
(iii) Experience in Account department-3 yr
(ii) Marks in graduation (B.Com.)-60%
(iv) Marks in selection process-49%
(iii) Experience in Account department-5 yr
Here, Pratibha Kale does not fulfil the condition (iv) Marks in selection process-55%.
(iv) So, Pratibha Kale is not to be selected. Parbha Dixit fulfils the sub-condition (A) in
place of condition (i). So, the case is to be
referred to GM-Accounts.
How to Crack Test of Reasoning ANALYTICAL
232

5. (b) Amul Verma


(i) Age (on 01.11.2011)-24 yr and 9 months
(ii) Marks in graduation (B.Com.)-50%
(iii) Experience in Account department-4 yr
(iv) Marks in selection process-60%
Amul Verma fulfils the sub-condition (B) in place of condition (ii). So, the case is to be referred to
VP-Accounts.
Solutions (Q. Nos. 6-10) All the information can be summarised as in the given table

Candidate Conditions fulfilled


(i) or (A) (ii) (iii) or (B) (iv) (v) (vi)
Surbhi Gaurav P – – P – P P P
Vineet Pehal – – P P – P P –
Malik Aanand P – P O P P P P
Shobha Vijay P – P P – P P –
Alankrita Mangal P – P P – P P P

A
M
6. (e) Surbhi Gaurav has appeared in five interviews, but it is not given that all the interviews were for Bank PO or any
other competitive exam. So, data is insufficient to take the decision.

H
7. (e) Age of Vineet Pehal is not clear from the given information and no information is given about his education. So,

TT
the data is insufficient to take the decision.
8. (d) Malik Aanand did not get 60% marks in the written exam conducted by the company. So, his case is to be
referred to the Assistant Vice President of the company. A
W
9. (e) The age of Shobha Vijay is not clear from the given information. So, data is insufficent to take the decision.
SH

10. (a) Alankrita Mangal satisfies all the given conditions. So, she is to be selected as CM.

Solutions (Q. Nos. 11-15) All the information can be summarised as in the given table
A
@

Conditions fulfilled
Name
(i) A (ii) (iii) (iv) B
Sachin P — P P O P
Anjali O — P P P —
Mohan Rao P — — P P —
K. Shiv Kumar O P P P P —
Rajeev Andhare O P P P P —

11. (b) Sachin’s case will be referred to the General Manager of the company, because he fulfils sub-condition (B) in
place of condition (iv).
12. (e) Anjali fulfils all the criteria except (i). Hence, she is not to be selected.
13. (d) The data provided are not adequate to take a decision.
14. (c) K. Shiv Kumar’s case may be referred to the VP of the company, because he fulfils sub-condition (A) in place of
condition (i).
15. (c) According to sub-condition A, Rajeev Andhare’s case may be referred to the VP of the company.
Chapter 20 ELIGIBILITY TEST
233

Solutions (Q. Nos. 16-25) All the information can be summarised as in the given table
Conditions fulfilled
(i) (ii) (iii) (iii)/(A) (iv) (iv)/(B) (v)
Name Age 25-35 yr Graduate in PG Comm MBA-F Work Exp CA/ICWA+ Interview
on 1-1-10 Comm ≥ 55% ≥ 60% ≥ 65% ≥ 6 yr 1 yr Exp.
45%
Prashant Mishra P P P — P — P
Samir Malhotra P P O — P — P
Sudha Agarwal P P — P P — P
Arun Ramnathan P P P — — — P
Mohan Das P P P — O P P
Atul Ghosh P P P — P — —
Seema Jaiswal P P P — P — P
Navin Ghosh — P O — P — P
Kapil Sonawane P P P — O P P

A
P O P P P P

M
Sonam Khanna —

H
16. (d) Prashant Mishra fulfils all the conditions. 25. (e) Sonam Khanna does not satisfy condition (ii).

TT
Hence, he is to be selected. Hence, she is not to be selected.
17. (e) Samir Malhotra does not satisfy Solutions (Q. Nos. 26-29) All the information can be
condition (iii). Hence, he is not to be A
summarised as in the given table
W
selected.
Conditions fulfilled
SH

18. (b) Sudha Agarwal does not satisfy condition (iii) Candidates
but she satisfies sub-condition (A). Hence, (i) (ii) (A) (iii) (iv) (B) (v)
her case is to be referred to GM-accounts. Rita Bhatt P P — P P — P
A

19. (c) Data provided are not enough to make a Ashok Pradhan P P — P O P P
@

decision, as the department in which Arun Alok Verma O P — P P — P


Ramnathan has been working for the past 6
yr is not given. Swapan Ghosh P O P P P — P
20. (a) Mohan Das does not satisfy condition (iv) but 26. (e) Rita Bhatt fulfils all the conditions.
he satisfies sub-condition (B). Hence, his Hence, she is to be selected.
case is to be referred to executive director.
27. (c) Ashok Pradhan does not satisfy the condition (iv)
21. (c) Data provided are not adequate to make a but he satisfies sub-condition (B). Hence, his case
decision as percentage of marks of personal is to be referred to President- HR.
interview is not specified.
28. (a) Alok Verma does not satisfy condition (i).
22. (d) Seema Jaiswal fulfils all the conditions.
Hence, he is not to be selected.
Hence, she is to be selected.
29. (d) Swapan Ghosh does not satisfy condition (ii) but he
23. (e) Condition (iii) and sub-condition (A), both are satisfies sub-condition (A).
not satisfied. Hence, the candidate is not be Hence, his case is to be referred to GM-HR.
selected.
30. (b) For selection, Candidate must have a
24. (a) Kapil Sonawane does not satisfy condition post-graduate degree/diploma in marketing
(iv) but he satisfies sub-condition (B). management not in finance.
Hence, his case is to be referred to executive
director.
234 How to Crack Test of Reasoning ANALYTICAL

21
Syllogism
Syllogism is a Greek word, which means l
Copula (C) A copula is that part of a
inference or deduction. Syllogism is a proposition which says about the relation
logical argument where a quantified between the subject and the predicate.

A
statement of a specific form (the
conclusion) is inferred from two or more Term

M
quantified statements. To find a logical deduction from two statements

H
In this chapter, questions are based on there should be three terms present. Those three

TT
some statements and their conclusions and terms are
you are required to identify the correct 1. Major term It is the predicate of the
conclusion(s). You are not supposed to A
conclusion and is denoted by ‘P’ (for
W
apply extra information except from the predicate).
SH

information given in statements, while 2. Minor term It is the subject of the conclusion
drawing the conclusion. and is denoted by ‘S’ (for subject).
A

3. Middle term It is the term common to both


Important Terms Used in
@

the statements and is denoted by ‘M’ (for


Syllogism middle).
Note The middle term should not appear in the
Components of Proposition conclusion.
A proposition, also known as a premise, is e.g. Statements
a grammatical sentence which comprises a All ships are buses.
subject, a predicate and a copula. All cars are ships.
e.g. All aeroplanes are trains Conclusion All cars are buses.
↓ ↓ ↓ ↓ Here, ‘buses’ is the predicate of the conclusion.
Quantifier Subject Copula Predicate Therefore, it is major term, P. ‘Cars’ is the subject
l
Quantifier The word ‘All’, ‘No’ and of the conclusion. Therefore, it is the minor term,
‘Some’ are called quantifier as they S. ‘Ships’ is the term common to both the
specify a quantity. statements. Therefore, it is the middle term, M.
l
Subject (S) A subject is that part of a Major and Minor Statements
proposition about which something is
said. The ‘major statement’ is that in which the middle
term is the subject and the ‘minor statement’ is
l
Predicate (P) A predicate is that part of a that in which the middle term is the predicate.
proposition which is affirmed detail
about the subject.
Chapter 21 SYLLOGISM 235

e.g. Statements Types of


Venn Diagram
All dogs are cats. Propositions
All cats are white. Either Some S are P
S p [Some S are not P]
Here, the middle term ‘cats’ is the subject of
second statement.
or S Some P are S
So, it is the major statement and the middle P [All S are P]
term, ‘cats’ is the predicate of first statement. So, I : Some S are P
it is the minor statement. P Some S are not P
or
S [All P are S]

Classification of Propositions Either S p


Some S are not P
[Some S are P]
Propositions have been classified on the basis of
quality and quantity of the propositions. Quality
denotes whether the proposition is affirmative or or P Some S are not P
negative in nature. Quantity denotes whether the O : Some S are not P S [All P are S]
proposition is universal or particular.
or S p Some S are not P
Types of proposition can be understood by the [No S are P]

A
following diagram

M
Proposition
Inference

H
Inference is a logical conclusion derived

TT
Universal Particular
from premises or statements known or
Affirmative Negative Affirmative Negative A
assumed to be true.
W
All A are B No A is B Some A are B Some A are
(A-type) (E-type) (I-type) not B (O-type)
Types of Inferences
SH

There are two types of inferences


The four-fold proposition can be summarised as (conclusions), which are as given below
A

below
@

Symbols Proposition Quantity Quality 1. Immediate Inference


A All A are B Universal Affirmative When an inference is drawn from a single
statement, then it is known as ‘immediate
E No A is B Universal Negative
inference’.
I Some A are B Particular Affirmative e.g. Statement All books are pages.
O Some A are not B Particular Negative Conclusion Some pages are books.
To draw valid inferences from the statements, the In the above example, conclusion is drawn
candidate is required to have a clear from a single statement and does not require
understanding of A, E, I, O relationships. the second statement to be referred. Hence,
the inference is called an immediate
Venn Diagram Representation of inference.
Four Propositions Method to Draw Immediate
Types of
Venn Diagram Inference
Propositions
There are various methods to draw
immediate inferences like conversion,
A : All S are P Always S
P
observation, contraposition, etc., but we are
required to study only following two given
E : No S are P Always S p methods
236 How to Crack Test of Reasoning ANALYTICAL

(i) Implications While drawing a conclusion Conversion Table


through implication, subject remains the Types of Proposition Gets converted into
subject and predicate remains the
predicate. A I
e.g. All boys are good. E E
Here, it is clear that if all boys are good, I I
then some boys will definitely be good
because some is a part of all. O Never gets converted

e.g. No bags are roses.


Here, it is clear that if no bags are roses, 2. Mediate Inference
then certainly some bags are not roses. In mediate inference, conclusion is drawn
In the above cases, subject remains the from two given statements.
subject and predicate remains the e.g. Statements All dogs are cats
predicate.
All cats are black.
Immediate inference through implication
Conclusion All dogs are black.
can be presented as
In the above example, conclusion is drawn

A
Types of Propositions Types of Inferences from the two statements. In other words, both

M
A I the statements are required to draw the

H
conclusion. Hence, the above conclusion is
E O know as mediate inference.

TT
I —
Method to Draw Mediate Inference
O — A
Step I Properly align pair of propositions, i.e.
W
(ii) Conversions In conversions, while the common term must be the
SH

drawing a conclusion, subject and predicate of the first proposition and


predicate of a proposition are the subject of the second.
A

interchanged, but the quality of the Let us consider the following pair of
@

proposition remains the same, i.e. the statements.


affirmative remains affirmative and the I. Some bats are chairs  Non-aligned
negative remains negative. II. Some cats are bats  pair
e.g. I. Some cats are bats 
I. Statement All lamps are mangoes.  Aligned pair
II. Some bats are chairs 
Conclusion Some mangoes are
lamps. Priority order The priority should be
given while aligning. According to the
II. Statement No men are wise. following rule.
Conclusion No wise are men. I E A
III. Statement Some rivers are ponds.
Conclusion Some ponds are rivers. i.e. importance for conversion should
be given first to I-type statement, then
In above cases, subject and predicate change to E-type statement and finally to
their positions but the quality remains same. A-type statement.
Chapter 21 SYLLOGISM 237

Step II After having aligned statements, if immediate inference, a combination I + I does not
required, inference can be drawn produce any valid mediate inference. Hence, no
using the table as given below mediate inference can be drawn.
It is important to note here, that conclusion I,
Types of I Types of II Types of ‘Some cars are buses’ is not valid, because there is
statement statement inference a possibility of ‘No cars are buses’, likewise
A + A = A conclusion II, ‘No cars are buses’ is invalid because
A + E = E there is a possibility of ‘Some cars are buses’. In
other words, both the conclusions are invalid
E + A = O* individually. However, we can say that either of
E + I = O* conclusion I or II is true. Hence, both the
conclusions make a complementary pair of
I + A = I
conclusions.
I + E = O A complementary pair of conclusions must follow
the following the conditions:
The conclusion will follow the pattern in such
I. Both of them must have the same subject and the
a way that subject of the first statement is the
same predicate.
subject of the inference and predicate of
II. They are anyone of three types of pairs
second statement is the predicate of the

A
inference except the cases O* as shown in the (i) I-O type (ii) A-O type (iii) I-E type

M
table. Note Here, we have already discussed the A, E, I

H
Where conclusion will follow the reverse and O types of the statements.

TT
pattern, i.e. subject of the first statement will
be the predicate of inference and predicate of Venn Diagram Representation
the second statement will be the subject of the
A
Statements Some cars are scooters. Some
W
inference. The given table shows the scooters are buses.
combination of statements which produce Here, we can draw all possible cases as given
SH

definite and valid results. All other below.


combinations will not produce definite
A

Scooters
inferences.
@

Buses
Complementary Pair of Conclusions
Scooters

(‘Either or’ Situation) Car Buses Car


In drawing mediate inference from given
statements, students are required to be more (a)
(b)
attentive to select complementary pair of
conclusions, where neither of the conclusions Here, using both diagrammatical represen-
is definitely true but a combination of both tations we can conclude either ‘some cars are
makes a complementary pair. buses’ or ‘No cars are buses’. Hence, atleast
Let us consider the example one of the conclusions must be true.
Statements Possibility Cases in Syllogism and Solving
them using Venn Diagrams
Some cars are scooters.
Some scooters are buses. In syllogism, when there is no exact
conclusion for the given statements, then
Conclusions possibility case arises.
I. Some cars are buses.
If conclusion is false in one diagram and true
II. No cars are buses. in another diagram, then the conclusion is
Solution Both the statements are properly aligned, deduced as partially true. In this case,
therefore fulfil the first requirement but both the partially true is considered as true.
statements are of I-type and as per table for
238 How to Crack Test of Reasoning ANALYTICAL

Statements Some walls are roads. So, it is clear that rivers are not big, so no
All roads are floors. book is big. So, only Conclusion I is true.
Conclusions Rule 2 If one statement is particular, then the
I. All roads being walls is a possibility. conclusion must be particular.
II. All walls being floors is a possibility. e.g. Statements Some men are wise.
Solution All possible diagrams are as follows All wise are doctors.
Conclusions
Walls
I. Some men are doctors.
Floors II. All doctors are wise.
Walls Roads or
Roads Conclusion II cannot follow from the given
statements because if one statement is
Floors
particular, then the conclusion must be
(a) (b) particular.
Floors Therefore, Conclusion I follows.
Alternate Method
Walls
Walls Roads Doctor

A
Roads
Wise

M
Men
Floors

H
(c) (d) Clearly, Conclusion I follows.

TT
Here, the Conclusion I, i.e. ‘All roads being walls is Rule 3 If both the statements are affirmative,
a possibility’, is true in the diagrams (b) and (d).
And, the Conclusion II, i.e. ‘All walls being floors is A
then the conclusion must be affirmative.
W
e.g. Statements All books are schools.
a possibility’, is true in the diagrams (c) and (d). All schools are papers.
SH

Hence, both the conclusion follow.


Conclusions
Rules for Deduction I. All books are papers.
A

II. Some schools are not books.


@

There are some rules, which are extremely


helpful in deciding the conclusion whether it Conclusion II cannot follow, since both the
follows or not. They are as given below statements are affirmative and so the
conclusion should be affirmative. Thus, only
Rule 1 If one statement is negative, then the Conclusion I follows.
conclusion must be negative.
Alternate Method
e.g. Statements All books are rivers.
Papers
No river is big.
Books Schools
Conclusions
I. No book is big.
II. All books are big. So, it is clear that all books are papers.
Since, one statement is negative, the Hence, only Conclusion I follows.
conclusion must be negative. Thus, Rule 4 If both the statements are particular,
Conclusion II cannot follow, only Conclusion I then no conclusion follows.
follows. e.g. Statements Some pens are bags.
Alternate Method Some bags are dogs.
Rivers Conclusions
X
Books Big I. All pens are dogs.
II. Some dogs are pens.
Chapter 21 SYLLOGISM 239

Here, both the statements are particular. Rule 7 If the major statement is particular and
Therefore, no conclusion follows. the minor statement is negative, then no
Alternate Method conclusion follows.
Pens Dogs Pens Bags e.g. Statements Some lions are tigers.
No cats are lions.
or Conclusions
Dogs I. No lions are tigers.
Bags II. Some tigers are cats.
Here, we cannot find a definite conclusion. Here, the first statement contains the middle
Rule 5 If both the statements are negative, term ‘lions’ as the subject, so it is the major
then no conclusion follows. statement. The second statement contains the
e.g. Statements No man is woman. middle term ‘lions’ as the predicate, so it is
the minor statement. Since, the major
No woman is apple.
statement is particular and the minor
Conclusions statement is negative. So, no conclusion
I. No man is apple. follows.

A
II. Some apples are woman. Alternate Method

M
Here, both the statements are negative. Lion Tiger Lion Cat
Cat

H
Therefore, no conclusion follows. or

TT
Alternate Method
Tiger
Man Woman Apple

A
Here, no conclusion follow.
W
or
Types of Questions
SH

Man Apple Woman

There are two types of questions which are


A

Here, we cannot find a definite conclusion. generally asked in different competitive


@

examinations.
Rule 6 If major statement is affirmative, then
the conclusion must be particular.
e.g. Statements All animals are rats. Type 1
Some dogs are animals. Two or more Statements and
Conclusions
Conclusions
I. Some dogs are rats.
II. All rats are dogs. In this type of questions, two or more
Here, the first statement contains the middle statements are given followed by conclusions.
term ‘animals’ as the subject and is thus, the On the basis of the given statements, the
major statement and is also affirmative. So, correctness of conclusion is checked or
the conclusion must be particular. Therefore, conclusion follows from the statement or not.
Conclusion II cannot follow. Only Conclusion Directions (Illustrations 1-4) Two statements
I follows. are given in each of the following question,
Alternate Method followed by two Conclusions I and II. You have to
Rats take the given two statements to be true even, if
they seem to be at variance from commonly known
Animals Dogs
facts. Read the conclusions and then decide which
of the given conclusions logically follows from the
So, it is clear that some dogs are rats. two disregarding known facts.
So, Conclusion I follows.
240 How to Crack Test of Reasoning ANALYTICAL

Give Answer Illustration 3. Statements


(a) If only Conclusion I follows UP Police (Constable) 2018
(b) If only Conclusion II follows Some chalks are dusters.
(c) If neither I nor II follows All the dusters are boards.
(d) If both Conclusions I and II follow Conclusions
Illustration 1. Statements I. Some chalks are boards.
All huts are mansions. II. No duster is a board.
All mansions are temples. Solution (a)
Conclusions
Conclusions
Chalks I. 3
I. Some temples are huts. Dusters
II. 7
II. Some temples are mansions.
Boards
Solution (d)
Conclusion I − A + A = A
So, only Conclusion I follows.
∴ All huts are mansions + All mansions are
temples Illustration 4. Statements BSSC (CGL) 2015
conversion
= All huts are temples →

A
Some birds are clouds.
Some temples are huts.

M
Horse is a bird.
So, Conclusion I follows. Conclusions

H
Conclusion II All mansions are temples I. Some clouds are birds.

TT
conversion
→ Some temples are mansions. II. Horse is not a cloud.
So, Conclusion II follows.
A
Solution (a) According to the question,
W
Alternate Method Birds Birds
This shows the case of universal affirmative
SH
Clouds

proposition. So, Horse or Horse

ds
Temples
A

Clou
Mansions Conclusions
@

Huts I. 3
II. 3 Conclusions
I. 3 II. 7
Clearly, some temples are huts as well as some It is clear from diagram only Conclusion I follows.
temples are mansions. So, both conclusions follow.
Illustration 5. No parrots are black.
Illustration 2. Statements SSC (10+2) 2017 All crows are black.
Some camels are ships. From the above premises, which one of the
No ship is a boat. following conclusions is true? CLAT 2017
Conclusions (a) Some crows are not parrots.
I. Some ships are camels. (b) No crows are parrots.
II. Some camels are not boats. (c) Some parrots are not crows.
(d) No conclusion can be drawn.
Solution (d) According to the question,
× Conclusions Solution (b)
×
Camels Ships Boats I. 3 Parrots Crows
II. 3
Black
It is clear from diagram that both Conclusions I and
II follow. ∴ No crows are parrots is correct conclusion
according to the given statements.
Chapter 21 SYLLOGISM 241

Illustration 6 In the following question, three (a) None follows (b) Only I follows
statements numbered I, II and III and four (c) Only III follows (d) II and IV follow
conclusions are given. Select the correct answer (e) None of these
from the options given. SNAP 2014 Solution (d) According to the question,
Statements Teachers
Engineers
I. Some newspapers are ACs.
Doctors
II. Some ACs are trees. Lawyers Businessmen
III. All books are ACs.
Conclusions Conclusions I. 7 II. 3 III. 7 IV. 3
1. Some books are newspapers. So, II and IV follow.
2. All books are trees.
3. All trees are books. Type 2
4. Some books are not newspapers.
(a) Only 1 follows (b) 1 or 3 follows Find the Statements based on
(c) Either 1 or 4 follows (d) 2 and 3 follow the given Conclusions
Solution (c) According to the question,

A
In this type of questions, two or more
ACs conclusions followed by four or five sets of

M
News- Books Trees statements are given. The candidate has to

H
papers choose the correct set of statements that

TT
logically satisfies given conclusions.
Conclusions
1. 7 A
Illustration 8 In the question below are given
two conclusions followed by five set of
W
2. 7
Either statements. You have to choose the correct set
SH

3. 7 of statements that logically satisfies given


4. 7 conclusions either definitely or possibly.
A

Assume the given statements to be true even if


Conclusion 1 does not follow. Conclusion 4 also
they seem to be at variance from commonly
@

does not follow.


known facts.
But these two conclusions forms a complementary
pair. Therefore, either 1 or 4 follows.
Conclusions I. Some B is E. II. Some J is F.
Statements
Illustration 7 Some statements are given (a) All F is B. Some B is C. All C is D. All D is E.
followed by some conclusions. You have to Some E is J.
(b) Some F is B. All B is C. Some C is D. All D is E.
consider the statements to be true even, if they
Some E is J.
seem to be at variance from commonly known (c) Some F is E. All E is D. Some D is B. All B is C.
facts. You have to decide which of the given All C is J.
conclusions if any, follow from the given (d) All F is E. Some F is J. All J is B. Some B is D.
statements. All D is C.
Statements Some doctors are lawyers. All (e) None of the above
teachers are lawyers. Some engineers are Solution (d) From option (d),
lawyers. All engineers are businessmen.
SBI (PO) 2014 B C
Conclusions E
F J D
I. Some teachers are doctors.
II. Some businessmen are lawyers.
III. Some businessmen are teachers.
From the above diagram, it is clearly shown that
IV. Some lawyers are teachers. some B is E and also some J is F.
242 How to Crack Test of Reasoning ANALYTICAL

Let us Practice
Base Level Exercise
Directions (Q. Nos. 1-12) Two statements are 5. Statements SSC (Multitasking) 2014
given in each of the following questions, followed All crows are birds.
by two Conclusions I and II. You have to take the
All peacocks are crows.
two statements to be true even, if they seems to be
at variance from commonly known facts. Read the Conclusions
conclusions and then decide which of the given I. All peacocks are birds.
conclusion logically follows the given two
II. All birds are peacocks.
statements, disregarding the known facts.
Give Answer 6. Statements SSC (CGL) 2013
(a) If only Conclusion I follows Some keys are locks, some locks are numbers.
(b) If only Conclusion II follows All numbers are letters, all letters are words.
(c) If both Conclusions I and II follow

A
Conclusions
(d) If none of the conclusion follows

M
I. Some words are numbers.
1. Statements II. Some locks are letters.

H
UPSC (CSAT) 2015
Some men are great.

TT
7. Statements SSC (CPO) 2016
Some men are wise
Authors are learned people.
Conclusions
A
Some doctors are authors.
W
I. Men are either great or wise. Conclusions
II. Some men are neither great nor wise.
SH

I. Some doctors are learned people.


2. Statements SSC (Steno) 2012 II. Some learned people are doctors.
A

All players are doctors.


8. Statements SSC (CGL) 2016
Some doctors are actors.
@

All Hindus are God fearing.


Conclusions No Japanese is a Hindu.
I. Some doctors are players as well as Conclusions
actors. I. The Japanese are not God fearing.
II. All actors are doctors. II. All God fearing are Hindus.
3. Statements SSC (CGL) 2012 9. Statements SSC (CGL) 2017
All skaters are good swimmers. No lighters are fire.
All good swimmers are runners. All matchboxes are lighters.
Conclusions Conclusions
I. Some runners are skaters. I. No fire are matchboxes.
II. Some skaters are good swimmers. II. All matchboxes are fire.
4. Statements SSC (10+2) 2012 10. Statements SSC (10 + 2) 2017
Some buses are four wheelers. Some perfumes are incenses.
All four wheelers are vans. All perfumes are oils
Conclusions Conclusions
I. Some vans are buses. I. Some incenses are oils.
II. Some buses are vans. II. No oils are incenses.
Chapter 21 SYLLOGISM 243

11. Statements SSC (10 + 2) 2018 17. Statements


All chains are ropes. Some pens are erasers.
Some links are chains. All erasers are staplers.
Conclusions Conclusions
I. Some ropes are links. I. Atleast some staplers are pens.
II. All links are ropes. II. There is a possibility that some erasers
are neither staplers nor pens.
12. Statements SSC (10 + 2) 2018
All watches are ornaments. 18. Statements
All clocks are watches. No red is black.
Conclusions Some black are yellow.
I. All ornaments are clocks. Conclusions
II. All clocks are ornaments. I. No yellow is red.
Directions (Q. Nos. 13-26) In these questions, II. All red being yellow is a possibility.
two/three statements are given followed by two 19. Statements
Conclusions I and II. You have to take the given
All shores are beaches.
statements true even if they seem to be at variance
Some beaches are coasts.

A
from commonly known facts and then decide which
of the given conclusions logically follows from the All banks are coasts.

M
given statements disregarding commonly known Conclusions

H
facts. RRB (Assistant) 2016 I. Some banks are beaches.

TT
Give Answer II. No bank is a shore.
(a) If either Conclusion I or Conclusion II is true
(b) If both Conclusions I and II are true 20. Statements A
W
(c) If neither Conclusion I nor II is true All benches are parks.
(d) If only Conclusion II is true No park is a tree.
SH

(e) If only Conclusion I is true All trees are swings.


13. Statements All photos are signatures. Conclusions
A

All tubes are signatures. I. All benches being swings is a possibility.


@

Conclusions II. No tree is a bench.


I. Atleast some tubes are photos.
21. Statements
II. All photos are tubes.
All trains are buses.
14. Statements All horses are goats. All buses are jeeps
No goat is a donkey. All jeeps are aeroplanes.
Conclusions Conclusions
I. No horse is a donkey. I. Some buses are not aeroplanes.
II. Atleast some goats are horses.
II. All jeeps are trains.
15. Statements Some books are cars.
All books are tarts.
22. Statements
Conclusions Some parties are celebrations.
All celebrations are occasions.
I. Some cars are tarts.
No occasion is a festival.
II. All cars are tarts.
Conclusions
16. Statements No ring is a cap. I. No celebration is a festival.
Some bags are rings.
II. Some occasions are parties.
Conclusions
I. All caps are rings. 23. Statements
II. Some rings are definitely not caps. All papers are mills.
244 How to Crack Test of Reasoning ANALYTICAL

All mills are factories. Some perfumes are deodorants.


Conclusions All deodorants are cologne.
I. All mills are papers. Conclusions
II. All papers are factories. I. Atleast some deodorants are not
fragrances.
24. Statements II. All fragrance being colognes is a
No perfume is a fragrance. possibility.
Some perfumes are deodorants.
26. Statements
All deodorants are cologne.
Some stands are racks.
Conclusions No rack is a box.
I. Atleast some perfumes are colognes. All boxes are cartons.
II. No fragrance is a deodorant. Conclusions
25. Statements I. All stands can never be boxes.
No perfume is a fragrance. II. All racks being cartons is a possibility

A
Expert Level Exercise

M
H
Directions (Q. Nos. 1-5) Two/three/four (b) I and II follow

TT
statements are given in each of the following (c) II and III follow
question, followed by three/four Conclusions I, II, (d) III and IV follow
III and IV. You have to take the given statements
to be true even, if they seem to be at variance from
3. Statements A SSC (10+2) 2011
W
commonly known facts. Read all the conclusions
All dogs are rats.
SH

and then decide which of the given conclusions All rats are crows.
logically follows from the given statements All crows are parrots.
A

disregarding commonly known facts. Conclusions


@

1. Statements All cars are black. I. All dogs are parrots.


No black is shining. II. Some parrots are dogs.
Conclusions III. Some crows are dogs.
I. Some cars are shining. IV. All rats are dogs.
(a) I and II follow
II. No car is shining (b) I, II and III follow
III. No shining is car. (c) Either II or IV follows
(a) Conclusions I and II follow (d) Either I or II or III follows
(b) Conclusions II and III follow
(c) Conclusions II and III follow 4. Statements SSC (CGL) 2013
(d) All conclusions follow Some years are decades.
All centuries are decades.
2. Statements All aeroplanes are trains. Conclusions
Some trains are chairs.
I. Some centuries are years.
Conclusions
II. Some decades are years.
I. Some aeroplanes are chairs.
III. No century is a year.
II. Some chairs are aeroplanes. (a) Conclusions I and II follow
III. Some chairs are trains. (b) Conclusions I and III follow
IV. Some trains are aeroplanes. (c) Either I or III and II follow
(a) None follows (d) Conclusion II follows
Chapter 21 SYLLOGISM 245

5. Statements SSC (CPO) 2013 9. Statements All circles are triangles. Some
All philosophers are fools. All fools are triangles are rectangles. All rectangles are
illiterate. squares. IBPS (PO) Pre 2016
Conclusions Conclusions
I. All philosophers are illiterate. I. All rectangles being triangles is a
possibility.
II. All illiterates are philosopher.
II. All circles being square is a possibility.
III. All illiterates are fools.
IV. Some illiterates would be philosopher. 10. Statements All circles are triangles. Some
(a) Only IV follows triangles are rectangles. All rectangles are
(b) I and IV follow squares. IBPS (PO) Pre 2016
(c) Only II follows Conclusions
(d) III and IV follow I. Some triangles are not rectangles.
Directions (Q. Nos. 6-16) In these questions, II. No square is a circle.
two/three statements followed by two
conclusions are given. You have to take the given
11. Statements Some chairs are tables. Some bed
statements to be true even if they seem to be at are tables. No furniture is bed. IBPS (PO) Pre 2016
Conclusions

A
variance from commonly known facts and then
decide, which of the given conclusions logically I. All chairs being furniture is a possibility

M
follows disregarding commonly known facts. II. Some tables are not bed is a possibility.

H
Give Answer 12. Statements Some chairs are table. Some bed

TT
(a) If only Conclusion I follows are tables. No furniture are bed.
(b) If only Conclusion II follows IBPS (PO) Pre 2016
(c) If either Conclusion I or II follows Conclusions A
W
(d) If neither Conclusion I nor II follows I. Some tables are not furniture.
(e) If both Conclusions I and II follow
SH

II. All tables being furniture is a possibility.


6. Statements IBPS (RRB) 2016
13. Statements SBI (PO) Pre 2015
A

All copies are book. No factory is an industry.


@

No book is a pencil. All workshops are industries.


All pencils are rubbers. Some plants are workshops.
Conclusions Conclusions
I. No rubber is a book.
I. No workshop is a factory.
II. Some copies are rubbers.
II. Atleast some plants are industries.
7. Statements IBPS (RRB) 2016
All copies are book. 14. Statements SBI (PO) Pre 2015
No book is a pencil. Some movies are films.
All pencils are rubbers. No film is a show.
All shows are pictures.
Conclusions
I. No copy is a pencil. Conclusions
II. Some books are rubbers. I. Atleast some pictures are films.
II. No show is a movie.
8. Statements IBPS (RRB) 2016
Some mobiles are calculators. 15. Statements SBI (PO) Pre 2015
Some calculators are pens. Some actors are singers.
Some pens are scales. All singers are dancers.
Conclusions Some dancers are players.
I. No calculator is a scale. Conclusions
II. Atleast some calculators are scales. I. All actors being dancers is a possibility.
II. All dancers are singers.
246 How to Crack Test of Reasoning ANALYTICAL

16. Statements SBI (PO) Pre 2015 20. Statements Some pins are needles. All
Some actors are singers. needles are ropes. Some ropes are buckets.
All singers are dancers. All buckets are trees.
Some dancers are players. Conclusions I. Some buckets are pins.
Conclusions II. Some ropes are pins.
I. Atleast some dancers are actors. III. No bucket is pin.
II. No player is an actor. (a) Only I follows (b) Only II follows
(c) Only III follows (d) None follows
Directions (Q. Nos. 17-20) Four statements are (e) Either I or III and II follow
given in each of the following question, followed by
three conclusions numbered I, II and III. You have Directions (Q. Nos. 21-25) In these questions,
to take the given statements to be true even, if they four statements are given followed by five
seem to be at variance from commonly known facts. conclusions. One of which definitely does not
Read all the conclusions and then decide which of logically follow (or is not a possibility of
the given conclusions logically follows from the occurrence) from the given statements. That
given statements disregarding commonly known conclusion is your answer. IBPS (PO) 2014
facts. IDBI (Office Executive) 2018 Note You have to take the four given statements to

A
17. Statements Some villages are towns. be true even if they seem to be at variance
Some towns are huts. All huts are rivers. Some from commonly known facts and then decide

M
rivers are tents. which of the given conclusions logically does

H
not follow from given statements disregarding
Conclusions I. Some tents are towns.

TT
commonly known facts.
II. Some rivers are towns.
21. Statements Some logics are reasons. All
III. Some huts are villages.
A
reasons are arguments. All arguments are
W
(a) Only I follows (b) Only II follows
fights. No fight is a discussion.
(c) Only III follows (d) I and II follow
SH

Conclusions
(e) None of these
(a) All discussion being logics is a possibility
18. Statements All hotels are buses. (b) No discussion is an argument
A

Some buses are cars. All cars are trams. (c) All logics being discussion is a possibility
@

Some trams are clouds. (d) All reasons are fights


(e) No reason is a discussion
Conclusions I. Some trams are buses.
II. Some trams are hotels. 22. Statements All references are mails. All
mentions are references. All comments are
III. Some clouds are cars.
mentions. No mail is a declaration.
(a) Only I follows (b) Only II follows
Conclusions
(c) Only III follows (d) I and II follow
(a) No reference is a declaration
(e) None of these (b) All comments are mails
19. Statements All flowers are books. (c) No mention is a declaration
(d) All declaration being comments is a
All books are carpets. Some carpets are keys.
possibility
Some keys are locks.
(e) Atleast some mails are mentions
Conclusions I. Some keys are books.
23. Statements Some moments are flashers. All
II. Some keys are flowers.
moments are seconds. All flashers are
III. Some locks are books. instances. No instance is an hour.
(a) Only I follows Conclusions
(b) Only II follows (a) All hours being seconds is a possibility
(c) Only III follows (b) No second is a flash
(d) None follows (c) No hour is a flash
(e) All follow (d) Atleast some moments are instances
(e) Atleast some seconds are instances
Chapter 21 SYLLOGISM 247

24. Statements All circles are spheres. All 27. Conclusions Some air is system.
spheres are rectangles. No rectangle is a Some road is control.
pyramid. No pyramid is a triangle. Statements
Conclusions (a) All air is control. Some control is system. All
(a) Atleast some circles are pyramids system is idea. Some idea is traffic. All traffic is
(b) All triangles being circles is a possibility road.
(c) All rectangle being triangles is a possibility (b) All air is road. Some air is traffic. All traffic is
(d) Atleast some rectangle are circles control. All control is system. Some system
(e) No pyramid is a sphere is idea
(c) Some air is road. All road is idea. Some idea is
25. Statements No hotel is a motel. All motels traffic. All traffic is system. All system is control.
are apartments. All apartments are inns. (d) All air is traffic. Some traffic is control. All
No inn is a guesthouse. control is system. Some system is idea. All idea is
Conclusions road
(a) All hotels being inns is a possibility (e) None of the above
(b) No motel is guesthouse 28. Conclusions Some man is not doll. Some toy is
(c) All hotels being apartments is a possibility man.
(d) No motel is an inn
Statements

A
(e) No guesthouse is an apartment
(a) All toy is boy. Some boy is child. No child is

M
Directions (Q. Nos. 26-29) In the questions man. All man is girl. All girl is doll.

H
below are given two conclusions followed by five (b) Some toy is man. All man is doll. No doll is girl.
set of statements. You have to choose the correct All boy is girl. Some girl is child.

TT
set of statements that logically satisfies given (c) Some toy is child. All child is man. Some man is
conclusions either definitely or possibly. Assume girl. No girl is doll. All boy is doll.
the given conclusions to be true even if they A
(d) Some toy is girl. All girl is doll. No doll is child.
W
seem to be at variance from commonly known All boy is child. Some child is man.
facts. (e) None of the above
SH

26. Conclusions Some F is A. No E is C. 29. Conclusions Some red is white. Some yellow
is red.
A

Statements
Statements
@

(a) Some F is B. All B is C. No C is D. All D is


A. Some D is E. (a) All white is black. All black is red. Some red is
(b) Some F is B. All B is A. No A is C. All D is violet. All violet is yellow. Some yellow is green.
C. Some C is E. (b) All white is violet. Some violet is red. All red is
black. Some black is yellow. All yellow is green.
(c) All A is F. No F is C. All B is C. Some C is
D. All D is E. (c) All white is green. Some green is red. All red is
black. Some black is violet. All violet is yellow.
(d) Some A is E. All E is F. No F is C. All D is
(d) All white is yellow. Some yellow is red. All red is
C. Some C is B.
black. Some black is violet. All violet is green.
(e) None of the above
(e) None of the above

Answers
Base Level Exercise
1. (d) 2. (d) 3. (c) 4. (c) 5. (a) 6. (c) 7. (c) 8. (d) 9. (a) 10. (a)
11. (a) 12. (b) 13. (c) 14. (b) 15. (e) 16. (d) 17. (e) 18. (d) 19. (c) 20. (b)
21. (c) 22. (b) 23. (d) 24. (e) 25. (b) 26. (b)

Expert Level Exercise


1. (b) 2. (d) 3. (b) 4. (c) 5. (b) 6. (d) 7. (a) 8. (c) 9. (e) 10. (d)
11. (e) 12. (a) 13. (e) 14. (d) 15. (a) 16. (a) 17. (b) 18. (a) 19. (d) 20. (e)
21. (c) 22. (d) 23. (b) 24. (a) 25. (d) 26. (d) 27. (b) 28. (c) 29. (a)
248 How to Crack Test of Reasoning ANALYTICAL

Answer with Explanations


Base Level Exercise
1. (d) According to the statements, venn diagram is as 6. (c) According to the statements, venn diagram
follows is as follows
Words
Wise Men Great

Keys Locks Numbers


Conclusions I. 7 II. 7
From above, it is clear that none of the conclusions Letters
follows.
2. (d) According to the statements, venn diagram is as Conclusions I. 3
follows
II. 3
Hence, Conclusions I and II follow.
Players Actors
7. (c) According to the statements, venn diagram
Doctors is as follows
ple
Peo s

A
Conclusions I. 7 II. 7

arned

M
From above, it is clear that none of the conclusion Authors

H
follows.

Le
TT
3. (c) According to the statements, venn diagram is as Doctors
follows
Conclusions I. 3 II. 3
Good Swimmers A
So, both Conclusions I and II follow.
W
8. (d) According to the statements, venn diagram
Skaters
is as follows
SH

Runners Japanese
A

Conclusions I. 3 II. 3
@

Hence, both Conclusion I and II follow. Hindus


4. (c) According to the statements, venn diagram is as
follows God fearing

Four
Buses Conclusions I. 7
wheelers
II. 7
Vans Hence, neither Conclusion I nor II follows.
9. (a) According to the statements, venn diagram
Conclusions I. 3 II. 3 is as follows
Hence, both Conclusions I and II follow.
5. (a) According to the statements, venn diagram is as
follows Matchboxes × Fire
Birds
Crows
Lighters
Peacocks
Conclusions I. 3
II. 7
Conclusions I. 3 II. 7 Hence, only Conclusion I follows.
Thus, only conclusion I follows.
Chapter 21 SYLLOGISM 249

10. (a) According to the statements, venn diagram is as 15. (e) According to the statements, venn diagram is
follows as follows

Perfumes Incenses
Tarts Books Cars

Oils

Conclusions I. 3 II. 7
Conclusions I. 3 II. 7
Hence, only Conclusion I follows.
Hence, only Conclusion I is true.
11. (a) According to the statements, venn diagram is as
follows 16. (d) According to the statements, venn diagram is
as follows
Ropes Bags Rings × Caps

Links Chairs Conclusions I. 7 II. 3


Hence, only Conclusion II is true.
17. (e) According to the statements, venn diagram is
Conclusions I. 3 as follows
II. 7

A
Staplers
Hence, only Conclusion I follows.

M
Pens Erasers
12. (b) According to the statements, venn diagram is as

H
follows

TT
Ornaments Conclusions I. 3 II. 7
So, only Conclusion I follows.
Watches
Clocks
A
18. (d) According to the statements, venn diagram is
W
as follows
SH

Yellow
w or Black
Conclusions I. 7 Red Black llo Red
Ye
II. 3
A

Hence, only Conclusion II follows. Conclusions I. 7 II. 3


@

13. (c) According to the statements, venn diagram is as So, only Conclusion II follows.
follows
Signatures 19. (c) According to the statements, venn diagram is
as follows
Beaches Shores
Photos Tubes
Banks
Coasts

Conclusions I. 7 II. 7
Hence, neither Conclusion I nor II is true.
Conclusions I. 7 II. 7
14. (b) According to the statements, venn diagram is as
So, neither Conclusion I nor II follows.
follows
Goats 20. (b) According to the statements, venn diagram is
×
as follows
Horses Donkey Parks Swings

Benches Trees

Conclusions I. 3
II. 3 Conclusions I. 3 II. 3
Hence, both Conclusions I and II are true. So, both Conclusions I and II follow.
250 How to Crack Test of Reasoning ANALYTICAL

21. (c) According to the statements, venn diagram Conclusions I. 7 II. 3


is as follows So, only Conclusion II follows.
Aeroplanes 24. (e) According to the statements, venn diagram is as
Jeeps
Buses follows
Trains

es
m
rfu

es
Pe

gn
Conclusions I. 7 II. 7

lo
Co
So, neither Conclusion I nor II follows.
× Deodorants
22. (b) According to the statements, venn diagram
is as follows Fragrances
Parties Celebrations

Conclusions I. 3 II. 7
Festivals So, only Conclusion I follows.
25. (b) According to the above Sol. 24, venn diagram,
Conclusions I. 3 II. 3
Occasions So, both Conclusions I and II follow.

A
Conclusions I. 3 26. (b) According to the statements, venn diagram is as
II. 3

M
follows
So, both Conclusions I and II follow.

H
×
23. (d) According to the statements, venn diagram

TT
Stands Racks Boxes
is as follows
Cartons
A
W
Papers Mills Conclusions I. 3 II. 3
ies So, both Conclusions I and II follow.
SH

or
ct
Fa
A
@

Expert Level Exercise


1. (b) According to the statements, venn diagram is 3. (b) According to the statements, venn diagram is as
as follows follows

Cars × Shining

Black
Dogs
Conclusions I. 7 II. 3 III. 3 Rats
It is clear from diagram that Conclusions II and Crows
III follow.
Parrots
2. (d) According to the statements, venn diagram
is as follows Conclusions I. 3 II. 3 III. 3 IV. 7
Clearly, I, II and III follow.
4. (c) According to the statements, venn diagram is as
Aeroplanes Chairs follows
Years
Trains
Centuries
Conclusions I. 7 II. 7 III. 3 IV. 3
So, Conclusions III and IV follow. Decades
Chapter 21 SYLLOGISM 251

Conclusions I. 7 II. 3 III. 7 10. (d) According to the Sol. 9 venn diagram,
Conclusions I. 7
Complementary pair II. 7
From the above figure, it is clear that either I or III Hence, neither Conclusion I nor II follows.
and II follow. 11. (e) According to the statements, venn diagram is
5. (b) According to the statements, venn diagram is as follows
as follows ×

Tables
Illiterate
Chairs Beds Furnitures
Fools
Philosophers
Conclusions I. 3
II. 3
Hence, both Conclusions I and II follow.
Conclusions I. 3 II. ✗ III. 7 IV 3 12. (a) According to the Sol. 11 venn diagram,
So, only Conclusions I and IV follow. Conclusions I. 3
II. 7
6. (d) According to the statements, venn diagram is Hence, only Conclusion I follows.

A
as follows
13. (e) According to the statements, venn diagram is as

M
Books Rubbers follows

H
Industries
Copies Pencils

TT
×
Factories

Conclusions I. 7 II. 7 A
W
Plants
Hence, neither Conclusion I nor II follows. Workshops
SH

7. (a) According to the Sol. 6 venn diagram,


Conclusions I. 3 II. 3
Conclusions I. 3 II. 7
So, both conclusions are true.
A

Hence, only Conclusion I follows.


14. (d) According to the statements, venn diagram is as
@

8. (c) According to the statements, venn diagram is


as follows follows
Pictures
Calculators

Movies Films × Shows


Mobiles Pens Scales

Conclusions Conclusions I. 7 II. 7


I. ✗ So, neither Conclusion I nor II is true.
Complementary pair 15. (a) According to the statements, venn diagram is
II. ✗
Hence, either Conclusion I or II follows. as follow
9. (e) According to the statements, venn diagram is Dancers
as follows
Actors

Triangles Squares Singers


Rect-
Circles angles
Players

Conclusions I. 3 Conclusions I. 3
II. 3 II. 7
Hence, both Conclusions I and II follow. Hence, only Conclusion I is true.
252 How to Crack Test of Reasoning ANALYTICAL

16. (a) According to the Sol. 15 venn diagram, 21. (c) According to the statements, venn diagram is
Conclusions I. 3 II. 7 as follows
Hence, only Conclusion I is true.
Reasons ×
17. (b) According to the Sol. 15 venn diagram, Logics
Discussion
Arguments

Villages Towns Huts Fights


Tents
Rivers Conclusions
(a) ✓ (b) ✓ (c) ✗ (d) ✓
Conclusions I. 7 II. 3 (e) ✓
III. 7 Hence, conclusion (c) does not follow from the
Hence, only Conclusion II follows. given statements.
18. (a) According to the statements, venn diagram is 22. (d) According to the statements, venn diagram is
as follows as follows

Hotels Cars Comments


×
Buses Trams Clouds Mentions

A
Declaration

M
References
Conclusions I. 3 II. 7

H
III. 7 Mails

TT
Hence, only Conclusion I follows.
19. (d) According to the statements, venn diagram is as Conclusions
(a) ✓ (b) ✓ (c) ✓ (d) ✗ (e) ✓
follows
A
Hence, conclusion (d) does not follow from the
W
given statements.
SH

23. (b) According to the statements, venn diagram is


as follows
A

Flowers
Keys Locks ×
@

Moments Flashers
Hours
Books
Seconds Instances
Carpets
Conclusions
Conclusions I. 7 II. 7
(a) ✓ (b) ✗ (c) ✓ (d) ✓ (e) ✓
III. 7
Hence, no Conclusion follow. Hence, conclusion (b) does not follow from the
given statements.
20. (e) According to the statements, venn diagram is as 24. (a) According to the statements, venn diagram is
follows as follows
Pins
Needles Circles × ×
Pyramids Triangles
Buckets Spheres
Ropes
Rectangles
Trees
Conclusions I. 7 II. 3 III. 7 Conclusions
Complementary pair (a) ✗ (b) ✓ (c) ✓ (d) ✓ (e) ✓
Hence, conclusion (a) does not follow from the
Either I or III and II follow. given statements.
Chapter 21 SYLLOGISM 253

25. (d) According to the statements, venn 27. (b) According to the option (b), venn diagram is as follows
diagram is as follows
System
× Control
Motels Road
Hotels Guesthouse Air Traffic Idea
Apartments
×
Inns

Conclusions Clearly, some air is system and some road is control.


(a) ✓ 28. (c) According to option (c), venn digram is as follows
(b) ✓
(c) ✓ Toy Child Boy
(d) ✗ Girl
Man Doll
(e) ✓
Hence, conclusion (d) does not follow Clearly, some man is not doll and some toy is man.
from the given statements. 29. (a) According to the option (a), venn diagram is as follows
26. (d) According to the option (d) venn
diagram is as follows Red Yellow

A
F Black

M
A E × C D B
White Violet Green

H
TT
Clearly, some F is A and no E is C.

A
Clearly, some red is white and some yellow is red.
W
SH
A
@
254 How to Crack Test of Reasoning ANALYTICAL

22
Statement and
Assumptions
Statement is an information or a fact related to Directions (Illustrations 1-2) One statement

A
any general subject and assumption is the is given in each question mentioned below,

M
hidden or unsaid part of the statement which a followed by two assumptions I and II. You have

H
person assumes before stating some fact or to consider the statement and the following
statement. assumptions and decide which of the

TT
When we analyse assumption, we find that assumptions is implicit in the statement.
when one says or writes something, he does not Give Answer
A
(a) If only I is implicit
W
put everything into words and leaves some part
(b) If only II is implicit
unsaid or unwritten. He does so because he
SH

(c) If neither I nor II is implicit


takes this unsaid part for granted. In other
(d) If both I and II are implicit
words, he thinks this unsaid part will be
A

understood without saying and there is no need


Illustration 1. Statement Imprisonment for
@

to put this (unsaid part) into words.


27 yr made Nelson Mandela, the President.
In this type of questions, a statement is followed Assumptions
by two or more assumptions. The candidate is
I. Only who will be imprisoned for 27 yr will
required to assess the given statement and
become the President.
decide, which of the given assumptions is
implicit in the statement. II. To become the President, imprisonment is
a qualification.
Important points to be considered while
Solution (c) Nelson Mandela elected as the
selecting correct assumptions
President on the appeal of people as he struggled
l
It should be directly related to the statement. for the freedom of Africa and he was dedicated to
l
It should not be ambiguous. the nation. Here, statement does not clarify any of
l
It should not be superfluous. the assumptions.
l
It should not be merely repetition of the
statement. Illustration 2. Statement It is desirable to
l
It should not be the argument drawn from the put the child in school at the age of 5 as so.
statement. SSC (CPO) 2014, MPPSC 2014
l
It should not include words like ‘all’, ‘every’,
Assumptions
‘each’, ‘only’, etc.
I. At that age the child reaches appropriate
l
It may include words related to any
possibility, such as may be, possibly, etc. level of development and is ready to learn.
l
It can be many, in numbers, based on a single II. The schools do not admit children after
statement. 6 yr of age.
Chapter 22 STATEMENT AND ASSUMPTIONS 255

Solution (a) At the age of 5 the child reaches So, I is not implicit. Also, the statement is just an
appropriate level of development and is ready attempt to arouse the people to come and see the
to learn. So, it is necessary to put him/her in school shop. So, both II and III are implicit.
at this age. So, assumption I is implicit. Assumption
II is not implicit as no information is given regarding Illustration 4. Statement The Central
the age limit of a child to take admission. Government has directed the State
Governments to reduce government
Directions (Illustration 3) A statement is given,
expenditure in view of the serious resources
followed by several assumptions. Read the
crunch and it may not be able to sanction any
statement carefully and decide which of the
additional grant to the States for the next six
assumptions is implicit in the statement.
months.
Statement “We do not want you to see our Which of the following may be as assumption
product in newspaper, visit our shop to get a implicit in the above statement? SBI (MT) 2014
full view.” —An advertisement (a) The State Governments are totally dependent
on Central Government for its expenditure
Assumptions
(b) The Central Governments has reviewed the
I. People generally decide to purchase any expenditure account of the State Governments
product after seeing the same in the (c) The State Government would ignore such
directives as these have majority Government

A
advertisement.
(d) The Central Government should allocate more

M
II. Uncommon appeal may attract customers. funds to the States
III. People may come to see the product.

H
(e) None of the above
(a) Both I and III are implicit

TT
Solution (b) Assumption (a) is not implicit as nothing
(b) Both I and II are implicit is mentioned about the income of State
(c) Both II and III are implicit
(d) All are implicit A
Governments in the statement. Assumption (b) is
implicit as the Central government will direct the
W
(e) None is implicit
State Governments to reduce government
Solution (c) It can be inferred from the statement that
SH

expenditure only after reviewing the expenditure


people like to see a product before buying. account of State Governments.
A
@

Let us Practice
Base Level Exercise
Directions (Q. Nos. 1-21) In each question 1. Statement The government has recently
below is given a statement followed by two announced an incentive package for setting
assumptions. An assumption is something up new business ventures in the rural areas
supposed or taken for granted. You have to and promised uninterrupted power supply
consider the statement and decide which of the to all the units.
assumption is implicit in the statement. Assumptions
Give Answer I. The government may be able to supply
(a) If only assumption I is implicit.
adequate power to all such units.
(b) If only assumption II is implicit.
(c) If neither assumption I nor II is implicit. II. People living in the rural areas may
(d) If both assumption I and II are implicit. welcome the government decision.
256 How to Crack Test of Reasoning ANALYTICAL

2. Statement The municipal authority blocked 7. Statement Those who are appearing for
movement of traffic in and around the this examination for the first time, should be
temple on the main festival day. helped in filling up the forms.
Assumptions —An instruction to the invigilating staff.
I. Very large number of devotees may visit Assumptions
the temple on the main festival day. I. The form is somewhat complicated.
II. People travelling to the areas near the II. Candidates can appear more than once
temple may postpone their journey by a for this examination.
day unless they have very urgent work
8. Statement Be humble even after gaining
in the areas.
victory.
3. Statement The government has instructed Assumptions
all the private schools in the city to maintain
I. Many people are humble after gaining
the current fees for atleast two more years.
victory.
Assumptions
II. Generally, people are not humble.
I. The authorities of private schools may
not follow the government instruction as 9. Statement It is dangerous to lean out of a
they are not dependent on government moving train.

A
funds. Assumptions

M
II. The parents of the students of private I. All those who lean out of a train take the

H
schools of the city may still be eager to risk of being hurt.

TT
pay higher fees. II. Generally, people don’t like to get hurt.
4. Statement The municipal authority has
decided to demolish the old bridge on a bus A
10. Statement “Buy pure and natural honey of
company X”—An advertisement in a
W
road for constructing new flyover. newspaper says.
SH

Assumptions
Assumptions
I. The traffic department may be able to
I. No other company supplies pure and
A

divert movement of vehicles through


natural honey.
alternate roads.
@

II. People read advertisement.


II. The people travelling in the nearby areas
may demonstrate to protest against the 11. Statement The target of a fiscal deficit of 5%
authority’s decision. of GDP could not be met because of major
short fall in revenue collection.
5. Statement Rich people are more prone to
have heart attacks. SSC (10+2) 2012 Assumptions
Assumptions I. Shortfall in revenue collection leads to an
I. Most of the deaths among rich people increase in fiscal deficit.
are due to heart attacks. II. Shortfall in revenue collection leads to a
II. Poor people do not have heart attacks. decrease in fiscal deficit.

6. Statements Only good singers are invited in 12. Statement We need not worry about errors
the conference. No one without sweet voice but we must try to learn from our errors.
is a good singer. Assumptions OPSC 2018
Assumptions I. Errors may take place when we are
I. All invited singers in the conference have carrying out certain work.
sweet voice. II. We are capable of benefitting from the
II. Those singers who do not have sweet past and improve our chances error free
voice, are not invited in the conference. work.
Chapter 22 STATEMENT AND ASSUMPTIONS 257

13. Statement Drastic increase in pollution is Assumptions


now a global phenomenon. MPPSC 2018 I. Training helps in boosting the
Assumptions performance of employees.
I. All countries have realized it and II. Employees have no skill sets before
working towards the control. training is provided to them.
II. Growing population, industries, 18. Statement Take a ferry or a boat instead of a
highways and constructions are beyond bus to reach the Kravi Islands faster.
control.
PNB (MT) 2011
14. Statement Send employee XYZ for a training Assumptions
in UK to gain more insight into the project I. The islands being in remote location are
which he is handling at present. not easily accessible.
SBI Associates (PO) 2014 II. Ferries and boats are available to travel
Assumptions to Kravi Islands.
I. Some similar training programs are
available in the employee’s own country
19. Statement The government has decided to
run all commercial vehicles on biofuels in
as well.
order to save the depleting fossil fuel

A
II. With the present skill sets, employee reserves. PNB (MT) 2011
XYZ is incapable of handling the project.

M
Assumptions

H
15. Statement Travelling by trains rather than I. It is possible to switch over from fossil
cabs is more convenient and economical in fuel to biofuels for vehicles.

TT
Paris. SBI (PO) 2014 II. Sufficient amount of biofuel can be
Assumptions
A
produced in the country to run all
commercial vehicles.
W
I. Paris is an expensive city.
II. Train services are reasonably good in 20. Statement A Physics teacher in the class
SH

Paris. announced to keep a copy of Concepts of


16. Statement The school environment for Physics written by H.C. Verma. MPPSC 2018
A

students should be made ‘‘free from fear, Assumptions


@

trauma and anxiety’’ to make the right to I. The book is well-written.


education a grand national movement. II. Every student can afford to buy the
PNB (MT) 2011 book.
Assumptions
21. Statement Tuberculosis (TB) is a pandemic
I. The Right to Education Movement is
issue now. MPPSC 2018
desirable for the betterment of society.
Assumptions
II. Children feel a lot of anxiety and stress at
I. Government should work towards
school.
complete eradication of diseases like
17. Statement Give adequate job related training polio.
to the employees before assigning them full II. Tuberculosis is not a serious disease and
fledged work. PNB (MT) 2011 treatment is available.
258 How to Crack Test of Reasoning ANALYTICAL

Expert Level Exercise


Directions (Q. Nos. 1-4) A statement is given in 4. Statement An advertisement in the
each of the following question, followed by two newspaper – ‘Learn a foreign language
assumptions I and II. An assumption is something course to get a high paying job’. PNB (MT) 2011
supposed or taken for granted. You have to Assumptions
consider the statement and the following
assumptions and decide which of the assumptions I. All those who learn foreign language get
is implicit in the statement. highly paid jobs.
II. Only a foreign language can get someone
Give Answer
(a) If only I is implicit a high paying job.
(b) If only II is implicit Directions (Q. Nos. 5 and 6) A statement is
(c) If either I or Ii is implicit given for each of the question given below,
(d) If neither I nor II is implicit followed by three assumptions I, II and III. You
(e) If both and II are implicit have to consider the statement and the following
assumptions and decide which of the assumption
1. Statement The impact of economic sanctions is implicit in the statement, then decide which of
on economy, that is already so weak, could the answers (a), (b), (c) and (d) is a correct answer

A
be devastating. and indicate it on your answer sheet.

M
Assumptions
5 Statement “Use Riya cold cream for fair

H
I. Economic sanctions impact only a weak
complexion”—An advertisement. MAT 2005

TT
economy.
Assumptions
II. The impact of economic sanctions varies
I. People like to use cream for fair
from economy to economy.
complexion.A
W
2. Statement Minute traces of nitrate II. People are easily fooled.
SH

sometimes present in cattle fodder B may be III. People respond to advertisements.


responsible for the poor health of the cattle
(a) Only I is implicit
therefore use fodder G to ensure that your
A

(b) Both I and II are implicit


cattle remain healthy. SBI Associates (PO) 2014
@

(c) Only II is implicit


Assumptions (d) Both I and III are implicit
I. Cattle fodder G does not contain nitrate.
6. Statement “God! if my intentions have
II. Nitrate is toxic to all living beings. been pure and if my efforts sincere, I pray
3. Statement Due to inadequate rainfall this that, by your majestic grace, I emerge a
year, the harvest obtained by farmers of successful woman!”—A woman in a church.
village C is 50% of what was expected. As a SSC (CGL) 2008
result this year we will be buying the crop Assumptions
from village Z as it is been convenient and I. God helps only who pray to him.
cheaper for us to buy the crop from the II. God helps only those whose intentions
farmers of one village rather than buying it are pure.
from farmers of different villages—Statement
III. Prayers do help people.
by a distributor.
(a) Both I and II are implicit
I. Atleast some farmers in village Z produce
(b) Both II and III are implicit
the same crop as farmers in village C.
(c) Only II is implicit
II. The harvest in village Z was much more
(d) Only III is implicit
than expected.
Chapter 22 STATEMENT AND ASSUMPTIONS 259

7. 'Attend a free seminar on how to crack MAT Directions (Q. Nos. 10-12) A statement is
examination.'—An advertisement by an given for each of the question mentioned below,
institute. followed by three assumptions I, II and III. An
Which of the following can be an assumption assumption is something supposed or taken for
of the advertiser? SBI (PO) 2017 granted.
(a) There are other institutes which are also
organising such seminars
You have to consider the statement and the
(b) These seminars are just a gimmick by the following assumptions and decide which of the
advertiser to attract students assumption is implicit in the statement, then
(c) There will be people who will respond to this decide which of the answers (a), (b), (c) and (d)
kind of advertisement is a correct answer and indicate it on your
(d) The advertisements many a time turn out to be answer sheet.
exaggerated
(e) None of the above 10. Statement The Prime Minister is expected
to announce an expansion in his Ministry
8. Statement The ‘X’ finance company has shortly. A newspaper report.
launched a new tractor loan scheme. It has Assumptions SNAP 2014
brought prosperity among the farmers. Which
I. The newspaper has quoted authentic
of the following assumptions is implicit in the
sources.

A
above statement? IBPS (PO) 2013
II. The newspaper has reliable sources for

M
(a) The prosperity of farmers solely depends upon
the tractor loan scheme the news.

H
(b) Tractor is an important input for farmers to III. The Prime Minister has the power to

TT
enhance the farm activities expand his cabinet.
(c) Earlier there was no tractor loan scheme for (a) I and II are implicit
farmers (b) A
II and III are implicit
W
(d) Farmers do not prefer to take loans for buying
(c) I and III are implicit
tractors
(d) All are implicit
SH

(e) None of the above

9. Many children today eat so much sugary 11. Statement “If you desire to enjoy the best
A

processed food that tooth decay has become a holiday to Europe, join our exclusive EURO
@

more serious problem than ever. Periodic package.” An advertisement of a travel


brushing can protect children’s teeth only if agency in an Indian newspaper.
they also receive regular twice-yearly SSC (CPO) 2008
professional cleaning. Hence, the dental Assumptions
reimbursement plans offered by most of the I. Many Indians travel to European
companies are adequate to protect the dental countries to spend their holidays.
health of atleast some children.
II. There are other travel agencies in
Which of the following is an assumption
India which organise holiday tours to
implied in the argument? SBI (PO) 2017
Europe.
(a) In the past, children did not require
twice-yearly professional cleaning to protect III. Many people may still travel to Europe
their teeth from decay through other travel agencies.
(b) Some dental reimbursement plans offered by (a) All are implicit
companies are adequate to protect the dental
(b) Both I and II are implicit
health of children who do not consume sugary
processed food (c) Only I is implicit
(c) No single dental reimbursement plan suits the (d) Both I and III are implicit
dental health needs of all families
(d) The dental reimbursement plans offered by
12. Statement “There is very heavy traffic on
some companies do not provide for regular the road between 5 to 7 pm. We need to
twice-yearly professional cleaning of children’s have flyover in this area.”—A planning
teeth engineer said in a meeting.
(e) None of the above
260 How to Crack Test of Reasoning ANALYTICAL

Assumptions (a) People prefer coloured portaits now-a-days as


I. Heavy traffic is sought to be maintained. compared to pencil sketch portraits
II. Previous planning engineers did not do (b) With availability of good cameras at
much about heavy traffic. affordable prices, people prefer photographs
now-a-days
III. A flyover is likely to solve the problem of
(c) computer drawn portraits have option of
heavy traffic colour filling as well
(a) Only II is implicit
(d) Good artists who can draw pencil sketch
(b) Only III is implicit portraits have been giving up their
(c) Both I and II are implicit professions now-a-days.
(d) Both II and III are implicit
(e) sketches drawn by computer absolutely like
13. Study the following statement and decide the hand drawn sketches and it is difficult to
which of the given assumption(s) is implicit in make out any difference with the naked eye
the statement. SBI (PO) 2017 15. A famous singer recently won a lawsuit
Statement ‘‘Bar Council of India (BCI) has against an advertising firm for using
decided to go on hunger strike to protest the another singer in a commercial to evoke the
implementation of the Legal Services famous singer’s well-known rendition of a
Authority (Amendment) Act.’’ — Chairman of certain song. As a result of the lawsuit,

A
BCI advertising firms will stop using imitators

M
Assumptions in commercials. Therefore, advertising costs

H
A. The amendment has several loopholes and will rise, since famous singer‘s services cost

TT
is bound to hurt the litigants’ interests. more than those of their imitators.
B. All the members of BCI will welcome the The conclusion above is based on which of
decision. A
the following assumption? IBPS RRB (PO) 2017
W
(a) Most people are unable to distinguish a
C. The hunger strike held in front of the famous singer’s rendition of a song from a
court will put a pressure on the respective
SH

good imitator’s rendition of the same song


authority. (b) Commercials using famous singers are
(a) Only A (b) A and B usually more effective than commercial using
A

(c) A and C (d) All A, B and C imitators of famous singers


@

(e) None of the above (c) The original versions of some well known
songs are unavailable for use in commercials
14. The popularity of hand drawn black and white (d) Advertising firms will continue to use
pencil sketch portraits is set to decline sharply imitators to mimic the physical mannerisms
as now computer are available which can draw of famous singers
pencil sketch portraits as well. Which of the (e) The advertising industry will use well-known
following is an underlying assumption in the renditions of songs in commercials
above conclusion? IBPS (SO) 2014

Answers
Base Level Exercise
1. (d) 2. (d) 3. (c) 4. (a) 5. (c) 6. (d) 7. (d) 8. (c) 9. (a) 10. (b)
11. (a) 12. (d) 13. (c) 14. (c) 15. (b) 16. (d) 17. (a) 18. (b) 19. (d) 20. (a)
21. (c)

Expert Level Exercise


1. (b) 2. (a) 3. (a) 4. (d) 5. (d) 6. (d) 7. (c) 8. (b) 9. (a) 10. (b)
11. (b) 12. (b) 13. (e) 14. (e) 15. (e)
Chapter 22 STATEMENT AND ASSUMPTIONS 261

Answer with Explanations


Base Level Exercise
1. (d) A promise is made only after assuming that it 9. (a) Instruction in the statement is based on the
would be fulfilled. So, Assumption I is implicit. As assumption that those who lean out of a train are in
the new business ventures are to be set up in rural the risk of being hurt. It is not inferred from the
areas, so the people of these areas may welcome statement that people don’t like to get hurt. Hence,
the government decision. So, Assumption II is also only I is implicit.
implicit.
10. (b) It is an advertisement, every company
2. (d) It is expected that a large number of devotees recommends itself but it cannot be said that no
may visit the temple on main festival day. So, other company supplies the same. Also,
municipal authority is blocking the movement of advertisement are given for reading.
traffic in that area. Therefore, Assumption I is
11. (a) It is clear from the statement which is based on
implicit. People travelling to that areas may
the assumption that short fall in revenue collection
postpone their journey because of blocked
leads to an increase in fiscal deficit. So, only
movement. Therefore, Assumption II is also
assumption I is implicit.
implicit.

A
12. (d) It is clear from the statement that errors may
3. (c) The government has taken that decision after

M
occur while doing a certain work and we must try to
analysing many facts and parents never want to

H
learn from them to improve the chances of error
pay more fees. So, neither Assumption I nor II is
free work.

TT
implicit.
13. (c) From the given statement we cannot assume
4. (a) Assumption I is implicit because diversion
would be necessary when the construction is A
that the countries are working towards the control
of pollution and it is not correct to say that growing
W
going on. Assumption II is not implicit as negative
population, industries high ways and constructions
reactions are not assumed.
SH

are beyond control. Hence, neither Assumption I


5. (c) In the given statement there is no explanation nor II is implicit.
about death of rich people as it may other causes
A

14. (c) From the given statements, it is clear that the


of death so assumption I is not implicit. There is
employee needs some more skills to handling the
@

not given about the poor people. So, none of the


project more efficiently but it does not mean that
assumption follow.
right now he is incapable of handling it. Also,
6. (d) It is given in the statement that good singers nothing is said about the similar training
are invited in conference and good singers have programmes in his own country. So, neither
sweet voice. So, we could say that both Assumption I nor II is implicit.
Assumptions I and II are implicit.
15. (b) Because of good and economical train services
7. (d) It is given in the statement that students in Paris the travelling by trains is more convenient.
appearing for this examination for the first time, Hence, Assumption II is the only assumption that
should be helped. So, it is clear that candidates go with the statement.
can re-apply for this exam. Students are helped as
16. (d) In statement, it is clearly stated that school
form is somewhat complicated. Hence, both
environment should be stress and anxiety free. So
Assumptions I and II are implicit.
that right to education movement make the society
8. (c) Statement is an advice. So, it is clear that, this better.
advice is given, as many people are not humble
17. (a) Only the Assumption I is implicit as training will
after being victorians. Also, it cannot be said that
help the employees to get more knowledge about
generally people are humble.
the work and hence performance will enhance.
262 How to Crack Test of Reasoning ANALYTICAL

18. (b) In statement, it is clearly stated that ferry or 20. (a) Assumption I is implicit as a teacher will
boat can save time to reach to the island that recommend a book only if it is well-written.
means they are available to travel to Kravi Island. Assumption II is not implicit as no information is given
So, Statement II is implicit. regarding whether students can afford it or not.
19. (d) Both statements are implicit as government 21. (c) Assumption I is not implicit as it is not about
decision of switching fuel supports both the Tuberculosis. Assumption II is also not implicit as it
assumptions. is opposite to the given statement.

Expert Level Exercise


1. (b) It is assumed in the light of statement that 8. (b) The prosperity among farmers has come through
impact of economic sanctions varies from economy tractor loan, so it can be assumed that tractor is an
to economy. Only Assumption II is implicit. important input for farmers to enhance the farm
activities.
2. (a) From given statement, it is clear that by using
fodder G cattle will remain healthy, so it is clear 9. (a) We can easily assumed from the argument that
that cattles fodder G does not contain nitrate. It is in the past, children did not eat so much sugary
true that nitrate is toxic but nothing is given about processed food and hence, did not require twice
all living beings. So, only Assumption I is implicit. yearly professional cleaning to protect their tooth

A
from decay.

M
3. (a) It is clear from the statement that atleast some
farmers in village Z produce the same crop as 10. (b) The newspaper has not quoted authentic

H
farmers in village C. So, only Assumption I is sources, so I is not implicit, but II and III are implicit.

TT
implicit.
11. (b) This advertisement is given as people travel to
4. (d) The word ‘all’ makes the assumption I Europe. There are other agencies also (clear from
ambiguous. Similarly word ‘only’ in assumption II A
word ‘our agency’). But III does not clarify that after
W
makes it ambiguous. So, both assumptions are this advertisement people will go to other agencies
not true as per the statement so they are not
SH

or not.
implicit.
12. (b) Assumption I is just opposite of truth.
5. (d) Assumption I is implicit, II is not implicit. As, it
A

II is not implicit as no hint of previous planning is


is vague. III is implicit as advertisements are given. Only assumption III is implicit.
@

given with the hope that people would know the


qualities of the product and buy it. 13. (e) A is not implicit, will it hurt the litigants interest or
the lawyers’? B is not implicit because of the word
6. (d) The woman asks God to help her, if her ‘all’. C deviates by mentioning the venue.
intentions have been pure. But this does not
imply that God would not have helped her if her 14. (e) The sketches drawn by computer look absolutely
intentions were not pure. like that of hand drawn so now people do not need
Hence, only Assumption III is implicit. to go to portrait artists. They can make sketch
portraits by themselves.
7. (c) If an advertiser gives any advertisement, then
the basic assumption is that people will respond 15. (e) The advertising industry will use well known
to the advertisement. renditions of songs in commercials.
Chapter 23 STATEMENT AND CONCLUSIONS 263

23
Statement and
Conclusions
‘Conclusion’ means ‘a fact that can truly be l
If definitive words like all, always, atleast,
inferred from the contents of a given only, exactly and so on are used, then such
statement or passage’. It is an opinion or words make the conclusion invalid or

A
decision that is formed after a period of ambiguous.

M
thought or research on some facts or l
If the conclusion is provided with a stated

H
sentence stated by someone. A consequent example, then the conclusion is invalid.

TT
effect has always to be analysed before
reaching to final result or conclusion of a Types of Questions
given premise.
A
There are two types of questions based on
W
You are required to analyse the given statement and conclusion which are generally
statements, understand their direct/indirect
SH

asked in various competitive examinations.


implications and then decide which of the
given conclusions follows logically.
Type 1
A

e.g. Statement Indian president is elected


@

through secret ballot. One Statement and Two Conclusions


Conclusion Indian president is not the In this type of questions, we have one statement
supreme officer of India. and two conclusions. We have to analyse which
Here, the statement gives us the information of the conclusions support the statement.
that the President of India is elected through Directions (Illustrations 1-3) In each of the
a secret ballot which is absolutely true. Now, following questions, a statement is given followed
the conclusion given i.e., Indian President is by two conclusions numbered I and II.
not the supreme officer of India cannot be Give Answer
concluded from the statement, so it seems to (a) If only conclusion I follows
be invalid. Also, according to the constitution (b) If only conclusion II follows
it is an established fact that President is the (c) If either conclusion I or II follows
supreme head or officer of India. (d) If neither conclusion I nor II follows
Hence, the given conclusion goes against (e) If both conclusions I and II follow
the established fact, so it is invalid. Illustration 1. Statement These apples are too
Important points to be considered while expensive to be bad.
reaching on a conclusion. Conclusions
l
To reach to a conclusion think only about I. When apples are in short supply, the prices
the given information. There is no need to go up.
use, assume anything else or add any II. The higher the selling price, the superior is the
further or extra information from outside. quality of the commodity.
264 How to Crack Test of Reasoning ANALYTICAL

Solution (b) Clearly, ‘Too expensive to be bad’ Illustration 4 Statements 60% of government
means that it cannot be bad because it is employees went on strike.
expensive it means that apples with higher cost Mr. Gopal is a government employee.
are good . But conclusion I, short supply favours BSSC (CGL) 2015
rising of price is irrelevant to the statement.
Conclusions
Illustration 2. Statement Cases of bride I. Mr. Gopal went on strike.
burning for dowry are not uncommon.
II. Mr. Gopal did not participate in the strike.
Conclusions
Solution (d) Either of the situation is possible. If
I. Inspite of anti-dowry laws, the ill
Mr. Gopal was one of the member of 60% employees,
practice continues.
then he went on strike. If he was not in group of 60%,
II. The punishment inflicted on the party then he did not participate in the strike.
concerned is not sufficient. Hence, either conclusion I or II follows.
Solution (a) These cases are not uncommon
means inspite of anti-dowry laws, the ill Illustration 5 Statements
practice continues. Lawyers marry only fair girls.
Shobha is very fair. UPPSC 2014
Illustration 3. Statement Only good men
die on time. Conclusions

A
Conclusions I. Shobha is married to a lawyer.

M
I. No good people live till being old. II. Shobha is not married to a lawyer.

H
II. Every person who live till being old is Solution (d) The statement I is talking about a condition

TT
bad. with the lawyer that they marry only fair girls. But it is
Solution (d) Both conclusions have the same not talking about any condition with Shobha. So,
meaning. Means bad person live till they are A
Shobha can marry either a lawyer or anyone else.
W
old. But according to statement, only good men
die in time. Hence, neither I nor II follows. Illustration 6 Statements Amit and Subhash are
SH

friends. Subhash is friendly with all. Amit has


many enemies. Rahul and Amit do not like each
Type 2
A

other.
@

More than One Statements and Conclusions


Two/Three/Four Conclusions I. Amit, Rahul and Subhash form a clique.
II. Rahul and Subhash are friends.
In this type of questions, we deal with III. Subhash is friendly with Amit’s friends.
problems, which have more than one
IV. Amit and Rahul are both friends of Subhash.
statements and two/three/four conclusions.
We have to analyse which of the The conclusion(s) correctly drawn is/are
(a) III and IV (b) II and III
conclusions supports the statement.
(b) I and II (d) II, III and IV
Directions (Illustrations 4-5) In the Solution (d) According to the statement,
following questions two statements are given
Amit's friend → Subhash …(i)
followed by two conclusions I and II. You
have to consider the two statements to be true Subhash's friend → Amit, Rahul and friendly with all
even if they seem to be at variance from …(ii)
commonly known facts. You have to decide Rahul's friend → Subhash …(iii)
which of the given conclusions, if any follow Now from Eq. (iii), Conclusion II is correct that Rahul
from the given statements. and Subhash are friends.
Give Answer From Eq. (ii) Conclusion III is correct as Subhash is
(a) Only conclusion I follows friendly with all thus he is friendly with Amit’s friends
(b) Only conclusion II follows also. From Eqs. (i) and (ii), Conclusion IV is correct that
(c) Both conclusions I and II follow Amit and Rahul are both friends of Subhash.
(d) Either conclusion I or II follows
Chapter 23 STATEMENT AND CONCLUSIONS 265

Let us Practice
Base Level Exercise
Directions (Q. Nos. 1-4) In each of the Conclusions
following questions, a statement is followed by two I. At present, 60% villages in the State
conclusions I and II. Consider the statement and have no access to electricity at all.
the following conclusions and decide which of the II. Even after a couple of years, more than
following conclusions from the statement.
that of the villages may lack access to
Give Answer 24 h electricity.
(a) If only conclusion I follows
(b) If only conclusion II follows Directions (Q. Nos. 5-13) The questions given
(c) If either conclusion I or II follows below have a statement followed by two
(d) If neither conclusion I nor II follows conclusions I and II. Consider the statement and
(e) If both conclusions I and II follow the following conclusions and decide which of the
conclusions follows from the statement.

A
1. Statement Smoking is one of the human Give Answer

M
weaknesses, which tends to test the (a) If conclusion I follows

H
willpower of the smoker to the edge. (b) If conclusion II follows

TT
Conclusions (c) If neither conclusion I nor II follows
I. It is very difficult for the smokers to give (d) If both conclusions I and II follow
up smoking even if they want to.
A
5. Statement All the TV sets manufactured in
W
II. Human beings have other weaknesses as India, ‘Solar’ brand has the largest sale.
well.
SH

SSC (FCI) 2012, SNAP 2014


2. Statement No country is absolutely self Conclusions
A

dependent these days. I. Volume of sales of all brands of TV sets


manufactured is known.
@

Conclusions
I. It is impossible to grow and produce all II. The production of no other TV set is as
that a country needs. large as that of ‘Solar’.
II. Country’s men in general have become 6. Statement According to a recent report, a
lazy. glass of wine daily for women with breast
3. Statement The nation X faced the increased cancer could boost the success rate of
international opposition due to its decision to treatment. MAT 2012
go on with eight nuclear explosions. Conclusions
Conclusions I. Women taking a glass of wine daily will
I. The citizens of the nation have favoured never suffer from breast cancer.
the decision. II. A glass of wine will cure the women
II. Some powerful nations don’t want others suffering from breast cancer.
to become powerful. 7. Statement Sick people need medicine.
4. Statement At present 10% of our villages Conclusions SSC (CGL) 2013
receive 24 h electricity. In the next 2 yr, the I. Healthy people do not need medicine.
State government would strive to add 16% II. People keep medicine in their home.
more villages to this list. SBI Associates (PO) 2014
266 How to Crack Test of Reasoning ANALYTICAL

8. Statement The best way to escape from a Directions (Q. Nos. 14-20) In each of the
problem is to solve it. MAT 2013 following questions, a statement is given followed
Conclusions by two conclusions I and II. Consider the statement
I. Your life will be dull, if you do not face and the following conclusions and decide which of
a problem. the conclusions follows from the statement.
II. To escape from problems, you should Give Answer
(a) If only conclusion I follows
always have some solutions with you. (b) If only conclusion II follows
9. Statement India’s economy is dependent (c) If either conclusion I or II follows
mainly on forests. MAT 2013 (d) If neither conclusion I nor II follows
(e) If both conclusions I and II follow
Conclusions
I. Trees should be preserved to improve 14. Statement The Prime Minister emphatically
the Indian economy. stated that this government will make every
possible effort for the upliftment of poor farmers
II. India wants only maintenance of forests and farmhands.
to improve economic conditions.
Conclusions
10. Statement This world is neither good nor I. Except poor farmers and farmhands, all
evil, each man manufactures a world for others have got benefits of fruits of

A
himself. MAT 2013 development.

M
Conclusions II. No serious efforts have been made in the

H
I. Some people find this world quite good. past for upliftment of farmers.

TT
II. Some people find this world quite bad. 15. Statement Company X has a record in making
cameras with quality and moder- nisation to
11. Statement I know nothing except the fact
of my ignorance. MAT 2013 A
ensure that you do not lose even a single shot
W
despite bad weather.
Conclusions
SH

Conclusions
I. Writer ‘s knowledge is very poor.
I. No other company has any recognition in
II. The world of knowledge is too vast to
A

camera industry.
be explored by a single person.
@

II. Any common man can take good


12. Statements The increasing population of photographs using camera of Company X.
our nation will lead to depletion of many 16. Statement Interview panel may select a
essential resources. SSC (CPO) 2013 student who is neither possessing the abilities of
Conclusions desired level nor any value and assumptions.
I. Population of our nation can be Conclusions
controlled.
I. Inclusion of experts in interview panel
II. The nation will not be able to provide a does not ensure that the selection will be
decent living to its citizens. made properly.
13. Statement The government will review the II. Interview procedure for admission has
present policy of the diesel price in view of some limitations.
further spurt in the international oil prices. 17. Statement He emphasised the need to
CAT 2011 replace the present training programme by
Conclusions other methods which will bring out the real
I. The government will increase the price merit of the managers.
of the diesel after the imminent spurt in Conclusions
the international oil prices.
I. It is important to bring out the real merit
II. The government will not increase the
of the managers.
price of diesel to maintain the proper
balance between expenditure and II. The present training programme does not
growth rate. bring out the real merit of the managers.
Chapter 23 STATEMENT AND CONCLUSIONS 267

18. Statement Company X has marked its 22. Statements


product; “Go ahead and purchase it, if I. Demonetisation has evolved need for leaning
price and quality are your considerations.” Electronic Payment System (EPS).
Conclusions II. EPS will enable people to go cashless and
I. The product must be good in quality. use plastic money for most of their
II. The price of the product must be transactions.
reasonable. Conclusions
I. EPS is quick, safe and easy way of transaction
19. Statement In case of the outstanding of money from one account to another.
candidates, the condition of previous
II. Cashless transactions will further curb the
experience of social work may be
black marketing of money in upcoming time.
waived by the Admission Committee for
(a) Only conclusion II follows
MA (social work).
(b) Both conclusions I and II follow
Conclusions (c) Neither I nor II follow
I. Some of the students for MA (social (d) Only conclusion I follows
work) will have previous experience 23. Statements
of social work.
I. For protection of Indian museums Central

A
II. Some of the students for MA (social Government is responsible.

M
work) will not have previous
II. Victoria Memorial Hall is national property.
experience of social work.

H
SSC (10+2) 2013

TT
20. Statement While presenting a stage Conclusions
show recently, the famous actor declared I. Indian museum is national property.
that he has a practice of either taking full
payment or none for his stage shows. A
II. Historical property of nation is protected by
W
the Central Government.
(a) Only conclusion I follows
SH

Conclusions
(b) Only conclusion II follows
I. The actor has taken full payment for
(c) Both conclusions I and II follow
A

his recent stage show. (d) None of the conclusions follows


@

II. The actor did not take any money for


his recent stage show. 24. Statements
I. Leaders are human beings.
Directions (Q. Nos. 21-26) In the following II. All human beings need rest.
questions two/three statements are given
followed by two conclusions I and II. You have Conclusions
to consider the statements to be true even if I. All human beings are not leaders.
they seem to be at variance from commonly II. Leaders need rest.
known facts. You have to decide which of the (a) Both conclusions I and II follow
given conclusions, if any, follow from the (b) Neither conclusion I nor II follows
given statements. Indicate your answer. (c) Only conclusion I follows
(d) Only conclusion II follows
21. Statements
I. All students in my class are intelligent. 25. Statements
I. The constitution assures the fundamental
II. Sunil is not intelligent. SSC (10+2) 2013
rights.
Conclusions
II. Parliament has right to amend the
I. Sunil is not a student of my class. constitution. SSC (CGL) 2013
II. Sunil must work hard. Conclusions
(a) Only conclusion I follows
I. Parliament included fundamental rights in
(b) Only conclusion II follows
the constitution.
(c) Both conclusions I and II follow
(d) Neither conclusion I nor II follows II. Parliament did not assure the fundamental
rights.
268 How to Crack Test of Reasoning ANALYTICAL

(a) Only conclusion I follows Directions (Q. Nos. 31-32) Read the given
(b) Only conclusion II follows statements carefully and then answer the questions
(c) Both conclusions I and II follow accordingly. IB (ACIO) 2017
(d) None of the above
31. A. Either he is happy or he is poor.
26. Statements B. He is a happy.
I. Rabindranath Tagore wrote many poems. Inference He is poor.
II. Every poet has aesthetic knowledge. (a) The inference is definitely true
(b) The inference is definitely false
III. Aesthetic is a part of axiological study.
(c) The inference is probably false or true
SSC (10+2) 2014 (d) The inference cannot be drawn
Conclusions
32. A. Those who are honest are good teachers.
I. Rabindranath Tagore did different
B. Hard working people are honest.
axiological study.
Inference Hard work is the necessary quality
II. He followed the base of logic and ethics.
of a good teacher.
(a) Only conclusion I
(a) The inference is definitely true
(b) Both conclusions I and II
(b) The inference is definitely false
(c) Only conclusion II
(c) The inference is probably false or true
(d) None of the above

A
(d) The inference cannot be drawn
27. From the statement ’Every library has

M
33. Based on the following statements, which is
books’, it follows that CGPSC 2014
the correct conclusion drawn?

H
(a) Some libraries do not have readers
Only gentlemen can become members of the

TT
(b) Books are only in library
club. Many of the members of the club are
(c) Libraries are meant for books only
officers. Some of the officers have been
(d) No library is without books
A
invited for dinner.
W
(e) None of the above
(a) All the members of the club have been invited
SH

Directions (Q. Nos. 28-30) Which alternative for dinner


applies to the following statement or assumptions? (b) Some of the officers are not gentlemen
Choose the most appropriate option. CLAT 2017 (c) All gentlemen are members of the club
A

(d) Only gentlemen have been invited for dinner


28 ‘Only ignorant people believe in witchcraft,
@

is equivalent to 34. From the statement ‘Ability is poor man’s


(a) some ignorant persons are not those who wealth’, it follows that CGPSC 2014
believe in witchcraft (a) A poor man can earn wealth if he has ability
(b) all persons who believe in witchcraft are (b) A wealthy man is always able
ignorant (c) A poor man is always able
(c) there is no link between ignorance and (d) A poor man has the ability to earn wealth
witchcraft (e) None of the above
(d) no ignorant persons are those who do not Directions (Q. Nos. 35-37) Each question given
believe in witchcraft below consists of a statements, followed by three or
29 ‘There is no man that is not naturally good’ four conclusions (a), (b), (c) and (d). You have to
is equivalent to the proposition decide which of the conclusions is/are true from
(a) no men are good the given statements. Read the statement and
(b) all men are naturally good answer the correct conclusion below.
(c) some men are not naturally good 35. Statement All guilty politicians were
(d) some men are naturally good arrested. Kishan and Chander were among
30. ‘Some of the valuable books are seldom those arrested.
read’, means Conclusions
(a) all the valuable books are not read (a) All politicians are guilty
(b) some of the valuable books are read (b) All arrested people are politicians
(c) all the valuable books are read (c) Kishan and Chander were not politicians
(d) some of the valuable books are not read (d) Kishan and Chander were guilty
Chapter 23 STATEMENT AND CONCLUSIONS 269

36. Statement Ministers arrived at the public 37. Statement Many business offices located in
functions in their cars. building having two to eight floors. If a
Conclusions building has more than three floors, it has a lift.
(a) All ministers are rich Conclusions
(b) Ministers have cars (a) All floors may be reached by lifts
(c) Ministers come to the public functions (b) Only floors above the third floors have lifts
(d) Both (b) and (c) follow as minister have cars (c) Seventh floor has lift
and come to public functions (d) Second floor have lifts

Expert Level Exercise


Directions (Q. Nos. 1 and 2) In each of the 3. Examine the following statement.
following question, a statement followed by two I. None but the rich can afford air-travel.
conclusions I and II have been given. Study the II. Some of those who travel by air become sick.
conclusions based on the given statement and III. Some of those who become sick require
select the appropriate answer. treatment.
1. Statement Government has spoiled many Which one of the following conclusions can be drawn

A
top ranking financial institutions by from the above statements? UPSC (CSAT) 2013

M
appointing bureaucrats as directors of these (a) All the rich persons travel by air
institutions.

H
(b) Those who travel by air become sick
Conclusions (c) All the rich persons become sick

TT
I. Government should appoint directors of (d) All those who travel by air are rich
the financial institutes taking into
consideration the expertise of the A
4. Read the given statement and choose the
W
answer from given options.
person in the area of finance.
SH

II. The director of the financial institute Statement All scientists working in America
should have expertise commensurate are talented, some are Indian.
with the financial work carried out by Conclusions
A

the institute. CLAT 2015 I. None of the Indian scientists is talented.


@

(a) Only I follows (b) Only II follows II. Some talented Indian scientists have
(c) Both I and II follow (d) Neither I nor II follows migrated.
III. All talented scientists are in America.
2. Statement In Japan, the incidence of IV. Some Indian scientists are talented.
stomach cancer is very high, while that of
SSC (10+2) 2012
bowel cancer is very low. But Japanese (a) Only conclusion I follows
immigrate to Hawaii, this is reversed the rate (b) Only conclusion II follows
of bowel cancer increases but the rate of (c) Only conclusion III follows
stomach cancer is reduced in the next (d) Conclusions II and IV follow
generation. All this is related to nutrition.
The diets of Japanese in Hawaii are different
5. Consider the following three statements.
that those in Japan. I. Only students can participate in the race.
Conclusions II. Some participants in the race are girls.
I. The same diet as in Hawaii should be III. All girl participants in the race are invited
propagated in Japan also. for coaching. UPSC (CSAT) 2013

II. Bowel cancer is less severe than stomach Which one of the following conclusions can be
cancer. CLAT 2015
drawn from the above statements?
(a) All participants in the race are invited for
(a) Only I follows
coaching
(b) Only II follows (b) All students are invited for coaching
(c) Both I and II follow (c) All participants in the race are students
(d) Neither I nor II follows (d) None of the above
270 How to Crack Test of Reasoning ANALYTICAL

6. Consider the following statements. 9. A new apparel store has been opened by a
I. All machines consume energy. telecom company XYZ, from which
II. Electricity provides energy. customers have very high expectations.
Which of the following statements can be
III. Electricity operated machines are cheap to
inferred from the given statement?
maintain.
(An inference is something by which you can
IV. Electrically operated machines do not logically deduce something to be true based on
cause pollution. known/given premises.) Andhra Bank (PO) 2015
Which one of the following inferences can be drawn (a) Previously operated in telecom industry only
from the above statements? UPSC (CSAT) 2013 (b) Expansion is the best strategy to have
(a) All machines are run by electric energy competitive edge in the market
(b) There is no form of energy other than electricity (c) To be successful in an expansion, one needs
(c) Most machines are operated on electric energy to be a leader telecom industry
(d) Electrically operated machine are preferable to (d) XYZ has already established brand name in
use the market through its telecom business
(e) XYZ’s apparel store offers premium products
7. India’s neighbouring countries have seen a for youngsters only
surge in their imports of the metal. This gold is
then being carried across into India by 10. Read the following information and

A
smugglers. Which of the following inferences answer the question which follows.

M
can best be drawn from the above statements? ‘The upcoming marathon seems to be an

H
MPPSC 2018 impossible task for me. I can only run 5 km
at a stretch. Even if I double that I would

TT
(a) There is duty differential between India and its
neighbouring countries still not reach half way through the
(b) The tough restrictions on the metal’s imports marathon’ Rohan.
could be eased A
Which of the following statements can be
W
(c) India has a high current account deficit inferred from Rohan's statement?
SH

(d) There has been a sharp depreciation of the rupee Indian Bank (PO) 2016
8. Read the given information carefully and (a) Any runner who can complete the marathon
can definitely finish first five kilometres of
A

answer the given question.


the marathon faster than Rohan
@

Charity store do not have to charge VAT from (b) If Rohan runs four times his capacity, he
customers. Therefore, charity stores can charge would finish the marathon
lower prices as compared to the other stores, (c) Had the specified time for marathon been
which are not registered as charity. one hour, Rohan would have completed it in
two hours time
A. Charity stores pay less tax than (d) The specified distance of the upcoming
Non-Charity stores. marathon is definitely more than 20 km
B. VAT leads to increment in prices of things (e) If Rohan runs 5 more kilometres, he will
C. The stores recover their taxes from the reach exactly half way through the
customers. marathon’s specified distance

D. Since Charity stores do not charge VAT, 11. Read the following information carefully
their products are of lower quality. and answer the question that follows.
Which of the following can be inferred from the State Government imposes a monthly tax
given statements? SBI Associates (PO) 2014 on the salary of all employees. The tax
(a) None of the given statements amount varies according to the income
(b) Both C and D slab. The Central Government also imposes
(c) Only C
tax on the same income which is known as
(d) Only A
the income tax. This is against the theory of
(e) Both B and C
taxation. SBI (PO) 2014
Chapter 23 STATEMENT AND CONCLUSIONS 271

Which of the following conclusion can be (c) Training programmes for other profesions are
drawn from the above statement? not as good as teachers training programmes
(a) Tax should be imposed only once on an (d) Very high entrance exam is also one of the
individual reasons behind plunging number of applicants
(b) Income tax should not vary according to the for teachers training programmes
income slabs (e) In the years to come, the schools would face a
(c) The Central Government should not collect any crunch in terms of availability of qualified
tax on income teachers, if the salaries and working conditions
(d) A regular collection, irrespective of the income of teachers do not improve
slab, by the Central Government is the income tax 14. Read the given information and answer the
(e) Its upto an individual to pay tax or not
question.
12. The Sensex has only generated a paltry The MB Road near highway has an average
return of 9.19% during the last 20 yr, well of one pothole or patched piece of asphalt for
below the 10.46% returns generated by safe every yards of road. The primary reason for
debt products like PPF during the same the condition of the road is not age, weather
period. or rush hour traffic. It’s 18-wheeler trucks.
Which of the following inferences can be made These heavy trucks roar down this route that
from the above statement? MPPSC 2018 was never meant to carry weights above

A
(a) Investors need to time their entry and exit 40 tonnes. While regular traffic on the road

M
(b) Equity will provide fabulous returns if you averages 2 tonnes in terms of weight per
avoid timing and invest regularly through vehicle, the average weight per 18-wheeler

H
Systematic Investment Plans (SIPs) truck is 35 times this weight.

TT
(c) Equity investors have actually lost money after
doing SIP on the Sensex for full 10 yr Which of the following can be concluded from the
(d) The common man should not invest in shares
A
given statement?
NABARD Asst. Manager (Grade A) 2016
W
13. Read the following information carefully and A. The highway close to the MB Road can
SH

answer the given question. definitely carry atleast 35 times more


One of the main reasons behind the lack of number of vehicles than that possible on
A

applicants for teachers training/degree MB road.


programmes is that teachers have not
@

B. 18-wheeler trucks which are 30 tonnes


experienced any improvement in working lighter will not cause harm to the
conditions and their salaries have not kept mentioned road.
pace with salaries in other professions.
C. Restricting small vehicles while
SBI Clerk (Mains) 2016 permitting only 18-wheeler trucks may
Which of the following can be inferred from the reduce harm caused to the road.
given paragraph? D. A road is constructed considering the
(a) No direct relationship can be established type of vehicles plying on it.
between the work conditions of a particular
profession and preference for it amongst the (a) Only C
qualified candidates (b) A and C
(b) Number of applicants for teachers training (c) B and D
programmes will improve, if the salaries in (d) A and D
other professions are reduced (e) None of those given as options
272 How to Crack Test of Reasoning ANALYTICAL

Answers
Base Level Exercise
1. (e) 2. (d) 3. (d) 4. (b) 5. (a) 6. (b) 7. (a) 8. (b) 9. (a) 10. (d)
11. (b) 12. (b) 13. (a) 14. (d) 15. (d) 16. (e) 17. (a) 18. (e) 19. (e) 20. (c)
21. (a) 22. (c) 23. (d) 24. (d) 25. (d) 26. (a) 27. (d) 28. (b) 29. (b) 30. (d)
31. (b) 32. (c) 33. (d) 34. (a) 35. (d) 36. (d) 37. (c)

Expert Level Exercise


1. (c) 2. (d) 3. (d) 4. (d) 5. (c) 6. (d) 7. (a) 8. (e) 9. (d) 10. (d)
11. (a) 12. (a) 13. (e) 14. (c)

Answer with Explanations

A
Base Level Exercise

M
H
1. (e) The statement clearly means that smoking 6. (b) From the statement, we can clearly say that
is the weakness of human which tends to test ‘A glass of wine will cure the woman suffering from

TT
the will power of human at the extreme point. breast cancer’.
Hence, both the conclusion follow. 7. (a) As per conclusion I, healthy people do not need
2. (a) The second conclusion is an irrational A
medicine is right but people keep medicine in their
W
inference. But first conclusion is correct as it is home is not right because statement says nothing
SH

impossible for a country to grow and produce about conclusion II.


all that it needs. Hence, only conclusion I follows.
A

3. (d) We don’t know the citizens of nation X have 8. (b) The conclusion that follows the statement is
favoured the decision of nuclear explosion so, conclusion II because the only way to escape the
@

conclusion I does not follow. problem is to face it.


Also we don’t know the reason behind the 9. (a) Only conclusion I follows as to improvise the
opposition of nation X. Hence, none of the economy of India. One should preserve the forests.
conclusions follow from the statement.
10. (d) Both conclusions follow as it is human thinking
4. (b) Nothing is given in the statement about the which manufactures a world for himself.
villages which have no access to electricity at
all. So, conclusion I does not follow. It is clear 11. (b) Here, only conclusion II follows because this
from the statement that even after a couple of statement is a quote given by the Greek philosopher
years, more than that of the villages may lack socrates which implies that knowledge of word is too
access to 24 h electricity. vast to be explored by a single person.
5. (a) ‘Of all the TV sets manufactured in India, 12. (b) Only conclusion II follows as the nation will not be
‘Solar’ brand has the largest sale’, it can be able to provide a decent living to its citizens because
said only when volume of sales of all brands of of the increasing population of our nation.
TV sets manufactured is known.
13. (a) Only conclusion I follows from the statement as the
Thus, conclusion I follows. Nothing can be said government will increase the diesel price after the
about conclusion II as we don’t have any immense shoot up of diesel prices in International
information about the production of different
market.
brands of TV.
Chapter 23 STATEMENT AND CONCLUSIONS 273

14. (d) Given, both conclusions are not related to the 25. (d) From the given two statements it is not clear
statement, which gives the right conclusions. whether the parliament can assure the
15. (d) Clearly, companies other than ‘X’ may also fundamental right or not.
have recognition in camera industry. So, 26. (a) Only conclusion I follows as Rabindranath
conclusion I does not follow. Also conclusion II Tagore did the study of Aesthetic which is a part of
does not follow as in the statement the quality of axiological study.
camera is defined not of the cameraman. Hence,
neither conclusion I nor II follows. 27. (d) From the given statement it follows that no
library is without books.
16. (e) Both the conclusions follow the statement as
both correctly explain the given statement. 28. (b) ‘Only ignorant people believe in witchcrafts’ is
17. (a) Only conclusion I follows as the person equivalent to all persons who believe in witchcraft
emphasised the need to replace the present are ignorant.
training programme by other method, to bring out 29. (b) ‘There is no man that is not naturally good’ is
the real merit of the managers. Conclusion II is equivalent to all men are naturally good.
doubtful.
30. (d) ‘Some of the valuable books are seldom read’
18. (e) It is clear from the statement that a product of
means that some of the valuable books are not read.
company ‘X’ has good quality and is also available
at a reasonable price. Hence, both the conclusions 31. (b) Since, he is happy, so he cannot be a poor.

A
follow.
32. (c) From the statements, hard work is the quality of

M
19. (e) Both the conclusions follow. The waiver is a good teacher but it is not the only quality that he
incorporated because some of the students for MA

H
must have. There are other qualities needed by
will have previous experience and some of the him to be a good teacher. It is probably false or

TT
students will not have previous experience of true because the necessary qualities are not given
social work. here.
20. (c) As given in the condition in the statement, the A
33. (d) Clearly, only gentlemen have been invited for
W
actor either will take the full payment for his recent
show or will not take any money for the same. dinner as all members are gentlemen of the club.
SH

Hence, either of the conclusions follows. 34. (a) From the given statement, it follows that a poor
21. (a) All the students in my class are intelligent and man can earn wealth if he has ability.
A

Sunil is not intelligent. So, Sunil is not a student of


35. (d) As Kishan and Chander were arrested
@

my class.
amongst them. So, they must be guilty politicians.
22. (c) Both of the conclusions cannot be definitely
concluded from given statements. 36. (d) Both (b) and (c) follow as minister have cars
and come to public functions.
23. (d) Neither conclusion I nor II follows.
24. (d) From the given statements, 37. (c) ‘If a building has more than three floors, it has a
‘‘Leaders are human beings’’, lift’ clearly shows that seventh floor of a building
has a lift.
and ‘‘All human beings need rest’’,
‘‘Leaders need rest’’ follows.

Expert Level Exercise


1. (c) According to the statement, government has 2. (d) The statement neither propagates the diet
spoiled financial institutions by appointing of any of the countries nor compares the two types
bureaucrats as directors. This means that only of cancer. So, neither I nor II follows.
those persons should be appointed as directors 3. (d) Option (d) can be concluded from the
who are experts in finance and are acquainted with statement I, as only rich can afford air-travel.
the financial work of institute. So, both I and II So, all those who travel by air are rich.
follow.
274 How to Crack Test of Reasoning ANALYTICAL

4. (d) Both conclusions II and IV follow. These 9. (d) It is clear from the statement that the
conclusions are exist in the option (d). customers have very high expectations from
5. (c) From conclusions I, it can be concluded that ‘all XYZ company because it has already
participants in the race are students’. Since, it is given established brand name in the market through
in statement III that ‘all girls participants in the race its telecom business.
are invited for coaching’ so options (a) and (b) cannot 10. (d) It is clearly given in the statement that Rohan
be concluded. Hence, correct answer is option (c). can run only 5 km at a stretch and even he
6. (d) From statement I and II, we can conclude that doubles it i.e. 10 km. He would still not reach
electricity is one of the mode for providing energy. half way. Therefore, it is clearly shown that the
So, options (a) and (b) are ruled out. Option (c) is not specified distance of the upcoming marathon is
correct as numbers are not talked about. Statements definitely more than 20 km.
III and IV, talk about two basic advantages of 11. (a) Tax should be imposed only once on an
electrically operated machines. So, it can be inferred individual, so option (a) is correct.
that electrically operated machines are preferable to
use. 12. (a) If investors do not time their investment in
equities, it is more profitable to invest in safe
So, option (d) is correct. debt products.
7. (a) It is the duty differential that has led gold to other 13. (e) Clearly, conclusion (e) can be inferred from
destinations and its consequent smuggling. the given statement.

A
8. (e) Both (B) and (C) can be inferred from the given 14. (c) A road is constructed considering the type of

M
statement. According to the statement, stores vehicles plying on it and 18-wheeler trucks
increases their prices of things when they have to pay

H
which are 30 tonnes lighter will not cause harm
VAT. It implies that they recover their taxes from

TT
to the mentioned road.
customers.

A
W
SH
A
@
Chapter 24 STATEMENT AND ARGUMENTS 275

24

Statement and
Arguments

A
M
Argument is a fact that is being given by a

H
person to support or oppose a statement. Important Points

TT
In this type of questions, a statement Following points should be taken into consideration
concerned with an issue is given, followed by
certain arguments in favour or against that A
while choosing a strong argument.
W
l
A strong argument should give the realistic
statement. You have to identify the logical diagonosis of the situation described in the statement.
SH

correct arguments. l
A strong argument should give deep analysis of the
Generally, both the arguments are contrary to topic dealing with the statement.
A

each other and refer to the positive and l


A strong argument should relate with the
negative results of the action as mentioned in
@

statement and be supported up by facts or


the statement issue. established notions.
Arguments can be of two types l
An argument, which is based on universal truth is
• Strong Argument An argument is called always strong.
strong, if it touches the practical and real Following points should be taken into consideration
aspect of the situation as described in the while choosing a weak argument.
statement. It is backed up by reasons and facts l
Weak arguments are ambiguous in nature.
related to the situation. l
Weak arguments are too simple and sometime
• Weak Argument An argument is called weak, superfluous in nature.
if it is not directly related to the given statement l
Weak arguments are contrary to established facts.
and it does not address all the points put l
Weak arguments are those which contain the words
forward in the given statement. A weak only, definitely, the best untill, unless, will be.
argument is of minor importance or may be l
Weak arguments are those which are half explained.
related to trival aspect of the statement.
276 How to Crack Test of Reasoning ANALYTICAL

Examples given below, will give you a idea Solution (b) Only argument II is strong because
about the types the questions asked in various a common entrance test will brought a
competitive exams. uniformity in the standard of students, which is
necessary.
Directions (Illustrations 1-3) Study the
following instructions carefully and then answer the
Illustration 3. Statement Should computers
questions that follow.
be used in all possible sectors in India?
In making decisions about important questions it is
desirable that a candidate should be able to Arguments
distinguish between ‘strong’ and ‘weak’ arguments I. Yes, it will bring efficiency and
so far as they relate to the questions. accuracy in the work.
‘Weak’ arguments may not be directly related to the II. No, it will be an injustice to the
questions and may be of minor importance or may monumental human resources which
be related to the trivial aspect of the question. Each are at present underutilised.
question given below is followed by two arguments Solution (e) Use of computers does have the
I and II. You have to decide which of the arguments positive impact, as mentioned in argument I but
is a ‘strong’ argument and which one is ‘weak’ the negative impact as mentioned in argument II
argument. is also worth considering. So, both arguments are

A
Give Answer strong.

M
(a) If only I is strong
(b) If only II is strong
Directions (Illustrations 4-5) The giving

H
statement is followed by three or four
(c) If either I or II is strong
arguments. Decide which of the options is

TT
(d) If neither I nor II is strong
correct about the given arguments.
(e) If both I and II are strong
Illustration 1. Statement Should small states be A
Illustrations 4. Statement Should seniority be
W
only criterion for promotion?
formed out of bigger states in India? UKPSC (Pre) 2014
SH

Arguments Arguments
I. Yes, there will be greater administrative I. No, it would be an injustice to those
A

convenience. juniors who are more deserving and


suitable for higher position.
@

II. No, it will jeopardise the national


integration. II. Yes, otherwise seniors will feel
humiliated.
Solution (e) Clearly, if we form smaller states out of
bigger one in India, there will be of greater III. Yes, seniors are more experienced and
administrative convenience. Hence, argument I is must be rewarded.
strong. Also, it may hamper the national integration. (a) I and II
So, argument II is also strong. Hence, both arguments (b) II and III
(c) Only I
are strong.
(d) None of the above
Illustration 2. Statement Should there be only one Solution (c) Argument I is a strong argument,
entrance test for all the medical colleges—both because it is rightly said that deserving and
government and private in India? suitable persons feel injustice. At the same time,
Arguments it is necessary to give higher positions to
I. No, each college has unique requirements deserving and suitable persons to get better
and decisions on admissions should be left result.
to the concerned colleges. Argument II is a weak argument as it is illogical.
II. Yes, this will bring in uniformity in the Argument III is also weak, because experience
standard of students at the time of should not be the only criteria for promotion as
admission and will reduce multiplicity. efficiency also matter.
Chapter 24 STATEMENT AND ARGUMENTS 277

Illustrations 5. Statement Should the Illustrations 6. This question consists of an


consumption of aerated drinks be banned information and two statements numbered I and
in India? II given below it. You have to decide which of the
Arguments given statements weaken/s or strengthen/s the
I. Yes, this is the only way to reduce the given information and decide the appropriate
risk of exposing people to some answer. IBPS (SO) 2016
disease. Information : Examination committee of
College X has recommended that the negative
II. No, each individual should have right
marking in the exam should be removed.
to choose what he wants.
Statements
III. No, there is no confirmed evidence
that such products have adverse I. The students tend to give answers simply
effects on human body. by guessing when there is no negative
marking in the exam.
IV. Yes, it is banned in many countries
also. II. Selecting the best candidates from the
(a) Only I is strong
population becomes difficult in the absence
of negative marking.
(b) I and II are strong
(a) Both Statement I and Statement II weaken
(c) Only III is strong the information.

A
(d) I and IV are strong (b) Statement I strengthens the information while

M
Solution (c) Argument I is a weak argument, Statement II weakens the information.

H
because of the word ‘only’. Argument II is also (c) Statement I weakens the information while
weak argument, because it is the duty of the

TT
Statement II strengthens the information.
government to save its citizens from harmful (d) Both Statements I and Statement II are
products. Argument III is strong as a product
must not be banned unless its harmful effects A
neutral statements.
(e) Both Statements I and II Strengthen the
W
have been proved. Argument IV is weak, information.
SH

because we cannot blindly follow other Solution (a) Both Statements I and II weakens the
countries. information.
A
@

Let us Practice
Base Level Exercise
Directions (Q. Nos. 1-16) Study the following instructions carefully and then answer the questions
that follow.
In making decisions about important Give Answer
questions, it is desirable that a candidate (a) If only I is strong
is able to distinguish between ‘strong’ and (b) If only II is strong
‘weak’ arguments so far as they relate to (c) If either I or II is strong
the questions. ‘Weak’ arguments may not (d) If neither I nor II is strong
be directly related to the trivial aspect of (e) If both I and II are strong
the question. Each question below is 1. Statement Are nuclear families better than joint
followed by two arguments numbered families?
I and II. You have to decide which of the Arguments
arguments is a ‘strong’ argument and
I. No, joint families ensure greater security.
which is a ‘weak’.
II. Yes, nuclear families ensure greater freedom.
278 How to Crack Test of Reasoning ANALYTICAL

2. Statement Should strikes in the field of 8. Statements Should parents invest as


education be banned? much in educating their daughters as
Arguments much as they spend on educating their
I. Yes, it is against professional ethics. sons? UP Police (Constable) 2018
Arguments
II. Yes, it affects the students adversely.
I. No, almost all data points to the fact
3. Statement Should school education be made that boys are way more intelligent
free in India? OPSC 2018 than girls.
Arguments II. No, though girls may be intelligent,
I. Yes, this is the only way to improve the parents have to keep money aside for
level of literacy. their marriages.
II. No, it should add to the already heavy 9. Statements Should a cricket team have
burden on the exchequer. more than one captain during a match?
UP Police (Constable) 2018
4. Statement Should foreign films be banned in
India? Arguments
Arguments I. No, one needs to make decisions on the
spot and there won't be time to resolve
I. Yes, they depict an alien culture which

A
conflicting ideas between the captains
adversely affects our values. on the field if such a scenario emerges.

M
II. No, foreign films are of a high artistic II. Yes, it is always better to have more

H
standard. brains coming to an understanding

TT
5. Statement Should the sex determination during before taking a decision.
pregnancy be completely banned? 10. Statement Should one year of army
Arguments A
training be compulsory for all Indian
W
I. Yes, this leads to discriminate female citizens? SSC (CGL) 2017
SH

foeticide and eventually will lead to social Arguments


imbalance. I. No, the costs of training will be
II. No, people have the right to know about prohibitive and one year of labour
A

their unborn child. will be lost.


@

6. Statement Should India support all the II. Yes, army training helps make better
international policies of USA? citizens.
Arguments 11. Statement Should the opinion polls
I. No, many other powerful countries don’t predicting outcome of elections before the
support the same. elections be banned in India?
II. Yes, this is the only way to gain USA Arguments
development funds. I. Yes, this may affects the voters mind
7. Statements Should a child from a and may affect the outcome.
not-so-affluent family in India be allowed to II. No, such polls are conducted all over
follow his/her passion? UP Police (Constable) 2018 the world.
Arguments: 12. Statement Should persons convicted of
I. Yes, these days if a child is allowed to criminal offences in the past be allowed to
pursue her/his interests, she/he can excel contest elections in India?
in it, even financially. Arguments
II. No, even today our country does not have I. No, such persons cannot serve the
a strong social security system and cause of the people and country.
establishing oneself in unconventional
II. Yes, it is democracy—Let people
fields is time-consuming and can be
decide whom to vote.
financially taxing
Chapter 24 STATEMENT AND ARGUMENTS 279

13. Statement Should the provision to dissolve Arguments


the assembly prematurely be amended? I. Yes, this is the only way to teach
MPPSC (Pre) 2014 discipline to the employees.
Arguments II. No, government cannot deprive its
I. Yes, on many occasion the provision has citizens of their basic rights.
been used by ruling governments to III. Yes, this is the only way to maximise
fulfil vested interests. productivity without disruption of
II. No, to fulfil the constitutional obligations work.
and norms, it sometimes becomes the (a) Only argument I is strong
need of the hour to dissolve the (b) Only argument II is strong
assembly prematurely. (c) Only argument III is strong
14. Statement Should only reputed NGO's be (d) None is strong
authorised to distribute the commodities to 18. Statement Should there be a complete ban
the public under the programme of Public on celebration of various’ ‘day’ in colleges?
Distribution System (PDS)? MPPSC (Pre) 2014 Arguments
Arguments
I. No, there is nothing wrong in celebrating
I. Yes, the move will be helpful to
the days and enjoying once in a while.

A
implement the programme more
II. Yes, children are giving more importance

M
effectively and will keep a tab on
various problems like black marketing of to such celebrations than the studies.

H
the commodities supplied under PDS. III. No, this type of celebration gives

TT
II. Yes, NGO's have helped government on opportunity for children to express their
many occasions. feelings.
15. Statement Should the private companies be A
(a) Arguments I and II are strong
W
allowed to operate passenger train services (b) Arguments II and III are strong
SH

in India? MPPSC (Pre) 2014 (c) Only argument III is strong


Arguments (d) Only argument II is strong
(e) None of the above
A

I. Yes, this will improve the quality of


service in Indian Railways as it will have
@

19. Decision : The authorities of cricket team of


to face severe competition. country B decided to replace the captain of the
II. No, the private companies may not agree team with a relatively new player for the
to operate in the non-profitable sectors. upcoming series to be played against country G.
16. Statement Should open book examination I. Most of the strategies of the old captain
be introduced for professional courses in failed to help the team with the series
India? MPPSC (Pre) 2014 held in the past two years.
Arguments II. The captain of the team was found guilty
I. No, it will not seriously improve the of ball-tampering in one of the major test
value and importance of present series held during recent time.
Indian Bank PO (Pre) 2017
examination system.
II. Yes, all candidates can pass easily and (a) Both Statement I and Statement II weaken the
can start their professional life. decision
(b) Statement I Strengthens the decision while
Directions (Q. Nos. 17and 18) In the statement II weakens the decision.
following questions, a statement is given followed (c) Both Statements I and II are neutral statement.
by three/four arguments. Study the statement (d) Statement I weakens the decision while
carefully and mark the appropriate alternative. Statement II strengthens the decision.
17. Statement Should the government ban all (e) Both Statements I and II strengthen the
forms of protests including strikes and decision.
processions? SSC (10+2) 2011
280 How to Crack Test of Reasoning ANALYTICAL

Expert Level Exercise


Directions (Q. Nos. 1-5) Study the following II. No, let us first solve problems of our own
instructions carefully and then answer the people like poverty and malnutrition.
questions that follow. 4. Statement Are the fabulous prices demanded
In making decisions about important questions, by the art dealers for the original paintings of
it is desirable that a candidate is able to old masters justified?
distinguish between ‘strong’ and ‘weak’ Arguments
arguments so far as they relate to the questions.
I. Yes, those are unattainable antique pieces
Each question below is followed by two
of art, hence the price for the collection of
arguments numbered I and II. You have to
art is justified.
decide which of the arguments is a ‘strong’
argument and which is a ‘weak’. II. No, modern painters can paint as well, if
not better than them and for much less
Give Answer price.
(a) If only I is strong
(b) If only II is strong 5. Statement Should all professional colleges in
(c) If either I or II is strong India be encouraged to run their own courses

A
(d) If neither I nor II is strong without affiliations to any university?
(e) If both I and II are strong

M
Arguments
1. Statement Should there be only one rate I. Yes, it is the only way to create more

H
of interest for term deposit of varying opportunities for those who seek

TT
duration in banks? professional training.
II. No, this will dilute the quality of
Arguments
I. No, people will refrain from keeping A
professional training as all such colleges
W
may not be equipped to conduct such
money for longer duration resulting courses.
SH

into reduction of liquidity of the


banks. Directions (Q. Nos. 6-10) In the following
A

II. Yes, this will be much simple for the questions, a statement is given followed by
three/four arguments. Study the statement
@

common people and they may be


encouraged to keep more money in carefully and mark the correct alternative.
banks. 6. Statement Should the government impose
2. Statement Should the oil companies be restrictions on access to sensitive information
allowed to fix the price of petroleum to the journalists to avoid the media hype?
products depending on market conditions? Arguments
Arguments I. Yes, the media creates hype and publishes
distorted information at times.
I. Yes, this is the only one way to make
the oil companies commercially viable. II. No, journalists should have an access to all
the information as media is the best
II. No, this will put additional burden on source to expose the malfunctions in the
the retail prices of essential society.
commodities and will cause a lot of
hardships to the masses. III. Yes, at times it leads to harassment of
those who are affected and alleged to be
3. Statement Should India become a involved in the crisis.
permanent member of UN’s Security
(a) All I, II and III are strong
Council?
(b) Arguments I and II are strong
Arguments (c) Arguments II and III are strong
I. Yes, India has emerged as a country (d) Only argument II is strong
which loves peace and amity. (e) None of the above
Chapter 24 STATEMENT AND ARGUMENTS 281

7. Statement Should the government introduce a (a) Only argument I is strong


system of obtaining bond from students for (b) Arguments I and III are strong
working in India before sanctioning (c) Arguments III and IV are strong
education loan for higher studies? (d) Arguments I and IV are strong
Arguments (e) None of the above
I. No, this is not a workable solution and 10. Statement Should all the school teachers be
will obstruct the development of young debarred from giving private tuitions?
talent in the country. Arguments
II. Yes, this is the only way to ensure use of I. No, the needy students will be deprived
the talent of our country for the of the expertise of these teachers.
development of the country and not II. Yes, this is an injustice to the
only an individual. unemployed educated people who can
III. No, this step will be too harsh. earn their living by giving tuitions.
(a) Only argument I is strong III. Yes, only then the quality of teaching in
(b) Only argument II is strong schools will improve.
(c) Arguments I and II are strong
IV. Yes, now salary of these teachers is
(d) None is strong
reasonable.

A
(e) None of the above (a) Arguments I and III are strong

M
8. Statement Should people with educational (b) Arguments I, II and III are strong
qualification higher than the optimum (c) Arguments III and IV are strong

H
requirements be debarred from seeking jobs? (d) Arguments II, III and IV are strong

TT
Arguments (e) None of the above
I. No, it will further aggravate the problem
of educated unemployment. A
Direction (Q. No. 11) Read the following
W
information carefully and answer the given
II. Yes, it creates complexes among question. SBI (PO) 2013
SH

employees and affects the work Despite repeated announcements that mobile
adversely. phones are not allowed in the examination hall,
A

III. No, this goes against the basic rights of three students were caught with their mobile
the individuals.
@

phones.
IV. Yes, this will increase productivity A. Mobile phones now-a-days have a lot of
(a) Argument III is strong features and it is easy to cheat with their help.
(b) All are strong
B. The invigilator must immediately confiscate
(c) Arguments II and IV are strong
the mobile phones and ask the students to
(d) None of the above
leave the exam hall immediately.
9. Statement Should all the youngsters below C. Mobile phones are very expensive and
21 yr of age be disallowed from going to a leaving them in bags outside the exam hall is
beer bar? not safe.
Arguments D. There have been incidents where students
I. No, it is not correct to prevent matured who left the exam hall early stole the mobile
youngsters above 18 yr of age who can phones kept in the bags of the students who
vote, from having fun. were writing the exam.
II. Yes, the entry fee to such pubs should 11. Which of the following among A, B, C and D
also be hiked. may be a strong argument in favour of the
III. No, there is no such curb in Western three students who were caught with the
countries. mobile phone?
IV. Yes, this will help in preventing (a) Only A (b) Both A and B
youngsters from getting into bad (c) Both C and D (d) Only C
company and imbibing bad habits. (e) Both B and D
282 How to Crack Test of Reasoning ANALYTICAL

Direction (Q. No. 12) Read the following (c) Many online shopping portals offer the ‘cash on
statements carefully and answer the given delivery’ feature which is for those who are
questions. IBPS (PO) 2014
sceptical about online payments
(d) Many consumers prefer shopping at retail stores
12. Cocoas and chocolate products have been which are nearer to their houses
used as medicine in many cultures for (e) In online shopping the customer may be deceived
centuries. Chocolate is made from plants as he cannot touch the product he is paying for
which means it contains many of the
health benefits of leafy vegetables. Which Direction (Q. Nos. 14) The given information is
of the following statements weakens the followed by two statements. Read carefully and
above arguments? answer the given questions. IBPS (PO) 2014
A. Dark chocolate contains a large 14. Many parents have written a plea to the
number of antioxidants which slow administration department of school X to
downs the aging process. discontinue the rule of wearing ties to school.
B. A small study revealed that regular A. The school has kept different coloured ties
intake of chocolate increase insulin for different academic scorers as part of
sensitivity thus, lowering the chances their uniform. Thus, the low scoring
of diabetes. children of the school feel discriminated.

A
C. Green leafy vegetables have B. The sport uniform of the school does not
have a tie which is to be worn only on

M
flavonoids which protect skin from
UV rays. Wednesday.

H
(a) Statement A weakens but statement B
D. Chocolates have 3 types of fats one

TT
strengthens the argument
out of which increase the cholesterol (b) Both statement A and statement B weaken the
level.
E. Cocoas increases blood flow to the A
argument
W
(c) Statement B weakens but statement A
retina thus giving a boost to vision. strengthens the argument
SH

(a) Only D (d) Both statement A and statement B strengthen


(b) A and E the argument
A

(c) Only C (e) Statement A strengthens the argument and


statement B is a neutral statement
@

(d) None of the given statements


(e) Both C and D 15. Read the given information and answer the
question.
Direction (Q. No. 13) Read the following
information carefully and answer the given Company Z and company F launched similar
question. SBI (PO) 2013 high end cars last year. However, six months
The convenience of online shopping is what I later it was found that the popularity of car
like best about it. Where else can you shop manufactured by company Z sky-rocketed
even at mid-night wearing your night suit? while the sales of the one manufactured by
You do not have to wait in a line or wait till the company F did not pick up at all. Manager X:
shop assistant is ready to help you with your “The only reason our car did not succeed was
purchases. It is a much better experience as the price of the car. Had our car been priced at
compared to going to a retail store. a little lower value, the sales of our cars would
—A consumer’s view also have been as good as the one
13. Which of the following can be a strong manufactured by company Z”.
argument in favour of retail store owners? Which of the following weakens the statement of
(a) Online shopping portals offer a great deal of manager X? NABARD Asst. Manager 2016
discounts which retail stores offer only
(a) Experts have suggested that there is always a
during the sale season
segment of population which reviews the car after
(b) One can compare a variety of products it has been launched and purchases it only after a
online which cannot be done at retail stores certain amount of time has passed after the launch
Chapter 24 STATEMENT AND ARGUMENTS 283

(b) A few potential buyers had a problem with the (d) The car launched by company F is available in
fact that the only two air bags while the one limited colours, although the owners can
manufactured by company F had six request for the colour of their choice
(c) In the past two years, all cars launched by (e) A few other cars launched by company F
company F were of mediocre quality and several years ago had been priced lower than
needed frequent repairs and maintenance thus most other cars in the market and they worked
tarnishing the name of the company to a great very well
extent

Answers
Base Level Exercise
1. (e) 2. (e) 3. (b) 4. (d) 5. (a) 6. (d) 7. (a) 8. (d) 9. (a) 10. (a)
11. (a) 12. (a) 13. (c) 14. (a) 15. (a) 16. (d) 17. (b) 18. (b) 19. (e)

Expert Level Exercise

A
1. (a) 2. (b) 3. (a) 4. (a) 5. (b) 6. (a) 7. (a) 8. (a) 9. (a) 10. (e)

M
11. (c) 12. (a) 13. (e) 14. (e) 15. (c)

H
TT
Answer with Explanations A
W
SH

Base Level Exercise


1. (e) There is more security in joint family as there 7. (a) Only argument I is strong. These days if a child
A

are other members to help. Also in nuclear is allowed to pursue her/his in terests, she/he
@

families, there are lesser persons, so lesser can excel in it, even financially.
responsibilities and more freedom. Hence, 8. (d) In the given statement it is not mentioned about
both arguments are strong. intelligence of girls and boys. So, none of the
2. (e) Strikes are against the professional ethics, also arguments is strong.
affects the students adversely. 9. (a) Every Person can have different opinion. So, to
3. (b) Argument I is not strong as it contains ‘only’, make a decision on the spot it may need more
while argument II is strong as such step would time. Hence, only argument I is strong.
require immense funds and will lead to financial 10. (a) As per the given statement, if one year of army
drain. training be compulsory for all Indian citizens,
4. (d) Foreign films depict the alien culture but they then it is not so beneficial because the cost of
also help in learning. Hence, both arguments are training will be prohibitive and one year of
not strong. labour will be lost. It is not necessary to join
army to be a better citizen.
5. (a) This determination is done as parents mostly
want a boy child. Also, people who have a right Hence, only argument I is strong.
to know, they should only know about the health, 11. (a) The opinion polls may influence the thinking of
development and general well being of the child an individual and thus, divert his mind. Also,
before its birth. Hence, argument I holds. blindly imitating a policy followed by other
6. (d) Word ‘only’ makes argument II vague and countries holds no relevance. Hence, only
argument I is the example of other countries. argument I holds.
Hence, neither argument I nor II holds. 12. (a) Persons with criminal background cannot stand
to serve as the representatives of the common
284 How to Crack Test of Reasoning ANALYTICAL

people, so they should not allowed to contest 16. (d) Argument I is not strong as professional courses
elections. Hence, only argument I holds strong. do not need knowledge of books. While
13. (c) Either argument I or II is strong because many Argument II is of little importance and hence can
times ruling government misuse it while be rejected.
sometimes dissolving the assembly becomes 17. (b) Word ‘only’ makes argument I and argument III
the need of the hour. vague. So, argument II is strong.
14. (a) Argument I is strong because it will reduce the 18. (b) Arguments II and III hold strong because at times
problem of black marketing of commodities student pays more attention in celebrating these
supplied under PDS. day which disturb their studies but these days are
argument II is not strong because it is not important also because they provide them with a
directly related with the statement.
stage to express their feelings about various
15. (a) Argument I is strong because competition things and aspects in life.
increases the quality of service. From the
19. (e) Clearly, both the statements strengthen the
statement, it is not clear that railways is a
non-profitable sector. So, argument II is not decision.
strong.

Expert Level Exercise

A
M
1. (a) If we apply the proposed scheme, people will criteria. Arguments I, II and IV are vague hence

H
not deposit their money for a longer period and are weak argument.
thus liquidity with the bank will fall drastically.

TT
9. (a) Argument I is strong as it states the genuine
So, only argument I holds.
reason against the ban. Arguments II, III and IV
2. (b) We know that, oil is an essential commodity and
its prices govern the prices of other essential the cause. A
are weak as they any how does not support
W
commodities. Also, the common people must
10. (e) All four arguments are weak because either they
SH

be taken care of. Hence, argument II holds


strong. are vague or they are less important for such
casue.
3. (a) Clearly, India seeks to bring different nations
A

on friendly terms with each other. Also, internal 11. (c) The strong arguments are C and D. As, mobile
@

problems of a nation should not discourage it phones are very expensive and leaving them in
from strengthening international relations. bags outside the exam hall is not safe. And there
Hence, only argument I holds strong. have been incidents where students who left the
4. (a) Fabulous prices for old master prices are exam hall early stole the mobile phones kept in
demanded because of their originality and the bags of the students who were writing the
innovative hard effects. exam.
5. (b) Argument I is not strong as it contains the word 12. (a) Chocolates have three types of fats, one out of
only. Argument II is strong. which increase level of cholestrol weakens the
6. (a) All the three arguments are strong. arguments because it shows negative quality of
chocolates. Hence, only statement D weakens
7. (a) As India is democratic country and every
the given argument.
person has its right about where he wants to
work and singing such bonds does not ensure 13. (e) We know that, in online shopping the customer
the correct use of talent of a country. So, cannot touch the product he is paying for.
argument I is strong but argument II is weak
14. (e) Classification of wearing tie as per academic
also argument III is a vague and hence is weak
scores is the cause of discrimination of the low
argument.
scorer, so statement A strengthens the argument
8. (a) Only argument III holds strong because this best while statement B is neutral in nature.
goes against the basic rights as an individual
15. (c) Option (c) weakens the statement of manager X.
can seek any Job if he fulfills the eligibility
Chapter 25 STATEMENT AND COURSE OF ACTION 285

25
Statement and
Course of Action
Course of Action is the feasible step to be Examples given below, will give you a better

A
taken after an event happened or is about to idea about the types of questions asked in

M
happen. various competitive exams.

H
In this type of questions, a situation is Directions (Illustrations 1-2) Each

TT
presented and some courses of action are Illustration given below has a statement followed
suggested in the context of that situation.You by two courses of action numbered I and II. You
have to identity the correct course of action,
which either reduce the problem or improve A
have to assume everything in the statement to be
true and on the basis of the information given in
W
the situation, which is created by that the statement, decide which of the suggested
SH

situation. Course of action should be feasible courses of action logically follows for pursuing.
and should relate with the practical aspect of Give Answer
A

life. (a) If only I follows (b) If only II follows


@

(c) If either I or II follows (d) If neither I nor II follows


Basic Points to be Considered (e) If both I and II follow
while Choosing a Correct
Illustration 1. Statement
Course of Action If the retired professors of some institutes are
To solve these questions, the candidates are also invited to deliberate on restructuring of
advised to keep in mind the following important the organisation, their contribution may be
points beneficial to the institute.
l
Simple problem must have simple course of Courses of Action
action and not a complicated one which may I. Management may seek opinion of the
create more problems than to solve or reduce it. employees before calling retired
l
The correct course of action should be a professors.
positive step towards the solution of the II. Management should involve
problem rather harsh and undemocratic. experienced people for the systematic
l
If any course of action is followed with word restructuring of the organisation.
‘only’ it makes action weak. Solution (b) We know it very clearly that retired
l
In most of the cases, a situation has more professors have lots of experience and can give
than one courses of action, but they are valuable suggestions. Hence, management should
never exclusive to each other. So, the answer involved such experienced people. Seeking
should be both follow or not either of them permission from the employees is not the right
follows. course of action.
286 How to Crack Test of Reasoning ANALYTICAL

Illustration 2. Statement Statement Nuclear power cannot make a


Incessant rain for the past several days has country secure.
posed the problem of overflowing and flood as Courses of Action
the river bed is full of silt and mud.
I. We should stop further increasing our
Courses of Action nuclear power.
I. The people should be made aware about II. We should destroy out nuclear capability.
the eminent danger over radio/ television. III. We should focus on improving diplomatic
II. The silt and mud from the river bed relations. UKPSC (Pre) 2014
should be cleared immediately after (a) Only I follows
receding of water level. (b) Only II follows
Solution (e) Each of the courses will provide relief to (c) Only III follows
the victims. Hence, both the given courses of (d) I and II follow
action will improve the solution. Solution (c) Action I is not a good action, because
Illustration 3. Consider the following statements nuclear power can be used in power generation.
and courses of action and decide which Destroying nuclear capability will raise another
course/courses of action logically problem, so II is also not good. Action III is a good
follows/follow for pursuing. option to secure a country.

A
M
Let us Practice

H
TT
Base Level Exercise A
W
SH

Directions (Q. Nos. 1-16) Each question given II. More CCTV cameras should be installed
below has a statement followed by two courses of near the dining and reception areas of
A

action numbered I and II. A course of action is a the hotel where these incidents took place.
@

step or administrative decision to be taken for


2. Statement Local villagers have reported
improvement, follow up or further action in
that instances of illegal cutting of trees have
regard to the problem, policy, etc on the basis of
increased over the last few months in the
the information given in the statement. You have
forest area.
to assume everything in the statement to be true,
Courses of Action
then decide which of the two suggested courses of
action logically follows for pursuing. I. The local villagers should be encouraged
to report any such activities in the future
Give Answer as well.
(a) If only I follows
(b) If only II follows II. Authorities should immediately look into
(c) If either I or II follows the matter and put a stop to such illegal
(d) If neither I nor II follows activities.
(e) If both I and II follow 3. Statement Most of the children in India are
1. Statement Four cases of pick pocketing not able to get education, because they get
were reported at one of the most renowned employed to earn livelihood in their
five star hotels last evening. childhood only.
Courses of Action Courses of Action
I. The hotel staff should be instructed to be I. Education should be made compulsory
vigilant and report any suspicious person for all children upto the age of 14.
or activity. II. Employment of children below the age of
14 yr should be banned.
Chapter 25 STATEMENT AND COURSE OF ACTION 287

4. Statement Most of the development plans are 10. Statement People see tax as a burden and
on paper only. thus devise ways to underpay or avoid it
Courses of Action altogether.
I. The incharges should be instructed to Courses of Action
supervise the field work regularly. I. Government should educate and
II. The supply of paper to such department inform citizens about the ways in
should be cut short. which taxes help in development of
the nation.
5. Statement Annual results of school G are II. Tax rates should be increased so that
continuously falling from past three years as the under-recovery in collection is
the school is unable to hire adequate number of compensated.
teachers due to lack of funds.
Courses of Action 11. Statement Despite repeated warnings to
I. School G should increase the fees of students and parents from the college,
students by 30% in order to generate funds some students have finally not fulfilled the
for hiring new teachers. mandatory criteria of 75% attendance in
order to appear for exams.
II. Classes of school G should be divided into two
shifts and should be continued with the Courses of Action

A
existing teachers. Andhra Bank (PO) 2015 I. The college should stop adhering to

M
this particular criteria.
6. Statement Many villages face flood fury every

H
II. Either the parents or guardians of the
year in the monsoon.

TT
defaulters should be called for a
Courses of Action CGPSC 2018
meeting.
I. Timely evacuation of the people is
necessary. A
12. Statement The authorities of ‘Emperor’s
W
II. The government should take measures to Garden’, a famous tourist spot in City F,
SH

control floods by building dams etc. which is open for public visit free of cost
for two months in a year, have complained
7. Statement The police harass the common about the cleanliness issue in the garden
A

man. and damage to property caused by the


@

Courses of Action public. Canara Bank (PO) 2018


I. The government should take firm steps Courses of Action
against the erring policeman. I. Authorities should restrict entry to a
II. The police force should be educated for certain age group only.
public dealing. CGPSC 2018 II. All those found damaging the
8. Statement School uniform is a burden on the property should be penalised in
poor parents. monetary terms.
Courses of Action CGPSC 2018
13. Statement The sales of ball-point pens
I. Poor parents should stop buying school manufactured by company Lixus have
uniform. gone down considerably ever since the
II. Schools can provide subsidy on school same company introduced a gel-ink pen
uniforms. in the market.
9. Statement Trade Unions slow down the Courses of Action
industrial production. I. Ball-point pens should not be
Courses of Action CGPSC 2018 manufactured by Lixus any more.
I. Trade Unions should be banned. II. Lixus should immediately withdraw
II. Industry should employ persons on all gel-ink pens from the market so as
temporary basis. to force people to buy ball-point pens.
UBI (PO) 2011
288 How to Crack Test of Reasoning ANALYTICAL

14. Statement The police department has Courses of Action


come under a cloth with recent I. All the owners should strictly give the
revelations that atleast two senior police responsibility of their properties to only
official are suspected to have been one broker
involved in the illegal sale of a large II. The brokers should be instructed to
quantity of weapons from the state police mandatorily disclose the list of all the
armoury. SBI (PO) 2014 properties they will be showing the
Courses of Action customers on a particular day before taking
I. A thorough investigation should be them to the actual site.
ordered by the State Government to Directions (Q. Nos. 17 and18) Each question
bring out all those who are involved in contains one statement and two courses of action I
the illegal sales of arm. and II. Assuming the statements to be true, decide
II. State police armoury should be kept which of the two courses of action most logically
under Central Government’s control. follow. CLAT 2013

15. Statement In a competitive exam held Give Answer


(a) If only I follows
across country Banimia, more than 8 lakhs (b) If only II follows
candidates appeared. But, about 4000

A
(c) If either l or II follows
candidates were found to be using unfair (d) If neither I nor II follows

M
means, 75% of which were from State K of
17. Statement Despite of child labour laws,

H
the country. RBI 2016
children can be seen working in hotels, shops,

TT
Courses of Action
houses very frequently.
I. Government of ‘Banimia’ should
Courses of Action
improve inspection and invigilation in A
I. The Government should not make such
W
all test centres of State K for all
competitive exams. laws which cannot be enforced.
SH

II. From now on for any competitive II. A proper education system for the primary
exams in Banimia, any application level particular for lower caste community
A

from State K should not be entertained. may eradicate this problem.


@

16. Statement Most of the people looking for 18. Statement School dropout rate is very high
buying/renting properties these days in the rural areas as children support their
complain of being taken to the same parents in income earning activities.
property by more than 6-7 brokers. So, Courses of Action
even after contracting multiple agents, I. Public awareness programme on primary
they end up having usually the same education should be expanded
options. RBI 2016 immediately to educate parents.
II. Compensation is not a remedy.

Expert Level Exercise


1. Below is given a statement followed by II. This problem should be raised in the
three courses of action. Parliament.
Statement There is significant increase in III. Hospitals in the city should be equipped
the number of patients affected by some properly for the treatment of patients.
disease in a city. CGPSC 2014 Which of the following course(s) of action logically
Courses of Action follows(follow)?
(a) I and II follow (b) Only III follow
I. This problem should be raised in the
(c) I and III follow (d) All follow
Municipal Corporation of the city.
(e) None of the above
Chapter 25 STATEMENT AND COURSE OF ACTION 289

Directions (Q. Nos. 2-7) In each question Courses of Action


below is given a statement followed by three I. All such patients who are suffering from
courses of action numbered I, II and III. malaria should immediately be discharged
Assuming the statements to be true, decide from the hospital.
which of the three courses of action most II. The hospital authority should immediately
logically follows. put a ban on admitting new patients into
2. Statement There has been a continuous the hospital.
increase in the number of dropout of III. All such patients who are suffering from
students of Government run primary malaria should be kept in an isolated ward.
schools in the state. SBI (PO) 2014 (a) Only I follows (b) Only II follows
Courses of Action (c) Only III follows (d) None follows
I. Government should immediately set (e) None of these
up a committee to review the situation 5. Statement Many public sector undertakings
and suggest measures to reverse the have been making losses for the past few years
trend. and the situation is equally bad in the current
II. Government should conduct year.
orientation programmers for parents of Courses of Action

A
the students, emphasizing on the need I. These loss making public sector companies

M
for educating their children. should immediately be closed down.
II. The government should scout for potential

H
III. Government should close down such
state-run primary schools where buyers in the private sector to sell these

TT
dropout rates are more than fifty per companies to get back part of the
cent. investments made by the government.
(a) Only I follows (b) Only II follows A
III. All the employees of these companies
W
(c) Only III follows (d) I and II follow should be retrenched with adequate
SH

(e) None of the above compensation and the fixed assets may be
3. Statement Every year during monsoon, the put up for sale.
(a) I and II follow (b) II and III follow
A

condition of most of the roads in the city


(c) All I, II and III follow (d) None of the above
@

deteriorates causing immense problem to


the commuters. SBI (PO) 2014 6. Statement Many shops in the local market
Courses of Action have extended their shops and occupied most
I. The civic body should include a heavy part of the footpath in front of their shops.
penalty clause while awarding future Courses of Action
contracts for road repairs. I. The civic authority should immediately
II. The civic officials in charge of activate a task force to clear all the
maintenance of city roads should be footpaths encroached by the shop owners.
asked to explain why the condition of II. The civic authority should charge hefty
the roads worsens every year. penalty to the shop owners for occupying
III. General public should avoid taking the footpath.
their vehicles out during monsoon. III. The civic authority should set up a
(a) Only I follows monitoring system so that such
(b) Only II follows encroachments, do not recur in future.
(c) I and II follow (a) None follows (b) I and II follow
(d) II and III follow (c) II and III follow (d) All I, II and III follow
(e) None of these
7. Statement The vehicular traffic in New Delhi
4. Statement During the past few days, more area has increased considerably these days
and more number of indoor patients of the making it difficult for people to reach on time
local government hospital have been to Railway Station as vehicles frequently face
diagnosed with malaria. SBI (PO) 2014 traffic jams at various places. UBI 2017
290 How to Crack Test of Reasoning ANALYTICAL

Courses of Action (c) Scholarships should be declared for those


I. The vehicles which are not bound to students who perform well in tenth and
Railway Station should not be allowed to twelfth standard examinations
ply on the road connecting the New (d) All the colleges and universities should
Delhi area to the Railway Station. be asked to emphasise the importance of
II. The load of vehicular traffic should be higher education and benefits of
diverted through several link roads studying in their institute should also be
during the peak hours. highlighted
III. The time table of trains should be (e) The Government should make education
prepared so as to avoid the congestion free for all people of the State
during the peak hours.
9. Study the following information carefully to
Which of the courses of action logically answer the given question. OICL (SO) 2016
follow(s) for pursuing? (A ‘course of action
is a step or administrative’ decision to be Even though Country X has developed a
taken for improvement, follow-up or further new vaccine called stagniere for the
action in regard to the problem, policy etc.) treatment of an earlier incurable disease, the
Health Ministry of Country X has recently
(a) Only I and II follows
ordered to discontinue its usage until further
(b) Only I and III follow

A
(c) Only II follows
refinements.

M
(d) Only II and III follow Which of the following strengthens the order

H
(e) All I, II and III follow given by the Health Ministry of Country X?

TT
8. Read the following information carefully and (a) Country B has also started developing
answer the question which follows. same medicine for the same disease.
SBI Associate (PO) 2015
A
(b) ‘Cyamine’, the mixture of chemicals
W
Most students from State X prefer to move which is crucial for the development of
base to other States for higher education. The stagniere, is available only with the
SH

government of the State is concerned about scientists of Country B.


this problem as these students might not come (c) The vaccine stagniere, though highly
A

back even after completing their education. effective for targeted disease has some
@

Which of the following statements may be a long term side effects like skin infection,
course of action to deal with this problem? liver enlargements, swelling of bones etc.
(a) The Government should set-up well (d) Each vaccine costs around $69 which is
equipped State of the Art universities not affordable by many citizens of the
which will provide quality education country.
(b) The Government should not give much (e) The vaccine is made from the pollen of
importance to this problem as there is some flower which is available only in
high density of population per square the forests in some remote locations of
km in the State the country.

Answers
Base Level Exercise
1. (e) 2. (e) 3. (e) 4. (a) 5. (b) 6. (b) 7. (e) 8. (b) 9. (b) 10. (a)
11. (b) 12. (b) 13. (d) 14. (a) 15. (a) 16. (e) 17. (b) 18. (a)

Expert Level Exercise


1. (c) 2. (d) 3. (c) 4. (c) 5. (d) 6. (d) 7. (c) 8. (d) 9. (c)
Chapter 25 STATEMENT AND COURSE OF ACTION 291

Answer with Explanations


Base Level Exercise
1. (e) According to the statement, both the courses 9. (b) If the industry employ persons on temporary basis
of action are strong. it can improve its production. So, course of action
2. (e) According to the statement, both the courses II follows.
of action are strong. 10. (a) Course of action I can be followed. It will help
3. (e) Both courses of action I and II are people to understand the benefits of taxes and
constitutional obligations. Hence, both encourage them to submit taxes.
actions follow. 11. (b) Either the parents or guardians of the defaults
4. (a) Main problem according to the question is should be called for a meeting.
that most of the development plans are on 12. (b) Only course of action II is advisable. The
paper only. But if the incharge will supervise authorities should penalised all those who found
the field work properly the plans will take place damaging the property.
in real world. Hence, only action I follows. 13. (d) In the given courses of action, none is strong as
5. (b) If the classes are divided into two shifts the per the question.

A
school management will be able to effectively 14. (a) Course of action I is advisable because it is a

M
manage the school with the existing teachers. Judicious step. But course of action II is irrelevant

H
So, course of action II follows. to the given statement.

TT
6. (b) The government should build dams and also 15. (a) Only course of action I is suitable for pursuing.
take other measures to control floods, so that Course of action II is too harsh regarding the
the villages will not have to face flood fury
every year in the monsoon.
problem.
A
W
16. (e) Obviously, both the courses of action are suitable
7. (e) Both the courses of action follow. for pursuing.
SH

8. (b) The correct course of action for the given 17. (b) Only course of action II follows as primary
statement is, schools should provide subsidy education may eradicate the problem.
A

on school uniforms. 18. (a) Only course of action I can follow.


@

Expert Level Exercise


1. (c) The immediate action can be taken by the 5. (d) Only action II is a good step as it will compensate
Municipal Corporation not by the Parliament the government loss. Actions I and III are not
and the situtation needs immediate action. It reasonable.
is necessary to equip the hospitals properly 6. (d) All courses of action follow. Action I would put an
for the treatment of patients. So, courses of end to the current round of encroachment. Action
action I and III follows. II would work as a deterrent. Action III is the
2. (d) Only courses of action I and II follows. Action I ultimate preventive step.
will find out the reason and measures to 7. (c) Only course of action II seems to be feasible.
remove problems. Action II will convince Others are not reasonable.
parents to send their childrens to school.
Action Ill is not reasonable. 8. (d) When the students get to know the benefits of
studying in college and universities of their own
3. (c) Action I follows because such a clause would State, then they will think twice before going
act as a deterrent to inferior standard outside the State for higher education. Hence,
construction. Action II is also a good step as it correct action will be option (d).
will make officials responsible.
9. (c) The vaccine stagniere is highly effective for the
4. (c) Only action III follows as it will prevent the targeted disease. But it has serious side effects.
disease from spreading to a large number of So, the Health Ministry of Country X might have
people. ordered to discontinue its use. Thus, option (c)
strengthens the order given by the Health Ministry
of Country X.
How to Crack Test of Reasoning ANALYTICAL
292

26
Cause and Effect
Cause is the reason of an event that has occured Principal Cause It is the most important
and Effects are the outcomes of a cause. reason behind the effect. e.g. Rohit fails in
The necessary condition for an event to occur is annual exam because he does not study.
a cause which supplements an event to occur. Here, the principal cause is ‘he does not

A
For a cause to be valid, it must be either study’

M
sufficient or necessary. Common Cause Two effects given in two

H
1. Necessary Condition for an event to take statements may be caused by a third
unmentioned event which is called the

TT
place is that condition without which the
event will not occur. common cause of the given events. e.g.
Statement 1. Global warming is increasing.
2. Sufficient Condition for an event to take
A
Statement 2. The glaciers are melting.
W
place is that condition under whose
presence event must occur. Here, the common cause will be the
SH

e.g. For life to exist on earth we require “amount of carbon dioxide” is increasing.
(a) air (b) water (c) food.
A

Condition (a), (b) and (c) together makes Types of Questions


@

sufficient conditions for life to exist on


Earth but individually they are necessary There are mainly two types of questions,
condition for life to exist on Earth. which are generally asked in various
Therefore, we can say that there may be more competitive examinations.
than one necessary conditions for the occurrence
of an event and all those necessary conditions Type 1
must be included in the sufficient conditions.
Statement and Direction
Different Types of Causes Based Questions
In this type of questions, two statements are
Immediate Cause An immediate cause is one
given and the candidate has to identify
which occurs immediately before the effect.
whether they are independent causes or
e.g. I slapped Rohan after that he slapped me.
effects of independent causes or a common
Here, the immediate cause is “I slapped Rohan”.
cause etc.
Chapter 26 CAUSE AND EFFECT
293

Directions (Illustrations 1-3) In the given


questions, there are given two statements A and
Type 2
B. These statements may be either independent Direct Cause and
causes or may be effects of independent causes or
a common cause. One of these statements may be Effect Based Questions
the effect of the other statement. Read both the In this type of questions, a cause or an effect is
statements and decide which of the following given followed by 4 or 5 options. The
answer choices correctly depicts the relationship candidate has to choose the correct effect or
between these two statements. cause from the options which suits the given
Give Answer statements most.
(a) If Statement A is the cause and Statement B is
the effect Illustration 4. Cause A severe cyclonic storm
(b) If Statement B is the cause and Statement A is swept away most part of the state during the
the effect last two days.
(c) If both the Statements A and B are independent Which of the following cannot be a possible effect of
causes the above cause? PNB (PO) 2009
(d) If both the Statements A and B are effects of (a) Heavy rainfall was reported in most part of the
independent causes state during the last two days

A
(e) If both the Statements A and B are effects of (b) Many people were rendered homeless as their

M
common cause houses were flown away
(c) The communication system of the state was

H
Illustration 1. A. Ahmed is a healthy boy. severely affected and continues to be out of gear

TT
B. His mother is very particular about the (d) Government has ordered that all the offices and
schools should be kept open
food he eats.
Solution (b) Since, Ahmed’s mother take care of what A
(e) All above are possible effects
W
Solution (d) In such a catalcysmic scenario, the
he eats, Ahmed has a good health. Hence, (B) is
Government is likely to order the closure of offices
the cause and (A) is its effect. Therefore, (b) is
SH

and schools.
correct.
Hence, statements (d) cannot be a possible effects of
A

Illustration 2. A. The staff of airport authorities the given cause.


@

called off the strike they were observing in


Illustration 5. Effect The prices of food grains
protest against privatisation .
and vegetables have increased by about 30% in
B. The staff of airport authorities went on the past three months.
strike, anticipating a threat to their jobs.
Which of the following can be a probable cause of the
Solution (d) Clearly, calling of the strike and going above effect? CGPSC 2013
on strike are events that may not be backed by the (a) The farmers have decided to change their farming
same cause. Therefore, they must have been style
effects triggered by separate independent causes. (b) The prices of other products have increased more
than 30%
Illustration 3. A. Party ‘X’ won clear majority in (c) The number of farmers has reduced
the recently held state assembly elections. (d) Occupation of farming has not been viewed as a
B. Of late, there was unrest in public and also reputed work
among the members of the ruling party of the (e) None of the above
state. BOI (PO) 2007 Solutions (b) The prices of foodgrains and vegetables
Solution (e) It can be safely assumed that party ‘ X’ have increased by about 30% due to increase of
was earlier in the opposition and has benefitted more than 30% in the price of other products,
from anti incumbancy. But A and B are because farmers will need 30% more funding for
consequences of a common cause, i.e., bad purchasing other products.
governance by the ruling party. Hence, statement (b) can be a probable cause of the
given effect.
How to Crack Test of Reasoning ANALYTICAL
294

Let us Practice
Base Level Exercise
Directions (Q. Nos. 1-14) Below in each 5. A. Worldwide recession has created
question are given two Statements A and B. These uncertainity in job market.
statements may be either independent causes or B. Many people are opting for change from
may be effects of independent causes or of a private sector to public sector.
common cause. One of these statements may be the MHA-CET 2009
effect of the other statement. Read both the
statements and decide which of the following 6. A. A large number of employees could not
answer choices correctly depicts the relationship report to the duty on time.
between these two statements. B. Police had laid down barricades on the
road to trap the miscreants.
Give Answer MHA-CET 2009
(a) If Statement A is the cause and Statement B is its
effect 7. A. The farmers have decided against

A
(b) If Statement B is the cause and Statement A is its selling their Kharif crops to the

M
effect Government agencies.
B. The Government has reduced the

H
(c) If both statements are independent causes
(d) If both statements are effects of independent procurement-price of Kharif crops

TT
causes starting from last month to the next six
(e) If both statements are effects of some common months.
cause A
W
8. A. There has been mass recruitment of IT
1. A. Ravi died while on the way to the professionals by Indian IT companies.
SH

hospital. B. Many developed countries are


B. A car dashed into the motorcycle Ravi increasingly outsourcing IT related
A

was driving. functions to India and China.


@

UBI (PO) 2009


2. A. Most of the shopkeepers in the locality
closed their shops for the second 9. A. The price of aircraft fuel has risen
continuous day. during the past few months.
B. Two groups of people living in the B. Many passenger airlines in India have
locality have been fighting with each been forced to cut their air fares by
other with bricks and stones, forcing about 10%. Punjab and Sindh Bank (PO) 2009
people to stay indoors. SBI (PO) 2010 10. A. The Government has decided to
3. A. The prices of fruits have dropped increase the prices of LPG cylinders
substantially during the last few days. with immediate effect.
B. The prices of foodgrains have increased B. The Government has decided to
substantially during the last few days. increase the prices of Kerosine with
Andhra Bank (PO) 2009 immediate effect. BOI (PO) 2010

4. A. The road traffic between the two towns in 11. A. Most of the students enrolled
the state has been disrupted since last themselves for the educational tour
week. scheduled for next month.
B. The rail traffic between the two towns in B. The school authority cancelled the
the state has been disrupted since last educational tour scheduled for next
week. Andhra Bank (PO) 2009 month. Andhra Bank (Clerk) 2009
Chapter 26 CAUSE AND EFFECT
295

12. A. The Reserve Bank of India has recently 17. Statement I The prices of petrol and diesel in
put restrictions on few small banks in the domestic market have remained
the country. unchanged for the past few months .
B. The small banks in the private and Statement II The crude oil prices in the
cooperative sector in India are not in a international market have gone up
position to withstand the competitions substantially in the last few months.
of the bigger ones in the public sector.
18. Statement I It is the bounden duty of each
13. A. The braking system of the tourist bus member of the civil society to control the air
carrying 40 passengers failed while pollution by contributing their best in this
negotiating a stiff climb on a hilly road. endeavour to safeguard the health of their
B. The tourist bus fell into the gorge killing countrymen.
at least ten passengers and seriously Statement II The alarming air pollution in
injuring all the remaining. our country is causing asthma cases to
IPPB PO (Mains) 2017 constantly multiply.
14. A. The State Government has decided to 19. Statement I The Government has recently
boost English language education in all fixed the fees for professional courses offered
the schools from the next academic year. by the unaided institutions which are much

A
B. The level of English language of the lower than the fees charged last year.

M
school students of the State is Statement II The parents of the aspiring

H
comparatively lower than that of the students launched a severe agitation last year
neighbouring states. IPPB PO (Mains) 2017

TT
protesting against the high fees charged by
Directions (Q. Nos. 15-19) In each of these the unaided institutions.
questions, two statements marked as I and II are A
20. In this question there are two Statements A
W
provided. These may have a cause and effect and B. These statements may either be
relationship or may have independent causes or independent causes or effects of independent
SH

be the effects of independent causes. Read both causes or a common cause. One of these
the statements and give answer. statements may be the effect of the other
A

IBPS Clerk (Mains) 2017 statement. Read both the statements carefully
@

(a) If Statement I is cause and Statement II is effect and decide which of the given answer choices
(b) If Statement II is cause and Statement I is effect correctly depicts the relationship between
(c) If Statement I and II both are independent these two statements.
causes
(d) If Statement I and II both are the effect of A. Despite giving their best performance,
independent causes more than 100 employees of XYZ Pvt.
(e) If both Statements are effect of common cause Ltd. have been laid off.
B. More than 40% employees of XYZ Pvt.
15. Statement I There has been an increase in Ltd. who have crossed the age of 50 yrs
the underground water level column at all have been asked to take voluntary
places in Delhi due to the last year's retirement from their services.
monsoon rains.
Andhra Bank (PO) 2016
Statement II Many trains had to be (a) Statement A is the cause and statement B is·its
cancelled last year due to water-logging on effect
the railway tracks. (b) Statement B is the cause and statement A is its
effect
16. Statement I The prices of sugar had risen (c) Both statements are effects of independent
very sharply in Indian markets last year. causes
Statement II The Government imported (d) Both statements are independent causes
large quantities of sugar as per trade (e) Both statements are effects of some common
agreements with other countries last year. cause
How to Crack Test of Reasoning ANALYTICAL
296

21. Statement Most part of the boundary walls of III. The college should be blacklisted by the
the local school collapsed completely last university for holding final examinations
night. atleast for a year.
Which of the following can be a possible (a) Only I (b) Only II
consequence of the facts stated in the above (c) Only III (d) Both II and III
(e) None of these
statement? OBC (PO) 2010
I. The local school authority will close 24. Cause All the major rivers in the state have
down the school till the boundary walls been flowing way over the danger level for
are erected. the past few weeks.
II. The Government will levy penalty to the Which of the following is/are possible effect(s) of
school management for their negligence. the above cause? Canara Bank (PO) 2007
III. The management of the school will erect I. Many villages situated near the
temporary fences till the boundary walls riverbanks are submerged, forcing the
are erected. residents to flee.
(a) Only I (b) Only II II. Government has decided to provide
(c) Only III (d) Both I and III alternate shelter to all the affected
(e) None of the above villagers residing near the river banks.

A
22. Statement The prices of foodgrains and III. The entire state has been put on high

M
other essential commodities have decreased flood alert.
(a) Only I (b) Both I and II

H
for the second consecutive week.
Which of the following can be possible (c) Both II and III (d) All I, II and III

TT
consequence of the facts stated in the above (e) None of these
statement? OBC (PO) 2010
I. The consumer price index will come A
25. Cause Government has recently decided to
W
hike the procurement price of paddy for the
down considerably. rabi crops.
SH

II. People will increases their purchase of Which of the following will be a possible effect of
quantity of essential commodities and the above cause? RBI (Grade ‘B’) 2009
A

foodgrains. (a) The farmers may be encouraged to cultivate


@

III. Government will increase its taxes on paddy for the rabi season
essential commodities and foodgrains. (b) The farmers may switch over to other cash
(a) Both I and II (b) Both II and III crops in their paddy fields
(c) Both I and III (d) All I, II and III (c) There was a drop in production of paddy
(e) None of these during Kharif season
(d) Government may not increase the
23. Statement Many students were caught while procurement price of paddy during the next
using unfair means during the final Kharif season
examinations by the special team of the (e) Government will buy paddy from the open
university. market during the next few months
Which of the following can be a possible
26. Effect Atleast 20 school children were
consequence of the facts stated in the above
seriously injured while going for a school
statement? OBC (PO) 2010
picnic during the weekend.
I. The teacher responsible for invigilation in
all such examination halls where the Which of the following can be a probable cause of
the above effect? RBI (Grade ‘B’) 2009
students were caught are to be suspended
(a) The teacher accompanying the school children
from their services.
fell ill during the journey
II. All those students who were caught (b) The bus in which the children were travelling
while using unfair means are to be met with an accident while taking turn on the
debarred from appearing in these main highway
examinations for a year.
Chapter 26 CAUSE AND EFFECT
297

(c) The driver of the bus in which the children were (d) The company owns large tracts of land in
travelling did not report after the break at the the state which will fetch huge sum of its
halting place on their journey owners
(d) The school authority banned all school picnics (e) None of the above
for the next six months with immediate effect
(e) None of the above 28. Cause School authority has increased the
tution fee excessively from the current
27. Effect Majority of the employees of the ailing academic session.
organisation opted for voluntary retirement Which of the following can be possible effect of
scheme and left the organisation with all their the cause mentioned above?
retirement benefits within a fortnight of
I. Parents of the students of this school
launching the scheme.
will get their child admitted in other
Which of the following can be a probable cause of schools.
the above effect? IOB (PO) 2010
II. No new student will get admission in
(a) The company has been making huge losses for
the past 5yr and is unable to pay salary to its
this academic session.
employees in time III. Parents of the students of this school
(b) The management of the company made huge will contact the school authority for the
personal gains through unlawful activities roll back of the increased fee.

A
(c) One of the competitions of the company went (a) None (b) Only ll (c) Only III (d) Only I

M
bankrupt last year (e) Both II and III

H
TT
Expert Level Exercise
Directions (Q. Nos. 1-5) In each question, two A
2. Statement A India is ranked fifth most
W
Statements A and B are provided. These may have powerful country in the world, next to US,
a cause and effect relationship or may have
SH

China, Russia and Japan, in the hierarchy


independent causes. of top 50 nations, identified on the basis of
Give Answer their GDP, as per national security index.
A

(a) If the Statement A is the cause and Statement B is Statement B The assessment is based on
@

its effect defence capability, economic strength,


(b) If the Statement B is the cause and Statements A is effective population, technological
its effect capability and energy security of top 50
(c) If both statements are effects of independent causes countries. MAT 2012
(d) If both statements are effects of some common
cause 3. Statement A Sri Lankan kipper Kumar
Sangakara justified his decision to step
1. Statement A A mobile phone, if kept on down from the captaincy of the ODI and
during an air flight, can actually disrupt the
T20 teams by saying, ‘I will be 37 by the
plane's electronic systems and eventually lead
next World Cup and I cannot be sure of my
to a crash, a study has revealed. Older planes
place in the team. It is better that Sri Lanka
are most at risk to mobiles, laptop and i-pads.
is now led by a player who will be at the
Statement B One case involved a Boeing 747 peak of his career during that tournament’.
flying at a height of 4500 ft whose pilot Statement B Remarkably, unlike most
disengaged by itself. When the flight skippers whose individual performance
attendants went through the cabin, they found drops after assuming the leadership role,
four passenger using their cell phones. Once Sangakara has actually batted better as
these were switched off, the flight carried on captain in all three formats of the game.
without any problem. MAT 2012
How to Crack Test of Reasoning ANALYTICAL
298

4. Statement A Indian women do not believe (d) The airline companies can now charge unlimited
in donating blood. According to the first additional charge for peak time flights
ever data bank on gender distribution of (e) None of the above
blood donors, India has only 6% blood 7. Effect During the last six months the sale of
donations by women. The rest 94% were four wheelers has come down in comparison
male donors. to the sale of four wheelers last year for the
Statement B Just twenty five countries in same period.
the world collect more than 40% of their Which of the following can be the possible cause for
blood supplies from female donors, these the effect mentioned above?
include Australia, US, Thailand, I. Government has increased the excise duty
Switzerland, Portugal, New Zealand,
on the four wheelers from the beginning of
Mongolia, Zimbabwe, etc.
this year.
5. Statement A Six months after Glaxo Smith II. Cost of petrol has increased immensely
Kline (GSK) consumer healthcare India during the last eight months.
brought in its global oral care brand III. Interest rates on car and home loans has
Sensodyne to the Indian market, been increasing from last seven months.
Sensodyne has garnered a 10% share of the IBPS (PO) 2011

A
sensitive toothpaste market. (a) I, II and III

M
Statements B GSK has not only invested (b) Both I and II
heavily in the advertising and promotion (c) Both II and III

H
of the brand but has contacted 15000 (d) Only ll

TT
dentists in two months and is keen on (e) Only I
building on the Sensodyne equity.
6. Effect Government has allowed all the A
8. Cause The Government has recently increased
its takes on Petrol and Diesel by about 10%.
W
airlines to charge additional amount as Which of the following can be a possible effect of the
SH

peak time congestion charges for the flights above cause? CBI (PO) 2010
landing between 6 : 00 am and 10: 00 am. (a) The Petroleum companies will reduce the prices
A

Which of the following is a probable cause of of petrol and diesel by about 10%
@

the above effect? Canara Bank (PO) 2007 (b) The Petroleum companies will increase the price
(a) All the airline companies had threatened to of petrol and diesel by about 10%
suspend their services during peak hours (c) The Petroleum companies will increase the
(b) The Government has increased its tax for prices of petrol and diesel by about 5%
peak time flights (d) The petrol pumps will stop selling petrol and
(c) The aircrafts are routinely put on hold over diesel till the taxes are rolled back by the
the airports while landing during peak time, Government
causing extra fuel consumption (e) None of the above

Answers
Base Level Exercise
1. (b) 2. (b) 3. (d) 4. (e) 5. (a) 6. (b) 7. (b) 8. (b) 9. (d) 10. (e)
11. (c) 12. (b) 13. (a) 14. (b) 15. (e) 16. (a) 17. (d) 18. (b) 19. (b) 20. (e)
21. (c) 22. (a) 23. (d) 24. (d) 25. (a) 26. (b) 27. (a) 28. (c)

Expert Level Exercise


1. (a) 2. (b) 3. (c) 4. (c) 5. (d) 6. (c) 7. (a) 8. (e)
Chapter 26 CAUSE AND EFFECT

Answer with Explanations


Base Level Exercise
1. (b) Ravi died because of the accident. Hence, B 15. (e) Clearly, both the statements are the effects of
is the cause and A is the effect. some common cause.
2. (b) The fighting has led to the closure of the 16. (a) Statement I is the cause and Statement II is its
shops. Hence, B is the cause and A is its effect. effect.
3. (d) A has happened because of increased 17. (d) Statement I and II both are the effect of
supply of fruits. B has happened because of independent causes.
decreased supply of foodgrains.
18. (b) Statement II is cause and Statement I is effect.
4. (e) A and B both seem to have a common
19. (b) Statement II is cause and Statement I is its effect.
cause, agitation on a large scale.
20. (e) Both the statements are effects of some common
5. (a) Many people are opting for change from
cause.
private sector to public sector because
worldwide recession has created uncertainity in 21. (c) It can be a possible consequence.
the job market. It is considered that public 22. (a) I and II can be possible consequences.

A
sector jobs are more certain than the private
23. (d) II and III can be possible consequences.

M
sector jobs. So, Statements A is the cause while
Statements B is the effect. 24. (d) All, I, II and III are possible effects.

H
6. (b) A large number of employees could not 25. (a) The decision of hiking the procurement price of a

TT
report to the duty on time because police had crop may encourage the farmers to cultivate it.
laid down barricades on the road to drop the
miscreants. So, Statement B is the cause while injuries. A
26. (b) We often hear of accidents leading to such
W
Statement A is its effect.
27. (a) It should be the obvious cause for the given effect.
SH

7. (b) The reduction in procurement price of crops


must have instigated the farmers not to sell their 28. (c) Only III can be the possible effect.
produce to Government agencies.
A

8. (b) The outsourcing has led to creation of jobs in Expert Level Exercise
@

IT in India. 1. (a) Statement A is the cause and Statement B is the


9. (d) A is usually the effect of hike in global effect.
petroleum prices. B is usually the effect of 2. (b) Statement B is the cause and Statement A is its
competition in aviation. effect because India's rank as the fifth most powerful
10. (e) Both are the effects of common cause as country is based on its defence capability, economic
prices of both LPG cylinders and kerosine strength, effective population, technological capability
increases adruptly. It seems that the cause may and energy security among top 50 countries.
be the increase in price of pertrolium. 3. (c) Both statements are effects of independent
11. (c) A happened so that students could see more causes.
of the world. B happened so that the school 4. (c) Both statements are effects of an independent
may attend to other important task. Hence, both cause.
are independent causes.
5. (d) Both statements are effects of some common
12. (b) The inability of the small banks to compete cause.
with the bigger ones shall not ensure security
and good service to the customers, which is an 6. (c) Extra fuel consumption is a probable cause of the
essential concomitant that has to be looked into given effect.
by the RBI. It seems to be a remedial step for 7. (a) All may be the possible causes.
the same. 8. (e) Companies may2 hike the price to cover the loss
13. (a) Brake failure led to the accident. because of the increased tax. But increase in price is
14. (b) Since the level is lower, the Government has totally the matter of company, we cannot make any
decided to boost English language education. guesses about it.
How to Crack Test of Reasoning ANALYTICAL

27

Assertion and
Reason

A
Illustration 1. Assertion (A) Shimla is colder than Delhi.

M
Assertion is a strong or forceful statement
and Reason is an explanation about that Reason (R) Shimla is at higher altitude as

H
particular statement. compared to Delhi.

TT
The questions consist of two statements, Solution (a) Both ‘A’ and ‘R’ are true and ‘R’ is the correct
an assertion and a reason. The candidate explanation of ‘A’. As compared to Delhi, Shimla is
must first determine whether each A
situated at a higher altitude and temperature decreases
W
statement is true. It both are true, then it by 1°C for every 165m of ascent.
SH

is required to analyse whether the reason Illustration 2. Assertion (A) Silver is not used to
is an optimum and correct explanation of make wires.
the assertion.
A

Reason (R) Silver is a bad conductor.


@

Sometimes both assertion and reason are Solution (c) Silver is not used to makes wires, hence
correctly stated facts but the reason does Assertion is true but silver is not a bad conductor, it is a
not correctly explain the assertion. good conductor. Hence, Reason is false.
So, different possibilities can exist Illustration 3. Assertion (A) In India, females have
between these two statements and higher life expectancy than the males.
accordingly the correct answer is marked Reason (R) Females receive a better diet.
from the given alternatives.
Solution (e) Because of high birth rate and negligence,
Directions (Illustration 1-4) In each of the females have lower life expectancy than males in India.
following question, there are two statements Although a better diet is a requirement for females, they
labelled as Assertion (A) and Reason (R). do not receive that. Thus, both 'A’ and ‘R’ are false.

Give answer Illustration 4. Assertion (A) Tamil Nadu gets most of


(a) If both ‘A’ and ‘R’ are true and ‘R’ is the the rainfall in winter.
correct explanation of ‘A’ Reason (R) Tamil Nadu gets rainfall from
(b) If both ‘A’ and ‘R’ are true but ‘R’ is not the
retreating monsoons. NIFT (PG) 2014
correct explanation of ‘A’
Solution (a) Rainfall in Tamil Nadu is caused by the
(c) If ‘A’ is true but 'R' is false
retreating monsoons which occur in winter. Hence, both
(d) If ‘A’ is false but 'R' is true
(e) If both ‘A’ and 'R' are false (A) and (R) are true and (R) is the correct explanation of (A).
Chapter 27 ASSERTION AND REASON 301

Let us Practice
Base Level Exercise
Directions (Q. Nos. 1-14) Each of these questions Reason (R) India become independent on
has an Assertion (A) and a Reason (R). 15th August 1947.
Give answer 9. Assertion (A) In India 2nd October is
(a) If both ‘A’ and ‘R’ are true and ‘R’ is the correct observed as Martyr's Day.
explanation of ‘A’
(b) If both ‘A’ and ‘R’ are true but ‘R’ is not the Reason (R) Pt Nehru was the first Prime
correct explanation of ‘A’ Minister of India.
(c) If ‘A’ is true but ‘R’ is false
(d) If ‘A’ is false but ‘R’ is true
10. Assertion (A) River Narmada flows
westward.
1. Assertion (A) Leakages in household gas Reason (R) Narmada falls into the Bay of
cylinders can be detected.
Bengal.

A
Reason (R) LPG has a strong smell.

M
11. Assertion (A) Clothes are not washed
2. Assertion (A) Salt is added to cook food at properly in hard water.

H
higher altitudes.

TT
Reason (R) Hard water contains many
Reason (R) Temperature is lower at higher minerals. NIFT (PG) 2014
altitudes.
A
12. Assertion (A) The common value of all
W
3. Assertion (A) Inside the Earth, metals are religion should be taught to the children in
present in molten state. NIFT (UG) 2014 our schools.
SH

Reason (R) Earth absorbs the Sun’s rays. Reason (R) Yes, it will help inculcating moral
4. Assertion (A) India’s Republic Day’ falls on values amongst the children of our country.
A

26th January. MAT 2014


@

Reason (R) Constitution of India, declaring 13. Assertion (A) The steam engine was
India as a ‘Republic’, came into force on 26th invented by James Watt.
January 1950. MAT 2013 Reason (R) There was a problem of taking
5. Assertion (A) India is a democratic country. out water from flooded mines. NIFT (PG) 2014
Reason (R) India is a developing country. 14. Assertion (A) We feel colder on mountains
MAT 2013 than on plains.
6. Assertion (A) In India, the judiciary is Reason (R) Temperature decreases with
independent of the executive. altitude. NIFT (UG) 2014
Reason (R) Judiciary favours the government 15 Assertion (A) Autism is a development
and helps in the implementation of its plans. disability.
NIFT (UG) 2014
Reason (R) Heredity and lower development
7. Assertion (A) An apple a day keeps the of brain are the causes of the Autism.
doctor away. SSC (CGL) 2014
(a) ‘A’ is false and ‘R’ is true
Reason (R) Apple are very costly. MAT 2012
(b) Both ‘A’ and ‘R’ are false
8. Assertion (A) India celebrates its (c) Both ‘A’ and ‘R’ are True
independence day on 15th August. (d) ‘A’ is true and ‘R’ is false.
302 How to Crack Test of Reasoning ANALYTICAL

6. Assertion (A) Interpersonal roles relate to a


Expert Level Exercise person's behaviour that focuses on
Directions (Q. Nos. 1-3) For the Assertion (A) interpersonal contact.
and Reason (R) below, choose the correct Reason (R) Such roles are derived directly
alternative from the following. from the authority and status.
Give answer 7. As per a recent survey, 85% of the work force
(a) If ‘A’ is true but ‘R’ is false today is either actively looking for a job or
(b) If both ‘A’ and ‘R’ are true but ‘R’ is not the open to talking to recruiters and relevant
correct explanation of ‘A’ opportunities; the ones who are satisfied with
(c) If both ‘A’ and ‘R’ are true and ‘R’ is the
correct explanation of ‘A’
their job are few in number.
(d) If ‘A’ is false but ‘R’ is true Which of the following may be a reason for the
above mentioned results of the survey? MPPSC 2018
1. Assertion (A) In the ordinary fire (a) As there is a wide array of opportunities today
extinguisher, carbon dioxide is generated and headhunters are always seeking for
by the chemical reaction of sodium professionals, people are not willing to let go of
bicarbonate and dilute sulphuric acid. an opportunity to do something new and
Reason (R) Carbon dioxide can be used to challenging
extinguish fire. MAT 2011 (b) As per a recent survey more than 40% people

A
today are dissatisfied with their jobs as there is the
2. Assertion (A) All machines suffer a loss of

M
wide gap between what they study in the class
efficiency due to the frictional force acting room as compared to the actual work they do

H
on any moving member of that machine. (c) Some people in the country take education loan

TT
Reason (R) The frictional force can be for completing higher education and are
reduced by providing a thin film of lubricant expected to repay the loans themselves after
between any two mating parts. MAT 2011 A
they start working
W
3. Assertion (A) The escape velocity from the (d) As employees today are of different age group
surface of the moon is less than that from and background it becomes difficult for the HR
SH

the Earth’s surface. department to organise employment


engagement activities to improve the job
Reason (R) The moon has no atmosphere.
A

satisfaction of employees
MAT 2011
@

Directions (Q.Nos. 4-6) Choose the correct 8. Travellers should generally be careful about
alternative from the following options for the booking a hotel at the time when a big
Assertion (A) and Reason (R) given below. convention is in town.
Which of the following is the most plausible reason
Give answer
(a) If Both ‘A’ and ‘R’ are true and ‘R’ is the for exercising this caution? MPPSC 2018
correct explanation of ‘A’ (a) Hotels boost rates when conventions are going
(b) If Both ‘A’ and ‘R’ are true but ‘R’ is not the on in town
correct explanation of ‘A’ (b) Terrorist threats are likely at the time of
(c) If ‘A’ is true but ‘R’ is false Conventions
(d) If ‘A’ is false but ‘R’ is true (c) Conventions are a breeding ground for
(e) If both ‘A’ and ‘R’ are false swindlers of all types
(d) If you go to a town where a convention is going
4. Assertion (A) Red-green colour blindness on, you are most likely to spend all your time
occurs with more frequency in males than attending the convention
in females.
Reason (R) Females have two
9. It tooks the soaring rhetoric of US Ex-
President Barack Obama - in his eulogy to
chromosomes and males have one.
Mandela-to finally bury the embarrassing
5. Assertion (A) The western coast of India is legacy of American policy towards the South
characterised by the location of several sea African leader during his lifetime. It has taken
ports. a black president of the US to make amends
Reason (R) Western coast has evidence of for treating Mandela as an enemy, long after
deep sea water. the Cold War had ended.
Chapter 27 ASSERTION AND REASON 303

Which of the following can be the best reason for (a) The tea was available only in a single flavour
the assertion made in the above statement? (b) Media preservatives said that the tea has
MPPSC 2018 contained preservatives more than the
(a) Mandela was opposed to racism permissible limit.
(b) Obama is opposed to racism (c) The ready to mix tea was only in large
(c) The Cold War has now ended packaging
(d) US Presidents are entitled to change their (d) The prices of the tea were unusually higher
foreign policy. than other such items of regular use.
(e) The introductory scheme of free gift voucher
10 Read the following information and answer with the tea was withdrawn from the market
the given question. with 15 days of launch of the product.
The number of workers joining trade union in 12 Read the following information and answer
company B has decreased tremendously in the question.
the past few years.
Export of tea by Country X to Country Z, a
Which of the following can be a reason for the major importer of tea from country X, were
decreased number of workers joining these unions? reduced by more than half in the present
Indian Bank (PO) 2016
(a) The wages and other benefits offered in year as compared to previous year even
company B are better than that received by though the production of tea in Country X
workers in other companies was increased by 25% in the said year.

A
(b) Social events and activities for the welfare of Which of the following statements cannot be a

M
society are organised by the employers of reason for the given problem?
company B regularly. NABARD Assit. Manager (Grade A, B) 2016, 15

H
(c) The employers in company B ensure that the (a) Until last year, tea was exempt from export

TT
grievances of worker's are received well in time. duty in Country X but this year the
(d) The management of company B considers the view government levied the export duty of
of workers in all decision related to their welfare
A
` 10 per kg.
W
(e) The intervention of government in regulation of (b) Country Z signed agreement with three new
trade unions has decreased in recent times. countries which provide good quality tea at
SH

comparatively much lower prices than


11 Read the following information to answer the country X.
given question. (c) Many harmful chemicals were reportedly
A

A tea manufacturing brand V, introduces a found in the tea of Country X upon being
@

new ready to mix packaging of tea. But it had tested by Country Z last year
to withdraw its product from the market in a (d) The market price of tea was reduced by 5%
short span soon after the product was for consecutive two years by Country X for
launched, it faced heavy losses. its foreign buyers.
(e) Tea producers of Country X started using a
Which of the following cannot be a reason for the
new type of fertilizer in the last year which
withdrawal of new product from the market? promised good production but at the cost of
IBPS Clerk (Mains) 2017
quality.

Answers
Base Level Exercise
1. (a) 2. (b) 3. (c) 4. (a) 5. (b) 6. (c) 7. (c) 8. (a) 9. (d) 10. (c)
11. (b) 12. (a) 13. (a) 14. (a) 15. (c)

Expert Level Exercise


1. (c) 2. (b) 3. (b) 4. (a) 5. (a) 6. (c) 7. (a) 8. (a) 9. (b) 10. (e)
11. (c) 12. (d)
304 How to Crack Test of Reasoning ANALYTICAL

Answer with Explanations


Base Level Exercise 14. (a) Both A and R are true and R is the correct
explanation of A. Above the sea level,
1. (a) LPG contains ethyl mercaptan which has strong temperature decreases with an increase in
smell. Because of this, small leakages in altitude which makes mountain peaks colder.
household gas cylinders can be detected. 15. (c) Both A and R are true and ‘R’ is the correct
explanation of ‘A’.
2. (b) As pressure decreases at higher altitudes, water
boils much below 100°C, so that the food does
not get sufficient heat for being cooked. Salt Expert Level Exercise
increases the boiling point of water.
1. (c) Both A and R are true and R is the correct
Reason is also true. But not the correct
explanation of A.
explanation of assertion.
2. (b) Both A and R are true but R is not the correct
3. (c) Inside the Earth, the high temperature and
explanation of A.
pressure keep the metals in molten state. The
Earth does not absorb the Sun’s rays but reflects 3. (b) Both A and R are true but R is not the correct

A
them. explanation of A.

M
4. (a) Both A and R are true and R is the correct 4. (a) A Men are much more likely to be colourblind
explanation of A. than women because the genes responsible

H
for the most common, inherited colour

TT
5. (b) Both A and R are true but R is not the correct
blindness are on the X chromosome. Male
explanation of A.
only have one X chromosome, while females
6. (c) A is true but R is false. As in India, the judiciary is A
have two X chromosome. So, R is correctly
W
completely independent of the executive and the explain A.
government has no interference in the judicial
5. (a) Both ‘A’ and ‘R’ are true and ‘R’ is the correct
SH

affairs.
explanation of ‘A’.
7. (c) An apple a day keeps the doctor away is definitely
6. (c) A is true, but R is false.
A

true but the statement R may or may not be true.


@

Hence, A is true but R is false. 7. (a) There is clear relation between ‘relevant
opportunities’ and a wide array of opportunities
8. (a) India became independent on 15th August 1947.
and between ‘recruiters’ and ‘headhunters’.
So, it celebrates independence day on 15th
August. 8. (a) This one is obvious from demand-supply
match.
9. (d) A is false but R is true.
9. (b) Attitude of US has change under the leadership of
10. (c) River Narmada flows westward and drains into Obama. Clearly, he is a votary of
the Arabian sea. So, ‘A’ is true but ‘R’ is false. non-discrimination on the basis of colour-that
11. (b) Both A and R are true but R is not the correct Obama is a ‘‘black president’’ has been
explanation of A. Clothes are not washed properly highlighted.
in hard water because it does not form lather with 10. (e) Statement given in option (e) is a reason for the
soap. However, it is true that hard water contains decreased number of workers joining these
many minerals. unions.
12. (a) Both A and R are true and R is the correct 11. (c) The withdraw of new product from the market
explanation of A. is not due to the packaging size of tea.
13. (a) Both A and R are true and R is the correct 12. (d) When company X reduced its tea price by 5%
explanation of A. The problem of pumping out
for consecutive 2 yr then why company Z
water from the flooded mines required the need
would reduced import of tea from them.
of a self-working engine which led James Watt to
invent the same. Hence, it cannot be the reason of the given
problem.
Chapter 28 DATA SUFFICIENCY 305

28
Data Sufficiency
Data sufficiency is a method to analyse the Illustration 1. How is P related to R?
given set of information and decide whether CGPSC (Pre) 2014
the given information is sufficient to answer I. Q is the son of R.

A
the question. Data sufficiency questions II. Q is the brother of P.

M
include problems based on Solution (e) Statement I
Coding-Decoding, Blood-Relations, R

H
Direction Sense Test, Ranking and Time

TT
Sequence Test, Arithmetical Reasoning, Son
Mathematical Operations and Puzzle Test
etc. A Q
W
Statement II
The problems based on this topic consist of Brother
Q P
SH

two or more than two statements containing


the information related to it. You have to Now, from both statements,
decide whether the problem can be solved Eit
A

h er
by using the information from the given R s
@

on
statements combined or individually. or
Son da
ugh
In these questions, it is also possible that ter
either of the given statements is sufficient to Brother
Q P
answer the question.
Here, P is either the son or daughter of R. So, we
Directions (Illustrations 1-4) The questions cannot find the exact relation between P and R.
given below consist of a question followed by Hence, Statements I and II together are not sufficient
two statements labelled as I and II. We have to to answer the question.
decide whether these statements give enough
information required to answer the question or Illustration 2. Who is older Ritu or Situ?
not. I. Situ was born in the year 1980 and is 5 yr
Give Answer younger than Ritu’s brother.
(a) If Statement I alone is sufficient to answer the II. Ritu is twice as old as her brother.
question
(b) If Statement II alone is sufficient to answer Solution (c) From Statement I, it is clear that Situ is
the question younger than Ritu's brother.
(c) If Statements I and II together are needed to From Statement II, it is clear that Ritu is older than her
answer the question brother.
(d) If Statement I or II alone is sufficient to So, from both the statements, we get
answer the question
(e) If Statements I and II together are not
Ritu > Ritu’s Brother > Situ
sufficient to answer the question Hence, Ritu is older than Situ.
306 How to Crack Test of Reasoning ANALYTICAL

Illustration 3. What is the sum of first two even A B C or E D C or E F


natural numbers out of a group of five even
1 2 3 4 5 6
natural numbers?
Positions of C and E can be either 3rd or 5th.
I. The second number is double of the first From the Statement I, we get that E is to the right of
number. A and hence, position of E can either be 3rd or 5th.
II. All the five numbers are consecutive Therefore, Statement I alone is not sufficient to
even numbers. answer the question. Statement II fixed the
Solution (c) From Statement II, the five numbers are positions of C at 5th and E at 3rd place,
consecutive even numbers. respectively. Therefore, we get our answer from
Statement II.
From Statement I, second number is double of first
number. On combining both Statement I and II, Direction (Illustration 5) In the question given
we get below, three Statements I, II and III are given.
First number = 2 and second number = 4 You are required to find out which of the given
[since, this is the only possibility in case of statements is/are sufficient to answer the
consecutive even numbers, where second number question. IBPS (PO) 2011
is double of the first number] Illustration 5. On which day Suresh went to

A
∴Sum of first two numbers = 2 + 4 = 6 Chennai, if week starts on Monday?

M
So, both Statements I and II are required to answer I. Suresh took leave on Wednesday.
the question.

H
II. Suresh went to Chennai the next day of

TT
Illustration 4. Six persons A, B, C, D, E and F are the day his mother came to his house.
sitting in a row facing North. A and F are III. Suresh’s mother came to his house
sitting at two extreme ends of the row. B is to
A
neither on Monday nor Friday.
W
the immediate right of A and D is 2nd left of F. (a) II and III
What is the position of E with respect to A?
SH

(b) I and II
I. E is to the right of A. (c) I and III
(d) All are needed
A

II. E is to the left of C.


(e) Even I, II and III together are not sufficient
@

Solution (b) On the basis of the statements given in


Solution (e) Statements I, II and III together are not
the question, we can arrange the persons in a row
sufficient as the required day can be anyone of
in the following way
Wednesday, Thursday, Friday and Sunday.

Let us Practice
Base Level Exercise
Directions (Q. Nos. 1-7) Each of the following (b) If the data in Statement II alone is sufficient to
question below consists of a question and two answer the question, while the data in Statement I
statement numbered I and II. You have to decide alone is not sufficient to answer the question
whether the data provided in the statements are (c) If the data either in Statement I alone or in
sufficient to answer the question. Statement II alone is sufficient to answer the
question
Give Answer (d) If the data in both Statements I and II together
(a) If the data in Statement I alone is sufficient to are necessary to answer the question
answer the question, while the data in Statement II
(e) If the data in both the Statements I and II together
alone is not sufficient to answer the question
are not sufficient to answer the question
Chapter 28 DATA SUFFICIENCY 307

1. Who amongst L, M, N, O and P is the Directions (Q. Nos. 8-14) Each question
shortest? NICL 2014 consists of a question and two statements
I. O is shorter than P, but taller than N. numbered I and II given below it. You have to
II. M is not as tall as L. decide, whether the data provided in the
statements are sufficient to answer the question.
2. Point A is towards which direction from Read both the statements and mark the
Point B? NICL 2014 appropriate answer.
I. If a person walks 4 m towards the North Give Answer
from Point A and takes two consecutive (a) If the data even in both statements together are not
right turns, each after walking 4 m, he sufficient to answer the question
would reach Point C, which is 8 m away (b) If the data in Statement I alone is sufficient to
from Point B. answer the question while the data in
II. Point D is 2 m towards the East of point Statement II alone is not sufficient to answer
the question
A and 4 m towards the West of point B.
(c) If the data either in Statement I alone or in
3. How many brothers does Bharat have? Statement II alone is sufficient to answer the
NICL 2014 question
I. Shiela, the mother of Bharat, has only three (d) If the data in both statements together are
necessary to answer the question

A
children.
(e) If the data in Statement II alone is sufficient to

M
II. Meena, the grandmother of Bharat, has
answer the question while the data in
only one granddaughter.

H
Statement I is not sufficient to answer the
4. Is T the grandmother of Q? question

TT
NICL 2014
I. P is the mother of Q. Q is the son of R. R is 8. How many sisters does Madhu have?
son of T. Statements A SBI Clerk (Mains) 2016
W
II. L is the father of N and N is the daughter I. Madhu’s parents have four children.
of T. II. Madhu has three brothers.
SH

5. Are all the five friends viz. Leena, Amit, 9. Is R the granddaughter of C?
Arun, Ali and Ken who are seated around a
A

Statements SBI Clerk (Mains) 2016


circular table facing the centre? NICL 2014
@

I. The only sister of A is the mother of R’s


I. Leena sits second to the left of Amit. Amit brother, B.
faces the centre. Arun sits second to the II. C, the mother of A has only one grandson,
right of Leena. B.
II. Ali sits third to the left of Ken. Ken faces the
10. Four friends A, B, C and D are seated in
centre. Amit sits to the immediate left of Ali
circle facing the centre but not necessarily in
but Ken is not an immediate neighbour of the same order. Is anyone seated exactly
Amit. between C and D, when counted from the
6. On which day of the month is definitely left of C? SBI Clerk (Mains) 2016
Meena’s birthday? If Statements
I. Meena’s brother correctly remembers that I. B is seated to the immediate right of C.
Meena’s birthday is after 25th but before II. B is seated to the immediate left of A. D is
29th of this month. not an immediate neighbour of B.
II. Meena’s father correctly remembers that 11. Among five friends M, N, O, P and Q (each
Meena’s birthday is after 27th but before earning a different amount), who earns the
31st of this month. least? SBI Clerk (Mains) 2016
Statements
7. What is the value of 144$16H7#9? If
I. M earns more than O, P and N.
I. ‘$’ means ‘÷’, ‘H’ means ‘×’ and ‘#’ means ‘+’.
II. P earns more than only O.
II. 16 $ 4 H 2 # 2 =10
308 How to Crack Test of Reasoning ANALYTICAL

12. Among A, B, C, D and E, seated in a straight II. S stands at extreme left end of the line. T
line, but not necessarily in the same order, stands to the extreme right end of the line.
facing North, who sits exactly in the middle Only one person stands between S and U.
of the line? Only one person stands between T and W.
Statements
14. How far is point M from point K?
I. A sits third to left of D. B sits to the IBPS (SO) 2017
immediate right of C. I. Point D is 5m to the South of Point P. Point
II. B sits second to right of A. E is not an M is 8m to the West of point D. Point S is
immediate neighbour of D. 2.5m to the North of point M. Point O is
13. How many people are standing in a straight 10m to the east of Point S. Point K is 2.5m
line (Note: All are facing North)? IBPS SO 2017 to the South of point O.
I. U stands third from the left end of the line. II. Point K is 10m to the East of point M. Point
U is an immediate neighbour of P and W. U is 8m to the West of point M. Point D is
Only one person stands between W and T. to the East of M. Point M is the midpoint of
Only two people stand to the right of W. the lines formed by joining points U and D.

A
Expert Level Exercise

M
H
Directions (Q. Nos. 1-5) Each question consists 2. What is the direction of M with respect to K?

TT
of a question and two statements numbered I and I. K is 8 m to the East of S. Q is to the North of
II given below it. You have to decide whether the S. P is 4 m away from Q. M is 2 m to the
data given in the statements are sufficient to A
North of P.
W
answer the questions. Read both the statements
II. M is 12 m to the West of J. Q is 6 m to the
and Andhara Bank (PO) 2015
SH

South of M. K is to the South-East of Q.


Give Answer
(a) If the data in Statement I alone is sufficient to 3. How is P related to M?
A

answer the question, while the data in I. M is the wife of C. M and I are the only
@

Statement II alone is not sufficient to answer the children of R. G is the only daughter of C.
question
P is the granddaughter of R.
(b) If the data in Statement II alone is sufficient to
answer the question, while the data in II. G is married to L. M is mother-in-law of L.
Statement I alone is not sufficient to answer the M is the only daughter of R and A. P is the
question grand child of R.
(c) If the data either in Statement I alone or in
Statement II alone is sufficient to answer the 4. Five people M, N, O, P and Q live on five
question different floors of a building but not
(d) If the data even in both statements together are not necessarily in the same order. The lowermost
sufficient to answer the question floor of the building is numbered one, the
(e) If the data in both statements together are one above that is numbered two and so on
necessary to answer the question till the topmost floor is numbered five. How
1. Among buildings J, K, L, M, N and O (each many people live between M and Q?
building is of different height), which is the I. M lives on floor numbered three. Only
tallest? Only one person lives between M and P. N
I. M is taller than J and O but shorter than K. lives on an odd numbered floor
N is taller than M but not the tallest. K is immediately above Q.
not the tallest. II. O lives on an even numbered floor
II. K is shorter than only two buildings. J is immediately below M. Only one person
shorter than M and K but taller than O. N lives between O and Q. N lives on the
is taller than J. topmost floor.
Chapter 28 DATA SUFFICIENCY 309

5. Six people are sitting in two parallel rows 7. Point Q is in which direction with respect to
containing three people each, in such a way Point P? SBI (PO) 2014
that there is an equal distance between I. Point P is in East of Point L. Point L is in
adjacent persons. In row - 1, U, V and W North of Point M. Point M is in West of
are seated and all of them are facing South. Point N. Point N is in South of Point O.
In row-2, P, Q and R are seated and all of Point O is in West of Point Q.
them are facing North. (Therefore, in the
given seating arrangement, each member II. Point L is in West of Point P and North of
seated in a row faces another member of Point M. Point M is in East of Point N which
the other row). Who sits at extreme left end is South of Point O. Point Q is in East of
of row-2? Point O and North of Point L.
I. V sets at one of the extreme ends of the 8. Six persons viz. A, B, C, D E and F are sitting
line. Only one person sits between Q and on a circular table for lunch, who among them
the one who faces V. P is an immediate sits immediate left of F (If all the persons are
neighbour of Q. facing towards the centre)?
II. Only one person sits between Q and R. W I. A sits third of right of F. One person sits
faces one of the immediate neighbours of between A and C. E sits third to left of C.

A
R. V is an immediate neighbour of W. II. Only one person sits between E and D. F sits

M
second to right of D. A sits third to left of F.
Directions (Q. Nos. 6-13) Each of the

H
following question below consists of a question 9. Sharvan’s birthday is in which of the following

TT
and two statements numbered I and II given month in a year?
below it. You have to decide whether the data I. Sharvan’s mother correctly remembers
provided in the statements are sufficient to
answer the question. Read both the statements A
that Sharvan’s birthday will come after
W
September but not in the month which
and give answer. IDBI (Office Executive) 2018
has 30 days.
SH

Give Answer
(a) If the data in Statement I alone is sufficient to II. Sharvan’s father correctly remember that
answer the question, while the data in Sharvan’s birthday will not come in first and
A

Statement II alone is not sufficient to answer last month of the year.


@

the question
(b) If the data in Statement II alone is sufficient to 10. How far and in which direction is Point B with
answer the question, while the data in respect to Point A? Canara Bank (PO) 2018
Statement I alone is not sufficient to answer Statements
the question I. Point G is 6 m to the East of Point A. Point
(c) If the data either in Statement I alone or in C is 9 m to the North of Point G. Point F is
Statement II alone is sufficient to answer the
question 3 m to the West of Point C. Point B is 6 m
(d) If the data in both Statements I and II together away from Point F.
are necessary to answer the question II. Point M is 8 m to the West of Point B. Point R
(e) If the data in both the Statements I and II is 8 m to the South of Point M. Point A is
together are not sufficient to answer the
question 11 m to the East of Point R. Point C is to the
North-East of Point A.
6. How is ‘also’ written in a code language?
I. ‘he also show data’ is written as ‘sx fa 11. Amongst the people viz. A, B, C, D, E and F
mn ca’ and ‘now many person also’ is sitting around a circular table facing the
written as ‘zb ct sx ya’ in that code centre, who sits second to the right of A?
Canara Bank (PO) 2018
language.
Statements
II. ‘she visit the also’ is written as ‘sx lm nc I. A sits second to the right of F. Only two
ty’ and ‘she visit the always’ is written as people sit between A and D. B is neither
‘lm kc ty nc’ in that code language. an immediate neighbour of D nor F.
310 How to Crack Test of Reasoning ANALYTICAL

II. Only one person sits between A and F (c) If the statement I and II are sufficient to answer
(either from left or right). Only two people the question, but statement III is not sufficient to
answer the question.
sit between F and B. C sits to the
(d) If all the statements are sufficient enough to
immediate left of D. D is not an immediate answer the question.
neighbour of B. (e) If statement I and III are sufficient to answer the
question but statement II alone is not sufficient
12. Five boxes viz. A, B, C, D and E are kept one to answer the question IBPS Clerk (Mains) 2016
above the other in a stack. How many boxes
are kept between box E and box B? 14. Which of the following represents ‘come’ in
Canara Bank (PO) 2018 a code language?
Statements I. ‘pit na ja od’ means ‘you may come here’
I. Only two boxes are kept between box C in that language.
and box D. Only one box is kept II. ‘ja ta ter’ means ‘come and go’ in that code
between C and box B. Only two boxes language.
are kept between box E and box A. Box E III. ‘od na pit ter’ means ‘you may go home’ in
is kept at one of the positions above box that code language.
A and below box C.
15. How many sons does D have?
II. No box is kept between box D and box B.

A
I. B and F are brothers of A.
Only two boxes are kept between box C

M
II. C is the sister of A and F.
and box D. Box E is kept at one of the

H
positions above box A but below Box C. III. C and E are daughters of D.

TT
13. Among six people viz. C, D, E, F, G and H, Directions (Q. Nos. 16-18) Each of the question
sitting in a straight line with equal distance, given below consists of a question and three
between each other and facing North, who A
statements numbered I, II and III. You have to
W
sits second to the left of G? decide whether the data provided in the statements
are sufficient to answer the question.
SH

Canara Bank (PO) 2018


IBPS (Clerk) 2011
Statements
I. C sits third from the left end of the line Give Answer
A

(a) If the data in Statements I and II are sufficient to


Only one person sits between C and H.
@

answer the question while the data in Statement


Only two people sit between E and G. III are not required to answer the question
G sits at one of the position to the right (b) If the data in Statements I and III are sufficient
of E. to answer the question, while the data in
Statement II are not required to answer the
II. E sits third to the left of G. G does not sit at question
any extreme end of the line. More than (c) If the data in Statements II and III are sufficient
three people sit between H and F. F sits at to answer the question, while the data in
one of the positions to the right of H. Statement I are not required to answer the
question
Directions (Q. Nos 14 and 15) Each of the (d) If the data in either Statement I alone or
question below consists of a question and three Statement II alone or Statement III alone are
statements numbered I, II and III. You have to sufficient to answer the question
decide whether the data provided in the (e) If the data in all the Statements I, II and III
statements are sufficient to answer the questions. together are necessary to answer the question

Read all the three statements and give answer 16. Among six people P, Q, R, S, T and V each
(a) If the statements II and III are sufficient to lives on a different floor of a six storey
answer the question, but statement I alone is not building having six floors numbered one to
sufficient to answer the question. six (the ground floor is numbered1, the floor
(b) If all the statement taken together are not above it, number 2 and so on and the
sufficient to answer the question topmost floor is numbered 6). Who lives on
the topmost floor?
Chapter 28 DATA SUFFICIENCY 311

I. There is only one floor between the floors on III. Both N and W are placed
which R and Q live, P lives on an even numbered immediately next to S. The word
floor. does not begin with R. A is not
II. T does not live on an even numbered floor. Q placed immediately next to W.
lives on an even numbered floor. Q does not live 18. Point D is in which direction with
on the topmost floor. respect to point B?
III. S lives on an odd numbered floor. There are two I. Point A is to the West of point B.
floors between the floors on which S and P live. Point C is to the North of point B.
T lives on a floor immediately above R’s floor. Point D is to the South of point C.
17. There are six letters W, A, R, S, N and E. Is II. Point G is to the South of point D.
‘ANSWER’ the word formed after performing the Point G is 4 m from point B. Point D
following operations using these six letters only? is 9 m from point B.
I. E is placed fourth to the right of A. S is not III. Point A is to the West of Point B.
placed immediately next to either A or E. Point B is exactly midway between
II. R is placed immediately next (either left or right) points A and E. Point F is to the
to E. W is placed immediately next (either left or South of Point E. Point D is to the

A
right) to S. West of point F.

M
Answers

H
TT
Base Level Exercise
1. (e) 2. (b) 3. (d) 4. (d) 5. (c) 6. (d) 7. (a) 8. (d) 9. (d) 10. (e)
11. (e) 12. (d) 13. (b) 14. (c) A
W
Expert Level Exercise
SH

1. (a) 2. (b) 3. (a) 4. (c) 5. (d) 6. (c) 7. (b) 8. (d) 9. (d) 10. (b)
A

11. (b) 12. (a) 13. (d) 14. (c) 15. (b) 16. (e) 17. (b) 18. (b)
@

Answer with Explanations


Base Level Exercise
1. (e) From Statement I, P > O > N 3. (d) From both Statements it is clear that Meena,
From Statement II, L>M the grandmother of Bharat has only one
Here, we cannot find the shortest and tallest by both granddaughter and Shiela the mother of Bharat
the statements. Hence, the data in both the has only three children. It means one is Bharat,
statements are not sufficient to answer the question. second his sister and third one definitely his
brother because grandmother has only one
2. (b) From Statement II, granddaughter, So, Bharat has one brother.
N Hence, both the statements together are necessary.
W E 4. (d) From Statement I, T
A 2m D 4m B Son
S Couple
P R
Now, it is clear form the diagram that, point A is in s r
West direction from point B. Mother
Hence, statement II alone is sufficient.
n
So

Q
r
312 How to Crack Test of Reasoning ANALYTICAL

From Statement II, 8. (d) From Statement I, Madhu has three siblings.
Married From Statement II, Madhu has three brothers.
Couple So, from both statements, we can say that Madhu
T L
has no sister.
Da

Father
u gh 9. (d) From Statements I and II,
te
r C–
N
Mother
From Statement I, it is clear that T is either the

grandmother or grandfather of Q. A
Sister
And, from Statement II, it is clear that T is female.
So, T is the grandmother of Q. Hence, both the Mother
statements together are necessary. –
R B+
5. (c) From Statement I, Brother
Arun Hence, R is the granddaughter of C.

Amit 10. (e) From Statement I,

A
Leena

M
C

H
Hence, Leena faces outside.
From Statement II,

TT
From Statement II, D
Ali
A C A
W
Amit
B
SH

So, from Statement II, we can say that there is no


Ken
person between C and D.
A

11. (e) From Statement I,


@

Hence, Ali faces outside.


M > O, P, N
It is clear from diagrams that all the five friends are
Nothing is given about Q.
not facing the centre of circular table.
Hence, either Statement I or II alone is sufficient to From Statement II,
answer the question. ...... P > O
6. (d) Statement I Birthday according to the Here, P earns more than only O. So, O earns
brother of Meena 26, 27, 28. least. Hence, only Statement II is sufficient to
answer the question.
Statement II Birthday according to the father of
Meena 28, 29, 30. 12. (d) From Statement I,
As, common date = 28
So, both the statements are necessary to answer E A C B D
or
the question.
7. (a) From Statement I, A C B D E
144 $16 H 7 # 9 From Statement II, E is not an immediate
144
⇒ 144 ÷ 16 × 7 + 9 = × 7 + 9 = 63 + 9 neighbour of D and B sits second to the right of A.
16 From Statements I and II,
= 72
So, Statement I alone is sufficient to answer the E A C B D
question.
Hence, C sits exactly in the middle of the line.
Chapter 28 DATA SUFFICIENCY 313

13. (b) From Statement I 3. (a) From Statement I


P U W T R

hter
From, Statement II,

g
d-dau
S U ....... W T Couple
I M
(–)
C
(+)

Hence, data in Statement I alone is sufficient to

Gr a n

ce
answer the question, while data in Statement II is

Nie
not. Daughter
(–)
14. (c) From Statement I, P (–)
G

10 m P N So, the data in Statement I is sufficient to answer


S O the question.
2.5 m 5 m 2.5 m W E From Statement II,
M D K
8m Couple
S R A
Clearly, point M, is 10m away from point K. h ild
d -c Daughter
From Statement II, Point K is 10m to the East of an
r (–)
point M. G M

A
Hence, data in either of the two Statements is Mother-in-law
sufficient to answer the question. P

M
G L
Couple
Expert Level Exercise

H
Hence, the data in Statement II is not sufficient to

TT
1. (a) From Statement I, >K/N>M>J/O give answer.
But, neither K nor N is tallest. Hence, L is tallest. 4. (c) From Statement I,
No conclusion can be drawn from Statement II. 5 N A
W
Hence, the data in Statement I alone is sufficient to 4 Q
SH

answer the question. 3 M


2 O
2. (b) From Statement I,
1 P
A

M So, the data in Statement I is sufficient to give answer.


@

N
2m N-W From Statement II,
4m N-E
Q P 5 N
W E 4 Q
3 M
S-W S-E 2 O
S 8m K S 1 P
Direction of P with respect to Q is not clear. So, So, the data in Statement II is sufficient to give
data is not sufficient. answer.
From Statement II, 5. (d) From Statement I,
M 12 m V V
J Row-1
6m or
Row-2
Q Q P R R P Q
K
So, data in Statement I is not sufficient to give the
Hence, data in Statement II is sufficient to give
answer.
answer.
314 How to Crack Test of Reasoning ANALYTICAL

From Statement II, Now, from Statements I and II,


Row-1 V/U W U/V U/V W V/U F
or
Row-2 B C
Q P R R P Q
So, data in Statement II is not sufficient to give D
E
the answer. A
6. (c) From Statement I, Clearly, C sits to the immediate left of F.
he also show data sx fa mn ca Hence, data in both the statements are required
to answer the question.
now many person also zb ct sx ya
9. (d) From Statement I,
∴also = sx Months according to Sharvan’s mother = October,
December
From Statement II,
From Statement II,
she visit the also sx lm nc ty
Months according to sharvan’s father = February
she visit the always lm kc ty nc to November

A
∴ also = sx ∴From Statements I and II,

M
Hence, the data in either Statement I or II in Sharvan’s birthday = October.

H
Statement II alone is sufficient to answer the
Hence, the data in both the statements are
qustion.

TT
necessary to answer the question.
7. (b) From Statement II,
10. (b) From Statement I,
O Q
N-W
N
B 6m A F 3m C 3m B
W
N-E
SH

W E N
L P 6m 9m
A

S-W S-E W E
N M S B
@

S
Clearly, Q is in North-West direction with respect
to P. A 6m G
Hence, data in Statement II alone is sufficient.
We can not determine the position of B.
8. (d) From Statement I, From Statement II,
A A
M 8m B

E C or C E
8m C
F F
From Statement II, A
R 11 m
F
So, B is in the North-West direction with respect
to A.
Thus, Statements II alone is sufficient to answer
E D the question.
A
Chapter 28 DATA SUFFICIENCY 315

11. (b) From Statement I, 15. (b) From all the three statements, the sex of A is
B not clear. So, we cannot determine how many
C/E sons D have.
16. (e) From the given statements, we get
D A Floors People
6 P
5 T
C/E 4 R
F 3 S
C or E is second to right of A. 2 Q
From Statement II, 1 V
D Therefore, the data in Statements I, II and III
F together are necessary to answer the question.
17. (b) From Statement I,
E C A _ S _ E _ or _A_S_E
From Statement II,

A
B E R _ or R E and S W or W S

M
A From Statement III, N S W or W S N
C is sitting second to the right of A.

H
A is not placed next to W and the word does not
Hence, Statement II alone is sufficient to answer

TT
begin with R.
the question. From Statements I and III together,
12. (a) From Statement I,
C
ANSWER
A
W
Hence, data only in Statements I and III are
E
sufficient to answer the question, while the data in
SH

B
Statement II is not required to answer the question.
D
A 18. (b) From Statement I,
A

C C
So, no box is kept between box E and box B.
↑ ↓
@

Hence, statement I alone is sufficient to answer A ← B, D


the question.
From Statement III,
13. (d) From Statements I and II, A ← B → E

H E C D G F D ← F
Hence, C sits second the left of G. The data in By combining Statements I and III, we get
both Statements I and II are together necessary to C
answer the question.
14. (c) From Statement I, A B E
You may come here → pit na ja od
From Statement II, D F
come and go → ja ta ter So, the data in Statements I and III are sufficient
From Statements I and II together, come → ja to answer the question, while the data in
Hence, it is clear that Statements I and II are Statement II is not required to answer the
sufficient to answer the question. question.
316 How to Crack Test of Reasoning ANALYTICAL

29
Input-Output
Input is a group of words, numbers or Points to be remembered while solving
data that is fed into the system to input-output questions
achieve output or a result, while output l
First of all, observe the given input line of words or

A
means an artifact of the arrangement numbers and the last step of rearrangement, so

M
that has been created by someone or that candidate may get an idea about the changes
some process.

H
in various steps of rearrangement.
In this chapter, the questions are based

TT
l
In order to know what changes have been made in
on a given input of line of words or each step, observe two consecutive steps
numbers or combination of words and
numbers that are organised step-by-step
carefully.
A
W
l
Now, correlate the input, the last step and any one
and follow a specific rule to arrive at the
of the middle steps. This will enable you to
SH

final step.
identify the rule of arrangement.
Candidates are required to observe the
Observe the pattern to know that how many
A

change in order of words or numbers in


elements are arranged in each step i.e. one or
@

each step sequentially to specify the


more than one.
rule of arrangement of words and
numbers.
l
The arrangement can be done in any direction, i.e.
from left to right or right to left.
Generally, each set of questions on
input-output is based on a particular
l
The words can also be arranged according to
rule. The different sets of questions vowels and consonants.
involve different rules of arrangement of
words or numbers. Types of Questions
Most common type of questions on
input-output involves arrangement of There are mainly four types of questions that are asked
words as per dictionary or an from input-output i.e. shifting, arranging,
alphabetical order or arrangement of mathematical operations and miscellaneous.
numbers in ascending or descending
order. Type 1
Generally, one word or number or both
are arranged in every step and
Problems Based on Shifting
procedure is continued until all the In this type of questions, the given words of the
words or numbers or both get arranged input are shifted from one position to another
according to a specific rule. according to a certain set pattern.
Chapter 29 INPUT-OUTPUT 317

Directions (Illustrations 1-5) Read the following information carefully and answer the questions.
In a SSB interview for the NDA, a group testing officer allocates the following instructions to the
candidates divided into groups, who have to perform the tasks according to the input.
Group I Monkey crawling long jump and rope climbing.
Group II Monkey crawling rope jump and long climbing.
Group III Rope crawling monkey climbing long and jump.
Group IV Climbing monkey crawling rope jump and long.
Group V Climbing monkey and rope jump crawling long.
1. If the task of II group is ‘Nothing is without be achieved to determination’, then which will be the
VI group?
(a) Without is nothing determination to achieved to (b) Achieved nothing determination to is be without
(c) Nothing is achieved without be to determination (d) Data is inadequate
(e) None of the above
2. If ‘Terrorism bush job incurbing remarkable did a’ is V group task, then which is ‘Bush did
incurbing remarkable job a terrorism’ group?
(a) V (b) IV (c) II (d) VI (e) None of these

A
3. Asad is first selected for I group “You are my inspiration, guide and instructor”, after sometime he

M
was shifted to group IV. What is his new group task?
(a) Instructor you are and inspiration guide my (b) You are my guide instructor and inspiration

H
(c) Inspiration and guide instructor you are my (d) Data inadequate

TT
(e) None of the above
4. If the last VII group is “Not job you for this selected are”, then which group is I?
(a) You are selected for this not job A
(b) Selected for this job you are not
W
(c) Not you are selected for this job (d) You are not selected for this job
SH

(e) None of the above


5. If the task of VI group is “Pollution is becoming cause of deadly diseases”, then what is III group?
A

(a) Pollution is cause of deadly diseases becoming (b) Diseases becoming is cause of deadly pollution
(c) Becoming is pollution diseases deadly of cause (d) Data inadequate
@

(e) None of the above


Solutions (Illustrations 1-5) The given input follows the pattern shown below.
Group I to II Based on arrangement of words to different positions. The third word of previous group becomes
sixth and sixth become third, interchanging their places.
Group II to III First three words get reversed and so the group of last four.
Group III to IV This time we reverse the group of first 4 words and then the last three.
Group IV to V In group V same rule follows as in group I to group II.
To solve the problems more easily, we can give numbers to each word.
Group I Monkey crawling long jump and rope climbing
(1) (2) (3) (4) (5) (6) (7)
Group II Monkey crawling rope jump and long climbing
(1) (2) (6) (4) (5) (3) (7)
Group III Rope crawling monkey climbing long and jump
(6) (2) (1) (7) (3) (5) (4)
Group IV Climbing monkey crawling rope jump and long
(7) (1) (2) (6) (4) (5) (3)
Group V Climbing monkey and rope jump crawling long
(7) (1) (5) (6) (4) (2) (3)
Group VI And monkey climbing long crawling jump rope
(5) (1) (7) (3) (2) (4) (6)
Group VII Long climbing monkey and rope jump crawling
(3) (7) (1) (5) (6) (4) (2)
318 How to Crack Test of Reasoning ANALYTICAL

1. (b) Group II-nothing (1) is (2) without (6) be 6. Step III of an input is ‘‘15 yes 24 80 today
(4) achieved (5) to (3) determination (7). With same never go 59’’, which of the following will
logic group VI will be, achieved (5) nothing definitely be the input?
(1) determination (7) to (3) is (2) be (4) without (6). (a) 24 80 today never go 59 15 yes
2. (c) Group V-terrorism (7) bush (1) job (5) incurbing (b) 24 80 today yes never go 59 15
(6) remarkable (4) did (2) a (3). It can also be (c) 24 15 yes 80 today never go 59
solved by going back but the above system is (d) Cannot be determined
more convenient. So, Bush (1) did (2) incurbing (e) None of the above
(6) remarkable (4) job (5) a (3) terrorism (7) will be 7. Input man 79 over 63 like 43 joy 15. How
group II. many steps will be required to complete the
3. (a) Group I-you (1) are (2) my (3) inspiration (4) rearrangement?
guide (5) and (6) instructor (7). Group IV will be, (a) Six (b) Seven (c) Eight (d) Nine
instructor (7) you (1) are (2) and (6) inspiration (4) (e) None of these
guide (5) my (3). 8. Step III of an input is 18 tower 38 basket 82
4. (d) Last group VII-not (3) job (7) you (1) for (5) this 76 hall new, which of the following will be
(6) selected (4) are (2). With same logic I group is, step VII?
you (1) are (2) not (3) selected (4) for (5) this (6) job (a) 18 tower 38 hall 76 new basket 82

A
(7). (b) 18 tower 38 new 76 hall 82 basket
(c) 18 tower 38 hall 76 new 82 basket

M
5. (e) Group VI-pollution (5) is (1) becoming (7) cause (d) There will be no such step
(3) of (2) deadly (4) diseases (6). Group III will be,

H
(e) None of the above
diseases (6) of (2) is (1) becoming (7) cause

TT
(3) pollution (5) deadly (4). 9. Input 94 join for 81 style home 32 48, which
of the following steps will be the last?
Type 2 (a) VI A
(b) V (c) VII (d) IX
W
(e) None of these
Problems Based on Arrangement
SH

10. Step II of an input is 27 world go 57 48 stem


35 kite. How many more steps will be
In this type of questions, the numbers or required to complete the rearrangement?
A

words are arranged as per a given order. The (a) Five (b) Four (c) Three (d) Six
@

given order can be alphabetical order in case (e) None of these


of words and ascending or descending order Solutions (Illustrations 6-10)
in case of numbers. 6. (d) Input cannot be determined as we cannot judge
Directions (Illustrations 6-10) Study the the initial position of words and numbers.
following information carefully and answer the 7. (e) Input man 79 over 63 like 43 joy 15
given questions. Step I 15 man 79 over 63 like 43 joy
Step II 15 over man 79 63 like 43 joy
A word and number arrangement machine when
Step III 15 over 43 man 79 63 like joy
given an input line of words and numbers
Step IV 15 over 43 man 63 79 like joy
rearranges them following a particular rule in
Step V 15 over 43 man 63 like 79 joy
each step. The following is an illustration of input
So, five steps are required to complete the
and rearrangement.
rearrangement.
Input rose petal 29 32 86 goal 41 toll 8. (b) Step III 18 tower 38 basket 82 76 hall new
Step I 29 rose petal 32 86 goal 41 toll Step IV 18 tower 38 new basket 82 76 hall
Step II 29 toll rose petal 32 86 goal 41 Step V 18 tower 38 new 76 basket 82 hall
Step III 29 toll 32 rose petal 86 goal 41 Step VI 18 tower 38 new 76 hall basket 82
Step IV 29 toll 32 rose 41 petal 86 goal Step VII 18 tower 38 new 76 hall 82 basket
Step IV is the last step of rearrangement. So, the correct step VII is given in option (b).
Chapter 29 INPUT-OUTPUT 319

9. (a) Input 94 join for 81 style home 32 48 12. If in the first step, ‘59’ interchanges its
Step I 32 94 join for 81 style home 48 position with ‘lakes’ and ‘omits’ also
Step II 32 style 94 join for 81 home 48 interchanges its position with ‘12’ then
Step III 32 style 48 94 join for 81 home which element will be to the immediate left
of ‘41’?
Step IV 32 style 48 join 94 for 81 home
(a) 59 (b) omits
Step V 32 style 48 join 81 94 for home
(c) 12 (d) elect
Step VI 32 style 48 join 81 home 94 for (e) lakes
So, last step is step VI.
13. Which of the following combinations
10. (a) Step II 27 world go 57 48 stem 35 kite represent the first two and the last two
Step III 27 world 35 go 57 48 stem kite elements in the step V of the given input?
Step IV 27 world 35 stem go 57 48 kite (a) 41, train and omits, 36
Step V 27 world 35 stem 48 go 57 kite (b) 12, lakes and 36, elect
Step VI 27 world 35 stem 48 kite go 57 (c) 12, lakes and omits, 36
Step VII 27 world 35 stem 48 kite 57 go (d) lakes, 41 and 36, elect
So, five more steps are required to complete the (e) lakes, 41 and 59, omits
rearrangement. 14. Which element is fourth to the right of the

A
Directions (Illustrations 11-15) Study the one which is ninth from the right end in
step V of the give input?

M
following information carefully and answer the
given questions. (a) 36 (b) yards

H
(c) 85 (d) elect
When a word and number arrangement machine

TT
(e) 12
is given an input line of words and numbers, it
arranges them following a particular rule. The 15. In which step are the elements ‘train 85
following is an illustration of input and A
yards 59’ found in the same order?
W
rearrangement. (All the numbers are two digit (a) Third (b) Fourth
(c) Second (d) Fifth
SH

numbers) Andhra Bank (PO) 2015


(e) The given order of elements is not found in any
Input 41 sprain 10 early 97 noble 26 65 ankle step.
A

death.
Solutions (Illustrations 11-15) According to the given
@

Step I 97 41 10 early noble 26 65 ankle death information, we have the following arrangement
sprain. Input omits 36 59 yards 41 elect train 12 lakes 85
Step II noble 97 41 10 early 26 ankle death Step I 85 omits 36 59 41 elect train 12 lakes yards
sprain 65 Step II train 85 omits 36 41 elect 12 lakes yards 59
Step III 41 noble 97 10 26 ankle death sprain Step III 41 train 85 36 elect 12 lakes yards 59 omits
65 early Step IV lakes 41 train 85 elect 12 yards 59 omits 36
Step IV death 41 noble 97 10 ankle sprain Step V 12 lakes 41 train 85 yards 59 omits 36 elect
65 early 26
Step V 10 death 41 noble 97 sprain 65 early 11. (e) ‘elect’ comes exactly between ‘85’ and lakes in
step III.
26 ankle
Step V is the last step of the above arrangement 12. (e) After interchanging, we get
as the intended arrangement is obtained. 85 12 36 lakes 41 elect train omits 59 yards.
As per the rules followed in the given steps, find ∴ ‘lakes’ is to the immediate left of ‘41’.
out the appropriate steps for the given input. 13. (b) ‘12 lakes’ are first two and ‘36 elect’ are last two
Input omits 36 59 yards 41 elect train 12 lakes 85 elements in the step V.
11. Which element comes exactly between 85 14. (b) ‘yards’ is fourth to the right of the ninth element
and lakes in Step III of the given input. from the right end in step V.
(a) 59 (b) yards (c) train (d) omits 15. (d) ‘train 85 yards 59’ are found in the same order in
(e) elect step V.
320 How to Crack Test of Reasoning ANALYTICAL

Type 3
Problems Based on Mathematical Operations
In this type of questions, the input has some 19. What will be the step V of the following
numbers. Different steps are obtained by input?
taking the numbers of the input and different Input 37, 48, 91, 22, 49
arithmetic operations are performed after that. (a) 10, 12, 10, 4, 13
Directions (Illustrations 16-20) Study the (b) 4, 4, 1, 3, 1
following information to answer the following (c) 1, 3, 1, 4, 4
questions. (d) Cannot be determined
(e) None of the above
A number arrangement machine, when given a
particular input, rearranges it following a 20. Find the step III of the following input.
particular rule. The following is the illustration of Input 17, 50, 37, 23, 35
the input and the steps of the arrangement. (a) 51, 150, 111, 69, 105
Input 44 35 18 67 22 28 36 (b) 8, 5, 10, 6, 8
(c) 150, 51, 111, 69, 105
Step I 36 27 10 59 14 20 28

A
(d) Cannot be determined
Step II 16 15 8 42 4 16 18 (e) None of the above

M
Step III 132 105 54 201 66 84 108 Solutions (Illustrations 16-20) The steps are as follow

H
Step IV 50 41 24 73 28 34 42 Step I (Each number of the input) – 8

TT
Step V 8 8 9 4 4 1 9 Step II Product of the digits of each number of the
input
Step VI 64 64 81 169 16 100 81
Step VII 20 19 12 46 8 20 22 A
Step III (Each number of the input) ×3
Step IV (Each number of the input) +6
W
16. What will be the 4th step of the following Step V Keep adding the digits of each number of
SH

input? the input till they are converted into single


digit
Input 24, 88, 22, 34, 81, 90, 38
Step VI (Digit sum of each number of input) 2
A

(a) 30, 94, 28, 40, 87, 92, 40


Step VII (Each number of step II) +4
@

(b) 30, 94, 28, 40, 87, 96, 44


(c) 44, 96, 87, 40, 28, 94, 30 16. (b) Step IV = (Each number of the input) + 6
(d) Cannot be determined 17. (a) Step VII = (Each number of step II) + 4
(e) None of the above 18. (a) As given arrangement
17. The second step of a given input is 45, 27, 35, = (Digit sum of each number of input) 2
28, 42, 15. What will be step VII for the Which is the logic of step VI.
input? 19. (c) As in step V, digits of each number of given input
(a) 49, 31, 39, 32, 46, 19 (b) 50, 31, 40, 22, 37, 19 are added till each number get converted into
(c) 19, 46, 32, 39, 31, 49 (d) Cannot be determined single digit. Let us see
(e) None of these 37 ⇒ 3 + 7 = 10 ⇒ 1 + 0 = 1
48 ⇒ 4 + 8 = 12 ⇒ 1 + 2 = 3
18. In how many steps would the following 91 ⇒ 9 + 1 = 10 ⇒ 1 + 0 = 1
arrangement be yielded by the given input? 22 ⇒ 2 + 2 = 4
Input 91, 45, 67, 51, 32, 17 49 ⇒ 4 + 9 = 13
Arrangement 100, 81, 169, 36, 25, 64 ⇒ 1+ 3 = 4
(a) VI (b) III (c) V (d) VII 20. (a) Step III = (Each number of the input) × 3
(d) None of these
Chapter 29 INPUT-OUTPUT 321

Directions (Illustrations 21-25) Study the 25. If digit is exchanged within the each block
given information carefully and answer the then find the multiplication of two new
questions basen on it. numbers obtained in step II.
An input-output is given in different steps. Some (a) 1500 (b) 1812 (c) 1902 (d) 1802
mathematical operations are done in each step. (e) None of these
No mathematical operation is repeated in next Solutions (Illustrations 21-25) In the given arrangement,
step but it can be repeated with some other the patterns followed in each step are as follow
mathematical operation (as multiplication can be
Input A B C D E F G H I J K L
used with subtraction in step 1 and same can be
Step I A~C=P E~ G = R I~ K = T
used with addition in step 2).
B~D=Q F~ H = S J~ L = U
3 1 6 5 5 2 9 4 1 5 9 1
PQ R S T U

Step II P + R + T = W
2 2 2
Q + S + U2 = X
2 2

Step I 3 4 4 2 8 4 W X
w1 w2 x1 x2

Step II 8 9 3 6 Step III w1 * x1 = y w2 * x2 = z


Y Z

A
y1 y2 z1 z2
Step III 2 4 5 4
y1x[z1 + z 2 ] + y 2 x [z1 + z 2 ] = ANS

M
Step IV
Step IV 5 4

H
AN S

TT
As per the rules followed in the steps given above,
find out in each of the following questions the Now, the steps of the given input are as
appropriate step for the given input. follow
4 1 1 6 A 2 5 3 8 2 5 7 8
W
4 1 1 6 2 5 3 8 2 5 7 8
4–1

3–2

7–2
=5

=3

=3
21. Find the multiplication of two numbers
SH 6 –1

8 –5

8 –5
=3

=1

=5
obtained in step II.
Step I 3 5 1 3 5 3
(a) 1409 (b) 1505
A

(c) 1615 (d) 1705 (32+12+52) (52+32+32)


@

(e) None of these =35 =43


Step II 3 5 4 3
22. Find the addition of the three numbers
obtained in step I. (3×4)=12 (5×3)=15
(a) 101 (b) 100
(c) 102 (d) 103 Step III 1 2 1 5
(e) None of these 1×(1+5)+2(1+5)
23. Find the difference between the two =1×6+2×6
numbers obtained in step III. =18
Step IV 1 8
(a) 3 (b) 4
(c) 5 (d) 6 21. (b) Required answer = 35 × 43 = 1505
(e) None of these 22. (a) Required answer = 35 + 13 + 53 = 101
24. Find the cube of number which is obtained 23. (a) Required answer = 15 − 12 = 3
in Step IV. 24. (d) Required answer = (18)3 = 5832
(a) 4096 (b) 3375
25. (d) Numbers obtained after interchanging the digits
(c) 6859 (d) 5832 53 and 34
(e) None of these ∴ Required answer = 1802
322 How to Crack Test of Reasoning ANALYTICAL

Type 4
Miscellaneous Problems
In this type of questions, the input can be 29. If step I of an input is 4, 16, 121, 8, 17, then
anything and machine performs a set of find the input.
random operations on this. (a) 45, 163, 1217, 87, 178
(b) 46, 163, 1213, 85, 172
Directions (Illustrations 26-30) Study the (c) 41, 161, 1216, 82, 176
following informations to answer the given questions.
(d) Cannot be determined
A number arrangement machine when given an (e) None of the above
input line of numbers, rearranges them following
a particular rule in each step. The following is an
30. What will be step IV for the following
illustration of input and steps of arrangement. input?
220, 197, 15, 37, 89, 75
Input 23 132 38 27 430 287 (a) 3, 2, 2, 4, 9, 8
Step I 2 13 3 2 43 28 (b) 1, 8, 6, 8, 10, 6
Step II 3 32 8 7 30 87 (c) 6, 10, 8, 6, 8, 1
(d) 8, 9, 4, 2, 2, 3
Step III 32 231 83 72 034 782
(e) None of the above

A
Step IV 3 2 4 3 5 3 Solutions (Illustrations 26-30)

M
Step V 34 233 49 38 531 388 Step I Rightmost digit of each number of input

H
26. What will be step V for the following input? gets disappeared

TT
Input 135, 88, 24, 215, 16 Step II Leftmost digit of each number of input is
(a) 236, 99, 35, 316, 27 (b) 136, 99, 25, 216, 17 disappeared
(c) 17, 216, 25, 99, 136 (d) Cannot be determined A
Step III Leftmost and rightmost digits of each
number of input are interchanged
W
(e) None of these
Step IV Leftmost digit of each number of input + 1
27. If step II is 16, 9, 22, 416, 25, 67, then find the
SH

Step V Leftmost and rightmost digits of each


input. number of input increase by 1
(a) 18, 11, 24, 418, 27, 69 26. (a) In step V, leftmost and rightmost digits of each
A

(b) 11, 18, 24, 418, 27, 69 number of input increase by 1.


@

(c) 69, 27, 418, 24, 11, 18 27. (d) In step II leftmost digit of each number of input
(d) Cannot be determined gets disappeared but it is impossible to get
(e) None of these leftmost digit of the numbers of input from
28. What will be the step III for the following step II.
input? 28. (c) In step III, leftmost and rightmost digits of each
number of input are interchanged.
Input 777, 29, 435, 115, 61, 37
29. (d) In step I, rightmost digit of each number of input
(a) 777, 92, 436, 116, 62, 37
gets disappeared but practically it is impossible
(b) 777, 92, 534, 16, 511, 73
to find out rightmost digit of input from step I.
(c) 777, 92, 534, 511, 16, 73
30. (a) ∴ Step IV = Leftmost digit of each number of
(d) Cannot be determined
input + 1
(e) None of the above
Chapter 29 INPUT-OUTPUT 323

Let us Practice
Base Level Exercise
Directions (Q. Nos. 1-5) Study the following 4. If the pass-code for the second batch is,
information to answer the given questions. children for not is good watching television,
Given an input, a coding machine generates what will be the pass-code for the fifth
pass-codes for six batches every day as follows. batch?
Input the shopkeeper offered discount to (a) children for good not is watching television
customer. (b) children for good is not watching television
Pass-code for (c) children good for is not television watching
Batch I customer the shopkeeper offered (d) Cannot be determined
discount to (e) None of the above
Batch II customer discount the shopkeeper 5. If the input is ‘necessary arrangements have
offered to
already been made.’

A
Batch III customer discount offered the
How many batches are required to complete

M
shopkeeper to and so on, until the
arrangement is completed. the arrangement?

H
(a) Three (b) Four
After the arrangement is completed the next

TT
(c) Five (d) Six
batch gets the same code as that for batch I.
(e) None of the above
Duration of each batch is 1 h. There is a break of
1 h after the fourth batch. Sixth batch is the last A
Directions (Q. Nos. 6-10) Read the information
W
batch. Now, answer the following questions. carefully and answer the questions given below.
IDBI (Office Executive) 2018
SH

1. If the pass-code for the second batch is, do


lean window out of not, what will be the A word and number arrangement machine when
pass-code for the fourth batch? given a particular input, rearranges it following a
A

(a) do lean of not out window


particular rule. The following is the illustration of
@

(b) do lean of out not window input and the steps of arrangement.
(c) do lean not out window of Input cup for hot 34 69 72 tea 27
(d) do lean not of window out Step I 27 cup for hot 34 69 72 tea
(e) None of the above Step II 27 tea cup for hot 34 69 72
2. If the input is, please do not delay the Step III 27 tea 34 cup for hot 69 72
matter, what will be the pass-code for third Step IV 27 tea 34 hot cup for 69 72
batch? Step V 27 tea 34 hot 69 cup for 72
(a) do please not delay the matter Step VI 27 tea 34 hot 69 for cup 72
(b) delay do matter please not the
Step VII 27 tea 34 hot 69 for 72 cup
(c) delay do matter not please the
(d) delay do matter not the please And Step VII is the last step of the above
(e) None of the above arrangement
As per the rules followed in the above steps, find
3. If the pass-code for third batch is, brisk every
out in each of the following question, the
for morning go walk, what will definitely be
appropriate step for the given input.
the input?
(a) morning brisk go walk for every 6. Input kind 12 96 heart water 59 42 yes
(b) morning go brisk walk for every How many steps will be required to
(c) morning go walk brisk for every complete the rearrangement?
(d) Cannot be determined (a) Three (b) Four (c) Five (d) Six
(e) None of the above (e) None of these
324 How to Crack Test of Reasoning ANALYTICAL

7. Input Jungle 43 mode 25 basket 39 target 19 11. Step III of an input : year 92 ultra 15 23
Which of the following steps will be the last strive house 39.
but one? How many more steps will be required to
(a) VII (b) VIII (c) IX (d) VI complete the rearrangement?
(e) None of these (a) Three (b) Four (c) Two (d) Five
(e) None of these
8. Step III of an input is : 12 world 31 ask cart
ball 87 75 12. Input any how 49 24 for wide 34 69.
Which of the following will definitely be the Which of the following steps will be the last
input? but one?
(a) 31 ask cart ball 87 75 world 12 (a) VI (b) VII (c) V (d) VIII
(b) 31 ask cart ball 87 75 12 world (e) None of these
(c) 31 ask 12 world cart ball 87 75 13. Step II of an input is town 74 pair 15 31 nice
(d) Cannot be determined job 42. Which of the following is definitely
(e) None of the above the input?
9. Step II of an input is: 24 year 56 43 last part (a) pair 15 31 town nice job 42 74
64 over (b) pair 15 town 31 74 nice job 42
(c) pair 15 town 74 31 nice job 42
How many more steps will be required to

A
(d) Cannot be determined
complete the rearrangement? (e) None of the above

M
(a) Five (b) Six (c) Seven (d) Four

H
(e) None of these Directions (Q. Nos. 14-18) Study the following

TT
10. Step III of an input: 32 station 46 81 73 information carefully and answer the given
march go for questions. IBPS (PO) 2012

Which of the following will be step VI A


A word and number arrangement machine when
W
(a) 32 station 46 march 73 go for 81 given an input line of words and numbers
(b) 32 station 46 march 73 81 go for rearranges them following a particular rule in
SH

(c) 32 station 46 march 73 go 81 for each step. The following is an illustration of


(d) There will be no such step input and rearrangement. (All the numbers given
A

(e) None of the above in the arrangement are two-digit numbers).


@

Input gone over 35 69 test 72 park 27


Directions (Q. Nos. 11-13) Study the following
information carefully and answer the given Step I 27 gone over 35 69 test 72 park
questions. Step II 27 test gone over 35 69 72 park
A word and number arrangement machine when Step III 27 test 35 gone over 69 72 park
given an input line of words and numbers Step IV 27 test 35 park gone over 69 72
rearranges them following a particular rule in Step V 27 test 35 park 69 gone over 72
each step. The following is an illustration of input
Step VI 27 test 35 park 69 over gone 72
and rearrangement.
Input But 32 71 glory fair South 65 84 Step VII 27 test 35 park 69 over 72 gone
Step I South but 32 71 glory fair 65 84 And step VII is the last step of the rearrangement
Step II South 84 but 32 71 glory fair 65 of the above input as the desired arrangement is
obtained.
Step III South 84 glory but 32 71 fair 65
As per the rules followed in the above steps, find
Step IV South 84 glory 71 but 32 fair 65 out the appropriate step for the given input in
Step V South 84 glory 71 fair but 32 65 each of the following questions.
Step VI South 84 glory 71 fair 65 but 32 and 14. Input 86 open shut door 31 49 always 45.
Step VI is the last step of the rearrangement. How many steps will be required to
As per the rules followed in the above steps, find complete the rearrangement?
out in each of the following questions the (a) Five (b) Six (c) Seven (d) Four
appropriate step for the given input. (e) None of these
Chapter 29 INPUT-OUTPUT 325

15. Step III of an input is 25 yes 37 enemy joy And Step V is the last step of the arrangement of
defeat 52 46. the above input as the intended arrangement is
Which of the following is definitely the obtained.
Input? As per the rules followed in the above steps, find
(a) enemy 25 joy defeat yes 52 37 46
out in each of the following questions the
(b) 37 enemy 25 joy yes defeat 52 46
appropriate steps for the given input.
(c) enemy joy defeat 25 52 yes 46 37 Input for 52 all 96 25 jam road 15 hut 73 bus
(d) Cannot be determined stop 38 46 (All the numbers given in the
(e) None of the above arrangement are two-digit numbers).
16. Step II of an input : 18 win 71 34 now if 19. Which word/number would be at the 6th
victory 61. position from the left in step V?
(a) 25 (b) stop (c) jam (d) all
How many more steps will be required to
(e) road
complete the rearrangement?
(a) Three (b) Four (c) Five (d) Six 20. Which of the following would be the step III?
(e) More than six (a) hut for bus all 25 jam road 15 stop 38 96 73 52 46
(b) for bus all 25 jam road 15 hut 38 stop 96 46 73 52
17. Input where 47 59 12 are they going 39.
(c) hut for bus all jam road 15 stop 38 96 73 52 46 25
Which of the following steps will be last but

A
(d) for bus all 25 jam road 15 hut stop 38 46 96 73 52
one?

M
(e) None of the above
(a) VII (b) IV (c) V (d) VIII
21. Which word/number would be at 8th

H
(e) None of these
position from the right in step IV?

TT
18. Step II of an input is 33 store 81 75 full of (a) 15 (b) road (c) hut (d) jam
goods 52. (e) stop
Which of the following will be step VI? A
22. Which of the following would be step VII?
W
(a) 33 store 52 of 75 81 full goods (a) stop road jam hut for bus all 15 96 73 52 46 38 25
SH

(b) 33 store 52 of 75 full 81 goods (b) road jam hut for bus all stop 15 25 38 46 52 73 96
(c) 33 store 52 of 75 goods 81 full (c) stop road jam hut for bus all 96 73 52 46 38 25 15
(d) There will be no such step
A

(d) jam hut for bus all 25 road stop 15 96 73 52 46 38


(e) None of the above
@

(e) There will be no such step as the arrangement


Directions (Q. Nos. 19-23) Study the following gets established at step VI
information to answer the given questions. 23. Which step number would be the following
IBPS (Clerk) 2011
output?
A word and number arrangement machine when bus all for 52 25 jam road 15 hut stop 38 46
given an input line of words and numbers 96 73.
rearranges them following a particular rule. The (a) VI (b) III (c) II (d) V
following is an illustration of input and (e) There will be no such step
rearrangement. (All the numbers are two-digit
numbers). Directions (Q. Nos. 24-28) Study the following
Input sine 88 71 cos theta 14 56 gamma information to answer the given questions.
delta 26 A word and number arrangement machine when
Step I cos sine 71 theta 14 56 gamma delta 26 given an input line of words and number
88 rearranges them following a particular rule. The
Step II delta cos sine theta 14 56 gamma 26 following is an illustration of output and
88 71 rearrangement. NICL (AO) 2014
Step III gamma delta cos sine theta 14 26 88 Input 52 peak 91 snow freeze 46 cold 15 high
71 56 31 73 trek
Step IV sine gamma delta cos theta 14 88 71 Step I 15 52 peak snow freeze 46 cold high 31
56 26 73 trek 91
Step V theta sine gamma delta cos 88 71 56 Step II 15 31 52 peak snow freeze 46 cold high
26 14 trek 73 91
326 How to Crack Test of Reasoning ANALYTICAL

Step III 15 31 46 peak snow freeze cold high Directions (Q. Nos. 29-33) Study the following
trek 52 73 91 information to answer the following questions.
Step IV 15 31 46 cold peak snow freeze high A number arrangement machine, when given an
trek 52 73 91 input line of numbers, rearranges them following
Step V 15 31 46 cold freeze peak snow high a particular rule in each step. The following is the
trek 52 73 91 illustration of the input and the steps of
Step VI 15 31 46 cold freeze high peak snow arrangement.
trek 52 73 91 Input 13 25 9 17 15 32 7 20
Step VI is the last step of the rearrangement. Step I 8 20 4 12 10 27 2 15
As per the rules followed in the above steps, find Step II 22 34 18 26 24 41 16 29
out in each of the following questions the Step III 121 529 49 225 169 900 25 324
appropriate steps for the given input.
Step IV 169 625 81 289 225 1024 49 400
Input 67 hot sun 19 best 83 ice 49 ace 77 cut
Step V 144 576 64 256 196 961 36 361
37
Step VI 676 2500 324 1156 900 4096 196 1600
24. How many steps would be needed to
complete the rearrangement? 29. What will be the fourth step of the following
(a) X input?

A
(b) VIII Input 8 18 28 38 48 58 68 27

M
(c) IX (a) 46, 289, 1444, 2304, 3364, 4624, 729
(d) VII (b) 64, 324, 784, 1444, 2304, 3364, 4624, 729

H
(e) None of the above (c) 64, 324, 748, 1444, 2364, 4624, 729

TT
25. Which step number would be the following (d) 64, 748, 324, 1444, 2304, 3364, 4624, 729
output? (e) None of the above
19 37 49 ace best hot sun ice cut 67 77 83 A
30. The sixth step for a given input is
W
(a) II (b) VI (c) V (d) IV 1156, 5776, 2304, 324, 1296, 2916,7056, 576
SH

(e) None of these What will be the input?


26. Which of the following would be the step I? (a) 15, 36, 22, 7, 16, 25, 40, 10
A

(a) 19 37 49 hot sun best ice ace cut 67 77 83 (b) 17, 38, 24, 9, 18, 27, 42, 12
@

(b) 83 67 hot sun best ice 49 ace 77 cut 37 19 (c) 19, 40, 26, 11, 20, 29, 44, 14
(c) 19 67 ace best hot sun ince 49 77 cut 37 83 (d) 21, 42, 28, 13, 22, 31, 46, 16
(d) 19 67 hot sun best ice 49 ace 77 cut 37 83 (e) None of the above
(e) None of the above
31. In how many steps in the input given below,
27. Which of the following would be the final the following arrangement would be arrived?
arrangement? Input 11, 17, 22, 34, 8, 25, 38, 43
(a) 67 77 83 ace best cut hot ice sun 19 37 49 Arrangement
(b) 19 37 49 ace best cut hot ice sun 67 77 83
(c) 19 37 49 67 77 83 ace best cut hot ice sun
100, 256, 441, 1089, 49, 576, 1369, 1764
(d) 19 37 49 ace ice best cut hot sun 67 77 83 (a) 3 (b) 5 (c) 4 (d) 6
(e) None of the above (e) None of these

28. In Step IV, which of the following 32. The second step of a given input is
word/number would be on 7th position 57, 41, 37, 53, 44, 26, 17, 32
(from the right)? What will be the input?
(a) sun (a) 109, 1404, 38, 58, 1928, 32, 829, 925
(b) best (b) 2304, 1024, 784, 1936, 1225, 289, 64 529
(c) 67 (c) 4023, 1024, 874, 1936, 1225, 289, 64, 529
(d) cut (d) 2304, 1024, 874, 1936, 1225, 289, 64, 529
(e) None of the above (e) None of the above
Chapter 29 INPUT-OUTPUT 327

33. What would be step VI for the following 34. What will be the step V for the following
input? input?
8, 12, 16, 24, 6, 15, 2, 13 Input : you are the one great person
(a) 17, 21, 25, 33, 15, 24, 11, 21 (a) u e e e eat rson
(b) e e u e eat rson
(b) 3, 7, 11, 19, 1, 10, − 3, − 8
(c) yo ar th on gr pe
(c) 256, 576, 1024, 2304, 144, 900, 16, 676
(d) Cannot be determined
(d) 64, 144, 256, 576, 36, 225, 4, 169 (e) None of the above
(e) None of the above
35. If step IV of an input is ‘e i a oo ou’, then
Directions (Q. Nos. 34-36) Study the following find the input.
information to answer the given questions. (a) get will an spoon you
A word arrangement machine when given an (b) left wit at noon you
input line of words, rearranges them following a (c) net wit at noon you
particular rule in each step. The following is an (d) Cannot be determined
illustration of input and steps of arrangement. (e) None of the above
Input every now and then same 36. Find step II for the following input.
Step I very ow nd hen ame Input : you must be agree
Step II ever no an the sam (a) ou ust e gree

A
(b) yu mst be ee
Step III vry nw nd thn sm

M
(c) yo mus b agre
Step IV ee o a e ae

H
(d) Cannot be determined
Step V ery w d en me (e) None of the above

TT
Expert Level Exercise
A
W
Directions (Q. Nos 1-5) Study the following Step VI is the last step of the above input. As per
SH

information carefully and answer the questions the rules followed in the above steps, find out in
given below. SBI (Clerk Mains) 2016 each of the following questions the appropriate
step for the given input.
A

A word and number arrangement machine when


Input 31 rise gem 15 92 47 aim big 25 does 56
@

given an input line of words and numbers


rearranges them following a particular rule in not 85 63 with moon
each step. The following is an illustration of input 1. How many steps will be required to
and rearrangement (All the numbers are two complete the rearrangement?
digit numbers). (a) Eight (b) Six (c) Seven (d) Five
(e) Nine
Input gate 20 86 just not 71 for 67 38 bake
2. Which word/number would be at seventh
sun 55
position from the left of step IV?
Step I bake gate 20 just not 71 for 67 38 sun (a) rise (b) aim (c) big (d) 15
55 86 (e) does
Step II for bake 20 just no 67 38 sun 55 86 71 3. Which step number is the following output?
Step III gate for bake 20 just not 38 sun 55 86 Rise not moon gem does big aim 15 with 92
71 67 85 63 56 47 31 25
Step IV just gate for bake 20 not 38 sun 86 71 (a) Step V (b) Step VII (c) Step IV (d) Step VIII
67 55 (e) Step III
Step V not just gate for bake 20 sun 86 71 67 4. Which of the following represents the
55 38 position of 92 in step VI
Step VI sun not just gate for bake 86 71 67 55 (a) Ninth from the left (b) Fifth from the right
38 20 (c) Sixth from the right (d) Ninth from the right
(e) Seventh from the left
328 How to Crack Test of Reasoning ANALYTICAL

5. Which word/number would be at fifth (d) family 32 beautiful 17 proud girl 48 55 97 rich 61
position from the right in the last step? 72 nice life
(a) Gem (b) 63 (c) 56 (d) 85 (e) girl 48 life 55 family 32 beautiful 17 proud 97
rich 61 72 nice
(e) Other than those given as options
Directions (Q. Nos. 6-10) Study the given 9. What is the position of ‘nice’ from the left
information and answer the following questions. end in the final step?
SBI (PO) 2013 (a) Fifth (b) Sixth (c) Seventh (d) Eighth
When a word and number arrangement machine (e) Ninth
is given an input line of words and numbers, it 10. Which element is third to the right of
arranges them following a particular rule. The
‘family’ in Step V?
following is an illustration of input and rearran-
(a) Beautiful (b) 17 (c) Proud (d) 97
gement. (All the numbers are two-digit numbers)
(e) 32
Input 40 made butter 23 37 cookies salt extra
52 86 92 fell now 19 Directions (Q. Nos. 11-17) Study the following
Step I butter 19 40 made 23 37 cookies salt information carefully and answer the questions
extra 52 86 92 fell now based on it. SBI (PO) 2014
Step II cookies 23 butter 19 40 made 37 salt A set of words and numbers is passed through an
extra 52 86 92 fell now arrangement machine and following

A
Step III extra 37 cookies 23 butter 19 40 made rearrangement is obtained.

M
salt 52 86 92 fell now Input talk 48 11 rude 97 84 35 walk jug home

H
Step IV fell 40 extra 37 cookies 23 butter 19 25 bag 77 along
made salt 52 86 92 now

TT
Step I 97 talk 48 11 rude 84 35 walk jug home
Step V made 52 fell 40 extra 37 cookies 23 25 77 along bag
butter 19 salt 86 92 now
Step VI now 86 made 52 fell 40 extra 37 cookies A
Step II 97 84 talk 48 11 rude 35 walk jug 25 77
W
along bag home
23 butter 19 salt 92
Step III 97 84 77 talk 48 11 rude 35 walk 25
SH

Step VII salt 92 now 86 made 52 fell 40 extra 37


along bag home jug
cookies 23 butter 19
Step IV 97 84 77 48 talk 11 35 walk 25 along
A

Step VII is the last step of the above arrangement


bag home jug rude
as the intended arrangement is obtained. As per
@

the rules followed in the given steps, find out the Step V 97 84 77 48 35 11 walk 25 along bag
appropriate steps for the given input. home jug rude talk
Input 32 proud girl beautiful 48 55 97 rich family Step VI 97 84 77 48 35 25 11 along bag home
61 72 17 nice life jug rude talk walk.
6. How many steps will be required to Step VI is the last step of the rearrangement.
complete the given input? Now, a set of words and letters is given below. As
(a) Five (b) Six (c) Seven (d) Eight per the rules followed in above steps, answer the
(e) Nine questions based on it.
Input 81 who sit 19 32 not but ink flow 51 27 van
7. Which of the following is the third element 68 92
from the left end of step VI?
(a) Beautiful (b) Life (c) 61 11. Which is the last step of this input?
(d) Nice (e) 17 (a) V (b) VI (c) VII (d) VIII
(e) None of these
8. Which of the following is step III of the given
input? 12. Which of the following is the step III?
(a) proud 72 girl 48 family 32 beautiful 17 55 97 rich (a) 92 81 68 who sit 19 32 not 51 27 van but flow ink
61 nice life (b) 92 81 68 51 who sit 32 19 not 27 van but flow ink
(b) life 55 girl 48 family 32 beautiful 17 proud 97 (c) 92 81 68 51 who sit 19 32 not 27 van but flow ink
rich 61 72 nice
(d) 92 81 68 51 32 who sit 19 not 27 van but flow ink
(c) girl 48 family 32 beautiful 17 proud 55 97 rich 61
72 nice life (e) None of the above
Chapter 29 INPUT-OUTPUT 329

13. Which step is ‘92 81 68 51 32 27 who 19 van but 18. How many elements are there between
flow ink not sit’? ‘with’ and ‘born’ in the step IV?
(a) Step IV (b) Step V (c) Step III (d) Step VI (a) None (b) One
(e) There is number such step (c) Two (d) Three
14. Which is third element to the right of the sixth (e) Five
element from the right end in step II? 19. In which of the following steps is ‘15 some
(a) 32 (b) van (c) 27 52’ found consecutively in the same order?
(d) flow (e) who (a) Step V (b) Step VI
15. How many elements are there between “who” (c) Step II (d) Step I
and “van” in step IV? (e) Step III
(a) 3 (b) 5 (c) 4 20. Which of the following is the second
(d) 0 (e) 1 element to the left of the ninth element
16. In step V, if ‘92’ is related to ‘sit’ and ‘81’ is from the left in the second step?
related to ‘not’, similarly, 68 is related to which (a) 39 (b) like
element? (c) with (d) role
(a) ink (b) flow (c) but (e) air
(d) sit (e) None of these
21. In step I, ‘some’ is related to ‘77’ following

A
17. In step II, if ‘not’ is related ‘92’, ‘51’ is related to a certain pattern. Following the same

M
‘68’, ‘van’ is related to ‘sit’, in the same way pattern, in step V ‘role’ is related to ‘96’. In
‘flow’ is related to which element? step IV, to which of the following is ‘born’

H
(a) 81 (b) who (c) 19 related to

TT
(d) 32 (e) None of these (a) with (b) 63 (c) like (d) 39
Directions (Q. Nos. 18-22) Study the following (e) role
information carefully and answer the questions. A
22. In step II, which element appears exactly
W
When a word and number arrangement machine is between ‘77’ and ‘15’?
SH

given an input line of words and numbers, it (a) Only ‘role’ (b) Both ‘born’ and 52
arranges them following a particular rule. The (c) Only ‘air’ (d) Only ‘some’
A

following is an illustration of input and (e) Both ‘like’ and ‘39’


rearrangement. (All the numbers are two - digit
@

Directions (Q. Nos. 23-25) Study the given


numbers). Indian Bank (PO Pre) 2016
information carefully and answer the questions
Input turn 12 84 mist hike 45 vast 26 site gate given below. SBI (PO Mains) 2017
72 56 An input-output is given in different steps.
Step I gate turn 12 mist hike 47 vast 26 site 72 Some mathematical operations are done in each
56 84 step. No mathematical operation is repeated in
Step II 72 gate turn 12 mist 45 vast 26 site 56 84 next step.
hike
5 3 1 2 2 4 2 1 4 3 2 4
Step III mist 72 gate turn 12 45 vast 25 site 84
hike 56
Step IV 45 mist 72 gate turn 12 vast 26 84 hike
56 site Step I 5 6 3 8 8 8
Step V turn 45 mist 72 gate 12 vast 84 hike 56
Step II 1 6 2 2
site 26
Step VI 12 turn 45 mist 72 gate 84 hike 56 site 26 Step III 3.5 2
vast
Step IV 1.5
Step VI is the last step of the above arrangement
as the intended arrangement is obtained. As per the rules followed in the steps given
As per the rules followed in the given steps, find the above, find out in each of the following
appropriate steps for the given input. questions the appropriate step for the given
input.
Input 15 role air 96 63 born with 77 like 39 some
52. 4 2 5 1 2 9 3 2 7 1 1 4
330 How to Crack Test of Reasoning ANALYTICAL

23. Find the addition of two numbers obtained in As per the rules followed in the steps given
step III. above, find out in each of the following
(a) 1.5 (b) 3 (c) 7 (d) 3.5 questions the appropriate steps for the given
(e) Other than those given as options input.
24. Find the difference between sum of numbers Input 6 3 5 6 6 3 4 1 2 6 3 5
obtained in 1st step and sum of numbers 26. Find the addition of the three numbers
obtained in all other steps. obtained in step I.
(a) 232 (b) 185 (c) 188 (d) 183.5 (a) 57 (b) 56
(e) Other than those given as options (c) 55 (d) 58
25. Find the multiplication of the numbers (e) None of these
obtained in step II. 27. Find the difference of two numbers
(a) 426 (b) 462 (c) 188 (d) 98 obtained in step II.
(e) Other than those given as options (a) 41 (b) 61
(c) 91 (d) 56
Directions (Q. Nos. 26-30) Study the given
(e) None of these
information carefully and answer the given questions.
An input-output is given in different steps. Some 28. Find the multiplication of the two
mathematical operations are done in each step. No numbers obtained in step III.

A
mathematical operation is repeated in next step but (a) 2007 (b) 1747

M
it can be repeated with some other mathematical (c) 3087 (d) 1841

H
operation (as multiplication can be used with (e) None of these

TT
subtraction in step 1 and same can be used with 29. Find the multiplication of unit and tens
addition in step 2) digit of number which is obtained in step
Input 6 8 3 5 4 9 1 3 1 2 2 7 IV.
A
W
(a) 72 (b) 60
(c) 50 (d) 36
SH

Step I 2 2 2 3 1 2 (e) 40
30. Which of the following number is obtained
A

Step II 1 7 4 3 in step IV ?
@

(a) 34 (b) 98
Step III 1 6 6 3 (c) 48 (d) 66
2 5 (e) None of these
Step IV

Answers
Base Level Exercise
1. (d) 2. (b) 3. (d) 4. (e) 5. (c) 6. (c) 7. (e) 8. (d) 9. (d) 10. (c)
11. (b) 12. (c) 13. (d) 14. (b) 15. (d) 16. (b) 17. (e) 18. (c) 19. (a) 20. (d)
21. (b) 22. (c) 23. (c) 24. (d) 25. (c) 26. (d) 27. (b) 28. (a) 29. (b) 30. (b)
31. (b) 32. (e) 33. (c) 34. (a) 35. (d) 36. (c)

Expert Level Exercise


1. (a) 2. (d) 3. (b) 4. (c) 5. (c) 6. (c) 7. (d) 8. (c) 9. (a) 10. (b)
11. (c) 12. (c) 13. (b) 14. (b) 15. (a) 16. (a) 17. (d) 18. (d) 19. (b) 20. (b)
21. (d) 22. (c) 23. (d) 24. (b) 25. (b) 26. (a) 27. (d) 28. (c) 29. (e) 30. (e)
Chapter 29 INPUT-OUTPUT 331

Answer with Explanations


Base Level Exercise 9. (d) Step II 24 year 56 43 last part 64 over
1. (d) Batch II do lean window out of not Step III 24 year 43 56 last part 64 over
Batch III do lean not window out of Step IV 24 year 43 part 56 last 64 over
Batch IV do lean not of window out Step V 24 year 43 part 56 over last 64
Step VI 24 year 43 part 56 over 64 last
2. (b) Input please do not delay the matter Hence, four more steps are required to
Batch I delay please do not the matter complete the rearrangement.
Batch II delay do please not the matter 10. (c) Step III 32 station 46 81 73 March go for
Batch III delay do matter please not the Step IV 32 station 46 March 81 73 go for
3. (d) Input cannot be determined Step V 32 station 46 March 73 81 go for
4. (e) Batch II children for not is good watching Step VI 32 station 46 March 73 go 81 for
television 11. (b) Step III year 92 ultra 15 23 strive house 39
Batch III children for good not is watching Step IV year 92 ultra 39 15 23 strive house
television Step V year 92 ultra 39 strive 15 23 house

A
Batch IV children for good is not watching Step VI year 92 ultra 39 strive 23 15 house
television

M
Step VII year 92 ultra 39 strive 23 house 15
Batch V children for good is not television So, four more steps are required.

H
watching. 12. (c) Input any how 49 24 for wide 34 69

TT
5. (c) Input necessary arrangements have already Step I wide any how 49 24 for 34 69
been made Step II wide 69 any how 49 24 for 34
Batch I already necessary arrangements have A
Step III wide 69 how any 49 24 for 34
W
been made Step IV wide 69 how 49 any 24 for 34
Batch II already arrangements necessary have Step V wide 69 how 49 for any 24 34
SH

been made Step VI wide 69 how 49 for 34 any 24


Batch III already arrangements been necessary So, step V will be the last step but one.
A

have made
13. (d) Input cannot be determined
Batch IV already arrangements been have
@

necessary made 14. (b) Input 86 open shut door 31 49 always 45


Batch V already arrangements been have Step I 31 86 open shut door 49 always 45
made necessary Step II 31 shut 86 open door 49 always 45
So, the last batch is V. Step III 31 shut 45 86 open door 49 always
6. (c) Input kind 12 96 heart water 59 42 yes Step IV 31 shut 45 open 86 door 49 always
Step I 12 kind 96 heart water 59 42 yes Step V 31 shut 45 open 49 86 door always
Step II 12 yes kind 96 heart water 59 42 Step VI 31 shut 45 open 49 door 86 always
Step III 12 yes 42 kind 96 heart water 59 Hence, six steps are required to complete the
Step IV 12 yes 42 water kind 96 heart 59 rearrangement.
Step V 12 yes 42 water 59 kind 96 heart 15. (d) Input cannot be determined of the given
7. (e) Input jungle 43 mode 25 basket 39 target 19 step III.
Step I 19 jungle 43 mode 25 basket 39 target
16. (b) Step II 18 win 71 34 now if victory 61
Step II 19 target jungle 43 mode 25 basket 39
Step III 18 win 34 71 now if victory 61
Step III 19 target 25 jungle 43 mode basket 39
Step IV 18 win 34 victory 71 now if 61
Step IV 19 target 25 mode jungle 43 basket 39
Step V 18 win 34 victory 61 71 now if
Step V 19 target 25 mode 39 jungle 43 basket Step VI 18 win 34 victory 61 now 71 if
Hence, Step IV is the last but one. Four more steps are required to complete the
8. (d) Input cannot be determined rearrangement.
332 How to Crack Test of Reasoning ANALYTICAL

17. (e) Input where 47 59 12 are they going 39 Step IV 19 37 49 ace hot sun best ice cut 67 77 83
Step I 12 where 47 59 are they going 39 Step V 19 37 49 ace best hot sun ice cut 67 77 83
Step II 12 where 39 47 59 are they going Step VI 19 37 49 ace best cut hot sun ice 67 77 83
Step III 12 where 39 they 47 59 are going Step VII 19 37 49 ace best cut hot ice sun 67 77 83
Step IV 12 where 39 they 47 going 59 are
24. (d) VII steps would be needed to complete the
Hence, step III is the last but one.
rearrangement.
18. (c) Step II 33 store 81 75 full of goods 52 25. (c) V step number would be the given output.
Step III 33 store 52 81 75 full of goods 26. (d) ‘19 67 hot sun best ice 49 ace 77 cut 37 83’
Step IV 33 store 52 of 81 75 full goods would be the step I.
Step V 33 store 52 of 75 81 full goods 27. (b) ‘19 37 49 ace best cut hot ice sun 67 77 83’
would be the final arrangement.
Step VI 33 store 52 of 75 goods 81 full
28. (a)‘sun’ would be at the 7th position from the right.
Solutions (Q. Nos. 19-23) The word and number
arrangement stepwise is as follow Solutions (Q. Nos. 29-33)
Input for 52 all 96 25 jam road 15 hut 73 bus stop Step I (Each number of input) – 5
38 46 Step II (Each number of input) + 9
Step I all for 52 25 jam road 15 hut 73 bus stop 38 Step III (Each number of input – 2) 2
46 96 Step IV (Each number of input) 2

A
Step II bus all for 52 25 jam road 15 hut stop 38 46 Step V (Each number of input −1) 2

M
96 73 Step VI (Each number of input ×2) 2

H
Step III for bus all 25 jam road 15 hut stop 38 46 96
73 52
29. (b) Step IV = (Each number of input) 2

TT
Step IV hut for bus all 25 jam road 15 stop 38 96 73 30. (b) Step VI = (Each number of input × 2 )2
52 46
Step V jam hut for bus all 25 road 15 stop 96 73 52 A
⇒ Each number of input =
Step VI
W
2
46 38
31. (b) Step V = (Each number of input –1) 2
SH

Step VI road jam hut for bus all 15 stop 96 73 52 46


38 25 32. (e) Step II = (Each number of input) + 9
⇒ Each number of input = (Step II) – 9
A

Step VII stop road jam hut for bus all 96 73 52 46 38


25 15 Hence, it is clear that out of the given options
@

19. (a) ‘25’ would be at the 6th position from the left in (a), (b), (c) and (d), no one can be correct.
step V. 33. (c) Step VI = (Each number of input × 2 )2
20. (d) ‘for bus all 25 jam road 15 hut stop 38 46 96 73
Solutions (Q. Nos. 34-36)
52’ would be the step III.
Step I First letters disappear
21. (b) ‘road’ would be at 8th position from the right in
step IV. Step II Last letters disappear
Step III Vowels disapper
22. (c) ‘stop road jam hut for bus all 96 73 52 46 38 25
15’ would be step VII. Step IV Consonants disappear
23. (c) Step II would be the given output. Step V First two letters disappear

Solutions (Q. Nos. 24-28) 34. (a) In step V, first two letters in each word of input
get disappeared.
Input 67 hot sun 19 best 83 ice 49 ace 77 cut 37
Step I 19 67 hot sun best ice 49 ace 77 cut 37 83 35. (d) It is very obvious that we cannot find the input.
Step II 19 37 67 hot sun best ice 49 ace cut 77 83 36. (c) In step II, last letter in each word of input gets
Step III 19 37 49 hot sun best ice ace cut 67 77 83 disappeared.
Chapter 29 INPUT-OUTPUT 333

Expert Level Exercise


Solutions. (Q. Nos. 1-5) According to the given 7. (d) ‘nice’ is the third element from the left end of
information, we have the following arrangement step VI.
Input 31 rise gem 15 92 47 aim big 25 does 56 not 8. (c) ‘girl 48 family 32 beautiful 17 proud 55 97 rich 61
85 63 with moon 72 nice life’ is step III of the given input.
Step I aim 31 rise gem 15 47 big 25 does 56 not 85
9. (a) nice is fifth from the left end in the final step.
63 with moon 92
Step II big aim 31 rise gem 15 47 25 does 56 not 63 10. (b) ‘17’ is third to the right of ‘family’ in step V.
with moon 92 85 Solutions (Q. Nos. 11-17)
Step III does big aim 31 rise gem 15 47 25 56 not with Input 81 who sit 19 32 not but ink flow 51 27 van 68
moon 92 85 63 92
Step IV gem does big aim 31 rise 15 47 25 not with Step I 92 81 who sit 19 32 not ink flow 51 27 van 68
moon 92 85 63 56 but
Step V moon gem does big aim 31 rise 15 25 not with Step II 92 81 68 who sit 19 32 not ink 51 27 van but
92 85 63 56 47. flow
Step VI not moon gem does big aim rise 15 25 with 92 Step III 92 81 68 51 who sit 19 32 not 27 van but
85 63 56 47 31 flow ink

A
Step VII rise not moon gem does big aim 15 with 92 85 Step IV 92 81 68 51 32 who sit 19 27 van but flow
63 56 47 31 25

M
ink not
Step VIII with rise not moon gem does big aim 92 85 63 Step V 92 81 68 51 32 27 who 19 van but flow ink

H
56 47 31 25 15 not sit

TT
1. (a) Eight steps are required to complete the Step VI 92 81 68 51 32 27 19 who but flow ink not
rearrangement. sit van
2. (d) ‘15’ is at seventh position from the left end in
step IV. A
Step VII 92 81 68 51 32 27 19 but flow ink not sit
W
van who
3. (b) The given output is the step VII. 11. (c) Step VII is the last step of the given input.
SH

4. (c) ‘92’ is at sixth position from the right end in 12. (c) ‘92 81 68 51 who sit 19 32 not 27 van but flow
step VI. ink’ is the step III.
A

5. (c) ‘56’ is at fifth position from the right in the last 13. (b) Step V is ‘92 81 68 51 32 27 who 19 van but flow
@

step. ink not sit.’


Solutions (Q. Nos. 6-10) 14. (b) ‘Van’ is third element to the right of the sixth
Input 32 proud girl beautiful 48 55 97 rich family 61 element from the right end in step II.
72 17 nice life 15. (a) There are three elements between ‘who’ and
Step I beautiful 17 32 proud girl 48 55 97 rich family ‘van’ in step IV.
61 72 nice life 16. (a) ‘68’ is related to ‘ink’.
Step II family 32 beautiful 17 proud girl 48 55 97 rich 17. (d)‘flow’ is related to ‘32’.
61 72 nice life
Solutions (Q.Nos. 18-22) According to the given
Step III girl 48 family 32 beautiful 17 proud 55 97 rich information
61 72 nice life Input 15 role air 96 63 born with 77 like 39 some 52.
Step IV life 55 girl 48 family 32 beautiful 17 proud 97 Step I air 15 role air 63 born with 77 like 39 some
rich 61 72 nice 52 96
Step V nice 61 life 55 girl 48 family 32 beautiful 17 Step II 77 air 15 role 63 with like 39 some 52 96 born
proud 97 rich 72 Step III like 77 air 15 role with 39 some 52 96 born 63
Step VI proud 72 nice 61 life 55 girl 48 family 32 Step IV 52 like 77 air 15 with 39 some 96 born 63 role
beautiful 17 97 rich
Step V some 52 like 77 air 15 with 96 born 63 role 39
Step VII rich 97 proud 72 nice 61 life 55 girl 48 family 32
Step VI 15 some 52 like 77 air 96 born 63 role 39 with
beautiful 17
18. (d) Three elements viz ‘39’ ‘some’ and ‘96’ are there
6. (c) Seven steps will be required to complete the between ‘with’ and ‘born’ in step IV.
given input
334 How to Crack Test of Reasoning ANALYTICAL

19. (b) In step VI, ‘15 some 52’ is found consecutively in and sum of numbers of all other
the same order. steps = 21 + 22 + 15
. + 2 + 0.5 = 47
20. (b) ‘like’ is the second element to the left of the ninth ∴Required difference = 232 − 47 = 185
element from the left in step II. 25. (b) Here numbers, obtained in Step II
21. (d) ‘born’ is related to ‘39’ following the same = 21 and 22
pattern as given in question. ∴ Required multiplication = 21 × 22 = 462
22. (c) ‘air’ appears exactly between ‘77’ and ‘15’ in Solutions (Q.Nos. 26-30) From the given information
step II arrangement is as follows,
Solutions (Q.Nos. 23-25) From given information
Input 6 3 5 6 6 3 4 1 2 6 3 5
arrangement is as follows,
4 2 5 1 2 9 3 2 7 1 1 4 (6×5–6×3) (6×4–3×1) (5×6–2×3)

Step I 1 2 2 1 2 4
Step I 4×2 2×3 5×1 7×1 2×4 9×1
(13+23+23) (23+13+43)
8 6 5 7 8 9
Step II 1 7 7 3
Step II 8+5+8=21 6+7+9=22
(1×72) (7×32)
2 1 2 2

A
Step III 4 9 6 3

M
2+1 2+2
Step III = 1.5 =2 (94–36)
2 2

H
1.5 2 5 8

TT
Step IV
Step IV 2 – 1.5 = 0.5
26. (a) Required answer = 12 + 21 + 24 = 57
0.5 A
27. (d) Required difference = 73 − 17 = 56
W
23. (d) Addition of two numbers obtained in Step III
28. (c) Required answer = 3087
SH

= 15
. + 2 = 3.5
29. (e) Required answer = 8 × 5 = 40
24. (b) Here, sum of numbers of Step I
= 86 + 57 + 89 = 232
A

30. (e) ‘58’ is obtained in Step IV.


@
Chapter 01 ANALOGY 335
NON-VERBAL REASONING

01
Analogy
Analogy refers to correspondence or similarity the second part. You have to select that figure
in relationship. When a figure exhibits some from the set of answer figures which would
kind of relationship with another figure on replace the question mark “?”.
some basis then the two figures are said to be Illustration 1.

A
analogous to each other.
Problem Figures

M
The basis for establishing such relationship

H
between the different figures are given below
:: ?

TT
l
Based on shape and size
l
Based on orientation of figure
l
Based on number of elements Answer Figures A
W
l
Based on inversion (vertically or
SH

horizontally)
l
Rotation of figure
(a) (b) (c) (d) (e)
A

l
Movement of figure
Solution (d) Considering the first two figures, both the
@

l
Replacement of positions of elements
figures interchange their positions and get reversed
In the questions based on analogy, two sets of at the new position. The arrow head is shifted to
figures namely problem figures and answer the arm of U-shaped figure and becomes reversed.
figures are given.
Illustration 2.
The first part comprises of two figures, which
Problem Figures
have some relationship between them on the
basis of a certain rule.
?
The second part comprises of one figure and a
sign of ‘?’. The candidate is required to find
MPPSC 2018
out the the figure from the given alternatives,
Answer Figures
which have the same relationship to the third
figure, as there is between the first two.
Directions (Illustrations 1-5) The second
figure of the first part of the problem figures bears (a) (b) (c) (d)
a certain relationship to the first figure.
Similarly, one of the figures in answer figures Solution (b) The shaded figure moves inside the
bears the same relationship to the first figure of pentagon and the lower half of it becomes unshaded.
336 How to Crack Test of Reasoning NON-VERBAL

Illustration 3. Answer Figures


Problem Figures

:: ? (a) (b) (c) (d)

Solution (d) The number of sides in the main figure


Answer Figures increases by one and the number of line
segments attached to the main figure decreases by
one.

Illustration 5.
(a) (b) (c) (d) (e) Problem Figures
Solution (b) The lower element gets enlarged and
becomes the outer most element. The upper
element also gets enlarged and becomes the middle
:: ?
element while a new element with one more side
than the lower element appears at the innermost Answer Figures

A
position.

M
Illustration 4.

H
TT
Problem Figures (a) (b) (c) (d) (e)

:: ? A
Solution (d) The line attached to the small black
W
dot rotates 90° anti-clockwise. The remaining portion
of the figure rotates 90° clockwise and is inverted.
SH
A

Let us Practice
@

Base Level Exercise


Directions (Q. Nos. 1-27) The second figure in the first part of the problem figures bears certain
relationship to the first figure. Similarly, one of the figures of answer figures bears the same relationship
to the first figure of the second part. You have to select a figure from the set of answer figures which
would replace the question mark (?).

1. Problem Figures 2. Problem Figures

:: ? :: ?

Answer Figures Answer Figures

(a) (b) (c) (d) (a) (b) (c) (d)


Chapter 01 ANALOGY 337

3. Problem Figures 7. Problem Figures

:: ? :: ?

UP Police (Constable) 2018 Answer Figures


Answer Figures

(a) (b) (c) (d)


(a) (b) (c) (d)
8. Problem Figures
4. Problem Figures
:: ?
:: ?
Answer Figures

A
M
SSC (CPO) 2010

H
Answer Figures

TT
(a) (b) (c) (d)

A
9. Problem Figures
W
(a) (b) (c) (d)
SH

5. Problem Figures
:: ?
A
@

SSC (Multitasking) 2012


:: ?
Answer Figures

Answer Figures

(a) (b) (c) (d)


(a) (b) (c) (d) (e)

6. Problem Figures 10. Problem Figures

:: ? :: ?

Answer Figures
Answer Figures

(a) (b) (c) (d) (e)


(a) (b) (c) (d)
338 How to Crack Test of Reasoning NON-VERBAL

11. Problem Figures 15. Problem Figures

:: ? :: ?

Answer Figures DSSSB (PRT) 2017

Answer Figures

(a) (b) (c) (d) (e)

12. Problem Figures (a) (b) (c) (d)

16. Problem Figures


:: ?
:: ?
Answer Figures

A
M
Answer Figures

H
TT
(a) (b) (c) (d) (e)

13. Problem Figures (a) A


(b) (c) (d) (e)
W
17. Problem Figures
SH

:: ?
:: ?
A
@

Answer Figures
Answer Figures

(a) (b) (c) (d) (e)


(a) (b) (c) (d) (e)
14. Problem Figures
18. Problem Figures
: :: : ?
:: ?
NICL (AO) 2015; SSC (CPO) 2011
Answer Figures Answer Figures

(a) (b) (c) (d) (e)


(a) (b) (c) (d)
Chapter 01 ANALOGY 339

19. Problem Figures Answer Figures

:: ?

MPPSC 2018; SSC (CPO) 2014 (a) (b) (c) (d) (e)
Answer Figures 24. Problem Figures

:: ?

(a) (b) (c) (d) DSSSB (PRT) 2017


Answer Figures
20. Problem Figures

:: ?
(a) (b) (c) (d)

A
Answer Figures
25. Problem Figures

M
H
H
: H :: : ?

TT
H
(a) (b) (c) (d) (e)
SSC (10+2) 2013
21. Problem Figures Answer Figures A
W
SH

:: ? H H H H
A

SSC (Steno) 2012 (a) (b) (c) (d)


Answer Figures
@

26. Problem Figures

:: M
?
(a) (b) (c) (d)
Answer Figures
22. Problem Figures
M

M
:: ? M

(a) (b) (c) (d)


Answer Figures
27. Problem Figures

:: ?
(a) (b) (c) (d) (e)
Answer Figures
23. Problem Figures

:: ?
(a) (b) (c) (d) (e)
340 How to Crack Test of Reasoning NON-VERBAL

Expert Level Exercise


Directions (Q. Nos. 1-22) The second figure in the first part of the problem figures bears a certain
relationship to the first figure. Similarly, one of the figure of answer figure bears the same relationship
to the first figure of the second part. You have to select a figure from the set of any figures which would
replace the questions mark (?).

1. Problem Figures 5. Problem Figures


+
:: ? :: ?
S S C
Answer Figures Answer Figures
C C C + +
C + +
C + – +
(a) (b) (c) (d) (a) (b) (c) (d) (e)

A
2. Problem Figures 6. Problem Figures

M
:: ?

H
:: ?

TT
Answer Figures
Answer Figures
A
W
SH

(a) (b) (c) (d)


(a) (b) (c) (d) (e)
A

3. Problem Figures
@

7. Problem Figures
:: ?
:: ?
SSC (CGL) 2011
Answer Figures
Answer Figures

(a) (b) (c) (d) (a) (b) (c) (d) (e)


4. Problem Figures
8. Problem Figures
?
+ X :: X ?
SSC (10+2) 2013
Answer Figures Answer Figures

+ + + × ×
(a) (b) (c) (d) (a) (b) (c) (d) (e)
Chapter 01 ANALOGY 341

9. Problem Figures Answer Figures

T
P
P S C S S S
P :: ?
S

S
T S + + + # + C + C

T
(a) (b) (c) (d) (e)
Answer Figures
T T
P
T T T 14. Problem Figures
P P
P
+
P
+

S
S
S S S S :: ?

C
(a) (b) (c) (d) (e)

C
C S

S
10. Problem Figures Answer Figures
S S S S S
C C C C C
:: ?
(a) (b) (c) (d) (e)
Answer Figures
15. Problem Figures

A
M
:: ?

H
(a) (b) (c) (d) (e)

TT
11. Problem Figures Answer Figures
A
W
:: ?
SH

(a) (b) (c) (d) (e)


Answer Figures
A

16. Problem Figures


@

:: ?
(a) (b) (c) (d) (e)

12. Problem Figures Answer Figures


T
T
T

:: ?
S G G
S
S

(a) (b) (c) (d) (e)


Answer Figures
17. Problem Figures
T

S S S S :: ?
(a) (b) (c) (d) (e)
1 2 3 4
13. Problem Figures Answer Figures
C +
:: ?
? S
(a) (b) (c) (d) (e)
342 How to Crack Test of Reasoning NON-VERBAL

18. Problem Figures Answer Figures


C S S
:: ? S S S C
C
1 2 3 4 (a) (b) (c) (d) (e)
Answer Figures
20. Problem Figures
T S
S :: ? C T
(a) (b) (c) (d) (e) S C C
1 2 3 4
19. Problem Figures BPS (PO) 2011
Answer Figures
O O ? C C C C S
S T T
T T S T
(a) (b) (c) (d) (e)

A
Answers

M
H
Base Level Exercise

TT
1. (c) 2. (d) 3. (a) 4. (a) 5. (b) 6. (b) 7. (a) 8. (c) 9. (a) 10. (b)
11. (d) 12. (a) 13. (b) 14. (d) 15. (b) 16. (e) 17. (e)
A 18. (e) 19. (d) 20. (d)
W
21. (d) 22. (d) 23. (b) 24. (d) 25. (d) 26. (c) 27. (b)
SH

Expert Level Exercise


A

1. (a) 2. (a) 3. (a) 4. (d) 5. (a) 6. (b) 7. (d) 8. (b) 9. (a) 10. (e)
@

11. (b) 12. (a) 13. (c) 14. (a) 15. (b) 16. (c) 17. (c) 18. (d) 19. (c) 20. (b)

Answer with Explanations


Base Level Exercise
1. (c) First design of first pair of problem figures is 5. (b) The outer figure rotates 135° clockwise and the
inverted and then placed on the original inner figure rotates 45° anit-clockwise and get
design to get the second figure. Similar inverted at its new position.
concept is used in second pair.
6. (b) The symbol ‘o’ is moving from one corner to
2. (d) Second figure of each pair is mirror image of another in clockwise direction and one line is
first figure. removed to get the second figure of the pair.
3. (a) The outer figure comes inside and becomes 7. (a) In first pair of problem figures, one more arm/side
shaded. The inner figure comes outside. is added to the first figure and the half-circle at the
4. (a) From first figure to second figure, the number corner of the sides, which were at the outside,
of figures attached to the vertical line comes inside. Similarly, in second pair of problem
increase by 1 and the outer line segment also figures, one more side is added to the first figure
increase by 1. and the half-circle at the corner of the sides, which
were at the inside, comes outside.
Chapter 01 ANALOGY 343

8. (c) First design of first pair of problem figures is 18. (e) The complete circle is converted into a hexagon
rotated by 90° and then it is placed on the and semi-circle is converted into half hexagon.
original to get the second figure of the pair. Similar
19. (d) Here, the figure at the bottom rotates 90° and
concept is used in second pair to get the right
becomes the outermost figure. The outer figure
answer.
at the top becomes the innermost figure, and
9. (a) In the second figure, a figure is generated similar the inner figure at the top gets inverted and
to the figure inside the circle. becomes the middle figure.
10. (b) Half of the petal is added to the right side and 20. (d) The larger figure becomes small, gets inverted
figure is rotated 45° clockwise.
and occupies the top position inside the small
11. (d) The figure is rotated 45° clockwise in such a way figure which gets enlarged.
that one curved line is deleted and one arrow is
21. (d) From the first figure to second figure, the
added. The arrows get inverted to form their new
designs are inverted.
position.
22. (d) Both the upper and lower pair of figures
12. (a) Both the figures interchange positions. Single
interchange positions.
figure is converted into three figures and the
vice-versa. 23. (b) The given figure rotates 90° clockwise. The top
13. (b) The figure rotates 180° and three lines are and bottom elements interchange their
added to the lower portion of the figure. positions. Also, the bottom element reverses its

A
side.
14. (d) Second figure of each pair is mirror image of first

M
figure. 24. (d) The inner part of the figure is removed and a

H
15. (b) Here, the size of both the figures reduces and horizontal line is added in the middle to get the
second figure.

TT
the half part of inner figure moves to the top of
the outer figure. 25. (d) Inner elements of the figure becomes outer
16. (e) The figure rotate 90° clockwise and heads of the
arrows are reversed. A
elements and vice-versa.
W
26. (c) Second figure is the mirror image of the first
17. (e) The main figure rotates 180° and both the leaves figure, but the middle line remains same.
SH

at the extreme ends get deviated by 45° and


27. (b) Straight line in the figure intersect and curved
three curved lines are added to the figure,
positions of the figure are deleted.
A

pointing in anti-clockwise direction.


@

Expert Level Exercise


1. (a) The mirror image of the symbol in the middle 5. (a) The movement of elements are as follow
position shifts to the top left position, symbol at
the top right position shifts to the bottom right
position and is shaded and vice-versa. Symbol
at bottom left position rotates 90° anti-clockwise
and occupies the central position. + ÷ C

2. (a) The central element becomes the right element Similarly, ÷ ÷ +


after rotating 90°. The top element rotates 180° × C + ×
and becomes the central element covered by the
enlarged bottom element which rotates by 90°. 6. (b) Number of lines equal to the number of sides
are added inside the main figure. The outer
3. (a) From problem figure I to II, the inner most figure designs interchange their positions and half of
becomes outer most figure and outer most these are shaded.
figure becomes the inner most figure.
7. (d) Both the figures are superimposed on each other
4. (d) Circle and arrow at the corner gets interchanged and the figure thus obtained is rotated 90°
and circle in the middle of the figure gets clockwise.
converted to square. Same pattern is followed to As,
obtain missing figure.
⇒ ⇒
344 How to Crack Test of Reasoning NON-VERBAL

Similarly, 14. (a) All the signs change positions as shown in the
diagram and the signs rotate in a specific order.
⇒ ⇒

8. (b) The figure is rotated 90° anti-clockwise, curved


portion is pressed and the figure is pressed
from one side. The symbol inside the figure
rotated 45° and comes out on the smaller side, 15. (b) The second figure is the water image of first
the triangle moves 90° clockwise and is figure. Also black dots become white and a line
shaded and a new sign is added opposite side segment from the head of the figure is removed.
of triangle. 16. (c) The figure at the top right position rotates 135°
9. (a) The symbol at the top left position rotates 180° clockwise, the symbol at the central position
and comes at the centre, the symbol at top rotates 90° anti-clockwise and moves to the
right position rotates 90° clockwise and comes bottom position and the figure at bottom left
at the bottom right position, the symbol at the position is inverted and moves to the central
bottom right position rotates 90° clockwise and position.
comes to the top left position, the symbol at 17. (c) From figure (1) to (2), the figure at extreme right
the bottom left position rotates 90°

A
gets inverted and enlarged. The left one moves to
anti-clockwise and shifts at top right corner. the middle and the middle figure gets inverted

M
The symbol at central position rotates 90° and reduced in size and becomes the innermost

H
clockwise, gets reversed and shifts at the element.

TT
bottom left position. 18. (d) The figure at the bottom left rotates 180° in
10. (e) The main figure rotates 45° clockwise and the clockwise direction and moves one and half step
dots outside the figure move to the other side A
in anti-clockwise direction. The figure at the top
W
of the line. right moves one step in anti-clockwise direction
and becomes darkened and two new small
SH

11. (b) The design of the first figure rotates 180° in the
figures are added.
second figure. So to obtain same design in the
fourth figure, the third figure will be rotated 19. (c) In the first figure, there are four designs but in
A

180°. second design, they become five in number and


@

all the figures inside it interchange their place.


New

12. (a) Sign at the top left position is being inverted,


sign at top right position rotates 90° clockwise,
sign at the bottom left position rotates Similarly, the same change is done from third
90° clockwise, sign at the central position figure to fourth figure.
rotates 90° anti-clockwise and the sign at C
bottom right position remains unchanged.
S
13. (c) All the symbols interchange positions as S
shown in the diagram. Also, each of the
symbols change their direction at the new 20. (b) From figure (2) to (1), the changes in position of
position in a specific order and the symbol in elements is given as follows
place of * is replaced by new symbol. N

(N) = New Symbol


Chapter 02 CLASSIFICATION 345

02
Classification
Classification is the process of putting Solution (d) In all the figures except figure (d), there are
things or objects into a group and then two pins and three arrows. In figure (d), there are three
finding the different object or thing that pins and two arrows, thus it is different from the others.
does not belong to the group. Illustration 2.

A
In this chapter, we deal with questions

M
which have a set of four/five figures, out of

H
which all, except one, are alike or have

TT
some common nature/characteristics. You
(a) (b) (c) (d) (e)
have to select that exclusively different
figure from the given set. A
Solution (e) All the figures, except figure (e) are made of
W
straight lines. But figure (e) is not, thus, it is different.
Criteria for Selecting
SH

Illustration 3.
Similar-Dissimilar Figures
A

There are several criteria used for


@

establishing similar characteristics between


the figures, like rotation of same figure,
division of figures, number of elements or (a) (b) (c) (d)
line, interior-exterior consideration of UP Police (Constable) 2018
elements etc. Solution (a) All the figures except figure (a) contain the
same number of pins ( —• ) as the number of sides of
Examples given below, will give you a better
the main figure (outer figure), e.g. The third figure is a
idea about the type of questions asked in
pentagon and thus, have five pins inside in it.
various competitive examinations. Similarly, the second and fourth figure follow the same
Directions (Illustrations 1-8) Out of these pattern.
five/four given figures, four/three are similar Illustration 4.
in a certain way and so, form a group. Find
out the figure which does not belong to the
group.
Illustration 1. (a) (b) (c) (d)

Solution (c) In all the figures except figure (c), the


rotation of the arrows is in clockwise direction. But in
figure (c), four arrows are moving in anti-clockwise
(a) (b) (c) (d) (e) direction.
346 How to Crack Test of Reasoning NON-VERBAL

Illustration 5. Illustration 7.

(a) (b) (c) (d) (e) (a) (b) (c) (d)


SSC (10+2) 2013
Solution (d) Except figure (d), all other figures have four
black and three white squares. Solution (d) All the figures except figure (d), are
rotated forms of the same figure. i.e. all are same
Illustration 6. when rotated. But in figure (d), the shaded portion
is at opposite side.
Illustration 8.

(a) (b) (c) (d) (e)

Solution (d) In each figure except figure (d), the


(a) (b) (c) (d) (e)
upper right element is identical to the lower part of
Solution (e) In all the figures except figure (e), the

A
the left element and the lower right element is
identical to the upper part of the left element. areas of shaded portions are different.

M
H
TT
Let us Practice A
W
SH

Base Level Exercise


A

Directions (Q. Nos. 1-37) In each of the following problem, out of four/five figures marked (a), (b),
@

(c) and (d), three/four are similar in a certain manner, but one figure is different from others. Choose
the figure which is different from others.
1. 4.

(a) (b) (c) (d)


(a) (b) (c) (d)
5.
UP Police (Constable) 2013
H
2. (a) (b) (c) (d)
6.
(a) (b) (c) (d)
(a) (b) (c) (d)
3. UP Police (Constable) 2018

7.

(a) (b) (c) (d)


(a) (b) (c) (d)
Chapter 02 CLASSIFICATION 347

8. 18.

(a) (b) (c) (d) (a) (b) (c) (d)


9. ∨ ∨ ∧ ∧ ∨ ∨ > >
SSC (10+2) 2012
19.
∨ ∨ ∧ ∧ ∧ ∨ > >
(a) (b) (c) (d) MPPSC 2018

10. (a) (b) (c) (d)


20.
(a) (b) (c) (d)
11.
(a) (b) (c) (d) (e)

21.

A
(a) (b) (c) (d)

M
12.

H
(a) (b) (c) (d) (e)

TT
22.
(a) (b) (c) (d)
AFCAT 2017 A
W
(a) (b) (c) (d)
13.
SH

RRB (ECRC) 2009, 2008

23.
A

(a) (b) (c) (d)


@

14. 3 3 3 3
3 3
3 3
3
3
(a) (b) (c) (d)
SSC (Constable) 2012
(a) (b) (c) (d)
UP Police (Constable) 2018 24.
15. 2 1 7 5

2 4 1 0 3 4 2 3
(a) (b) (c) (d) (e)
(a) (b) (c) (d)
25.
16.

(a) (b) (c) (d) (e)


(a) (b) (c) (d)
26.
17.

(a) (b) (c) (d) (e)


(a) (b) (c) (d)
348 How to Crack Test of Reasoning NON-VERBAL

27. 32.

(a) (b) (c) (d) (e)


(a) (b) (c) (d)
AFCAT 2017 33.
28.

(a) (b) (c) (d) (e)


(a) (b) (c) (d) (e) 34.

29.
(a) (b) (c) (d) (e)
35. x x
x
x x x
x x x x x x x
(a) (b) (c) (d) x +++++ x xx x x
x x x x x x x x
AFCAT 2017 x x x x x

A
(a) (b) (c) (d) (e)

M
30. 36.

H
TT
(a) (b) (c) (d) (e) (a) (b) (c) (d) (e)
31 A
W
37.
SH

(a) (b) (c) (d)


A

RRB (ECRC) 2009 (a) (b) (c) (d) (e)


@

Expert Level Exercise


Directions (Q. Nos. 1-27) In each of the 3.
problem, out of five/four figures, four/three are
similar in a certain manner. Choose the figure
which is different from others.
1. 4.

(a) (b) (c) (d)


SSC (10+2) 2013
5.
2.

(a) (b) (c) (d) (a) (b) (c) (d) (e)


Chapter 02 CLASSIFICATION 349

6. 15.

(a) (b) (c) (d) (e) (a) (b) (c) (d) (e)
7. 16. x
x
CC

X
xx XX

X
X
(a) (b) (c) (d) (a) (b) (c) (d) (e)
MPPSC 2017 17.
8.

(a) (b) (c) (d) (e)

(a) (b) (c) (d) 18.


SSC (Steno) 2012

A
9.

M
(a) (b) (c) (d) (e)

H
19.

TT
(a) (b) (c) (d) (e)

10.
(a) A
(b) (c) (d) (e)
W
20.
SH

11.
A

(a) (b) (c) (d) (e)


@

21.
(a) (b) (c) (d) (e)
12.
(a) (b) (c) (d)
SSC (10+2) 2013
(a) (b) (c) (d) (e)
22.
13.

(a) (b) (c) (d) (e)


(a) (b) (c) (d) (e)
23.
14.

(a) (b) (c) (d) (e)


(a) (b) (c) (d) (e)
350 How to Crack Test of Reasoning NON-VERBAL

24. 26.
+

+
+
+

(a) (b) (c) (d) (e) (a) (b) (c) (d) (e)
25. 27.

(a) (b) (c) (d) (e) (a) (b) (c) (d) (e)

Answers
Base Level Exercise
1. (d) 2. (c) 3. (b) 4. (c) 5. (d) 6. (c) 7. (c) 8. (c) 9. (c) 10. (d)
11. (c) 12. (d) 13. (d) 14. (d) 15. (a) 16. (d) 17. (b) 18. (c) 19. (c) 20. (d)
21. (e) 22. (d) 23. (b) 24. (c) 25. (c) 26. (c) 27. (d) 28. (b) 29. (b) 30. (c)

A
31. (a) 32. (e) 33. (c) 34. (e) 35. (b) 36. (b) 37. (c)

M
H
Expert Level Exercise

TT
1. (c) 2. (d) 3. (d) 4. (a) 5. (e) 6. (b) 7. (a) 8. (c) 9. (c) 10. (d)
11. (a) 12. (d) 13. (d) 14. (d) 15. (b) 16. (e) 17. (d)
A 18. (a) 19. (c) 20. (c)
W
21. (b) 22. (e) 23. (c) 24. (c) 25. (e) 26. (b) 27. (e)
SH
A

Answer with Explanations


@

Base Level Exercise


1. (d) Except figure (d), all other figures are made up of 7. (c) All figures except figure (c), are closed figures.
four lines. 8. (c) Except figure (c), in all other figures, the inner
2. (c) Figure (c) has one closed loop while others do design consists of less number of sides than
not have such loop. that of the outer design.
In option (c), the inner design has more sides
3. (b) Figure (b) is divided into two parts while others
than outer design.
are being divided into four parts.
9. (c) Except figure (c), in all other figures the given
4. (c) Direction of arrows in all figures except figure (c),
four designs are pointing in same direction.
is in clockwise direction.
10. (d) In all the figures except figure (d), the main
5. (d) Except figure (d), all other figures are made up of
figure is divided into four equal parts.
straight lines only.
11. (c) In all the figures except figure (c), arrow is
6. (c) In all other figures, except figure (c), the sum of
incomplete.
the number of dots is 14.
Chapter 02 CLASSIFICATION 351

12. (d) Except figure (d), all other figures have even 26. (c) In all other figures except (c), both the arrows
number of sides. are perpendicular to each other.
13. (d) Two lines on the extreme ends of all the figures, 27. (d) In all other figures, all the designs face different
except figure (d) have same direction of arrows. directions, but in figure (d) two of the four
In figure (d), the direction of arrows are different. designs face the same directions.
14. (d) Except (d), in all other figures, the symbols in 28. (b) In figure (b) only, the small line segment
opposite sectors are same. attached to the main line segment at the bottom
is perpendicular to it.
15. (a) Except (a), in all other figures, the sum of two
lower numbers is given on top of each figure. 29. (b) Except figure (b), in all other figures, the shaded
portion is between two circles.
16. (d) In all the figures except figure (d), the arrow is
added at the side of the figure while in figure (d) 30. (c) Except figure (c), all other figures have common
it is added to the corner. features, i.e. two squares and a line.
17. (b) All the figures except figure (b), are rotated 31. (a) Except figure (a), all figures are made up of four
forms of the same figure. lines, while figure (a) is made up of five lines.
18. (c) Except figure (c), all have undivided leave. 32. (e) All the figures are made of two circles of different
sizes, each circle is having a line within it. One
19. (c) Except (c), all figures have their legs pointing
line is vertical and the other horizontal. But in

A
upside.
figure (e), both the lines are horizontal.

M
20. (d) In all other figures, except (d), one of the line is
perpendicular to the vertical line.
33. (c) Except figure (c), all other figures have common

H
feature of having a single square either shaded

TT
21. (e) Only in figure (e), all the arrows are facing to the or unshaded at the intersection of lines.
same direction.
34. (e) Except figure (e), in all other figures the difference
22. (d) Figure (d) is made up of two geometrical
A
in number of lines and number of dots is two
W
figures while others are of single geometrical while in figure (e), the difference is one.
figures.
35. (b) All the figures except figure (b), consist of
SH

23. (b) In figure (b), position of S is different from other ‘×’ (cross) signs only. But figure (b) consist of ‘×’
figures. (cross) and ‘+’ (plus) signs.
A

24. (c) In all other figures except (c), both the line 36. (b) In all the other figures except (b), the designs half
@

segments are perpendicular to each other. arrow and half dot are facing each other.
25. (c) Only in figure (c), the cross inside the circle is 37. (c) Only in figure (c), the straight line divides the
attached in line with the main line segment. figure into two equal parts.

Expert Level Exercise


1. (c) In all other figures except (c), the shaded part 5. (e) The two small lines at the end of the lines are
and the triangle have one empty space between pointing in the same direction but in figure (e),
them similarly the shaded part and the small they are pointing in different direction (clockwise
circle have one empty space in between. and anti-clockwise).
2. (d) In all other figures, all the three designs face 6. (b) Except figure (b), all other figures contain figures
different directions, but in figure (d), two of the with one, two, three and four number of sides while
three designs face the same directions. figure (b) contains two figures with two sides each.
3. (d) In all other figures except (d), all the numbers 7. (a) All the figures except figure (a), are rotated forms
from 1 to 5 are in a serial order in anti-clockwise of the same figure.
direction.
8. (c) Except (c), all figures have shaded portion
4. (a) Except figure (a), both unshaded squares opposite to each other.
attached to both the lines are in same direction.
352 How to Crack Test of Reasoning NON-VERBAL

9. (c) In all other figures except (c), all the four small 19. (c) All the arcs along the sides of the figure are
figures in it are arranged in anti-clockwise direction arranged in such a way that any three arcs
in ascending order of their sides. adjustment are placed in similar way, i.e.
they face sides of the figure or faces outside.
10. (d) In all other figures, the number of black blocks
Figure (c) does not follow this rule and
inside the square is odd while in figure (d), it is even.
hence is different from other figures.
11. (a) In all other figures except (a), one dot is outside the
figure and one is inside.
20. (c) All other figures except (c), have a line of
symmetry.
12. (d) In all other figures except figure (d), two arcs are
facing inward and two arcs are facing outward.
21. (b) Only in figure (b), two straight lines are
present inside the square and the circle
13. (d) In all other figures, shaded portion of the central while it is absent in rest of the figure.
figure and sign ‘=’ are on the same side. In figure
(d), these two are on the opposite sides.
22. (e) Except figure (e), all other figures have
common features i.e. corners of inner figure
14. (d) Except figure (d), all other figures are made up of six and outer figure are joined with straight line.
line segments while figure (d) is made up of five line
segments.
23. (c) All other figures except (c), are similar in a
way that protruding in and out portions of
15. (b) In all other figures except (b), the number of arrows each of the figures are identical.
facing inside is more than the number of arrows

A
facing outside.
24. (c) Only in figure (c), both the semi-circles, white

M
and black, lie on the same side of the line
16. (e) In all other figures except (e), symbols inside and segment.

H
outside the figures are placed along the different
25. (e) Only in figure (e), the difference of lines of

TT
sides of the figures. In figure (e), these symbols are
the figure and lines in it is two whereas in
placed in the same direction.
others the difference is three.
17. (d) In all other figures except figure (d), the outer line is A
26. (b) Only in figure (b), placement of symbols ‘•’
W
placed adjacent to the unshaded circle.
and ‘+’ is different from rest of the figures.
18. (a) All the figures are placed straight with base down,
SH

then one of the pins lies along base and the other
27. (e) In all other figures, two bent lines segments
faces clockwise direction and single bent
pin lies along left side of the figure. Figure (a) does
A

line faces anti-clockwise direction. In figure


not follow this rule and hence is different from
(e), the case is just the opposite.
@

others.
Chapter 03 SERIES 353

03
Series
Series is a continuous sequence of figures Different positions have been marked in the
following a certain defined pattern. given figure.
A series of figures is formed when each of the Centre/Central Top/Upper middle
consecutive figures of the series is obtained design design
from the previous figure by following a certain
pattern like clockwise or anti-clockwise Upper left ψ

A
X = Upper right
rotation, movement of symbols inside the figure, design design

M
addition or deletion of designs etc. Middle left S ? ∆ Middle right

H
The problems based on series consist of design design

TT
four/five figures following a definite sequence Lower left ↑ C O Lower right
forming a set of problem figures followed by design design
four/five other figures forming a set of answer A
Down/Bottom middle design
W
figures. You are required to select one of the
figures from the set of answer figures which
3. Angular Movement of Designs
SH

will continue the same sequence correctly.


The candidate should be aware about the
To solve questions on series a candidate must
A

angular movement of designs in clockwise


have a clear vision of the concept like rotation,
@

and anti-clockwise directions. Clockwise and


angles, steps of movement, different positions
anti-clockwise movement of angles is shown
etc. which are discussed below
in the given figure.
1. Rotational Direction Clockwise movement
The rotational direction basically states the
clockwise (in the direction of motion of clock’s A B C
hands) and anti-clockwise (opposite to the
direction of motion of clock’s hands) directions. 45º 90º
360º 135º
H D
12 1 12 1 315º 180º
11 11
10 2 10 2
9 3 9 3
270º 225º
8 4 8 4
G F E
7 6 5 7 6 5

Clockwise movement Anti-clockwise movement


A → B = 45 ° , A→ C = 90 °
2. Position of Designs A → D = 135 °, A→ E = 180 °
The candidate should be aware of the designs A → F = 225 °, A→ G = 270 °
and their positions to gauge their movement. A → H = 315 °, A→ A = 360 °
354 How to Crack Test of Reasoning NON-VERBAL

Anti-clockwise movement Anti-clockwise movement

A B C A B C

360º 315º
45º 270º
H D H D
90º 225º
135º 180º

G F E G F E

A → H = 45 °, A → G = 90 ° 1
A→H= arm/step, A → G = 1 arm/step
A → F = 135 °, A → E = 180 ° 2
A → D = 225 °, A → C = 270 ° 1
A → F = 1 arms/steps, A → E = 2 arms/steps
A → B = 315 °, A → A = 360 ° 2
1

A
4. Movement of Designs A → D = 2 arms/steps, A → C = 3 arms/steps

M
2
Through Distance 1

H
A → B = 3 arms/steps, A → A = 4 arms/steps
The clockwise and anti-clockwise movement 2

TT
of designs through distance in clockwise and
anti-clockwise directions are shown below. 5. Movement of Designs Through
Clockwise movement A
Angles and Distance
W
The concept of angular movement and
SH

A B C movement through distance can be combined


together and described as below
A
@

H D 1 Step 1
Step
2 2
or 45° or 45°
1 Step 1 Step
2 2
G F E or 45° or 45°
1 Step 1 Step
2 2
1 or 45°
A→B= arm/step or 45°
1 Step 1 Step
2 2
2
A → C = 1 arm/step or 45° or 45°
1
A → D = 1 arms/steps
2 1
A → E = 2 arms/steps From the given figure, 45 ° = step,
2
1
A → F = 2 arms/steps 1
2 90 ° = 1 step, 135 ° = 1 steps,
2
A → G = 3 arms/steps
1 180 ° = 2 steps and so on.
A → H = 3 arms/steps These concepts will be very helpful in
2
solving the problems based on symmetry and
A → A = 4 arms/steps
visual designs.
Chapter 03 SERIES 355

Examples given below, will help you to Solution (b) In each successive problem figures, two,
understand the concept of series. three, four,… elements are added and the previous
elements/designs rotate through 180°.
Directions (Illustrations 1-7) Each of the
following question consists of four/five figures as Illustration 4.
the problem figures followed by four/five figures Problem Figures
marked (a), (b), (c), (d) and (e) as the answer
figures. Select correct answer figure which will
continue the series as established by the problem S C
figures. C
Illustration 1. Answer Figures
Problem Figures C S

C
(a) (b) (c) (d) (e)
Answer Figures
Solution (c) The first two symbols in anti-clockwise
direction starting from empty space interchange their

A
positions while the third symbol moves one step
anti-clockwise and is replaced by a new one in

M
(a) (b) (c) (d) (e)
each step.

H
Solution (b) In each successive problem figure, a pin
is added on the left hand side of existing pin/pins. Illustration 5.

TT
The head of the pin is in a direction opposite to Problem Figures
adjacent pin.
A
W
Illustration 2.
Problem Figures
SH

Z Answer Figures
A
@

+ + + + Z +

Answer Figures
Z Z Z (a) (b) (c) (d) (e)
+ Z Solution (e) In each successive figure, the lowermost
+ $ + × C + ? line segment is converted into a curve. In the
(a) (b) (c) (d) second step, the second line segment also gets
converted into a curve and the existing curve is
Solution (d) In each successive problem figure, the inverted. This process continues and the curve from
element at the bottom middle position changes its bottom gets converted into a straight line segment.
position and a new element comes at its place.
Illustration 6.
Illustration 3.
Problem Figures
Problem Figures

MPPSC 2018
Answer Figures Answer Figures

(a) (b) (c) (d)


(a) (b) (c) (d)
356 How to Crack Test of Reasoning NON-VERBAL

Solution (c) The two elements together move two Answer Figures
spaces (each space is equal to half-side of the
square boundary) and three spaces in
anti-clockwise direction alternately. Also, in first
step, ‘ ’ symbol rotates 180°, in second and third
steps, it rotates 90° clockwise. This process will (a) (b) (c) (d)
continue in further steps. SSC (CPO) 2014
Solution (a) In each successive figure, the number of
Illustration 7. arrows in top left corner increases by one and the
Problem Figures number of dots in top right corner decreases by one.
The arrow in bottom left corner rotates 90 ° in
clockwise direction and changes its position from left
? to right block. The other figure in bottom right corner
also changes its position from right to left block.

Let us Practice

A
M
H
Directions (Q. Nos. 1- 28) In each of the question given below which one from the four/five answer

TT
figures should come at the right of the problem figures to complete the series logically.
Answer Figures
1. Problem Figures
A
W
SH

Answer Figures (a) (b) (c) (d) (e)


A

4. Problem Figures
@

(a) (b) (c) (d) (e)


Answer Figures
2. Problem Figures

(a) (b) (c) (d) (e)


Answer Figures 5. Problem Figures

(a) (b) (c) (d) (e) Answer Figures


3. Problem Figures

(a) (b) (c) (d) (e)


Chapter 03 SERIES 357

6. Problem Figures Answer Figures

(a) (b) (c) (d) (e)


Answer Figures
11. Problem Figures

(a) (b) (c) (d) (e)


Answer Figures
7. Problem Figures

(a) (b) (c) (d) (e)


Answer Figures 12. Problem Figures DSSSB PRT 2017

A
M
?

H
TT
(a) (b) (c) (d) (e)
Answer Figures
8. Problem Figures
A
W
SH

(a) (b) (c) (d)


13. Problem Figures
Answer Figures
A

? ?
@

? ?

(a) (b) (c) (d) (e) Answer Figures


9. Problem Figures
? ? ?
? ?
(a) (b) (c) (d) (e)

Answer Figures 14. Problem Figures


1 4 6 5 4 3 4 3 5 6 3 7 3 7 6

3 7 5 6 7 1 7 1 6 5 1 4 1 4 5
(a) (b) (c) (d) (e) Answer Figures
10. Problem Figures
7 6 3 5 4 1 5 7 1 5 7 1 5 7 1

4 5 1 6 7 3 3 4 6 6 4 3 6 4 2
(a) (b) (c) (d) (e)
358 How to Crack Test of Reasoning NON-VERBAL

15. Problem Figures Answer Figures


S S B S B S S
B

(a) (b) (c) (d) (e)


Answer Figures
20. Problem Figures
Z Z
Z
(a) (b) (c) (d) (e) Z Z

16. Problem Figures DSSSB PRT 2017 Answer Figures


Z Z Z
E S M ?
E

Z
Z
Answer Figures (a) (b) (c) (d) (e)

A
21. Problem Figures

M
M

M T T
M

H
TT
(a) (b) (c) (d)
17. Problem Figures Answer Figures
A
W
SH

Answer Figures (a) (b) (c) (d) (e)


A

22. Problem Figures


@

(a) (b) (c) (d) (e)


18. Problem Figures Answer Figures

LIVER RLIVE CRDIE ECRDI PEARI

(a) (b) (c) (d) (e)


Answer Figures
23. Problem Figures
IEPAR ZPRJI IPEAR ZPJRI IPAER
C C C
(a) (b) (c) (d) (e) C C

19. Problem Figures Answer Figures


C C C
S S S
B S C

S C
(a) (b) (c) (d) (e)
Chapter 03 SERIES 359

24. Problem Figures AFCAT 2017 Answer Figures


S
O S S O S O
O
Answer Figures (a) (b) (c) (d)
Directions (Q. Nos. 29-49) Which figure from
the answer figures will come in place of question
mark (?) in the following questions?
(a) (b) (c) (d) 29. Problem Figures SSC (Multitasking) 2012
25. Problem Figures S
?
S S

Answer Figures
Answer Figures S S S

A
S

M
(a) (b) (c) (d)

H
(a) (b) (c) (d) (e) 30. Problem Figures SSC (Constable) 2012

TT
26. Problem Figures
A ?
W
SH

Answer Figures
A

Answer Figures
@

C C O V

(a) (b) (c) (d)


T T T T T
(a) (b) (c) (d) (e) 31. Problem Figures SSC (CPO) 2012

27. Problem Figures


?

Answer Figures
Answer Figures

(a) (b) (c) (d)


(a) (b) (c) (d) (e)
32. Problem Figures SSC (CPO) 2012
28. Problem Figures AFCAT 2018

R S O
? ? R ?
R ? O S
360 How to Crack Test of Reasoning NON-VERBAL

Answer Figures Answer Figures

(a) (b) (c) (d) (e)


(a) (b) (c) (d)
37. Problem Figures
33. Problem Figures SSC (Steno) 2012
+ + ↑ + ↑
N
N


N ↑ ↑ + N ↑
N


+


?
Answer Figures


Answer Figures
↑ ↑ ↑


(a) (b) (c) (d) (e)
38. Problem Figures

A
(a) (b) (c) (d)

M
H
34. Problem Figures SSC (10+2) 2013

TT
Answer Figures
?
A
W
SH

Answer Figures (a) (b) (c) (d) (e)


39. Problem Figures
A
@

(a) (b) (c) (d)

35. Problem Figures SSC (10+2) 2013 Answer Figures

?
(a) (b) (c) (d) (e)
Answer Figures 40 Problem Figures

(a) (b) (c) (d)


Answer Figures
36. Problem Figures

(a) (b) (c) (d) (e)


Chapter 03 SERIES 361

41. Problem Figures Answer Figures

(a) (b) (c) (d) (e)


Answer Figures
44. Problem Figures

(a) (b) (c) (d) (e)


42. Problem Figures Answer Figures

(a) (b) (c) (d) (e)


Answer Figures

A
45. Problem Figures SBI (PO) 2012

M
8 Z X X 8

H
3 T X T Z 8 8 T Z S

TT
(a) (b) (c) (d) (e) Z X T 8 Z X

43. Problem Figures SBI (PO) 2013


Answer Figures
A
W
Z S Z Z
S X 8 Z X Z X X B
SH

X 8 8 S 8 S 8
(a) (b) (c) (d) (e)
A
@

Answers
1. (b) 2. (c) 3. (a) 4. (c) 5. (a) 6. (a) 7. (c) 8. (a) 9. (a) 10. (c)
11. (c) 12. (d) 13. (b) 14. (d) 5. (e) 16. (c) 17. (d) 18. (c) 19. (e) 20. (e)
21. (e) 22. (b) 23. (b) 24. (a) 25. (c) 26. (e) 27. (b) 28. (d) 29. (a) 30 (c)
31. (b) 32. (d) 33. (d) 34. (c) 35. (d) 36. (b) 37. (a) 38. (e) 39. (a) 40. (c)
41. (b) 42. (d) 43. (d) 44. (d) 45. (c)

Answer with Explanations


1. (b) Central element is rotating 90° clockwise and 4. (c) The diagram moves 180°, 90° alternatively in
corner elements are interchanging their positions. clockwise direction and point disappears from
circle in every alternate step.
2. (c) The dot moves 90°, 135°, 180°, 225° respectively, in
clockwise direction. 5. (a) The two elements move clockwise from side
to side and alternately replaced by a new
3. (a) In each step a new line is added as a side to each
one of the pre-existing lines, a new line appears element.
as a new element and a line appears inside the 6. (a) In each step, one sign is removed from the
square, when the square is completely formed. symbol in this order, circle, arrow head and
line.
362 How to Crack Test of Reasoning NON-VERBAL

7. (c) The main figure rotates 90°, 45°, 90°, 45° and 90° 18. (c) From problem figure first to second, third to
anti-clockwise in each step. fourth and fifth to answer figure, last letter
The shaded square become circle and circle becomes first letter and each letter moves one
repeat twice and square comes after circle. step towards right.
The shaded portion in the main figure shifts on 19. (e) The pattern followed by figure first to figure
the sides of line attached to square or circle. second; figure third to figure fourth and figure
8. (a) One and two parts of the line disappear fifth to answer figure is
alternately and the disappearance of line is * New design
anti-clockwise.
9. (a) Designs in figure first, third and fifth are shaded
from top to bottom row. Similarly, designs in
second, fourth and answer figure are shaded 20. (e) Pattern followed from figure first to figure
from top to bottom. second; figure third to figure fourth and figure
fifth to answer figure is
10. (c) In each step the two designs ‘H’ and ‘o’ are
moving one step in clockwise direction. Also,
two designs ‘ ’ and ‘∆’ are moving one step in
anti-clockwise direction inside the hexagon and
are alternately shaded.

A
11. (c) The two dots are removed and one cross is 21. (e) The elements move half step anti-clockwise in

M
added in each subsequent figure. So, in each step. One element is added in each step,
continuation with the given series of figures, the first to the right and then to the left of the main

H
next figure would be answer figure (c). element i.e .

TT
12. (d) Here, in each step, the inner figure comes in 22. (b) Here, two arcs and one arc is added in the
place of the outer figure and a new figure comes successive figures alternatively and existing
at the place of the inner figure. A
arcs rotate 90° anti-clockwise in each step.
W
13. (b) The upper and lower left 23. (b) As we move from figure first to second and
elements move to upper and
SH

figure third to fourth and figure fifth to answer


lower right positions figure the given pattern is followed.
respectively, the upper right (i) Symbol C moves from centre to the corner
A

comes to lower left, the lower diagonally opposite to symbol X.


@

right comes to the centre and central element


(ii) Symbol X moves from corner to centre.
comes to the upper left in each subsequent
step. The pattern is shown in figure. (iii) Symbol o remains at original position.

14. (d) Pattern followed by figure first to second, 24. (a) Here, the element (<) moves one step after
figure third to fourth and figure fifth to answer rotating 90° in anti-clockwise direction, in the
figure is middle row. Also, the element at the upper right
position changes into a new element and moves
one step and two steps alternately in clockwise
direction.
25. (c) One line in the lower design is added and one
15. (e) From problem figure first to second, third to line from the upper design is removed in a
fourth and fifth to answer figure, each symbol is particular order in each successive figure.
moving one step in clockwise direction after 26. (e) In each step the central element rotates 90°
rotating 180°. clockwise. The elements diagonally opposite to
each other interchanges their places after
16. (c) Here, in each step, the element at middle right
rotating 90° in clockwise direction. Also, a new
position rotates 90° in anti-clockwise direction element is added at the corners starting from
and moves to middle left position and a new top right corner and moving in clockwise
element comes at middle right position. direction.
17. (d) From problem figure first to second, third to 27. (b) On close observation, we find that arrow rotates
fourth and fifth to answer figure, each symbol is 45° clockwise and the other line rotates 45°
moving one step in anti-clockwise direction after anti-clockwise in each successive figure.
rotating 90° in clockwise direction.
Chapter 03 SERIES 363

28. (d) From figure first to second and figure fourth to fifth, 39. (a) In each alternate step, one more (l) and one
the elements at the top and bottom interchange more (|) is removed.
their place and the elements at right and left
position interchange their place. 40. (c) As we move from problem figure first to
second, the outer arc is inverted and the
Also, from figure second to third and figure fifth to semi-circle in the inner figure moves half
answer figure the elements move one place in
side. From problem figure second to third,
clock-wise direction.
the outer arc rotates 90° clockwise and the
29. (a) The sequence of the problem is as follows inner figure rotates 90° in anti-clockwise
direction. The same process is repeated in
the subsequent figures.
41. (b) Figure first is the reversed figure of figure
fourth and figure second is reversed figure of
figure fifth, then answer figure will be
From Fig. 1st to Fig. 2nd Fig. 2nd to Fig. 3rd
From Fig. 3rd to answer figure.
reversed figure of figure third.
30. (c) The figure rotates 90° in clockwise direction and the 42. (d) The main figure in problem figure first is
circle attached to the figure remains inside the same as in problem figure fifth. Therefore,
figure in even numbered figures and outside the main figure second will be same as the
figure in odd numbered figures. answer figure. Black circles and white circles

A
are moving in anti-clockwise direction,
31. (b) In each step, the arrow moves clockwise and small

M
alternatively.
circle moves anti-clockwise by 90° and the main

H
figure gets inverted in every second step. 43. (d) Figure third is obtained by rotating figure first
by 180° and adding a leaf to right of it.

TT
32. (d) In each successive problem figure, the small
figures at the end of vertices of the triangle moves Similarly, figure fourth is obtained by rotating
figure second by 180° and adding a leave to
clockwise and the arrow moves in and out
alternately in each successive steps. A
the right of it. So, the correct answer figure
W
will be (d).
33. (d) In each successive problem figure, black dot moves
SH

three steps clockwise and white circle moving two 44. (d) In given figure ‘ ’ and ‘X’ elements move as
steps clockwise.
A

34. (c) In each step the figure is rotated by


@

90° in anti-clockwise direction and


the small circle is moving one block
in clockwise direction. So, the correct Movement Movement
of figure and of figure
answer figure is
35. (d) In each step the arrow head gets inverted. The small 45. (c) When moved form figure 1 to 2 elements
line disappears in alternate steps. Also, a new line changed as
containing circle is added to the left side in the
inverse direction of pre-existing line containing
circle.
New
36. (b) Problem figures first, third and fifth are same. element
Problem figures second, fourth and answer figure
are same. Now, when move form figure 2 to 3 elements
37. (a) While moving from problem figure first to second, all change as
the signs move one step in clockwise direction and
from problem figure second to third, the signs
exchange their positions diagonally opposite to one
another. The same process is repeated in the
subsequent figures.
38. (e) Main figure moves in anti-clockwise direction and We can detect that the pattern in figure 5 to
three diagram attached to semi-circle changes the answer figure will be similar as in figure 2 to
place in cyclic order. 3.
364 How to Crack Test of Reasoning NON-VERBAL

04
Mirror Image
-

The figure obtained by putting a mirror in front of 2. When Mirror is


the real object is known as mirror image or we can Placed Horizontally
say that the reflection of an object into the mirror is

A
When a mirror is placed at the top or
called its mirror image.
bottom of the object, then the image, so

M
obtained is like the water image. Here,
Position of Mirror

H
the top and bottom of an object

TT
There are two positions of mirror to get the image interchange their places while LHS and
from it which are as follow RHS remains at their position.

1. When Mirror is Placed Vertically


A
Horizontal mirror image is obtained by
W
vertical inversion of the object.
SH

When a mirror is placed to the left or right of the The figure given below will help you to
object, then standard form of mirror image is understand the formation of image formed
formed. Here, the Left Hand Side (LHS) and Right by a horizontal mirror.
A

Hand Side (RHS) of an object interchange their


@

Real Object
places while top and bottom remains at their
position. Vertical mirror image is obtained by
lateral inversion of the object.
The figure given below will help you to
understand the formation of image formed by a Mirror
vertical mirror
Top Top

LHS RHS LHS RHS


Mirror image

In the above figure, you can see LHS and


Bottom Bottom RHS of the image remains unchanged but
Real Object Mirror Mirror Image the top and bottom interchange their
positions.
In the above figure, you can see clearly that Left
Hand Side (LHS) and Right Hand Side (RHS) Note In mirror image, if the position of mirror is not
have interchanged their position but top and given, then always consider mirror to be placed
bottom remains at the same place. on right side.
Chapter 04 MIRROR IMAGE 365

Mirror Images of English Alphabet Letters and Numbers


Images of capital and small letters of English Alphabet and numbers formed by vertical mirror
are given below
Mirror Images of Capital Letters
Real Image A B C D E F G H I J K L M
Mirror Image A B C D E F G H I J K L M
Real Image N O P Q R S T U V W X Y Z
Mirror Image O P Q R S T U V W X Y Z

Note The letters —A, H, I, M, O, T, U, V, W, X and Y have the same mirror images i.e. their left hand and right
hand sides do not appear to change their positions.
Mirror Images of Small Letters
Real Image a b c d e f g h i j k l m
Mirror Image a b c d e f g h i j k l m
Real Image n o p q r s t u v w x y z

A
Mirror Image n o p q r s t u v w x y z

M
Note The letters — i, l, o, v, w and x have the same mirror images i.e. their left hand and right hand sides do not

H
appear to change their positions.

TT
Mirror Images of Numbers
Real Image 1 2 3 4 5 6 7 A
8 9 0
W
Mirror Image 1 2 3 4 5 6 7 8 9 0
SH

Note Numbers 0 and 8 have the same mirror images.

Types of Questions
A

Illustration 1. FUN
@

(a) NUF (b) UNF (c) NUF (d) NUF


There are two types of questions, which are
generally asked in various competitive exams, Solution (a) If we put a mirror in front of the
regarding mirror images. word, we will get the following image
FUN NUF

T ype 1 Illustration 2. TRIUMPHS


(a) SHPMUIRT (b) SPMIURT
Letter/Number Images (c) STRIUMPH (d) SHPMUIRT
In this type of questions, a number, letter or a Solution (d) If we put a mirror in front of the
combination of letters and/or numbers is given word, we will get the image like
and the candidates are required to find their TRIUMPHS SHPMUIRT
mirror images out of given alternatives. Illustration 3. 2 3 4 5
5 42 2345 5 2
3 5432 34
Directions (Illustrations 1-4) In each of the given
question, a group of letters/numbers/combination of Solution (c) If we put a mirror in front of the
letters and numbers are given followed by four number, we will get the image like
alternatives. You have to select one alternative, which 2345 5432
exactly matches with the mirror image of the group of Illustration 4. 73AP4OD8
letters/number/combination of letters and numbers (a) 7 3 A P 4 O D 8 (b) 8 D O 4 P A 3 7
in the question. (c) 7 3 A P 4 O D 8 (d) 8 D O 4 P A 3 7
366 How to Crack Test of Reasoning NON-VERBAL

Solution (b) If we put a mirror in front of the Solution (d) Here, mirror is taken vertically to right
combination of letters and number, we will get the side. Hence, mirror image of figure (A) will be like
image like M
73AP4OD8 8DO4PA37
N

T ype 2 Illustration 8.
Geometrical Images
In this type of questions, the candidate is
required to find the mirror image of various (A) (a) (b) (c) (d)
geometrical shapes.
Solution (c) Here, mirror is taken horizontally to the
Directions (Illustrations 5-8) In each of the bottom. Hence, mirror image of figure (A) will be like
following question, choose the correct mirror
image from alternatives (a), (b), (c) and (d) of the
figure (A).
Illustration 5.

A
> < > < >

M
> > > > > > < > > >
> > > > < > > < >

H
>

TT
(A) (a) (b) (c) (d)
Solution (b) Here, mirror is taken vertically to right
T ype 3
side. Hence, mirror image of figure (A) will be like Mirror Image of ClocksA
W
> In this type of questions, a candidate has to find
>
SH

>
> the original positions of the hour hand and
>
minute hand of a clock from the positions of the
Figure (A) Mirror image
A

hour hand and the minute hand as seen in a


@

Illustration 6. SSC (10+2) 2018 mirror. On the basis of time indicated by mirror
image of the clock, we have to find the actual
time in the clock. The following examples will
help understand the concept better.

(A) (a) (b) (c) (d) Illustration 9. A clock seen through a mirror
shows quarter to three. What is the correct
Solution (b) Here, mirror is taken horizontally to the
time shown by the clock?
bottom. Hence, mirror image of figure (A) will be
like (a) 8 : 15 (b) 9 : 12 (c) 8 : 17 (d) 9 : 15
Solution (d) Here,
Time = 2 : 45 A Time = 9 : 15
M N

Fig. (X) B Fig. (Y)


Illustration 7.
M It is clear from Fig. (X) that if seen through a
mirror the time indicated by clock is 2.45. When
the mirror is placed vertically at AB, the mirror
N image i. e., Fig. (Y) shows the actual time in the
(A) (a) (b) (c) (d)
clock i. e. , (9 : 15).
Chapter 04 MIRROR IMAGE

Let us Practice
Base Level Exercise
Directions (Illustrations 1-19) In each of the following question, you are given a combination of
alphabets/numbers followed by four alternatives (a), (b), (c) and (d). Choose the alternative which
resembles the mirror image of the given combination.
1. NIRMAL A 14. 2 4 7 5 9 6
(a) ALAMRIN (b) ALAMRIN (a) 695742
(c) NRILAMA (d) INRMALA (b) 965742
(c) 247596
2. VINAYAKA
(d) 695742
(a) INVAYAKA (b) AKAYANIV
(c) AKAYANIV (d) NIVYAAKA 15. PROCRASTINATE
3. OBSTINATE (a) ETANITSARCORP (b) ETANITSARCORP

A
(c) RPORCASTNITAE (d) ETPROCRASTINA
(a) ETANITSBO (b) BOSTINATE

M
(c) ETANITSBO (d) SOBTNIATE 16. PRECARIOUS

H
4. VISHAL (a) SUOIRACERP (b) SUOIRACERP

TT
(a) LAHSIV (b) VISHAL (c) LAHSIV (d) VISHAL (c) SUOPRECARI (d) SPRECARIOU
17. PERFECTION
5. FIXING
(a) GNIXIF (b) FIXING (c) GNIXIF (d) GNIXIF (a) NOITCEFERP A (b) RPEFECTION
W
(c) NOITCEFREP (d) ERPFECTION
6. ZEBRA
SH

(a) ARBEZ (b) ARBEZ 18. RUN69test


(a) t s e t 6 9 N U R (b) t s e t 9 6 NU R
(c) ARBEZ (d) ARBEZ UR
(c) RU N 6 9 t s 2 t
A

(d) N 6 9 t es t
7. BUZZER
@

19. ANS43Q12
(a) REZZUB (b) RUZZEB (a) ANS43Q12 (b) 21Q34SNA
1 Q4
(c) ERZZUB (d) REZZBU (c) SNA 3 2 (d) 12Q43AN S
8. 1 3 9 4 20. How many letters of the English alphabet
(a) 4 6 3 1 (b) 4 9 3 1 (c) 4 9 3 1 (d) 4 9 3 1 appears same when looked at in a mirror?
9. GUIDED CGPSC 2013
(a) 9 (b) 10 (c) 11 (d) 12
(a) DEDIUG (b) DEDIUG (c) DE DIUG (d) DE IUG
(e) 13
10. disturbu
r t i 21. Which of the following collections of letters
(a) p s q (b) brutsid (c) disturd (d) brutsid
will look the same in the mirror?
11. FANTASY (a) OSMIHOM (b) VHRTRVH
(a) YSATNAF (b) FNTASAY (c) HIMOSTA (d) AOVIVOA
(c) YSATNAF (d) YFANTSAY 22. What is the mirror image of the following
12. RADIANT figure? SSC (CGL) 2016
(a) TNAIDAR (b) TNAIDAR Problem Figure Answer Figures
(c) TRADIAN (d) TIANRAD
13. BENEDICTION S S S
(a) NOITCIDENEB (b) NEBEDICTION
(c) NOITCIDENEB (d) NOIBENEDICT (a) (b) (c) (d)
368 How to Crack Test of Reasoning NON-VERBAL

Directions (Q. Nos. 23 and 24) In the following Directions (Q. Nos. 26-28) In each of the given
question, if a mirror is placed on the line MN, question, If mirror is placed on the line MN, then
then which of the answer figures is the right which of the answer figures is the right image of
image of the given figure? SSC (CPO) 2013 the given figure.
23. Problem Figure Answer Figures 26.
M
M
N
(A) (a) (b) (c) (d)
N
(a) (b) (c) (d)
27. M
24. Problem Figures Answer Figures
M
N
(A) (a) (b) (c) (d)
N
(a) (b) (c) (d) 28.
M
25. If a mirror is placed on the line MN, then

A
which of the answer figures is the right image
of the given figure? SSC (Multitasking) 2013

M
N
Problem Figure Answer Figures

H
M

TT
(a) A(b) (c) (d)
W
N (a) (b) (c) (d)
SSC (10+2) 2017, 2018
SH

Expert Level Exercise


A

Directions (Q. Nos. 5-13) In each of the


@

1. Looking into a mirror, the clock shows 9 : 30


as the time. The actual time is following question, if mirror is placed on the line
(a) 2 : 30 (b) 3 : 30 MN, then which of the answer figures is the right
(c) 4 : 30 (d) 6 : 30 image of the given figure.
2. When seen through a mirror, a clock shows Problem Figure Answer Figures
8h and 30 min (8:30). The correct time is 5. M
CGPSC 2014
(a) 2 : 30 (b) 3 : 30
(c) 4 : 30 (d) 5 : 30 N
(e) None of these (a) (b) (c) (d)

3. By looking in a mirror, it appears that it is SSC (CPO) 2015


6 : 30 in the clock. What is the real time? M
6.
(a) 6 : 30 (b) 5 : 30
(c) 6 : 00 (d) 5 : 50 N
(a) (b) (c) (d)
4. A clock seen through a mirror shows quarter BSSC (CGL) 2015
past three. What is the correct time shown by 7. M
the clock?
(a) 9 : 45 (b) 9 : 15 N
(c) 8 : 45 (d) 3 : 15 (a) (b) (c) (d)
SSC (CPO) 2015
Chapter 04 MIRROR IMAGE 369

Problem Figure Answer Figures Directions (Q. Nos. 14-17) In each of the
8. M 2 2
following question, if a mirror is placed on the
2 2 2
line MN, then which of the answer figures is the
G G G
G
G right image of the given problem figure?
N
(a) (b) (c) (d) 14. Problem Figure Answer Figures
SSC (CPO) 2015 M N
9. M

N
(a) (b) (c) (d) (a) (b) (c) (d)
10. M SSC (CGL) 2012
S U N U S S U U S

15. M N
N S U N N N
N
(a) (b) (c) (d)
SSC (CPO) 2016
(a) (b) (c) (d)
11. M

A
APPROACH HCAORPPA
R A
HC O PPA HCAORPPA APPROACH
SSC (CGL) 2017

M
N 16. M

H
(a) (b) (c) (d)

TT
MPPSC 2018, SSC (CPO) 2014
N
12. M (a) (b) (c) (d)
A SSC (Steno) 2016
W
N 17. M
(a) (b) (c) (d)
SH

SSC (10+2) 2014


13. M
A
@

N (a) (b) (c) (d)


N SSC (CGL) 2013
(a) (b) (c) (d)
SC (CPO) 2017, SSC (CGL) 2014

Answers
Base Level Exercise
1. (b) 2. (c) 3. (a) 4. (a) 5. (d) 6. (a) 7. (a) 8. (c) 9. (a) 10. (b)
11. (a) 12. (b) 13. (c) 14. (d) 15. (b) 16. (a) 17. (c) 18. (b) 19. (b) 20. (c)
21. (d) 22. (a) 23. (b) 24. (b) 25. (a) 26. (d) 27. (a) 28. (d)

Expert Level Exercise


1. (a) 2. (b) 3. (b) 4. (c) 5. (c) 6. (c) 7. (c) 8. (d) 9. (c) 10. (b)
11. (c) 12. (c) 13. (b) 14. (d) 15. (b) 16. (b) 17. (a)
How to Crack Test of Reasoning NON-VERBAL

Answer with Explanations


Base Level Exercise
1. (b) NIRMAL A ALAMRIN 24. (b) M

2. (c) VINAYAKA AKAYANIV


3. (a) OBSTINATE ETANITSBO
N
4. (a) VISHAL LAHSIV
25. (a) M
5. (d) FIXING
6. (a) ZEBRA ARBEZ
7. (a) BUZZER REZZUB N
8. (c) 1 3 9 4 4 9 3 1 M
26. (d)

A
9. (a) GUIDED DEDIUG

M
10. (b) disturb b r u t s i d

H
N
11. (a) FANTASY YSATNAF

TT
27. (a) M
12. (b) RADIANT TNAIDAR
13. (c) BENEDICTION NOITCIDENEB A
W
N
14. (d) 2 4 7 5 9 6 6 9 5 7 4 2
SH

28. (d) M
15. (b) PROCRASTINATE ETANITSARCORP
A

16. (a) PRECARIOUS SUOIRACERP


@

N
17. (c) PERFECTION NOITCEFREP

18. (b) RUN69test t s e t 96 NUR Expert Level Exercise


19. (b) ANS43Q12 21Q34SNA
1. (a) Here,
20. (c) The english alphabets which appear same in
mirror are. A, H, I, M, O, T, U, V, W, X and Y.
21. (d) The mirror image of AOVIVOA will look exactly
9:30 2:30
like it in the mirror.
22. (a) Clearly, this clocks shows the time 2 : 30.
M
S S
2. (b) Here,
N
23. (b) M

8:30 3:30
N Clearly, this clocks shows the time 3 : 30.
Chapter 04 MIRROR IMAGE 371

3. (b) Here, 11. (c) M

APPROACH HCAORPPA

N
6:30 5:30
12. (c) M
Clearly, this clock shows the time 5 : 30.
4. (c) Here,

N
13. (b) M
3:15 8:45
Clearly, this clock shows the time 8 : 45.
5. (c) M N

14. (d)

A
N

M
6. (c) M
M N

H
TT
N
A
W
7. (c) M 15. (b)
SH
A

M N
N M N
@

8. (d) M

16. (b) M
N
9. (c) M N
17. (a) M
N
10. (b) S U
M U S

N
N
N N
372 How to Crack Test of Reasoning NON-VERBAL

05

Water Image
Top
When an object is placed near a water source like river, pond or Real object
water tub etc, then the image formed in the water is called water
image of that object. LHS RHS
It is obtained by inverting an object vertically i.e. upside down.

A
Bottom
The water image of the figure looks like the mirror image of the object,

M
when the mirror is placed horizontally at the bottom of the object. Water
source

H
Top
From the adjacent figure, it is clear that the water image is an

TT
inverted form of a real object in which LHS (Left Hand Side) and LHS RHS
RHS (Right Hand Side) remains unchanged but the top and bottom
of the object gets interchanged i.e. top becomes bottom and bottom A Water Image
W
becomes top.
Bottom
SH

Water Images of English Alphabet Letters and Numbers


Water Images of Capital Letters
A

Real Image A B C D E F G H I J K L M
@

A B C D E F G H I J K L M
Water Image
Real Image N O P Q R S T U V W X Y Z
N O P Q R S T U V W X Y Z
Water Image

Note The letters which have the same water images are — C, D, E, H, I, O and X.
Water Images of Small Letters
Real Image a b c d e f g h i j k l m
a b c d e f g h i j k l m
Water Image
Real Image n o p q r s t u v w x y z
Water Image n o p q r s t u v w x y z

Note The letters which have the same water images are — c, l, o and x.
Water Images of Numbers

Real Image 0 1 2 3 4 5 6 7 8 9
0 1 2 3 4 5 6 7 8 9
Water Image
Note Numbers 0, 3 and 8 have the same water images.
Chapter 05 WATER IMAGE 373

Types of Questions Directions (Illustrations 4-6) In each of the


following question, choose the correct water
There are two types of questions based on water image of the figure (A) from the given four
images, which can be asked in various alternatives (a), (b), (c) and (d).
competitive examinations. Illustration 4.

pe
Letter/Number Images (A) (a) (b) (c) (d)
In this type of questions, a combination of letters Solution (b) The correct water image of the given
and/or numbers is given and the candidates are figure (A) is as shown below
required to find their water images out of given
alternatives. object
water
Directions (Illustrations 1-3) In each of the
following question, a combination of alphabets
and/or numbers followed by four alternatives (a), water
(b), (c) and (d) showing possible water images of that image

A
word or number is given. Out of these four

M
alternatives choose the alternative which shows the Illustration 5.

H
correct water image of that word and/or number.

TT
Illustration 1. FROG
FR G FR G FR
(a) O (b) GORF (c) O (d) OG
Solution (a) The correct water image of given word is as (A) A
(a) (b) (c) (d)
W
shown below Solution (c) The correct water image of the given
SH

FROG
FROG figure (A) is as shown below

Illustration 2. 765492
A

7654 2 7654 2 7654 2 object


@

(a) 6 (b) 765492 (c) 9 (d) 6 water

Solution (a) The correct water image of given numbers is


as shown below water
765492 image
765492

Illustration 3. 96FSH52 Illustration 6.


96F F 52 F 52
(a) SH52 (b) 96FSH52 (c) 69 SH (d) 69 SH

Solution (c) The correct water image of given group of


letters and numbers is as shown below (A) (a) (b) (c) (d)
96FSH52
96FSH52 Solution (b) The correct water image of the given
figure (A) is as shown below
pe 2 object
water
Geometrical Images
In this type of questions, some figures/shapes
are given and the candidates are required to water
image
find corresponding water images out of given
alternatives.
374 How to Crack Test of Reasoning NON-VERBAL

Let us Practice
Directions (Q. Nos. 1-12) In each of the 13. The number of English alphabets (in capital
following question, a word/group of letters or letters) whose water images are same as the
numbers or both is followed by four alternatives original letters, is CGPSC 2014
(a), (b), (c) and (d) showing possible water image (a) 8 (b) 7
of that word/group of letters or numbers or both. (c) 6 (d) 5
One, out of these four alternatives, is the exact (e) None of these
water image of that word/group of letters or
Directions (Q. Nos. 14-20) In each of the
numbers or both. Choose the alternative which is
following question, a figure marked (A) is followed
the correct water image of that word/group of
by four other figures (a), (b), (c) and (d) showing
letters or numbers or both.
the possible water images of figure (A). Choose the
1. CORDIAL
C RD O AI R L A
correct water image of the figure (A) out of given
(a) L (b) CO DI four alternatives.
R A A
(c) CO DI L (d) L IDROC 14.

A
2. POLEMIC
L

M
(a) PO EI CM (b) CIMELOP
P L M P (A) (a) (b) (c) (d)
(c) O E I C (d) OLWCIE

H
3. FECUND 15.

TT
U F F U
(a) DN CE (b) EC N D
F F EC D NU
(c) ECDNU (d)
(A) A
(a) (b) (c) (d)
W
4. RADIANT
R D IA TI A R TNA A UP Police (Constable) 2018
(a) N (b) DI
SH

R T A A 16.
(c) DI N (d) TNAIDAR
5. MUNDANE
A

M A UM NA
(a) UN EN D (b) ND E
@

UM A (A) (a) (b) (c) (d)


(c) ND NE (d) ENADNUM
6. riser e 17.
s
i i e
(a) (b) esir (c) r s e (d) sir
7. wrote (A) (a) (b) (c) (d)
t rw te r t
(a) wrote ro o oe
w e w
(b) (c) (d) 18.
8. 013 1 013 ××××
××××
(a) 0 3 (b) 013 (c) (d) 013 ××××
××××

9. 3713 (A) (a) (b) (c) (d)


371 3 3713 1 3173
(a) (b) (c) 3 73 (d)
SSC (CGL) 2013
10. SUPERFLOUS UPER LOU 19.
(a) SUOLFREPUS (b) S F S
SUPES O UPERFLOUS
(c) U LFR (d) S
11. D6Z7F4
D 7 4 Z D6CZ7F4 D6Z7F4 D ZF4
(A) (a) (b) (c) (d)
(a) 9 F (b) (c) (d) 6 7
20.
12. ab45CD67
45CD 7 a 45CD67
aq
(a) 6 (b) q
aq45CD 7 a 45CD 7
(c) 6 (d) q 9 (A) (a) (b) (c) (d)
Chapter 05 WATER IMAGE 375

Answers
1. (b) 2. (c) 3. (b) 4. (c) 5. (c) 6. (a) 7. (c) 8. (c) 9. (b) 10. (d)
11. (c) 12. (b) 13. (b) 14. (c) 15. (a) 16. (c) 17. (d) 18. (d) 19. (d) 20. (c)

Answer with Explanations


1. (b) The correct water image of given CORDIAL 14. (c) The correct water image of the given figure (A) is
word is as shown CORDIAL as shown below

2. (c) The correct water image of given POLEMIC


word is as shown POLEMIC
15. (a) The correct water image of the given figure (A) is
3. (b) The correct water image of given FECUND as shown below
word is as shown FECUND

A
4. (c) The correct water image of given RADIANT

M
word is as shown RADIANT
16. (c) The correct water image of the given figure (A) is

H
as shown below

TT
5. (c) The correct water image of given MUNDANE
word is as shown MUNDANE

6. (a) The correct water image of given word A


W
rise
is as shown rise
SH

17. (d) The correct water image of the given figure (A) is
as shown below
7. (c) The correct water image of given word wrote
A

is as shown wrote
@

8. (c) The correct water image of given 013


numbers is as shown 013
18. (d) The correct water image of the given figure (A) is
as shown below
9. (b) The correct water image of given 3713
numbers is as shown 3713

10. (d) The correct water image of given SUPERFLOUS


word is as shown SUPERFLOUS

11. (c) The correct water image of given 19. (d) The correct water image of the given figure (A) is
D6Z7F4 as shown below
group of letters and numbers is as D6Z7F4
shown

12. (b) The correct water image of given ab45CD67


group of letters and numbers is as ab45CD67
shown
20. (c) The correct water image of the given figure (A) is
13. (b) There are 7 English alphabets (in capital letters) as shown below
whose water images are same as the original
letters, i.e. C, D, E, H, I, O and X.
376 How to Crack Test of Reasoning NON-VERBAL

6
Paper Folding
and Cutting

A
M
H
Paper folding and cutting problems are based Examples given here, will give you a better

TT
on a transparent sheet which is folded along a idea about the types of questions asked in
dotted line or folded and cut (or punched) in a various competitive exams.
particular manner.
A
Illustration 1. In the following problem, a square
W
Problems based on ‘Paper Folding’ involves transparent sheet with a pattern is given. Figure
folding a transparent sheet along a dotted
SH

out from amongst the four alternatives as how


line. So, that the design on one side of the
the pattern would appear when the transparent
dotted line gets superimposed on the design
A

on the other side. sheet is folded along the dotted line.


@

Transparent Answer
In such type of questions, a transparent sheet
Sheet Figures
having a certain design and a dotted line on it is
given. The candidates are required to identify
the design (or pattern) that would be formed
when the sheet is folded along the dotted line.
In the questions based on paper cutting, few (a) (b) (c) (d)
figures are given showing the way in which a
Solution (c) The right half of the transparent sheet is
piece of paper is to be folded and then cut from a being folded along the dotted line and placed on
particular section. The dotted line is the reference the left half of the sheet. Thus, the figure obtained
line along which the paper is to be folded and the resembles the answer figure (c).
arrow indicates the direction of the fold.
The candidates are required to identify the
design or pattern that would be formed when
the sheet is unfolded.
Chapter 6 PAPER FOLDING AND CUTTING 377

Illustration 2. In the following question, a set of then after unfolding the last fold the transparent
three figures (X), (Y) and (Z) have been given, sheet will look like as given in option (d) i.e.
showing a sequence in which paper is folded
and finally cut from a particular section.
These figures are followed by a set of answer
figures marked (a), (b), (c) and (d) showing
the design which the paper actually acquires Illustration 3. A paper sheet is folded in a particular
when it is unfolded. You need to select the manner and several punches (cuts) are made.
answer figure which is closest to the unfolded When unfolded the paper sheet looks like the
piece of paper. question figure (X). From the given options
select the one that follows the manner in which
Problem Figures
the paper is folded and punched.
Question Figure

X Y Z

A
Answer Figures [SSC (CPO) 2013]
X

M
Answer Figures

H
TT
(a) (b) (c) (d)
Solution (d) In figure (X), the square sheet of paper (a) A (b) (c) (d)
W
is being folded along the vertical line of symmetry,
Solution (a) When we fold the question figure, then it
so that right half of the sheet overlaps the left half.
SH

looks like as answer figure (a).


In figure (Y), the sheet is folded further to a quarter.
In figure (Z), two squares are punched in the folded
A

sheet. Clearly, the punched squares will


@

be created in each quarter of the paper


and after unfolding the first fold the
figure will look like as,
378 How to Crack Test of Reasoning NON-VERBAL

Let us Practice
Directions (Q. Nos. 1-12) In each of the following question, a figure marked as transparent
sheet is given and followed by four answer figures, one out of these four options resembles the
figure which is obtained by folding the transparent sheet along the dotted line. Find the answer
from these figures.
Transparent Answer Transparent Answer
Sheet Figures Sheet Figures
1. 9.

(a) (b) (c) (d)


(a) (b) (c) (d)
2.
10.

A
M
(a) (b) (c) (d)

H
(a) (b) (c) (d)
3.

TT
11.
(a) (b) (c) (d) A
W
(a) (b) (c) (d)
4.
SH

12.
A

(a) (b) (c) (d)


@

5. (a) (b) (c) (d)

13. If the following pattern is drawn on a


(a) (b) (c) (d) transparent rectangular sheet and folded
along the dotted line, how does it appear?
6.
Problem Figure SSC (10+2) 2012

(a) (b) (c) (d)

7.
Answer Figures
(a) (b) (c) (d)

8.

(a) (b) (c) (d) (a) (b) (c) (d)


Chapter 6 PAPER FOLDING AND CUTTING 379

Directions (Q. Nos. 14-23) A piece of paper is folded and a cut is made as shown below. From the
given responses indicate how it will appear when opened?
14. Problem Figures SSC (Multitasking) 2012 18. Problem Figures SSC (Steno) 2016

Answer Figures

Answer Figures

(a) (b) (c) (d)

(a) (b) (c) (d) 19. Problem Figures SSC (CPO) 2014

15. Problem Figures SSC (10+2) 2017

A
Answer Figures

M
H
Answer Figures

TT
(a) (b) (c) (d)
A
W
(a) (b) (c) (d)
20. Problem Figures SSC CGL (Tier I ) 2017
SH

16. Problem Figures SSC (CPO) 2016


A
@

Answer Figures

Answer Figures

(a) (b) (c) (d)


(a) (b) (c) (d)
21. Problem Figures SSC CGL (Tier I ) 2017
17. Problem Figures SSC (CPO) 2016

Answer Figures
Answer Figures

(a) (b) (c) (d) (a) (b) (c) (d)


380 How to Crack Test of Reasoning NON-VERBAL

22. Problem Figures SSC CGL (Tier I ) 2017 Problem Figure Answer Figures
27.

X (a) (b) (c) (d)


Answer Figures
28.

X (a) (b) (c) (d)


(a) (b) (c) (d)
29.
23. Problem Figures SSC (Steno) 2012

X (a) (b) (c) (d)


SSC (10+2) 2006
Answer Figures

A
30.

M
H
TT
X (a) (b) (c) (d)
(a) (b) (c) (d)

Directions (Q. Nos. 24-30) In each of the A


Directions (Q. Nos. 31-37) In each of the
W
following question, a set of three figures (X), (Y)
following question, a sheet (square/circle/ and (Z) has been given, showing a sequence in
SH

triangle) of paper is folded and punch is made. which a paper is folded and finally cut at a
When unfolded the paper sheet looks like the particular section. Below these figures a set of
A

question figure. See the answer figures and answer figures marked as (a), (b), (c) and (d)
@

select the one that follows the manner in which showing the design which the paper actually
the paper is folded and punch is made. acquires when it is unfolded, are also given.
Problem Figure Answer Figures You have to select the answer figure which is
closest to the unfolded piece of paper.
24.
31.

X (a) (b) (c) (d)


MAT 2009 X Y Z

25.

(a) (b) (c) (d)


X (a) (b) (c) (d)

26. 32.

X (a) (b) (c) (d) X Y Z


RRB (TC/CC) 2009
Chapter 6 PAPER FOLDING AND CUTTING 381

Directions (Q. Nos. 38-42) A piece of paper is


folded and cut as shown below in the problem
figures. From the given answer figures, indicate
(a) (b) (c) (d)
how it will appear when opened?
38. Problem Figures SSC (Multitasking) 2014
33. Cut

X Y Z

Answer Figures

(a) (b) (c) (d)


34.
39. Problem Figures SSC (Multitasking) 2013

A
M
X Y Z

H
Answer Figures

TT
(a) (b) (c) (d)

35. A
W
(a) (b) (c) (d)
SH

X Y Z SSC (CPO) 2014 40. Problem Figures BSSC (CGL) 2015


A
@

(a) (b) (c) (d)


Answer Figures
36.

X Y Z SSC (CGL) 2014

(a) (b) (c) (d)

41. Problem Figures SSC (FCI) 2012

37.

Answer Figures
X Y Z

(a) (b) (c) (d) (a) (b) (c) (d)


382 How to Crack Test of Reasoning NON-VERBAL

42. Problem Figures SSC (CPO) 2015 Problem Answer


Figure Figures
45.

Answer Figures
X (a) (b) (c) (d)

46.

(a) (b) (c) (d)


X (a) (b) (c) (d)
Directions (Q. Nos. 43-50) In each of the
following question, a sheet 47.
(square/circle/triangle) of paper is folded and
punch is made. When unfolded the paper sheet
looks like the question figure. See the answer X (a) (b) (c) (d)
figures and select the one that follows the

A
manner in which the paper is folded and punch 48.

M
is made.

H
Problem Answer

TT
Figure Figures X (a) (b) (c) (d)

43. 49.
A
W
SH

X (a) (b) (c) (d) X (a) (b) (c) (d)

50.
44.
A
@

X (a) (b) (c) (d) (a) (b) (c) (d)


SSC (CPO) 2015

Answers
1. (a) 2. (c) 3. (d) 4. (d) 5. (a) 6. (b) 7. (a) 8. (c) 9. (a) 10. (b)
11. (d) 12. (c) 13. (d) 14. (d) 15. (b) 16. (b) 17. (b) 18. (a) 19. (b) 20. (c)
21. (a) 22. (c) 23. (c) 24. (c) 25. (d) 26. (a) 27. (b) 28. (a) 29. (a) 30. (b)
31. (a) 32. (c) 33. (c) 34. (d) 35. (d) 36. (b) 37. (c) 38. (a) 39. (a) 40. (d)
41. (d) 42. (b) 43. (a) 44. (b) 45. (a) 46. (d) 47. (b) 48. (c) 49. (a) 50. (a)
Chapter 6 PAPER FOLDING AND CUTTING

Answer with Explanations


1. (a) The right half of the transparent sheet is folded 7. (a) The lower half of the transparent sheet is folded
along the dotted line and is placed on the left along the dotted line and is placed on the upper
half. The figure, thus obtained resembles the half. The figure, thus obtained resembles the
figure as shown in option (a) figure as shown in option (a)

8. (c) The right half of the transparent sheet is folded


2. (c) The right half of the transparent sheet is folded along the dotted line and is placed on the left
along the dotted line and is placed on the left half. The figure, thus obtained resembles the
half. The figure, thus obtained resembles the figure as shown in option (c)
figure as shown in option (c)

A
9. (a) The top left half of the transparent sheet is

M
3. (d) The bottom right half of the transparent sheet is folded along the dotted line and is placed on the
folded along the dotted line and is placed on the bottom right half. The figure, thus obtained

H
top left half. The figure, thus obtained resembles resembles the figure as shown in option (a)

TT
the figure as shown in option (d)

A
W
10. (b) The right half of the transparent sheet is folded
SH

along the dotted line and is placed on the left


4. (d) The left half of the transparent sheet is folded half. The figure, thus obtained resembles the
along the dotted line and is placed on the right
A

figure as shown in option (b)


half. The figure, thus obtained resembles the
@

figure as shown in option (d)

11. (d) The right half of the transparent sheet is folded


along the dotted line and is placed on the left
5. (a) The upper half of the transparent sheet is folded half. The figure, thus obtained resembles the
along the dotted line and is placed on the lower figure as shown in option (d)
half. The figure, thus obtained resembles the
figure as shown in option (a)

12. (c) The lower half of the transparent sheet is folded


6. (b) The lower half of the transparent sheet is folded along the dotted line and placed on the upper
along the dotted line and is placed on the upper half of the sheet. Then, the left half of the sheet is
half. The figure, thus obtained resembles the folded along the dotted line and placed on the
figure as shown in option (b) right half of the sheet. The figure, thus obtained
resembles the answer figure (c)
384 How to Crack Test of Reasoning NON-VERBAL

13. (d) The right half of the transparent sheet is folded 21. (a) After unfolding the last fold the transparent
along the dotted line and is placed on the left sheet will look as given in option (a)
half. The figure, thus obtained resembles the
figure as shown in option (d)

22. (c) After unfolding the last fold the transparent


sheet will look as given in option (c)

14. (d) After unfolding the last fold the transparent


sheet will look as given in option (d)
23. (c) After unfolding the last fold the transparent
sheet will look as given in option (c)

15. (b) After unfolding the last fold the transparent


sheet will look as given in option (b)

A
24. (c) The right half of the transparent sheet is folded

M
along the dotted line and is placed on the left

H
half. Then, the upper half of the sheet is folded
16. (b) After unfolding the last fold the transparent

TT
along the dotted line and placed on the lower
sheet will look as given in option (b) half of the sheet. The figure, thus obtained
resembles the figure as shown in option (c).
A
W
SH

17. (b) After unfolding the last fold the transparent


sheet will look as given in option (b)
A

25. (d) The right half of the transparent sheet is folded


@

along the dotted line and is placed on the left


half. Then, the upper half of the sheet is folded
along the dotted line and placed on the lower
18. (a) After unfolding the last fold the transparent half of the sheet. The figure, thus obtained
sheet will look as given in option (a) resembles the figure as shown in option (d).

19. (b) After unfolding the last fold the transparent


sheet will look as given in option (b)
26. (a) The left half of the transparent sheet is folded
along the dotted line and is placed on the right
half. Then, the upper half of the sheet is folded
along the dotted line and placed on the lower
half of the sheet. The figure, thus obtained
20. (c) After unfolding the last fold the transparent resembles the figure as shown in option (a).
sheet will look as given in option (c)
Chapter 6 PAPER FOLDING AND CUTTING 385

27. (b) The left half of the transparent sheet is folded 32. (c) After unfolding the last fold the transparent
along the dotted line and is placed on the right sheet will look as given in option (c)
half. Then, the upper half of the sheet is folded
along the dotted line and placed on the lower
half of the sheet. The figure, thus obtained
resembles the figure as shown in option (b).

33. (c) After unfolding the last fold the transparent


sheet will look as given in option (c)

28. (a) The left half of the transparent sheet is folded


along the dotted line and is placed on the right
half. Then, the upper half of the sheet is folded 34. (d) After unfolding the last fold the transparent
along the dotted line and placed on the lower sheet will look as given in option (d)
half of the sheet. The figure, thus obtained
resembles the figure as shown in option (a)

A
M
35. (d) After unfolding the last fold the transparent
sheet will look as given in option (d)

H
29. (a) The upper half of the transparent sheet is folded
along the dotted line and is placed on the lower

TT
half. Then, the top left of the sheet is folded and
placed on the bottom right of the right half of the
sheet. After that the top right is folded and A
W
placed at bottom left half. The figure, thus 36. (b) After unfolding the last fold the transparent
obtained resembles the figure as shown in sheet will look as given in option (b)
SH

option (a).
A
@

37. (c) After unfolding the last fold the transparent


sheet will look as given in option (c)
30. (b) The left half of the transparent sheet is folded
along the dotted line and is placed on the right
half. Then, the upper half of the sheet is folded
along the dotted line and placed on the lower
half of the sheet. The figure, thus obtained
resembles the figure as shown in option (b). 38. (a) After unfolding the last fold the transparent
sheet will look as given in option (a)

31. (a) After unfolding the last fold the transparent 39. (a) After unfolding the last fold the transparent
sheet will look as given in option (a) sheet will look as given in option (a)
386 How to Crack Test of Reasoning NON-VERBAL

40. (d) After unfolding the last fold the transparent 46. (d) The left half of the transparent sheet is folded
sheet will look as given in option (d) along the dotted line and is placed on the right
half. Then, the upper half of the sheet is folded
along the dotted line and placed on lower half of
the sheet.The figure, thus obtained resembles
the figure as shown in option (d)

41. (d) After unfolding the last fold the transparent


sheet will look as given in option (d)

47. (b) The bottom left half of the transparent sheet is


folded along the dotted line and is placed on the
42. (b) After unfolding the last fold the transparent top right half. Then, the upper half of the sheet is
sheet will look as given in option (b) folded along the dotted line and placed on lower
half of the sheet. The figure, thus obtained
resembles the figure as shown in option (b)

A
43. (a) The left half of the transparent sheet is folded

M
along the dotted line and is placed on the right

H
half. Then, the upper half of the sheet is folded 48. (c) The left half of the transparent sheet is folded
along the dotted line and placed on lower half of

TT
along the dotted line and is placed on the right
the sheet. The figure, thus obtained resembles half. Then, the upper half of the sheet is folded
the figure as shown in option (a) along the dotted line and placed on lower half of
A
the sheet. The figure, thus obtained resembles
W
the figure as shown in option (c)
SH
A

44. (b) The right half of the transparent sheet is folded


along the dotted line and is placed on the left
@

half. Then, the upper half of the sheet is folded 49. (a) The transparent sheet is folded in six equal parts
along the dotted line and placed on lower half of along the dotted line. The figure, thus obtained
the sheet. The figure, thus obtained resembles resembles the figure as shown in option (a)
the figure as shown in option (b)

45. (a) The left half of the transparent sheet is folded 50. (a) The left half of the transparent sheet is folded
along the dotted line and is placed on the right along the dotted line and is placed on the right
half. Then, the upper half of the sheet is folded half. Then, the upper half of the sheet is folded
along the dotted line and placed on lower half of along the dotted line and placed on lower half of
the sheet. The figure, thus obtained resembles the sheet. The figure, thus obtained resembles
the figure as shown in option (a) the figure as shown in option (a)
Chapter 07 GROUPING OF IDENTICAL FIGURES 387

07
Grouping of
Identical Figures

A
M
Grouping of different figures on the basis of certain common properties/ qualities, is called

H
grouping of identical figures.

TT
In this chapter, we have a set of few figures or geometrical shapes which are numbered as 1, 2,
3, 4 and so on. We are required to analyse these figures and classify them into groups consisting
of figures having same properties. Grouping of figures can be done on the basis of their shape, A
W
orientation, the number of elements and other such characteristics. The best answer is to be
selected from a given set of fairly close alternatives.
SH

Examples given below, will give you a better idea about the type of questions asked in
examinations.
A
@

Direction (Illustration 1) In the following question, group the given figures into three classes using
each figure only once.

(1) (2) (3) (4) (5) (6) (7) (8) (9)


(a) (7, 8, 9); (2, 4, 3); (1, 5, 6) (b) (1, 3, 2); (4, 5, 7); (6, 8, 9)
(c) (1, 6, 8); (3, 4, 7); (2, 5, 9) (d) (1, 6, 9); (3, 4, 7); (2, 5, 8)
Solution (d) By observing the given figures, we found that, figures (1), (6) and (9) can be grouped together, as
they all are triangles. Figures (3), (4) and (7) can be grouped together, as they all are quadrilaterals and figures
(2), (5) and (8) can be grouped together, as they all are pentagons.
Direction (Illustration 2) There are two classes of three figures each. Class ‘A’ figures differ in certain
way from the figures in class ‘B’. Which two of the four answer figures belong to class ‘A’?
Answer Figures

Class A Class B (1) (2) (3) (4)


(a) Both 1 and 3 (b) Both 1 and 2 (c) Both 2 and 4 (d) Both 2 and 3
Solution (d) By observing the given figures, we found that, each figure in class A consists of two similar closed
figures, which are placed one inside the other.
388 How to Crack Test of Reasoning NON-VERBAL

Let us Practice
Directions (Q. Nos. 1-17) In each of the following question, group the given figures into three classes
using each figure only once.

1.
SSC (CGL) 2009
(1) (2) (3) (4) (5) (6)

(a) (1, 4); (2, 3); (5, 6) (b) (1, 5); (2, 6); (4, 3)
(c) (1, 6); (2, 3); (4, 5) (d) (1, 2); (3, 6); (4, 5)

2.

(1) (2) (3) (4) (5) (6) (7) (8) (9)


(a) (1, 3, 4); (6, 7, 8); (2, 5, 9) (b) (1, 2, 3); (4, 5, 6); (7, 8, 9)

A
(c) (1, 5, 9); (2, 4, 7); (3, 6, 8) (d) (3, 7, 8); (1, 6, 5); (4, 2, 9)

M
3.

H
TT
1 2 3 4 5 6 7 8 9 SSC (CGL) 2004
(a) (1, 5, 7), (2, 4, 6), (3, 9, 8)
A
(b) (1, 5, 7), (2, 4, 8), (3, 6, 9)
W
(c) (1, 5, 7), (4, 8, 9), (2, 3, 6) (d) (1, 5, 7), (3, 8, 9), (2, 4, 6)
SH

4.
A
@

(1) (2) (3) (4) (5) (6) (7) (8) (9)


(a) (1, 4, 9); (2, 6, 8); (3, 5, 7) (b) (1, 4, 5); (3, 6, 8); (2, 7, 9)
(c) (1, 4, 6); (2, 7, 8); (3, 5, 9) (d) (1, 3, 6); (2, 4, 7); (5, 8, 9)

5.

(1) (2) (3) (4) (5) (6) (7) (8) (9)


(a) (1, 2, 4); (3, 5, 6); (8, 7, 9) (b) (9, 7, 6); (5, 3, 1); (4, 8, 2)
(c) (2, 3, 4); (7, 6, 5); (9, 8, 1) (d) (1, 4, 7); (2, 6, 9); (3, 5, 8)
6.

1 2 3 4 5 6 7 8 9 SSC (CGL) 2005


(a) (1, 5, 7), (2, 3, 9), (4, 6, 8) (b) (1, 3, 9), (2, 4, 6), (5, 7, 8)
(c) (3, 4, 7), (2, 4, 9), (1, 6, 8) (d) (6, 7, 9), (1, 3, 4), (2, 4, 8)

7.
SSC (CGL) 2002
(1) (2) (3) (4) (5) (6) (7) (8) (9)
(a) (1, 2, 3); (4, 8, 9); (5, 7, 6) (b) (4, 5, 7); (3, 1, 2); (7, 8, 9)
(c) (1, 3, 7); (4, 8, 9); (2, 5, 6) (d) (3, 5, 6); (8, 7, 4); (9, 1, 2)
Chapter 07 GROUPING OF IDENTICAL FIGURES 389

8.

(1) (2) (3) (4) (5) (6) (7) (8) (9)


(a) (1, 4, 8); (2, 5, 7); (3, 9, 6) (b) (1, 4, 6); (2, 5, 8); (3, 7, 9)
(c) (1, 4, 6); (2, 5, 7); (3, 8, 9) (d) (1, 2, 3); (4, 5, 6); (7, 8, 9)

9.

(1) (2) (3) (4) (5) (6) (7) (8) (9)


(a) (1, 5, 6); (2, 3, 4); (7, 8, 9) (b) (1, 2, 4); (3, 5, 8); (6, 7, 9)
(c) (5, 6, 7); (1, 2, 4); (3, 8, 9) (d) (1, 2, 4); (3, 5, 7); (6, 8, 9)
10.

1 2 3 4 5 6 7 8 9 MAT 2007

A
(a) (1, 5, 7), (2, 3, 8), (4, 6, 9) (b) (3, 6, 7), (1, 4, 8), (2, 5, 9)

M
(c) (1, 4, 9), (2, 5, 6), (3, 7, 8) (d) (1, 2, 6), (3, 4, 5), (7, 8, 9)

H
11.

TT
SSC (CPO) 2010
(1) (2) (3) (4) (5) (6) (7) A (8)
W
(a) (1, 4, 6); (3, 5); (2, 7, 8) (b) (2, 5, 8); (1, 3, 8); (4, 6)
SH

(c) (3, 7); (14, 5); (2, 5, 8) (d) (2, 5, 8); (1, 6); (4, 7)
12.
A
@

1 2 3 4 5 6 7 8 9 SSC (10 + 2) 2009


(a) (1, 5, 8), (3, 4, 7), (2, 6, 9) (b) (1, 3, 6), (4, 5, 9), (2, 7, 8)
(c) (1, 3, 6), (2, 5, 7), (4, 8, 9) (d) (6, 7, 8), (1, 3, 7), (2, 4, 9)

13.

(1) (2) (3) (4) (5) (6) (7) (8) (9)


(a) (1, 5, 8); (2, 6, 7); (3, 4, 9) (b) (1, 5, 7); (2, 6, 8); (3, 4, 9)
(c) (1, 7, 8); (2, 6, 9); (3, 4, 5) (d) (1, 5, 8); (2, 4, 7); (3, 6, 9)

14.
A M B H W D E N U
(1) (2) (3) (4) (5) (6) (7) (8) (9)

(a) (1, 3, 6); (2, 8, 9); (4, 7, 5) (b) (1, 4, 8); (2, 5, 7); (3, 6, 9)
(c) (1, 4, 7); (2, 5, 8); (3, 6, 9) (d) (1, 4, 7); (3, 5, 8); (2, 6, 9)
390 How to Crack Test of Reasoning NON-VERBAL

15.
SSC (CGL) 2003

(1) (2) (3) (4) (5) (6) (7) (8) (9)

(a) (1, 4, 7); (3, 6, 9); (2, 5, 8) (b) (1, 4, 5); (2, 6, 8); (3, 7, 9)
(c) (1, 7, 9); (3, 6, 8); (2, 4, 6) (d) (1, 6, 9); (2, 5, 8); (3, 4, 7)

16.
SSC (CGL) 2003
(1) (2) (3) (4) (5) (6) (7) (8) (9)

(a) (1, 4, 7); (3, 6, 9); (2, 5, 8) (b) (1, 4, 7); (2, 6, 9); (3, 5, 8)
(c) (1, 6, 9); (2, 4, 7); (3, 5, 8) (d) (1, 5, 7); (2, 6, 9); (3, 4, 8)
17.

(1) (2) (3) (4) (5) (6) (7) (8) (9)


SSC (CGL) 2003

A
M
(a) (1, 4, 7); (2, 5, 9); (3, 8, 6) (b) (2, 6, 9); (1, 4, 7); (5, 8, 3)

H
(c) (1, 4, 7); (2, 3, 6); (5, 8, 9) (d) (3, 5, 1); (4, 7, 8); (6, 2, 9)

TT
18. There are two classes of three figures each. Class ‘A’ figures differ in certain way from the figures
in class B. Which two of the four answer figures belong to class A?
A
W
SH

Class A Class B (1) (2) (3) (4)


A

(a) Both 1 and 3 (b) Both 1 and 2


@

(c) Both 2 and 4 (d) Both 2 and 3

Answers
1. (a) 2. (a) 3. (b) 4. (a) 5. (d) 6. (a) 7. (c) 8. (b) 9. (b) 10. (b)
11. (a) 12. (c) 13. (d) 14. (b) 15. (a) 16. (b) 17. (c) 18. (c)
Chapter 07 GROUPING OF IDENTICAL FIGURES 391

Answer with Explanations


1. (a) By observing the given figures, we found that, Figures (2, 5 and 8) are similar in shape.
figures (1 and 4) are similar. In figures (3, 7 and 9), there are eight small
Figures (2 and 3) are similar, as they all are circles around a big circle.
Triangles. 9. (b) By observing the given figures, we found that,
Figures (5 and 6) are similar. figures (1, 2 and 4) are made of three lines.
2. (a) By observing the given figures, we found that, Figures (3, 5 and 8) are made of four lines.
figures made by three lines = (1, 3, 4) Figure (6, 7 and 9) are made of five lines.
Figures made by four lines = (6, 7, 8) 10. (b) By observing the given figures, we found that,
Figures made by five lines = (2, 5, 9) Figures (3, 6 and 7) form a group of figures
3. (b) By observing the given figures, we found that, which are divided into three unequal parts.
figures (1, 5 and 7) are composed of two similar Figures (1, 4 and 8) form a group of figures
figures, one inside the other. which are divided into three equal parts.
Figures (2, 4 and 8) contain a figure placed Figures (2, 5 and 9) form a group of figures in
inside a different figure. Figures (3, 6 and 9) are which the figures are divided into three parts in
such a way that one part is half and the other

A
figures with thick boundaries.
two parts are one-fourth of the original figure.

M
4. (a) By observing the given figures, we found that, 11. (a) By observing the given figures, we found that,
figures (1, 4 and 9) are the similar. grouping the figure on the basis of their use,

H
Figures (2, 6 and 8) are the similar. figures (1, 4, 6) are used for lighting. Figures

TT
Figures (3, 5 and 7) are the similar. (3 and 5) are used for playing and figures
(2,7 and 8) are used for learning.
5. (d) By observing the given figures, we found that,
figures of four sides = (1, 4, 7) A
12. (c) By observing the given figures, we found that,
W
Figures of one big and small circle = (2, 6, 9) figures (1, 3 and 6) contain one complete circle
each. Figures (2, 5 and 7) contain a semi-circle
SH

Figures (3, 5 and 8) are wide from one side and


narrow from other. each. Figures (4, 8 and 9) contain a triangle each.
13. (d) By observing the given figures, we found that,
A

6. (a) By observing the given figures, we found that,


figures (1, 5 and 7) contain two similar figures, figures (1, 5 and 8) form a group of figure having
@

one inside the other. Figures (2, 3 and 9) form a a circle and a triangle. The triangle is divided
group of figures in which half portion of the into two equal parts. Figures (2, 4, and 7)
figure is enclosed inside that figure. contain figures which have curved portions.
Figures (4, 6 and 8) contain two straight lines Figures (3, 6 and 9) form a group of figures
perpendicular to each other dividing the figure formed by straight lines only.
into four parts. 14. (b) By observing the given figures, we found that,
7. (c) By observing the given figures, we found that, figures (1, 4 and 8) are made of three lines.
figures (1, 3 and 7) have three petals, three Figures (2, 5 and 7) are made of four sides.
rectangles and three triangles, respectively. So, Figures (3, 6 and 9) have curved portions.
they should be grouped in one group. Figures 15. (a) By observing the given figures, we found that,
(4, 8 and 9) have one triangle, one rectangle and figures (1, 4, 7); (3, 6, 9) and (2, 5, 8) are similar.
one oval respectively. So, they should be
grouped in one group. 16. (b) By observing the given figures, we found that,
Figures (2, 5 and 6) have five petals, five figures (1, 4, 7); (2, 6, 9) and (3, 5, 8) are similar.
triangles and five squares, respectively. So, they 17. (c) By observing the given figures, we found that,
should be grouped in one group. figures (1, 4, 7); (2, 3, 6) and (5, 8, 9) are similar.
8. (b) By observing the given figures, we found that, 18. (c) By observing the given figures, we found that,
figures (1, 4 and 6) have two triangles, two each figure in class A consists of two similar
circles and two hexagons, respectively. intersected figures.
392 How to Crack Test of Reasoning NON-VERBAL

08

Formation of
Figures

A
Formation of figures refers to the use of Answer Figures

M
pieces of different designs to construct a

H
desired figure. It requires a high spatial

TT
visualisation skill, as different fragmented
parts of a figure are combined to form the (a) (b) (c) (d)
desired figure. A
Solution (a) Figure given in option (a) can be formed by
W
In this chapter, two sets of figures are joining the pieces given in question figure, as shown
below.
SH

provided namely problem figure and


answer figures. A candidate is asked to
A

arranged the different components of the


problem figure, so as to form one of the
@

answer figure. Illustration 2. In the question given below, find out


Besides this, there are questions in which which of the figures can be formed from the
one problem figure is given and the pieces given in the problem figure.
candidate is asked to find out the correct Problem Figure
answer figure in which all the pieces
which are required to form the question
figure are present.
Examples given below, will give you a better
idea about the types of questions generally Answer Figures
asked in various competitive exams.
Illustration 1. In the question given below,
find out which of the figures can be
formed from the pieces given in the (a) (b) (c) (d)
problem figure. Solution (b) Figure given in option (b) can be formed by
joining the pieces given in question figure, as shown
Problem Figure
below.
Chapter 08 FORMATION OF FIGURES 393

Illustration 3. In the question given below, find (a) ABC (b) BCD
the figure given in answer figures, that can (c) CDE (d) BDE
form the problem figure. SSC (CGL) 2013 Solution (c) An equilateral triangle has each angle
measure equal to 60 ° and the lengths of all its three
Problem Figure Answer Figures
sides are also equal.
Now, we cannot measure the exact value of angle
and sides but we have to closely approximate these
measures and values. So, keeping these points in
(a) (b) (c) (d) mind, we first combine figure E and C which will
Solution (a) The problem figure can be formed by look like as
joining the pieces given in option (a), as shown
below.

Now, the missing figure which can fit in the above


Illustration 4. In the following question, five
figure is D. So, the complete figure is shown as
figures are given. Out of them, find the three
figures that can be joined to form an

A
equilateral triangle. E C

M
D

H
TT
So, the correct group of figures is C, D and E.
(A) (B) (C) (D) (E)
A
W
SH

Let us Practice
A
@

Directions (Q. Nos. 1-11) In each of the following question, find out which of the figures (a), (b), (c)
and (d) can be formed from the pieces given in problem figures.
1. Problem Figure Answer Figures 3. Problem Figure Answer Figures

(a) (b) (c) (d)


(a) (b) (c) (d)
SSC (CGL) 2010
SSC (CGL) 2010 4. Problem Answer
2. Problem Figure Answer Figures Figure Figures

(a) (b) (c) (d) (a) (b) (c) (d)


394 How to Crack Test of Reasoning NON-VERBAL

5. Problem Answer
Directions (Q. Nos. 12-20) In each of the
following questions, find the figure given in the
Figure Figures
answer choices, that can form the problem figure
12. Problem Figure Answer Figures

(a) (b) (c) (d)


SSC (Multitasking) 2013; (CPO) 2008
6. (a) (b) (c) (d)
SSC (CPO) 2008

13.
(a) (b) (c) (d)

7.
(a) (b) (c) (d)
14.
(a) (b) (c) (d)

A
8.

M
(a) (b)

H
SSC (10+2) 2006

TT
15.
(a) (b) (c) (d)

9. A
W
(a) (b) (c) (d)
SH

16.
(a) (b) (c) (d)
A

SSC (10+2) 2014


@

10. (a) (b) (c) (d)


MAT 2007
17.
(a) (b) (c) (d)
SSC (CGL) 2015
11. Problem Figure
(a) (b) (c) (d)

18.

Answer Figures (a) (b) (c) (d)

SSC (CGL) 2013


19.

(a) (b) (c) (d)


SSC (CPO) 2011 (a) (b) (c) (d)
Chapter 08 FORMATION OF FIGURES 395

20. Problem Figure Answer Figures 23. Which of following answer figure can be
combined with the question figure, so as to
form a square.
Problem Figure Answer Figures
(a) (b)
SSC (CPO) 2015
21. Which of the answer figures includes the
separate components found in the question (a) (b) (c) (d)
figure?
Problem Figure Answer Figures 24. In the following question, a set of five figures
marked (A), (B), (C), (D) and (E) are given
followed by four alternatives (a), (b), (c) and
(d). Select the alternative which contains those
figures which, if fitted together, will form a
(a) (b) (c) (d) complete square.
22. Identify the response figure in which the
figures given in problem figure are found.

A
Problem Figure Answer Figures
(A) (B) (C) (D) (E)

M
(a) ADE

H
(b) BED

TT
(c) BCD
(a) (b) (c) (d) (d) CDE
A
W
SH

Answers
A

1. (b) 2. (b) 3. (a) 4. (b) 5. (b) 6. (c) 7. (b) 8. (b) 9. (d) 10. (b)
@

11. (c) 12. (c) 13. (c) 14. (d) 15. (c) 16. (c) 17. (c) 18. (c) 19. (b) 20. (d)
21. (b) 22. (b) 23. (a) 24. (c)
How to Crack Test of Reasoning NON-VERBAL

Answer with Explanations


1. (b) Figure given in option (b) can be formed by 8. (b) Figure given in option (b) can be formed by
joining the pieces given in question figure, joining the pieces given in question figure, as
as shown below. shown below.

2. (b) Figure given in option (b) can be formed by 9. (d) Figure given in option (d) can be formed by
joining the pieces given in question figure, as joining the pieces given in question figure, as
shown below. shown below.

3. (a) Figure given in option (a) can be formed by 10. (b) Figure given in option (b) can be formed by

A
joining the pieces given in question figure, as joining the pieces given in question figure, as
shown below.

M
shown below.

H
TT
4. (b) Figure given in option (b) can be formed by
joining the pieces given in question figure, as A
11. (c) Figure given in option (c) can be formed by
W
shown below. joining the pieces given in question figure, as
shown below.
SH
A
@

5. (b) Figure given in option (b) can be formed by


joining the pieces given in question figure, as 12. (c) The problem figure can be formed by joining the
shown below. pieces given in option (c), as shown below.

6. (c) Figure given in option (c) can be formed by 13. (c) The problem figure can be formed by joining the
joining the pieces given in question figure, as
pieces given in option (c), as shown below.
shown below.

7. (b) Figure given in option (b) can be formed by 14. (d) The problem figure can be formed by joining the
joining the pieces given in question figure, as pieces given in option (d), as shown below.
shown below.
Chapter 08 FORMATION OF FIGURES 397

15. (c) The problem figure can be formed by joining the 21. (b) Figure given in option (b) can be formed by
pieces given in option (c), as shown below. joining the pieces given in question figure,
as shown below

16. (c) The problem figure can be formed by joining the


pieces given in option (c), as shown below. 22. (b) Figure given in option (b) can be formed by
joining the pieces given in question figure,
as shown below

17. (c) The problem figure can be formed by joining the


23. (a) Figure given in option (a) can be formed by
pieces given in option (c), as shown below
joining the given problem figure, as shown
below

A
M
18. (c) The problem figure can be formed by joining the 24. (c) The only figure with right angle is figure (C). If it

H
pieces given in option (c) as shown below is fitted with figure (B), the resultant figure will

TT
look like below figure (X).

A B C
W
19. (b) The problem figure can be formed by joining the (X)
SH

pieces given in option (b) as shown below Now, when the figure (D) is fitted in figure (X),
the resultant figure will be a square as shown in
figure (Y).
A

D
@

B C
20. (d) The problem figure can be formed by joining the
pieces given in option (d) as shown below (Y)
Hence, combination of figures as presented by
alternative (c) is the correct combination of
figures to form a square.
398 How to Crack Test of Reasoning NON-VERBAL

09

Counting of
Figures

A
M
Counting of figures is simply the realisation of Number of horizontal lines
different geometrical figures from a complex one. = AC, LD, KE, JF, IG = 5

H
The figures which are asked for counting can be Number of vertical lines = AI, BH, CG = 3

TT
straight lines, triangles, square, rectangles, Number of slant lines = AG, IC = 2
circles etc. To find the accurate number of
figures, firstly, a candidate needs to find the A
\ Total number of straight lines
W
= 5 + 3 + 2 = 10
required figures formed by individual section of
SH

the given figure, then the figures formed by Illustration 2. The number of squares in the
combination of two figures and so on. figure is SSC (CPO) 2016
A

Examples given below, will give you a better idea


@

about the types of questions generally asked in


various competitive examinations.
Illustration 1. How many straight lines are there in
the following figure UP B.Ed. 2011

(a) 14 (b) 10
(c) 8 (d) 12
Solution (b) Naming the figure,
M N J

(a) 5 (b) 10 A I B
(c) 9 (d) 8 H
O P
Solution (b) Naming the figure, E G
A B C D F C
L D L Q K
M Clearly, there are 10 squares in the given figure
K E
namely, OMNE, NJPE, PKQE, OLQE, AIEH,
J F
IBGE, GCFE, FDHE, ABCD, MJKL.
I H G
Chapter 09 COUNTING OF FIGURES 399

Illustration 3. How many rectangles are there in Illustration 5. Find the total number of circles in
the question figure? SSC (10+2) 2014 the figure given below. SSC (10+2) 2012

(a) 6 (b) 7
(c) 8 (d) 9 (a) 10 (b) 11
(c) 12 (d) 13
Solution (d) Naming the figure,
Solution (d) Counting the circles,
A E G B
M H 2 9
F K L 6
3 10
1 7 13
4 11
8
D C

A
N 5 12

M
Clearly, there are 9 rectangles in the given figure Clearly, there are 13 circles in the given figure.

H
namely, AEFM, EGHF, AGHM, GBLK, KLCN,

TT
GBCN, MHND, ABCD and AGND. Illustration 6. Consider the figure given below.
Illustration 4. How many triangles are there in
the given figure? RRB (ASM) 2009, RPSC 2013 A
W
SH
A

Find the least number of colours to be used to


@

(a) 28 (b) 24
(c) 25 (d) 26
fill the figure, if not two adjacent areas can
have the same colour.
Solution (a) Naming the figure,
(a) 3 (b) 4
E F G H (c) 5 (d) 6
Solution (a) According to the question,
O P Q

A B C D III I
II
Clearly, there are 28 triangles in the given figure III II I

namely, EOF, AOE, AOB, BOF, ABF, BEF, ABE, I


I
III
III
AEF, BPF, FPG, CPG, BPC, BFG, BCG, CFG, BCF, I III I

GQC, CDQ, DQH, GQH, GDC, GDH, GHC, III I

CDH, AFC, BGD, EBG and FCH. So, there are only 3 colours needed to fill the figure.
400 How to Crack Test of Reasoning NON-VERBAL

Let us Practice
1. How many straight lines in the given figure 6. Find the number of triangles in the figure.
CISF 2018 SSC (CGL) 2017

(a) 11 (b) 14 (c) 16 (d) 17


(a) 8 (b) 9
2. How many triangles are there in the figures (c) 11 (d) 13
given below? CGPSC 2017
Directions (Q. Nos. 7-8) Study the figure
given below to answer the questions that follow.
RRB (TC/CC) 2005

A
M
H
TT
(a) 5 (b) 6 (c) 8 (d) 10 7. How many squares are there in this figure?
(e) None of these
(a) 2 (b) 4
3. How many triangles are there in there figure? (c) 6 A (d) 8
W
SSC (CPO) 2016 8. Count the number of triangles in the above
SH

figure.
(a) 22 (b) 23
(c) 28 (d) 15
A
@

9. How many triangles are there in the given


figure? WBPSC 2018
(a) 12 (b) 16 (c) 9 (d) 8

4. How many triangles are there in the following


figure? SSC (Steno) 2016

(a) 11 (b) 12
(c) 13 (d) 9

(a) 16 (b) 18 (c) 20 (d) 22 10. How many circles are there in this figure?
SSC (CGL) 2015
5. How many triangles are there in the given
figure? MPPSC 2014

(a) 13 (b) 16
(a) 10 (b) 12 (c) 14 (d) 16 (c) 17 (d) 22
Chapter 09 COUNTING OF FIGURES 401

11. Find out the number of circles in the given 16. How many triangles are there in the given
figure? figure? SSC (CPO) 2017

(a) 13 (b) 14 (c) 12 (d) 15


17. How many squares are there in the following
(a) 14 (b) 16 (c) 17 (d) 18 figure?
12. How many parallelograms are there in the
following figure? SSC (CPO) 2008

(a) 16 (b) 17 (c) 26 (d) 30

(a) 3 (b) 4 (c) 5 (d) 6


18. How many triangles are there in the following
figure? CGPSC 2013

A
13. Find the minimum number of straight lines

M
required to make the given figure.

H
IB (ACIO) 2017

TT
A
W
(a) 16 (b) 17
SH

(c) 18 (d) 19 (a) 20 (b) 22 (c) 16 (d) 24 (e) 25

14. How many squares are there in the given 19. How many squares are there in this figure?
A

figure? MPPSC 2018, SSC (CPO) 2011 SSC (10 + 2) 2012


@

(a) 24 (b) 23 (c) 27 (d) 26


(a) 10 (b) 13
(c) 12 (d) 14
20. Find out the number of squares in the given
pattern. SSC (Multitasking) 2013
15. Consider the following figure and answer the
item that follows

(a) 26 (b) 30 (c) 35 (d) 38

21. How many total squares are there in the


following figure?

What is the total number of triangles in the


above grid? UPSC (CSAT) 2012, UPSSSC 2016

(a) 27 (b) 26
(c) 23 (d) 22 (a) 12 (b) 13 (c) 16 (d) 17
402 How to Crack Test of Reasoning NON-VERBAL

22. Find out the number of triangles in the given 25. Find the number of triangles in the given
pattern. SSC (Multitasking) 2013 figure. IB (ACIO) 2017

(a) 23 (b) 26 (c) 27 (d) 29


(a) 18 (b) 20
23. The number of triangles in this figure is (c) 24 (d) 27

26. Find the number of triangles in the diagram


given below. UPPSC (RO) 2014

(a) 20 (b) 22 (c) 26 (d) 28

24. How many triangles are there in the given


figure? BSSC (CGL) 2015

(a) 32

A
(b) 31

M
(c) 30

H
(a) 19 or more (b) 10 (c) 16 (d) 18 (d) 29

TT
Answers A
W
SH

1. (b) 2. (c) 3. (b) 4. (b) 5. (b) 6. (d) 7. (b) 8. (d) 9. (b) 10. (c)
11. (c) 12. (d) 13. (b) 14. (d) 15. (c) 16. (d) 17. (d) 18. (b) 19. (c) 20. (c)
A

21. (d) 22. (c) 23. (d) 24. (a) 25. (c) 26. (a)
@

Answer with Explanations


1. (b) Naming the figure, 2. (c) Naming the figure,
A B C D E A

L B
K C
L F
J D

I E
G H F
K J I H
Horizontal lines = AE, LF, KG = 3 G
Vertical lines = AK, BJ, CI, DH, EG = 5 There are 8 triangles in the given figure
Slant lines = CL, CF, IL, IF, AG, EK = 6 namely, ABL, BCD, DEF, FGH, HIJ, JKL, AIE
\Total number of straight lines = 3 + 5 + 6 = 14 and KCG
Chapter 09 COUNTING OF FIGURES 403

3. (b) Naming the figure, Solutions (Q. Nos. 7-8) Naming the figure,
A F P O E

J G S N
E F
Q J K R
O A D
D I K B H I B C L M
7. (b) There are 4 squares in the given figure
H L G namely, GHIQ, FPJQ, OERK and RNML.
C 8. (d) There are 15 triangles in the given figure
There are 16 triangles in the given figure namely, AGH, GFQ, FPS, SJK, OEK, EKR,
namely, AFE, FBG, GCH, HDE, EJI, JFK, KLG, ERN, NMD, FIA, FQK, FKO, GFK, FKE, ELD
LHI, IJO, JKO, KLO, LIO, IJK, ILK, JIL, JKL and EKN.
9. (b) Naming the figure,
4. (b) Naming the figure,
A
A B
F G
J D E
G

A
I H C

M
B
E D C F

H
There are 18 triangles in the given figure There are 12 triangles in the given figure

TT
namely, ABD, AED, AEB, DBE, AEJ, ABJ, BJD, namely, DGC, CGF, GEF, DCF, CFE, DFA,
EJD, FGJ, GHJ,JHI,JIF, BCD, FGH, FIH, FIG, ACF, AFB, AEB, EBF, CEB and ABC.
HIG, BEC.
A
10. (c) Counting the circles,
W
5. (b) Naming the figure,
2
E 7,8
SH

13 14
1
17
D
A

O
3
@

5,6 9,10
A B C
There are 12 triangles in the given figure 15 16
namely, AOB, BOC, COD, DOE, EOA, AOC, 11,12
4
COE, ABE, CBE, EAD, DAC and ACE
There are 17 circles in the given figure.
6. (d) Naming the figure,
11. (c) Counting the circles,
A
1 circle
B 3 circles
H 3 circles
C
F 1 circle
G D 1 circle
1 circle

E 3 circles
3 circles
There are 13 triangles in the given figure 1 circle
namely, AFB, FBD, DEF, BCD, DEG, FHG,
AHF, FGE, AFE, AEC, AHB, AFG and FHE. There are 12 + 5 = 17 circles in the given figure.
404 How to Crack Test of Reasoning NON-VERBAL

12. (d) Naming the figure, 16. (d) Naming the figure,
B D F A

D
A C E G
There are 6 parallelograms in the given figure B G C
namely, ABDC, ABFE, BCED, BCGF, CDFE
There are 15 triangles in the given figure
and DEGF.
namely, BGD, BGE, BGA, CGD, CGE, CGA,
13. (b) Naming the figure, BDC, BEC, BAC, ABD, ABE, ACD, ACE, BED,
B CED.
A C
H I J D E 17. (d) Naming the figure,
E F G B
A
K L M
Q R S
P H
G F T U V
O I

A
There are 17 straight lines in the given figure W X Y
N J
namely, AC, HE, GF, KM, HG, IK, BL, DM, EF,

M
BI, BD, IG, KG, DF, MF, AI and CD. D C
M L K

H
14. (d) Naming the figure,
There are 16 squares which are equal parts of

TT
A P big square ABCD and the other squares are

B
O
N A
AFUO, FUIB, OULD, ULCI, PRXN, RXJH
QSYW, TVKM, TVGE, AGYN, EBJW, PSKD and
W
QHCM.
Q M L
SH

C Hence, there are 30 squares in this figure.


R S K J 18. (b) Naming the figure,
D
A

B D
@

E C
F G H I
A E
There are 14 squares in the given figure
namely, ABOP, BCQO, OQMN, CDRQ, QRSM, K L
MSKL, DEFR, RFGS, SGHK, KHIJ, BDSN,
CEGM, DFSQ and QFHL. J F
H
15. (c) Naming the figure,
I G
A
The number of triangles in the upper most and
lower most part of the figure
D E
= 2 + 2 = 4, i.e. ABC, CDE, JHI and HFG.
G
F H The number of small triangles in the middle
J K part of the figure = 8, i.e. AKJ, AKC, CKH,
I L HKJ, CLH CLE, ELF and HLF.
The number of medium triangles in the middle
B M N O C part of the figure = 8,
i.e. AJH, ACH, ACJ, JHC, CFH, CFE, CEH and
There are 23 triangles in the given figure HFE.
namely, ADE, DGE, DGF, EGH, FGJ, GJK, The number of big triangles in the middle part
GKH, HKL, JMN, FJI, JNK, HNC, HNF, NFB, of the figure = 2, i.e. JCF and AHE
FHA, EMC, DOB, GMO, DKI, EJL, AIL, BOD,
ABC. Total number of triangles = 4 + 8 + 8 + 2 = 22
Chapter 09 COUNTING OF FIGURES 405

19. (c) Naming the figure, 21. (d) Naming the figure,
A B C D E E
A B

F I M J
X
A¢ B¢
Q U R
P W Y X N
C¢ D¢ V¢ H F

T V S
Y Z E¢ F¢ G¢
W G L K
S¢ T¢ O

V Q¢ H¢ H
P¢ W¢ U¢ D G C

U I There are 17 squares in the given figure,
N’ J¢
O¢ M¢ L¢ K¢ I¢ namely, ABCD, QRST, IJKL, QUYW, WYVT,
T J
URXY, YXSV, MNOP, EFGH, IMYP, MJNY,
YNKO, PYOL, AEYH, EBFY, HYGD and
S R Q P O N M L K YFCG.

A
There are 27 squares in the given figure namely, 22. (c) Number of triangles made of one unit

M
ABA¢X, DEFB¢, C¢D¢T¢S¢, D¢V¢E¢T¢, WYP¢V, S¢T¢W¢Q¢, = 1 + 3 + 5 + 7 = 16

H
T¢E¢U¢W¢, G¢GHH¢, TO¢SR, M¢L¢OP, L¢K¢NO, I¢JKL, Number of triangles made of 4 units

TT
ACT¢W, CEGT¢, WT¢OS, T¢GKO, AEKS, C¢V¢U¢Q¢, = 1+ 2 + 3 + 1 = 7
WZN¢U¢, ZT¢R¢N¢, T¢F¢J¢R¢, F¢GIJ¢, UN¢QS, N¢R¢OQ, Number of triangles made of 9 units
R¢J¢MO, J¢IKM and ZF¢MQ.
A = 1+ 2 = 3
W
20. (c) Namely the figure, Number of triangles made of 16 units = 1
Total number of triangles
SH

A B C D E
= 16 + 7 + 3 + 1 = 27
Q R S
A

P F 23. (d) Namely the figure,


@

T U V A B C
O G
I J K
W X Y
N H O P Q
D R S T U E
M
L K J I V W X
L N
There are 16 squares in the given figure namely, M
ABQP, BCRQ, CDSR, DEFS, PQTO, QRUT, H
RSVU, SFGV, OTWN, TUXW, VYXU, VGHY, F G
NWLM, WXKL, XYJK and YHIJ The simple outer triangles are AID, AIB, BIK,
Squares formed with four triangles QCSU, SGYU, BCK, CKE, KEN, ENH, NGH, NGL, LFG,
WUYK and WOQU = 4 DLF and DLI, i.e. 12 triangles. The simple
Squares formed with four small squares are middle triangles are IJR, ORV, RLM, VXM,
ACUO, BDVT, CEGU, PRXN, QSYW, RFHX, UMN, QUX, KJU and JOQ, i.e. 8 triangles.
OUKM, TVJL, UGIK and OCGK is formed from The simple inner most triangles are OSP,
small = 10 PQT, SVW and TWX, i.e. 4 triangles. The
Squares formed with nine small squares are bisected triangles are ABD, BCE, DFG and
ADYN, BEHW, PSJM, QFIL = 4 EGH, i.e. 4 triangles.
Largest square = AEIM = 1 Total number of triangles
\Total squares = 16 + 4 + 10 + 4 + 1 = 35 = 12 + 8 + 4 + 4 = 28
406 How to Crack Test of Reasoning NON-VERBAL

24. (a) Naming the figure, 26. (a) In order to count the triangles, we first
A B C D name the vertices
A
H
I
G E
J
F
There are 20 triangles in the given figure namely, ABG, D E
O N M
BHJ, GHJ, BCI, CID, BIJ, DIE, JIE, GJF, FJE, BJG,
G L
BJE, DEJ, BDE, DBJ, GFE, DFE, BDI, GBE and FIE.
H K
25. (c) Naming the figure.
B I J
A
L C
I
K D B F C
J N O Number of triangles in region AED =
M
H P AOD, OAN, NAM, MAE, DAN, OAM,
G Q

A
E NAE, DAM,OAE, DAE = 10
F

M
There are 24 triangles in the given figure namely, Similarly, DBDF and DCEF will have 10
and 10 triangles. DDEF and DABC will

H
ABK, AKH, ABH, ILK, IJK, ILJ, BLC, HJG, ICM, IGM,
IGC, CMP, GMP, GCP, CDO, GFQ, ONP, QNP, QOP, count 2, hence total number of triangles

TT
DNE, FNE, DFE, CIP and GIP = 10 + 10 + 10 + 2 = 32

A
W
SH
A
@
Chapter 10 EMBEDDED FIGURES
407

10
Embedded
Figures
A figure is said to be hidden or embedded in Directions (Illustrations 1-4) In the following

A
another figure when the one figure is questions, figure (A) is embedded in any one of the

M
contained in the other. Let us consider four alternative figures (a), (b), (c) and (d). Find

H
following two figures. the alternative which contains figure (A) as its

TT
part.

Illustration 1.
A
W
(X) (Y)
SH

By careful observation of fig (Y), we find that fig


(A) (a) (b) (c) (d)
(X) is embedded in it, as shown in the figure
A

given below by solid line. SSC (CPO) 2016


@

Solution (a) By careful observation, we find that the


figure (A) is embedded in figure (a) as shown
below.

Types of Questions
Illustration 2.
There are two types of questions, which are
generally asked in various competitive exams.

Type 1 (A) (a) (b) (c) (d)


When Question Figure Embedded Solution (d) By careful observation, we find that the
in Answer Figure figure (A) is embedded in figure (d) as shown
below.
In this type, the question figure is embedded
in one of the answer figures. The candidate is
required to find out the answer figure in which
the given question figure is embedded.
How to Crack Test of Reasoning NON-VERBAL
408

Illustration 3. Directions (Illustrations 5-6) In the following


questions, a question figure is given with four
answer figures (a), (b), (c) and (d). Find out that
answer figure which is embedded in the question
figure.
(A) (a) (b) (c) (d)
Illustration 5.
Solution (d) By careful observation, we find that the
figure (A) is embedded in figure (d) as shown
below.

(A) (a) (b) (c) (d)

Solution (a) By carefully observation, we find that the


Illustration 4. figure (a) is embedded in figure (A) as shown
below.

(A) (a) (b) (c) (d)


Illustration 6.

A
[SSC (CPO) 2015]

M
Solution (b) By carefully observation, we find that the
figure (A) is embedded in figure (b) as shown

H
below.

TT
(A) (a) (b) (c) (d)

A
Solution (b) By carefully observation, we find that the
W
figure (b) is embedded in figure (A) as shown below.
SH

Type 2
When Answer Figure Embedded
A

Alert! There may be some questions in which the


in Question Figure
@

question figure is not directly embedded in any of


In this type, one of the answer figures is the answer figures. In these type of questions,
change the orientation of question figure to find the
embedded in question figure and that
correct answer figure.
particular answer figure has to be found out.

Let us Practice
Directions (Q. Nos. 1-29) In each of the following problem, choose the alternative figure in which the
problem figure is embedded.
1. Problem Answer 2. Problem Answer
Figure Figures Figure Figures

(a) (b) (c) (d) (a) (b) (c) (d)


SSC (CGL) 2016
Chapter 10 EMBEDDED FIGURES
409

3. Problem Answer 10. Problem Answer


Figure Figures Figure Figures

+ + + +
+
(a) (b) (c) (d) (a) (b) (c) (d)
4. Problem Answer SSC (CGL) 2017
Figure Figures 11. Problem Answer
Figure Figures

(a) (b) (c) (d)


SSC (CPO) 2017
(a) (b) (c) (d)
5. Problem Answer
SSC (10+2) 2011
Figure Figures
12. Problem Answer
Figure Figures

A
M
(a) (b) (c) (d)

H
SSC (CGL) 2015

TT
6. Problem Answer (a) (b) (c) (d)
Figure Figures SSC (Steno) 2012
13. Problem A Answer
W
Figure Figures
SH

(a) (b) (c) (d)


A

7. Problem Answer
@

(a) (b) (c) (d)


Figure Figures 14. Problem Answer
Figure Figures

(a) (b) (c) (d)


SSC (CGL) 2014
(a) (b) (c) (d)
8. Problem Answer
15. Problem Answer
Figure Figures
Figure Figures

(a) (b) (c) (d)


SSC (CPO) 2014 (a) (b) (c) (d)

9. Problem Answer 16. Problem Answer


Figure Figures Figure Figures

(a) (b) (c) (d) (a) (b) (c) (d)


How to Crack Test of Reasoning NON-VERBAL
410

17. Problem Answer 24. Problem Answer


Figure Figures Figure Figures

(a) (b) (c) (d)


(a) (b) (c) (d)
SSC (CGL) 2012
25. Problem Answer
18. Problem Answer Figure Figures
Figure Figures

(a) (b) (c) (d)


(a) (b) (c) (d)
19. Problem Answer 26. Problem Answer
Figure Figures
Figure Figures

A
(a) (b) (c) (d)

M
(a) (b) (c) (d) 27. Problem Answer

H
SSC (CPO) 2010 Figure Figures

TT
20. Problem Answer
Figure Figures
A
W
(a) (b) (c) (d)
SH

28. Problem Answer


(a) (b) (c) (d) Figure Figures
A

SSC (CGL) 2010


21. Problem Answer
@

Figure Figures
(a) (b) (c) (d)
SSC (Steno) 2016
29. Problem Answer
(a) (b) (c) (d) Figure Figures
SSC (CPO) 2016
22. Problem Answer
Figure Figures
(a) (b) (c) (d)
Directions (Q. Nos. 30-46) In each of the
following question, choose the answer which is
(a) (b) (c) (d) embedded in the problem figure.
23. Problem Answer 30. Problem Answer
Figure Figures Figure Figures

(a) (b) (c) (d) (a) (b) (c) (d)


SSC (Multitasking) 2012 SSC (CGL) 2015
Chapter 10 EMBEDDED FIGURES
411

31. Problem Answer 38. Problem Answer


Figure Figures Figure Figures

(a) (b) (c) (d) (a) (b) (c) (d)


SSC (10+2) 2014 39. Problem Answer
32. Problem Answer Figure Figures
Figure Figures

(a) (b) (c) (d) (a) (b) (c) (d)

33. Problem Answer


40. Problem Answer
Figure Figures
Figure Figures

A
M
(a) (b) (c) (d)

H
(a) (b) (c) (d) 41. Problem Answer

TT
34. Problem Answer Figure Figures
Figure Figures
A
W
SH

(a) (b) (c) (d)


(a) (b) (c) (d)
42. Problem Answer
A

35. Problem Answer Figure Figures


@

Figure Figures

(a) (b) (c) (d)


(a) (b) (c) (d) 43. Problem Answer
Figure Figures
36. Problem Answer
Figure Figures

(a) (b) (c) (d)


SSC (10+2) 2008
(a) (b) (c) (d)
37. Problem Answer 44. Problem Answer
Figure Figures
Figure Figures

(a) (b) (c) (d)


(a) (b) (c) (d)
SSC (CGL) 2010
SSC (Steno) 2009
How to Crack Test of Reasoning NON-VERBAL
412

45. Problem Answer 46. Problem Answer


Figure Figures Figure Figures

(a) (b) (c) (d) (a) (b) (c) (d)


SSC (CPO) 2008 SC (CGL) 2008

Answers
1. (b) 2. (b) 3. (d) 4. (a) 5. (a) 6. (c) 7. (d) 8. (b) 9. (c) 10. (d)
11. (c) 12. (b) 13. (b) 14. (a) 15. (b) 16. (a) 17. (b) 18. (d) 19. (d) 20. (b)
21. (d) 22. (a) 23. (a) 24. (b) 25. (b) 26. (c) 27. (d) 28. (d) 29. (b) 30. (b)
31. (c) 32. (b) 33. (d) 34. (b) 35. (a) 36. (c) 37. (c) 38. (d) 39. (d) 40. (d)

A
41. (c) 42. (a) 43. (b) 44. (c) 45. (a) 46. (a)

M
H
Answer with Explanations
TT
1. (b) We find that the problem figure is embedded in A
5. (a) We find that the the problem figure is embedded
W
figure (b) as shown in figure (a) as shown
SH
A
@

6. (c) We find that the problem figure is embedded in


2. (b) We find that the problem figure is embedded in figure (c) as shown
figure (b) as shown

7. (d) We find that the problem figure is embedded in


3. (d) We find that the problem figure is embedded in figure (d) as shown
figure (d) as shown

+
8. (b) We find that the problem figure is embedded
4. (a) We find that the problem figure is embedded in in figure (b) as shown
figure (a) as shown
Chapter 10 EMBEDDED FIGURES
413

9. (c) We find that the problem figure is embedded in 18. (d) We find that the problem figure is embedded in
figure (c) as shown figure (d) as shown

19. (d) We find that the problem figure is embedded in


10. (d) We find that the problem figure is embedded in
figure (d) as shown
figure (d) as shown

20. (b) We find that the problem figure is embedded in


11. (c) We find that the problem figure is embedded in
figure (b) as shown
figure (c) as shown

21. (d) We find that the problem figure is embedded in


12. (b) We find that the problem figure is embedded in
figure (d) as shown

A
figure (b) as shown

M
H
TT
22. (a) We find that the problem figure is embedded in
13. (b) We find that the problem figure is embedded in figure (a) as shown
figure (b) as shown
A
W
SH

14. (a) We find that the problem figure is embedded in 23. (a) We find that the problem figure is embedded in
A

figure (a) as shown


figure (a) as shown
@

24. (b) We find that the problem figure is embedded in


15. (b) We find that the problem figure is embedded in figure (b) as shown
figure (b) as shown

25. (b) We find that the problem figure is embedded in


16. (a) We find that the problem figure is embedded in figure (b) as shown
figure (a) as shown

26. (c) We find that the problem figure is embedded in


17. (b) We find that the problem figure is embedded in figure (c) as shown
figure (b) as shown
How to Crack Test of Reasoning NON-VERBAL
414

27. (d) We find that the problem figure is embedded 35. (a) We find that the answer figure (a) is embedded
in figure (d) as shown in problem figure.
36. (c) We find that the answer figure (c) is embedded
in problem figure.
37. (c) We find that the answer figure (c) is embedded
28. (d) We find that the problem figure is embedded in in problem figure.
figure (d) as shown
38. (d) We find that the answer figure (d) is embedded
in problem figure.
39. (d) We find that the answer figure (d) is embedded
in problem figure.
29. (b) We find that the problem figure is embedded in
figure (b) as shown 40. (d) We find that the answer figure (d) is embedded
in problem figure.
41. (c) We find that the answer figure (c) is embedded
in problem figure.

30. (b) We find that the answer figure (b) is embedded 42. (a) We find that the answer figure (a) is embedded
in problem figure.

A
in problem figure.

M
31. (c) We find that the answer figure (c) is embedded 43. (b) We find that the answer figure (b) is embedded
in problem figure.

H
in problem figure.
44. (c) We find that the answer figure (c) is embedded

TT
32. (b) We find that the answer figure (b) is embedded
in problem figure. in problem figure.

33. (d) We find that the answer figure (d) is embedded A


45. (a) We find that the answer figure (a) is embedded
in problem figure.
W
in problem figure.
46. (a) We find that the answer figure (a) is embedded
SH

34. (b) We find that the answer figure (b) is embedded


in problem figure. in problem figure.
A
@
Chapter 11 FIGURE COMPLETION
415

11
Figure Completion
Figure completion is a process to find out the Illustration 2. Select a figure from the four
missing part of an incomplete figure to alternatives, which when placed in the missing
complete it. portion (?) of the original figure, as shown by
In this chapter, we deal with the questions in figure (X), would complete the pattern.
SSC (CGL) 2014

A
which a part, generally 1/4th part of the
Problem Figure

M
figure is missing, and the candidate is asked
to find the missing part from the given option

H
figures.

TT
To find the missing part, you are required to ?
understand the general structure of the
figure, then choose the correct alternative.
(X)
A
W
Answer Figures
The examples given below, which will give
SH

you a better idea about the types of questions


generally asked in various competitive
A

examinations.
@

(a) (b) (c) (d)


Illustration 1. In the following question,
complete the missing segment by selecting the Solution (d) If figure shown in option (d), is placed in
appropriate figure from the given alternatives, the missing portion of the original figure, it
completes the original figure as shown below
(a), (b), (c) and (d).
Problem Figure Answer Figures

?
(a) (b) (c) (d) Illustration 3. Select a figure from the four
SSC (CPO) 2017 alternatives, which when placed in the missing
Solution (b) If figure shown in option (b), is placed portion (?) of the original figure, as shown by
in the place of missing portion of the original figure (X), would complete the pattern.
figure, then it is completed as shown below Problem Figure

(X)
How to Crack Test of Reasoning NON-VERBAL
416

Answer Figures Problem Figure

?
(a) (b) (c) (d)
Solution (b) It is clear that figure given in option (b) (X)
completes the original figure, as shown below Answer Figures

(a) (b) (c) (d)

This array of lines is rotating 45° in clockwise Solution (b) Clearly, the figure given in option (b)
direction. completes the original figure as shown below

Illustration 4. Select a figure from the four

A
alternatives, which when placed in the missing

M
portion (?) of the original figure, as shown by

H
figure (X), would complete the pattern.

TT
A
W
SH

Let us Practice
A
@

Directions (Q. Nos. 1-42) In each of the following problem, select a figure from the given four
alternatives, which when placed in the blank space of problem figure would complete the pattern.
1. Problem Figure 2. Problem Figure

? ?
SSC (Steno) 2011
Answer Figures Answer Figures

(a) (b) (c) (d) (a) (b) (c) (d)


Chapter 11 FIGURE COMPLETION
417

3. Problem Figure 7. Problem Figure

?
?
Answer Figures
Answer Figures

(a) (b) (c) (d) (a) (b) (c) (d)


4. Problem Figure 8. Problem Figure

A
?

M
AFCAT 2018 AFCAT 2018

H
Answer Figures Answer Figures

TT
A
W
(a) (b) (c) (d) (a) (b) (c) (d)
SH

5. Problem Figure 9. Problem Figure


A

?
@

?
SSC (CPO) 2013
SSC (CGL) 2015
Answer Figures Answer Figures

(a) (b) (c) (d) (a) (b) (c) (d)


6. Problem Figure 10. Problem Figure

? ?
Answer Figures Answer Figures

(a) (b) (c) (d) (a) (b) (c) (d)


How to Crack Test of Reasoning NON-VERBAL
418

11. Problem Figure 15. Problem Figure

SSC (CGL) 2014 SSC (Steno) 2016


Answer Figures Answer Figures

(a) (b) (c) (d) (a) (b) (c) (d)


12. Problem Figure 16. Problem Figure

A
SSC (10+2) 2014

M
SSC CGL (Tier I) 2017
Answer Figures

H
Answer Figures

TT
(a) (b) (c) (d) A
W
(a) (b) (c) (d)
13. Problem Figure 17. Problem Figure
SH

?
A
@

?
SSC (CGL) 2016
Answer Figures Answer Figures

(a) (b) (c) (d) (a) (b) (c) (d)


14. Problem Figure 18. Problem Figure

?
SSC CGL (Tier I) 2017
Answer Figures Answer Figures

(a) (b) (c) (d) (a) (b) (c) (d)


Chapter 11 FIGURE COMPLETION
419

19. Problem Figure 23. Problem Figure

SSC (Multitasking) 2013 BSSC (CGL) 2015

Answer Figures Answer Figures

(a) (b) (c) (d) (a) (b) (c) (d)

20. Problem Figure 24. Problem Figure

A
M
SSC (Steno) 2011 SSC (10+2) 2014

H
Answer Figures Answer Figures

TT
A
W
(a) (b) (c) (d) (a) (b) (c) (d)
21. Problem Figure
SH

25. Problem Figure


A
@

?
SSC (Multitasking) 2013 ?
Answer Figures RRB (ASM) 2009
Answer Figures

(a) (b) (c) (d)


22. Problem Figure (a) (b) (c) (d)
26. Problem Figure
+
+

? ?
+

SSC (10+2) 2011


BSSC (10+2) 2015
Answer Figures
Answer Figures
+

(a) (b) (c) (d)


(a) (b) (c) (d)
How to Crack Test of Reasoning NON-VERBAL
420

27. Problem Figure 31. Problem Figure

SSC (Multitasking) 2014


Answer Figures
Answer Figures

(a) (b) (c) (d)


(a) (b) (c) (d)
28. Problem Figure
32. Problem Figure

? ?

A
M
Answer Figures Answer Figures

H
TT
(a) (b) (c) (d) (a)
A
(b) (c) (d)
W
29. Problem Figure 33. Problem Figure
Ý
SH

rr
Ý
A

@ ?
@

r b ?
a
SSC (10+2) 2013 SSC (CGL) 2011
Answer Figures Answer Figures
r r a b
@ b @ a b r r @
a b @ a
(a) (b) (c) (d)
(a) (b) (c) (d)
30. Problem Figure
34. Problem Figure

?
SSC (CGL) 2013
?
Answer Figures Answer Figures

(a) (b) (c) (d) (a) (b) (c) (d)


Chapter 11 FIGURE COMPLETION
421

35. Problem Figure 39. Problem Figure

SSC (CPO) 2015


Answer Figures Answer Figures

(a) (b) (c) (d) (a) (b) (c) (d)


36. Problem Figure
40. Problem Figure

A
?

M
Answer Figures
Answer Figures

H
TT
(a) (b) (c) (d) (a) (b) A (c) (d)
W
37. Problem Figure 41. Problem Figure
SH

?
A
@

?
SSC (Steno) 2012 SSC (10+2) 2003
Answer Figures Answer Figures

(a) (b) (c) (d) (a) (b) (c) (d)


38. Problem Figure
42. Problem Figure
?

?
SSC (CGL) 2015
Answer Figures
Answer Figures

(a) (b) (c) (d)


(a) (b) (c) (d)
How to Crack Test of Reasoning NON-VERBAL

Answers
1. (c) 2. (c) 3. (d) 4. (c) 5. (c) 6. (a) 7. (b) 8. (d) 9. (c) 10. (b)
11. (c) 12. (b) 13. (d) 14. (d) 15. (b) 16. (c) 17. (c) 18. (a) 19. (d) 20. (c)
21. (d) 22. (d) 23. (c) 24. (a) 25. (c) 26. (d) 27. (d) 28. (c) 29. (c) 30. (c)
31. (d) 32. (c) 33. (b) 34. (d) 35. (c) 36. (d) 37. (d) 38. (b) 39. (a) 40. (d)
41. (a) 42. (d)

Answer with Explanations


1. (c) If figure shown in option (c), is placed in the 5. (c) If figure shown in option (c), is placed in the
place of missing portion of the original figure, place of missing portion of the original figure,

A
then it is completed as shown below then it is completed as shown below

M
H
TT
2. (c) If figure shown in option (c), is placed in the A
6. (a) If figure shown in option (a), is placed in the
W
place of missing portion of the original figure,
place of missing portion of the original figure,
then it is completed as shown below
then it is completed as shown below
SH
A
@

3. (d) If figure shown in option (d), is placed in the 7. (b) If figure shown in option (b), is placed in the
place of missing portion of the original figure, place of missing portion of the original figure,
then it is completed as shown below then it is completed as shown below

4. (c) If figure shown in option (c), is placed in the 8. (d) If figure shown in option (d), is placed in the
place of missing portion of the original figure, place of missing portion of the original figure,
then it is completed as shown below then it is completed as shown below
Chapter 11 FIGURE COMPLETION
423

9. (c) If figure shown in option (c), is placed in the 15. (b) If figure shown in option (b), is placed in the
place of missing portion of the original figure, place of missing portion of the original figure,
then it is completed as shown below then it is completed as shown below

10. (b) If figure shown in option (b), is placed in the 16. (c) If figure shown in option (c), is placed in the
place of missing portion of the original figure, place of missing portion of the original figure,
then it is completed as shown below then it is completed as shown below

11. (c) If figure shown in option (c), is placed in the 17. (c) If figure shown in option (c), is placed in the
place of missing portion of the original figure, place of missing portion of the original figure,

A
then it is completed as shown below then it is completed as shown below

M
H
TT
A
W
12. (b) If figure shown in option (b), is placed in the 18. (a) If figure shown in option (a), is placed in the
place of missing portion of the original figure, place of missing portion of the original figure,
SH

then it is completed as shown below then it is completed as shown below


A
@

13. (d) If figure shown in option (d), is placed in the 19. (d) If figure shown in option (d), is placed in the
place of missing portion of the original figure, place of missing portion of the original figure,
then it is completed as shown below then it is completed as shown below

14. (d) If figure shown in option (d), is placed in the 20. (c) If figure shown in option (c), is placed in the
place of missing portion of the original figure, place of missing portion of the original figure,
then it is completed as shown below then it is completed as shown below
How to Crack Test of Reasoning NON-VERBAL
424

21. (d) If figure shown in option (d), is placed in the 27. (d) If figure shown in option (d), is placed in the
place of missing portion of the original figure, place of missing portion of the original figure,
then it is completed as shown below then it is completed as shown below

28. (c) If figure shown in option (c), is placed in the


22. (d) If figure shown in option (d), is placed in the place of missing portion of the original figure,
place of missing portion of the original figure, then it is completed as shown below
then it is completed as shown below

29. (c) If figure shown in option (c), is placed in the


place of missing portion of the original figure,
23. (c) If figure shown in option (c), is placed in the then it is completed as shown below
place of missing portion of the original figure, Ý

A
then it is completed as shown below
rr

M
Ý
@ a
r b b r

H
a @

TT
30. (c) If figure shown in option (c), is placed in the
place of missing portion of the original figure,
24. (a) If figure shown in option (a), is placed in the
place of missing portion of the original figure, A
then it is completed as shown below
W
then it is completed as shown below
SH
A
@

31. (d) If figure shown in option (d), is placed in the


place of missing portion of the original figure,
25. (c) If figure shown in option (c), is placed in the then it is completed as shown below
place of missing portion of the original figure,
then it is completed as shown below

32. (c) If figure shown in option (c), is placed in the


26. (d) If figure shown in option (d), is placed in the place of missing portion of the original figure,
place of missing portion of the original figure, then it is completed as shown below
then it is completed as shown below
+
+
+

+
Chapter 11 FIGURE COMPLETION
425

33. (b) If figure shown in option (b), is placed in the 38. (b) If figure shown in option (b), is placed in the
place of missing portion of the original figure, place of missing portion of the original figure,
then it is completed as shown below then it is completed as shown below

39. (a) If figure shown in option (a), is placed in the


34. (d) If figure shown in option (d), is placed in the
place of missing portion of the original figure,
place of missing portion of the original figure,
then it is completed as shown below
then it is completed as shown below

35. (c) If figure shown in option (c), is placed in the 40. (d) If figure shown in option (d), is placed in the
place of missing portion of the original figure, place of missing portion of the original figure,

A
then it is completed as shown below then it is completed as shown below

M
H
TT
36. (d) If figure shown in option (d), is placed in the A
41. (a) If figure shown in option (a), is placed in the
W
place of missing portion of the original figure, place of missing portion of the original figure,
SH

then it is completed as shown below then it is completed as shown below


A
@

37. (d) If figure shown in option (d), is placed in the 42. (d) If figure shown in option (d), is placed in the
place of missing portion of the original figure, place of missing portion of the original figure,
then it is completed as shown below then it is completed as shown below
426 How to Crack Test of Reasoning NON-VERBAL

12
Figure Matrix
The representation of figures following a certain pattern, in a matrix form is called figure matrix.
In this chapter, we deal with questions which have either 2 ´ 2 or 3 ´ 3 matrix. These matrices

A
are formed by a group of figures. Corresponding rows or corresponding columns follow a certain
pattern. You are required to analyse each of the sets to find out that pattern and on the basis of

M
that, the missing figure is to be identified from a set of alternatives, so as to complete the matrix.

H
Directions (Illustrations 1-3) In each of the following question, find out which of the answer figures

TT
(a), (b), (c) and (d) completes the figure matrix?
Illustration 1. Problem Figure Answer Figures A
W
SH

(a) (b) (c) (d)


A

?
@

Solution (d) Clearly, in the first row, the number of dots in the second figure is twice the number of dots in the
first figure. Similarly, in the second row, the number of dots in the second figure must be twice the number of
dots in the first figure.
So, number of dots in missing segment = 2 ´ 3 = 6

Illustration 2. Problem Figure Answer Figures

+ + + +
+ + + (a) (b) (c) (d)
+ +

?
UP (B. Ed.) 2015
Solution (a) In each row, the number of objects increases by 1 at each step from left to right.
So, in third row, the missing segment contain 2 + 1 = 3 dots.
Chapter 12 FIGURE MATRIX 427

Illustration 3. Problem Figure Answer Figures

(a) (b) (c) (d)


?

Solution (a) In each row, the central part of the first figure rotates either 90° clockwise or 90° anti-clockwise to
form the central part of the second figure and the central part of the second figure rotates 90° to form the
central part of the third figure. Also, in each row, there are three types of geometrical figures, i.e. rectangles,
circles and triangles.

Let us Practice
Directions (Q. Nos. 1-17) In each of the following question, find out which of the answer figures (a),

A
(b), (c) and (d) completes the figure matrix.

M
1. Problem Figure 3. Problem Figure

H
TT
?

A
W
? SSC (CPO) 2017
SH

Answer Figures
Answer Figures
A
@

(a) (b) (c) (d)


(a) (b) (c) (d)
4. Problem Figure
2. Problem Figure

Answer Figures
Answer Figures

(a) (b) (c) (d)


(a) (b) (c) (d)
428 How to Crack Test of Reasoning NON-VERBAL

5. Problem Figure Answer Figures

(a) (b) (c) (d)


? 9. Problem Figure
SSC (CPO) 2013
Answer Figures

(a) (b) (c) (d)


?
6. Problem Figure
Answer Figures

A
?

M
(a) (b) (c) (d)

H
10. Problem Figure

TT
Answer Figures

A
W
SH

(a) (b) (c) (d)


7. Problem Figure ?
A
@

Answer Figures

? (a) (b) (c) (d)


SSC (CGL) 2014
Answer Figures 11. Problem Figure

(a) (b) (c) (d)


8. Problem Figure
?
SSC (Cabinet Secretariat) 2013, SSC (10+2) 2012
Answer Figures

? (a) (b) (c) (d)


Chapter 12 FIGURE MATRIX 429

12. Problem Figure 15. Problem Figure

?
MPPSC 2018
Answer Figures
Answer Figures

(a) (b) (c) (d)


(a) (b) (c) (d)
13. Problem Figure 16. Problem Figure

A
M
H
TT
? ?
Answer Figures Answer Figures A
W
SH
A

(a) (b) (c) (d)


(a) (b) (c) (d)
@

14. Problem Figure 17. Problem Figure

?
RRB (TC/CC) 2008 ?
Answer Figures
Answer Figures

(a) (b) (c) (d)


(a) (b) (c) (d)

Answers
1. (d) 2. (b) 3. (a) 4. (d) 5. (b) 6. (d) 7. (a) 8. (a) 9. (d) 10. (c)
11. (b) 12. (b) 13. (d) 14. (a) 15. (d) 16. (a) 17. (c)
430 How to Crack Test of Reasoning NON-VERBAL

Answer with Explanations


1. (d) In each row the number of dots in the second 11. (b) Observing the figure in columnwise manner
figure is thrice the number of dots in the first we notice that in column first the figure rotates
figure. 90° clockwise in each step.
2. (b) The second figure in each row is obtained by In column second, the figure rotates 90°
joining the lower part of first figure with its upper anti-clockwise in each step.
part. The common side is then removed to form Similarly, in column third the figure rotates 90°
the second figure. clockwise in each step.
3. (a) If we move clockwise the triangle rotates 90° in 12. (b) In each row we have three figures, i.e a simple
clockwise direction in each block. triangle, a triangle with circle inside it and a
4. (d) The pattern followed is triangle with two line segments. Also, in each
row two triangle faces top while one triangle
faces the bottom.
13. (d) Clearly, there are three types of shadings of
circles in each row, one is unshaded, another
In each row the second figure is obtained by has its right half shaded and yet another has
joining the adjacent end points of the intersecting its upper half shaded. Also, two of the figures
lines. in each row have one triangle shaded.

A
5. (b) Starting with the top left figure, one line is 14. (a) There are three types of arrows, a single

M
increasing in clockwise direction. So, four lines will headed arrow, a double headed arrow and a

H
come in fourth part. triple headed arrow. There are three positions
(d) In each row, the next figure is obtained by rotating of arrows upwards, downwards and towards

TT
6.
the first figure 135° anti-clockwise. right. The arrows have three types of bases
square, rectangular and circular. Each of
7. (a) In each row, the number of circles decreases by
one and number of triangles inicreases by one as A
these features is used once in each row. So,
W
the missing figure consists of the double
we move from left to right.
headed arrow with rectangular base points
SH

8. (a) In figure, each row consists of a circle and two line pointing towards right.
segments, three line segments and four line
15. (d) Clearly, in each row, the second figure is at the
segments. So, in third row only the figure with
A

innermost and outermost position and first


three line segments is missing.
figure at middle position of the third figure.
@

9. (d) The third figure in each row comprises of parts


16. (a) Each column has one ‘+’, one ‘o’ and one ‘×’.
which are common in the figure.
First figure of each column is without boundary
10. (c) In each row we have three figures, i.e. square, and second one is in the square. Third figure is
triangle and circle. in two squares. As, there are ‘+’ and ‘o’ in third
In third row we have circle and square present in column. So, figure ‘×’ will be surrounded by
outermost portion, so triangle is the outermost two squares.
portion in missing figure. We have also noticed 17. (c) Clearly, in each row, the number of elements in
that the figure inside the main figure contains the third figure is equal to the difference in the
triangle and circle, so square is missing. number of elements in the first and second
Hence, we can say that the missing part consist of figures. Also, the third figure has the same
a figure with triangle containing square. types of elements as that of first figure.
Chapter 13 CUBE AND DICE 431

13
Cube and Dice
A cube or dice is a three-dimensional object l
Number of smaller cubes with two surfaces
bounded by six square faces or surfaces. painted = (n - 2) ´12
l
Number of smaller cubes with one surface
Cube painted = (n - 2)2 ´ 6
l
Number of smaller cubes with no surface
A cube is a three-dimensional figure, having 8
painted = (n - 2)3

A
corners, 6 surfaces and 12 edges.
Here, n = Number of parts on an edge of the

M
S M S bigger cube after division
S

H
M C M Length of edge of bigger cube
=

TT
M
S M S M Length of edge of one smaller cube
S
C
S M S
M
S
A
Illustration 1. All the surfaces of a cube of 15 cm
W
M side are painted with red colour and then it is
M C M
cut into smaller cubes of 3 cm side. Then,
SH

S
S M S (i) How many smaller cubes are there
having only one surface painted with red
A

After cutting a cube, following type of smaller colour?


@

cubes are obtained (a) 18 (b) 24 (c) 36 (d) 54


l
Corner cubes = S (exist at each corner) (ii) How many smaller cubes are there having
l
Middle cubes = M (exist at the middle of two surface painted with red colour?
each edge) (a) 8 (b) 24 (c) 36 (d) 54
l
Central cubes = C (exist at the middle of each (iii) How many smaller cubes are there
face) having only three surface painted with
l
Nuclear cube/Inner central cube = N (hidden red colour?
and exist at the centre of the larger cube) (a) 8 (b) 24 (c) 36 (d) 54
(iv) How many smaller cubes are there
Number of Cubes When a Large having 4 or more faces painted with red
Cube is Painted and Cut colour?
If a cube is painted on all of its surfaces with (a) 0 (b) 8 (c) 36 (d) 81
the same colour and then cut into smaller (v) How many smaller cubes are there
cubes of equal size, then after separation, having no surfaces painted with red
number of smaller cubes, so obtained will be colour?
calculated as under (a) 3 (b) 9 (c) 27 (d) 81
l
Number of smaller cubes = n3 (vi) How many smaller cubes are obtained
l
Number of smaller cubes with three surfaces from larger cube?
painted = 8 (a) 5 (b) 25 (c) 75 (d) 125
432 How to Crack Test of Reasoning NON-VERBAL

Side of bigger cube 15 Illustration 3. Count the number of cubes in the


Solution (i) (d) n = = =5
Side of smaller cube 3 given figure. SSC (10+2) 2009
\Required number of smaller cubes = 6 (n - 2) 2
= 6 (5 - 2) 2
= 6 ´ 9 = 54
(ii) (c) Required number of cubes
= 12 (5 - 2)
= 12 ´ 3 = 36
(iii) (a) As cubes with three faces painted are
always 8.
(iv) (a) As number of cubes with four or more faces
painted is 0. (a) 6 (b) 8 (c) 10 (d) 12
(v) (c) Required number of cubes Solution (c) Clearly, in the figure, 1 column contains
= (n - 2)3 = (5 - 2)3 3 cubes, 2 columns contains 2 cubes each and
= (3)3 = 27 3 columns contains 1 cube each.
\ Total number of cubes = (1 ´ 3) + (2 ´ 2) + (3 ´1)
(vi) (d) Total number of smaller cubes = 3 + 4 + 3 = 10

A
= (n)3 = (5)3 = 125
Illustration 4. Count the number of blocks in the

M
Illustration 2. Little wooden cubes each with a given figure.

H
side of one inch are put together to form a

TT
solid cube with a side of three inches. This big
cube is then painted red all over on the
outside. When the big cube is broken up into A
W
the original little ones, how many cubes will
SH

have paint on two sides? SSC (CGL) 2010

(a) 7 (b) 11 (c) 13 (d) 15


A

Solution (a) Number of blocks at the top = 3


@

Number of blocks at bottom = 4


\Total blocks = 3 + 4 = 7

Dice
Dice is a three-dimensional figure with
(a) 4 (b) 8 6 surfaces. It may be in the form of a cube or a
(c) 12 (d) 0 cuboid.
Solution (c) A smaller cube with two sides painted is In the questions based on dice one or more
present on each of the 12 edges of the cube. figures showing the different positions of a
Therefore, total number of cubes having paint on same dice are given. After observing these
two sides = 12 figures, we have to find the different side
(opposite or adjacent) of the dice.
Counting the Number E H
of Cubes A D
When the number of cubes (or blocks/
G
cuboids) in a figure needs to be counted, the F
procedure is described with the help of the B C
following illustrations.
Chapter 13 CUBE AND DICE 433

Front surface ¾¾® ABCD ù opposite, Sum of the adjacent faces


Back surface ¾¾® EFGH ûú
1 + 2 = 3, 3 + 2 = 5, 4 + 1 = 5, 5 + 1 = 6,
Top surface ¾¾® ADHE ù 6 + 1 = 7, 4 + 2 = 6, 5 + 2 = 7, 6 + 3 = 9,
opposite,
Bottom surface ¾¾® BCGF úû 4 + 3 = 7, 5 + 3 = 8, 6 + 4 = 10,
Left surface ¾¾® ABFE ù opposite 6 + 5 = 11
Right surface ¾¾® DCGH úû
How to fill unfolded
Types of Dice faces with digits?
Dice having digits/dots from 1 to 6 on its Two positions of a dice are given below. In
surfaces can be divided into two types order to fill the digits in the given unfolded
1. Standard Dice When the sum of figure follow the following steps
digits/dots on opposite faces is equal to
7, then the dice is called standard dice. 2 3
4 6
6 5
Let us see
(a) (b)
1

A
2 Step (i) Starting with common digit write the

M
4 3 digits moving in anti-clockwise
5
direction as below.

H
6
Position (a) 6 ¾® 4 ¾® 2

TT
Sum of the opposite faces (I) (II) (III)
1+ 6 =7 4 + 3 =7 2 + 5 =7 A
Position (b) 6 ¾® 3 ¾® 5
W
Sum of the adjacent faces (I) (IV) (V)
SH

1 + 2 = 3, 3 + 1 = 4, 4 + 1 = 5, Step (ii) Now, place the above digits in the


5 + 1 = 6, 6 + 2 = 8, 3 + 2 = 5, unfolded figure at their respective
position as below.
A

4 + 2 = 6, 5 + 3 = 8, 6 + 3 = 9,
@

5 + 4 = 9, 6 + 4 = 10, 6 + 5 = 11 III 2
2. General Dice When the sum of IV I II 3 6 4
digits/dots on opposite faces is not equal
to 7, then the dice is called general dice. V 5
Let us see
2 Step (iii) Now, fill the blank space with the
3 remaining digit, i.e.1.
5 4
1
2
6
3 6 4
Sum of the opposite faces 5
1 + 3 = 4, 4 + 5 = 9,
2 + 6 =8 1
434 How to Crack Test of Reasoning NON-VERBAL

Dice or Cube Formation


When a dice or a cube is formed by folding a sheet of paper, then some of the forms or patterns
are as follows
1 lies opposite to 6 1 lies opposite to 5
2 lies opposite to 4 1 2 2 lies opposite to 4 1
3 lies opposite to 5 3 3 lies opposite to 6 2 3 4
4 5
5 6 6

1 lies opposite to 3 1 1 lies opposite to 4 1


2 lies opposite to 5 2 2 lies opposite to 6
2 3
4 lies opposite to 6 3 4 3 lies opposite to 5
4
5
5 6
6

A
1 lies opposite to 4 1

M
2 lies opposite to 5 2 3
3 lies opposite to 6

H
4 5

TT
6

A
W
Important Rules Related to Dice
SH

S.No. Rule Example Explanation


I. In a dice, one face is adjacent to four Here, the adjacent faces of 1 are 3, 5,
A

1 1
other faces and the remaining sixth 2 and 6.
@

face is opposite to it. 5 6


3 2 So, the opposite face of ‘1’ must be
the remaining face, i.e. 4.
(i) (ii)

II. When a number is common in two Here, 4 is common and it is on the


4 4
positions of a dice and it is on the same face in both the positions.
same face in both the positions, then 1 2
5 3 So, 5 and 3 are opposite, 1 and 2 are
the remaining faces in one position will opposite and hence 4 and 6 (the
be opposite to their corresponding (i) (ii) remaining numbers) are opposite.
faces in the other position.
III. When a number is common in two Here, 3 is common in both the
1 3
positions of a dice but it is placed on positions but it is placed on different
different faces, then moving in 3 5 faces. So, starting from 3 in clockwise
2 4
clockwise direction from the common direction, 2 [in position (i)] and 5 [in
number in both the positions, first face (i) (ii) position (ii)] are opposite. Also, 1 [in
will be opposite to first face and position (i)] and 4 [in position (ii)] are
second face will be opposite to second opposite. The remaining number, i.e. 3
face. is obviously opposite to 6.
IV. When two numbers are common in the Here, 5 and 6 are common in both the
4 6
two positions of a dice, then the positions.
remaining numbers are opposite to 6 3
5 5 So, the remaining numbers, i.e. 4 and
each other. 3 are opposite.
(i) (ii)
Chapter 13 CUBE AND DICE 435

Methods to Find Digits/Dots/ Solution (a) From positions (i) and (iii), adjacent
faces of 2 are 1, 3, 4 and 6. So, the remaining
Figures/Symbols at the Blank Face face, i.e. 5 is opposite to 2. Now, from positions
In the following figure, three positions of a (ii) and (iv), adjacent faces of 4 are 1, 2, 5 and 6.
single dice are given. So, the remaining face 3 is opposite to 4.
Hence, the remaining two faces, i.e. 1 and 6 are
1 3 4 opposite.
3 4 ? Alternate Method
6 5 2
Numbers 4 and 2 are common in positions (ii) and
I II III (iii), so according to rule number IV, 1 and 6 are
To find (?) firstly we take opposite. Hence, 1 is at bottom when 6 is at top.
positions I and II of the given 6 Illustration 6. Choose the cube which will be
dice as these two positions have formed on folding the question figure.
5 3 1
a common digit 3. Question Figure SSC (CGL) 2014
Now, the unfolded figure will 4
be as shown. Here, 1 and 5 are O
opposite faces. 2 and 3 are 2 K
L

T
opposite faces. 4 and 6 are

A
M U
opposite faces.

M
Now, the face having digit 1 in the unfolded Answer Figures

H
figure in pushed beside the face having digit 2

TT
in the following way K L K O
M U L
T

6 L
A K T K
W
5 3 1
(a) (b) (c) (d)
SH

4 Pushed
2
Solution (a) The unfolded dice can be represented as
1
A

O
@

Now, compare the position III of the dice with


unfolded figure obtained by pushing and go K Opposite
anti-clockwise from the face having digit 2 to
Opposite
L
T

get the following situation.


From position III of the dice M U
2 ? 4
From unfolded figure obtained 2 1 4
by pushing Opposite
\ ? =1
Here, O and L are opposite, K and M are opposite.
Illustration 5. Four positions of a dice are given
T and U are opposite.
below. Identify the number at the bottom when
In answer figure (b), K and M are shown adjacent
the number at the top is 6. SSC (CPO) 2014
to each other, in answer figure (c), T and U are
shown adjacent to each other and in answer figure
2 4 6 5 (d), O and L are shown adjacent to each other.
According to the question figure these are
1 3 1 2 4 2 4 6
opposite to each other, so these alternatives are
not possible. Only the cube given in answer figure
(i) (ii) (iii) (iv) (a) can be formed because K, L and T can be on
(a) 1 (b) 3 (c) 4 (d) 5 adjacent faces.
How to Crack Test of Reasoning NON-VERBAL

Let us Practice
1. How many cubes are there in this diagram? 6. All the surfaces of the dice contain different
numbers in the form of dots. Consider both
the figures of dice and tell how many dots
are there on the face opposite to the face that
contains four dots?

(a) 16 (b) 12 (c) 10 (d) 8


(i) (ii)
2. How many cubes are there in the group?
(a) 2 (b) 3 (c) 5 (d) 6
7. When number 1 is at the top, which number
will come at the bottom?

A
M
H
(i) (ii)

TT
(a) 10 (b) 16 (c) 18 (d) 20
(a) 1 (b) 2 (c) 3 (d) 6
3. Two position of a cube are shown below.
What will come opposite to face A
8. Two positions of a dice are shown below.
W
containing ‘9‘? SSC (CPO) 2017 When there are two dots at the bottom, the
number of dots at the top will be
SH

7 8
9 6
8 4
A

(i) (ii)
@

(i) (ii)
(a) 7 (b) 4 (c) 6 (d) 4 or 6
(a) 2 (b) 3
4. Two positions of a dice are shown below (c) 5 (d) 6
when number 1 is on the top. What number
will be there at the bottom? 9. Which number will appear on the face
opposite to 1? SSC (10+2) 2012
6 6
1 4 4 5 2 2
4 3 4 1
(i) (ii)
(a) 2 (b) 3 (a) 6 (b) 5
(c) 5 (d) Data inadequate (c) 4 (d) 3
5. What number is at the opposite of 3 in the 10. Two positions of a cube are shown below.
figure shown below? The given two positions When the face at the top has 3 dots, then
are of the same dice whose each surface bears how many dots will be there at bottom face?
a number among 1, 2, 3, 4, 5 and 6. SSC (CGL) 2015

2 1
6 5 5 4
(i) (ii)
(i) (ii)
(a) 2 (b) 4 (c) 5 (d) 6 (a) 5 (b) 1 (c) 4 (d) 2
Chapter 13 CUBE AND DICE 437

11. Which digit will appear on the face opposite Directions (Q. Nos. 16-20) In each of the
to the face with number 4? DSSSB 2017 following question, four positions of the same dice
2 5 have been shown. You have to see these figures
3 3 and select the number opposite to the number as
1 6 asked in each of the question.
16. 3 2 5 6
(a) 1 (b) 2 (c) 3 (d) 4 1 3
4 4
5 6 2 5
12. Two positions of a block are shown below
(i) (ii) (iii) (iv)
Which number is at the opposite face of
number 5?
(i) (ii) (a) 6 (b) 5 (c) 1 (d) 3
When six is at the bottom, what number 17. 6 4 5 6
come at the top? 4 3 2 5
(a) 1 (b) 2 (c) 4 (d) 5 2 2 3 1

13. Three positions of a dice are given below (i) (ii) (iii) (iv)

3 3 4 Which number is at the opposite face of

A
number 2?
2 6 3

M
6 5 5 (a) 4 (b) 1 (c) 5 (d) 3

H
(i) (ii) (iii)
18. 2 6 5 1

TT
3 1 2 2
Identify the number on the face opposite 1 4 3 4
to 6. SSC (Steno) 2011 (i) (ii) (iii) (iv)
(a) 1 (b) 4 (c) 5 (d) 7 A
W
Which number is at the opposite face of
14. Three views of a cube are given below. Which number 5?
SH

number is printed on the opposite face to (a) 4 (b) 6 (c) 1 (d) 3


number 4 ? SSC (CPO) 2013
19.
A

2 4 1 3 3 2 4
4 2 6 6 1 1 6
@

3 1 5 5 6 3 1

(a) 5 or 6 (b) 3 (c) 6 (d) 2 (i) (ii) (iii) (iv)

15. The following figure represents a wooden Which number is on the opposite surface of
block of cube shape with 3 cm as its edge number 3?
and all the face of the cube are painted black. (a) 4 (b) 6 (c) 5 (d) 1
If the cube is cut along the dotted lines and 20. 2 5 1 6
27 new cubes are formed with a volume of 1 5 2 6 4
cm 3 each. What will be the number of 4 1 5 5
non-painted blocks? (i) (ii) (iii) (iv)
UPSSSC (Lower Subordinate) 2016
Which number is opposite to number 1?
(a) 4 (b) 6 (c) 2 (d) 3
21. How many dots lie opposite to the face
having three dots, when the given figure is
folded to form a cube?

(a) 1 (b) 3 (c) 6 (d) 9 (a) 2 (b) 4 (c) 5 (d) 6


438 How to Crack Test of Reasoning NON-VERBAL

Directions (Q. Nos. 22 and 23) The sheet of


paper shown in the figure given on the left hand
side of problem, is folded to form a box. Choose the
correct alternative which will truely represent the (a) (b) (c) (d)
position of various numbers/symbols/letters in
diagram. Directions (Q. Nos. 26-28) Read the following
information carefully to answer the questions that
22. 1 follow
4 3 6 A cube is coloured red on all of its faces. It is then
2 2 4 3 4 cut into 64 smaller cubes of equal size. The smaller
5 3 5 3
5 3 6 2 1 cubes, so obtained are now separated.
(a) (b) (c) (d) 26. How many smaller cubes have two surfaces
painted with red colour?
23. A (a) 24 (b) 8 (c) 12 (d) 20
C
27. How many smaller cubes have one surface
D B painted with red colour?
E C A C B (a) 16 (b) 24 (c) 32 (d) 8
D B A A

A
F E C E F
28. How many cubes have no surface painted

M
(a) (b) (c) (d) with red colour?
(a) 0 (b) 16 (c) 8 (d) 24

H
Directions (Q. Nos. 24 and 25) In each of the 29. From the given two postions of a single dice,

TT
following question, which of the following cube in find the letter at the face opposite to the face
the answer figure cannot be made based on the having letter V.
unfolded cube in the question figure? A SSC (FCI) 2008
W
SSC (10+2) 2018 V S
(i) J (ii) J
24.
SH

I U
A

(a) S (b) U (c) J (d) I


@

30. Four portions of a cube are shown below.


Identify the number at the bottom when top
is 6? SSC (CPO) 2016

3 4 5 6
6 2 5 1 3 1 5 4

(a) 1 (b) 6
(a) (b) (c) (d) (c) 5 (d) 2
25. 31. A dice is thrown four times and its different
positions are shown below. Which number is
opposite to face showing 2? SSC (CPO) 2013
2 4 1 2
4 2 6 6
3 1 5 1

(a) 6 (b) 3
(c) 4 (d) 5
Chapter 13 CUBE AND DICE 439

32. From the given four positions of a single 36. A cube which is painted red on the outer
dice, find the colour at the face opposite to the surface is of 2 inches height, 2 inches wide
face having green. SSC (10+2) 2008 and 2 inches across. If it is cut into one inch
n e r
cubes as shown by dotted lines, then indicate
Gree White Oran
g Silve the number of cubes which are red on two
n
Gree
d

d
sides?
Re

Re

Re
Orange Orange Violet Green

(i) (ii) (iii) (iv)

(a) Orange (b) Red


(c) Silver (d) Violet
(a) 8 (b) 0 (c) 4 (d) 6
33. Four positions of dice are given below.
Which letter will be opposite to D? 37. A cube has six numbers marked 1, 2, 3, 4, 5
SSC (CGL) 2013 and 6 on its faces. Three views of the cube
B E C are shown below
D
1 3 3
A C E B
C A D C 4 2 6
6 1 5

A
(i) (ii) (iii) (iv)
What possible numbers can exist on the two

M
(a) D (b) B faces marked A and B, respectively?

H
(c) A (d) C
B

TT
34. According to given figure of dice, which 5
A
option is correct. SSC (Multitasking) 2011
(a) 2 and 3 A (b) 6 and 1
W
1 (c) 1 and 4 (d) 3 and 1
SH

6 2 38. What will come in place of ‘?’.


3
5 3 6
A

4 5 4 1 ?
2 5 3
@

6 6 2 6
4 2 6 1 (a) 4 (b) 2 (c) 1 (d) 5
2 5 3 5
Directions (Q. Nos. 39 and 40) Read the
(a) (b) (c) (d) following information carefully to answer the
questions that follow.
35. Which cube can be made from the given net?
A cuboid of dimensions (6 cm ´ 4 cm ´ 1 cm) is
painted black on both the surfaces of dimensions
(4 cm ´ 1cm), green on the surfaces of dimensions
(6 cm ´ 4 cm) and red on the surfaces of
dimensions (6 cm ´ 1 cm). Now, the block is
divided into various smaller cubes of side 1 cm
each. The smaller cubes, so obtained are
separated.
39. How many cubes will have atleast two
colours?
(a) 16 (b) 12 (c) 10 (d) 8
40. How many cubes will have 4 coloured sides
and two faces without colour?
(a) (b) (c) (d) (a) 8 (b) 4 (c) 16 (d) 10
440 How to Crack Test of Reasoning NON-VERBAL

Directions (Q. Nos. 41 and 42) Read the 41. How many cubes will not have any coloured
following information carefully to answer the faces?
questions that follow. (a) 0 (b) 16
A cube is coloured red on two opposite faces, blue (c) 4 (d) 8
on two adjacent faces and yellow on two remaining
faces. It is then cut into two halves along the plane
42. How many cubes will not have any red faces?
parallel to the red faces. One piece is then cut into (a) 8 (b) 16
four equal cubes and the other one into 32 equal (c) 20 (d) 24
cubes.

Answers
1. (b) 2. (d) 3. (b) 4. (c) 5. (c) 6. (d) 7. (d) 8. (b) 9. (d) 10. (a)
11. (c) 12. (d) 13. (b) 14. (c) 15. (a) 16. (c) 17. (b) 18. (c) 19. (a) 20. (a)
21. (d) 22. (d) 23. (b) 24. (d) 25. (b) 26. (a) 27. (b) 28. (c) 29. (a) 30. (a)
31. (d) 32. (d) 33. (c) 34. (c) 35. (a) 36. (b) 37. (a) 38. (c) 39. (a) 40. (b)

A
41. (c) 42. (b)

M
H
Answer with Explanations

TT
1. (b) From figure, it is clear that the figure have
12 cubes as shown below A
6. (d) Face with 1 dot is common in both figures. So,
according to rule number 3, 2 is opposite to 5, 6
W
is opposite to 4 and 1 is opposite to 3.
SH

1 2 3 4 7. (d) In both figures, the position of 2 dots is


common. So, according to rule 2, 1 is opposite
A

12 5 to 6, 5 is opposite to 3 and 2 is opposite to 4.


@

11 6 8. (b) According to the rule number 3, 1 dot is


10 9 8 7
common in both the positions. So, 3 is opposite
to 2, 5 is opposite to 4 and 1 is opposite to 6.
2. (d) From the top most row of the figure, 9. (d) Accoding to the rule 4, 2 and 4 are common in
Cube in 1st row = 1, Cube in 2nd row = 3 both positions. So, 3 is opposite to 1.
Cube in 3rd row = 6, Cube in 4th row = 10
\ Total number of cubes = 1 + 3 + 6 + 10 = 20 10. (a) In both figures, the position of 6 dots is common.
So, according to the rule 2, 1 is opposite to 4, 3
3. (b) In both figures 8 is common. So, from rule 3, 7 is is opposite to 5 and 6 is opposite to 2.
opposite to 6 and 9 is opposite to 4. Hence, 5 is at the bottom when 3 is at the top.
4. (c) Here, the two faces 6 and 4 are common in both 11. (c) From both the figures, 1, 2, 5 and 6 are adjacent
the figures. So, from rule 4, 1 is opposite to 5. to 3. Hence, 4 is opposite to 3.
5. (c) The unfolded figure will appear as, 12. (d) From figures (i) and (ii), we conclude that the
6
numbers 1, 2, 3 and 4 appear adjacent to 6.
Thus, the number 5 will appear opposite to 6.
4 5 2 Therefore, when 6 is at the bottom, 5 will be at the
top.
1
13. (b) From positions (ii) and (iii), 3 and 5 are two
3
common numbers. Hence, 6 and 4 are
So, 5 is opposite to 3. opposite to each other.
Chapter 13 CUBE AND DICE 441

14. (c) The unfolded figure will appear as, 25. (b) The pairs of opposite faces are
4 ; and
5 1 2 Option figure (b) cannot be made as in this figure
6 and are adjacent to each other.
3
Solutions (Q. Nos. 26-28) According to the given
So, 4 is opposite to 6. information,

15. (a) Here, n = 3


\Number of non-painted blocks
= (n - 2 )3 = (3 - 2 )3 = 1

16. (c) From the figures (iii) and (iv), it is clear that
numbers adjacent to 5 are 3, 4, 2 and 6. Hence,
number 1 will be opposite to number 5. Here, n 3 = 64
17. (b) It is clear from all the figures that, numbers 6, 4, 3 \ n=4
and 5 appear on the adjacent surfaces of

A
26. (a) Number of smaller cubes with two surfaces
number 2. Hence, number 1 will be opposite to
painted = (4 - 2 ) ´ 12 = 24

M
number 2.

H
27. (b) Number of cubes with one surface painted
18. (c) It is clear from all the figures that, 2, 3, 4 and 6
= (n - 2 )2 ´ 6 = (4 - 2 )2 ´ 6 = 2 2 ´ 6 = 24

TT
appear on the adjacent surfaces of number 1.
So, number 1 will be opposite to number 5. 28. (c) Number of cubes with no surface painted
19. (a) From the figures (i), (ii) and (iii), it is clear that A
= (n - 2 )3 = (4 - 2 )3 = 2 3 = 8
W
numbers 5, 6, 1 and 2 are on the adjacent 29. (a) In both figures, the position of J is common. So,
surfaces of number 3. Hence, 4 is opposite to according to rule 2, V is opposite to S.
SH

number 3.
30. (a) The adjacent faces of 6 are 3, 2, 5 and 4. So, the
20. (a) From the positions (iii) and (iv), 5 and 6 are remaining number 1 will be opposite to 6.
A

common in both the positions. So, according to


@

rule 4, 1 is opposite to 4. 31. (d) From the first and fourth position of the dice, it is
clear that the numbers 6, 4, 3 and 1 appear
21. (d) When this figure is folded to form a cube, then
adjacent to 2. Thus, opposite face of 2 will be 5.
the face bearing six dots will be opposite to the
face bearing three dots. 32. (d) From the given four positions of a single dice,
22. (d) From the question figure, it is clear that 4 is faces adjacent to face having green colour
opposite to 6, 1 is opposite to 2 and 3 is = Orange, red, white, silver
opposite to 5. Figures (a), (b) and (c) will not be Clearly, violet is opposite to green.
identical to question figure because numbers
33. (c) In figures (ii) and (iii), E and C are common. So,
forming the opposite pairs appear on the
according to rule 4, D is opposite to A.
adjacent surfaces.
23. (b) From the question figure, it is clear that A is 34. (c) As, So, opposite
1 1 3
opposite to D, C is opposite to E and B is
6 2 2 4
opposite to F. Hence, figures (a), (c) and (d) are
not identical to question figure. 6 5
3
24. (d) The pairs of opposite faces are 4 5
; and
In figure (a), 2 and 4, in figure (b) 6 and 5 and in
Option figure (d) cannot be made as in this figure figure (d) 6 and 5 are shown adjacent to each
other. So, these dice are not possible.
and are adjacent to each other. Hence, only dice (c) can be formed.
442 How to Crack Test of Reasoning NON-VERBAL

35. (a) The pairs of opposite faces are Solution (Q. Nos.39-40) According to the given
information,
, and
Red (Top)

Here, the cubes given in options (b), (c) and (d) Black (Right)
cannot be formed because in these, the faces
(Front)
, and , Black (Left)
Red (Bottom)
respectively are given on adjacent faces. So,
only the cube given in option (a) can be formed. Green (Back)
36. (b) From the figure, there is no such cube which is
39. (a) All the sixteen cubes there on the boundary of
painted red only on two sides.
this block will have atleast two colours only.
37. (a) From figure first and second, it is clear that
40. (b) All the 4 cubes present on the corners will have
number 6, 2, 3 and 4 are adjacent to 1. Hence, 1
four faces painted and two faces unpainted.
is opposite to 5.
From figure second and third, it is clear that
Solutions (Q. Nos.41-42) According to the given
number 1, 2, 5 and 6 are adjacent to 3. Hence, 4 information,

A
is opposite to 3. And, therefore 2 is opposite to Red (top)

M
6. Thus, the numbers A and B, which are Blue (back)

H
adjacent to 5 could be any of the following four, Blue (right)
6 and 4, 4 and 2, 2 and 3, and 3 and 6. Out of

TT
these only 2 and 3 is given as option (a). Yellow (left)
Yellow (front)
38. (c) The unfolded figure will appear like this
A
W
4
SH

1 5 2 Red (bottom)
Pushed
1 3
A

41. (c) Only 4 smaller cubes inside the cube will have
6
@

none of their faces painted.


From position III of dice :3?6 42. (b) 16 cubes from the second layer from the
From unfolded figure obtained by pushing : 3 1 6 bottom will not have any red faces.
\ ? =1

You might also like